Maths Question Bank

You might also like

Download as pdf or txt
Download as pdf or txt
You are on page 1of 440

12

CBSE

MATHEMATICS

Ram Awatar Gupta


BSc (Hons) (Physics)
MSc (Mathematics)
Retd. Senior Lecturer
DTEA Sr. Sec. School, Delhi
(More than 40 Years of Teaching Experience)

Full Marks Pvt Ltd


(Progressive Educational Publishers)
New Delhi-110002
Published by:

9, Daryaganj, New Delhi-110002


Phone: 011- 40556600 (100 Lines)
Website: www.fullmarks.org
E-mail: info@fmpl.in

© Author

All rights reserved. No part of this publication may be reproduced or transmitted, in any form or by any means, without permission. Any person
who does any unauthorised act in relation to this publication may be liable to criminal prosecution and civil claims for damages.

Branches:
• Chennai • Guwahati

Marketing Offices:
• Ahmedabad • Bengaluru • Bhopal • Dehradun • Hyderabad • Jaipur • Jalandhar • Kochi • Kolkata • Lucknow
• Mumbai • Patna • Ranchi

NEW EDITION

“This book is meant for educational and learning purposes. The author(s) of the book has/have taken all reasonable care to ensure that the contents
of the book do not violate any existing copyright or other intellectual property rights of any person in any manner whatsoever. In the event the
author(s) has/have been unable to track any source and if any copyright has been inadvertently infringed, please notify the publisher in writing for
corrective action.”
Preface

Mathematics-12 is based on the latest curriculum guidelines specified by the CBSE.


It will certainly prove to be a torch-bearer for those who toil hard to achieve their goal.
This All-in-one Question Bank has been developed keeping in mind all the requirement of the students for Board
Examinations preparations like learning, practicing, revising and assessing.
Salient Features of the Book:
●● Each chapter is designed in ‘Topic wise’ manner where each topic is briefly explained with sufficient Examples and
Exercise. Exercise which covers all the possible variety of Questions.
●● Answers with hints are provided separately after the exercise.
●● Previous years’ Board Questions have been covered in every chapter.
●● 1 Solved and 2 Unsolved Sample Papers are given with hints & answers for self assessment.
●● Common Errors by the students are provided to make students aware what errors are usually done unknowingly.
●● The book has been well prepared to build confidence in students.
Suggestions for further improvement of the book, pointing out printing errors/mistakes which might have crept in spite
of all efforts, will be thankfully received and incorporated in the next edition.
–Author

(iii)
Mathematics 12
GLIMPSE OF A CHAPTER

1
EXERCISE 7.9
Relations and Functions I. Multiple Choice Questions (MCQs)
9.
 sin 4x − 4 
∫ 1 − cos 4x  e
x
dx [Delhi 2010]
1. ∫e
x
(1 − cot x + cot 2 x ) dx =
(a) ex cot x + C (b) –ex cot x + C
Topics covered   x
10. ∫  (2 − sin x)  e 2 dx [V. Imp.]
(c) ex cosec x + C (d) –ex cosec x + C  1 − cos x 
1.1 Relations 1.2 Types of Relations
( x + 3) e x
1.3 Functions 2. ∫ ( x + 4) 2
dx = [NCERT Exemplar]
11. ∫
(x − sin x) dx
1 − cos x
ex ex

C hapter map
(a) +C (b) +C  x + sin x 
x+4 x+3 12. ∫  1 + cos x  dx
1 ex
(c) +C (d) +C
( x + 4) 2 ( x + 4) 2  (2 − 5 sin 2x)  e5xdx
RELATIONS
2
13. ∫  1 − cos 2x 
 log x − 1 
Type of Relations
3. ∫  1 + ( log x )
2 

dx is equal to
 1 
14. ∫ log (log x) + (log x)2  dx [V. Imp.] [Delhi 2010 (C)]
x xe x
(a) +C (b) +C
( log x ) 2 + 1 1 + x2  1 + sin 2x  − x
15. ∫   e dx [V. Imp.]
Universal Empty Reflexive Symmetric Transitive Equivalence x log x
(c) +C (d) +C  1 − cos 2x 
x2 + 1 ( log x ) 2 + 1
1 − sin x  −2x 
Equivalence Relation Evaluate each of the following Integrals 16. ∫ e  dx [V. Imp.] [A.I. 2013 (C)]
1 + cos x  
II. Short Answer Type Questions-I
∫ (sin x) a
x
Functions (x − 3) e x dx 17. dx
1. ∫ (x + 1)3 [Delhi 2009]
2 x
III. Short Answer Type Questions-II
2. ∫ (x + 1) e 2 dx
Types of Functions
(x + 1)
[Delhi 2005, 2006 (C)
( x cos x) dx
−1

(x − 4) e x dx
1. ∫ 1 − x2
[A.I. 2014 (C)]
One-one/Injective Function Onto/Surjective Function Domain and Co-domain 3. ∫ [Delhi 2008]
(x − 2) 2
of the Function
(1 − x) 2 e x dx 2.  (
x + 1 log x 2 + 1 − 2 log x  dx
2
)
 [A.I. 2011, 2014 (C)]
4. ∫ ∫ x4
(x 2 + 1) 2
 1 − sin x  x
Bijective Function 2 3. ∫  e dx
 x + 2  1 − cos x 
5. ∫  x + 4  e x dx
IV. Long Answer Type Questions
Cartesian Product of Sets (Recap) (5x + 14) e5x dx
Let A and B be two non-empty sets. A set of all the ordered pairs (a, b) such that a ∈ A and b ∈ B is called cartesian product of set
6. ∫ (x + 3)2 (sin −1
x − cos −1 x dx )
1. ∫ (sin −1
x + cos −1 x )
A with set B and denoted as (A × B). 7. ∫ [sin (log x) + cos (log x)] dx [V. Imp.]
x dx 2
Or A × B = {(a, b) : a ∈ A and b ∈ B)} 2. ∫ ( x sin x + cos x)2 [Delhi 2012]
∫ 3x tan x + x sec x dx
2 3 2
8.
The number of ordered pairs in (A × B) = n(A × B) = n(A) × n(B)
= (number of elements in set A) × (number of elements in set B). Answers 7.9
Example 1. Given set A = {1, 2, 3} and set B = {3, 5}. Value of A × B and n(B × A) respectively are given by
∫e ( x + 3) e x ( x + 4 − 1) x
x
I. 1. (b) I = (1 − cot x + cot 2 x) dx
2. (a) I = ∫ ( x + 4) 2
dx = ∫
( x + 4) 2
e dx
(a) {(1, 3), (1, 5), (2, 3), (2, 5), (3, 3), (5, 3)} and 6
∫e
x
= {(1+ cot 2 x ) − cot x} dx
(b) {(1, 3), (1, 5) (2, 3), (5, 2), (3, 3), (5, 3)} and 5  1 1  x 1 x
= ∫ − 2
e dx = e +C

.(d)(c)E{(1,{(1,ach chapter is divided into topics and  x + 4 ( x + 4)  x+4


∫ e {cosec
x 2
3), (1, 5), (2, 3), (2, 5), (3, 3), (3, 5) and 6 = x + ( − cot x )}dx
 e x [ f ( x) + f ′ ( x)] dx = e x f ( x) 
3), (5, 1), (2, 3), (2, 5), (3, 3), (5, 3)} and 6  ∫ 
{ ( )}
explained We are given two sets: A = {1, separately.
= − ∫ e cot x + −cosec x dx
x 2

Solution. 2, 3} and B = {3, 5} 2


 log x − 1  (log x − 1) 2
 e x { f ( x) + f ′ ( x)} dx = e x f ( x)  3. (a) I = ∫  2 
dx = ∫ dx
A × B = {1, 2, 3} × {3, 5} = {(1, 3), (1, 5), (2, 3), (2, 5), (3, 3), (3, 5)} = –ex cot x + C  ∫
. Exercise is segregated according  1 + (log x )  {1 + (log x ) 2 }2
.We areChapter
given: n(A) = 3 and n(B) Map representation of the chapter.

=2
216 MatheMatics–12
\ n(B × A) = n(B) × n(A) = 2 × 3 = 6
Hence, option (c) is the correct answer.
to different variety of objective
(ii) B × A = {(2, 1), (2, 3), (2, 5), (4, 1), (4, 3), (v) n(A) = 3
11
(4, 5), (6, 1), (6, 3), (6, 5), (8, 1), (8, 3), (8, 5)} (vi) n(B) = 4
and subjective questions. Also
(iii) A × A = {(1, 1), (1, 3), (1, 5), (3, 1), (3, 3), (vii) n(A × B) = 12 (viii) n(B × A) = 12 includes Previous year board
(3, 5), (5, 1), (5, 3), (5, 5)}
(iv) B × B = {(2, 2), (2, 4), (2, 6), (2, 8), (4, 2),
(ix) n(A × A) = 9 (x) n(B × B) = 16 questions and HOTS questions.
(xi) A × B ≠ B × A
(4, 4), (4, 6), (4, 8), (6, 2), (6, 4), (6, 6), (6, 8),
(xii) n(A × B) = n(B × A)
(8, 2), (8, 4), (8, 6), (8, 8)}

1. relations
Relation from Set A to Set B
Let A and B be two non-empty sets, then the relation from set A to set B is a subset of set A × B. For
example, if set A = {1, 2, 3} and set B = {3, 4, 5}, then there can be many relations from set A to set B.
R1 = {1, 3), (2, 5)}; R2 = {(1, 5), (2, 4), (2, 3); R3 = {(1, 3), (1, 4), (1, 5), (2, 5)}; etc. are some of the
relations from set A to set B.
Number of all Possible Relations from Set A to Set B
Number of elements in set A = n(A). Number of elements in set B = n(B)
\ Number of ordered pairs in (A × B) = n(A × B) = n(A) × n(B) . Topic wise concepts are
\ Number of all possible relations from set A to set B = 2n(A × B) = 2n(A) × n(B).
Example 1. How many relations are possible from set A = {1, 2, 3} to set B = {6, 7, 8, 9}?
presented to remember them
Solution. We have A = {1, 2, 3}, B = {6, 7, 8, 9} easily.
⇒ n(A) = 3, n(B) = 4 ⇒ n(A × B) = 3 × 4 = 12
\ Number of possible relations from set A to set B = 212.
Domain of a Relation R
Relation R from set A to set B is a subset of set of (A × B) and A × B is a set of all possible ordered pairs
(a, b), where a ∈ A and b ∈ B. Now, a set of all the first coordinates of all the ordered pairs of relation
R is called the domain of relation R.
Or D = {a : (a, b) ∈ R}
In relation: R = {(2, 1), (3, 7), (5, 8)}, Domain = {2, 3, 5} . Each topic is well explained
Range of a Relation R with relevant example for better
Range of a relation R is a set of all the second coordinates of all the ordered pairs of relation R.
Or Range = {b : (a, b) ∈ R}
understanding.
In relation: R = {(3, 1), (7, 8), (–9, 5)}, Range = {1, 8, 5}
Example 2. If set A = {2, 3, 4, 7} and set B = {3, 4, 5, 6}. A relation R from set A to set B is defined as:
R : {(a, b): b is divisible by a and a ∈ A and b ∈ B}. Write the relation in roster form and with arrow
diagram. Also write the domain and range of relation R.
Solution. The given sets are: A = {2, 3, 4, 7} and B = {3, 4, 5, 6}
(i) R = {(2, 4), (2, 6), (3, 3), (3, 6), (4, 4)}
This is the roster form of the relation R.

(ii) Arrow diagram of relation R

(iii) Domain of R = {2, 3, 4} (iv) Range of R = {3, 4, 6}

Relations and Functions 13

(iv)
then a ⊗ e = a and e ⊗ a = a.
2. Inverse of an element a of ⊗ : A × A → A a binary operation: If a and b ∈ A and are inverse of
each other under the ⊗ operation then
a ⊗ b = e and b ⊗ a = e. COMMON ERRORS
3. Number of all possible relations from set A to set B = 2n(A × B) = 2n(A) × n(B).
ERRORS CORRECTIONS
4. (i) Commutative binary operations for ⊗ : A × A → A,
(i) In a set A = {1, 2, 3}, a relation R = {(1, 1), (2, 2), (3, (i) In reflexive relation, every element of set A must be related
a ⊗ b = b ⊗ a ∀ a, b ∈ A 2)}. Here R is not reflexive relation because (3, 3) is not to itself.
(ii) Associative binary operations for ⊗ : A × A → A, present in R. Students make a mistake that if only (1, 1)
(a ⊗ b) ⊗ c = a ⊗ (b ⊗ c) ∀ a, b, c ∈ A. or (1, 1), (2, 2) are present R is reflexive.
(ii) To check the symmetric relation R. (ii) Find R–1, if R = R–1 only then R is symmetric.
(iii) To check the transitivity of a relation R. (iii) Its every ordered pair must be tested.
(iv) If x ∈ N and x12 = x22 ⇒ x1 = ± x2 (iv) Here x1 = x2, because x1 = –x2 is not possible because x is
a natural number.
. Quite effective for a quick(v) revision
If x ∈ R and x before
=x ⇒ x exams.
=x 1
2
2
2
1 2 (v) Here x1 = ± x2, because x is a real number, here negative
number is possible.
Have the complete essence (vi) Whileof theinjectivity,
checking chapter. surjectivity of a function, (vi) The first step while checking injectivity, surjectivity, take
students ignore the set in which function is defined, i.e., f care of set under consideration.
: N → N or
f : Z → Z or f : R → R etc
Common errors (vii) Identification of co-domain, range (vii) Co-domain is the set of all second elements, i.e., if
f : A → B, then B is the co-domain.
errors CorreCtions Range is the set of all second entries of the ordered pairs.
(i) In a set A = {1, 2, 3}, a relation R = {(1, 1), (i) In reflexive relation, every element of set A
(2, 2), (3, 2)}. Here R is not reflexive relation must be related to itself.
because (3, 3) is not present in R. Students
make a mistake that if only (1, 1) or (1, 1), REVISION CHART—RELATIONS
(2, 2) are present R is reflexive.
–1 –1
empty relation reflexive relation
(ii) To check the symmetric relation R. (ii) Find R , if R = R only then R is symmetric. A relation R in set A is called empty A relation R in set A is called reflexive
(iii) To check the transitivity of a relation R. (iii) Its every ordered pair must be tested. relation if no element of set A related if (a, a) ∈ R for all a ∈ A or R is reflexive
to any element of set A, i.e, R = f. in set A if (a, a) ∈ R ∀ a ∈ A.
(iv) If x ∈ N and x12 = x22 ⇒ x1 = ± x2 (iv) Here x1 = x2, because x1 = –x2 is not possible
because x is a natural number.
(v) If x ∈ R and x12 = x22 ⇒ x1 = xuthor 2 A ’ C
Here x1 = ± x2, because x is a real number,
s (v)omments
here negative number is possible. relationS
(vi) Questionsof
While checking injectivity, surjectivity based
(vi)on following
The first steptypes
whileare very important
checking for Exams. So, students
injectivity, are advised
(i) Relation R from set A toto
setrevise
B: Relationthem
R from setthoroughly.
A to set B is a subset of Cartesian product (A × B)
or Relation from set A to set B = {(a, b) : a ∈ A and b ∈ B}
a function, students ignore the set 1. inTowhich
express a square matrix take
surjectivity, intocare
sumofofset
symmetric and skew-symmetric matrices.
under considered. If A = {1, 2} and B = {3, 4} then R = {(1, 3), (2, 4)}
function is defined, i.e., f : N → N 2. orQuestions based on property |adj A| = |A|n – 1. (ii) Relation R in set A: Relation R in set A is a subset of Cartesian product (A × A).
f : Z → Z or f : R → R etc 3. To find inverse of a square matrix. (Most Important) If A = {1, 2, 3} then Relation R = {(1, 1), (1, 2), (2, 3)}

(vii) Identification of co-domain, range (vii) Co-domain is the set of all second elements,
i.e., if f : A → B, then B is the co-domain.
Range is the set of all second entries of the Symmetric relation equivalence relation tranSitive relation
ordered pairs. COMMON ERRORS A relation R in set A is said to be If a relation R in set A is reflexive,
symmetric if (a, b) ∈ R and (b, a) ∈ R,
A relation R in set A is said to be
symmetric and transitive, then transitive if (a, b) ∈ R and (b, c) ∈ R
a, b ∈ A or if relation R = its inverse relation R is called an equivalence ⇒ (a, c) ∈ R for a, b, c ∈ A.
Errors and Functions 43
Relations
symmetric relation.
Corrections
relation R–1, then relation R is called relation.

(i) Wrong notation for matrices. (i) Matrices: ( ) or [ ].


(ii) Solution of equations: (ii) If AX = B, then X = A–1B.
If AX = B, then X = BA–1 is wrong.
.Common errors have been tagged to clear confusions
with cautions (iii) Not multiplying the first row elements with the second
andmatrix
answers for productive learning.
corresponding column elements and add
(iii) By memorising tips like Run and Jump remember while
32 MatheMatics–12
multiplying two matrices. More practice on various order
matrices for multiplication.

IMPORTANT FORMULAE
1. Multiplication of a Matrix by a Scalar Number
If A = [aij]m×n is a matrix and k is any scalar number, then kA = k[aij]m×n = [kaij]m×n.
2. Addition of Matrices
If matrix A = [aij]m×n and matrix B = [bij]m×n, then
A + B = [aij]m×n + [bij]m×n = [aij + bij]m×n.
3. Subtraction of Matrices
If matrix A = [aij]m×n and matrix B = [bij]m×n then
(A – B) = [aij]m×n – [bij]m×n = [aij – bij]m×n
4. Multiplication of a matrix with another matrix
[aij]m×n × [bjk]n×p = [cik]m×p
5. If A–1 is the inverse matrix of matrix A then AA–1 = A–1A = I.
6. If A′ is the transpose of matrix A, then
(i) (A′)′ = A (ii) (A + B)′ = A′ + B′
(iii) (A – B)′ = A′ – B′ (iv) (AB)′ = B′A′
(v) (ABC)′ = C′B′A′.
7. If A is a square matrix, then
(i) (A + A′) is a symmetric matrix. (ii) (A – A′) is a skew-symmetric matrix.
(iii) (AA′) and (A′A) are symmetric matrix.
8. Any square matrix A can be expressed as a sum of symmetric and skew-symmetric matrix:

A= 1
( A + A′ ) +
2 

1
(
A − A′
2 

)
Symmetric matrix Skew-symmetric matrix

Errors 73
.Important Formulae are given at the
end of the each chapter.

(v)
Dedicated to the everlasting memory

of my mother Shanti Devi and Grandmother Gopali Devi

who were a perpetual source of inspiration for me.


Syllabus

One Paper Max. Marks: 80


Units No. of Periods Marks
I. Relations and Functions 30 08
II. Algebra 50 10
III. Calculus 80 35
IV. Vectors and Three - Dimensional Geometry 30 14
V. Linear Programming 20 05
VI. Probability 30 08
Total 240 80
Internal Assessment 20

UNIT I: RELATIONS AND FUNCTIONS


1. Relations and Functions (15 Periods)
Types of relations: reflexive, symmetric, transitive and equivalence relations. One to one and onto functions.
2. Inverse Trigonometric Functions (15 Periods)
Definition, range, domain, principal value branch. Graphs of inverse trigonometric functions.

UNIT II: ALGEBRA


1. Matrices (25 Periods)
Concept, notation, order, equality, types of matrices, zero and identity matrix, transpose of a matrix, symmetric and skew symmetric
matrices. Operation on matrices: Addition and multiplication and multiplication with a scalar. Simple properties of addition,
multiplication and scalar multiplication. On-commutativity of multiplication of matrices and existence of non-zero matrices whose
product is the zero matrix (restrict to square matrices of order 2). Invertible matrices and proof of the uniqueness of inverse, if it
exists; (Here all matrices will have real entries).
2. Determinants (25 Periods)
Determinant of a square matrix (up to 3 × 3 matrices), minors, co-factors and applications of determinants in finding the area
of a triangle. Adjoint and inverse of a square matrix. Consistency, inconsistency and number of solutions of system of linear
equations by examples, solving system of linear equations in two or three variables (having unique solution) using inverse
of a matrix.

UNIT III: CALCULUS


1. Continuity and Differentiability (20 Periods)
Continuity and differentiability, chain rule, derivatives of inverse trigonometric functions, like sin x, cos x and tan–1 x,
–1 –1

derivative of implicit functions. Concept of exponential and logarithmic functions.


Derivatives of logarithmic and exponential functions. Logarithmic differentiation, derivative of functions expressed in parametric
forms. Second order derivatives.
2. Applications of Derivatives (10 Periods)
Applications of derivatives: rate of change of bodies, increasing/decreasing functions, maxima and minima (first derivative test
motivated geometrically and second derivative test given as a provable tool). Simple problems (that illustrate basic principles
and understanding of the subject as well as real-life situations).

(vii)
3. Integrals (20 Periods)
Integration as inverse process of differentiation. Integration of a variety of functions by substitution, by partial fractions and
by parts, Evaluation of simple integrals of the following types and problems based on them.
dx dx dx dx dx
∫ x2 ± a2 , ∫ 2
x ±a 2
,∫
2
a −x 2
,∫
ax 2 + bx + c ∫ ax 2 + bx + c
,

px + q px + q
∫ ax 2 + bx + c dx, ∫ 2
ax + bx + c
dx, ∫ a 2 ± x 2 dx,∫ x 2 − a 2 dx

∫ ax + bx + c dx
2

Fundamental Theorem of Calculus (without proof). Basic properties of definite integrals and evaluation of definite integrals.
4. Applications of the Integrals (15 Periods)
Applications in finding the area under simple curves, especially lines, circles/parabolas/ellipses (in standard form only).
5. Differential Equations (15 Periods)
Definition, order and degree, general and particular solutions of a differential equation. Solution of differential equations by
method of separation of variables, solutions of homogeneous differential equations of first order and first degree. Solutions
of linear differential equation of the type:
dy
+ py = q, where p and q are functions of x or constants.
dx
dx + px = q, where p and q are functions of y or constants.
dy

UNIT IV: VECTORS AND THREE-DIMENSIONAL GEOMETRY


1. Vectors (15 Periods)
Vectors and scalars, magnitude and direction of a vector. Direction cosines and direction ratios of a vector. Types of vectors
(equal, unit, zero, parallel and collinear vectors), position vector of a point, negative of a vector, components of a vector,
addition of vectors, multiplication of a vector by a scalar, position vector of a point dividing a line segment in a given ratio.
Definition, Geometrical Interpretation, properties and application of scalar (dot) product of vectors, vector (cross) product of
vectors.
2. Three - dimensional Geometry (15 Periods)
Direction cosines and direction ratios of a line joining two points. Cartesian equation and vector equation of a line, skew
lines, shortest distance between two lines. Angle between two lines.

UNIT V: LINEAR PROGRAMMING


1. Linear Programming (20 Periods)
Introduction, related terminology such as constraints, objective function, optimization, graphical method of solution for
problems in two variables, feasible and infeasible regions (bounded and unbounded), feasible and infeasible solutions, optimal
feasible solutions (up to three non-trivial constraints).

UNIT VI: PROBABILITY


1. Probability (30 Periods)
Conditional probability, multiplication theorem on probability, independent events, total probability, Bayes’ theorem, Random
variable and its probability distribution, mean of random variable.

(viii)
Question Paper Design for Mathematics (Class XII)
Time : 3 Hours Max. Marks : 80
Total
S. No. Typology of Questions % Weightage
Marks

1. Remembering: Exhibit memory of previously learned material


by recalling facts, terms, basic concepts, and answers.
Understanding: Demonstrate understanding of facts and ide- 44 55
as by organizing, comparing, translating, interpreting, giving
descriptions, and stating main ideas

2. Applying: Solve problems to new situations by applying ac-


20 25
quired knowledge, facts, techniques and rules in a different way.

3. Analysing :
Examine and break information into parts by identifying mo-
tives or causes. Make inferences and find evidence to support
generalizations
Evaluating:
Present and defend opinions by making judgments about in- 16 20
formation, validity of ideas, or quality of work based on a set
of criteria.
Creating:
Compile information together in a different way by combining
elements in a new pattern or proposing alternative solutions

TOTAL 80 100

1. No chapter wise weightage. Care to be taken to cover all the chapters


2. Suitable internal variations may be made for generating various templates keeping the overall weightage to different
form of questions and typology of questions same.
Choice(s):
There will be no overall choice in the question paper.
However, 33% internal choices will be given in all the sections

(ix)
CONTENTS

1. Relations and Functions...........................................................................................................................................11

2. Inverse Trigonometric Functions............................................................................................................................ 34

3. Matrices................................................................................................................................................................... 44

4. Determinants........................................................................................................................................................... 75

5. Continuity And Differentiability............................................................................................................................. 98

6. Application of Derivatives.................................................................................................................................... 137

7. Integrals................................................................................................................................................................. 178

8. Area Between Curves............................................................................................................................................ 246

9. Differential Equations........................................................................................................................................... 277

10. Vector Algebra....................................................................................................................................................... 309

11. Three Dimensional Geometry............................................................................................................................... 335

12. Linear Programming............................................................................................................................................. 355

13. Probability............................................................................................................................................................. 377

• Sample Paper-1 (Solved).................................................................................................................................. 410


• Sample Paper-2 (Unsolved).............................................................................................................................. 427


• Sample Paper-3 (Unsolved).............................................................................................................................. 434


(x)
E:\AMIT_WORKS\Exam_Guru\EG_Mathematics-12_(working_02-06-2022)\EG_Mathematics-12_working\Open_Files\Chap_1\Chap_1
\ 16-Aug-2022 Amit Proof-5 Reader’s Sign _______________________ Date __________


Topics
C
overed
1 Relations and Functions

1.1 Relations 1.2 Types of Relations


1.3 Functions

C hapter map
RELATIONS

Type of Relations

Universal Empty Reflexive Symmetric Transitive Equivalence

Equivalence Relation

Functions

Types of Functions

One-one/Injective Function Onto/Surjective Function Domain and Co-domain


of the Function

Bijective Function

Cartesian Product of Sets (Recap)


Let A and B be two non-empty sets. A set of all the ordered pairs (a, b) such that a ∈ A and b ∈ B is called cartesian product of set
A with set B and denoted as (A × B).
Or A × B = {(a, b) : a ∈ A and b ∈ B)}
The number of ordered pairs in (A × B) = n(A × B) = n(A) × n(B)
= (number of elements in set A) × (number of elements in set B).

Example 1. Given set A = {1, 2, 3} and set B = {3, 5}. Value of A × B and n(B × A) respectively are given by
(a) {(1, 3), (1, 5), (2, 3), (2, 5), (3, 3), (5, 3)} and 6
(b) {(1, 3), (1, 5) (2, 3), (5, 2), (3, 3), (5, 3)} and 5
(c) {(1, 3), (1, 5), (2, 3), (2, 5), (3, 3), (3, 5) and 6
(d) {(1, 3), (5, 1), (2, 3), (2, 5), (3, 3), (5, 3)} and 6
Solution. We are given two sets: A = {1, 2, 3} and B = {3, 5}
A × B = {1, 2, 3} × {3, 5} = {(1, 3), (1, 5), (2, 3), (2, 5), (3, 3), (3, 5)}

We are given: n(A) = 3 and n(B) = 2
\ n(B × A) = n(B) × n(A) = 2 × 3 = 6
Hence, option (c) is the correct answer.

11
E:\AMIT_WORKS\Exam_Guru\EG_Mathematics-12_(working_02-06-2022)\EG_Mathematics-12_working\Open_Files\Chap_1\Chap_1
\ 16-Aug-2022 Amit Proof-5 Reader’s Sign _______________________ Date __________

RECAP EXERCISE
I. Multiple Choice Questions (MCQs)
Choose the correct answer from the given options.
Given for Q1 to Q6: Set A = {–1, 0, 1} and Set B = {3, 7, 8}
1. The value of A × B will be
(a) {(–1, 0), (–1, –1), (–1, 3), (0, –1), (1, –1)}
(b) {(3, –1), (7, –1), (8, –1), (3, 0), (7, 0), (8, 0), (3, 1), (7, 1), (8, 1)}
(c) {(–1, 3), (–1, 7), (–1, 8), (3, 0), (7, 0), (8, 0), (3, 1), (7, 1), (8, 1)}
(d) {(–1, 3), (–1, 7), (–1, 8), (0, 3), (0, 7), (0, 8), (1, 3), (1, 7), (1, 8)}
2. The value of (B × A) is
(a) {(3, –1), (3, 0), (3, 1), (7, –1), (7, 0), (7, 1), (8, –1), (8, 0), (8, 1)}
(b) {(–1, 3), (0, 3), (1, 7), (–1, 7), (0, 7), (1, 7), (–1, 8), (0, 8), (1, 8)}
(c) {(3, 0), (3, –1), (7, 0), (–1, 7), (8, –1), (1, 7), (1, 8), (0, 8)}
(d) None of these
3. The value of A × A equals
(a) {(1, 1), (1, 0), (1, 1), (0, 1), (0, 0), (0, –1), (–1, 1), (–1, 0), (1, –1)}
(b) {(–1, 1), (1, 0), (1, –1), (0, 1), (0, 0), (0, –1), (1, 1), (1, 0), (1, 1)}
(c) {(–1, –1), (–1, 0), (–1, 1), (0, –1), (0, 0), (0, 1), (1, –1), (1, 0), (1, 1)}
(d) None of these
4. n(A) is equal to
(a) 1 (b) 2 (c) 3 (d) 6
5. n(A × B) is equal to
(a) 3 (b) 6 (c) 9 (d) None of these
6. n(A × A) is equal to
(a) 3 (b) 6 (c) 9 (d) None of these
Given for Q7 to Q8: Set A = {2, 3, 4}
7. (A × A ) is equal to
(a) {(2, 2), (2, 3), (2, 4), (3, 2), (3, 3), (3, 4), (4, 2), (4, 3), (4, 4)}
(b) {(2, 2), (2, 3), (2, 4), (2, 3), (3, 3), (4, 3), (2, 4), (3, 4), (4, 4)}
(c) {(2, 2), (3, 2), (4, 2), (2, 3), (3, 3), (3, 4), (4, 2), (4, 3), (4, 4)}
(d) None of these
8. n(A) is equal to
(a) 1 (b) 2 (c) 3 (d) 6

Answers and Hints


1. (d) {(–1, 3), (–1, 7), (–1, 8), (0, 3), (0, 7), (0, 8), (1, 3), 3. (c) {(–1, –1), (–1, 0), (–1, 1), (0, –1), (0, 0), (0, 1), (1, –1),
 (1, 7), (1, 8)}  (1, 0), (1, 1)}
4. (c) 3 5. (c) 9 6. (c) 9
2. (a) {(3, –1), (3, 0), (3, 1), (7, –1), (7, 0), (7, 1), (8, –1),
7. (a) {(2, 2), (2, 3), (2, 4), (3, 2), (3, 3), (3, 4), (4, 2), (4, 3), (4, 4)}
 (8, 0), (8, 1)} 8. (c) 3

Topic 1. Relations
Relation from Set A to Set B
Let A and B be two non-empty sets, then the relation from set A to set B is a subset of set (A × B). For example, if set A = {1, 2, 3}
and set B = {3, 4, 5}, then there can be many relations from set A to set B. R1 = {1, 3), (2, 5)}; R2 = {(1, 5), (2, 4), (2, 3)}; R3 = {(1, 3),
(1, 4), (1, 5), (2, 5)}; etc. are some of the relations from set A to set B.

12 Mathematics–12
E:\AMIT_WORKS\Exam_Guru\EG_Mathematics-12_(working_02-06-2022)\EG_Mathematics-12_working\Open_Files\Chap_1\Chap_1
\ 16-Aug-2022 Amit Proof-5 Reader’s Sign _______________________ Date __________

Number of all Possible Relations from Set A to Range of a Relation R


Set B Range of a relation R is a set of all the second coordinates of all
Number of elements in set A = n(A). Number of elements in set the ordered pairs of relation R.
B = n(B) Or Range = {b : (a, b) ∈ R}
In relation: R = {(3, 1), (7, 8), (–9, 5)}, Range = {1, 8, 5}
\ Number of ordered pairs in (A × B) = n(A × B)
Example 2. Given: set A = {2, 3, 4, 7} and set B = {3,
= n(A) × n(B) 4, 5, 6}. A relation R from set A to set B is defined as:
\ Number of all possible relations from set A to set R : {(a, b): b is divisible by a and a ∈ A and b ∈ B}. Domain of
relation R will be given by
B = 2n(A × B) = 2n(A) × n(B).
(a) {2, 4, 5} (b) {2, 3, 5} (c) {3, 5, 6} (d) {2, 3, 4}
Example 1. Number of relations possible from set A = {1, 2, 3} Solution. The given sets are: A = {2, 3, 4, 7} and B = {3, 4, 5, 6}
to set B = {6, 7, 8, 9} is given by R = {(2, 4), (2, 6), (3, 3), (3, 6), (4, 4)}
(a) 210 (b) 28 (c) 212 (d) 211 Domain of R = set of all first coordinates of all the ordered
Solution. We have A = {1, 2, 3}, B = {6, 7, 8, 9} pairs of relation R = {2, 3, 4}
⇒ n(A) = 3, n(B) = 4 ⇒ n(A × B) = n(A) × n(B) = 3 × 4 = 12 Hence, option (d) is the correct answer.
\ Number of possible relations from set A to set B = 212. Example 3. A relation R in set A = {1, 2, 3, 4, 5} is defined as:
R = {(a, b) : a is an odd number and b is an even number and a
Hence, option (c) is the correct answer.
∈ A and b ∈ A}. Range of R will be
Domain of a Relation R (a) {2, 3} (b) {2, 4} (c) {5, 4} (d) {1, 5}
Relation R from set A to set B is a subset of set (A × B) and Solution. The given set is A = {1, 2, 3, 4, 5}
A × B is a set of all possible ordered pairs (a, b), where a ∈ A In roster form R = {(1, 2), (1, 4), (3, 2), (3, 4), (5, 2),
and b ∈ B. Now, a set of all the first coordinates of all the ordered (5, 4)}
pairs of relation R is called the domain of relation R. \ Range of R = set of all second coordinates of all the ordered
Or D = {a : (a, b) ∈ R} pairs of relation R = {2, 4}.
In relation: R = {(2, 1), (3, 7), (5, 8)}, Domain = {2, 3, 5} Hence, option (b) is the correct answer.

EXERCISE 1.1
I. Multiple Choice Questions (MCQs) 4. A relation R is defined in set A = {1, 3, 5, 7, 9} such that
Choose the correct answer from the given options. R = {(a, b) : a + b ≤ 10} a ∈ A, b ∈ A}. Domain of the
relation R is given by
1. Number of relations possible in set A having 3 elements
will be (a) {1, 3, 5, 8, 9} (b) {1, 3, 5, 7, 9}
(a) 26 (b) 29 (c) 28 (d) 23 (c) {1, 3, 6, 7, 9} (d) {1, 6, 7, 9, 5}
2. Number of relations possible in set A = {2, 4, 7, 9} is given by II. Short Answer Type Questions-I
(a) 2 10
(b) 2 8
(c) 2 12
(d) 2 16 1. A relation R is defined from set A = {1, 3, 4, 5} to
3. A relation R is defined from set A = {1, 2, 3} to set B = {4, set B = {2, 4, 6} as R = {(a, b) : b is divisible by a}. Here,
5, 6, 7}. Number of relations possible from set A to set B a ∈ A and b ∈ B. Write the relation R in
will be (i) Roster form (ii) domain of R and
(a) 2 7
(b) 2 8
(c) 2 12
(d) 2 10 (iii) Range of the relation R

Answers 1.1
I. 1. (b) 29 4. (b) {1, 3, 5, 7, 9}

n(A) × n(A)
2 = 23 × 3 = 29 II. 1. (i) Relation R in roster form
16
2. (d) 2 R = {(1, 2), (1, 4), (1, 6), (3, 6), (4, 4)}
n(A) × n(A)
2 = 24 × 4 = 216
12 (ii) Domain of relation R = {1, 3, 4}
3. (c) 2
n(A) × n(B) (iii) Range of relation R = {2, 4, 6}
2 = 23 × 4 = 212

Topic 2. Types of Relations


Here, we shall discuss following types of relations: (i) Empty relation, (ii) Universal relation, (iii) Identity relation, (iv) Reflexive
relation, (v) Symmetric relation, (vi) Transitive relation and (vii) Equivalence relation.
(i) (a) Empty Relation from Set A to Set B: Let A and B be two non-empty sets. Then the relation R from set A to set B is called
an empty relation if no element of set A is related to any element of set B.

Relations and Functions 13


E:\AMIT_WORKS\Exam_Guru\EG_Mathematics-12_(working_02-06-2022)\EG_Mathematics-12_working\Open_Files\Chap_1\Chap_1
\ 16-Aug-2022 Amit Proof-5 Reader’s Sign _______________________ Date __________

For example, set A= {1, 2, 3, 4} and set B = {5, 6, 7}. is symmetric because (2, 3), (3, 2) and (2, 4), (4, 2) are
Now, consider a relation R from set A to set B defined present whereas relation
as: R = {(a, b) : a + b = 15, a ∈ A and b ∈ B}. R2 = {(2, 3), (2, 4), (3, 2), (4, 2), (3, 4)}
This is an empty relation as no pair (a, b) satisfies the is not a symmetric relation because for ordered pair (3, 4),
condition: a + b = 15. ordered pair (4, 3) is not present.
∴ R = φ ⊂ (A× B). (vi) Transitive Relation: A relation R in set A is called
(b) Empty Relation in Set A: Relation R in a non-empty transitive if for ordered pairs (a, b) and (b, c) present in
set A is called empty relation if no element of set A is relation R, there must be a third ordered pair (a, c) also
related to any element of set A i.e., R = φ ⊂ (A × A). present in relation R. Here a, b, c ∈ A
Consider a set A={1, 2, 3, 4}. Now, define a relation or A relation R in set A is called transitive if (a, b) ∈ R and
R in set A as: (b, c) ∈ R
R = {(a, b) : a – b = 10, a, b ∈ A}. ⇒ (a, c) ∈ R for a, b, c ∈ A
This is an empty set as no pair (a, b) satisfies the Take a set A = {1, 2, 3} and take a relation R1 in set A
condition a – b = 10. defined as R = {(1, 2), (2, 3), (1, 3), (3, 3), (3, 2)} is
(ii) (a) Universal Relation from Set A to Set B: Let A and transitive because
B be two non-empty sets, then the relation R from (1, 2) ∈ R and (2, 3) ∈ R ⇒ (1, 3) ∈ R
set A to set B is called universal relation if each element (2, 3) ∈ R and (3, 3) ∈ R ⇒ (2, 3) ∈ R
of set A is related to every element of set B, i.e., (1, 3) ∈ R and (3, 3) ∈ R ⇒ (1, 3) ∈ R
R = A × B. (3, 3) ∈ R and (3, 2) ∈ R ⇒ (3, 2) ∈ R
Take two sets, A = {1, 2, 3, 4} and B = {5, 6, 7}. Now, (3, 2) ∈ R and (2, 3) ∈ R ⇒ (3, 3) ∈ R
define a relation R from set A to set B as: (vii) Equivalence Relation: A relation R in set A is said to be
R = {(a, b) : |a – b| ≥ 0, a ∈ A and b ∈ B} is the universal an equivalence relation if R is reflexive, symmetric and
relation as each element of set A is related to every transitive.
element of set B. For example, a relation R defined in set Z of integers.
(b) Universal Relation in Set A: A relation R in set A is R = {(x, y) : x + y is an integer x ∈ Z and y ∈ Z} is an
called a universal relation if each element of set A is equivalence relation.
related to every element of set A i.e., R = A × A.
Take a set A = {2, 3, 4}. Relation R in set A is defined Reason
as: R = {(a, b : a + b < 10, a, b ∈ A}, is a universal (i) Reflexive: Take any integer x. Since, sum of two integers
relation as each element of set A is related to every is an integer, therefore x + x is an integer.
element of set A. \ (x, x) ∈ R ⇒ R is reflexive.
(iii) Identity Relation: Let A be a set, then relation IA on A is (ii) Symmetric: Take any two integers x and y. Since, sum of
called identity relation if every element of set A is related any two integers is an integer, therefore (x + y) and (y + x)
to itself only are integers.
or IA = {(a, a) : a ∈ A} \ (x, y) ∈ R and (y, x) ∈ R ⇒ R is symmetric.
For example: Take A = {1, 2, 3} then relation R = {(1, 1), (iii) Transitive: Take any three integers x, y and z. Since, sum
(2, 2), (3, 3)} is an identity relation on set A. of any two integers is an integer, therefore x + y, y + z and
(iv) Reflexive Relation: A relation R on set A is said to be x + z are integers.
reflexive if every element of set A is related to itself. Thus, \ (x, y) ∈ R and (y, z) ∈ R
R on a set A is not reflexive if there exists an element a ∈ ⇒ (x, z) ∈ R. So, relation R is transitive.
A such that (a, a) ∉ R. Above discussion proves that R is reflexive, symmetric and
Note: Every identity relation is reflexive but every reflexive transitive, therefore R is an equivalence relation.
relation is not identity relation. In set A = {2, 3} Example 1. A relation R in the set of natural number N, let R
(a) R1 = {(2, 2), (3, 3)} is identity and reflexive relation be defined as:
and R = {(x, y) : x + y = a natural number, x, y ∈ N}. R is
(b) R2 = {(2, 2), (3, 3), (2, 3)} is reflexive relation but not (a) Equivalence relation
identity relation due to presence of (2, 3). (b) Transitive, symmetric, not reflexive
(v) Symmetric Relation: A relation R on set A is said to be (c) Reflexive, symmetric, not transitive
symmetric relation when for every ordered pair (a, b) (d) None of these
present in relation R, there must present an ordered pair Solution. We have N = set of all natural numbers and R = {(x, y)
(b, a) also in relation R, here a, b ∈ A. : x + y = a natural number, x ∈ N and y ∈ N}.
Take a set A = {2, 3, 4} and a relation R1 in set A defined (i) R is reflexive relation: Let x be any natural number, then
as (x + x) is also a natural number.
R1 = {(2, 3), (2, 4), (3, 2), (4, 2)} \ (x, x) ∈ R ⇒ R is reflexive relation.

14 Mathematics–12
E:\AMIT_WORKS\Exam_Guru\EG_Mathematics-12_(working_02-06-2022)\EG_Mathematics-12_working\Open_Files\Chap_1\Chap_1
\ 16-Aug-2022 Amit Proof-5 Reader’s Sign _______________________ Date __________

(ii) R is symmetric relation: Take any two natural numbers x Example 3. A relation R in the set of integers Z is defined as:
and y. Since, sum of any two natural numbers is a natural R = {(x, y) : x ≥ y and x, y ∈ Z}. The relation R is
number, therefore (x + y) and (y + x) are natural numbers. (a) reflexive and transitive but not symmetric
(x, y) ∈ R and (y, x) ∈ R (b) reflexive, symmetric but not transitive
⇒ R is symmetric relation. (c) equivalence
(iii) R is transitive relation: Take any three natural numbers x, (d) None of these.
y and z. Since, sum of any two natural numbers is a natural Solution. We have R = {(x, y) : x ≥ y and x, y ∈ Z}
number, therefore, (x + y), (y + z) and (x + z) are natural (i) Relation R is reflexive: Relation R is : x ≥ y, x, y ∈ Z. If
numbers. we put y = x in relation x ≥ y and it remains true, then R is
Hence, (x, y) ∈ R and (y, z) ∈ R ⇒ (x, z) ∈ R. reflexive. So, put y = x in x ≥ y. It gives:
This shows that R is transitive relation. x ≥ x ⇒ x = x. It is true for any integers.
Above discussion proves that R is reflexive, symmetric \ R is reflexive.
and transitive and hence R is an equivalence relation. (ii) Relation R is transitive: Relation R is: x ≥ y, x, y ∈ Z.
Hence, option (a) is the correct answer. Take any three integers, x, y, z such that x ≥ y, y ≥ z. This
Example 2. A relation R in the set of natural numbers N is gives: x ≥ z.
defined as: Thus, x ≥ y and y ≥ z ⇒ x≥z
R = {(x, y) : x2 – 4xy + 3y2 = 0, x, y ∈ N} \ (x, y) ∈ R and (y, z) ∈ R ⇒ (x, z) ∈ R.
The relation R is Thus, relation R is transitive.
(a) reflexive, not symmetric and not transitive (iii) Relation R is not symmetric: Relation R is: x ≥ y, x, y ∈
Z. If x ≥ y, then it is not possible: y ≥ x. It can be true only
(b) equivalence
for equality and not for all numbers.
(c) symmetric
Thus, R is not symmetric.
(d) None of these
Hence, option (a) is the correct answer.
Solution. We have R = {(x, y) : x2 – 4xy + 3y2 = 0, x, y ∈ N} and
N = set of all natural numbers Equivalence Classes
(i) R is reflexive relation: To prove a relation is reflexive, Assume that R is an equivalence relation in a non-empty set A.
put y = x in x2 – 4xy + 3y2 = 0. If the relation remains true, Let a ∈ A, then the set of all those elements of set A, which are
then the relation is reflexive. related to a, is called equivalence class determined by a and is
x2 – 4xy + 3y2 = 0 ⇒ x2 – 4x(x) + 3x2 = 0 denoted by [a].
⇒ x2 – 4x2 + 3x2 = 0 ⇒ 0 = 0 (True) Example 4. Take a relation in Z, i.e., in integers defined as R =
⇒ R is reflexive relation. {(x, y) : (x – y) is divisible by 2 ∀ x ∈ Z and y ∈ Z}. Relation R
(ii) R is not symmetric relation: If we put x for y and y for x is an equivalence relation. Find the equivalence classes of [0], [1].
in the relation: x2 – 4xy + 3y2 = 0 and this relation remains Solution. [0] = {x ∈ Z : (0, x) ∈ R or (x, 0) ∈ R} = {x ∈ Z :
true, then the relation R will be symmetric otherwise not. (x – 0) or (0 – x) is divisible by 2}
Let us put x for y and y for x in: x2 – 4xy + 3y2 = 0, we get = {x ∈ Z, and x is divisible by 2}
y2 – 4yx + 3x2 = 0 ⇒ 3x2 – 4xy + y2 = 0. = {x = ..., .., –4, –2, 0, 2, 4, ...,}
It is not the same as x – 4xy + 3y2 = 0.
2
\ [0] = {..., ..., –4, –2, 0, 2, 4, ...,} is equivalence class
\ Thus, relation R is not symmetric. of 0.
(iii) R is not transitive relation: Relation R : x2 – 4xy + 3y2 = [1] = {x ∈ Z : (1, x) ∈ R or (x, 1) ∈ R} = {x ∈ Z :
0 is true for (9, 3) and (3, 1) but not true for (9, 1) (1, x) or (x, 1) is divisible by 2}
\ (9, 3) ∈ R and (3, 1) ∈ R ⇒ (9, 1) ∈ R = {x ∈ Z : (x – 1) or (1 – x) is divisible by 2}
Thus, R is not transitive relation. = {x ∈ Z : ..., –3, –1, 1, 3, ....,}
Hence, option (a) is the correct answer. \ [1] = {..., –3, –1, 1, 3, ...,} is equivalence class of 1.

EXERCISE 1.2
I. Multiple Choice Questions (MCQs) 2. Let R be a relation over the set of straight lines in a plane
Choose the correct answer from the given options. such that l1 R l2 such that l1 ^ l2, then R is
1. R is a relation over the set N × N and it is defined by (a) symmetric (b) reflexive
(a, b) R(c, d) ⇒ a + d = b + c, then R is (c) transitive (d) equivalence relation
(a) reflexive only (b) symmetric only 3. A relation R is defined on plane A of triangles in a given
(c) transitive only (d) equivalence relation planes defined as R = {(T1, T2) : T1 @ T2 and T1, T2 ∈ A},

Relations and Functions 15


E:\AMIT_WORKS\Exam_Guru\EG_Mathematics-12_(working_02-06-2022)\EG_Mathematics-12_working\Open_Files\Chap_1\Chap_1
\ 16-Aug-2022 Amit Proof-5 Reader’s Sign _______________________ Date __________

then R is 4. Let R be the equivalence relation in the set


(a) reflexive (b) symmetric A = {0, 1, 2, 3, 4, 5} given by R = {(a, b) : 2 divides (a – b);
(c) transitive (d) equivalence a, b ∈ A}. Write the equivalence class of [0].
4. Consider the non-empty set consisting of children in a [Delhi 2014]
family and a relation R defined as a Rb if a is a brother of 5. Let A = {1, 2, 3, 4} and R be a relation in set A given by
b. Then R is R = {(1, 1), (2, 2), (3, 3), (4, 4), (1, 2), (2, 1), (3, 1), (1, 3)}
(a) symmetric but not transitive then show that the relation R is reflexive and symmetric.
(b) transitive but not symmetric 6. Show that the relation R less than in the set of natural
(c) neither symmetric nor transitive numbers is transitive.
(d) both symmetric and transitive. 7. For real numbers x and y, we write xRy ⇒ x – y + 2
5. Let R be the relation on the set R of all real numbers defined is an irrational number, then show that the relation R is
by a R b iff |a – b| ≤ 1. Then R is reflexive.
(a) reflexive and symmetric 8. State the reason why a relation R = {(a, b) : a > b, a, b ∈ real
(b) symmetric only numbers} defined in the set of real numbers, is transitive.
(c) transitive only 9. A relation R = {(x, y) : x + y < 10; x, y ∈ A} defined in the
(d) anti symmetric only set A = {1, 2, 3, 4}. Is relation R symmetric? Give reason.
6. A relation R = {(a, b) : a divides b} on the set N of all 10. Take a relation R = {(a, b) : a = b, a, b ∈ A} defined on a
natural numbers is set A of real numbers. Is R a reflexive relation? Give reason.
(a) reflexive (b) symmetric
III. Short Answer Type Questions-I
(c) reflexive and transitive (d) symmetric and transitive
1. If set A = {1, 2, 3, 4}, set B = {a, b, c, d} and relation
7. Let R be a relation on the set N defined by {(x, y) :
R = {(1, a), (2, b), (3, c), (4, d)} is a relation defined from
x, y ∈ N, 2x + y = 41}. Then R is
set A to set B. Write relation R–1.
(a) reflexive (b) symmetric
2. A relation R defined in set A = {1, 2, 3, ...., 10} as
(c) transitive (d) None of these.
R = {(x, y) : 3x – y = 0, x, y ∈ A}. Show that the relation
8. Let S be the set of all real numbers. Then the relation
is not reflexive.
R = {(a, b) : 1 + ab > 0} on S is
3. Let A = {a, b, c} and a relation R is defined in set A as:
(a) reflexive and symmetric but not transitive.
R = {(a, a), (b, c), (a, b)}. Write the minimum number of
(b) reflexive and transitive but not symmetric.
ordered pairs be included in relation R to make it reflexive
(c) symmetric and transitive but not reflexive. and symmetric. [HOTS]
(d) reflexive, symmetric and transitive. 4. Check whether the relation R defined in the set A = {1, 2, 3,
9. Let W denote the words in the English dictionary. The 4, 5, 6} as R = {(x, y) : y = x + 1, x, y ∈ A} is reflexive or
relation R is defined by R = {(x, y) ∈ W × W : the words symmetric.
x and y have at least one letter in common}.
5. Show that the relation: R = {(1, 2), (2, 3), (3, 2), (1, 3),
Then R is
(2, 1), (2, 2), (3, 3), (3, 1), (1, 1)} is reflexive, symmetric
(a) not reflexive, symmetric and transitive and transitive.
(b) reflexive, symmetric and not transitive 6. Show that the relation: R ={(1, 1), (1, 2), (2, 1), (2, 2)} is
(c) reflexive, symmetric and transitive reflexive, symmetric and transitive.
(d) reflexive, transitive and not symmetric 7. Show that the relation: R = {(1, 1), (3, 3), (5, 5), (1, 3),
10. On the set N of all natural numbers define the relation R (3, 1), (3, 5)} is reflexive and is neither symmetric nor
by aRb if and only if the G.C.D. of a and b is 2, then R is transitive.
(a) reflexive but not symmetric 8. Take a set A={1, 3, 7} and find a relation R = A × A, then
(b) symmetric only show that it is reflexive, symmetric and transitive.
(c) reflexive and transitive 9. Relation R defined in a set of natural numbers N as: R =
(d) reflexive, symmetric and transitive. {(x, y) : x > y ∀ x, y ∈ N}. Show that R is transitive and
II. Very Short Answer Type Questions it is neither reflexive nor symmetric.
1. State the reason why the relation R = {(a, b) : a ≥ b2} 10. Relation R defined in a set of natural numbers N as:
defined in the set of real numbers is not reflexive. R = {(x, y) : x ≥ y ∀ x, y ∈ N}. Show that relation R is
2. State the reason of the relation R in the set {1, 2, 3} given reflexive and transitive but not symmetric.
by: R = {(1, 2), (2, 1)} not to be transitive. [A.I. 2011] 11. Relation R defined in a set of natural numbers N as:
3. If R = {(x, y) : x + 2y = 8} is a relation on N. Write the R = {(a, b) : (a + b) is an even natural number, where a, b
domain of R.[A.I. 2011] ∈ N}. Show that R is reflexive, symmetric and transitive.

16 Mathematics–12
E:\AMIT_WORKS\Exam_Guru\EG_Mathematics-12_(working_02-06-2022)\EG_Mathematics-12_working\Open_Files\Chap_1\Chap_1
\ 16-Aug-2022 Amit Proof-5 Reader’s Sign _______________________ Date __________

12. Relation R defined in a set of natural numbers N as: elements in A related to the right angled triangle T with
R = {(a, b) : (a – b) is an even natural number and a, b sides 3, 4 and 5? [NCERT]
∈ N}. Show that R is neither reflexive nor symmetric but 7. Let L be the set of all lines in the XY plane and R be the
transitive. relation in L defined as: R = {(L1, L2) : L1 is parallel to L2
13. Relation R defined in a set of real numbers R as: and L1, L2 ∈ L}. Show that R is an equivalence relation.
R ={(x, y) : (x + y) is divisible by 5 ∀ x, y ∈ R}. Show Find the set of all lines related to line y = 2x + 4.
that R is neither reflexive nor transitive but symmetric. 8. Prove that the relation R in the set A ={1, 2, 3, 4, 5} given
Note: In a set of real numbers R, if relation R = {(a, b) : by R ={(a, b) : |a – b| and a, b ∈ A is an even number} is
(a + b) is a multiple of 3, 4, 5, 6, …, n ∀ a, b ∈ R} then R is an equivalence relation.
neither reflexive nor transitive but symmetric. 9. Show that the relation S in the set R of real numbers defined
14. Show that the relation R in set A ={1, 2, 3} given by as: S = {(a, b) : a, b ∈ R and a ≤ b3} is neither reflexive
R = {(1, 1), (2, 2), (3, 3), (1, 2), (2, 3)} is reflexive but nor symmetric nor transitive. [HOTS]
neither symmetric nor transitive.
10. Let R be a relation defined on the set of natural numbers
15. Let L be the set of lines in a plane and R be the relation in N as follows:
L defined as: R = {(L1, L2) : L1 is perpendicular to L2, L1
and L2 ∈ L}. Show that R is symmetric but neither reflexive R = {(x, y) : x, y ∈ N and 2x + y = 24}
nor transitive. Find the domain and range of the relation R. Also find R is
an equivalence relation or not. [Delhi 2014 (C)]
IV. Short Answer Type Questions-II
1. Let T be the set of all the triangles in a plane with R a V. Long Answer Type Questions
relation in T given by: R = {(T1, T2) : T1 is congruent to 1. Let N denote the set of all natural numbers and R be the
T2, T1 and T2 ∈ T}. Show that R is an equivalence relation. relation on N × N defined by (a, b) R (c, d) if ad(b + c) =
2. Show that the relation R in the set A of all the books in a bc(a + d). Show that R is an equivalence relation.
library of a college given by R = {(x, y) : x and y have the [Delhi 2015] [HOTS]
same number of pages and x, y ∈ A} is an equivalence 2. Let n be a fixed positive integer. Define a relation R in Z as
relation. follows ∀ a, b ∈ Z, aRb if and only if (a – b) is divisible
3. Show that each of the relation R in the set A = {x ∈ Z : 0 by n. Show that R is an equivalence relation. [HOTS]
≤ x ≤ 12} given by
3. If R and S are equivalence relation in set A, show that
(A) R = {(a, b) : |a – b| is a multiple of 4 and a, b ∈ A}
R ∩ S is also an equivalence relation. [HOTS]
(B) R = {(a, b) : a = b and a, b ∈ A}
4. Let R be a relation defined as
are equivalence relations.
R = {(x, y) : x, y ∈ N and 2x + y = 41}
Find the set of all elements related to 1 in each case.
[NCERT] Find the domain and range of R also verify that R is neither
reflexive, nor symmetric nor transitive.
4. Show that the relation R in the set A of points in a plane
given by: R = {(p, q) : distance of the point p from the 5. Given the relation R = {(1, 2), (2, 3)} in set A = {1, 2, 3}.
origin is same as the distance of the point q from the origin} Find the minimum number of ordered pairs which when
is an equivalence relation. Further, show that the set of added to R makes it an equivalence relation.
all points related to a point p ≠ (0, 0) is the circle passing 6. Give an example to show that union of two equivalence
through p with origin as centre. [NCERT] relation on set A is not necessarily equivalence on A.
5. Show that the relation R defined in the set A of all triangles  [HOTS]
as: R = {(T1, T2) : T1 is similar to T2 and T1, T2 ∈ T} is an 7. Let R be a relation defined in N × N defined by:
equivalence relation. Consider three right angled triangles
R = {(a, b) R (c, d) if and only if ad = bc and a, b, c,
T1 with sides 3, 4, 5, T2 with sides 5, 12, 13 and T3 with
sides 6, 8, 10. Which triangles among T1, T2 and T3 are d ∈ N}
related?[NCERT] Prove that R is an equivalence relation.[V. Imp.][HOTS]
6. Show that the relation R defined in the set A of all polygons 8. Let R be a relation on the set of integers given by aRb ⇒
as: R = {(P1, P2) : P1 and P2 have the same number of a = 2Kb for some integer K. Show that R is an equivalence
sides} is an equivalence relation. What is the set of all the relation.

Answers 1.2
I. 1. (d) an equivalence relation R is not symmetric because if a is a brother of b, then b may not
2. (a) symmetric
be a brother of a, b can be a sister of a.
3. (d) equivalence ∀ a, b ∈ A, (a, b) ∈ R does not imply (b, a) ∈ R. Therefore, R
4. (b) transitive but not symmetric is not symmetric.
Let A be the non-empty set consisting of children in a family Again, ∀ a, b, c ∈ A, (a, b) ∈ R and (b, c) ∈R
and a relation R in the set A given by aRb, if a is a brother of b. ⇒ (a, c) ∈ R

Relations and Functions 17


E:\AMIT_WORKS\Exam_Guru\EG_Mathematics-12_(working_02-06-2022)\EG_Mathematics-12_working\Open_Files\Chap_1\Chap_1
\ 16-Aug-2022 Amit Proof-5 Reader’s Sign _______________________ Date __________

For if a is a brother of b and b is a brother of c, then a is brother 1 4


 1
of c. Therefore, R is transitive. and 1 +  −  (– 2) = 1 + = >0
 6 3 3
Hence, R is transitive but not symmetric.
But 1 + 3 (–2) = 1 – 6 = – 5 >/ 0
5. (a) reflexive and symmetric
∴ R is not transitive.
∀ a ∈ R , |a – a| = 0 < 1, ∴ aRa
Hence, the relation R is reflexive and symmetric but not
⇒ R is reflexive. transitive.
Now, aRb  ⇒ |a – b| ≤ 1 9. (b) reflexive, symmetric and not transitive
⇒  |b – a| ≤ 1 [ |a – b| = |b – a|] W denote the words in the English dictionary.
⇒  bRa R = {(x, y) ∈ W × W : the words x and y have at least one letter
∴ R is symmetric. in common}
Again, 1R2 and 2R1 but 1 ≠ 2 ∀ x ∈ W, (x, x) ∈ R
∴ R is not anti-symmetric [ Every letter of a word is common with itself]
[Note that a relation R in a set A is said to be an anti-symmetric ∴ R is reflexive.
relation iff (a, b) ∈ R and (b, a) ∈ R ⇒ a = b, ∀ a, b ∈ R] Again, (x, y) ∈ R ⇒ (y, x) ∈ R ∀ x, y ∈ W.
Now, 1R2, 2R3 but 1 R 3 [ |1 – 3| = |– 2| = 2 > 1] [ If the words x and y have at least one letter in common,
∴ R is not transitive. then words y and x have at least one letter in common]
Therefore, R is reflexive and symmetric. ⇒ R is symmetric.
6. (c) reflexive and transitive (x, y) ∈ R, (y, z) ∈ R but (x, z) ∉ R
As every natural number divides itself, i.e., ∀ a ∈ N, [The words ‘every’ and ‘word’ have ‘r’ common and ‘word’
(a, a) ∈ R. and ‘with’ have ‘w’ common, but ‘every’ and ‘with’ do not have
So, R is reflexive. any letter common].
2 divides 6 but 6 does not divide 2. Therefore a divides b does ⇒ R is not transitive.
not imply b divides a, i.e., (a, b) ∈ R need not imply (b, a) ∈ R. Therefore, R is reflexive, symmetric and not transitive.
Hence, R is not symmetric. 10. (b) symmetric only
Now, 2 divides 4 and 4 divides 8 ⇒ 2 divides 8 II. 1. R is not reflexive because a ≥ b2 does not imply a ≥ a2 ∀ a.
(a, b) ∈ R and (b, c) ∈ R ⇒ (a, c) ∈ R 2. Relation R is not transitive because: (1, 2), (2, 1) ∈ R but
(1, 1) ∉ R.
So, R is transitive.
3. Here R = {(2, 3), (4, 2), (6, 1)},
Hence, R is reflexive and transitive.
Domain of R = {2, 4, 6}
7. (d) None of these.
4. Equivalence class of [0] = {0, 2, 4}
On the set N of natural numbers, R is a relation defined by {(x,
y) : x, y ∈ N, 2x + y = 41} 5. Relation R is reflexive and symmetric because (1, 1),
(2, 2), (3, 3) ∈ R
R = {(x, y) : x, y ∈ N, 2x + y = 41} 1 ∈ N
⇒ R is reflexive and (1, 2), (2, 1), (3, 1), (1, 3) ∈ R
but (1, 1) ∉ R as 2(1) + 1 = 3 ≠ 41
⇒ R is symmetric.
So, R is not reflexive.
6. Reason: Take a, b, c ∈ N such that a < b and b < c
Again, (1, 39) ∈ R but (39, 1) ∉ R.
⇒ a<c \ (a, b) ∈ R and (b, c) ∈ R
∴ R is not symmetric.
⇒ (a, c) ∈ R.
Now, (20, 1) ∈ R and (1, 39) ∈ R but (20, 39) ∉ R.
Hence, relation R is transitive.
So, R is not transitive.
7. Relation R = x – y + 2 is reflexive because
∴  R is neither reflexive, nor symmetric nor transitive.
x R x = x –x + 2 = 2 .
8. (a) reflexive and symmetric but not transitive.
8. Same reason as given in solution of Q.No. 6.
R = {(a, b) : 1 + ab > 0 ∀ a ∈ S}
9. Relation R is symmetric because
1 + a ⋅ a = 1 + a2 > 0
x + y < 10 ⇒ y + x < 10
Therefore, aRa, i.e. the relation R is reflexive.
⇒ (x, y) ∈ R ⇒ (y, x) ∈ R
Again, ∀ a, b ∈ S, 1 + ab > 0 ⇒ 1 + ba > 0
10. Relation R is reflexive because a = a
[ ab = ba ∀ a, b ∈ S]
⇒ (a, a) ∈ R for all a ∈ A.
i.e. aRb = bRa
The relation R is symmetric. III. 1. R–1 = {(a, 1), (b, 2), (c, 3), (d, 4)}.
1 2. Given 3x – y = 0. Put y = x in 3x –y = 0
Now, we consider three real numbers 3, − and – 2. It gives 3x – x = 0 ⇒ 2x = 0.
6
 1 1 1 It is not true.
We have1 + 3  −  = 1 – = >0
 6 2 2 \ R is not reflexive.

18 Mathematics–12
E:\AMIT_WORKS\Exam_Guru\EG_Mathematics-12_(working_02-06-2022)\EG_Mathematics-12_working\Open_Files\Chap_1\Chap_1
\ 16-Aug-2022 Amit Proof-5 Reader’s Sign _______________________ Date __________

3. To make relation R reflexive, include (b, b) and (c, c). i.e., (x, y) ∈ R and (y, z) ∈ R
Now, (a, b) ∈ R and (b, c) ∈ R but (b, a) and (c, b) are not ⇒ (x, z) ∈ R \ R is transitive.
included in R. Therefore, include (b, a) and (c, b) to make R 11. R = {(a, b) : (a + b) is an even natural number and a, b ∈ N}
symmetric. (i) R is reflexive: Take a ∈ N. Since (a + a) is an even number
Hence, to make R reflexive and symmetric include (b, b), (c, c), ∀ a ∈ N.
(b, a) and (c, b). (a, a) ∈ R ⇒ R is reflexive.
4. (i) Relation: y = x + 1. (ii) Symmetric: Let a, b ∈ N.
If y = x makes it true, R will be reflexive If (a + b) is an even number then (b + a) is also an even
\ Put y = x in y = x + 1. number.
It gives x = x + 1 \ (a, b) ∈ R ⇒ (b, a) ∈ R
⇒ 0 = 1 (False) ⇒ R is symmetric.
(iii) Transitive: Since sum of two even numbers is even or sum
\ R is not reflexive.
of two odd numbers is even.
(ii) Relation: y = x + 1.
\ Let a, b, c ∈ N are either all even numbers or odd numbers.
Put x = y and y = x in y = x + 1. \ (a + b) is even and (b + c) is even
It gives x = y + 1. ⇒ (a + c) is also even
It is not same as y = x + 1. \ (a, b) ∈ R and (b, c) ∈ R ⇒ (a, c) ∈ R
Hence, R is no symmetric. ⇒ R is transitive.
5. (i) Reflexive: R is reflexive because 12. R = {(a, b) : (a – b) is an even natural number and a, b ∈ N}
{(1, 1), (2, 2), (3, 3)} ∈ R (i) Reflexive: Since (a – a) = 0 (not a natural no.)
(ii) Symmetric: R is symmetric because ⇒ (a, a) ∉ R \ R is not reflexive.
{(1, 2), (2, 1), (2, 3), (3, 2), (1, 3), (3, 1)} ∈ R (ii) Symmetric: If (a – b) is even natural number then (b – a)
(iii) Transitive: R is transitive because (1, 2) ∈ R and is not an even natural number.
(2, 3) ∈ R \ (a, b) ∈ R whereas (b, a) ∉ R.
⇒ (1, 3) ∈ R and so on. Hence, R is not symmetric.
6. (i) Reflexive: R is reflexive because (iii) Transitive: a, b, c ∈ N such that (a – b) and (b – c) are even
natural numbers then (a – c) will also be an even natural
{(1, 1), (2, 2)} ∈ R numbers.
(ii) Symmetric: R is symmetric because \ (a, b) ∈ R and (b, c) ∈ R
{(1, 2), (2, 1)} ∈ R ⇒ (a, c) ∈ R.
(iii) Transitive: R is transitive because {1, 2) ∈ R and Hence, R is transitive.
(2, 1) ∈ R 13. R = {(x, y) : (x + y) is divisible by 5 ∀ x, y ∈ R}
⇒ (1, 1) ∈ R and so on. (i) Reflexive: Take x ∈ R.
7. (i) Reflexive: R is reflexive because Since (x + x) is not divisible by 5 for any real number x.
{(1, 1), (3, 3), (5, 5)} ∈ R ⇒ R is not reflexive.
(ii) Symmetric: R is not symmetric because (3, 5) ∈ R but (ii) Symmetric: Let x, y ∈ R be such that (x + y) is divisible by
(5, 3) ∉ R. 5, then (y + x) is also divisible by 5.
(iii) Transitive: R is not transitive because (1, 3) ∈ R and (3, 5) \ (x, y) ∈ R ⇒ (y, x) ∈ R
∈ R does not imply (1, 5) ∈ R. Hence, R is symmetric.
8. A = {1, 3, 7} (iii) Transitive: Take x, y, z ∈ R such that (x + y) and (y + z) are
R = A × A = {(1, 1), (1, 3), (1, 7), (3, 1), (3, 3), (3, 7), (7, 1), divisible by 5, then (x + z) may or may not be divisible by
(7, 3), (7, 7)} 5.
(i) R is reflexive because {(1, 1), (3, 3), (7, 7)} ∈ R \ (a, b) ∈ R and (b, c) ∈ R does not imply
(ii) R is symmetric because (a, c) ∈ R.
{(1, 3), (3, 1) (1, 7), (7, 1), (3, 7), (7, 3)} ∈ R. Hence, R is not transitive.
(iii) R is transitive because (1, 3) ∈ R and (3, 7) ∈ R 14. R in set A = {1, 2, 3} is
⇒ (1, 7) ∈ R and so on. R = {(1, 1), (2, 2), (3, 3), (1, 2), (2, 3)}
9. See solution of Q.No. 6. (i) Reflexive: Since {(1, 1), (2, 2), (3, 3)} ∈ R
10. R = {(x, y) : x ≥ y, x, y ∈ N} ⇒ R is reflexive.
(i) R is reflexive: Since x ≥ x ⇒ (x, x) ∈ R (ii) Symmetric: Since (1, 2) ∈ R but (2, 1) ∉ R and (2, 3) ∈ R
⇒ R is reflexive. but (3, 2) ∉ R.
(ii) R is not symmetric: x ≥ y does not imply y ≥ x. ⇒ R is not symmetric.
\ R is not symmetric. (iii) Transitive: Since (1, 2) ∈ R and (2, 3) ∈ R does not imply
(iii) Transitive: Take three natural numbers x, y and z such that (1, 3) ∈ R.
x ≥ y and y ≥ z ⇒x≥z Hence R is not transitive.

Relations and Functions 19


E:\AMIT_WORKS\Exam_Guru\EG_Mathematics-12_(working_02-06-2022)\EG_Mathematics-12_working\Open_Files\Chap_1\Chap_1
\ 16-Aug-2022 Amit Proof-5 Reader’s Sign _______________________ Date __________

15. R = {(L1, L2) : L1 ^ L2 and L1, L2 ∈ L} 4. A = a set of points in a given plane.


(i) Reflexive: Let L1 ∈ L. Since line L1 cannot be perpendicular R = {(p, q) : distance of p and q from origin is same, p, q ∈ A}
to itself (i) Reflexive: Take p ∈ A.
\ (L1, L1) ∉ R Since (p, p) ∈ R \ R is reflexive.
Hence, R is not reflexive. (ii) Symmetric: Take p, q ∈ A such that distances of p and q
(ii) Symmetric: Let L1, L2 ∈ L. If line L1 ^ L2 then L2 ^ L1 from origin is same.
\ (p, q) ∈ R ⇒ (q, p) ∈ R
Hence (L1, L2) ∈ R and (L2, L1) ∈ R
⇒ R is symmetric.
⇒ R is symmetric.
(iii) Transitive: Take p, q, r ∈ A such that p and q are at equal
(iii) Transitive: Let L1, L2, L3 ∈ L and L1 ^ L2 and L2 ^ L3 then distances from O and q and r are also at equal distances from
L1 is not perpendicular to L3 O.
\ (L1, L2) ∈ R and (L2, L3) ∈ R does not imply that (L1, L3) Then (p, q) ∈ R and (q, r) ∈ R
∈ R. ⇒ (p, r) ∈ R
Hence, R is not transitive. Hence, R is transitive.
IV. 1. Do it yourself. Since R is reflexive, symmetric and transitive.
2. A = Set of all the books in a library Hence R is equivalence relation,
R = {(x, y) : x and y have the same number of pages and x, y 5. T1 is related to T3 because sides of T1 are 3, 4, 5 and sides of T3
∈ A} are 6, 8, 10 and hence are proportional.
(i) Reflexive: Since number of pages of any book x is equal to 6. Similar solution as given in solution of Q.No. 4 (Short answers-II).
number of pages of itself. Set of elements in set A related to the right angled triangle T
with sides 3, 4 and 5 is a set of all possible triangle in set A.
\ (x, x) ∈ R ⇒ R is reflexive.
7. Solution is similar to solution of Q.No. 5 (Short answers-II).
(ii) Symmetric: Take any two books x, y from the library such
that number of pages of book x is equal to number of pages Set of all lines related to the line y = 2x + 4 is y = 2x + C, C is
any constant.
of book y then the number of pages of book y is equal to
number of pages of book x. 8. A = {1, 2, 3, 4, 5} and
R = {(a, b) : |a – b| is an even number, a, b ∈ A}
\ (x, y) ∈ R ⇒ (y, x) ∈ R
(i) Reflexive: Take a ∈ A since |a – a| = 0 (an even number)
⇒ R is symmetric.
⇒ (a, a) ∈ R ⇒ R is reflexive
(iii) Transitive: Take three books x, y, z such that x and y are of
(ii) Symmetric: Take a, b ∈ A such that |a – b| is an even number,
equal pages and books y and z are of equal pages. Then x
then |b – a| also will be an even number.
and z will also have equal number of pages.
\ (a, b) ∈ R ⇒ (b, a) ∈ R
\ (x, y) ∈ R and (y, z) ∈ R
⇒ R is symmetric.
⇒ (x, z) ∈ R
(iii) Transitive: Take a, b, c ∈ A such that |a – b| and |b – c| are
Hence, R is transitive. even numbers. Then |a – c| will also be an even number.
3. (A) A = {x ∈ Z : 0 ≤ x ≤ 12} \ (a, b) ∈ R and (b, c) ∈ R
R = {(a, b) : |a – b| is a multiple of 4 and a, b ∈ A} ⇒ (a, c) ∈ R
(i) Reflexive: Take a ∈ A Hence R is transitive.
Since |a – a| = 0 is divisible by 4 Since R is reflexive, symmetric and transitive.
\ (a, a) ∈ R ⇒ R is reflexive. Hence, R is an equivalence relation.
(ii) Symmetric: Take a, b ∈ A such that |a – b| is divisible by 9. Do it yourself.
4 then |b – a| will also be divisible by 4. 10. R = {(x, y) : x, y ∈ N and 2x + y = 24}
\ (a, b) ∈ R ⇒ (b, a) ∈ R R = { ( 1 , 2 2 ) , ( 2 , 2 0 ) , ( 3 , 1 8 ) , ( 4 , 1 6 ) , ( 5 , 1 4 ) ,
(6, 12), (7, 10), (8, 8), (9, 6), (10, 4), (11, 2)}
Hence R is symmetric.
⇒ Domain of R = {1, 2, 3, ....., 11} and
(iii) Transitive: Take a, b, c ∈ A such that |a – b| and |b – c| are
Range of R = {2, 4, 6, ......, 20, 22}
divisible by 4 then |a – c| will also be divisible by 4.
Relation R is neither reflexive, nor symmetric, nor transitive.
\ (a, b) ∈ R and (b, c) ∈ R
V. 1. {(a, b) R (c, d) if ad(b + c) = bc(a + d), a, b, c, d ∈ N}
⇒ (a, c) ∈ R \ R is transitive.
(i) Reflexive: Take a, b ∈ N
Set of elements related to 1 = {1, 5, 9}
Since ab(b + a) = ba(a + b)
(B) A = {x ∈ Z : 0 ≤ x ≤ 12} \ (a, b) R (a, b)⇒ R is reflexive.
R = {(a, b) : a = b and a, b ∈ A} (ii) Symmetric: Take a, b, c, d ∈ N such that (a, b) R (c, d)
Prove in the same way as we have proved in Q.No. 3(A). ⇒ ad(b + c) = bc(a + d) ⇒ cb(d + a) = da(c + b)
Set of elements related to 1 = {1} ⇒ (c, d) R (a, b) ⇒R is symmetric

20 Mathematics–12
E:\AMIT_WORKS\Exam_Guru\EG_Mathematics-12_(working_02-06-2022)\EG_Mathematics-12_working\Open_Files\Chap_1\Chap_1
\ 16-Aug-2022 Amit Proof-5 Reader’s Sign _______________________ Date __________

(iii) Transitive: Take a, b, c, d, e, f ∈ N such that (a, b) R (c, d) 3. Do it yourself.


and (c, d) R (e, f) 4. Domain of R = {1, 2, 3, ...., 20}
⇒ ad(b + c) = bc(a + d) and Range of R = {1, 2, 3, ...., 37, 39}
cf (d + e) = de(c + f)
5. In a set A = {1, 2, 3}
b+c = a+d d +e c+ f
⇒ and = Relation R = {(1, 1), (1, 2), (1, 3), (2, 1), (2, 2), (2, 3), (3, 1),
bc ad de cf
(3, 2), (3, 3)}
1 1 1 1
⇒ + = + and 1 + 1 = 1 + 1 will be an equivalence relation.
c b d a e d f c
1+1+1+ 1 = 1 +1+ 1 +1
⇒ 7 ordered pairs will be included.
⇒ (on adding)
c b e d d a f c 6. Hint: A = {1, 2, 3}
1 1 1 1 b+e = a+ f R = {(1, 1), (2, 2), (3, 3), (1, 2), (2, 1)}
⇒ + = + ⇒
b e a f be af S = {(1, 1), (2, 2), (3, 3), (2, 3), (3, 2)}
⇒ a f(b + e) = be (a + f) Here R and S each is an equivalence relation on set A, but their
⇒ (a, b) R (e, f) ⇒ R is transitive. union R ∪ S is not an equivalence relation.
2. R = {(a, b) : (a – b) is divisible by a fixed positive integer, a, b 7. R = {(a, b) R (c, d) if and only if ad = bc; a, b, c, d ∈ N}
∈ Z}
(i) Reflexive: Take a, b ∈ N since ab = ba
(i) Reflexive: Let a ∈ Z.
⇒ (a, b) R (a, b)⇒ R is reflexive.
Since (a – a) = 0 is divisible by n.
\ (a, a) ∈ R ⇒ R is reflexive. (ii) Symmetric: Take a, b, c, d ∈ N such that (a, b) R (c, d)
(ii) Symmetric: Let a, b ∈ Z such that (a – b) is divisible by n. ⇒ ad = bc ⇒ cb = da
If (a – b) is divisible by n then (b – a) is also divisible by n. ⇒ (c, d) R (a, b)⇒ R is symmetric.
\ (a, b) ∈ R ⇒ (b, a) ∈ R (iii) Transitive: Take a, b, c, d, e, f ∈ N such that (a, b) R (c, d)
⇒ R is symmetric. and (c, d) R (e, f)
(iii) Transitive: Let a, b, c ∈ Z such that (a – b) and (b – c) are ⇒ ad = bc and cf = de ⇒ ad = bc and de = cf
divisible by n. ad bc a b
⇒ = ⇒ =
Then (a – c) also will be divisible by n. de cf e f
[... a – b + b – c = a – c] ⇒ af = be ⇒ (a, b) R (e, f)
Hence (a, b) ∈ R and (b, c) ∈ R ⇒ R is transitive.
⇒ (a, c) ∈ R ⇒ R is transitive 8. Do it yourself.

Topic 3. Functions
Let A and B be two non-empty sets. Then, a relation from set Image and Pre-Image of an Element of a Function
A to set B is called a function if each element of set A has a If the element x ∈ A corresponds to y ∈ B under a function f :
correspondence with exactly one element of set B. A → B, y = f (x), then y is called image of x under f and x is called
A function is denoted by the symbols: y = f (x), y = g (x), pre-image of y under f.
y = h(x), etc. Range of a Function
A function y = f (x) from set A to set B is represented as: In a function f : A → B, y = f (x) the subset of set B containing the
f : A → B and it is read as ‘function f such that A maps into B’. images of the elements of set A is called range of the function f.
Note: In a function f : A → B, y = f (x) if expressed in roster
form, then
(i) No element of set A should be left without correspondence
of only one element of set B.
(ii) No first element of any ordered pair should repeat. For example: f : A → B is a function such that A = {1, 2, 3}
and B = {7, 8, 10, 11}. Let f represents the rule under which
Domain of a Function element 1 of set A corresponds to element 7 of set B; element 2
In a function f : A → B, y = f (x) set A is called domain of the of set A corresponds to element 8 of set B and element 3 of set A
function. corresponds to elements 10 of set B.
This function f : A → B can be expressed in roster form as given,
Co-Domain of a Function f = {(1, 7), (2, 8), (3, 10)} and in arrow diagram as shown alongside.
In a function f : A → B, y = f (x) set B is called co-domain of the It is a function because every element of set A has its exactly one
function. image in set B.

Relations and Functions 21


E:\AMIT_WORKS\Exam_Guru\EG_Mathematics-12_(working_02-06-2022)\EG_Mathematics-12_working\Open_Files\Chap_1\Chap_1
\ 16-Aug-2022 Amit Proof-5 Reader’s Sign _______________________ Date __________

One-One (or Injective) Function Example 3. A function f : N → N, given by f (x) = 5x + 3 is


A function f : X → Y is called one-one (or injective) if the images (a) one-one but not onto (b) onto but not one-one
of distinct elements of set X under f are distinct i.e., for every (c) bijective (d) None of these.
x1, x2 ∈ X; f (x1) = f (x2) implies x1 = x2. Otherwise f is called Solution. The given function is f : N → N and f (x) = 5x + 3.
many-one function. (i) One-one: Take any two natural numbers x1 and x2 such
that
f (x1) = f (x2) ⇒ 5x1 + 3 = 5x2 + 3
⇒ 5x1 = 5x2 ⇒ x1 = x2
⇒ given function is one-one.
The function f in figure given alongside is one-one. (ii) Onto: The given function is f : N → N and f (x) = 5x + 3.

Onto or (Surjective) Function  y − 3


Taking it as: y = 5x + 3 ⇒ x=
5 
A function f : X → Y is said to be onto (or surjective), if every Now, for every natural number y, there does not exist a natural
element of set Y is the image of some element of set X under f, i.e.,
number x or for any natural number y, y − 3
for every y ∈ Y, there exists an element x ∈ X such that f (x) = y. 5
is not a natural number. For example, for y = 1, there is no
natural number x. Hence, given function is not onto.
Hence, option (a) is the correct answer.
Example 4. A function f : Z → Z given by f (x) = 5x + 3 is
(a) one-one but not onto. (b) bijective
Here, function f1 and f2 are onto functions because every element (c) onto but not one-one (d) None of these
of Y has a pre-image in X.
Solution. The given function: f : Z → Z given by f (x) = 5x + 3.
Note: A function f : X → Y is onto if and only if Range of f = Y.
(i) One-one: Take two integers x1 and x2,
One-One and Onto or (Bijective) Function such that f (x1) = f (x2) ⇒ 5x1 + 3 = 5x2 + 3
A function f : X → Y is said to be one-one and onto (or bijective) ⇒ 5x1 = 5x2 ⇒ x1 = x2
if f is both one-one and onto. ⇒ given function is one-one.
y −3
(ii) Onto: Take y = 5x + 3 ⇒ x= 5
For every integer y, there does not exist an integer x.

  y−3 
 is not an integer for all y 
5 
Thus, f (x) is not onto.
Function f in figure given alongside is one-one and onto both.
Hence, option (a) is the correct answer.
Example 1. Take a relation from set A = {1, 3, 5, 7} to set B =
Example 5. A function f : R → R is given by: f (x) = 5x + 3. The
{1, 2, 3, 4, 5} given in roster form as:
function f (x) is
R = {(1, 1), (3, 4), (7, 5)}. It is
(a) one-one but not onto (b) bijective
(a) a function (b) an injective function
(c) onto but not one-one (d) None of these.
(c) not a function (d) a bijective function
Solution. Given function f : R → R is f (x) = 5x + 3.
Solution. It is not a function because element 5 of set A does not
have any image in set B. (i) One-One: Take two real numbers x1 and x2 such that f (x1)
Hence, option (c) is the correct answer. = f (x2)
Example 2. Take a relation from set ⇒ 5x1 + 3 = 5x2 + 3    ⇒ 5x1 = 5x2   ⇒ x1 = x2
A to set B shown on R.H.S. in arrow ⇒ f (x) is one-one function.
diagram. Which of the following is y −3
(ii) Onto: Take y = 5x + 3 ⇒ x= .
true about it? 5
(a) It is a function. Here for every real number y, there exists a real number x.
(b) It is an injective function.   y−3 
 is a real number for all y 
(c) It is a bijective function. 5 
(d) It is not a function. \ f (x) is an onto function.
Solution. It is also not a function because element 3 of set A has Since given function f (x) is one-one and onto
two images 7 and 8 in set B. both.       \ f (x) is a bijective function.
Hence, option (d) is the correct answer. Hence, option (b) is the correct answer.

22 Mathematics–12
E:\AMIT_WORKS\Exam_Guru\EG_Mathematics-12_(working_02-06-2022)\EG_Mathematics-12_working\Open_Files\Chap_1\Chap_1
\ 16-Aug-2022 Amit Proof-5 Reader’s Sign _______________________ Date __________

Example 6. A function f : N → N defined as f (x) = 5x2 + 3. The Horizontal Line Test for One-One Function
function f (x) is If any horizontal line cuts the graph of a relation in one point only,
(a) bijective (b) one-one but not onto. it is one-one. In the given graph of y = sin x, any line drawn parallel
(c) onto but not one-one (d) None of these to x-axis will cut the graph at many points, therefore the function
Solution. Given function f : N → N is f (x) = 5x2 + 3. y = sin x is not one-one.
(i) One-One: For any two natural numbers x1 and x2, Notes:
let f (x1) = f (x2) 1. The number of functions from a finite set A into finite set

B = [n(B)]n(A).
⇒ 5x12 + 3 = 5x22 + 3    ⇒ 5x12 = 5x22   ⇒ x12 = x22
Example: (i) The number of functions from {1, 2, 3}
⇒ x1 = x2 (for natural numbers)⇒ f (x) is one-one function into {6, 7} = 23 = 8.
y −3 (ii) Number of functions from {a, b} into
(ii) Onto: Take y = 5x2 + 3 ⇒ x2 =
5 {2, 3, 4} = (3)2 = 9.
y −3 2. The number of one-one functions that can be defined from

⇒ x=
5 a finite set A into a finite set B = n(B)Pn(A) if n(B) ≥ n(A)
For every natural number y, there does not exist a natural otherwise 0.
y −3 Example: (i) The number of one-one functions from {a,
number x =
5 b, c} into {1, 2} = 0. n(A) > n(B).
\ f (x) is not onto. (ii) The number of one-one functions from {1,
Hence, option (b) is the correct answer. 2} into {a, b, c} = 3P2 = 6.
(iii) The number of one-one functions from {1,
Vertical Line Test for a Function
2, 3} into {5, 6, 7} = 3P3 = 3! = 6.
A relation is a function if each vertical line meets its graph in only
3. The number of onto functions that can be defined from
one point and relation is not a function if any vertical line cuts
the graph in more than one point. a finite set A containing n elements onto a finite set B
containing 2 elements = 2n – 2.
Example: The number of onto functions from {1, 2, 3}
onto {p, q} = 23 – 2 = 8 – 2 = 6.
4. The number of onto functions that can be defined from
finite set A onto a finite set B is equal to number of ways
of dividing n(A) things into n(B) groups so that no group
is empty if n(A) ≥ n(B) and is 0 otherwise.
Take a function: y = sin x. Its graph is on RHS. You draw any Example: The number of onto functions from {1, 2, 3}
vertical line. It will cut the graph only at one point, hence y = sin onto {p, q} = 3! + 3! = 3 + 3 = 6
x is a function. 1! 2 ! 2 ! 1!

DOMAIN OF FOLLOWING FUNCTIONS

1. (a) The domain of a 2 − x 2 is [–a, a] or –a ≤ x ≤ a.


1
(b) The domain of is (–a, a) or –a < x < a.
a − x2
2

2. (a) The domain of x 2 − a 2 is (–∞, –a] ∪ [a, ∞) or x ≥ a or x ≤ –a.


1
(b) The domain of is (–∞, –a) ∪ (a, ∞) or x > a or x < –a.
x − a2
2

3. (a) The domain of ( x − a ) (b − x ) when a < b is [a, b] or a ≤ x ≤ b.


1
(b) The domain of when a < b is (a, b) or a < x < b.
( x − a ) (b − x )
4. (a) The domain of ( x − a ) ( x − b) when a < b is (–∞, a] ∪ [b, ∞) or x ≥ b or x ≤ a.
1
(b) The domain of when a < b is (–∞, a) ∪ (b, ∞) or x > b or x < a.
( x − a ) ( x − b)

Relations and Functions 23


E:\AMIT_WORKS\Exam_Guru\EG_Mathematics-12_(working_02-06-2022)\EG_Mathematics-12_working\Open_Files\Chap_1\Chap_1
\ 16-Aug-2022 Amit Proof-5 Reader’s Sign _______________________ Date __________

5. (a) The domain of x − a when a < b is (–∞, a] ∪ (b, ∞) or x > b or x ≤ a.


x−b
(b) The domain of x − a when a > b is (–∞, b) ∪ [a, ∞) or x ≥ a or x < b.
x−b
x−a
(c) The domain of when a < b is [a, b) or a ≤ x < b.
b−x
x−a
(d) The domain of when a > b is (b, a] or b < x ≤ a.
b−x
6. (a) The domain of log (a2 – x2) is (–a, a).
(b) The domain of log (x2 – a2) is (–∞, a) ∪ (a, ∞).
7. (a) The domain of log (x – a) (b – x) when a < b is (a, b).
(b) The domain of log (x – a) (x –b) when a < b is (–∞, a) ∪ (b, ∞).
 8. (a) The domain of tan mx when m ∈ N is R − ( 2n + 1) π , m ∈ Z
2m { }
(b) The domain of cot nx when m ∈ N is R −

m
: n ∈Z . { }
9. (a) Range of f (x) = x 2 − a 2 is [0, a].
(b) Range of f(x) = a cos x + b sin x + c is c − a 2 + b 2 , c + a 2 + b 2  .
 
10. If f (x) is a function, then
(a) f (x) + f (–x) is an even function. (b) f (x) – f (–x) is an odd function.
x+c
11. The domain of the function f ( x ) = is R – [c] and range = {–1, 1}.
x+c

Thus
Example 7. The domain of x − 1 + 8 − x is
(a) [1, 8] (b) (–8, 8) (c) [1, 8)
Solution. Clearly, x – 1 ≥ 0 and 8 – x ≥ 0 i.e. x ≥ 1 and 8 ≥ x
(d) (1, 8) ( 4   20 )
Range of f = −∞, − 1  ∪  − 1 , ∞ = R − − 1 , − 1
4 20 { }
Hence, option (b) is the correct answer.
i.e. x ≥ 1 and x ≤ 8 i.e. 1 ≤ x ≤ 8 ⇒ x ∈ [1, 8] Example 9. Let D be the domain of a real valued function defined
Hence, option (a) is the correct answer.
as: f ( x ) = 25 − x 2. The domain D will be
x+2 (a) [–2, 2] (b) [–3, 3] (c) [–4, 4] (d) [–5, 5]
Example 8. The range of function y = is
x 2 − 8x − 4

(
2

{ }
2
1 Solution. f (x) = 25 − x ⇒ 25 – x ≥ 0
−∞, −  1 1
(a) (b) R − − , − ⇒ 25 ≥ x 2
⇒ –5 ≤ x ≤ 5
4  4 20

)
⇒ Domain = [–5, 5].
(c)  − 1 , ∞ (d) None of these. Hence, option (d) is the correct answer.
 20
2
Solution. For y to be defined x – 8x – 4 ≠ 0, i.e. x ≠ 4 ± 2 5 Example 10. Let D be the domain of a real valued function defined
1
\ Domain of y = R – {4 + 2 5 , 4 – 2 5 }
as: f ( x ) = , find D.
2
4x − 25
x+2 1
Now y = 2 ⇒ x2y – (8y + 1)x – (4y + 2) = 0 Solution. f (x) = ⇒ 4x2 – 25 > 0
x − 8x − 4 2
4x − 25
For x to be real 25 5 5
2
⇒x > ⇒ x> or x < −
(8y + 1)2 + 4y(4y + 2) ≥ 0 ⇒ 80y2 + 24y + 1 ≥ 0 4 2 2


2
⇒ y +
3
10
y+
1
80
≥0 ⇒ ( y+
3
) ( )
2

1
2
≥0
⇒ Domain = x > { 5
2
5
or x < − , x ∈ R
2 }
20 10
Example 11. Let D be the domain of a real valued function
3 1
⇒ y + 3 ≤ − 1 or y +
≥ [Q x2 ≥ a2 ⇒ x ≥ a defined as f ( x ) = x2 − x − 6 .
20 10 20 10
2
or x ≤ –a] Solution. f (x) = x − x − 6 ⇒ x2 – x – 6 ≥ 0
5 1 ⇒ (x –3)(x + 2) ≥ 0 ⇒ x ≥ 3 or x ≤ –2
⇒y≤ −
or y ≥ − ⇒ y ≤ − 1 or y ≥ − 1
20 20 4 20 ⇒ Domain ={x ≥ 3 or x ≤ –2, x ∈ R}

24 Mathematics–12
E:\AMIT_WORKS\Exam_Guru\EG_Mathematics-12_(working_02-06-2022)\EG_Mathematics-12_working\Open_Files\Chap_1\Chap_1
\ 16-Aug-2022 Amit Proof-5 Reader’s Sign _______________________ Date __________

EXERCISE 1.3
I. Multiple Choice Questions (MCQs) 6. A function f : R → R is defined as f (x) = 2x2 + 3. Show
Choose the correct answer from the given options. that f (x) is neither one-one nor onto.
1. A function f : N → N defined as y = x2 is 7. A function f : N → N is defined as f (x) = 2x3 + 3. Show
(a) one-one (b) onto that f (x) is one-one but not onto.
(c) bijective (d) None of these. 8. A function f : R → R is defined as f (x) = 2x3 + 3. Show
that f (x) is one-one and onto.
2. Domain of the function: f : R → R defined as y = x + 4 is
9. Write Domain and Range of modulus function.
(a) Real numbers (b) Natural numbers
10. Write Domain and Range of signum function.
(c) Negative real numbers (d) None of these.
11. Write Domain and Range of the greatest integer function
3. Domain of a real valued function f ( x ) = 36 − x 2 is: f : R → R and f (x) = [x].
(a) [–6, 6] (b) (–6, 6) (c) (–6, 6] (d) [–6, 6) III. Short Answer Type Questions-II
4. Number of function from set {1, 2} into set {a, b, c} are 1. Let A be the set of all 50 students of Class X in a school.
(a) (2)3 (b) (3)2 (c) (2)2 (d) (3)3 Let f : A → N be a function defined by f (x) = roll numbers
5. Number of one-one functions that can be defined from a of the student x. Show that f is one-one, but not onto.
set {3, 4, 5} into a set {1, 2} are [NCERT]
(a) 2P3 (b) 0 (c) 2P2 (d) 3P3 2. Show that the function f : N → N given by f (x) = 2x, is
6. The function f : R → [–1, 1] defined by f (x) = cos x is one-one but not onto. [NCERT]
(a) both one-one and onto 3. Prove that the function f : R → R given by f (x) = 2x, is
(b) not one-one, but onto one-one and onto. [NCERT]
(c) one-one, but not onto 4. Show that the function f : N → N given by f (1) =
(d) neither one-one, nor onto f (2) = 1 and f (x) = x – 1 for every x > 2 is onto but not
one-one.[NCERT]
7. Every function is a
 x + 1 if x is odd
(a) relation. (b) onto function 5. Show that f : N → N given by f ( x ) = 
(c) injective function (d) bijective function is both one-one and onto.  x − 1 if x is even
8. A function which is injective and surjective is called [NCERT] [HOTS] [A.I. 2011]
(a) onto function only (b) bijective function 6. Show that the function defined by f : R+ → R+ and given by
(c) one-one function (d) None of these. 1
f ( x ) = is one-one and onto, where R+ is the set of all non-
9. An onto function from set A to set B means every element x
zero positive real numbers. Is the result true, if the domain
of set B has a pre-image in R+ is replaced by N with co-domain being same as R+?
(a) Set B × A (b) Set A
7. Prove that the greatest integer function f : R → R given by
(c) Set A × B (d) None of these. f (x) = [x], is neither one-one nor onto, where [x] denotes
10. A function f : R → R is defined as f(x) = x3 + 1. Then the the greatest integer less than or equal to x.
function has
[NCERT] [HOTS]
(a) no minimum value
8. Show that the signum function f : R → R given by
(b) no maximum value
 1 if x > 0
(c) both maximum and minimum value 
f ( x ) =  0 if x = 0 is neither one-one nor onto.
(d) neither maximum value nor minimum value   − 1 if x < 0 [HOTS]

II. Short Answer Type Questions-I
9. Let A = {3, 7, 9}, B ={2, 4, 10} and let f = {(3, 2), (7, 4),
1. A function f : N → N is given by f (x) = 2x + 3. Show that (9, 10)} be a function from A to B. Show that f is one-one.
f (x) is one-one but not onto.
10. In a function given below, state whether the function is
2. A function f : Z → Z is given by f (x) = 2x + 3. Show that one-one, onto or bijective. Justify your answer. f : R → R
f (x) is one-one but not onto. defined by f (x) = 1 + x2.[NCERT] [HOTS]
3. A function f : Q → Q is defined as f (x) = 2x + 3. Show that 11. Let A and B be two sets. Show that f : A × B → B × A such
f (x) is one-one and onto. that f (a, b) = (b, a) is bijective function. [NCERT]
4. A function f : R → R is defined as f (x) = 2x + 3. Show that 12. Let A = R – {3} and B = R – {1}. Consider the
f (x) is one-one and onto.
5. A function f : N → N is defined as f (x)= 2x2 + 3. Show that
function f : A → B defined by f ( x ) = x − 2 . Is
x−3 ( )
f (x) is one-one but not onto. f one-one and onto? Justify your answer. [NCERT]

Relations and Functions 25


E:\AMIT_WORKS\Exam_Guru\EG_Mathematics-12_(working_02-06-2022)\EG_Mathematics-12_working\Open_Files\Chap_1\Chap_1
\ 16-Aug-2022 Amit Proof-5 Reader’s Sign _______________________ Date __________

13. Let A = {– 1, 0, 1, 2}, B = {– 4, – 2, 0, 2} and f, g : 19. Show that the function f : W → W defined by
A → B be functions defined by f (x) = x2 – x, x ∈ A and  x + 1 when x is odd
1
f ( x) =  is bijective function.
g(x) = 2 x − − 1, x ∈ A. Are f and g equal? Justify your  x − 1 when x is even
2
[HOTS] [A.I. 2011 (C), 2012]
answer. [HOTS]
IV. Long Answer Type Questions
[Hint: One may note that two functions f : A → B and g :
A → B such that f (a) = g (a) ∀ a ∈ A, are called equal 1. Show that the modulus function f : R → R given by f (x) =
functions]. |x| is neither one-one nor onto, where |x| is x if x is positive
14. Find the Domain and Range of each of the following or 0 and |x| is (– x), if x is negative. [HOTS]
functions. 2. Let the function f : R → [–1, 1] defined by f (x) = cos x, ∀

{( }
x ∈ R. Show that f (x) is neither one-one nor onto.[HOTS]
(i) f ( x ) = )
x, 4 − x 2 : x ∈ R .  [HOTS] 3. L e t f : R → R b e t h e f u n c t i o n d e f i n e d a s
1
 2   f ( x) = , ∀ x ∈ R . Find the range of f (x).
(ii) f ( x ) =  x, x − 9  : x ∈ R and x ≠ 3 2 − cos x
  x − 3    [HOTS]
{
(iii) f ( x ) = ( x, x − x ) : x ∈ R  } [HOTS]
4. If f : [0, ∞) → [0, ∞) and f ( x ) = x , then prove that
1+ x
 f (x) is one-one but not onto.
(iv) f ( x ) =  x, 2 2  : x ∈ R  5. Let f : R → R be a function defined by f (x) = 2x3 + 5, then
 4 − x  
show that f(x) is bijective. [HOTS]
 x + 1 if x is odd
 6. Find the domain of the function:
15. Let f : N → N be defined as f ( x ) =  2 ,
x if x is even (i) f ( x ) = 1  [HOTS]
2 x− x
x ∈ N. State whether function f is bijective. 1
16. Let f : X → Y be a function. Determine a relation R in (ii) f ( x ) =  [HOTS]
x+ x
X given by R = {(a, b) : f (a) = f (b)}. Show that R is an
7. Find the domain:
equivalence relation.
(i) f (x) = cos–1(3x – 1) [HOTS]
17. Show that the function f : R → R given by f (x) = ax + b,
−1
where a, b ∈ R, a ≠ 0 is bijective. (ii) f ( x ) = sin x  [HOTS]
18. If f : R → R be the function defined by f (x) = 4x3 + 7. Show x
that f (x) is bijective. [Delhi 2011 (C)] (iii) f(x) = sin–1 (log2 x) [HOTS]

Answers 1.3
I. 1. (d)
None of these y − 3
2. (a)
Real numbers (ii) Onto: Take y = 2x + 3 ⇒ x=
 2 
3. [–6, 6] 4. (b) (3)2
(a) 5. (b) 0  n(A) > n(B)  y − 3
For every natural number y, is not a natural number.
 2 
6. (b)
not one-one, but onto \ f (x) is not onto.
f (x) =cos x, R → [–1, 1] 2. Similar to solution of Q.No. 1.
One-one: Take x1, x2 ∈ R such that f (x1) = f (x2)
3. Hint: (i) One-one: Take x1, x2 ∈ Q such that
⇒ cos x1 = cos x2 ⇒ x1 = ±x2
f (x1) = f (x2)
Hence, f (x) is not one-one
⇒ 2x1 + 3 = 2x2 + 3 ⇒ x1 = x2
Onto: f (x) =cos x ⇒ f (x) is one-one.

Take y = cos x ⇒ x = cos–1 y y − 3
(ii) Onto: Take y = 2x + 3 ⇒ x = 
For every value of y ∈ [–1, 1], cos–1 y exists  2 
\ f (x) is onto. y −3
Now, for every rational number y, is a rational number.
7. (a) relation. 2
\ f (x) is onto.
8. (b) bijective function
4. Try yourself as Q.No. 3.
9. (b) Set A
5. Hint: (i) One-one: Take x1, x2 ∈ N such that
10. (b) neither maximum value nor minimum value
II. 1. Hint: (i) One-one: Take x1, x2 ∈ N such that: f (x1) = f (x2)

f (x1) = f (x2) ⇒ 2x12 + 3 = 2x22 + 3 ⇒
x12 = x22

⇒ 2x1 + 3 = 2x2 + 3 ⇒ x1 = x2 ⇒ x1 = x2[ x1 and x2 are natural numbers]

⇒ f (x) is one-one. ⇒ f (x) is one-one.

26 Mathematics–12
E:\AMIT_WORKS\Exam_Guru\EG_Mathematics-12_(working_02-06-2022)\EG_Mathematics-12_working\Open_Files\Chap_1\Chap_1
\ 16-Aug-2022 Amit Proof-5 Reader’s Sign _______________________ Date __________

7. f : R → R and f (x) = [x]


y −3
(ii) Onto: Take y = 2x2 + 3 ⇒ x= Hint: One-one: Take x1 and x2 ∈ R such that
2
f (x1) = f (x2)  
⇒ [x1] = [x2]
For every natural number y, y − 3 is not a natural number.
2 ⇒ x1 ≠ x2 ⇒ f is not one-one.
\ f (x) is not onto. Onto: y = [x], here for every real number y, there does not exist
6. Hint: (i) One-one: Take x1, x2 ∈ R such that a real number x.
f (x1) = f (x2) ⇒ f is not onto.
 1 if x > 0
⇒ 2x12 + 3 = 2x22 + 3
⇒ x12 = x22 

⇒ x1 = ± x2 ⇒ f (x) is not one-one function. 8. Hint: f : R → R and f (x) =  0 if x = 0
 −1 if x < 0
y −3 
(ii) Onto: Take y = 2x2 + 3 ⇒ x= One-One: Take x1, x2 ∈ R such that f (x1) = f (x2)
2
y − 3 is not a real number. ⇒ x1 ≠ x2 because f (2) = f (3) but 2 ≠ 3.
For every real number y,
2 ⇒ f (x) is not one-one.
\ f (x) is not onto. Onto: Take y = f (x). Here, we have value of x, only for three
7. Try as we have solved Q.No. 6. values of y i.e., y = – 1, 0, 1. If y = 4. There is no value of
8. Try as we have solved Q.No. 6. x ⇒ f is not onto.
9. (i) Domain of modulus function = R. 9. True. Try yourself.
(ii) Range of modulus function = non-negative real numbers. 10. Neither one-one nor onto.
 −1; x < 0 Hint: f : R → R and f (x) = 1 + x2.
( )  One-One: Take x1, x2 ∈ R such that f (x1) = f (x2)
Signum function f x =  0; x = 0
 1 x>0 ⇒ x12 = x22 ⇒ x1 = ± x2

10. (i) Domain of signum function = real numbers. ⇒ f is not one-one.
(ii) Range of signum function = {–1, 0, 1}.
Onto: y = 1 + x2 ⇒ x= y −1.
11. (i) Domain of greatest integer function = real numbers.

(ii) Range of greatest integer function = Integers. For every real number y , y −1 is not a real number ⇒ f is not
III. 1. Hint: One-One: Every student has a different roll number onto.
from 1 to 50. No two or more students in the class can have 11. Hint: A and B are two sets: f : A × B → B × A such that f (a, b)
same roll number. Therefore, f is one-one. = (b, a) is bijective.
Onto: Roll numbers of the students are from 1 to 50. There One-one: Take two ordered pairs (a1, b1) and (a2, b2) such that
is no student whose roll number is 51, 52 or 53. So, every a1, b1 ∈ A and a2, b2 ∈ B. Now assume that
number in the codomain of natural numbers is not a roll
f (a1, b1) = f (a2, b2)
number. Therefore, f is not onto.
2. Try yourself. 3. Try yourself. ⇒ (b1, a1) = (b2, a2) ⇒ b1 = b2 and a1 = a2
4. Hint: f : N → N. ⇒ f is one-one.
One-one: f (1) = f (2) = 1 Onto: For every element (b, a) from co-domain, there exists a
⇒ f is not one-one. corresponding element (a, b) in domain of f. ∴ f is onto.
Onto: When y = 1, then x = 1 or 2. When x > 2, 12. Hint: One-one: Take x1, x2 ∈ R such that f (x1) = f (x2).
then x = y + 1. Hence, for every natural number x − 2 x2 − 2
⇒ 1 = ⇒ x1 = x2
y, (y + 1) is a natural number ∴ f is onto. x1 − 3 x2 − 3
 x + 1 if x is odd ⇒ f(x) is one-one.
5. Hint: f : N → N and f ( x ) = 

 x − 1 if x is even x−2 3y − 2
Onto: Take y =  
⇒ x= ,y≠1
One-One: Let x1 and x2 be any two odd numbers, and x−3 y −1
f (x1) = f (x2) Here for every real number y ≠ 1, there exists a real number
⇒ x1 + 1 = x2 + 1 ⇒ x1 = x2.
⇒ f is one-one. 3y − 2
Now, take x1 and x2 any two even numbers, and f (x1) = f (x2) y −1
⇒ x1 – 1 = x2 – 1  ⇒ x1 = x2
⇒ f is onto.
13. Yes.

f is one-one.
Hint: Show that:f (–1) = g(–1) = 2;
Onto: Let y = x + 1, here x is odd ⇒ x = y – 1. For every even

number y, y – 1 is an odd number. In the same way, prove f (0) = g(0) = 0
when x is even. f (1) = g(1) = 0; f (2) = g(2) = 2
\ f is onto.
14. (i) Domain = [– 2, 2] and Range = [–2, 2]
6. Try yourself. Result is true when R+ is replaced by N.
4 – x2 ≥ 0 ⇒ –2 ≤ x ≤ 2

Relations and Functions 27


E:\AMIT_WORKS\Exam_Guru\EG_Mathematics-12_(working_02-06-2022)\EG_Mathematics-12_working\Open_Files\Chap_1\Chap_1
\ 16-Aug-2022 Amit Proof-5 Reader’s Sign _______________________ Date __________

(ii) Domain = R – {3}, Range = R – [6]. 2. Try yourself.


(iii) Domain = Real numbers, Range = (– ∞, 0) 1
3. Hint: y =
{ }
1 2 − cos x
(iv) Domain = R – {– 2, 2}, Range = y≥ ; y ∈R 2y − 1
2 ⇒ cos x = = 2− 1
y y
15. f is onto and not one-one and hence not bijective.
Since –1 ≤ cos x ≤ 1
16. Hint: Reflexive: Take any element a ∈ X. Since f (a) = f (a)
⇒ (a, a) ∈ R  
⇒ −1 ≤  2 − 1  ≤ 1
⇒ R is reflexive.  y
 
Symmetric: Let a and b be two real numbers. Let (a, b) ∈ R ⇒ −3 ≤  − 1  ≤ −1
 y
⇒ f (a) = f (b) ⇒ f (b) = f (a)
⇒ (b, a) ∈ R ⇒ 3≥ 1 ≥1
y
⇒ R is symmetric. 1 ≤ y ≤1

Transitive: Take a, b, c ∈ X such that (a, b) ∈ R, (b, c) ∈ R 3
1
⇒ f (a) = f (b) and f (b) = f (c) ⇒ Range of f(x) =  , 1
⇒ f (a) = f (c)  3 
4. Try yourself
⇒ (a, c) ∈ R ⇒ R is transitive.
5. Try yourself.
IV. 1. f :R → R and f (x) = |x|.
6. (i) Domain of f (x) = f
One-One: Take x1, x2 ∈ R and f (x1) = f (x2)
⇒ | x1 | = | x2 | (ii) Domain of f (x) = (0, ∞)
⇒ x1 ≠ x2 ⇒ f is not one-one.  2
7. (i) D of f (x) = 0, 
 3
Onto: Take y = |x| for every real number y, there does not
exist a real number x. For example, when y = – 1, there is no (ii) D of f (x) = [–1, 1] – {0}
corresponding number x.
1 
⇒ f is not onto. (iii) D of f (x) =  , 2
2 

Case Based Questions


1. There are two small bookshelves, shelf A and shelf B. Both Two books of a shelf are associated to each other by a
the shelves have four books each. Shelf A has different relation. The difference in their number of pages is at
books for science students whereas the shelf B has different most 10.
books for non-science students. On the basis of above information, answer the following:
(i) As per the given definition, the relation on shelf A is:
(a) {(Maths, Physics), (Chemistry, Biology)}
(b) {(Maths, Physics), (Chemistry, Biology), (Physics,
Biology)}
(c) {(Maths, Maths), (Chemistry, Physics), (Biology,
Maths)}
(d) None of above
(ii) With reference to Q1, the relation on shelf A is:
(a) reflexive only (b) symmetric only
(c) transitive only (d) None of these
Number of pages of each book of both the shelves is given (iii) As per the definition given, the relation on shelf B is:
in the table below:
(a) {(Economics, Accountancy), (Economics, Geography),
Shelf A Shelf B (Accountancy, History)}
Book No. of Book No. of (b) {(Economics, Accountancy), (Economics, History),
Pages Pages (Accountancy, History)}
Maths 132 Economics 145 (c) {(History, Geography), (Accountancy, Geography)}
(d) None of above
Physics 140 Accountancy 149
(iv) With reference to Q3, the relation on the shelf B is:
Chemistry 160 History 154
(a) reflexive only (b) symmetric only
Biology 165 Geography 170 (c) transitive only (d) None of above

28 Mathematics–12
E:\AMIT_WORKS\Exam_Guru\EG_Mathematics-12_(working_02-06-2022)\EG_Mathematics-12_working\Open_Files\Chap_1\Chap_1
\ 16-Aug-2022 Amit Proof-5 Reader’s Sign _______________________ Date __________

(v) Let {(Maths, Maths), (Physics, Physics), (Chemistry, (v) From the given sets n(A × A), n(B × B) and n(A × B) are
Chemistry), (Biology, Biology)} be a relation defined in respectively:
a different manner on shelf A. Then the relation is: (a) 25, 16 and 20 (b) 20, 10 and 15
(a) reflexive only (b) identity only (c) 22, 14 and 18 (d) 24, 15 and 19
(c) reflexive and identity both (d) None of above Ans. (i) (c) 220 (ii) (d) an equivalence relation
Ans. (i) (a) {(Maths, Physics), (Chemistry, Biology)} (iii) (c) 210 (31) (iv) (d) not a function
(ii) (d) None of these (v) (a) 25, 16, 20
(iii) (b) {(Economics, Accountancy), (Economics, 3. A general election of Lok Sabha is a gigantic exercise.
History), (Accountancy, History)} About 911 million people were eligible to vote and voter
(iv) (c) transitive only. turnout was about 67%, the highest ever.
(v) (c) reflexive and identity both
2. There are two different sections of Class 12: Section A and ONE-NATION
Section B including girls as well as boys. A college girl ONE-ELECTION
Teena forms two sets with these students as her college FESTIVAL OF
project. Let A = {a1, a2, a3, a4, a5} and B = {b1, b2, b3,
DEMOCRACY
b4}, where ai’s and bi’s are the students of Section A and
Section B respectively. GENERAL ELECTION —2019

Let I be the set of all citizens of India who were eligible to


exercise their voting right in general election held in 2019.
A relation ‘R’ is defined on I as follows:
R = {(V1, V2) : V1, V2 ∈ I and both use their voting right
in general election – 2019}
(i) Two neighbours X and Y ∈ I. X exercised his voting right
while Y did not cast her vote in general election – 2019.
Which of the following is true?
Section A Section B (a) (X, Y) ∈ R (b) (Y, X) ∈ R
Teena decides to explore these sets for various types of (c) (X, X) ∉ R (d) (X, Y) ∉ R
relations and functions using the information given, answer (ii) Mr. ‘X’ and his wife ‘W’ both exercised their voting right
the following: in general election – 2019. Which of the following is true?
(i) Teena wishes to know the number of reflexive relations (a) Both (X, W) and (W, X) ∈ R
defined on set A. How many such relations are possible? (b) (X,W) ∈ R but (W, X) ∉ R
(a) 0 (b) 25 (c) Both (X, W) and (W, X) ∉ R
(c) 220 (d) None of above (d) (W, X) ∈ R but (X, W) ∉ R
(ii) Let R : A → A, R = {(x, y) : x and y are students of same (iii) Three friends F1, F2 and F3 exercised their voting right
sex}. Then relation R is: in general election – 2019, then which of the following is
(a) reflexive only true?
(b) reflexive and symmetric but not transitive (a) (F1, F2) ∈ R, (F2, F3) ∈ R and (F1, F3) ∈ R
(c) reflexive and transitive but not symmetric (b) (F1,F2 ) ∈ R, (F2, F3) ∈ R and (F1, F3) ∉ R
(d) an equivalence relation (c) (F1, F2 ) ∈ R, (F2, F2) ∈ R but (F3, F3) ∉ R
(iii) Teena and her friend Reena interested to know the number (d) (F1, F2) ∉ R, (F2, F3) ∉ R and (F1, F3) ∉ R
of symmetric relations defined on both the sets A and B, (iv) The above defined relation R is
separately. She decides to find the symmetric relation on
set A, while Reena decides to find the symmetric relation (a) symmetric and transitive but not reflexive
on set B. What is difference between their result? (b) universal relation
(a) 1024 (b) 210 (15) (c) equivalence relation
10
(c) 2 (31) (d) None of these (d) reflexive but not symmetric and transitive
(iv) Let R : A → B, R = {(a1, b1), (a1, b2), (a2, b1), (a3, b3), (a4, (v) Mr. Shyam exercised his voting right in General Election –
b2), (a5, b2)}, then R is: 2019, then Mr. Shyam is related to which of the following?
(a) neither one-one nor onto (a) All those eligible voters who cast their votes
(b) one-one but, not onto (b) Family members of Mr. Shyam
(c) only onto, but not one-one (c) All citizens of India
(d) not a function (d) Eligible voters of India

Relations and Functions 29


E:\AMIT_WORKS\Exam_Guru\EG_Mathematics-12_(working_02-06-2022)\EG_Mathematics-12_working\Open_Files\Chap_1\Chap_1
\ 16-Aug-2022 Amit Proof-5 Reader’s Sign _______________________ Date __________

Ans. (i) (d) (X, Y) ∉ R 5. An organization conducted bike race under two different
(ii) (a) Both (X, W) and (W, X) ∈ R categories–boys and girls. Totally there were 250
(iii) (a) (F1, F2) ∈ R, (F2, F3) ∈ R and (F1, F3) ∈ R participants. Among all of them finally three from Category
(iv) (c) equivalence relation 1 and two from Category 2 were selected for the final race.
(v) (a) All those eligible voters who cast their votes. Ravi forms two sets B and G with these participants for
4. Sherlin and Danju are playing Ludo at home during the final race.
Covid-19. While rolling the dice, Sherlin’s sister Raji Let B = {b1, b2, b3} and G = {g1, g2} where B represents
observed and noted the possible outcomes of the throw the set of boys selected and G the set of girls who were
every time belongs to set {1, 2, 3, 4, 5, 6}. Let A be the selected for the final race.
set of players while B be the set of all possible outcomes.
Then, A = {S, D} and B = {1, 2, 3, 4, 5, 6}

Ravi decides to explore these sets for various types of


relations and functions.
(i) Ravi wishes to form all the relations possible from B to G.
How many such relations are possible?
(a) 26 (b) 25 (c) 0 (d) 23
(i) Let R : B → B be defined by R = {(x, y): y is divisible by (ii) Let R : B → B be defined by R = {(x, y): x and y are students
x} is of same sex}, Then this relation R is
(a) reflexive and transitive but not symmetric (a) equivalence
(b) reflexive and symmetric and not transitive (b) reflexive only
(c) not reflexive but symmetric and transitive (c) reflexive and symmetric but not transitive
(d) equivalence (d) reflexive and transitive but not symmetric
(ii) Raji wants to know the number of functions from A to B. (iii) Ravi wants to know among those relations, how many
How many functions are possible? functions can be formed from B to G?
(a) 62 (b) 26 (c) 6! (d) 212 (a) 22 (b) 212 (c) 32 (d) 23
(iii) Let R be a relation on B defined by R = {(1, 2), (2, 2), (iv) Let R : B → G be defined by R = {(b1, g1), (b2, g2),
(1, 3), (3, 4), (3, 1), (4, 3), (5, 5)}. Then R is (b3, g1)}, then R is
(a) symmetric (b) reflexive (a) injective
(c) transitive (d) None of these (b) surjective
(iv) Raji wants to know the number of relations possible from (c) neither surjective nor injective
A to B. How many relations are possible? (d) surjective and injective
(a) 62 (b) 26 (c) 6! (d) 212 (v) Ravi wants to find the number of injective functions from
(v) Let R : B → B be defined by R = {(1, 1), (1, 2), (2, 2), B to G. How many injective functions are possible?
(3, 3), (4, 4), (5, 5), (6, 6)}, then R is (a) 0 (b) 2! (c) 3! (d) 0!
(a) symmetric (b) reflexive and transitive Ans. (i) (a) 2 6
(ii) (a) equivalence
(c) transitive and symmetric (d) equivalence (iii) (d) 23 (iv) (b) surjective
Ans. (i) (a) reflexive and transitive but not symmetric (v) (a) 0
(ii) (a) 62 6. Students of Grade 12, planned to plant saplings along
(iii) (d) None of these straight lines, parallel to each other to one side of the
(iv) (d) 212 playground ensuring that they had enough play area. Let
(v) (b) reflexive and transitive us assume that they planted one of the rows of the saplings

30 Mathematics–12
E:\AMIT_WORKS\Exam_Guru\EG_Mathematics-12_(working_02-06-2022)\EG_Mathematics-12_working\Open_Files\Chap_1\Chap_1
\ 16-Aug-2022 Amit Proof-5 Reader’s Sign _______________________ Date __________

along the line y = x – 4. Let L be the set of all lines which children. Raji found that the swing traced the path of a
are parallel on the ground and R be a relation on L. parabola as given by y = x2.

(i) Let relation R be defined by R = {(L1, L2): L1 || L2 where (i) Let f : R → R be defined by f (x) = x2 is
L1, L2 ∈ L} then R is (a) neither surjective nor injective
(a) equivalence relation (b) surjective
(b) only reflexive relation (c) injective
(c) not reflexive relation (d) bijective
(d) symmetric but not transitive relation
(ii) Let f : N → N be defined by f (x) = x2 is
(ii) Let R = {(L1, L2) : L1 ^ L2 where L1, L2 ∈ L}. Which
(a) surjective but not injective
of the following is true?
(b) surjective
(a) R is symmetric but neither reflexive nor transitive
(c) injective
(b) R is reflexive and transitive but not symmetric
(d) bijective
(c) R is reflexive but neither symmetric nor transitive
(iii) Let f : {1, 2, 3,….} → {1, 4, 9,….} be defined by
(d) R is an equivalence relation f (x) = x2 is
(iii) The function f : R → R defined by f (x) = x – 4 is (a) bijective
(a) bijective (b) surjective but not injective
(b) surjective but not injective (c) injective but not surjective
(c) injective but not surjective
(d) neither surjective nor injective
(d) neither surjective nor injective
(iv) Let : N → R be defined by f (x) = x2 . Range of the function
(iv) Let f : R → R be defined by f (x) = x – 4. Then the range among the following is
of f (x) is
(a) {1, 4, 9, 16,…} (b) {1, 4, 8, 9, 10,…}
(a) R (b) Z (c) W (d) Q
(c) {1, 4, 9, 15, 16,…} (d) {1, 4, 8, 16,…}
(v) Let R = {(L 1 , L 2 ) : L 1 is parallel to L 2 and L 1 : y =
x – 4}, then which of the following can be taken as L 2 ? (v) The function f : Z→Z defined by f (x) = x2 is
(a) 2x – 2y + 5 = 0 (b) 2x + y = 5 (a) neither injective nor surjective
(c) 2x + 2y + 7 = 0 (d) x + y = 7 (b) injective
Ans. (i) (a) equivalence relation (c) surjective
(ii) (a) R is symmetric but neither reflexive nor transitive (d) bijective
(iii) (a) bijective (iv) (a) R Ans. (i) (a) neither surjective nor injective
(v) (a) 2x – 2y + 5 = 0 (ii) (c) injective (iii) (a) bijective
7. Raji visited the exhibition along with her family. The (iv) (a) {1, 4, 9, 16,…}
exhibition had a huge swing, which attracted many (v) (a) neither injective nor surjective

Author’s Comments
Questions based on following types are very important for Exams. So, students are advised to revise them thoroughly.
1. Relations: To show that the given relation is reflexive, symmetric and transitive or (equivalence relation).
2. Functions: To show that given function is one-one and onto (bijective).

Relations and Functions 31


E:\AMIT_WORKS\Exam_Guru\EG_Mathematics-12_(working_02-06-2022)\EG_Mathematics-12_working\Open_Files\Chap_1\Chap_1
\ 16-Aug-2022 Amit Proof-5 Reader’s Sign _______________________ Date __________

COMMON ERRORS
ERRORS CORRECTIONS
(i) In a set A = {1, 2, 3}, a relation R = {(1, 1), (2, 2), (3, (i) In reflexive relation, every element of set A must be related
2)}. Here R is not reflexive relation because (3, 3) is not to itself.
present in R. Students make a mistake that if only (1, 1)
or (1, 1), (2, 2) are present R is reflexive.
(ii) To check the symmetric relation R. (ii) Find R–1, if R = R–1 only then R is symmetric.
(iii) To check the transitivity of a relation R. (iii) Its every ordered pair must be tested.
(iv) If x ∈ N and x12 = x22 ⇒ x1 = ± x2 (iv) Here x1 = x2, because x1 = –x2 is not possible because x is
a natural number.
(v) If x ∈ R and x12 = x22 ⇒ x1 = x2 (v) Here x1 = ± x2, because x is a real number, here negative
number is possible.
(vi) While checking injectivity, surjectivity of a function, (vi) The first step while checking injectivity, surjectivity, take
students ignore the set in which function is defined, i.e., f care of set under consideration.
: N → N or
f : Z → Z or f : R → R etc
(vii) Identification of co-domain, range (vii) Co-domain is the set of all second elements, i.e., if
f : A → B, then B is the co-domain.
Range is the set of all second entries of the ordered pairs.

REVISION CHART—RELATIONS
Empty Relation Reflexive Relation
A relation R in set A is called empty A relation R in set A is called reflexive
relation if no element of set A related if (a, a) ∈ R for all a ∈ A or R is reflexive
to any element of set A, i.e, R = f. in set A if (a, a) ∈ R ∀ a ∈ A.

Relations
(i) Relation R from set A to set B: Relation R from set A to set B is a subset of Cartesian product (A × B)
or Relation from set A to set B = {(a, b) : a ∈ A and b ∈ B}
If A = {1, 2} and B = {3, 4} then R = {(1, 3), (2, 4)}
(ii) Relation R in set A: Relation R in set A is a subset of Cartesian product (A × A).
If A = {1, 2, 3} then Relation R = {(1, 1), (1, 2), (2, 3)}

Symmetric Relation Equivalence Relation Transitive Relation


A relation R in set A is said to be If a relation R in set A is reflexive, A relation R in set A is said to be
symmetric if (a, b) ∈ R and (b, a) ∈ R, symmetric and transitive, then transitive if (a, b) ∈ R and (b, c) ∈ R
a, b ∈ A or if relation R = its inverse relation R is called an equivalence ⇒ (a, c) ∈ R for a, b, c ∈ A.
relation R–1, then relation R is called relation.
symmetric relation.

32 Mathematics–12
E:\AMIT_WORKS\Exam_Guru\EG_Mathematics-12_(working_02-06-2022)\EG_Mathematics-12_working\Open_Files\Chap_1\Chap_1
\ 16-Aug-2022 Amit Proof-5 Reader’s Sign _______________________ Date __________

REVISION CHART—FUNCTIONS

One-one (or injective function)


A function f : X → Y is called one-one
(or injective) if the images of the
distinct elements of set X under f are
distinct, i.e., for every x1, x2 ∈ X;
Domain of the Function Co-domain of the Function f
In a function f : A → B, y = f (x); set In a function f : A → B, y = f (x); set B
A is called domain of the function f. is called co-domain of the function f.

Functions
Let A and B be two non-
empty sets. Then a relation
Image and Pre-image of an Element
of a Function
from set A to set B is called a Range of a Function
function if each element of
If the element x ∈ A corresponds to In a function f : A → B, y = f (x) the
y ∈ B under the function f : A → B, set A has correspondence subset of B containing the image of
y = f (x) then y is called image of x with exactly one element the elements of set A is called range
under f and x is called pre-image of of set B. of the function f.
y under f.
A function f from set A
to set B is denoted as:
f : A → B and y = f (x).

Onto or (Surjective) function One-one and Onto or (Bijective)


A function f : X → Y is called onto function Function
if every element of set Y is the image of A function f : X → Y is said to be
some element of set X under f, i.e., for one-one and onto or Bijective, when
every y ∈ Y there exists an element function f is both one-one and onto
x ∈ X such that f (x) = y. function.

Relations and Functions 33


E:\AMIT_WORKS\Exam_Guru\EG_Mathematics-12_(working_02-06-2022)\EG_Mathematics-12_working\Open_Files\Chap_2\Chap_2
\ 16-Aug-2022 Amit Proof-4 Reader’s Sign _______________________ Date __________

Topics Covered
2 Inverse Trigonometric Functions

2.1 Inverse of a Function


2.2 Principal Value Branch and Graphs of an inverse trigonometric function.

C hapter map
INVERSE TRIGONOMETRIC FUNCTIONS

Domain and Range Inverse of a Function

sine cosine tangent secant cosecant cotangent

Principal Value Branch

sin–1 x cos–1 x tan–1 x sec–1 x cosec–1 x cot–1 x

We have studied trigonometric functions in detail in Class 11. Let us revise the Domains and Ranges of Trigonometric Ratios.

Trigonometric Ratios, their Domains and Ranges (Recap)


Following are the six trigonometric ratios with their respective domains and ranges:
1. y = sin x, Domain = {x : x ∈ R},
Range = {y : y ∈ [– 1, 1]} or y = sin x : R → [– 1, 1].
2. y = cos x, Domain = {x : x ∈ R},
Range = {y : y ∈ [– 1, 1]} or y = cos x : R → [– 1, 1]

{
3. y = tan x, Domain = R − x : x = (2n + 1) π , n ∈ Z . Range = R.
2 }
{ π
or y = tan x : R − x : x = (2n + 1) , n ∈ Z → R
2 }
4. y = cot x, Domain = R – {x : x = nπ, n ∈ Z}. Range = R
or y = cot x : R – {x : x = np, n ∈ Z} → R

{ }
5. y = sec x, Domain = R − x : x = ( 2n + 1) π , n ∈ Z , Range = R – (–1, 1)
2

{ π
or y = sec x : R − x : x = (2n + 1) , n ∈ Z} → R − ( −1,1)
2
6. y = cosec x, Domain = R – {x : x = np, n ∈ Z}, Range = R – (– 1, 1)
or y = cosec x : R – {x : x = np, n ∈ Z} → R – (–1, 1)

34
E:\AMIT_WORKS\Exam_Guru\EG_Mathematics-12_(working_02-06-2022)\EG_Mathematics-12_working\Open_Files\Chap_2\Chap_2
\ 16-Aug-2022 Amit Proof-4 Reader’s Sign _______________________ Date __________

Topic 1. Inverse of a Function


If f : X → Y such that f (x) = y, x ∈ X and y ∈ Y, is one-one and 1
onto, then we can define a unique function g : Y → X such that (ii) sin −1 x ≠
sin x
g(y) = x where x ∈ X and y ∈ Y. Here domain of g = range of f, (iii) After this point we shall write inverse of sine function
and range of g = domain of f. Such a function g is called inverse −π π
as y = sin–1 x : [–1, 1] →  ,  .
of function f and is denoted as f –1. Here function g is one-one  2 2 
and onto, g also has its inverse. Inverse of function g is function Graph of y = sin x and y = sin–1 x
f, therefore f and g are inverse of each other.

( ) ( )
−1 −1
Thus: g −1 = f −1 = f or f −1 = g −1 =g

Thus : ( f –1 of )(x) = f –1 [ f (x)] = f –1 (y) = x and


( f of –1 )(y) = f [ f –1(y)] = f (x) = y
Now let us define inverse of trigonometric functions.

Inverse of Sine Function


x = sin q : R → [–1, 1] is the sine function whose domain is R y = sin x :  −π , π  → [ −1,1] y = sin −1 x : [ −1,1] →  −π , π 
(all real numbers) and range is [–1, 1]. For the given domain R,  2 2   2 2 
x = sin q is not one-one and onto function. So, for this domain, Graph of y = sin x Graph of y = sin–1 x
we cannot define inverse of sine function. Inverse of Cosine Function
To make x = sin q, one-one and onto function we shall have x = cos q : R → [–1, 1] is cosine function with domain R and
to re-define the domain of the sine function. The sine function range [–1, 1]. Cosine function with domain R is not a one-one
can become one-one and onto function if we choose its domain and onto function. Hence, we cannot define inverse of x = cos q
as one of the intervals: for this domain R.
−3π −π   −π π   π 3π  To make it one-one and onto function, we shall re-define its
⋅⋅⋅  , , , , , ⋅⋅⋅ .
 2      domain. If we select one of the intervals out of: ... [–p, 0], [0, p],
2   2 2   2 2 
[p, 2p] ... etc. as domain, x = cos q becomes one-one and onto.
−π π
Interval  ,  is taken as domain of the sine function. [0, p] is taken as the domain of the cosine function to make it
 2 2  one-one and onto. Hence, x = cos q : [0, p] → [–1, 1] is cosine
−π π
The sine function: x = sin q with domain  ,  and function which is one-one and onto (bijective) function and hence
 2 2  its inverse can be defined.
range [–1, 1] is one-one and onto function and hence its inverse
Inverse of x = cos q : [0, p] → [–1, 1] is q = cos–1 x : [–1, 1]
can be defined.
→ [0, p]. This shows that domain of cos­–1 x is [–1, 1] and range
Inverse of x = sin q is denoted as: q = sin–1 x and read as ‘sine is [0, p].
inverse x’, or inverse of (x = sin q) is: This means: –1 ≤ x ≤ 1 and 0 ≤ cos–1 x ≤ p.
−π π
θ = sin −1 x : [ −1,1] →  ,  Principal Value Branch of cos–1 x
 2 2 
The range of cos–1 x, [0, p] is called principal value branch
–1
Hence, q = sin x is the inverse of sine function. Domain of cos–1 x. This means all the value of cos–1 x will lie in the class
−π π interval [0, p].
and range of inverse of sine function are [–1, 1] and  , 
 2 2  Note: (i) cos–1 x is an angle whereas cos x is a number.
respectively.
(ii) cos −1 x ≠ 1 or ( cos x )
−1

Principal Value Branch of sin–1 x cos x


(iii) Now onward, inverse of cos function will be written
−π π as y = cos–1x.
The range  ,  of sin –1 x is called its principal-
 2 2 
Graph of y = cos x and y = cos–1 x
value-branch. This means, now every value of sin–1 x will
−π π
be taken in the interval  ,  .
 2 2 
Principal Value of an Inverse Trigonometric
Function
The value of an inverse trigonometric function which lies in its
principal value branch is called principal value of the inverse
trigonometric functions. Graph of y = cos x : Graph of y = cos–1 x :
Note: (i) q = sin–1 x is an angle whereas sin q is a number [0, p] → [–1, 1] [–1, 1] → [0, p]

Inverse Trigonometric Functions 35


E:\AMIT_WORKS\Exam_Guru\EG_Mathematics-12_(working_02-06-2022)\EG_Mathematics-12_working\Open_Files\Chap_2\Chap_2
\ 16-Aug-2022 Amit Proof-4 Reader’s Sign _______________________ Date __________

In a similar way, we can define inverse of cosec x, inverse of sec x, inverse of tan x and inverse of cot x. Their principal value branches
and graphs are given below:
Inverse of Cosecant Function
−π π
We can define inverse of x = cosec q as q = cosec–1 x : R – (–1, 1) →  ,  − {0} . It is read as cosec inverse x.
 2 2 
−π π
Here q = cosec–1 x has domain R – (–1, 1) and Range is  ,  − {0} .
 2 2 
Principal Value Branch of cosec–1 x
−π π
The range of cosec −1x =  ,  − {0} is the principal value branch of the cosec–1 x.
 2 2 
Note: (i) cosec–1 x is an angle whereas cosec x is a number.
(ii) cosec −1x ≠ ( cosec x ) or
−1 1
.
cosec x
(iii) Now onward, inverse of cosec function will be written as: y = cosec–1 x.
Graph of y = cosec x and y = cosec–1 x

 −π π  Graph of y = cosec −1x = R − ( −1,1) →  −π , π  − {0}


Graph of y = cosec x =  2 , 2  − {0} → ( −1,1)
   2 2 

Inverse of Secant Function


π
We can define inverse of x = sec q as q = sec–1 x : R − ( −1,1) → [ 0, π ] − 2 {}
It is read as ‘secant inverse x’. Hence, q = sec x has domain: R – (–1, 1) and Range: [ 0, π ] − π .
–1
–1
2 {}
Principal Value Branch of sec x
π
The range of sec–1 x, i.e., [ 0, π ] −
2 {}
is the principal value branch of sec–1 x.

Note: (i) sec–1 x is an angle whereas sec x is a number.


(ii) sec −1 x ≠ 1 or (sec x )
−1
sec x
(iii) Now onward, inverse of secant function will be represented as: y = sec–1 x.
Graph of y = sec x and y = sec–1 x

2 {}
Graph of y = sec x : [ 0, π ] − π → R − ( −1,1) Graph of y = sec −1 x = R − ( −1,1) → [ 0, π ] − π
2 {}
36 Mathematics–12
E:\AMIT_WORKS\Exam_Guru\EG_Mathematics-12_(working_02-06-2022)\EG_Mathematics-12_working\Open_Files\Chap_2\Chap_2
\ 16-Aug-2022 Amit Proof-4 Reader’s Sign _______________________ Date __________

Inverse of Tangent Function


π
x = tan θ : R −  x : x = ( 2n + 1) , n ∈ Z → R is the tangent function with domain R −  x : x = ( 2n + 1) π , n ∈ Z and range R.
 2   2 

( )
Now q = tan–1 x : R → −π , π is the inverse function of x = tan q: with domain R and range −π , π .
2 2
–1
2 2 ( )
Principal Value Branch of tan x

( )
The range of q = tan x, i.e., −π , π is the principal value branch of q = tan–1 x.
–1
2 2
Note: (i) tan–1 x is an angle whereas tan x is a number.
(ii) tan −1 x ≠ ( tan x ) or 1
−1
tan x
(iii) Now onward, inverse of tangent function will be represented as: y = tan–1 x.
Graph of y = tan x and y = tan–1 x

2 2 ( )
Graph of y = tan x : −π , π → R Graph of y = tan −1 x : R → −π , π
2 2 ( )
Inverse of Cotangent Function
x = cot q : R – {x : x = np, n ∈ Z} → R is a cotangent function with domain: R – {x : x = np, n ∈ Z} and range R.
Hence: q = cot–1 x : R → (0, p) is the inverse function of x = cot q : (0, p) → R. q = cot–1 x : R → (0, p) is read as ‘cot inverse x’.
Domain and range of q = cot–1 x are R and (0, p) respectively.
Principal Value Branch of q = cot–1 x
The range of q = cot–1 x i.e., (0, p) is the principal value branch of q = cot–1 x.
Note: (i) cot–1 x is an angle whereas cot x is a number.
1
or ( cot x ) .
−1
(ii) cot −1 x ≠
cot x
(iii) Now onward, inverse function of cotangent will be represented as: y = cot–1 x.
Graph of y = cot x and y = cot–1 x

Graph of y = cot x : R – {x : x = np, n ∈ Z} → R Graph of y = cot–1 x : R → (0, p)

Inverse Trigonometric Functions 37


E:\AMIT_WORKS\Exam_Guru\EG_Mathematics-12_(working_02-06-2022)\EG_Mathematics-12_working\Open_Files\Chap_2\Chap_2
\ 16-Aug-2022 Amit Proof-4 Reader’s Sign _______________________ Date __________

The Domain and Ranges (Principal Value Branches) of Inverse Trigonometric Functions
Inverse Trigonometric Functions Domains Ranges (Principal Value Branches)
 −π , π 
y = sin–1 x [–1, 1]
 2 2 
y = cos–1 x [–1, 1] [0, p]

 −π , π  − {0}
y = cosec–1 x R – (–1, 1)
 2 2 

y = sec–1 x R – (–1, 1) {}
[0, π ] − π
2

y = tan–1 x R ( )
−π π
,
2 2
y = cot–1 x R (0, p)

Example 1. The principal value of inverse trigonometric function


q= π
()
\
1 is 3
sin −1

(a) π
2

3
(b) π
4
(c) π
6
(d) π
⇒ P.V. of sec–1 (2) =
π
3
∈[ 0, π ] −
π
2 {}
Hence, option (c) is the correct answer.

2 ()
Solution. sin −1 1 : P.V.B. of sin −1 x :  −π , π 
 2 2  Example 4. The principal value of inverse trigonometric function
cosec–1 (1) is
Let sin −1
1
2 ()=q ⇒ sin q =
1
2
(a)
π
3
(b) π
6
(c) π
4
(d) π
2
π  − π π
⇒ sin q = sin –1
Solution. cosec (1) : P.V.B. of cosec x :–1
,  − {0}
6  2 2 
π  −π π  π
⇒ q = ∈ ,  Let cosec–1 (1) = q ⇒ cosec θ = 1 = cosec
6  2 2 2

⇒ P.V. of sin
−1
() 1
2
π −π π
= ∈ , 
6  2 2 
Since ∈
π  −π π 
, − {0}
2  2 2 
\ θ= π
2
π −π π
Hence, option (c) is the correct answer. ⇒ P.V. of cosec–1 (1) = ∈  ,  − {0}
2  2 2 
Example 2. The principal value of inverse trigonometric function
Hence, option (d) is the correct answer.
tan–1 (1) is
π Example 5. The principal value of inverse trigonometric function
(a) π (b) π (c) (d) π
3 4 2 6  − 3
sin −1 
( )
is
Solution. tan (1) : P.V.B. of tan x :
−1 −1 −π π
,  2 
2 2
−π π π
π (a) (b) π (c) (d)
Let tan–1 (1) = q ⇒ tan q = 1 = tan 3 3 6 4
4
⇒ θ= ∈ π
4

(
π
2 2
,
π
) −1  − 3 
Solution. sin 
 2 
 
−π π
: P.V.B. of sin −1 x :  , 
 2 2 
⇒ P.V. of tan–1 (1) = π ∈ −π , π
4 2 2 ( )  − 3
Let sin −1  =q ⇒ sin θ =
− 3
Hence, option (b) is the correct answer.  2  2

() ( ) ( )
Example 3. The principal value of inverse trigonometric function π −π −π  −π π 
sec–1 (2) is ⇒ sin q = − sin = sin since ∈ ,
3 3 3  2 2 
π
(a) π (b) p (c) π (d)
2 3 2 −π

{}
\ q=
π 3
Solution. (c) sec ( 2) : P.V.B. of sec x : [ 0, π ] −
−1 −1
2
 − 3  −π  −π π 
Let sec–1 (2) = q ⇒ sin −1  = ∈ ,
 2  3  2 2 
π π
⇒ sec q = 2 = sec . Since ∈[ 0, π ] −
3 3
π
2 {} Hence, option (a) is the correct answer.

38 Mathematics–12
E:\AMIT_WORKS\Exam_Guru\EG_Mathematics-12_(working_02-06-2022)\EG_Mathematics-12_working\Open_Files\Chap_2\Chap_2
\ 16-Aug-2022 Amit Proof-4 Reader’s Sign _______________________ Date __________

Example 6. The principal value of inverse trigonometric function


−1  −2  −π π
Solution. cosec   : P.V.B. of cosec x :  ,  − {0}
( )
−1
−1  3  2 2
cos −1 is
2

(a) π (b)
π 2π
(c) 3
π
(d) 6
−1  −2 
Let cosec   = q
 3
⇒ cosec θ =
−2
3
= − cosec
π
3 ()
4
( ) ( )
3
−π −π  −π π 
 2 2  { }
( ) −1 ⇒ cosec q = cosec , since ∈ , − 0
Solution. cos −1 : P.V.B. of cos −1 x : [ 0, π ] 3 3
2
−1  −2 

( )
−π −π  −π π 
Let cos −1
( )−1
2
=q ⇒ cos θ =
−1
2
= − cos
π
3
\ q=
3
 ⇒ cosec   =
 3
∈ , − {0}
3  2 2 

( ) ( ) ( )[
Hence, option (b) is the correct answer.
π 2π 2π
cos q = cos π − = cos since ∈ 0, π ] Example 9. The principal value of inverse trigonometric function
3 3 3
sec–1 (–1) is
\ q=

3
−1
⇒ cos − ( ) 1
2
=

3
∈[ 0, π ]
(a)
π
2
(b)
−π
3
(c) π
π
(d) 3

{}
Hence, option (c) is the correct answer.
π
Example 7. The principal value of inverse trigonometric function Solution. sec −1 ( −1) : P.V.B. of sec −1 x : [ 0, π ] −
2
 −1 
tan −1   is Let sec–1 (–1) = q ⇒ sec q = –1 = – sec 0 = sec p
 3

(a)
π
(b)
π
(c)
π
(d)
−π
Since π ∈[ 0, π ] − π
2 {} \ q=p

{}
6 4 3 6
⇒ P.V.B. of sec–1(–1) = p ∈ [0, p] – π
 
Solution. tan −1  −1  : P.V.B. of tan −1 x : −π , π
 3 2 2 ( ) Hence, option (c) is the correct answer.
2

Example 10. The principal value of inverse trigonometric


−1  −1   −1 
Let tan   = q
 3
⇒ tan θ =  
 3 function cot −1 − 3 ( ) is

tan q = − tan () ( )
π
6
= tan −
π
6
since
−π
6

−π π
,
2 2 ( )( ) π
(a) 6 (b)

3

(c) 4 (d)

6

( )
\ q=
−π
6
 
⇒ tan −1  −1  = −π ∈ −π , π
 3 6 2 2 ( ) Solution. cot −1 − 3 : P.V.B. of cot −1 x : ( 0, π )

Hence, option (d) is the correct answer.


Let cot −1 (− 3) = q ( )
⇒ cot θ = − 3 = − cot π
6
Example 8. The principal value of inverse trigonometric function
 −2 
cosec −1   is

π
cot q = cot π −
6 ( ) ( )
= cot

6
, since

6
∈( 0, π )

 3 \ q=

6
⇒P.V.B.of cot −1 − 3 =

6 ( )
∈ ( 0, π )
π −π π π
(a) (b) (c) (d) 4 Hence, option (d) is the correct answer.
3 3 6

EXERCISE 2.1
I. Multiple Choice Questions (MCQs) 3. The principal value of inverse trigonometric function
Choose the correct answer from the given options. sec– 1(2) is
π π π
1. The principal value of inverse trigonometric function (a) 4 (b) (c) 3 (d) π
2
sin −1
1
2 ()
is 4. The principal value of inverse trigonometric function
cosec–1(1) is
π π π
(a) π (b) (c) 4 (d)
6 8 3 π π π π
(a) (b) (c) (d)
2. The principal value of inverse trigonometric function 3 6 2 4
tan–1(1) is
π π π
5. The value of sin  π − sin −1 −1  is
3 2  ( )
(a) π (b) (c) 3 (d)
2 4 6 (a) 3 (b) –1 (c) 2 (d) 1

Inverse Trigonometric Functions 39


E:\AMIT_WORKS\Exam_Guru\EG_Mathematics-12_(working_02-06-2022)\EG_Mathematics-12_working\Open_Files\Chap_2\Chap_2
\ 16-Aug-2022 Amit Proof-4 Reader’s Sign _______________________ Date __________

 9π 
6. The principal value of tan–1  tan  is
 8
 
 2  2 ( )
(ii) 2 sin −1  3  − cos −1 −1 + 3 tan −1(1)

π
8
(a) (b)

8
(c) −
π
8
(d) −

8
 7π 1 
(iii) tan  − 2 sin −1  .
 12 2  ()
[CBSE 2022] III. Long Answer Type Question
7. What is the principal value of sec–1 (–2)?

(a)

(b)

(c) 3
4π π
(d) 2
 3
 2 
1
1. Find the value of: 2 sin −1   − cos −1 − + 3 tan −1 (1)
2 ( )
2 3
 1
8. The principal value of cos–1   + sin–1
 2
 1 
 − 
 7π
 12
1 
2. Find the value of: tan  − 2 sin −1  .
2  ()
2

(a)
π
(b)
π
(c)
π
(d)
π 3. Find the value of: tan −1 (1) + cos −1 ( )
−1
2
+ sin −1
−1
2 ( )
12 4 3 6
4. Find the value of:
[CBSE 2022]
II. Short Answer Type Questions-I 
cos cos −1
 2 ( )
−1 π  π  − 3 
+  + sin  − sin −1 
3 6  2  
.
1. (i) Write the principal value of tan −1 ( ) ( )
3 − cot −1 − 3
5. Find the value of:
[A.I. 2013]
1
(ii) Write the principal value of tan −1 (1) + cos −1 − .
2 ( )
π
 6 

cos  + 2 tan −1 (1) + sin 3 sin −1

1
2
1 
+ 2 cos −1  .
2  () ()
[Delhi 2013]
2 ( ) ( )
6. Find the value of: 3 tan −1 (1) − cos −1 −1 + 2 sin −1 −1 .
2

(iii) Write the value of tan −1 sin

−π 
2  ( )
.[A.I. 2014] 7. Find the value of:
(iv) Write the principal value of cos–1[cos (680°)].
[Delhi 2014]
2 tan −1
( 3 ) − sec ( − 2 ) + cosec
−1 −1  2 
  .
3

(v) Write the value of cos −1


−1
+ 2 sin −1 ( )
1
. () 8. Find the value of:

( )
2 2  − 3 −1  −1 
[Foreign 2014] 3 sin −1 
+ 2 cos −1 + tan −1   .
 2  2  3
2. Find the principal values of each of the following inverse 9. Find the value of:
trigonometric functions:

(i) tan −1(1) + cos −1


−1
2 ( )
+ sin −1
−1
2 ( )


3 sin

−1 −1
2 ( )
 − 3
+ 2 sin −1 
 2
−1  −1  
 − tan  3   .

Answers 2.1
I. 1. (a)
π
6
sec–1(2) : P.V.B. of sec–1 x = [ 0,π ] −
π
2 {}
sin –1 1
2() −π π
: P.V.B of sin −1 x =  , 
 2 2  Let sec–1 (2) = q
⇒ sec q = 2 = sec
π
3
Let sin–1
1
2 ()
= q ⇒ sin q = = sin
1
2
π
6 ⇒ q=
π
3
∈[ 0, π ] −
π
2 {}
π  −π π  π
⇒ q = ∈ ,  4. (c)
6  2 2 2
π  −π π 
2. (b) cosec–1 (1) : P.V.B. of cosec–1 x =  ,  − {0}
4  2 2
tan (1) : P.V.B. of tan–1 x =
–1 −π π
2 2
,( ) –1
Let cosec (1) = q ⇒ cosec q = 1 = cosec
π
2
π π ∈  −π , π  0
Let tan–1 (1) = q ⇒ tan q = 1 = tan ⇒ q= −{ }
4 2  2 2 
⇒ q=
π ∈ −π , π
( ) 5. (d) 1

( )
3. (c) π
4 2 2 
 3 2

sin  π − sin −1 −1  = sin  π + sin −1 1 
  3 2  ()
3
[Q sin–1(–x) = –sin–1 x]

40 Mathematics–12
E:\AMIT_WORKS\Exam_Guru\EG_Mathematics-12_(working_02-06-2022)\EG_Mathematics-12_working\Open_Files\Chap_2\Chap_2
\ 16-Aug-2022 Amit Proof-4 Reader’s Sign _______________________ Date __________

π π
= sin  +  = sin
 3 6 
π =1
2
 ()  −1 1
()
π
Qsin 2 = 6  (ii) tan −1 (1) + cos −1 −
1
2 ( )
= tan (1) + π − cos
−1 −1 1
2 ()
π [Q cos (– x) = p – cos–1 x]
–1
6. (a)
8 π +π− π
=
4 3
7. (b) 2π 3π + 12π − 4π = 11π
3 =
12 12
sec–1(–2) : P.V.B. of sec–1 x = [ 0,π ] −
π
2 {} (iii)
−π
4
(iv) 40° (v) p

Let sec–1 (–2) = q ⇒ sec q = –2 3π 3π


2. (i) (ii) (iii) 1
4 4

⇒ –sec q = 2
3π 3π
π III. 1. 2. 1 3. 4. 0

⇒ sec(p – q) = 2 ⇒ p–q= 4 4
3 1
5. − 2 6. −π


π 2π ∈ 0, π − π
q= π− =
3 3
[ ] 2 {} 4

π
7. 2 tan −1 ( 3 ) − sec ( − 2 ) + cosec
−1 −1 
2 
 
3
8. (a)
12
= 2 tan −1 ( 3 ) − π + sec ( )
−1  
2 + cosec −1  2 
II. 1. (i) tan −1 ( 3 ) − cot ( − 3 )
−1

[Q sec–1 (–x) = p – sec–1 x]


 3
−1
= tan 3 −  π − cot −1 3 
On solving, we get
[Q cot–1 (–x) = p – cot–1 x]
π π π π
= tan −1 3 − π + cot −1 3 = 2 × − π + + =
3 4 3 4
π − π + π = π − π = −π π
= 8. 6 9. –p
3 6 2 2

Case Based Questions


1. (iii) ∠BCA = b =
B
(a) tan–1 1
2 () (b) tan–1 (2)

A
x
 
d
C
(c) tan–1  1 
 3
(d) tan–1 ( 3)
D

(iv) ∠ABC =
Two men on either side of a temple of 30 metres high (a) π (b) π
observe its top at the angles of elevation a and b 4 6
π
respectively (as shown in the figure above). The distance (c) π (d)
2 3
between the two men is 40 3 metres and the distance
between the first person A and the temple is 30 3 metres. (v) Domain and Range of cos−1 x =
Based on the above information answer the following: (a) ( −1, 1 ), (0, π) (b) [ −1, 1 ], (0, π)
(i) ∠CAB = a = (c) [ −1, 1 ], [0, π] (d) ( −1, 1 ) ,  − π , π 
 2 2 
 
(a) sin–1  2 
 3
(b) sin–1 ()
1
2  1
Ans. (i) (b) sin–1  
 3
(ii) (c) cos–1  
   2  2
(c) sin–1(2) (d) sin–1  3 
 2  π
(iii) (d) tan–1 ( 3 ) (iv) (c)
(ii) ∠CAB = a = 2
(v) (c) [ −1, 1 ], [0, π]
(a) cos–1 1
5 () (b) cos–1 ()
2
5 Case Study–II
2. The Government of India is planning to fix a hoarding
 3
(c) cos–1  
 2 
(d) cos–1 ()
4
5
board at the face of a building on the road of a busy market
for awareness on COVID-19 protocol. Ram, Robert and

Inverse Trigonometric Functions 41


E:\AMIT_WORKS\Exam_Guru\EG_Mathematics-12_(working_02-06-2022)\EG_Mathematics-12_working\Open_Files\Chap_2\Chap_2
\ 16-Aug-2022 Amit Proof-4 Reader’s Sign _______________________ Date __________

Rahim are the three engineers who are working on this


project. “A” is considered to be a person viewing the
(c) tan−1 ()
4
3
(d) tan−1 (4)
hoarding board 20 metres away from the building, standing
at the edge of a pathway nearby. Ram, Robert and Rahim (iii) Measure of ∠EAB =
suggested to the firm to place the hoarding board at three (a) tan−1 (11) (b) tan−1 3
different locations namely C, D and E. “C” is at the height
of 10 metres from the ground level. For the viewer A, the (c) tan−1 ( )
2
11
11
(d) tan−1 2 ( )
angle of elevation of “D” is double the angle of elevation
of “C”. The angle of elevation of “E” is triple the angle (iv) A' is another viewer standing on the same line of
of elevation of “C” for the same viewer. Look at the observation across the road. If the width of the road is
figure given and based on the above information answer 5 metres, then the difference between ∠CAB and ∠CA′B is
the following:
E (a) tan–1 ()
1
2
(b) tan−1 1
8 ()
D (c) tan−1 ()
2
5
(d) tan−1
11
21 ( )
C
(v) Domain and Range of tan−1x =

10 m
(a) R+, − π , π
2 2 ( ) (
(b) R–, − π , π
2 2 )
A 5m A 20 m
B (c) R, − π , π
2 2 ( ) ( )
(d) R, 0,
π
2
(i) Measure of ∠CAB =

(a) tan−1 (2) (b) tan−1


1
2 () Ans. (i) (b) tan–1 1
2 () (ii) (c) tan–1 4
3 ()
(c) tan−1 (1)
(ii) Measure of ∠DAB =
−1
(d) tan (3) (iii) (d) tan–1 11
2 ( ) (iv) (b) tan–1 1
8 ()
(a) tan−1 4
3
() (b) tan−1 (3)
(v) (c) R, − π , π
2 2 ( )
Author’s Comments
Questions based on following types are very important for Exams. So, students are advised to revise them thoroughly.
1. To find principal value of a given trigonometric function.
2. To convert one inverse trigonometric function into another equal trigonometric function.

IMPORTANT FORMULAE
The Domain and Range (Principal Value Branches) of Inverse Trigonometric Functions
Inverse Trigonometric Functions Domains Range (Principal Value Branches)

 −π , π 
y = sin–1 x [–1, 1]  2 2 

y = cos–1 x [–1, 1] [0, p]

 −π , π  − {0}
y = cosec–1 x R – (–1, 1)  2 2 

y = sec–1 x R – (–1, 1) {}
[0,π ] − π
2

y = tan–1 x R ( )
−π , π
2 2

y = cot–1 x R (0, p)

42 Mathematics–12
E:\AMIT_WORKS\Exam_Guru\EG_Mathematics-12_(working_02-06-2022)\EG_Mathematics-12_working\Open_Files\Chap_2\Chap_2
\ 16-Aug-2022 Amit Proof-4 Reader’s Sign _______________________ Date __________

COMMON ERROR
ERROR CORRECTION
(i) Students generally do not remember the principal value (i) Write down the principal value branch of the respective
branches of different inverse trigonometric functions and trigonometric function.
commit mistakes.

REVISION CHART

Inverse of a Functions
If f : X → Y such that f(x) : y, x ∈ X and y ∈ Y, is one-one and onto, then we can define a unique function g : Y → X such that
g(y) = x where x ∈ X and y ∈ Y.

Domain and Principal Value Branch (Range)of


All Inverse Trigonometric Functions

Function Domain Range

 −π π 
sin–1x [–1, 1]  2 , 2
 

cos–1x [–1, 1] [0, p]

 −π π 
cosec–1x R – (–1, 1)  2 , 2  − {0}
 

π
sec–1x R – (–1, 1) [0, π ] −  2 
 

 −π π 
tan–1x R  , 
2 2

cot–1x R (0, p)

Inverse Trigonometric Functions 43


E:\AMIT_WORKS\Exam_Guru\EG_Mathematics-12_(working_02-06-2022)\EG_Mathematics-12_working\Open_Files\Chap_3\Chap_3
\ 19-Aug-2022 Amit Proof-4 Reader’s Sign _______________________ Date __________

3 Matrices
Topics Covered
3.1. Matrix 3.2. Operations on Matrices
l Types of Matrices l Transpose of a Matrix

l Equality of Matrices l Symmetric and Skew Symmetric Matrix

l Multiplication of a Matrix by a Scalar Number

l Addition of Matrices

l Subtraction of Matrices

l Multiplication of a Matrix with Another Matrix

l Inverse of a Square Matrix

C hapter map
MATRIX

Types of Matrices

Row Column Zero Identity Scalar Diagonal Inverse of a


Matrix Matrix Matrix Matrix Matrix Matrix Square Matrix

Order of a 3×3 Order Square


Matrix General Matrix Rectangular Invertible
Matrix Matrix Matrix

Symmetric and Skew


Symmetric Matrix

Operations on Matrices

Addition of Equality of Multiplication


Matrices Matrices of a matrix

Elements of Subtraction Transpose of


a Matrix of Matrices a Matrix

Topic 1. Matrix
Arrangement of numbers in horizontal and vertical lines is called Elements or Members of the Matrix
a matrix. The numbers used to form a matrix are called its elements or
7 −8 5  members. These numbers are enclosed in ( ) or [ ] braces.
 4
 1 3  1 3 0     3 0 2
For example,   ,  −5 2 4 ,  2 0 1 are matrices. For example: 
 4 0      is a matrix. Here 3, 0, 2, –4, 5
0 3 2  −4 5 7 
  and 7 are its elements or members.

44
E:\AMIT_WORKS\Exam_Guru\EG_Mathematics-12_(working_02-06-2022)\EG_Mathematics-12_working\Open_Files\Chap_3\Chap_3
\ 16-Aug-2022 Amit Proof-4 Reader’s Sign _______________________ Date __________

A matrix is usually denoted by a capital letter, A, B or C, e.g., \ Matrix A can have any one of the possible orders: 1 × 6,
2 3 7 2 × 3, 3 × 2 or 6 ×1.
 3 1 4   Example 1. A matrix has 12 elements. Number of possible orders
A=  ; B = 1 5 8
0 5 7  having it is
 4 6 9 
(a) 5 (b) 4 (c) 6 (d) 7
Rows of a Matrix Solution. A matrix has 12 elements. Number 12 can be factorised
In a matrix, horizontal lines of numbers are called rows. In a as:
 2 1 3 12 =1 × 12 = 2 × 6 = 3 × 4 = 4 × 3 = 6 × 2 = 12 × 1
matrix A =  4 0 −5 , there are three rows and these are: \The number of possible orders of the matrix is 6, i.e.: 1 ×
 
 6 7 8 12, 2 × 6, 3 × 4, 4 × 3, 6 × 2, 12 × 1.
Hence, option (c) is the correct answer.
 2 1 3 ← First row
 4 0 −5 ← Second row 3 0 2 
 
 6 7 8 ← Third row Example 2. If a matrix A =  1 −5 7  , then the order of the
 
matrix A is 8 4 9
Columns of a Matrix
In a matrix, vertical lines of numbers are called columns. In a (a) 2 × 3 (b) 3 × 3 (c) 3 × 2 (d) 2 × 2
 3 1 −1  3 0 2 
 
matrix B = 0 5 2 , there are three columns and the three Solution. The given matrix: A =  1 −5 7 
 
 1 4 7  8 4 9
columns are: Q Since, there are 3 rows and 3 columns in the above given
3 1 −1 matrix, therefore, its order is 3 × 3.
0 5 2 Hence, option (b) is the correct answer.
1 4 7 Example 3. The values of elements: a23­, a31, a22, and a13, in the
matrix given in Example 2, are
First Second Third
column column column (a) 7, – 5, 8, 2 (b) 7, 2, – 8, 5
(c) 7, 8, –5, 2 (d) 5, 7, –2, 8
Order of a matrix Solution. a23 = 7, a31 = 8, a22 = –5 and a13 = 2.
If a matrix has m-rows and n-columns, then its order is m × n. (It Hence, option (c) is the correct answer.
is read as m by n).
(A) Types of Matrices
In m × n, first number m indicates number of rows, and second
Following are different types of matrices:
number n indicates number of columns in the given matrix
7 1 −1 Row matrix
Take a matrix: A = 
 4 5 0 A matrix having only one row and any number of columns is
 
called a row matrix. For example: [3]1×1, [4 –7]1×2, [5 0 –7]1×3
There are 2 rows and 3 columns, hence its order is 2 × 3 and
are row matrices.
this matrix has 2 × 3 = 6 elements.
3 × 3 Order General Matrix Column Matrix
A matrix having only one column and any number of rows is
 a11 a12 a13 
called a column matrix.
A 3 × 3 order general matrix can be written as: A =  a21 a22 a23 
 a31 a32 a33   1
3  
3× 3
For example: [ 7 ]1×1 ,   ,  0 are column matrices.
This general representation of a 3 × 3 matrix is very important 5
  2 ×1  
because the number 11, 12, 13, etc. written with elements of the  −7  3×1
matrix tell us in which row and column that particular element Zero or Null Matrix
is situated.
A matrix of any order having its all elements zero, is called zero
For example, a23 means, this element is situated in second
matrix or null matrix.
row and third column of the matrix.
If you are given number of elements in a matrix, you can work  0  0 0 0  0 0
Forexample:[0]1×1,[0 0]1×2,   ,   , 
out all possible orders of the given matrix. Suppose a matrix A 0 2 ×1 0 0 0 2 × 3 0 0 2 × 2
has 6 elements, then 6 = 1 × 6 = 2 × 3 = 3 × 2 = 6 × 1. etc. are null or zero matrices.

Matrices 45
E:\AMIT_WORKS\Exam_Guru\EG_Mathematics-12_(working_02-06-2022)\EG_Mathematics-12_working\Open_Files\Chap_3\Chap_3
\ 16-Aug-2022 Amit Proof-4 Reader’s Sign _______________________ Date __________

Square Matrix Or
A matrix in which number of rows is equal to number of columns A square matrix A = [aij]n×n is a scalar matrix if:
is called a square matrix.
α when i = j
aij = 
 1 3 2  0 when i ≠ j
 3 0
For example, [ −7 ]1×1 ,   and  2 0 5 are
 −5 2 2 × 2  4 0 0
 −5 4 7  3× 3 7 0  0 4 0
For example, [ ]1×1 
5 ,  , are scalar
square matrices.  0 7  2 × 2  

 0 0 4  3× 3
Rectangular Matrix matrices.
A matrix in which number of rows is not equal to number of Note: (i) A scalar matrix is also a diagonal matrix.
columns is called a rectangular matrix.
(ii) A scalar matrix is a times of an identity matrix of same order.
 3 5
1 1 0 Construction of a Matrix when its General Term
For example, [1 2 3]1× 3 ,   and  1 0 are
 2 3 4  2×3 and Order are Given
 4 3 3× 2
To construct a matrix with given general element and order, first
rectangular matrices.
write the general matrix of the required order and then fill in the
Diagonal of a Square Matrix values of the elements with the help of given general element.
If A = [aij]n×n is a square matrix of order n × n then the elements Example 4. A matrix of order 3 × 3 whose general element is aij
a11, a22, a33 ..., an×n of this matrix form the main diagonal or just = 3i + 4j, is given by
diagonal of the square matrix A.
 7 5 11  21 5 11
 
 a11 a12 a13  (a) 14 10 18 (b) 10 14 18
 
Thus: A =  a21 a22 a23 
 is a square matrix of order 17 13 21  3 17 7 
 a31 a32 a33 
3× 3
 7 11 15  7 5 11
3 × 3. Here a11, a22 and a33 elements of the matrix A form the
(c) 10 14 18 (d) 14 18 10
main diagonal of the square matrix.    
13 17 21 13 21 17 
Diagonal Matrix
A square matrix A = [aij]n×n is called diagonal matrix if its all aij Solution. General element of the required matrix of order 3 × 3
= 0 for i ≠ j. is aij = 3i + 4j.
Or To construct it, take a 3 × 3 general matrix:
A square matrix whose all elements except diagonal elements are  a11 a12 a13 
zero is called a diagonal matrix. A =  a21 a22 a23 

 4 0 0  a31 a32 a33 
 3 0  0 3 0 3× 3
For example, [ ]1×1 0 5
7 , and   are Now aij = 3i + 4j
  2× 2  0 0 7  3×3
(3 × 1 + 4 × 1) (3 × 1 + 4 × 2) (3 × 1 + 4 × 3) 
diagonal matrices.
A = (3 × 2 + 4 × 1) (3 × 2 + 4 × 2) (3 × 2 + 4 × 3)
Identity Matrix or Unit matrix (3 × 3 + 4 × 1) (3 × 3 + 4 × 2) (3 × 3 + 4 × 3) 3×3
A square matrix in which each main diagonal element is 1 and all
other elements are zero, is called an identity matrix or unit matrix.  3 + 4 3 + 8 3 + 12   7 11 15
Or ⇒ A = 6 + 4 6 + 8 6 + 12 ⇒ A = 10 14 18
The square matrix A = [aij]n×n is called an identity or unit matrix  
9 + 4 9 + 8 9 + 12 3× 3 13 17 21 3× 3
1 when i = j
if: aij =  Hence, option (c) is the correct answer.
0 when i ≠ j
Note: An identity matrix of order n × n is denoted as In×n, e.g., 5i + 3j if i< j

Example 5. A matrix: A = [aij]3×3 when aij =  7 if i= j
1 0
I2× 2 =  . is given by 4i − 3j if i> j

0 1
Scalar Matrix  7 5 8 11 7 14
 
A square matrix whose all non-diagonal elements are zero and all (a) 11 14 9 (b)  9 6 7 
 
diagonal elements are equal is called scalar matrix.  6 7 5  7 5 6

46 Mathematics–12
E:\AMIT_WORKS\Exam_Guru\EG_Mathematics-12_(working_02-06-2022)\EG_Mathematics-12_working\Open_Files\Chap_3\Chap_3
\ 16-Aug-2022 Amit Proof-4 Reader’s Sign _______________________ Date __________

7 11 14 7 11 14 a matrix whose (ij)th element is k(aij) for all i and j. Thus, kA =
    k[aij]m×n = [kaij]m×n.
(c) 7 5 19 (d)  5 7 19
OR
 9 6 7   9 6 7 
When we multiply a scalar number with a matrix of an order
Solution. General element of the required matrix of order 3 × 3 m × n, then every element of the matrix get multiplied by that
5i + 3j if i < j scalar number.
 3 0 5
is aij =  7 if i = j  
4i − 3j if i > j F o r e x a m p l e : i f m a t r i x A =  1 −1 −2 then

 2 4 8 3× 3
To construct a matrix A, take a 3 × 3 general matrix:
 a11 a12 a13  3 0 5 15 0 25
a  5A = 5  1 −1 −2 =  5 −5 −10
  
 21 a22 a23 
 a31 a32 a33   2 4 8 3× 3 10 20 40 3×33
3× 3
In a similar way if we want to take a number common from
 7 (5 × 1 + 3 × 2) (5 × 1 + 3 × 3)  a matrix, it is taken common from all the elements of the matrix.

⇒ A = ( 4 × 2 − 3 × 1) 7 (5 × 2 + 3 × 3)
 3 −6 0  1 −2 0
( 4 × 3 − 3 × 1) ( 4 × 3 − 3 × 2) 7  Matrix A =   = 3 
9 −15 −18 2 × 3 3 −5 −6 2 × 3
3× 3

7 11 14
⇒ A =  5 7 19 (D) Addition of Matrices
9 6 7  3× 3 If A = [aij]m×n and B = [bij]m×n are two matrices of same order
m × n, then their sum (A + B) is the matrix of the same order m
Hence, option (d) is the correct answer. × n and it is obtained by adding corresponding elements of the
matrix A and matrix B.
(B) Equality of Matrices
A + B = [aij]m×n + [bij]m×n ⇒ A + B = [aij + bij]m×n.
Two matrices A = [aij]m×n and B = [bij]m×n are called equal matrices
if: (i) they are of same order and (ii) each element of matrix A Note: The above definition of addition of matrices shows that only
is equal to corresponding element of the matrix B, i.e., aij = bij. same order matrices can be added and the order of the resultant
matrix also remains the same.
a b c 4 3 −1
For example: If  = , then a = 3 0 5 
d e f  2 × 3  2 7 − 4 2 × 3 Example 7. If two matrices A and B are such that: A =  
1 2 5 2×3
4, b = 3, c = –1, d = 2, e = 7 and f = – 4. 1 1 2 
and B =   , then 3A + 2B is given by
 x− y z 3 5 
3 2 5  2×3
Example 6. If  =  , the values of x, y, z
 2x + 3y w 11 −7 
19 11 2  9 11 19
and w are given by (a)   (b)  
(a) x = –2, y = 1, z = 5, w = 3 10 25 9  2 25 10
(b) x = 3, y = 5, z = –7, w = 7  9 2 19 11 2 19
(c)   (d)  
(c) x = 7, y = –7, z = 3, w = 2 11 25 10  9 10 25
(d) x = 4, y = 1, z = 5, w = –7
3 0 5 
 x− y z 3 5  Solution. We are given two matrices A and B as: A =  
Solution. We are given  = . 1 2 5 2×3
 2x + 3y w 11 −7 
1 1 2 
Since two matrices of equal orders 2 × 2 are given equal and B =  
so their corresponding elements will also be equal. 3 2 5  2×3
 x− y =3  z=5  3 0 5 1 1 2
\ and  . ⇒ 3A + 2B = 3   + 2 
2x + 3y = 11  w = −7 1 2 5 2 × 3 3 2 5  2 × 3
On solving these simultaneous equations for x, y, z and w, we 9 0 15 2 2 4
get x = 4, y = 1, z = 5 and w = –7. =   + 
3 6 15 2×3  6 4 10  2×3
Hence, option (d) is the correct answer.
11 2 19 
(C) Multiplication of a Matrix by a Scalar Number =  
 9 10 25 2×3
If A = [aij]m×n is a matrix and k is any scalar number, then kA is

Matrices 47
E:\AMIT_WORKS\Exam_Guru\EG_Mathematics-12_(working_02-06-2022)\EG_Mathematics-12_working\Open_Files\Chap_3\Chap_3
\ 16-Aug-2022 Amit Proof-4 Reader’s Sign _______________________ Date __________

11 2 19  20 5 10   3 −15 12 


⇒ 3A + 2B =   =   − 12 9 −12  2×3
 9 10 25 2 × 3  5 0 25  2×3 
Hence, option (d) is the correct answer.  17 20 −2 
=  
Properties of Addition of Matrices  −7 −9 37  2×3
Following are the properties of addition of matrices.  17 20 −2
1. Addition of matrices is commutative: Let A and B be ⇒ (5A – 3B) =  
 −7 −9 37  2 × 3
two matrices of same order then matrix addition is commutative
means: (A + B) = (B + A). Hence, option (c) is the correct answer.
2. Addition of matrices is associative: Take three matrices Example 9. A matrix X 2×3 if 5A – 7B + 3X = 0, where
A, B and C of same order then matrix addition is associative  2 1 3 0 9 1
means: (A + B) + C = A + (B + C). A=  and B =   is given by
 1 4 7  5 3 5
3. Existence of additive identity of a matrix: Let A and B
be two matrices of same order m × n, then the matrix B will be  30 5 1  30 1 0
(a)   (b)  
called additive identity of matrix A if (A + B) or (B + A) is equal  −10 −58 5
 −10 5 8
to matrix A. This is possible only when matrix B is a zero matrix
of same order as matrix A. Hence, every matrix A has its additive 1  −10 58 −8 1  10 18 8
(c) (d)
identity matrix and it is equal to zero matrix of the same order. 3  30 1 0 2  −2 6 5
4. Existence of additive inverse of a matrix: If sum of two
matrices of same order is zero matrix, then each matrix is called 2 1 3 0 9 1
Solution. It is given that A =  ,B =   and
additive inverse of the other matrix. 5A – 7B + 3X = 0 1 4 7   5 3 5
Let A and B be two matrices of same order and A + B = B + A ⇒ 3X = 7B – 5A
= 0. Then matrix A is additive inverse of matrix B and matrix B is 0 9 1 2 1 3
additive inverse of matrix A. If A + B = 0 then B = –A and A = –B. ⇒ 3X = 7   −5
 5 3 5 1 4 7 
\ For any matrix A, matrix (–A) is its additive inverse.
 0 63 7  10 5 15 
(E) Subtraction of Matrices ⇒ 3X =  − 
35 21 35  5 20 35
If A = [aij]m×n and B = [bij]m×n be two matrices of same order m
 −10 58 −8
× n, then their difference (A – B) is a matrix of same order m × ⇒ 3X =  
n and it is obtained by subtracting elements of matrix B from the  30 1 0
corresponding elements of matrix A. 1  −10 58 −8
⇒ X =
⇒ (A – B) = [aij]m×n – [bij]m×n ⇒ (A – B) = [aij – bij]m×n 3  30 1 0
The above definition suggests that a matrix B of order m × n can Hence, option (c) is the correct answer.
be subtracted from a same order matrix A and the resultant matrix  x2   x   −2
Example 10. Values of x and y when:  2  − 3   =   are
(A – B) also has same order m×n. y   2y   −9
given by
4 1 2 (a) x = 1, 2; y = 3 (b) x = 2, 3; y = –3
Example 8. If matrix A =   and matrix B =
 1 0 5 2×3 (c) x = –1, 2; y = 3 (d) x = 1, –2; y = 2
 1 −5 4  x2   x   −2
4 , then value of (5A – 3B) will be Solution. Given equation is:  2  − 3   =  
 3 − 4  2×3 y   2y   −9
7 20 21  3 7 21  x 2   3x   −2
(a)   (b)   ⇒  2 −   =  
 3 7 −9  20 3 −6  y   6y   −9
 17 20 −2  20 17 −2  x 2 − 3x   −2   x 2 − 3x = −2
(c)   (d)   ⇒  2  =   ⇒  2
 −7 −9 37   9 7 37   y − 6y   −9   y − 6y = −9
4 1 2  1 −5 4  x 2 − 3x + 2 = 0 ( x − 2) ( x − 1) = 0
Solution. We are given A =   and B = 4 ⇒  2 ⇒ 
3 − 4  2×3  ( y − 3) = 0
2
 1 0 5 2×3   y − 6y + 9 = 0

 4 1 2  1 −5 4  x = 1, 2
⇒ (5A – 3B) = 5  − 3 ⇒ 

 1 0 5 2 × 3 4 3 − 4 2 × 3 y = 3
Hence, option (a) is the correct answer.

48 Mathematics–12
E:\AMIT_WORKS\Exam_Guru\EG_Mathematics-12_(working_02-06-2022)\EG_Mathematics-12_working\Open_Files\Chap_3\Chap_3
\ 16-Aug-2022 Amit Proof-4 Reader’s Sign _______________________ Date __________

(F) Multiplication of a Matrix with Another Matrix Sum of the Sum of the 
The product AB of two matrices A and B is defined only if number  products of the product of the 
of columns of first matrix A is equal to number of rows of second corresponding corresponding 
 
matrix B. elements of first elements of first 
If matrix A = [aij]m×n has order m × n and another matrix B  row of matrix A row of matrrix A 
= [bjk]n×p has order n × p, then the product AB is defined because and first column and second column 
of matrix B. of the matrix B. 
number of columns of matrix A is n and number of rows of matrix
AB =  
B is also n. Sum of the Sum of the 
The product AB is defined as: AB = [aij]m×n × [bjk]n×p = [cik]  productts of the products of 
corresponding corrresponding 
m×p . elements of second 
To get the element cik of the matrix AB, we take ith row 
elements of second 
 row of matrix row of matrix A 
of matrix A and kth column of the matrix B and multiply them
 A and first column and second coolumn 
elementwise and take the sum of all the product. of matrix B.  2×2
of matrix B
In matrix A = [aij]m×n, ith row is [ai1 ai2 ai3 ... ain] and
in matrix B = [bik]n×p, the kth column is:
⇒ A B =
 b1k 
b   ( 2 × 1) + (1 × −1) + (3 × 0) ( 2 × 2) + (1 × 4) + (3 × 5) 

 2k 
b3k  ( −1 × 1) + ( 4 × −1) + ( 0 × 0) ( −1 × 2) + ( 4 × 4) + (0 × 5) 2× 2
   2 −1+ 0 4 + 4 + 15 
   ⇒ AB =  
bnk   −1 − 4 + 0 −2 + 16 + 0 2 × 2

Then (ik)th element of matrix AB = (ik)th element of matrix C  1 23
⇒ AB =  
cik = ai1b1k + bi2b2k + ... ainbnk ⇒ cik = (aijbjk), j = 1, 2, ....n  −5 14  2 × 2

 2 1 3 Hence, option (a) is the correct answer.


Example 11. If matrix A =   and matrix B =
 −1 4 0  2×3 Properties of Matrix Multiplication
 1 2 1. Matrix multiplication is not commutative:
 −1 4  , then AB will be given by (a) Let A and B be two matrices such that AB is defined,
 
 0 5 3×2 then it may be possible that BA is not defined. For
example, if matrix A has order 3 × 3 and matrix B has
 1 23  23 −5 order 3 × 2, then the product AB is defined but product
(a)   (b)   BA is not defined. Hence, equality AB and BA does
 −5 14  1 14
not exists.
14 5  23 14 (b) Let A and B be two matrices such that both products
(c)   (d)   AB and BA are defined. Now it may be quite possible
 −1 23  11 −1
that the order of the matrix AB is not equal to order of
 2 1 3 matrix BA. For example, if order of matrix A is 3 × 2
Solution. We are given two matrices: A =  and
 −1 4 0  2×3 and order of matrix B is 2 × 3. Then both the product
 1 2 AB and BA are defined but the order of AB is 3 × 3
B =  −1 4  and order of BA is 2 × 2. Hence AB ≠ BA.
 0 5 3×2 (c) Even if the orders of the matrix (AB) and (BA) are
equal, the two products are generally not equal.
Here matrix A has 3 columns and matrix B has 3 rows, hence
(d) The meaning of the above discussion is not that the
the product AB is defined. The order of the product matrix AB matrix AB can never be equal to matrix BA. In some
will be 2 × 2. cases we have AB = BA.
 1 2 2. Matrix multiplication is associative: If A, B and C be
 2 1 3  −1 4
AB =   × three matrices of such orders that the products, AB, (AB)
 −1 4 0  2 × 3  
C, (BC) and A(BC) are defined, then matrix multiplication
 0 5 3× 2
is associative, means:
(AB)C = A(BC).
3. Matrix multiplication is distributive over addition
and subtraction: If A, B and C have three matrices of
such orders that (B + C), (B – C), AB, AC, BA and CA

Matrices 49
E:\AMIT_WORKS\Exam_Guru\EG_Mathematics-12_(working_02-06-2022)\EG_Mathematics-12_working\Open_Files\Chap_3\Chap_3
\ 22-Aug-2022 Amit Proof-4 Reader’s Sign _______________________ Date __________

are defined, then matrix multiplication is distributive over 9 8 17 −5


addition and subtraction means: (c)   (d)  
 3 −4  11 −9
(i) A(B + C) = AB + AC (ii) (B + C)A = BA + CA
(iii) A(B – C) = AB – AC (iv) (B – C)A = BA – CA  4 5  8 3 
4. Existence of multiplicative identity: For every square Solution. It is given that: A  =  ...(1)
 2 3 10 4
matrix A, there exists an identity matrix I of same order,
such that: AI = IA a b 
Take A =   and put it in (1), we get
1 0 0 c d 
1 0  
I1×1 = [1] , I 2 × 2 =  , =
 3× 3  0 1 0 
I  a b   4 5  8 3
 0 1 
0 0 1   c d   2 3 = 10 4
    
Matrix Polynomial  4a + 2b 5a + 3b   8 3
3 2
If f (x) = ax + bx + cx + d is an algebraic polynomial and A is
⇒  4c + 2d 5c + 3d  = 10 4
   
a square matrix, then:
4a + 2b = 8 4c + 2d = 10
f (A) = aA3 + bA2 + cA + dI is called matrix polynomial. ⇒  and 
5 a + 3 b = 3 5c + 3d = 4
Here unit matrix I and square matrix A are of equal orders.
3 1 On solving for a and b, we get a = 9, b = – 14
2
Example 12. If matrix A =   and f (x) = 3x – 4x + 7, then On solving for c and d, we get c = 11, d = –17
 2 4 
f (x) is equal to
 a b   9 −14 
 11 17   28 17 
\ A=  = 
(a)   c d  11 −17 
 (b)  
 23 24 34 45 Hence, option (a) is the correct answer.
14 17  11 23
(c)   (d)   (G) Inverse of a Square Matrix
13 12 12 17  If A and B are two non-zero square matrices of same order such
that AB = BA = I, then matrix A is called inverse of matrix B
3 1 2
Solution. It is given that A =   and f (x) = 3x – 4x + 7. and matrix B is called inverse of matrix A. Inverse of matrix A is
 2 4 denoted as A–1. Here A and A–1 are inverse of each other.
3 1 3 1 3 1 \AA–1 = A–1 A = I
\ f (A) = 3A2 – 4A + 7I = 3   2 4 – 4 2 4 Also AI = IA = A and similarly A–1I = IA–1 = A–1
 2 4    
1 0  Invertible Matrix
+7   If a matrix A has inverse, it is called invertible matrix.
0 1 
 2 3  2 −3
9 + 2 3 + 4  3 1 1 0 Example 14. If matrix A =   and matrix B =   , then
= 3  − 4  +7   1 2   −1 2
 6 + 8 2 + 16  2 4 0 1
(a) AB ≠ BA
11 7  3 1 1 0 (b) matrices A and B are inverse of each other
= 3
14 18  − 4  2 4 + 7  0 1 
      (c) AB = BA = O (d) None of these [V. Imp.]
 33 21 12 4  7 0   2 3  2 −3
=  − +  Solution. It is given that A =   , B=  
 42 54   8 16   0 7  1 2  −1 2
33 − 12 + 7 21 − 4 + 0   28 17   2 3  2 −3  4 − 3 −6 + 6 
=  =  \ AB =   = 
 42 − 8 + 0 54 − 16 + 7  34 45  1 2   −1 2   2 − 2 −3 + 4 

2  28 17   1 0
\ f (A) = 3A − 4A + 7I =   =  =I ...(1)
34 45 0 1
Hence, option (b) is the correct answer.  2 −3  2 3  4 − 3 6 − 6 
Also BA =   = 
 4 5  8 3   −1 2   1 2   − 2 + 2 −3 + 4 
Example 13. A matrix A2×3, when A  =  , is
 2 3 10 4 1 0 
 9 11
=  =I ...(2)
 9 −14 0 1 
(a)   (b)  
11 −17  7 −3 From (1) and (2), we get AB = BA = I.

50 Mathematics–12
E:\AMIT_WORKS\Exam_Guru\EG_Mathematics-12_(working_02-06-2022)\EG_Mathematics-12_working\Open_Files\Chap_3\Chap_3
\ 16-Aug-2022 Amit Proof-4 Reader’s Sign _______________________ Date __________

\ Matrices A and B are inverse of each other.


1  3 −1
⇒ A–1 =
Hence, option (b) is the correct answer. 13  −2 5
 5 1
Example 15. If matrix A =  , and A2 – 8A + 13I = 0, then Hence, option (c) is the correct answer.
–1  2 3
value of A will be  5 1 2
Example 16. If matrix A =   , and A – 8A + 13I = 0, then
1 5 −1 1  3 5  2 3
(a) (b)
13 3 7  13  −1 −2 value of A3 will be

1  3 −1 1  −1 5  51 100 151 51


(c) (d) (a)   (b) 
13  −2 5 13  7 3  49 102

102 49

 5 1 102 49  49 50


Solution. It is given that A =  (c)   (d)  
  51 100 150 51
 2 3
and A2 – 8A + 13I = 0 ...(1)  5 1
Solution. It is given that A =  
–1 –1
To find A , multiply by A on both sides of equation (1), we get  2 3
A–1[A2 – 8A + 13I] = A–1 × 0 Take: A2 – 8A + 13I = 0
⇒ A–1A2 – 8A–1A + 13A–1I = 0 ⇒ A2 = 8A – 13I ...(1)
3
To find A , multiply by A on both sides of eqn. (1), we get
⇒ (A–1A)A – 8(A–1A) + 13(A–1) = 0
⇒ A3 = 8A2 – 13A
⇒ IA – 8I + 13A–1 = 0
⇒ A3 = 8[8A – 13I] – 13A
[Q A–1A = I and A–1I = A–1]
[Using (1)]
⇒ A – 8I + 13A–1 = 0 ⇒ 3
A = 64A – 104I – 13A
[Q IA = A] ⇒ A3 = 51A – 104I
⇒ 13A–1 = 8I – A  5 1 1 0
⇒ A3 = 51   − 104  
 1 0   5 1  2 3 0 1
⇒ 13A–1 = 8 − 
0 1   2 3  255 51  104 0 
 8 0   5 1
⇒ A3=  − 
13A–1 102 153  0 104
⇒ =  − 
0 8   2 3 151 51
A3 = 
⇒ A3 = 
102 49
 3 −1
⇒ 13A–1 =  
 −2 5 Hence, option (b) is the correct answer.

EXERCISE 3.1
I. Multiple Choice Questions (MCQs) 1  5 7 1  2 −3
(a) − (b) −
Choose the correct answer from the given options. 11  −3 2 11  −5 2

15 x + y   15 8 1  2 5 1  1 5
1. If  = , then the value of x will be (c) − (d)
2 y   x − y 3 11  −3 2 11 7 8

(a) 6 (b) 7  1 −1


4. If matrix A =   and A2 = kA, then the value of k
(c) 5 (d) 8 is equal to  −1 1
1 2  3 1  7 11 (a) 4 (b) 2
2. If   =  , then the value of x will be (c) 5 (d) 6
3 4  2 5  x 23
5. The value of (x – y + z) from the following equation:
(a) 17 (b) 18  x + y + z  9
 x + z  =  5 is given by
(c) 19 (d) 5    
 y + z  7 
 2 3
3. If A =   and AB = I, then matrix B is equal to (a) 4 (b) 1
 5 2 (c) 2 (d) 3

Matrices 51
E:\AMIT_WORKS\Exam_Guru\EG_Mathematics-12_(working_02-06-2022)\EG_Mathematics-12_working\Open_Files\Chap_3\Chap_3
\ 16-Aug-2022 Amit Proof-4 Reader’s Sign _______________________ Date __________

0 3  0 4a  α β  2 2
6. If A = 
2 −5  and kA =  −8 5b  , then value of k will be 9. If A = 
γ −α  is such that A = I, then show that 1 – a
     
(a) 4 (b) –4 – bg = 0.
(c) 5 (d) –5 cos θ − sin θ 
10. If f (θ ) =  , prove that
 −1 0 −1  1  sin θ cos θ 
7. If [ 2 1 3]  −1 1 0  0 = A , then order of the cos (θ + φ ) − sin (θ + φ )
   f (θ ) f ( φ ) = 
 = f (θ + φ ) .
 0 0 1  −1  sin (θ + φ ) cos (θ + φ )
matrix A will be
 cos θ sin θ 
(a) 1 × 1 (b) 2 × 1 11. If f (θ ) =   , show that f (q). f (–q) = I.
 − sin θ cos θ 
(c) 1 × 2 (d) 2 × 2
 2x + y + z   8 
 2 3
 1 −2 3   12. Solve for x, y and z if:  3y + 2z  = 10
8. If A =     
−4 2 5  and B =  4 5 and BA = [bij], then  5z  10
   2 1
b21 + b32 equals III. Short Answer Type Questions-II
(a) 18 (b) 17 1 2 2
(c) –18 (d) –17 1. If A =  2 1 2 , verify: A2 – 4A – 5I = 0.
 
II. Short Answer Type Questions-I  2 2 1 

 1 0  2 0 1
2. If A =  2 1 3 , find A2 – 5A + 4I and hence find the
2
1. If A =   , find k if A – 8A + kI = 0.
 −1 7  
 1 −1 0
1 2 3  1 matrix X such that A2 – 5A + 4I + X = 0. [Delhi 2015]
  
2. If [1 x 1]  4 5 6  −2 = 0 , find the value of x.
 1 2  x 
 3 2 5  3 3. If [ 2x 3]     = 0 , find x.[Delhi 2015]
 −3 0  3 
3 2  2 4. Construct a matrix:
3. If A =   , find the values of a and b such that A +
1 1   2i − 3j 
(i) A =  aij  , when aij = 
aA + bI = 0. 3× 3  2i + 3j 
4. If x and y are matrices of order 2 × 3, solve for x and y.
 (ii) A =  aij  , when aij = 3i + 2 j
 1 2 3 3× 3
x + y =  
 3 1 4  7i − 4j if i < j
 
(iii) A =  aij  , when aij =  11 if i = j
 x − y = 3 0 1 3× 3
  1 5 6 5i + 7j if i > j
   

 3i + j if i < j
 cos θ sin θ   sin θ − cos θ  
5. Simplify: cos θ   + sin θ cos θ  
(iv) A =  aij  , when aij = 2i + 3j if i = j
 − sin θ cos θ   sin θ  3× 3
 i + 3j if i > j

cos θ + sin θ 2 sin θ  3i + j if (i + j) is even
6. If A =   , then using PMI prove (v) A =  aij  , when aij =  i + j
 − 2 sin θ cos θ − sin θ  3× 3
 2 if (i + j) is odd
that:
5. If x and y each is a matrix of order 2 × 3, solve for x and y
cos n θ + sin n θ 2 sin n θ 
An =    1 − 1 2
 − 2 sin n θ cos n θ − sin n θ   5x + 2y =  
 [HOTS]  3 4 7 

7. Construct a matrix of order 3×3 when its general element 3x − 5y = 3 1 5
is aij = 3i + 2j.  5 2 3
  
 3 2 0
   3
8. If A = 1 4 0 , show that A2 – 7A + 10I = 0.
6. If A = 1  and B = [1 2 3], find (i) AB and (ii) BA.

0 0 5  
 2

52 Mathematics–12
E:\AMIT_WORKS\Exam_Guru\EG_Mathematics-12_(working_02-06-2022)\EG_Mathematics-12_working\Open_Files\Chap_3\Chap_3
\ 16-Aug-2022 Amit Proof-4 Reader’s Sign _______________________ Date __________

0 1 8. Find a matrix [A]2×2 if


7. If matrix A =   , show that:  1 −1 4  7 3 16
 0 0 (i) A ×   = 
(i) (aI + bA)2 = a2I + 2abA  2 3 1  −7 −13 −4
(ii) (aI + bA)3 = a3I + 3a2bA  2 −1  −1 −8
 1 0  A =  1 −2
 a − 1 1 1 (ii)     [AI 2017]
2 2 2
8. If A =   and B =   and (A + B) = A + B ,  −3 4   9 22
 2 − 1  b − 1
then find the values of a and b. 1 2 2
9. Find the condition for which: 9. If A =  2 1 2 , verify : A2 – 4A – 5I = 0 and hence
 
(i) (A + B) (A – B) = A2 – B2  2 2 1 
(ii) (A + B)2 = A2 + B2 find A–1.
 0 1 2  3 8
10. If A =   find the values of x and y when (xI + yA) = A. 10. If A =  2
 − 1 0  2 1 and A – mA + nI = 0, find the values of m
 
 0 α and n and hence find A–1.
− tan 
 2
11. If A =   and I is the identity matrix of 11. To promote the making of toilets for women, an
α
 tan 0  organisation tried to generate awareness through (i) house
 2 
order 2 × 2, then show that: calls (ii) letters, and (iii) announcements.
The cost for each mode per attempt is given below:
cos α − sin α 
(I + A) = (I − A)   [V. Imp.] (i) ` 50 (ii) ` 20
 sin α cos α  (iii) ` 40
IV. Long Answer Type Questions The number of attempts made in three villages X, Y and
Z are given below:
 cos θ sin θ   cos φ sin φ 
1. If A = 
− sin θ cos θ  and B =  − sin φ cos φ  then (i) (ii) (iii)
   
X 400 300 100
 cos (θ + φ) sin (θ + φ) 
show that: AB =   Y 300 250 75
 − sin (θ + φ) cos (θ + φ)  Z 500 400 150
cos θ + sin θ 2 sin θ  Find the total cost incurred by the organisation for the three
2. If f (θ ) =   , then show that: villages separately, using matrices. [A.I. 2015]
 − 2 sin θ cos θ − sin θ 
(i) [ f (q)]2 = f (2q) (ii) f (q) f (f) = f (q + f)  2 −1  5 2  2 5
12. If A = 
3 4  , B = 7 4 , C =  3 8 , find matrix D
(iii) f (q) f (–q) = I      
 cos 2 θ such that CD – AB = 0. [Delhi 2017]
sin θ cos θ 
3. If f (θ ) =   , then show that:
sin θ cos θ sin 2 θ   2 1   −3 2 1 0
13. Find the matrix A:   A = 
[ f (q)]2 = f (q).  3 2  5 −3 0 1 
 sin 2 θ sin θ cos θ  14. Three schools A, B and C organized a mela for collecting
4. If f (θ ) =   , then show that: funds for helping the rehabilitation of flood victims.
sin θ cos θ cos 2 θ 
π They sold hand made fans, mats and plates from recycled
f (q) f (f) = 0 where (q – f) is an odd multiple of . [V.Imp.]
2 material at a cost of ` 25, ` 100 and ` 50 each. The number
 1 tan θ   1 − tan θ  of articles sold are given below:
5. Show that:  − tan θ
 1 

 tan θ 1
2
 = sec θ I
 [V. Imp.]
( ) School → A B C

Articles ↓
2 1 1
 2 1  and f (x) = 2x2 – 5x + 7, find f (A). Hand Fans 40 25 35
6. If A = 1
1 1 2 Mats 50 40 50
Plates 20 30 40
7. Find a matrix [A]2×2 if
Find the funds collected by each school separately by
1 2  2 5 1 2  2 5
(i) A   =  (ii)   A=  selling the given articles. Also find the total funds collected
3 4   − 4 7  3 4   −4 7  for the purpose.  [Delhi 2015]

Matrices 53
E:\AMIT_WORKS\Exam_Guru\EG_Mathematics-12_(working_02-06-2022)\EG_Mathematics-12_working\Open_Files\Chap_3\Chap_3
\ 16-Aug-2022 Amit Proof-4 Reader’s Sign _______________________ Date __________

15. A trust fund, ` 35,000 is to be invested in two different types has 5 poor and 25 rich children and total monthly collection
of bonds. The first bond pays 8% interest per annum which is ` 26,000. Using matrix method find the monthly fees
will be given to orphanage and second bond pays 10% paid by each child of two types. [Foreign 2016]
interest per annum which will be given to an NGO (Cancer 17. A trust invested some money in two type of bonds. The first
Aid Society). Using matrix multiplication, determine how bond pays 10% interest and second bond pays 12% interest.
to divide ` 35,000 among two types of bonds if the trust The trust received ` 2,800 as interest. However, if trust had
fund obtains an annual total interest of ` 3,200. [A.I. 2015] interchanged money in bonds, they would have got ` 100 less
16. A coaching institute of English (subject) conduct classes in as interest. Using matrix method, find the amount invested
two batches I and II and fees for rich and poor children are by the trust. Interest received on this amount will be given
different. In batch I, it has 20 poor and 5 rich children and to Helpage India as donation.[A.I. (Punchkula) 2016]
total monthly collection is ` 9,000, whereas in batch II, it

Answers 3.1
I. 1. (c) 5.
 −10 2 21
15 x + y   15 8  
Hint: 
= ⇒ BA =  −16 2 37 
2 y   x − y 3  −2 −2 11

⇒ y = 3 and x + y = 8 b21 + b32 = –16 – 2 = –18
x=5
⇒ II. 1. k = 7
2. (a) 17  1 0
Hint: A=  
1 2  3 1 7 11  −1 7 
Hint:
 3 4   2 5 =  x
    23
\ A2 – 8A + kI = 0
 3 + 4 1 + 10  7 11  1 0  1 0  1 0 1 0

⇒ 9 + 8 3 + 20 =  x 23

⇒  −1 7   −1 7  − 8  −1 7  + k 0 1 = 0
         
 7 11  7 11  1+ 0 0 + 0   8 0  k 0

⇒ 17 23 =  x 23 ⇒ x = 17
⇒  −1 − 7 0 + 49 −  −8 56 +  0 k  = 0
         
 2 −3
3. (b) − 1   1 0   8 0  k 0
11  −5 2
⇒  −8 49 −  −8 56 +  0 k  = 0
     
4. (b) 2
 1 −1  1 −1  2 −2  −7 0   k 0  0 0
Hint: A2 =   = 
⇒  0 −7  +  0 k  = 0 0
 −1 1  −1 1  −2 2      
 1 −1 ⇒ –7 + k = 0
⇒ k=7
= 2 
 −1 1 1 2 3  1

⇒ k= 2 2. [1 x 1]  4 5 6  −2 = 0
5. (b) 1  3 2 5  3
x = 2, y = 4, z = 3 ⇒ x–y+z=1  1
6. (b) –4
⇒ [1 + 4x + 3 2 + 5x + 2 3 + 6x + 5]  −2 = 0
7. (a) 1 × 1  3
Hint: A1×3B3×3C3×1 = A1×3(BC)3×1 = (ABC)1×1.
 1
8. (c) –18

⇒ [ 4x + 4 5x + 4 6x + 8 ]  −2 = 0
 2 3
 1 −2 3    3
Hint: A=   and B =  4 5
 −4 2 5  2 1
⇒ [4x + 4 – 10x – 8 + 18x + 24] = 0
⇒ [12x + 20] = 0 ⇒ 12x + 20 = 0
 2 3 −20 −5
   1 −2 3 ⇒ x=
12
=
3

\ BA =  4 5  
 −4 2 5 3. a = –4, b = 1
 2 1
 2 − 12 −4 + 6 6 + 15  3 2  2
Hint: A = 
 and A + aA + bI = 0
  1 1 

⇒ BA =  4 − 20 −8 + 10 12 + 25
 2 − 4 −4 + 2 6 + 5 
\ A2 + aA + bI = 0

54 Mathematics–12
E:\AMIT_WORKS\Exam_Guru\EG_Mathematics-12_(working_02-06-2022)\EG_Mathematics-12_working\Open_Files\Chap_3\Chap_3
\ 16-Aug-2022 Amit Proof-4 Reader’s Sign _______________________ Date __________

3 2   3 2  3 2  1 0 α β 2

⇒ 1 1  1 1  + a 1 1  + b 0 1 = 0 9. Hint: A = 
γ −α  and A = I
        
9 + 2 6 + 2 3a 2a  b 0 α β α β  1 0

⇒  3 + 1 2 + 1 +  a a  + 0 b  = 0
⇒  γ −α   γ −α  = 0 1
          
11 + 3a + b 8 + 2a + 0 0 0  α 2 + βγ αβ − αβ  1 0

⇒  4+a+0 =
 3 + a + b  0 0
⇒  2
= 
 γα − γα βγ + α  0 1

⇒ 11 + 3a + b = 0, 2a + 8 = 0
α 2 + βγ 0  1 0
a+4=0 a+b+3= 0
⇒  = 
 0 βγ + α 2  0 1

⇒ a = –4, b = 1.

⇒ a2 + bg = 1
 2 1 2  −1 1 1
4. x =   and y =  
⇒ 1 – a2 – bg = 0
 2 3 5  1 −2 −1
cos θ − sin θ 
10. Hint: f (θ ) =  
1 2 3  sin θ cos θ 
Hint:
x+ y =  
3 1 4 
cos θ − sin θ  cos φ − sin φ 
3 0 1 
⇒ f (θ ) f ( φ ) =   
x− y =    sin θ cos θ   sin φ cos φ 
1 5 6
4 2 4   cos θ cos φ − sin θ sin φ − cos θ sin φ − sin θ cos φ 
=
2x =   sin θ cos φ + cos θ sin φ − sin θssin φ + cos θ cos φ 
 4 6 10 
 2 1 2 cos (θ + φ ) − sin (θ + φ )

⇒ x= =  = f (θ + φ )
 sin (θ + φ ) cos (θ + φ ) 
 2 3 5 
1 2 3 1 2 3  2 1 2  cos θ sin θ 
y=  −x= −  11. Hint: f (θ ) =  
3 1 4  3 1 4   2 3 5   − sin θ cos θ 

 −1 1 1  cos θ sin θ  cos θ − sin θ 



⇒ y=  f (θ ) f ( −θ ) =   
 1 −2 −1  − sin θ cos θ   sin θ cos θ 

1 0  cos 2 θ + sin 2 θ − sin θ cos θ + sin θ cos θ 


5.   = 
0 1 − sin θ cos θ + sin θ cos θ sin 2 θ + cos 2 θ
 
 cos θ sin θ   sin θ − cos θ  1 0
Hint: cos θ 
 + sin θ cos θ = =I
 − sin θ cos θ   sin θ  0 1 

 cos 2 θ sin θ cos θ   sin 2 θ − sin θ cos θ  12. x = 2, y = 2, z = 2.
=  + 
 − sin θ cos θ
2
cos θ  sin θ cos θ sin 2 θ  Hint: Try yourself.
1 2 2
cos 2 θ + sin 2 θ 0  1 0
= 2 2 
=  III. 1. Hint: Put A =  2 1 2 in A2 – 4A – 5I = 0 and verify it.
 0 cos θ + sin θ  0 1
 2 2 1 
6. Try yourself.
 −1 −1 −3 1 1 3
5 7 9
2. A2 – 5A + 4I =  −1 −3 −10 and X = 1 3 10 
7. A =  8 10 12
   −5 4 2 5 −4 −2
11 13 15
Hint: A = [aij]3×3 and aij = 3i + 2j Here X = –[A2 – 5A + 4I].
Try yourself.
 a11 a12 a13 
−3
A =  a21 a22 a23  3. x = 0,
2
. Try yourself.
 a31 a32 a33 
 −1 −1 −7 
3 ×1+ 2 ×1 3 ×1+ 2 × 2 3 ×1+ 2 × 3  5 7 9  5 2 11 
1 −5 
= 3 × 2 + 2 × 1 3 × 2 + 2 × 2 3 × 2 + 2 × 3 =  8 10 12 4. (i) A =  −1
 3 × 3 + 2 × 1 3 × 3 + 2 × 2 3 × 3 + 2 × 3 11 13 15 7 5 13 
 1
0 −1 
 3 5 
8. Hint: Put the values of A in A2 – 7A + 10I and now get the result.

Matrices 55
E:\AMIT_WORKS\Exam_Guru\EG_Mathematics-12_(working_02-06-2022)\EG_Mathematics-12_working\Open_Files\Chap_3\Chap_3
\ 16-Aug-2022 Amit Proof-4 Reader’s Sign _______________________ Date __________

 2i − 3j 
Hint: A = [aij]3×3 and aij = 
1 11 −3 20 = 1  −12 −8 −29
 2i + 3j  5. x = ,y
31  25 24 41 31  −16 2 6 
 a11 a12 a13  Hint: See the solution of Q.No. 4 (short answer).

   3 6 9

⇒ A =  a21 a22 a23 
 a31 a32 a33  6. AB = 1 2 3 , BA = [11]
Now put the values of i and j and get the answer.  2 4 6

 5 7 11 0 1
7. Hint: A =  
(ii) A = 11 13 17   0 0
 29 31 35 (aI + bA)2 = a2I2 + b2A2 + 2aIbA
Hint: A = [aij]3×3 and aij = 3 + 2
i j = a2I + b2A2 + 2abA
 a11 a12 a13  [Q I2 = I and IA = A]
  = a2I + b2 × 0 + 2abA

⇒ A =  a21 a22 a23 
 a31 a32 a33   2 0 1 
= a2I + 2abA Q A =   = 0
  0 0  
Now put the values of i and j in aij and get the answer.
8. Hint: a = 1, b = 4
11 −1 −5
(A + B)2 = A2 + B­2
(iii) A = 17 11 2 
⇒ A + B + AB + BA = A2 + B2 ⇒ AB = –BA
2 2

 22 29 11 
 a −1  1 1  a − b a + 1
AB =   b −1 =  2 − b
7i − 4j if i < j
  2 −1   3 
Hint: A = [aij]3×3 and aij =  11 if i = j

5i + 7j if i > j 1 1   a −1
 –BA = −   
b −1  2 −1
 a11 a12 a13 
   a+2 −2   −a − 2 2 

⇒ A =  a21 a22 a23  = −  = 
ab − 2 −b + 1 
 a31 a32 a33     −ab + 2 b − 1

Now put the values of i and j in aij and get the answer. ⇒ AB = –BA
\ a+1=2 ⇒ a=1
5 5 6 
3=b–1 ⇒ b=4
(iv) A = 5 10 9 
9. (i) AB = BA, (ii) AB = –BA
6 9 15
(i) Hint: (A + B)(A – B) = A2 – B2
 3i + j if i < j
 ⇒ A2 – AB + BA – B2 = A2 – B2
Hint: A = [aij]3×3 and aij = 2i + 3j if i = j

 i + 3j if i > j ⇒ –AB + BA = 0 ⇒ AB = BA
 (ii) (A + B)2 = A2 + B2
 a11 a12 a13  ⇒ A2 + B2 + AB + BA = A2 + B2
 

⇒ A =  a21 a22 a23  ⇒ AB + BA = 0 ⇒ AB = –BA
 a31 a32 a33 
10. x = ± 1 ; y = ± 1
2 2
Now put the values of i and j in aij and get the answer.
 0 1
 and ( xI + yA ) = A
2
 9 8 81  A=
−1 0
(v) A =  8 81 32   
81 32 729 ⇒ x I + y2A2 + 2xyAI = A
2 2

Hint: A = [aij]3×3 and


⇒ x2I + y2A2 + 2xyA = A
[Q I2 = I and AI = A]
1 0  0 1  0 1  0 1  0 1
3i + j if
aij =  i + j
(i + j ) is even
⇒ x2   + y2     +2xy  = 
2 if (i + j ) is odd 0 1  −1 0  −1 0  −1 0  −1 0

 x2 0  −1 0  0 2xy   0 1
 a11 a12 a13 
⇒  2
+ y2   +  −2xy 0  =  −1 0
  0 x   0 −1    
Take A =  a21 a22 a23 
 a31 a32 a33   x2 0   −y 2 0   0 2xy 

⇒  + +
2   −2xy
0 x2   0 −y   0 
Now put the values of i and j in aij and get the answer.

56 Mathematics–12
E:\AMIT_WORKS\Exam_Guru\EG_Mathematics-12_(working_02-06-2022)\EG_Mathematics-12_working\Open_Files\Chap_3\Chap_3
\ 16-Aug-2022 Amit Proof-4 Reader’s Sign _______________________ Date __________

 0 1  0 α
− tan 
=  ⇒ x2 – y2 = 0 and 2xy = 1. 1 0  2
 −1 0 LHS = I + A =  + 
0 1   tan α 0 
1  2 
Put y = in x2 – y2 = 0
2x
 1 α
− tan 

⇒ x − 12 = 0
2 4
⇒ 4x – 1 = 0

= 
2
 = RHS
4x
 tan α 1 
 2 

⇒ x=± 1
2 IV. 1. Try yourself. 2. Try yourself.
3. Try yourself.
1
Since x2 = y2 ⇒ y=±
2  sin 2 θ sin θ cos θ 
4. Hint: f (θ ) =  
sin θ cos θ cos 2 θ 
 0 α
− tan  f (q) f (f)

 2
11. A =  
 tan α 0  =
 sin 2 θ sin θ cos θ   sin 2 φ sin φ cos φ 
 2  2  
sin θ cos θ cos θ  sin φ cos φ cos 2 φ 

cos α − sin α  On multiplying the above two matrices, we get
RHS = ( I − A )  
 sin α cos α   sin θ sin φ sin θ cos φ 
cos (θ − φ )  
1 − tan 2 α cos θ sin φ cos θ cos φ 
  − tan α   −2 tan α 
 1 0  0 2  2 2 
=   − If (q – f) = odd multiple of p/2 then cos(q – f) = 0
  2α 
1 + tan 2 α 
 0 1  tan α 0    1 + tan
2 2  sin θ sin φ sin θ cos φ 
 2
    = 0 =0
 2 tan α 1 − tan 2 α  cos θ sin φ cos θ cos φ 
 2 2 
1 + tan 2 α 1 + tan 2 α   1 tan θ   1 − tan θ 
 2 2 5. Hint: LHS =    tan θ
 − tan θ 1  1 
1 − tan 2 α −2 tan α   1 + tan 2 θ − tan θ + tan θ 
 2 2  = 
 1 α   2α 2α − tan θ + tan θ 1 + tan 2 θ 
tan 1 + tan 1 + tan  
 2 2 2
=   sec2 θ 0  1 0
 − tan α 1   2 tan α 1 − tan 2α
= = sec2 θ  2
 = sec θ I ( )
 2  2 2 2  0 1
0 sec θ  
1 + tan 2 α 1 + tan 2 α   
 2 2 6. Try yourself.

 1 − tan 2 α + 2 tan 2 α 1  7 −1 
−2 tan α + tan α − tan 3 α  7. (i) A=
 2 2 2 2 2 2 37 −15
 2α 
 1 + tan 1 + tan 2 α 
2 2
=  1 2  2 5
 − tan α + tan 3 α + 2 tan α 2 tan 2α
+ 1 − tan 2α  A  =  
 2 2 2 2 2  3 4   −4 7 
 1 + tan 2 α 1 + tan 2 α 
 2 2 
1 2 1 2
−1
 2 5 1 2
−1
A
  =   
 3 4  3 4   −4 7  3 4
 1 + tan 2
2α − tan α 1 + tan 2 α  
2  2 

 1 + tan 2
=

1 + tan 2α
2



AI =    ( )
 2 5 −1  4 −2

 −4 7  2  −3 1 



2 (
 tan α 1 + tan 2 α
2 ) 1 + tan 2α
2


 A= ( )
−1  2 5   4 −2
2α 2α 2  −4 7   −3 1 
 1 + tan 2 1 + tan 
 2

 1

α
− tan 
A= ( )
−1  8 − 15 −4 + 5
2  −16 − 21 8 + 7 
2
= 
 tan α 1  −1  −7 1 
=
 2  2  −37 15

Matrices 57
E:\AMIT_WORKS\Exam_Guru\EG_Mathematics-12_(working_02-06-2022)\EG_Mathematics-12_working\Open_Files\Chap_3\Chap_3
\ 16-Aug-2022 Amit Proof-4 Reader’s Sign _______________________ Date __________

1  7 −1   −3 2  2 −1  1 0
=
2 37 −15

⇒ A =  
 5 −3  −3 2 0 1

 −8 −3  −3 2  2 −1
(ii) A=  
⇒ A = 
5 4  5 −3  −3 2
−1 −1
1 2  2 5  2 −1  −3 2
Hint: A = 
  
⇒ A=  
3 4  −4 7   −3 2  5 −3

3 2  Simplify it.
8. (i) A=   14. A = ` 7,000, B = ` 6,125, C = ` 7,875.
1 −4
Total amount ` 21,000
a c 
Hint: Let A = 
 Try yourself.
b d  15. x = ` 15,000, y = ` 20,000
 a c  1 −1 4  7 3 16  Hint: Let investment at 8% = ` x

 b d   2 3 1  =  −7 −13 −4
     and investment at 10% = ` y

Solve for a, b, c, d.  x + y = 35, 000 and


\  8x 10y
1 −2 100 + 100 = 3200
(ii) A=  
3 4 
 x + y = 35, 000

⇒ 
a c 
Hint: Take A = 
 8x + 10y = 3, 20, 000
b d 
1 1   x   35, 000 
 −3 2 2 
⇒ 8 10  y  = 3, 20, 000
1     
9. A–1 =  2 −3 2 
5 −1
 2 2 −3  x  1 1   35, 000 

⇒  y  = 8 10 3, 20, 000
Hint: First verify A2 – 4A – 5I = 0 by putting
     
1 2 2  x  1 10 −1  35, 000 
A =  2 1 2
⇒  y  = 2  −8 1  3, 20, 000
    
 2 2 1 
 x  1  3, 50, 000 − 3, 20, 000 
Now, A–1[A2 – 4A – 5I] = 0
⇒  y  = 2  −2, 80, 000 + 3, 20, 000
⇒ A–1A2 – 4A–1A – 5A–1I = 0    


⇒ A – 4I – 5A–1 = 0 ⇒ A −1 = 1 [ A − 4I ]

 x  1 30, 000 
5  y  = 2  40, 000
   
Put the values of A and I.
 x  15, 000 
 −1 8 
⇒  y  =  20, 000
10. m = 4, n = –13. A −1 = 1 
13  2 −3    

11. A = ` 30,000, B = ` 23,000, C = ` 39,000 ⇒ x = ` 15,000, y = ` 20,000



16. Fees for poor students = ` 200
 −191 −110
12. D = 
 77 44  Fees for rich students = ` 1000
Hint: Fees for poor students = ` x

Hint: CD – AB = 0

Fees for rich students = ` y

⇒ CD = AB ⇒ D = (C–1) (AB)
1 1 20x + 5y = 9000
13. A = 
\ 
 5x + 25y = 26000
1 0
 2 1   −3 2  1 0  4x + y = 1800
Hint: 
A = 
⇒ 
 3 2  5 −3 0 1  x + 5y = 5200
−1
 −3 2  2 1  1 0  4 1  x  1800 

⇒ A =   
⇒ 1 5  y  = 5200
 5 −3  3 2 0 1     

58 Mathematics–12
E:\AMIT_WORKS\Exam_Guru\EG_Mathematics-12_(working_02-06-2022)\EG_Mathematics-12_working\Open_Files\Chap_3\Chap_3
\ 16-Aug-2022 Amit Proof-4 Reader’s Sign _______________________ Date __________

−1
 x   4 1 1800  5x + 6y = 140000

⇒  y  = 1 5 5200
⇒ 
      6x + 5y = 135000

 x  1  5 −1 1800  5 6  x  140000



⇒  y  = 19  −1 4  5200
⇒ 6 5  y  = 135000
         

 x  1  9000 − 5200  −1
 x  5 6 140000

⇒  y  = 19  −1800 + 20800
⇒  y  = 6 5 135000
         
 x  1  3800   200 

⇒  y  = 19 19000 = 1000  x  −1  5 −6 140000
     
⇒  y  = 11  −6 5  135000
    
⇒ x = ` 200 and y = ` 1000.

 x  −1  700000 − 810000 
17. Investment in first bond = ` 10,000
⇒  y  = 11  −840000 + 675000
   
Investment in second bond = ` 15,000
Hint: Let investment in first bond = ` x
 x  −1  −110000 10000

⇒  y  = 11  −165000 = 15000
Investment in second bond = ` y      
10x + 12y = 2800 ⇒ x = ` 10,000, y = ` 15,000

100 100

⇒ 
12x + 10y = 2700
100 100

Topic 2. Operations on Matrices


(A) Transpose of a Matrix  2 3
If A = [aij]m×n be a m × n matrix, then the matrix obtained by  3 2 1  
Solution. It is given that: A =   and B = 1 0
 4 −1 3
changing rows into columns of matrix A is called transpose of  4 5
matrix A. Transpose of a matrix A is denoted by A′ or AT.
 2 3′
In other words, if matrix A = [aij]m×n then its transpose A′  3 2 1  
A + B′ =  4 −1 3 +  1 0 
= [aji]n×m.   
 4 5
Here: (i) aij of matrix A = aji of matrix A′.
 3 2 1  2 1 4   5 3 5
(ii) If order of matrix A is m × n then order of transpose = + =
 4 −1 3  3 0 5 7 −1 8
of matrix A or A′ is n × m.
 3 2 −1 Hence, option (c) is the correct answer.
For example: If matrix A =   ,  2 3
 4 −5 6  2×3  3 2 1 1 0
Example 2. If matrix A =   and matrix B =  
 4 −1 3
 3 4  4 5
then A′ =  2 −5 , then value of A′ – (B′)′ will be
 
 −1 6  3×2 2 5 4 2
 2 3

(a) 3 1 (b) 7 −1

  
 3 2 1    3 −2
Example 1. If matrix A =   and matrix B = 1 0  2 −2
 4 −1 3  4 5  3 5  1 1
, then value of A + B′ will be (c)  4 2 (d)  
   1 −1
 −1 3  −3 −2
 5 5 3  3 5 3
(a)   (b)  
7 8 1 7 4 1   2 3′ ′
3 2 1 ′  
 5 3 5  5 7 8 Solution. A′ – (B′)′ =   −   1 0  
(c)   (d)    4 −1 3   4 5 
7 −1 8  −1 2 3   

Matrices 59
E:\AMIT_WORKS\Exam_Guru\EG_Mathematics-12_(working_02-06-2022)\EG_Mathematics-12_working\Open_Files\Chap_3\Chap_3
\ 16-Aug-2022 Amit Proof-4 Reader’s Sign _______________________ Date __________

 3 4 Theorem: If A is a square matrix then prove that: P = A + A′ is


′  3 4  2 3  1 1
 2 −1 −  2 1 4   2 −1 −  1 0 =  1 −1 symmetric matrix.
=    3 0 5 =       Proof: Take any square matrix A.
 1 3  
 1 3  4 5  −3 −2
P = A + A′
Hence, option (d) is the correct answer. ⇒ P′ = (A + A′)′
⇒ P′ = A′ + (A′)′
Properties of Transpose Matrix
[Q (A + B)′ = A′ + B′]
1. For any matrix A, (i) (A′)′ = A and (ii) (kA)′ = kA′.
⇒ P′ = A′ + A [Q (A′)′ = A]
2. For any two matrices A and B of same order: ⇒ P′ = A + A′ [Q Matrix addition is
(i) (A + B)′ = A′ + B′ commutative]
(ii) (A – B)′ = A′ – B′ ⇒ P′ = P
(iii) (mA + nB)′ = mA′ + nB′ ⇒ P = (A + A′) is a symmetric matrix.
(iv) (mA – nB)′ = mA′ – nB′ Theorem: If matrix A is a square matrix, then prove that: Q =
Here m and n are scalar number. (A – A′) is a skew symmetric matrix.
3. Let A, B and C be three matrices such that their respective Proof: Take a square matrix A
orders are: m × n, n × p and p × q. Then Q′ = A – A′
(i) (AB)′ = B′A′ Q′ = (A – A′)′ = A′ – (A′)′
(ii) (ABC)′ = C′B′A′ [Q (A – B)′ = A′ – B′]
⇒ Q′ = A′ – A [Q (A′)′ = A]
(B) Symmetric and Skew Symmetric Matrix
⇒ Q′ = –A + A′
A square matrix A is called symmetric if its transpose is equal to [ Matrix addition is commutative]
the matrix A itself.
⇒ Q′ = –(A – A′)
OR ⇒ Q′ = – Q
A square matrix A is called symmetric if A′ = A ⇒ Q is a skew symmetric matrix.
 7 4 8 Theorem: Each diagonal element of a skew symmetric matrix
For example, matrix A =  4 3 9 is symmetric matrix is zero.
 
8 9 5 Proof: Let A = [aij]m×m be a skew symmetric square matrix.
\ A′ = –A ⇒ [aij]′ = –[aij]
7 4 8
⇒ [aji] = [–aij]
because A′ = 4 3 9 = A.

  ⇒ aji = –aij ∀ i and j...(i)
8 9 5
To take only diagonal elements, put j = i in (1), we get
 x A B \ aii = –aii ⇒ 2aii = 0
In general matrix A =  A y C is a symmetric matrix. ⇒ aii = 0
 
 B C z  Since aii are the elements of main diagonal, hence all the
elements of the main diagonal of a skew symmetric matrix are zero.
A square matrix A is called skew symmetric if its transpose is Theorem: A matrix which is both symmetric and skew symmetric
equal to negative of the matrix A itself. Each diagonal element is a zero matrix.
of a skew symmetric matrix is zero. Proof: Let A = [aij]m×m be square matrix which is both symmetric
Or and skew symmetric.
A square matrix A is called skew symmetric matrix if A′ = –A. Since A is a skew symmetric matrix, A′ = –A
 0 5 −7  ⇒ [aij] = –[aij] ⇒ [aji] = –[aij]
  ⇒ aji = –aij ...(1)
For example, matrix A =  −5 0 −8 is a skew symmetric
matrix because  7 8 0 Since A is a symmetric matrix \ A′ = A
⇒ [aij]′ = [aij] ⇒ [aji] = [aij]
 0 −5 7   0 5 −7  ⇒ aji = aij ...(2)
A′ =  5 0 8 = −  −5 0 −8 = −A From (1) and (2), we get: aij = –aij
 −7 −8 0  7 8 0 ⇒ 2aij = 0 ⇒ aij = 0 ∀ i and j

Hence all the elements of matrix A are zero i.e., A is a zero matrix.
 0 A −B
Theorem: If A is a square matrix, then prove that (i) P = AA′ is
In general, matrix P =  −A 0 C is a skew symmetric
 a symmetric matrix and (ii) Q = A′A is also symmetric matrix.
 B −C 0
matrix. Proof: Take any square matrix A.

60 Mathematics–12
E:\AMIT_WORKS\Exam_Guru\EG_Mathematics-12_(working_02-06-2022)\EG_Mathematics-12_working\Open_Files\Chap_3\Chap_3
\ 16-Aug-2022 Amit Proof-4 Reader’s Sign _______________________ Date __________

(i) Take P = AA′  0 3 −1


⇒ P′ = (AA′)′ ⇒ P′ = [(A′)′]A′  
= −  −3 0 −8 = −Q
[Q (AB)′ = (B′A′)]
 1 8 0 
⇒ P′ = AA′ [Q (A′)′ = A]
⇒ P′ = P ⇒ Q is a skew symmetric matrix.
\ P is a symmetric matrix. 2 4 2 2 1 3
(ii) Take Q = A′A  1 0 −3  4 0 5
(iii) R =AA′ ⇒ R=  
⇒ Q′ = (A′A)′  3 5 4   2 −3 4 
⇒ Q′ = A′(A′)′ [ (AB)′ = B′A′]
⇒ Q′ = A′A = Q [Q (A′)′ = A]  24 −4 34
 
⇒ Q′ = Q ⇒ R =  −4 10 −9 = R′
\ Q′ is also a symmetric matrix.  34 −9 50
 2 4 2 \ R is a symmetric matrix.
Example 3. If matrix A =  1 0 −3 , (iv) S = A′A
 
 3 5 4 2 1 3  2 4 2 
 4 0 5  1 0 −3
(i) work out P = (A + A′) and show that P is a symmetric ⇒ S=   
matrix.  2 −3 4   3 5 4 
(ii) work out Q = (A – A′) and show that Q is a skew symmetric
14 23 13
matrix.  
(iii) work out R = (AA′) and show that R is a symmetric matrix. =  23 41 28 = S′
 13 28 29 
(iv) work out S = (A′A) and show that S is a symmetric matrix.
\ S is a symmetric matrix.
 2 4 2
Theorem: Any square matrix can be expressed as the sum of a symmetric
Solution. It is given that: matrix A =  1 0 −3 and a skew symmetric matrices.
 
 3 5 4 Proof: Let matrix A be a square matrix.
\ A= A
 2 4 2  2 1 3
1 1
(i) P = A + A′
  
⇒ P =  1 0 −3 +  4 0 5
 ⇒ A = [ 2A ] = [ A + A ]
2 2
 3 5 4  2 −3 4 1
⇒ A = ( A + A′ ) + ( A − A′ )
2



 4 5 5
⇒ P =  5 0 2
 ⇒
1
2   
1
A =  ( A + A′ ) +  A − A′ 

2
 

( )
 5 2 8 Symmetric matrix Skew symmetric matrix

 4 5 5  3 5 2
 1 7 4
⇒ P′ = 5 0 2 = P
 Example 4. If matrix A =   , express matrix A as a sum of
 
 5 2 8  4 −5 8

⇒ P is a symmetric matrix. symmetric and skew symmetric matrices


(ii) Q = A – A′  3 5 2
 
Solution. It is given that: A =  1 7 4
2 4 2 2 1 3
 1 0 −3 −  4 0 5  4 −5 8
⇒ Q=    
 3 5 4   2 −3 4    3 5 2  3 1 4 
1 1
⇒ ( A + A′ ) = 2   1 7 4 +  5 7 −5
 0 3 −1 2   4 −5 8  2 4
     8
⇒ Q =  −3 0 −8
 1 8 0  
 3 3 3
 6 6 6
 0 −3 1 1   −1
  = 6 14 −1 = 3 7 
⇒ Q′ =  3 0 8 2 2
6 −1 16 
 −1 −8 0  3 −1 8
 2 

Matrices 61
E:\AMIT_WORKS\Exam_Guru\EG_Mathematics-12_(working_02-06-2022)\EG_Mathematics-12_working\Open_Files\Chap_3\Chap_3
\ 16-Aug-2022 Amit Proof-4 Reader’s Sign _______________________ Date __________

  3 5 2  3 1 4     
1 1   1 7 4  −  5 7 −5  3 3 3  0 2 −1
Also ( A − A′ ) = 2    
2  
  4 −5 8  2 4 8  −1  9
    ⇒ A = 3 7 +  −2 0 
 2  2
  3 −1 8  1 −9 0
0 2 −1  
 0 4 −2    2
  
2
1   0 9
 Symmetric matrix Skew symmetric matrix
=  −4 0 9  =  −2
2 2
 2 −9 0    3 5 2
 1 −9 0   
 2  So, it is clear that matrix A =  1 7 4 can be expressed
1 1  4 −5 8
Now A = ( A + A′ ) + ( A − A′ )
2 2 as the sum of symmetric and skew symmetric matrices.

EXERCISE 3.2
I. Multiple Choice Questions (MCQs) 2 3
3 5
(c) a = ,b= (d) a = − ,b=
Choose the correct answer from the given options. 7 2 3 2

 3  3 5
5. A =   is written as A = P + Q. If P is a symmetric matrix
1. If A =  5 and B = [1 0 4] , then (AB)′ equals 7 9
 
 2 and Q is skew symmetric matrix, then value of matrix P will be

 −1 2 1  2 5 8
     1 5 7 8
(a)  7 2 3 (b)  3 2 1 (a)   (b) 
3 4  9 4
 5 11 5  5 4 9
 3 7 3 6
(c)  (d) 
 3 5 2  3 2 5 6 7

6 9
   
(c)  0 0 0 (d) 7 8 11
 cos θ sin θ 
12 20 8 7 0 5 6. If f (θ ) =   , then [ f (q)]′ is equal to
 − sin θ cos θ 
a + b 2  6 5 ′
2. If  = , then a equals (a) f (q) (b) f (–q)
 5 b   2 2
 θ
(c) f (2q) (d) f  
(a) 3 (b) 4  2

(c) 6 (d) 5  2 1 3
7. If matrix A =  3 0 4 , then value of AA′ will be
 cos α sin α   
3. If A =  π 1 5 1 
 , a satisfying 0 < α < 2 and A +
 − sin α cos α 
A′ = 2 I 2 then value of a is  5 7 11 10 25 7 
   
(a) 17 3 8 (b) 10 7 8
π π
(a) (b) 14 18 10 11 12 3
3 4
π π 14 18 10 15 5 8
(c) (d)
 
6 2 (c) 18 25 7  (d)  7 8 11
 
 0 2b −2 10 7 27  13 12 13
4. Matrix A = 3  1 3 is given to be symmetric

3a 3 −a  3 4  2
8. If A =   , then (A )′ equals
matrix. The values of a and b will be 1 5
3 1 2 1 (a) (A′)2 (b) A′
(a) a = , b = (b) a = , b =
5 5 5 5 (c) A3 (d) AI

62 Mathematics–12
E:\AMIT_WORKS\Exam_Guru\EG_Mathematics-12_(working_02-06-2022)\EG_Mathematics-12_working\Open_Files\Chap_3\Chap_3
\ 16-Aug-2022 Amit Proof-4 Reader’s Sign _______________________ Date __________

 3 1 1 1 1
 2 1 3
, B =  −1 4 and C =  2 −1  
9. If A =      5 7  , (AB 3. If matrix A = 1 1 1 then using P.M.I. prove that
 4 7 −1  
 0 3 1 1 1
+ C)′ equals 3n −1 3n −1 3n −1
(a) (B′A′ + C′) (b) (A′B′ + C′)  
A n = 3n −1 3n −1 3n −1 where n ∈ N.  [V. Imp.]
(c) (C′ + BA′) (d) (C′ + AB′) 3n −1 3n −1 3n −1
 
10. If a matrix A is both symmetric and skey symmetric, then
A necessarily a 3 − 4 
(a) Diagonal matrix 4. If A =   , then using P.M.I. prove that:
1 − 1 
(b) Zero square matrix
1 + 2n − 4n 
(c) Square matrix An =  : n ∈N  [V. Imp.]
(d) Identity matrix  n 1 −2n 

II. Short Answer Type Questions-I 0 1 n


5. If A =   , then using P.M.I. prove that: (aI + bA) =
 2 3  1 5  0 0
1. If matrix A =  1 0 and matrix B =  3 2 , then find anI + nan – 1 bA, n ∈ N.
   
 3 4 0 −1 a b
AB′.
6. If A =   , a ≠ 1, using P.M.I. prove that:
 0 1
 cos θ sin θ 
2. If f (θ ) =   , show that [ f (q)]′ = f (–q).

 n
(
b an − 1 

)
 − sin θ cos θ  A = a
n
a − 1 , n ∈N .
0 1 
5 2   
3. If matrix A =   , express matrix A as a sum of a
1 4   cos θ − i sin θ 
symmetric and a skew symmetric matrix. 7. If f (θ ) =  , then show that: (i) [ f (q)]′
i sin θ cos θ 
 3 1 2
= f (–q) (ii) f (q) [ f (q)]′ = (cos 2q) I.
4. If A =   is a 2 × 2 matrix. Verify: |5A| = (5) |A| =
 4 5 
cos θ + sin θ − 2 sin θ 
25 |A|. 8. If f (θ ) =   , then show that:
 2 sin θ cos θ − sin θ 
5. Solve for x:
( x + 3) ( x − 1) x 1
= (i) [ f (q)]–1 = f (–q) (ii) f (q) f (–q) = I.
( x + 2) (3x − 1) −17 x
1 2 2
1 −1 2
9. If A = 2 1 −2 satisfies AA′ = 9I, find the values of

6. Find the value of the determinant: 3 2 3 .  
 x 2 y 
2 1 1
x and y.
0 sin α − cos α
 3 4
7. Evaluate: ∆ = − sin α 0 sin β  2 3 1  
10. If matrix A = 
1 4 5  and matrix B = 1 5 then
cos α − sin β 0    2 3
 3 1  2 3 verify: (AB)′ = B′A′.
8. If A =   and B =   , verify | AB | = | A | | B |.
 4 5 1 3  3
 
9. If A is a skew symmetric matrix of order 3, then prove that 11. If A =  1 and B = [1 –5 7] verify: (i) (AB)′ =
det A = 0. [Delhi 2017]  −7 
III. Short Answer Type Questions-II B′A′ (ii) (BA)′ = A′B′.
1. Using P.M.I., prove that (A′)n = (An)′, where n ∈ N.  3 1 2
 cos θ sin θ   
2. If matrix A =  12. If matrix A =  2 4 1
 , then using P.M.I. prove
 − sin θ cos θ   1 5 3
 cos nθ sin nθ  (i) work out P = (A + A′) and show that: P is a symmetric
that: A n =   where n ∈ N.
 − sin nθ cos nθ  matrix.

Matrices 63
E:\AMIT_WORKS\Exam_Guru\EG_Mathematics-12_(working_02-06-2022)\EG_Mathematics-12_working\Open_Files\Chap_3\Chap_3
\ 16-Aug-2022 Amit Proof-4 Reader’s Sign _______________________ Date __________

(ii) work out Q = (A – A′) and show that: Q is a skew The cost (in thousands rupees); average expenditure (in
symmetric matrix. rupees) and average returns (in rupees) of each ox, camel
(iii) work out R = (AA ′) and show that: R is a symmetric and tractor is given below:
matrix.  Cost Average Average 
(in thousand `) exp. (in `) income (in `)
(iv) work out S = (A′A) and show that: S is a symmetric  
Ox
matrix.  3.0 20 20 
Camel  
 4.0 20 40 
 5 3 1 Tractor
 50 100 200 
13. If matrix A =  7 4 5 express matrix A as a sum of
 
 2 9 3 Find by using matrix multiplication, the total cost, average
symmetric and skew symmetric matrices. expenditure and average income on oxen, camels and tractors.
IV. Long Answer Type Questions 7. In a legislative assembly election, a political group hired a
 0 1 3 7  public relation firm to promote its candidate in three ways,
1. If matrix A =   and B =   , then verify that
 −1 0 5 2  telephones, house-calls and letters. The cost per contact
P = BAB′ and Q = B′AB are skew symmetric matrix. (in paise) is given in matrix A.
 [HOTS] Cost per contact in paise
 40  Telephones
 0 −2 −3  0 5 −2 A=  
2. If matrix A =  2 0 −5 and matrix B =  −5 0 3
   100  House calls
 
 3 5 0  2 −3 0  50  Letters

are two skew symmetric matrices, then show that P = (AB
The number of contacts of each type made in two cities X
and Y is given below by matrix B.
– BA) is also a skew symmetric matrix. [HOTS]
Telephones House Calls Letters 

B =  1, 000 500 5, 000  X
a b c 
3. If matrix A =  b c a  where a, b and c are real positive
 3, 000 1, 000 10, 000  Y
 
 c a b 
Find the total amount spent by the group in the two cities
numbers. abc = 1 and A′A = I, then find the value of a3 + b3 + c3.
X and Y.
 [HOTS]
8. The shop of a particular school has 10 dozen Chemistry
 cos α sin α  –1 books, 8 dozen Physics books and 10 dozen Economics
4. If A =   and A′ = A , find the value of a.
 − sin α cos α  books. The selling prices are ` 80, ` 60 and ` 40 each
 [HOTS]

()
respectively. Find the total amount the book shop will
 −1 x 
 sin ( πx ) tan −1  receive from selling all the books using matrix algebra.
1 π
5. If A =   and
π  −1 x

sin
π ()
cot −1 ( πx )

9. A manufacturer produces three products x, y and z which
he sells in two markets. Annual sales are indicated below.

1
B= 

− cos −1 ( πx ) tan −1
x 
π  ()
 , then find (A – B).
Market
x
Products
y z
π

sin −1 x
π ()
− tan ( πx )
−1

I 10,000 2,000 18,000
II 6,000 20,000 8,000
6. Farm A and Farm B have oxen, camels and tractors as (a) If unit sale prices of x, y and z are ` 2.50, ` 1.50 and ` 1.00
given below: respectively, find the total revenue from each market with
Oxen Camels Tractors  the help of matrix algebra.
A  20 5 4  (b) If unit cost of the above three commodities are ` 2.0, `
B  10 6 6  1.00 and 50 paise, respectively. Find the gross profit.

64 Mathematics–12
E:\AMIT_WORKS\Exam_Guru\EG_Mathematics-12_(working_02-06-2022)\EG_Mathematics-12_working\Open_Files\Chap_3\Chap_3
\ 16-Aug-2022 Amit Proof-4 Reader’s Sign _______________________ Date __________

Answers 3.2
 3 5 2  2 3
1 3 0 
I. 1. (c)  0 0 0 Hint: Work out AB′ = 1 0 

  5 2 −1
12 20 8   3 4 

 3  3 0 12   cos θ sin θ 
Hint: AB =  5 [1 0 4] =  5 0 20
2. Hint: f (θ ) =  
     − sin θ cos θ 
 2  2 0 8 
cos θ − sin θ 
Now find (AB)′.
⇒  f (θ ) ′ =   and
 sin θ cos θ 
2. (b) 4 3. (b) π
4  cos ( −θ ) sin ( −θ ) 
f ( −θ ) =  
4. (d) a =
−2
,b=
3
 − sin ( −θ ) cos ( −θ )
3 2
cos θ − sin θ 
 0 2b −2  = 
  sin θ cos θ 
Hint:  3 1
3  is symmetric matrix.
3a 3 −a  [Q cos(–q) = cos q and sin(–q) = –sin q]


\ 2b = 3 and 3a = –2 ⇒ b = 3 and a = −2 =  f (θ ) ′
2 3
10 3 1  0 1
 3 6 3. A = 1  +
5. (d)   2  3 8 2  −1 0
6 9
1 A+A 1 1
Hint: P =
( ′) Hint: A = ( A + A′ ) + ( A − A′ )

2 2 2
6. (b) f (–q)
 3 1 15 5 
4. Hint: A =   ⇒ 5A =  
 cos θ sin θ   4 5  20 25
Hint: f (θ ) =  
 − sin θ cos θ  |5A| = 15 × 25 – 20 × 5
cos θ − sin θ  = 375 – 100 = 275 = 25 × 11 = (5)2|A|

⇒  f (θ ) ′ =   and
 sin θ cos θ 
Q |A| = 15 – 4 = 11 ⇒ |5A| = 25|A|
 cos ( −θ ) sin ( −θ )  cos θ − sin θ  5. x = 2, –9 6. D = –6
f ( −θ ) =   = 
 − sin ( −θ ) cos ( −θ )  sin θ cos θ  7. D = 0 8. Try yourself.
[Q cos(–q) = cos q and sin(–q) = –sin q] III. 1. Try yourself.
2. Try yourself. 3. Try yourself.
=  f (θ ) ′ 4. Try yourself. 5. Try yourself.
14 18 10  6. Try yourself.
7. (c) 18 25 7 
   cos θ −i sin θ 
10 7 27  7. (i) Hint: f (q) =  
i sin θ cos θ 
8. (a) (A′)2
 cos ( −θ ) −i sin ( −θ )
13 8 
A 2
( )′ = (A′ ) 2
= 
32 29

⇒ f (–q) = 
i sin ( −θ ) cos ( −θ ) 

9. (a) (B′A′ + C′)  cos θ i sin θ  ′


=   =  f (θ )
 7 10   −i sin θ cos θ 
( AB + C)′ = B′A′ + C′ =   (ii) Hint: f (q)[ f (q)]′
14 36
10. (b) zero square matrix  cos θ −i sin θ   cos θ i sin θ 
=
i sin θ cos θ   −i sin θ cos θ 
17 12 −3 
II. 1. AB′ =  1 3 0  cos 2 θ − sin 2 θ i sin θ cos θ − i sin θ cos θ 
 23 17 −4 = 
i sin θ cos θ − i sin θ cos θ cos 2 θ − sin 2 θ
 

Matrices 65
E:\AMIT_WORKS\Exam_Guru\EG_Mathematics-12_(working_02-06-2022)\EG_Mathematics-12_working\Open_Files\Chap_3\Chap_3
\ 16-Aug-2022 Amit Proof-4 Reader’s Sign _______________________ Date __________

cos 2θ
=
0  1 0 13. A = 1 ( A + A′ ) + 1 ( A − A′ )
0 cos 2 θ  = cos 2θ 0 1 2 2
   
5 5 3  0 −2 − 1 
= (cos 2q)I.  2  2
8. Try yourself. See the solution of Q.No. 7. =  5 4 7  +  2 0 −2 
9. x = –2, y = –1 3  1 
 7 3  2 2 0 
2  
Hint: AA′

Symmetric Matrix Skew Symmetric Matrix
 x + 2y + 4 
 1 0 
9  0 −29
  IV. 1. P = Q =  is skew symmetric matrix.
= 9 0 1
2x − 2y + 2   29 0
 9 
 
 x + 2y + 4 2x − 2y + 2 x2 + y 2 + 4   0 19 19
 9 9 9  2. P = ( AB − BA ) =  −19 0 −19
 
1 0 0  −19 19 0
= 9 0 1 0 is skew symmetric matrix.
0 0 1
 3. a3 + b3 + c3 = 4
Now solve for x and y. 4. a can have any value.
11 17  1
10. (AB)′ = B′ A′ =   5. A − B = I
 26 29 2

 3 1 −7  6. Total cost in Average Average


11. (i) ( AB) = B′ A′ = −15 −5 35
′  Thousand expenditure in income in
  rupees rupees rupees
 21 7 −49
280 900 1400
(ii) (BA)′ = A′B′ = –15
 6 3 3 354 920 1640
 
12. (i) P = (A + A′) =  3 8 6 = P′
7. X = ` 3400, Y = ` 7200
 3 6 6
8. ` 20,160

⇒ P is symmetric matrix
9. (a) Total revenue from market I = ` 46,000
 0 −1 1
(ii) Q = A – A′ =  1 0 −4 = −Q′ Total revenue from market II = ` 53,000
 −1 4 0 (b) ` 15,000, ` 17,000

⇒ Q is skew symmetric matrix. Hint:

14 12 14
(a) Work out the product:
(iii) R = AA′ = 12 21 25 = R ′
 2.50
14 25 35 10, 000 2, 000 18, 000  
 6, 000 20, 000 8, 000  1.50 

⇒ R is symmetric matrix.  
1.00 
14 16 11
(iv) S = A′A = 16 42 21 = S′ 10, 000 2, 000 18, 000 
(2.50 − 2.00) 
  
11 21 14  (b)   (1.50 − 1.00) 
 6, 000 20, 000 8, 000  
 (1.00 − 0.50) 

⇒ S is symmetric matrix.

66 Mathematics–12
E:\AMIT_WORKS\Exam_Guru\EG_Mathematics-12_(working_02-06-2022)\EG_Mathematics-12_working\Open_Files\Chap_3\Chap_3
\ 16-Aug-2022 Amit Proof-4 Reader’s Sign _______________________ Date __________

Case Based Questions


1. Three schools A, B and C organized a mela for collecting 2. Two farmers Velu and Krishna cultivates only three
funds for helping the rehabilitation of flood victims. varieties of rice namely Basmati, Permal and Naura. The
They sold hand made fans, mats and plates from recycled sale (in `) of these varieties of rice by both the farmers
material at a cost of ` 25, ` 100 and ` 50 each. in the month of September and October are given by the
following matrices A and B.

The number of articles sold are given below: September sales in `


School → A B C  Basmati Permal Naura 
Articles ↓ A = 10, 000 20, 000 30, 000 Velu

 
Hand made Fans 40 25 35  50, 000 30, 000 10, 000  Krishna

Mats 50 40 50 October sales in `

Plates 20 30 40  Basmati Permal Naura 


B =  5, 000 10, 000 6, 000  Velu
Based on the above information, answer the following  
 20, 000 10, 000 10, 000 Krishna
questions:
(i) The total money collected by the school A is Based on the above information, answer the following
questions:
(a) ` 700 (b) ` 7000
(i) The combined sales in September and October for farmer
(c) ` 6125 (d) ` 7875
Krishna for all varieties is
(ii) The total amount of money collected by schools B and
(a) ` 90000 (b) ` 130000
C is
(c) ` 8000 (d) ` 13500
(a) ` 14000 (b) ` 15725
(ii) The combined sales in September and October for farmer
(c) ` 21000 (d) ` 13125
Velu of all varieties is
(iii) The total amount of money collected by all the three
(a) ` 86000 (b) ` 90000
schools A, B and C is
(c) ` 80000 (d) ` 81000
(a) ` 15775 (b) ` 14000
(iii) The variety of rice which has the lowest selling value in
(c) ` 21000 (d) ` 17125 the month of September for the farmer Velu is
(iv) Number of handmade fans made by the students of (a) Permal (b) Naura
schools A, B and C are 20, 30 and 40 respectively and
(c) Basmati
number of plates made by the students of schools A,
(d) All of these have the same price
B and C are 40, 25 and 35 respectively. Moreover the
number of mats made by students of respective schools (iv) The variety of rice which has the highest selling value
remain unchanged. Then the total money collected by in the month of September for the farmer Krishna is?
all schools is (a) Naura (b) Permal
(a) ` 21250 (b) ` 6750 (c) Basmati
(d) All of these have the same price
(c) ` 21000 (d) ` 7000
(v) Total number of articles sold by all the three schools is (v) If both farmers receive 2% profit on gross sales, compute
the profit for each farmer for each variety sold in October.
(a) 230 (b) 130
(c) 430 (d) 330  Basmati Permal Naura 
 120  Velu
Ans. (i) (b) ` 7000 (ii) (a) ` 14000 (iii) (c) ` 21000 (a)  100 200
(iv) (a) ` 21250 (v) (c) 330  400 200 200  Krishna

Matrices 67
E:\AMIT_WORKS\Exam_Guru\EG_Mathematics-12_(working_02-06-2022)\EG_Mathematics-12_working\Open_Files\Chap_3\Chap_3
\ 16-Aug-2022 Amit Proof-4 Reader’s Sign _______________________ Date __________

 Basmati Permal Naura  (i) Total revenue of market A is


 (a) ` 64,000 (b) ` 60,400
(b)  100 200 120  Velu
(c) ` 46,000 (d) ` 40,600
 400 300 200  Krishna
(ii) Total revenue of market B is
 Basmati Permal Naura  (a) ` 35,000 (b) ` 53,000
 100  Velu (c) ` 50,300 (d) ` 30,500
(c)  100 200
 400 300 120  Krishna (iii) Cost incurred in market A is
(a) ` 13,000 (b) ` 30,100
 Basmati Permal Naura  (c) ` 10,300 (d) ` 31,000
 120  Velu
(d)  100 200 (iv) Profit in market A and B respectively are
 410 200 200  Krishna (a) (` 15,000, ` 17,000)
(b) (` 17,000, ` 15,000)
Ans. (i) (b) `1,30,000 (ii) (d) ` 81,000
(c) (` 51,000, ` 71,000)
(iii) (c) Basmati (iv) (c) Basmati (d) (` 10,000, ` 20,000)
 Basmati Permal Naura  (v) Gross profit in both market is
 120  Velu
(v) (a)  100 200 (a) ` 23,000 (b) ` 20,300
 400 200 200  Krishna (c) ` 32,000 (d) ` 30,200
Ans. (i) (c) `46,000 (ii) (b) ` 53,000 (iii) (d) ` 31,000
3. A manufacturer produces three stationery products Pencil,
Eraser and Sharpener which he sells in two markets. (iv) (a) (` 15,000, ` 17,000) (v) (c) ` 32,000
4. Amit, Biraj and Chirag were given the task of creating a
square matrix of order 2.
Below are the matrices created by them. A, B , C are the
matrices created by Amit, Biraj and Chirag respectively.
 1 2  4 0 2 0
A= 
−1 3 B =  1 5 C =  1 −2
     
If a = 4 and b = −2, based on the above information answer
the following:
(i) Sum of the matrices A, B and C, A + (B + C) is
 1 6  6 1
(a)   (b)  
2 7 7 2
7 2  2 1
(c)   (d)  
 1 6 7 6
(ii) (AT)T is equal to
 1 2 2 1
(a)   (b)  
 −1 3  3 −1
 1 −1  2 3
(c)   (d)  
 2 3  −1 1
Annual sales are indicated below.
(iii) (bA)T is equal to
Products (in numbers)
Market
Pencil Eraser Sharpener  −2 −4  −2 2
(a)   (b)  
A 10,000 2000 18,000  2 −6  −4 −6
B 6000 20,000 8,000  −2 2  −6 −2
(c)  (d) 
The unit Sale price of Pencil, Eraser and Sharpener are `  −6 −4  2 4
2.50, ` 1.50 and ` 1.00 respectively, and unit cost of the
above three commodities are ` 2.00, ` 1.00 and ` 0.50 (iv) AC − BC is equal to
respectively.  −4 −6  −4 −4
(a)  (b)  
Based on the above information answer the following:  −4 4  4 −6

68 Mathematics–12
E:\AMIT_WORKS\Exam_Guru\EG_Mathematics-12_(working_02-06-2022)\EG_Mathematics-12_working\Open_Files\Chap_3\Chap_3
\ 16-Aug-2022 Amit Proof-4 Reader’s Sign _______________________ Date __________

 −4 −4  −6 4 (iv) How much amount is given to each child by Seema?


(c)  (d)  (a) ` 32 (b) ` 30
 −6 4  −4 −4
(c) ` 62 (d) ` 26
(v) (a + b) B is equal to (v) How much amount Seema spends in distributing the
 0 8  2 10 money to all the students of the Orphanage?
(a)   (b)   (a) ` 609 (b) ` 960
10 2  8 0 (c) ` 906 (d) ` 690
 8 0  2 0 Ans. (i) (a) 5x – 4y = 40
(c)   (d)   5x – 8y = –80
 2 10  8 10
5 −4  x   40
(ii) (c)   =  (iii) (d) 32
 7 2 1 2
Ans. (i) (c)   (ii) (a)  
5 −8  y   −80
 1 6 −
 1 3 (iv) (b) ` 30 (v) (b) ` 960
 −2 2
(iii) (b)   5. In a city there are two factories A and B. Each factory
 −4 −6 produces sports clothes for boys and girls. There are three
 −4 −4 8 0 types of clothes produced in both the factories, type I, II
(iv) (c)  (v) (c) 
 −6 4 
 2 10 and III. For boys the number of units of types I, II and III
respectively are 80, 70 and 65 in factory A and 85, 65 and
4. On her birthday, Seema decided to donate some money to 72 are in factory B. For girls the number of units of types
children of an orphanage home. If there were 8 children
I, II and III respectively are 80, 75, 90 in factory A and 50,
less, everyone would have got ` 10 more. However, if
there were 16 children more, everyone would have got ` 55, 80 are in factory B.
10 less. Let the number of children be x and the amount
distributed by Seema for one child be y (in `).


Based on the above information, give the answer of the
Based on the information given above, answer the following questions:
following questions: (i) If P represents the matrix of number of units of each type
(i) The equations in terms x and y are produced by factory A for both boys and girls, then P is
(a) 5x – 4y = 40 (b) 5x – 4y = 40 given by:
5x – 8y = –80 5x – 8y = 80 Boys Girls
I II III
(c) 5x – 4y = 40 (d) 5x + 4y = 40 I 85 50
(a) II  65 55 (b) Boys  50 55 80 
5x + 8y = –80 5x – 8y = – 80   Girls 85 65 72

(ii) Which of the following matrix equations represent the III 72 80
information given above? Boys Girls
I II III
I 80 80
5 4   x   40 5 −4  x   40 (c) Boys 80 75 90 (d)
(a)    =  (b)   =  II 70 75
5 8  y   −80 5 −8  y   80 Girls 80 70 65  
III  65 90
5 −4  x   40 5 4  x   40 (ii) If Q represents the matrix of number of units of each
(c)    =   (d)   =  type produced by factory B for both boys and girls, then
5 −8  y   −80 5 −8  y   −80
(iii) The number of children who were given some money Q is given by:
by Seema, is Boys Girls
I II III
(a) 30 (b) 40 I 85 50
(a) II  65 55 (b) Boys  50 55 80 
(c) 23 (d) 32   Girls 85 65 72

III 72 80

Matrices 69
E:\AMIT_WORKS\Exam_Guru\EG_Mathematics-12_(working_02-06-2022)\EG_Mathematics-12_working\Open_Files\Chap_3\Chap_3
\ 16-Aug-2022 Amit Proof-4 Reader’s Sign _______________________ Date __________

Boys Girls (i) (ii) (iii)


I II III X 400 300 100
I 80 80
(c) Boys 80 75 90 (d)
II 70 75 Y 300 250 75
Girls 80 70 65   Z 500 400 150
III  65 90
Also, the chance of making of toilets corresponding to one
(iii) The total production of sports clothes of each type for
attempt of given modes is:
boys is given by the matrix:
(i) 2 % (ii) 4 %
I II III I II III
(a) (b) (iii) 20 %
[ 130 137]
165 [ 165 137]
130
Based on the above information, give the answer of the
I II III I II III following questions:
(c) (d)
[ 135 137]
165 [ 135 165]
137
(i) The cost incurred by the organisation on village X is:
(iv) The total production of sports clothes of each type for (a) ` 10,000 (b) ` 15,000
girls is given by the matrix: (c) ` 30,000 (d) ` 20,000
I II III I II III (ii) The cost incurred by the oraganisation on village Y is:
(a) (b)
[130 130 170] [170 130 130] (a) ` 25,000 (b) ` 18,000
I II III (c) ` 23,000 (d) ` 28,000
(c) (d) None of these
[130 170 130] (iii) The cost incurred by the organisation on village Z is:
(v) Let R be a 3 × 2 matrix that represents the total (a) ` 19,000 (b) ` 39,000
production of sports clothes of each type for boys and (c) ` 45,000 (d) ` 50,000
girls, then transpose of R is: (iv) The total number of toilets that can be expected after the
promotion in village X is:
(a) 165 135 137  (b) 130 130 170
130 130 170 165 135 138 (a) 20 (b) 30
(c) 40 (d) 50
165 132 130 168
(c) 135 130 (d) 130 135 (v) The total number of toilets that can be expected after the
    promotion in village Z, is:
137 170 170 137 
Boys Girls Boys Girls (a) 26 (b) 36
I 80 80 I 85 50 (c) 46 (d) 56
Ans. (i) (d) II 70 75 (ii) (a) II  65 55 Ans. (i) (c) `30,000 (ii) (c) ` 23,000 (iii) (b) ` 39,000
   
III  65 90 III 72 80
(iv) (c) 40 (v) (d) 56
I II III 6. Three car dealers, say A, B and C, deal in three types of
I II III
(iii) (c) [165 135 137 ] (iv) (a) cars, namely Hatchback car, Sedan car, SUV car. The
[130 130 170]
sales figure of 2019 and 2020 show that dealer A sold
(v) (a) 165 135 137  120 Hatchback, 50 Sedan, 10 SUV cars in 2019 and 300
130 130 170
Hatchback, 150 Sedan, 20 SUV cars in 2020; dealer B
6. To promote the making of toilets for women, an sold 100 Hatchback, 30 Sedan, 5 SUV cars in 2019 and
organisation tried to generate awareness through (i) houses 200 Hatchback, 50 Sedan, 6 SUV cars in 2020, dealer C
(ii) e-mails and (iii) announcements. sold 90 Hatchback, 40 Sedan, 2 SUV cars in 2019 and 100
Hatchback, 60 Sedan, 5 SUV cars in 2020.


The cost for each model per attempt is given below:
(i) ` 50 (ii) ` 20
(iii) ` 40 Based on the above information, give the answer of the
The number of attempts made in the village X, Y and Z are following questions:
given below: (i) The matrix summarising sales data of 2019 is:

70 Mathematics–12
E:\AMIT_WORKS\Exam_Guru\EG_Mathematics-12_(working_02-06-2022)\EG_Mathematics-12_working\Open_Files\Chap_3\Chap_3
\ 19-Aug-2022 Amit Proof-4 Reader’s Sign _______________________ Date __________

Hatchback Sedan SUV Hatchback Sedan SUV


A 300 150 20  A 420 200 30 
(a) B  200 50 6  (c) B  300 80 11 
   
C 100 60 5  C 190 100 7 

Hatchback Sedan SUV (d) None of these


A 120 50 10 
(b) B  100 30 5  (iv) The increase in sales from 2019 to 2020 is given by the
 
C 90 40 2  matrix:
Hatchback Sedan SUV
Hatchback Sedan SUV A 180 100 10 
A 100 30 5  (a) B  
(c) B  10 20 1
120 50 10   
  C 100 20 3 
C 90 40 2 
Hatchback Sedan SUV
Hatchback Sedan SUV A 10 20 3 
A 200 50 6  (b) B  
(d) B  100 20 1
100 30 5   
  C 180 100 10 
C 300 150 20 
Hatchback Sedan SUV
(ii) The matrix summarising sales data of 2020 is: A 180 100 10 
(c) B  100 20 1 
Hatchback Sedan SUV  
C 10 20 3 
A 300 150 20 
(a) B  200 50 6 
  Hatchback Sedan SUV
C 100 60 5  A 100 20 3 
(d) B  180 100 10 
Hatchback Sedan SUV  
C 10 20 3 
A 120 50 10 
(b) B  100 30 5 
  (v) If each dealer receive profit of ` 50,000 on sale of a
C 90 40 2 
Hatchback, ` 100,000 on sale of a Sedan and ` 200,000
Hatchback Sedan SUV
A 100 30 5  on sale of a SUV, then amount of profit received in the
(c) B  120 50 10 
  year 2020 by each dealer is given by the matrix.
C 90 40 2 
A 30000000 A 12000000 
Hatchback Sedan SUV (a) B 15000000  (b) B 16200000 
A 200 50 6     
C 12000000  C 34000000
(d) B  100 30 5 
 
C 300 150 20  A 34000000 A 15000000 
(3) B 16200000  (d) B 30000000
(iii) The total number of cars sold in two given years, by each    
C 12000000  C 12000000 
dealer, is given by matrix:
Hatchback Sedan SUV
Hatchback Sedan SUV A 120 50 10 
A 190 100 7 Ans. (i) (b) B  100 30 5 
  
(a) B  300 80 11  C 90 40 2 
 
C 420 200 30 
Hatchback Sedan SUV
Hatchback Sedan SUV A 300 150 20 
A 300 80 11 (ii) (a) B  200 50 6 
  
(b) B  190 100 7  C 100 60 5 
 
C 420 200 30 

Matrices 71
E:\AMIT_WORKS\Exam_Guru\EG_Mathematics-12_(working_02-06-2022)\EG_Mathematics-12_working\Open_Files\Chap_3\Chap_3
\ 22-Aug-2022 Amit Proof-4 Reader’s Sign _______________________ Date __________

Hatchback Sedan SUV Based on the above information, give the answer of the
A 420 200 30  following questions:
(iii) (c) B  300 80 11 
  (i) If ` 15000 is invested in bond X, then:
C 190 100 7 
Investment X Y
Hatchback Sedan SUV (a) X 15000  ; B = [0.1 0.08] interest rate
A= 
A 180 100 10  Y  20000
(iv) (c) B  100 20 1 
  Interest rate
C 10 20 3 X Y
 (b) A = Investment [15000 20000] ; X  0.1 
B= 
Y 0.08
A 34000000
(v) (c) B 16200000  Interest rate
  X Y
C 12000000  (c) A = Investment [20000 15000]; X 0.08
B= 
Y  0.1 
6. A trust fund has ` 35,000 that must be invested in two
(d) None of these
different types of bonds, say X and Y. The first bond pays (ii) If ` 15,000 is invested in bond X, then total amount of
10% interest p.a. which will be given to an old age home interest received on both bonds is:
and second one pays 8% interest p.a. which will be given (a) ` 2,000 (b) ` 2,100
to WWA (Woman Welfare Association). (c) ` 3,100 (d) ` 4,000
(iii) If the trust obtains an annual total interest of ` 3200,
then the investment in two bonds is:
(a) ` 15,000 in X, ` 20,000 in Y
(b) ` 17,000 in X, ` 18,000 in Y
(c) ` 20,000 in X, ` 15,000 in Y
IBMA (d) ` 18,000 in X, ` 17,000 in Y
TRUST FUND
(iv) The total amount of interest received on both bonds is
     
given by:
(a) AB (b) A′B
(c) B′A (d) None of these
(v) If the amount of interest given to old age home is ` 500,
then the amount of investment in bond Y is:
(a) ` 20,000 (b) ` 30,000
(c) ` 15,000 (d) ` 25,000
Interest rate
X Y X 0.1 
Ans. (i) (b) A = Investment [15000 20000] ; B = 
Y 0.08
(ii) (c) ` 3,100
(iii) (c) ` 20,000 in X, ` 15,000 in Y
(iv) (a) AB
Let A be 1 × 2 matrix and B be a 2 × 1 matrix, representing
the investment and interest rate on each bond respectively. (v) (b) ` 30,000

72 Mathematics–12
E:\AMIT_WORKS\Exam_Guru\EG_Mathematics-12_(working_02-06-2022)\EG_Mathematics-12_working\Open_Files\Chap_3\Chap_3
\ 16-Aug-2022 Amit Proof-4 Reader’s Sign _______________________ Date __________

Author’s Comments
Questions based on following types are very important for Exams. So, students are advised to revise them thoroughly.
1. To express a square matrix into sum of symmetric and skew-symmetric matrices.
2. Questions based on property |adj A| = |A|n – 1.
3. To find inverse of a square matrix. (Most Important)

COMMON ERRORS
Errors Corrections
(i) Wrong notation for matrices. (i) Matrices: ( ) or [ ].
(ii) Solution of equations: (ii) If AX = B, then X = A–1B.
If AX = B, then X = BA–1 is wrong.
(iii) Not multiplying the first row elements with the second (iii) By memorising tips like Run and Jump remember while
matrix corresponding column elements and add multiplying two matrices. More practice on various order
matrices for multiplication.

IMPORTANT FORMULAE
1. Multiplication of a Matrix by a Scalar Number
If A = [aij]m×n is a matrix and k is any scalar number, then kA = k[aij]m×n = [kaij]m×n.
2. Addition of Matrices
If matrix A = [aij]m×n and matrix B = [bij]m×n, then
A + B = [aij]m×n + [bij]m×n = [aij + bij]m×n.
3. Subtraction of Matrices
If matrix A = [aij]m×n and matrix B = [bij]m×n then
(A – B) = [aij]m×n – [bij]m×n = [aij – bij]m×n
4. Multiplication of a matrix with another matrix
[aij]m×n × [bjk]n×p = [cik]m×p
5. If A–1 is the inverse matrix of matrix A then AA–1 = A–1A = I.
6. If A′ is the transpose of matrix A, then
(i) (A′)′ = A (ii) (A + B)′ = A′ + B′
(iii) (A – B)′ = A′ – B′ (iv) (AB)′ = B′A′
(v) (ABC)′ = C′B′A′.
7. If A is a square matrix, then
(i) (A + A′) is a symmetric matrix. (ii) (A – A′) is a skew-symmetric matrix.
(iii) (AA′) and (A′A) are symmetric matrix.
8. Any square matrix A can be expressed as a sum of symmetric and skew-symmetric matrix:

A= 1
( A + A′ ) +
2 

1
(
A − A′
2 

)
Symmetric matrix Skew-symmetric matrix

Errors 73
E:\AMIT_WORKS\Exam_Guru\EG_Mathematics-12_(working_02-06-2022)\EG_Mathematics-12_working\Open_Files\Chap_3\Chap_3
\ 16-Aug-2022 Amit Proof-4 Reader’s Sign _______________________ Date __________

REVISION CHART

Matrix
An arrangement of numbers in horizontal and vertical lines is called a matrix.
4 1 3 
A =  0 −4 5  is a matrix
 
  columns of the matrix
rows of the matrix

4, 1, 3, 0, –4 and 5 are elements of the matrix. Matrix A has order 2 × 3 (Two rows ×
three columns)

Types of Matrices
(i) Row matrix, (ii) Column matrix, (iii) Zero or null matrix, (iv) Square matrix, (v) Rectangular matrix,
(vi) Diagonal matrix, (vii) Identity or Unit matrix, (viii) Scalar matrix

Equality of Matrices Multiplication of a Matrix Addition and Subtraction of matrices


Matrix A = [aij]m×n and matrix and a Scalar Matrix If A = [aij]m×n and B = [bij]m×n are two
B = [bij]m×n are called equal Let matrix A = [aij]m×n and matrices of equal order then
matrices if they are of same k is a scalar number then (A + B) = [aij]m×n + [bij]m×n = [aij + bij]m×n
order and aij = bij. kA = k [aij]m×n = [kaij]m×n. (A – B) = [aij]m×n – [bij]m×n = [aij – bij]m×n

Multiplication of a Matrix by another Matrix Symmetric Matrix


The product AB of two matrices A and B is defined only if A square matrix A is called symmetric if its transpose
number of columns of the first matrix A is equal to number is equal to the matrix A itself or a square matrix A is
of rows of the second matrix. symmetric if A = A′.
If A = [aij]m×n and [bjk]n×p then
 x A B
A =  A y C = A ′
AB = [aij]m×n × [bjk]n×p = [cik]m×p
If A, B and C are three matrices such that A = [aij]m×n
 B C z 
B = [bjk]n×p and C = [ckl]p×q then
(i) AB ≠ BA (ii) (AB)C = A(BC) \ A is a symmetric matrix.

Skew-Symmetric Matrix
A square matrix A is called skew
symmetric matrix if its transpose
is equal to negative of the matrix
A itself, i.e., A′ = –A.
 0 A −B 
A′ =  − A 0 C  = − A
 B − C 0 
\ A is a skew-symmetric matrix.

74 Mathematics–12
E:\AMIT_WORKS\Exam_Guru\EG_Mathematics-12_(working_02-06-2022)\EG_Mathematics-12_working\Open_Files\Chap_4\Chap_4
\ 16-Aug-2022 Amit Proof-5 Reader’s Sign _______________________ Date __________

Topics Covered
4 Determinants

4.1 Determinant 4.2 Area of a Triangle


4.3 Adjoint of a Square Matrix 4.4 Inverse of a Non-singular Square Matrix
4.5 Applications of Matrices

C hapter map
DETERMINANTS

Determinants of a Minors and Solution of


Square Matrix Co-factors Determinant Equation

Singular Evaluation of Area of a


Matrix Determinants Triangle

INVERSE OF A MATRIX BY ELEMENTARY OPERATIONS OF A MATRIX

Adjoint of a Square Inverse of a Application of


Matrix Non-Singular Square Martrix Martrices

Topic 1. Determinant
(A) Determinant of a Square Matrix Determinant of a (2 × 2) Matrix
To every square matrix A = [aij]n×n we can associate a number  a11 a12 
calculated with a rule. This number is called determinant of the If matrix A =   , then determinant of matrix
 a21 a22 
matrix. Determinant of a matrix is denoted by (det. A) or | A | or D.
 a11 a12 a13  a11 a12
  A = A = det A = = a11a22 − a12a21 .
Suppose A is a matrix which is denoted as A=  a21 a22 a23  , a21 a22
 a31 a32 a33  Note: Value of a determinant can be any positive, negative or
zero real number.
a11 a12 a13
7 2
then det A = | A | = D is denoted as a21 a22 a23 Example 1. If matrix A =   , then determinant A will be
given by 1 3
a31 a32 a33
(a) 19 (b) 17 (c) 15 (d) 16
Here we shall learn how to determine the special number
called determinant or value of determinant of a square matrix. 7 2
Solution. It is given that: matrix A =  
Note: Only square matrices have determinants, rectangular 1 3
matrices do not have determinants. 7 2
\ | A | = det A = = 7 × 3 − 2 × 1 = 21 − 2 = 19
1 3
Determinant of a (1 × 1) Matrix \ det A = 19.
If A = [a]1×1 is a 1 × 1 order matrix, then det A = | a | = a. Hence, option (a) is the correct answer.

75
E:\AMIT_WORKS\Exam_Guru\EG_Mathematics-12_(working_02-06-2022)\EG_Mathematics-12_working\Open_Files\Chap_4\Chap_4
\ 16-Aug-2022 Amit Proof-5 Reader’s Sign _______________________ Date __________

Theorem: If A is a 2 × 2 matrix, then | mA | = m2| A |. \ M11 = Minor of the element a11 of the det. A
 a11 a12  a11 a12 a13
Proof: Let A=   be a 2 × 2 matrix
 a21 a22  = a21 a22 a23
 a11 a12  a31 a32 a33
\ mA = m  
 a21 a22  a22 a23
⇒ M11 =
 ma11 ma12  a32 a33
⇒ mA =  
 ma21 ma22  ⇒ M11 = (a22 × a33 – a32 × a23)
ma11 ma12 Now, M12 can be worked out by deleting first row and second
⇒ | mA | column of the determinant A.
ma21 ma22
a11 a12 a13
⇒ | mA | = ma11ma22 – ma12 ma21
\ M12 = a21 a22 a23
⇒ | mA | = m2(a11a22 – a12a21)
a31 a32 a33
⇒ | mA | = m2| A |.
a21 a23
 3 1 ⇒ M12
Example 2. If a 2 × 2 matrix A =   , then | 3A | equals a31 a33
 4 5
⇒ M12 = (a21 a33 – a31 a23)
(a) 4 | A | (b) 3 | A |
(c) 6 | A | (d) 9 | A |
Similarly M13 can be worked out by deleting first row and third
 3 1 column of the determinant A.
Solution. It is given that: matrix A =  
 4 5 M13 = minor of element a13 of the det. A
3 1 a11 a12 a13
⇒ |A| = = 15 − 4 = 11 ...(1) a21 a22
4 5 ⇒ M13 = a21 a22 a23  ⇒ M13 =
a31 a32
 3 1  9 3  a31 a32 a33
Now 3A = 3  = 
 4 5 12 15 ⇒ M13 = (a21a32 – a31a22)
9 3 Now you have understood the way we work out minors of different
⇒ | 3A | = = 135 − 36
12 15 elements of det A.
⇒ | 3A | = 99 = 9 × 11  2 1 3
⇒ | 3A | = 9 × | A | = 9| A | [Using (1)] Example 3. If matrix A =  −1 4 7  , then matrix of minors
 
 1 8 5
Hence, option (d) is the correct answer. of determinant will be
(B) Minors 12 36 11  −12 11 2
To work out the value of the determinant of a 3 × 3 matrix or (a)  7 8 9 (b)  36 18 5
   
determinant of a 3 × 3 order matrix: We need to define minors  2 3 5  6 7 2
and cofactors of different elements of the matrix or determinant
of the matrix.  −36 −12 −12  −19 7 15
 
Minor of an element of a square matrix or determinant of (c)  −19 7 15 (d)  5 17 9

a square matrix: Minor of an element aij of a square matrix or  −5 17 9  12 12 31
determinant of a square matrix is the determinant obtained by
deleting its ith row and jth column in which the element aij lies.  2 1 3
Minor of an element aij is denoted by Mij. Solution. It is given that A =  −1 4 7 
 
Note: Minor of an element of a determinant of order n × n  1 8 5
(n ≥ 2), is a determinant of order (n – 1) × (n – 1).
2 1 3
Now take a matrix A of order 3 × 3:
⇒ | A | = −1 4 7
 a11 a12 a13  a11 a12 a13
1 8 5

A =  a21 a22 a23   ⇒ A = a21 a22 a23
4 7
 a31 a32 a33  a31 a32 a33 \ M11 = = 20 − 56 = −36
3× 3
8 5
Then minor of the element a 11 = M 11 , where M 11 is the
determinant obtained by deleting first row and first column of the −1 7
M12 = = −5 − 7 = −12
determinant A. 1 5

76 Mathematics–12
E:\AMIT_WORKS\Exam_Guru\EG_Mathematics-12_(working_02-06-2022)\EG_Mathematics-12_working\Open_Files\Chap_4\Chap_4
\ 16-Aug-2022 Amit Proof-5 Reader’s Sign _______________________ Date __________

−1 4 below before the minors of the different elements to get their


M13 = = −8 − 4 = −12 respective cofactors.
1 8
+ − +
1 3
M21 = = 5 − 24 = −19 − + −
8 5
+ − +
2 3
M22 = = 10 − 3 = 7  3 1 −2
1 5
Example 4. If matrix A =  4 5 9 , then matrix of co-factors
2 1  
M23 = = 16 − 1 = 15  8 7 6
1 8 of A will be given by
1 3
M31 = = 7 − 12 = −5 19 −35 11  11 13 −14
4 7 (a) 34 −4 7  (b)  34 −24 25
  
2 3  8 6 −9  −7 12 18
M32 = M 32 = = 14 + 3 = 17
−1 7
 −33 48 −12  −33 48 −12
2 1  
M33 = = 8 +1 = 9 (c)  −20 34 −13 (d)  −10 34 −3
 
−1 4  19 −35 11  19 −35 −11
\ Matrix of minors of determinant of matrix
 3 1 −2
 −36 −12 −12 Solution. It is given that A =  4 5 9
  
A =  −19 7 15  8 7 6
 −5 17 9
3 1 −2
Hence, option (c) is the correct answer.
⇒ |A| = 4 5 9
(C) Cofactors 8 7 6
Cofactor of an element aij of a determinant is denoted by Aij and Calculation of co-factors:
is defined as: 5 9
A11 = ( −1) = + (30 − 63) = −33
1+1
Aij = (–1)i + j(Mij) M11 = + M11 = +
7 6
Here Mij is the minor of the element aij.
4 9
A12 = ( −1) = − ( 24 − 72) = 48
1+ 2
Let us find out the minors and cofactors of the different elements M12 = −M12 = −
8 6
of a square matrix A or determinant of square matrix A, i.e., |A|.
 a11 a12 a13  a11 a12 a13 4 5
A13 = ( −1) = + ( 28 − 40) = −12
1+ 3
M13 = +M13 = +
a  8 7
A =  21 a22 a23  ⇒ A = a21 a22 a23
 a31 a32 a33  a31 a32 a33 1 −2
A21 = ( −1) = − ( 6 + 14) = −20
2 +1
M 21 = −M 21 = −
7 6
a22 a23
M11 = = ( a22a33 − a32a23 ) 3 −2
a32 a33 A22 = ( −1)
2+ 2
M 22 = + M 22 = + = + (18 + 16) = 34
8 6
[By eliminating first row and first column of matrix A or det. A]
and A11 = (–1)1+1 M11 = (–1)2 M11 = M11 3 1
A23 = ( −1) = − ( 21 − 8) = −13
2+ 3
M 23 = −M 23 = −
⇒ A11 = (a22a33 – a32a23) 8 7
a21 a23 1 −2
A31 = ( −1) = + (9 + 10) = 19
3+1
M12 = = ( a21a33 − a31a23 ) M 31 = +M 31 = +
a31 a33 5 9
and A12 = (–1)1+2M12 = (–1)3M12 = –M12 3 −2
A32 = ( −1) = − ( 27 + 8) = −35
3+ 2
M 32 = − M 32 = −
⇒ A12 = – (a21a33 – a31a23) 4 9
a21 a22 3 1
= ( a21a32 − a31a22 ) A33 = ( −1) = + (15 − 4) = 11
3+ 3
M13 = M 33 = +M 33 = +
a31 a32 4 5
and A13 = (–1)1+3M13 = (–1)4M13 = M13  −33 48 −12
⇒ A13 = (a21a32 – a31a22) 
\ Matrix of cofactors of Matrix A =  −20 34 −13
In place of writing (–1)i+j in front of the minors of a determinant  19 −35 11
of a matrix of order 3 × 3, we can put the +, – signs as shown Hence, option (c) is the correct answer.

Determinants 77
E:\AMIT_WORKS\Exam_Guru\EG_Mathematics-12_(working_02-06-2022)\EG_Mathematics-12_working\Open_Files\Chap_4\Chap_4
\ 16-Aug-2022 Amit Proof-5 Reader’s Sign _______________________ Date __________

Value of the Determinant of a Square Matrix A of D1 = a11A21 + a12A22 + a13A23 = 0;


order 3×3 D2 = a11A31 + a12A32 + a13A33 = 0
 a11 a12 a13  3 4 2
Take a general matrix A of order 3×3: A =  a21 a22 a23 
Example 5. In the determinant 2 1 −3 , the sum of the
 a31 a32 a33 
1 6 5
Value of its determinant D, can be worked out in 6 ways, along
R1, along R2, along R3, along C1, along C2 and along C3 products of the elements of first row with cofactors of second
row is
(i) Value of the determinant along
R1 = a11A11 + a12A12 + a13A13 (a) 0 (b) 5
(ii) Value of the determinant along (c) 6 (d) 8
R2 = a21A21 + a22A22 + a23A23 3 4 2
(iii) Value of the determinant along Solution. It is given that: D = 2 1 −3 . It is required to find
R3 = a31A31 + a32A32 + a33A33 1 6 5
(iv) Value of the determinant along the sum of the products of the elements of the first row with
C1 = a11A11 + a21A21 + a31A31 cofactors of second row.
(v) Value of the determinant along D = a11A21 + a12A22 + a13A23
C2 = a12A12 + a22A22 + a32A32
4 2 3 2 3 4
(vi) Value of the determinant along
= 3 ( −1) +4 + 2 ( −1)
C3 = a13A13 + a23A23 + a33A33 6 5 1 5 1 6
You can calculate the value of determinant of a square matrix of = –3(20 – 12) + 4(15 – 2) – 2(18 – 4)
order 3 × 3 using any one of the above discussed formulae, the = –3(8) + 4(13) – 2(14)
value of the determinant will be same. = –24 + 52 – 28 = 52 – 52 = 0
Theorem (without proof): If elements of a row or column of
⇒ D= 0
determinant of a square matrix are multiplied with cofactors of
any other row or column of the matrix, then their sum is zero. Hence, option (a) is the correct answer.
 a11 a12 a13  Theorem (without proof): The determinant of the product of two
matrices of same order is equal to the product of their respective
For example, A =  a21 a22 a23  then:
determinants, that is | AB | = | A | | B |, where A and B are two square
 a31 a32 a33  matrices of same order.

EXERCISE 4.1
I. Multiple Choice Questions (MCQs)
 5 7 9
Choose the correct answer from the given options.  
5. If A =  9 8 7  , then |A| =
 1 2  1 3
1. If A =   and B =   , the value of |AB| will 19 14 9
be 3 −1  −1 1
(a) 5 (b) 0 (c) 3 (d) 9
(a) –28 (b) –8 1 k 3
(c) –18 (d) 5 6. If ∆ = 3 k −2 = 0, then k =
2. For
( x + 3) ( x − 1) x 1
, value of x will be 2 3 −1
=
( x + 2) (3x − 1) −17 x
(a) – 1 (b) 0
(a) 9, 2 (b) 2, –9 (c) 9, –2 (d) –9, –2 (c) 1 (d) None of these.
0 sin α − cos α 0 p−q p−r
3. If ∆ = − sin α 0 sin β , then value of ∆ will be 7. q − p 0 q − r is equal to
cos α − sin β 0 r− p r−q 0
(a) 5 (b) 0 (a) 0 (b) (p – q) (q – r)(r – p)
(c) 7 (d) 1 (c) pqr (d) 3pqr
 3 1 2 3 1 2
4. If A =   and B = 1 , then | AB | equals 8. If A =   , then |2A| is equal to
 4 5   3  4 2
|A| (a) 2 |A| (b) 4 |A|
(a) | A | | B | (b) (c) | A |2 | B | (d) | B | | A |
|B| (c) 3 |A| (d) None of these.

78 Mathematics–12
E:\AMIT_WORKS\Exam_Guru\EG_Mathematics-12_(working_02-06-2022)\EG_Mathematics-12_working\Open_Files\Chap_4\Chap_4
\ 16-Aug-2022 Amit Proof-5 Reader’s Sign _______________________ Date __________

9. What positive value of x makes following pair of 2 3 4


determinant equal?
15. The value of the determinant −5 6 −10 is
2x 3 16 3
= 1 7 2
5 x 5 2
(a) – 440 (b) 0 (c) 328 (d) 488
(a) 1 (b) 2 (c) 3 (d) 4 II. Short Answer Type Questions-I
1 ω 6 ω8 α 2  3
1. If A =   and |A | = 125, then find the value of a.
10. The value of the determinant ω 6 ω 3 ω 7 where ω3 = 1  2 α 
ω8 ω 7 1 6 −1
is 2. If = 15, then find the value of x.
3 x
(a) 1 (b) 2 (c) 3 (d) 9 3 x 3 2
1 0 1  3. What is the values of x for which = ?
x 1 4 1
11. If A = 0 1 2 , then |3A| is equal to
  sin 10° − cos 10°
0 0 4 4. What is the value of ?
sin 80° cos 80°
(a) 3 |A| (b) 9 |A|
 5 10 3
(c) 27 |A| (d) None of these.
5. The matrix  −2 −4 6 is a singular matrix, then find the
6 −3 2  
 −1 −2 b 
12. The value of D = 2 − 1 2 is value of b.
−10 5 2 III. Long Answer Type Questions
(a) 0 (b) 1 (c) 8 (d) 12  1 17 
 3 5
1 2 k 1. If A =   and B =   , what is the value of |AB| ?
 2 0 0 −10
13. If k 4 5 = 0, then the possible values of k are
 1 2
5 6 7 2. If A =   , then what is the value of k if | 2A | = k| A | ?
8 8  4 2
(a) –3, 3 (c) −3, −1 (d) 3,
(b) −3, −
3 3 3 1 2 3
14. If Cij denotes the cofactor of element Pij of the matrix
3. Find the value of the determinant 0 sec x tan x .
 1 −1 2
P = 0 2 −3 . 0 tan x sec x
 3 2 4
0 2 0
(a) 5 (b) 24 (c) – 24 (d) – 5
4. What is the value of the determinant 2 3 4 ?
[CBSE 2020]
4 5 6

Answers 4.1
I. 1. (a) –28 2. (b) x = 2, –9 0 p−q p−r
3. (b) D = 0 4. (a) |A| |B| 5. (b) 0 Let D= q− p 0 q−r
5 7 9 r− p r−q 0
We have |A| = 9 8 7 Expanding along R1, we get
19 14 9
Expanding along first row, we get D = – (p – q) q − p q − r
r−p 0
|A| = 5(72 – 98) –7(81 – 133) + 9(126 – 152) q− p 0
+ (p – r)
= 5(–26) – 7(–52) + 9(–26) r− p r−q
= –130 + 364 – 234 = 364 – 364 = 0   = (p – q)(q – r)(r – p) + (p – r)(q – p)(r – q)
6. (d) None of these   = (p – q)(q – r)(r – p) – (p – q)(q – r) (r – p) = 0.
Expanding ∆ along R1, we get 8. (b) 4 |A|
(– k + 6) – k(– 3 + 4) + 3(9 – 2k) = 0 1 2  2 4
We have 2A = 2  4 2 = 8 4
⇒ –k + 6 – k + 27 – 6k = 0    
⇒ 33 – 8k = 0 2 4

⇒ |2A| = 8 4 = 2(4) – 8(4) = 8 – 32 = – 24
33
⇒ k=
8 1 2
7. (a) 0 Also |A| = 4 2 = 1(2) – 4(2) = 2 – 8 = – 6

Determinants 79
E:\AMIT_WORKS\Exam_Guru\EG_Mathematics-12_(working_02-06-2022)\EG_Mathematics-12_working\Open_Files\Chap_4\Chap_4
\ 19-Aug-2022 Amit Proof-5 Reader’s Sign _______________________ Date __________

⇒ 4|A| = 4 (– 6) = – 24 1 0 1   3 0 3
Hence, |2A| = 4|A|. 3A = 3 0 1 2 = 0 3 6
0 0 4 0 0 12
In general, |kA| = kn|A|, where n is the order of determinant.  
9. (d) 4 3 0 3
2x 3 16 3
∴ |3A| = 0 3 6
We have = 0 0 12
5 x 5 2
On expanding, we get Expanding along C1, we get
2x2 – 15 = 32 – 15 3 6 0 3 0 3
|3A| = 3 0 12 − 0 0 12 + 0 3 6
⇒ 2x2 – 15 = 17
⇒ 2x2 = 32 ⇒ x2 = 32 ÷ 2 = 16 = 3(36 – 0) – 0 + 0 = 108.
⇒ x = ± 16 ⇒ x = ±4 1 0 1
The positive value of x = 4 Again, |A| = 0 1 2
0 0 4
Hence, for x = 4, the given pair of determinant is equal.
Expanding along C1, we get
10. (c) 3
1 2
1 ω 6 ω8       |A| = 1 −0+0
0 4
∆ = ω6 ω3 ω7
ω8 ω 7 1 = 4–0=4
\ 27 |A| = 27(4)
1 ( ω ) ( ω ) .ω
3 2 3 2 2
= 108
∆= (ω )
3 2
ω ( ω ) .ω
3 3 2
Hence, |3A| = 27|A|.

( ω ) .ω ( ω ) .ω 1
3 2 2 3 2 12. (a) 0
6 −3 2
1 1 ω2 ∆= 2 −1 2
= 1 1 ω [Q ω3 = 1] − 10 5 2
ω2 ω 1 Expanding in terms of elements along the first row, we get
Expanding along first row, we get −1 2 2 2 2 −1
5 2 ( ) − 10 2
∆= 6 − −3 +2
∆ = 1(1 – ω2) – 1(1 – ω3) + ω2(ω – ω2) − 10 5
= 1 – ω2 – 1 + ω3 + ω3 – ω4 = 6 (– 2 – 10) + 3 (4 + 20) + 2 (10 – 10)
= 1 – ω2 – 1 + 1 + 1 – ω3.ω = – 72 + 72 + 2 (0) = 0.
= 2 – (ω2 + ω) 8
[Q 1 + ω + ω2 = 0 Q ω + ω2 = –1] 13. (d) 3, 14. (a) 5 15. (b) 0
3
= 2 – (– 1) = 2 + 1 = 3 II. 1. ±3 2. 2 3. ±2 2 4. 1 5. Any value
11. (c) 27 |A| III. 1. 100 2. 4 3. 1 4. 8

Topic 2. Area of a Triangle


If vertices of a triangle ABC are: A(x1, y1), B(x2, y2) and C(x3, y3), Example 1. Using determinants, the area of the triangle whose
then the area of the triangle is given by the following formula: D vertices are: A(1, 4), B(5, –3) and C(4, 7) is given by
1
=  x1 ( y2 − y3 ) + x2 ( y3 − y1) + x3 ( y1 − y2 ) (a) 16 (b) 15 (c) 16.5 (d) 14
2 Solution. The vertices of triangle ABC are: A(1, 4), B(5, –3)
Now this expression can be written in the form of determinant
and C(4, 7)
x1 y1 1
1 1 4 1
as: D = x y2 1 1
2 2 ⇒ D= 5 −3 1
x3 y3 1 2
4 7 1
Notes: (i) The value of this determinant can be positive or
1
negative, and area of a triangle is always a positive ⇒ D = 2 [1(–3 – 7) – 4(5 – 4) + 1(35 + 12)]
quantity, so always take absolute value of the above
determinant 1
= 2 [–10 – 4 + 47]
(ii) If the area of triangle is given, and you are to work out
1 1
value of some other variable, use both positive and ⇒ D = 2 [47 – 14] = 2 [33] = 33 = 16.5
negative values of the determinant for calculation and 2
\ Area of the DABC = 16.5
(iii) If three points are collinear then the area of the triangle
will be zero. Hence, option (c) is the correct answer.

80 Mathematics–12
E:\AMIT_WORKS\Exam_Guru\EG_Mathematics-12_(working_02-06-2022)\EG_Mathematics-12_working\Open_Files\Chap_4\Chap_4
\ 16-Aug-2022 Amit Proof-5 Reader’s Sign _______________________ Date __________

Example 2. Use determinants, and tell which three points, are 1


collinear ⇒ 2 [5(–1 – 4) – 1(k – 11) + 1(4k + 11)] = 0
(a) A(1, 4), B(3, 7) and C(4, 8) 1
(b) A(2, 4), B(0, 1) and C(4, 7) ⇒ 2 [–25 – k + 11 + 4k + 11] = 0
(c) A(5, 3), B(4, 4) and C(5, 4)
1
(d) A(7, 3), B(6, 1) and C(5, 3) ⇒ 2 [3k – 3] = 0 ⇒ 3k = 3 ⇒ k = 1
Solution. The three points are: A(2, 4), B(0, 1) and C(4, 7)
2 4 1 Hence, option (c) is the correct answer.
1 Example 4. The value of x (using determinants) if the area of the
⇒ D= 2 0 1 1
4 7 1 triangle ABC is 35 square units with vertices A(x, 4), B(2, –6) and
1 C(5, 4) will be given by
= 2 [2(1 –7) – 4(0 – 4) + 1(0 – 4)]
(a) 2, –12 (b) –2, 12 (c) –2, –12 (d) 2, 12
1
= 2 [–12 + 16 – 4] Solution. The three points are: A(x, 4), B(2, –6) and C(5, 4)

1 x 4 1
= 2 ×0=0 1
\ D= 2 −6 1 = ±35
2
\ The three given points are collinear. 5 4 1
Hence, option (b) is the correct answer. ⇒ x[–6 –4] – 4[2 – 5] + 1[8 + 30] = ±70
Example 3. The value of k (using determinants) when the three
⇒ –10x + 50 = ±70 ⇒ –x + 5 = ±7
points A(5, 1), B(k, –1) and C(11, 4) are collinear, will be given by
(a) 4 (b) 2 ⇒ –x + 5 = 7 or –x + 5 = –7
(c) 1 (d) 3 ⇒ x = –2 or x = 12
Solution. The three given points are: A(5, 1), B(k, –1) and C(11, 4) ⇒ x = –2, 12
5 1 1
1 Hence, option (b) is the correct answer.
⇒ D= k −1 1
2
11 4 1
EXERCISE 4.2
I. Multiple Choice Questions (MCQs) 2. Find the equation of the line joining the points A(1, 2) and
Choose the correct answer from the given options. B(3, 6) using determinant method.
1. The area of the triangle (using determinants) with vertices: 3. An equilateral triangle ABC has each side equal to a. If
A (– 2, 5), B (– 6, – 7), and C (– 5, – 4) is given by the coordinates of its vertices are A(x1, y1), B(x2, y2) and
(a) 6 (b) 5 (c) 0 (d) 8 x1 y1 1
3
2. Three points which are collinear given by C(x3, y3), then show that x2 y2 1 = (a)2
2
(a) A (5, 7), B (0, 1) and C (3, 5) x3 y3 1
(b) A (2, 4), B (0, 1) and C (4, 7) 4. The points (2, –3), (k, –1) and (0, 4) are collinear then find
(c) A (0, 1), B (5, 6) and C (1, 5) the value of k.
(d) none of these 5. Find the area of the triangle with vertices (1, 1), (3, 2) and
3. For what value of x, the three points A(x, 2 – 2x), (2, 3) in sq. units.
B(– x + 1, 2x) and C(– 4 – x, 6 – 2x) are collinear? III. Long Answer Type Questions
1 3 3 1 1. Using determinants, find the value of x if the area of the
(a) 1, − (b) 2, − 2 (c) –2, 2 (d) –1,
2 2 DABC is 35 square units with vertices A (x, 4), B (2, –6)
II. Short Answer Type Questions and C (5, 4).
1. Find the area of the triangle formed by three points 2. Find the equation of a line joining the points A(1, 3)
A(p + 1, 1), B(2p + 1, 3) and C(2p + 2, 2p) and show that
and B(0, 0) using determinants and find the value of k if
the points will be collinear if
D(k, 0) is a point such that the area of DABD is 3 sq. units.
p = 2 or −1 [A.I. 2013]
2

Determinants 81
E:\AMIT_WORKS\Exam_Guru\EG_Mathematics-12_(working_02-06-2022)\EG_Mathematics-12_working\Open_Files\Chap_4\Chap_4
\ 16-Aug-2022 Amit Proof-5 Reader’s Sign _______________________ Date __________

Answers 4.2
I. 1. (c) 0 Equating (1) and (2), we get
2. (b) A(2, 4), B(0, 1) and C(4, 7) x1 y1 1
1 3 2
2 4 1 x2 y2 1 = a
2 4
Q ∆ = 0 1 1 = 0 ⇒ Three points are collinear. x3 y3 1
4 7 1 x1 y1 1

⇒ x2 y2 1 = 3 2
x 2 − 2x 1 a
1 2
3. (d) x = –1,  Hint: Evaluate: ∆ = −x + 1 2x 1 =0 x3 y3 1
2
−4 − x 6 − 2x 1 4. The given points (2, –3), (k, –1) and (0, 4) are collinear if
1
II. 1. p = 2 or − 2 −3 1
2
Area of the ∆ = 1 (2p 2 − 3p − 2) . k −1 1 = 0
2 0 4 1
Points are collinear when p = 2 or − 1 Applying R2 → R2 – R1 and R3 → R3 – R1, we get
2
2 −3 1
Hint: Evaluate the determinant for area of the D: k −2 2 0 = 0
p +1 1 1 −2 7 0
1 2 1 3 1
p+ Expanding along C3, we get
2
2p + 2 2 p 1 k −2 2
= 0
−2 7
2. 2x = y
1 2 1
⇒ 7k – 14 + 4 = 0   ⇒ 7k – 10 = 0
Hint: Evaluate: 3 6 1 = 0
10

\ k=
x y 1 7
3
5.
3 2 2
3. a
2 x 4 1
x1 y1 1 1 2
III. 1. x = – 2, 12. Hint: Solve for x: −6 1 = ±35
Hint:
Since D = 1 x2 y2 1 ...(1) 2
2 5 4 1
x3 y3 1 2. k = ±2
3 a 2 ...(2) 1 3 1
and D =
4 Hint: 1 0 0 1 = ±3
2
[Area of an equilateral D with side ‘a’] k 0 1

Topic 3. Adjoint of a Square Matrix


Before discussing adjoint of a square matrix, first know about  2 3 −1
the singular matrix.  x + 4 −1 2 
Solution. A=  
Singular Matrix 3x + 1 2 −1
If value of the determinant of a square matrix is zero, then the
matrix is called singular matrix,otherwise it is called non-singular 2 3 −1
matrix. ⇒ | A | = x + 4 −1 2
 2 3 −1 3x + 1 2 −1
Example 1. If matrix A = x + 4 −1 2  is a singular matrix,

R1 → R1 – R3 and R2 → R2 + 2R3
 
3x + 1 2 −1
−3x + 1 1 0
then the value of x equals
| A | = 7x + 6 3 0
3
(a) 5 (b) 3x + 1 2 −1
16 13
3 3 Evaluate the det. along C3
(c) (d) −
7 16 | A | = –1[–9x + 3 – 7x – 6] = –[–16x – 3] = 16x + 3

82 Mathematics–12
E:\AMIT_WORKS\Exam_Guru\EG_Mathematics-12_(working_02-06-2022)\EG_Mathematics-12_working\Open_Files\Chap_4\Chap_4
\ 16-Aug-2022 Amit Proof-5 Reader’s Sign _______________________ Date __________

Since the given matrix is singular. 3 14 9  −13 −14 19


−3
∴ 16x + 3 = 0 ⇒   x = . (c) 8 7 14 (d)  2 −4 2
16   
3 −5 4  7 10 −9
Hence, option (d) is the correct answer.

Adjoint of a Square Matrix  1 4 3


 
The adjoint of a square matrix A is defined as the transpose of the Solution. Here A =  2 −1 4
matrix of the cofactors of the matrix A. It is denoted as (adj A)  3 2 5
 a11 a12 a13  Cofactors of matrix A are:
Take a matrix A =  a21 a22 a23 
−1 4
 a31 a32 a33  A11 = + = ( −5 − 8) = −13
2 5
Then matrix C = (a matrix of cofactors of matrix A)
2 4
 A11 A12 A13  A12 = − = − (10 − 12) = 2
3 5
⇒  C =  A 21 A 22 A 23 
 A31 A32 A33  2 −1
A13 = + = ( 4 + 3) = 7
3 2
Then (adj A) = Transpose of matrix C = C′
 A11 A 21 A31  4 3
A21 = − = − ( 20 − 6) = −14
∴ (adj of matrix A) = C′ =  A12 A 22 A32  2 5
 A13 A 23 A33  1 3
A22 = + = (5 − 9) = −4
 4 1 3 5
Example 2. If matrix A =  , then its adjoint matrix is
 3 5 1 4
given by A23 = − = − ( 2 − 12) = 10
3 2

 −1 5 5 −4 4 3
(a)  A31 = + = (16 + 3) = 19
 (b)   −1 4
 4 −1  3 2
1 3
 5 −1  4 3 A32 = − = − ( 4 − 6) = 2
(c)   (d)   2 4
 −3 4  −1 4
1 4
A33 = + = + ( −1 − 8) = −9
 4 1 2 −1
Solution. A =  
 3 5
 −13 2 7 
Cofactors of matrix A  
\ C =  −14 −4 10
A11 = (–1)1+1(5) = 5 A12 = (–1)1+2(3) = –3
 19 2 −9
A21 = (–1)2+1(1) = –1 A22 = (–1)2+2(4) = 4
 −13 −14 19
 5 −3  2 −4 2
\ Cofactors of matrix A is C =   ⇒ (adj A) = C′ = 
 −1 4  7 10 −9
 5 −1
⇒ Adj A = C′ =   Hence, option (d) is the correct answer.
 −3 4
Theorem: If A be any given square matrix of order n × n, then:
Hence, option (c) is the correct answer.
A(adj A) = (adj A) A = |A| I. Here I is a unit matrix of order n × n.
 1 4 3
 2 −1 4 Theorem: If A is a square matrix then: (adj A)′ = (adj A′)
Example 3. If matrix A =   , then its adjoint will be
 3 2 5 Theorem: If A and B be two non singular square matrices of same
order then adj (AB) = (adj B). (adj A).
 2 −4 2 13 14 11
Theorem: If A is non-singular square matrix of order 3 × 3, then
(a) 7 10 8 (b)  7 4 2
    |adj A| = |A|2 and in general if A is non-singular square matrix of
 3 −7 5  8 7 9
order n × n, then |adj A| = |A|n – 1

Determinants 83
E:\AMIT_WORKS\Exam_Guru\EG_Mathematics-12_(working_02-06-2022)\EG_Mathematics-12_working\Open_Files\Chap_4\Chap_4
\ 16-Aug-2022 Amit Proof-5 Reader’s Sign _______________________ Date __________

EXERCISE 4.3
I. Multiple Choice Questions (MCQs)
 2 3 1
Choose the correct answer from the given options. A =  1 2 −1
5. If   is a non-singular square matrix of
2 3 1  3 4 2
 
1. If matrix A = 1 4 2 , then (adj A) equals
order 3 × 3, then show that |adj A| = |A|2
 3 7 5
cos x sin x 1 0
6 8 −2  5 6 4 6. If A = and A(adj A) = k   , then find
  − sin x cos x 0 1
(a)  1 3 7  (b)  3 2 7 
  the value of k.
 5 4 5  −1 2 4
 6 −8 2  3 8 −7  III. Long Answer Type Questions
 
(c)  1 7 −3 (d)  3 2 4  1 −1 1

 −5 −5 5  −5 5 5  
1. If A =  0 2 −3 and B = (adj A), and C = 5A, then what
II. Short Answer Type Questions-I  2 1 0

 2 5 1 | adj B |
is the value of ?
1. If matrix A =  −1 2 4 , then verify |C|
 
 4 3 5 2. If k is a scalar and I is a unit matrix of order 3, then find
A (adj A) = (adj A) A = | A | I. the value of adj (kI).
 2 5 2 10 0 
2. If matrix A =  3 1 5 , then verify 3. For any 2 × 2 matrix A, if A(adj A) =   , then
   0 10
 4 −1 −3 find |A|.

(adj A′) = (adj A)′ 3 1


4. If A =  2 −3 , then find |adj A|.
 
 1 2 3  3 1 2
 
3. If matrix A =  1 0 2 and B =  2 5 0 , then  2 0 0
  5. If A =  0 2 0 , then find the value of |adj A|.
 −1 1 −1  4 3 5  0 0 2
verify adj (AB) = (adj B) . (adj A)
6. A is a square matrix of order 3 and |A| = 7. Find the value
 3 7 2 of |adj A|.
 
4. If matrix A =  4 5 3 , then find (adj A) A without
7. If A is a square matrix of order 3 such that |Adj A| = 64,
7 −1 4
then what is the value of |A|?
finding (adj A).
Answers 4.3
3. (adj A) . (adj B)
 6 −8 2
  4. |A| = 26, and (adj A) A = |A| I = 26 I.
I. 1. (c) (adj A) =  1 7 −3
 −5 −5 5  8 −2 −5
5. | A | = 1 and (adj A) =  −5 1 3
 −2 −22 18 
 6 −9  −2 1 1
II. 1. |A| =90; (adj A) =  21
 −11 14 9 and | (adj A) | = 1 ⇒ | (adj A) | = |A|2 = 1.
cos x sin x
1 0 0 6. |A| =
 0 1 0 − sin x cos x
and A(adj A) =(adj A) A = 90  
0 0 1 = cos2 x + sin2 x = 1
A (adj A) =(adj A) A = 90 I = | A |I Since (adj A)A = |A| I2
 2 29 −7 
⇒ (adj A)A = 1I = I
  But A(adj A) = kI
2. (adj A′) =(adj A)′ =  13 −14 22
 23 −4 −13
\ kI = I ⇒k=1

84 Mathematics–12
E:\AMIT_WORKS\Exam_Guru\EG_Mathematics-12_(working_02-06-2022)\EG_Mathematics-12_working\Open_Files\Chap_4\Chap_4
\ 16-Aug-2022 Amit Proof-5 Reader’s Sign _______________________ Date __________

1 −1 1 1 0 0
III. 1. |A| = 0 2 −3 2. Let I =  0 1 0
2 1 0 0 0 1
= 1(0 + 3) – 0 + 2(3 – 2) = 3 + 2 = 5
1 0 0  k 0 0 
Now, A11 = 3, A12 = – 6, A13 = – 4;

\ kI = k 0 1 0 =  0 k 0 
A21 = 1, A22 = – 2, A23 = – 3; 0 0 1  0 0 k 
A31 = 1, A32 = 3, A33 = 2
 3 1 1 k 2 0 0  1 0 0
adj (kI) = 0 k 0  = k 0 1 0
 2 2
B= adj A =  − 6 − 2 3

 − 4 − 3 2 2 0 0 1
0 0 k 
B11 = 5, B12 = 0, B13 = 10,
= k2I
B21 = –5, B22 = 10, B23 = 5,
B31 = 5, B32 = –15, B33 = 0 10 0 
3. A(adj A) = 
 5 −5 5  0 10
adj B =  0 10 −15
10 5 0 1 0

⇒ A(adj A) = 10  = 10 I
0 1
 1 −1 1
= 5  0 2 −3 = 5A We know that A(adj A) = (adj A) A = |A| I
 2 1 0

\ We get |A| = 10

\ adj B = C ⇒ |adj B| = |C|
4. –11 5. 64
| adj B |

\ =1 6. 49 7. ±8
|C|

Topic 4. Inverse of a Non-Singular Square Matrix


We will discuss the concept of inverse of a non-singular square 3 4 
matrix. If A and B are two non-singular square matrices of same Solution. A=   ⇒ | A | = 21 – 4 = 17
order and they are such that AB = BA = I, then matrix A is inverse 1 7 
of matrix B and matrix B is inverse of matrix A. Inverse of a ⇒ Matrix A is non-singular and hence its inverse exists. Let
matrix A is denoted as A–1. us find its cofactors.
Now we know that: (adj A) A = A(adj A) = | A | I A11 = + (7) = 7,
 1    A12 = – (1) = –1,
⇒ 
A
(adj A ) A = A  A1 (adj A ) A21 = – (4) = – 4,
   
1 A22 = +(3) = 3.

A
(adj A ) is inverse of matrix A.  7 −1
⇒ Cofactor’s matrix C =  
Hence A–1 = 1
(adj A )  −4 3
A
 7 −4
Also: (i) AI = IA = A ⇒ adj A = C′ =  
 −1 3
(ii)A–1I = IA–1 = A–1
1 1  7 −4
(iii)AA–1 = I and A–1A = I. ⇒
A
( adj A ) = 
A–1 =
17  −1 3
Notes: (i)If a square matrix A has inverse, it is called invertible
Hence, option (b) is the correct answer.
matrix.
(ii) A square matrix A is invertible if and only if matrix A  2 3 1
is a non-singular matrix. Example 2. If a square matrix A =  1 4 5 , then find A–1
 
Theorem: Inverse of non-singular square matrix is unique.  −1 2 3
3 4  –1 and verify that AA–1 = A–1A = I
Example 1. If non-singular matrix A =   , then A equals
1 7   2 3 1
1  −7 −3 1  7 −4 Solution. A =  1 4 5
(a) (b)  
13  −1 4 17  −1 3  −1 2 3
1  −4 −3 1  2 3 ⇒ | A | = 2(12 – 10) – 3(3 + 5) + 1(2 + 4)
(c) (d)
13  2 7  17  5 7  = 2×2–3×8+1×6

Determinants 85
E:\AMIT_WORKS\Exam_Guru\EG_Mathematics-12_(working_02-06-2022)\EG_Mathematics-12_working\Open_Files\Chap_4\Chap_4
\ 16-Aug-2022 Amit Proof-5 Reader’s Sign _______________________ Date __________

⇒ | A | = 4 – 24 + 6 = –14 On pre-multiplying A–1 on both sides of the above equation, we


⇒ Matrix A is non-singular and hence A exists. –1 have
 2 3 1 A–1(AB)(AB)–1 = A–1I
Let us find cofactors of the matrix A =  1 4 5 ⇒ (A–1A)B(AB)–1 = A–1
 
[Q Multiplication of matrices is associative and A–1I = A–1]
 −1 2 3
Cofactors of matrix A ⇒ IB(AB)–1 = A–1[Q A–1A = I]
⇒ B(AB)–1 = A–1[Q IB = B]
A11 = + (12 – 10) = 2 A12 = – (3 + 5) = –8
On pre-multiplying B–1 on both sides, we get
A13 = + (2 + 4) = 6 A21 = – (9 – 2) = – 7
B–1B(AB)–1 = B–1A–1
A22 = + (6 + 1) = 7 A23 = – (4 + 3) = – 7
⇒ I(AB)–1 = B–1A–1[Q B–1B = I]
A31 = + (15 – 4) = 11 A32 = – (10 – 1) = – 9
⇒ (AB)–1 = B–1A–1
A33 = + (8 – 3) = 5 Hence, (AB)–1 = B–1A–1.
 2 −8 6 3 4  2 1


C =  −7 7 −7  Example 3. If A =   and B =   are two square
1 5 3 7
 11 −9 5 matrices of same order, then verify that (AB)–1 = B–1A–1
 [V. Imp.]
 2 −7 11
   3 4   2 1 
⇒ (adj A) = C′ =  −8 7 −9 Solution. A=   , B = 3 7
1 5   
 6 −7 5
3 4  2 1  18 31
 2 −7 11 ⇒ AB =   = 
1 1  1 5  3 7  17 36
⇒ (A–1) =
A
(adj A ) = −14  −8 7 −9 ⇒ |AB| = 648 – 527 = 121
 6 −7 5
1  36 −31
⇒ (AB)–1 = ...(1)
 2 −7 11 121  −17 18
−1  
⇒ A–1 = 14  −8 7 −9 | B | = 14 – 3 = 11
 6 −7 5
1  7 −1
⇒ B–1 = , | A | = 11
11  −3 2
 2 3 1  2 −7 11
AA–1 = −1  1 4 5  −8 7 −9 1  5 −4
14   
⇒ A–1 =
 −1 2 3  6 −7 5 11  −1 3

 −14 0 0 1 0 0 1  7 −1  5 −4


−1  0 0 = 0 1 0 = I B–1A–1 =
= −14 11 × 11  −3 2  −1 3
14 
 0 0 −14 0 0 1 
Theorem: If A and B are two invertible matrices of same order, 1  36 −31
= ...(2)
then (AB)–1 = B–1A–1. 121  −17 18
Proof: From the definition of inverse of a matrix, we have: (AB) From (1) and (2), we get
(AB)–1 = I (AB)–1 = B–1A–1.
EXERCISE 4.4
I. Multiple Choice Questions (MCQs) 1
(a) det A (b)
Choose the correct answer from the given options. det A

1. Let A be a non-singular matrix of order (3 × 3). Then |adj (c) 1 (d) 0


A| is equal to
 2 3
(a) |A| (b) |A|2 3. If A =  1 −2 and A–1 = aA, then the value of a is
 
(c) |A|3 (d) 3|A|
(a) 7 (b) – 7
2. If A is an invertible matrix of order (2 × 2), then det (A–1) 1
(c) 1 (d) − .
is equal to 7 7

86 Mathematics–12
E:\AMIT_WORKS\Exam_Guru\EG_Mathematics-12_(working_02-06-2022)\EG_Mathematics-12_working\Open_Files\Chap_4\Chap_4
\ 16-Aug-2022 Amit Proof-5 Reader’s Sign _______________________ Date __________

 2 3 –1 3 4  7 8 
4. If A =   , then 19A is equal to 2. Find matrix [A]2 × 2, if A  = 2
.
 5 −2 1 2 1
(a) A (b) 2A
 3 2   2 5  7 2 
1
(c) A (d) 3A
3. Find matrix [A]2×2 if  A =  
2 1 4  2 3  − 4 3

−1 III. Long Answer Type Questions


 x y3 1 8  x y
5. If A =   =   , then 2 0  =
2 0   2 0    3 1  4 5 –1 –1 –1
1. If A =   , B=   , then verify (AB) = B A .
 4 0  3 4
 0 −2 1 0
(a)   (b)    5 0 4 1 3 3
 −2 1 0 1    
2. If A =  2 3 2 and B−1 = 1 4 3 , find (AB)–1.
 0 1
 0 −8  2 1 2 1  1 3 4
(c)   (d) 
 −2 1 1 1
 −  1 3 3
2 4
3. If A = 1 4 3 , then verify A–1A = I
 2 2 0 −1  
6. If A =   and B =
–1 –1 –1
 1 0 , then (B A ) = 1 3 4
 −3 2  
1 −1 1 
 2 −2  3 −2
(a)   (b)   4. If A =  2 −1 0 , then A 2 = A–1.
 2 3  2 2  
 1 0 0
 2 2
1 1  3 2
(c)  −2 3 (d)  −2 2  1 tan x 
10 10 5. If A =  , then show that A′A –1 =
   
 − tan x 1 
7. If A is an invertible matrix of order 2, then det (A –1) is
equal to cos 2x − sin 2x 
 sin 2x cos 2x 
(a) det A (b) 1 (c) 1 (d) 0  
det (A)  1 − 1 1
0 1 2  2 2 2
II. Short Answer Type Questions   
6. If A = 1 2 3 and A–1 =  − 4 3 c  , then show that

 2 −1  5 2  2 5  3 a 1   5 3 1
1. Let A =   , B=   and C =   . Find the  2
 3 4 7 4  3 8 2 2 
a = 1, c = –1.
matrix D such that CD – AB = 0.
Answers 4.4
I. 1. (b) |A|2 4. (a) A
Hint: If A is a matrix of order n × n then  2 3
A=  , then
|adj A| = |A|n – 1  5 −2
1
2. (b) 2 3
det A |A| = 5 −2 = – 4 – 15 = – 19 ≠ 0
3. (c) 1
7 A A   −2 −3
  adj A =  A11 A 21  = 
a b   d −b   12 22   −5 2
Adjoint of the matrix  c d  is 
   −c a 
adj. A 1  −2 −3 1  2 3
 −2 −3 \ A–1 = = =
\ adj A =  |A| − 19  −5 2 19  5 −2
 −1 2
 2 3
2 3 ⇒ 19A–1 =  5 −2 = A
Also |A| = =–4–3=–7≠0  
1 −2
adj A 1  −2 −3  0 1
⇒ A–1 = =  2
|A| −7  −1 2 5. (d) 
1 1
1  2 3  − 
= 2 4
7  1 −2
 x y3  1 8
1 Given that   = 
⇒ a=  2 0   2 0
7

Determinants 87
E:\AMIT_WORKS\Exam_Guru\EG_Mathematics-12_(working_02-06-2022)\EG_Mathematics-12_working\Open_Files\Chap_4\Chap_4
\ 16-Aug-2022 Amit Proof-5 Reader’s Sign _______________________ Date __________

By equality of matrices, we get


 −1 8 −12 1 3 3
A −1 = −  0 1 −2 , B− 1 = 1 4 3
1
x = 1, y3 = 8 ⇒ y = (8)3 = 2 2.
 x y 1 2  1 −10 15 1 3 4
Now,  2 0  = 
   2 0
 2 − 19 27 
1 2  0 −2
\ adj 
=  (AB) − 1 = B− 1A − 1 = −  2 − 18 25
 2 0 
  −2 1
 3 − 29 42
1 2 1 2
Also determinant  = 2 0 =0–4=–4
 2 0  7 −3 −3
−1 −1 3. A −1
=  −1 1 0 and A − 1A = I
 x y 1 2 1  0 −2

\ =  =  −1 0 1
 2 0   2 0 −4  −2 1

 0 1 0 0 1
 
=  1 12  4. A −1
= A = 0 −1 2
2

 2 − 4   1 −1 1

6. (a)  2 −2  1 − tan x   1 − tan x 


 2 3 A′A −1 = 
5.  × 12 
 tan x 1  sec x  tan x 1 
(B–1 A–1)–1 = (A–1)–1 (B–1)–1 = AB
 2 2 0 −1 0 1 2
=  − 3 2  1 0 6. |A| = 1 2 3
  
3 a 1
 0 + 2 − 2 + 0  2 − 2
=  0 + 2 3 + 0 =  = 0 – 1(1 – 9) + 2(a – 6) = 2a – 4
   2 3
1 The cofactors of different elements of |A|
7. (b)
det (A) A11 = 2 – 3a, A12 = 8, A13 = a – 6,
Since A is invertible matrix, therefore, AA–1 = I
A21 = 2a – 1, A22 = –6, A23 = 3
Now |AA–1| = | I | = 1 ⇒ |A| |A–1| = 1
1 A31 = –1, A32 = 2, A33 = – 1
⇒ | A–1 | =
|A|

\ A–1 = 1
Adj. A
–1 1 A
i.e., det (A ) =
det (A)
 2 − 3a 2a − 1 −1
II. 1. Hint: CD = AB 1  8
= 6 2  ...(1)
⇒ C – 1CD = C– 1(AB) ⇒ D = C – 1 (AB) ( 2a − 4)  a − 6 3 −1
−1
 7 8  3 4 
2. Hint: A–1 =     1 − 1 1
1 2 1 2  2 2 2
Given A–1 =  −4 3 c  ...(2)
7 8  1  2 −4  5 3 1
=  ×    2 − 2 2 
1 2 2  −1 3

 2 5 3 2 
−1
 7 2 −1 = 1
From (1) and (2), we get  [On comparing a13]
3. Hint: A   =    − 4 3 2a − 4 2
 2 3  1 4  
2 =c
and  [On comparing a23]
1  4 −2  7 2 1  36 2 2a − 4
= × =
10  −1 3  −4 3 10  −19 7  Now, –2 = 2a – 4
−1
⇒ 2a = 2
1  36 2  2 5 −1  3 −5
and A= =
10  −19 7   2 3 40  −2 3
⇒ a = 1 and 2
=c
2 (1) − 4
15 19   20 −19
III. 1. AB =   , (AB) −1 = −1  2
16 20 4  −16 15

−2
= c

 4 − 5 − 1 − 1  0 − 1
⇒ c = –1
B–1 =   , A = 4 − 4 3 
− 3 4    Hence, a = 1 and c = –1.

88 Mathematics–12
E:\AMIT_WORKS\Exam_Guru\EG_Mathematics-12_(working_02-06-2022)\EG_Mathematics-12_working\Open_Files\Chap_4\Chap_4
\ 16-Aug-2022 Amit Proof-5 Reader’s Sign _______________________ Date __________

Topic 5. Applications of Matrices


Here we shall discuss to solve the system of linear equations in On putting the values of A–1 and B in X = A–1B, we get
two and three variables and shall also discuss how to check the −1  −1 −2 12
consistency of the system of linear equations. ⇒ X=
7  −2 3  1 
Consistent System of Linear Equations: A system of
linear equations is said to be consistent if its solution (one or −1  −12 − 2 −1  −14
⇒ X= =
more) exists. 7  −24 + 3  7  −21
Inconsistent System of Linear Equations: A system of linear  2  x   2
equations is said to be inconsistent if its solution does not exist. ⇒ X =   ⇒   =   ⇒ x = 2, y = 3
 3  y   3
Solution of system of Linear Equations Using Inverse of Example 2. Using matrix method, find the value of 2x + y – z for
a Matrix: Take a system of linear equations in three variable:
 5x − y + z = 4
 a1x + b1y + c1z = d1 
the following linear equations 3x + 2y − 5z = 2 .

a2 x + b2 y + c2 z = d 2  x + 3y − 2z = 5

a x + b y + c z = d
 3 3 3 3  5x − y + z = 4

This can be expressed in matrix form as: Solution. The given system of equation is: 3x + 2y − 5z = 2
 a1 b1 c1   x   d1   x + 3y − 2z = 5

a b c   y  = d  1  x   4
 2 2 2    2 5 −1
 a3 b3 c3   z   d3      
AX = B   ⇒ 3 2 −5  y  =  2
 a1 b1 c1   x  d1   1 3 −2  z   5
 a b c  , X =  y  and B =  d  5 −1 1
Let A =  2 2 2     2 ⇒ X = A B , A = 3 2 −5
–1 
 a3 b3 c3   z   d3   
 1 3 −2
Then the above matrix equation can be written as: AX = B . ⇒ | A | = 5 (–4 + 15) + 1(–6 + 5) + 1(9 – 2)
Case I: If A is a non-singular matrix then its inverse exists. = 5 × 11 – 1 + 7 = 61 ≠ 0
On pre-multiplying A–1 on both sides of AX = B, we get: ⇒ Matrix A is non-singular and hence its inverse exists.
Cofactors of matrix A are:
A–1(AX) = (A–1B) ⇒ (A–1A)X = A–1B
A11 = + (– 4 + 15) = 11 A12 = – (– 6 + 5) = 1
[Q Matrix multiplication is associative]
A13 = + (9 – 2) = 7 A21 = – (2 – 3) = 1
⇒ IX = A B[Q A–1A = I]
–1
A22 = + (– 10 – 1) = –11 A23 = – (15 + 1) = – 16
⇒ X = A −1B [Q IX = X] A31 = + (5 – 2) = 3 A32 = – (– 25 – 3) = 28
A33 = + (10 + 3) = 13
This equation will give you a unique solution of the given
system of linear equation if A–1 exists. This method of solving 11 1 7 11 1 3
 1 −11 −16  1 −11 28
system of linear equations is called Matrix Method. ⇒ C=   ⇒ (adj A) = C′ =  
Case II. If A is a singular matrix, then | A | = 0. In this case  3 28 13  7 −16 13
A–1 does not exists. Now calculate [(adj A) B]. 11 1 3
If [(adj A) B] ≠ 0, then the system of linear equations has 1 1 
no solution. This type of system of linear equations is called
⇒A = –1
A
(adj A ) = 61  1 −11 28
inconsistent system of linear equations.  7 −16 13
If [(adj A) B] = 0, then the system of equations is consistent On putting matrices A–1 and B in X = A–1B, we get
with infinite number of solutions. 11 1 3  4
1 
Example 1. Using matrix method, find the values of x and y for ⇒ X= 1 −11 28  2
61 
3x + 2y = 12  7 −16 13  5
the following linear equations  .
 2x − y = 1  44 + 2 + 15   61 
3x + 2y = 12 1   1  
Solution. The given equations are:  ⇒ X= 4 − 22 + 140  = 122
61  61
 2x − y = 1  28 − 32 + 65  61 
 3 2  x  12 −1 1   x 1 
⇒  2 −1  y  =  1  ⇒ AX = B ⇒ X = A B      
     ⇒ X = 2 ⇒  y  =  2 ⇒ x = 1, y = 2, z = 1.
 
 3 2 −1  −1 −2 1   z  1 
Here A=   ⇒ A −1 =
 2 −1 7  −2 3  2x + y – z = 2 (1) + 2 – 1 = 2 + 2 – 1 = 3

Determinants 89
E:\AMIT_WORKS\Exam_Guru\EG_Mathematics-12_(working_02-06-2022)\EG_Mathematics-12_working\Open_Files\Chap_4\Chap_4
\ 16-Aug-2022 Amit Proof-5 Reader’s Sign _______________________ Date __________

EXERCISE 4.5
I. Multiple Choice Questions (MCQs) 2. Using matrix method, solve the following system of
Choose the correct answer from the given options.  2x − 3y + 5z = 11
1. Solution of system of equations 2x + 8y + 5z = 5, 
equations: 3x + 2y − 4z = −5
x + y + z = –2 and x + 2y – z = 2 is  x + y − 2z = − 3
(a) x = –3, y = 2, z = –1 (b) x = –3, y = –2, z = 1 
[Delhi 2011] [A.I. 2017]
(c) x = 3, y = –2, z = –1 (d) None of these
II. Short Answer Type Question-I  2 3 10
x+ y + z = 4
 2x + 5y = 1 
1. Solve by matrix method:   4 6 5
3x + 2y = 7 3. Solve by matrix method:  − + = 1
 x y z
[NCERT Exemplar]  6 9 20
1 2 1  x + y − z = 2
 
2. If P = 1 −1 −2 , find P −1 , and solve the following [NCERT Exemplar] [Delhi 2011, 2017]
1 1 3  x + y + z = 3
 1 1 1 
system of equations:  2x − y + z = 2  
4. If A = 1 0 2 , find A–1 and use A–1 to solve the
 x − 2y + 3z = 2

3 1 1   x+ y+ z = 6
3. Using matrix method examine the consistency of the 
following system of equations  x + 2z = 7
following system of equations:
3x + y + z = 12
2x + 3y = 10 
(i)  [Delhi 2011, 2012 (C), AI 2011, 2012 (C)]
 7x − 4y = 6
3x + 2y = 16  − 4 4 4  1 −1 1
(ii)  
5. If P = −7 1 3 and A =  1 −2 −2 , find
 
6x − 4y = 14 
 5 −3 −1  2 1 3
 3x + 5y = 7
(iii) 
9x + 15y = 21 PA and use this to solve the following system of
4. Using matrix method, examine the consistency of the  x− y+z = 4

following system of equations: equations  x − 2y − 2z = 9 .
 x + y − 2z = 1  2x − y + z = 4  2x + y + 3z = 1

 
(i)  2x + 3y − 5z = 3 (ii)  x + 2y + 3z = 2 [Delhi 2011, 2012 (C), AI 2011, 2012]
3x − 4y + z = −4 3x − 2y + z = 5
   1 −1 2  −2 0 1
5. Two schools A and B decided to award prizes to their 0 2 −3  9 2 −3
6. Use product    to solve the
students for three values, team, spirit, truthfulness  3 −2 4  6 1 −2
and tolerance at the rate of ` x, ` y and ` z per student
respectively. School A, decided to award a total of ` 1,100 system of equations:
for the three values to 3, 1 and 2 students respectively while x + 3z = 9, –x + 2y – 2z = 4, 2x – 3y + 4z = –3.

school B decided to award ` 1,400 for the three values to 1,
[Delhi 2017]
2 and 3 students respectively. If one prize for all the three
values together amount to ` 600, then  2 3 1
(i) represent the above situation by a matrix equation after  
7. If A =  1 2 2 find A–1 and hence solve the system
forming linear equations.  −3 1 −1
(ii) is it possible to solve the system of equations so of equations:
obtained using matrices?
2x + y – 3z = 13, 3x + 2y + z = 4, x + 2y – z = 8.
(iii) which value you prefer to be awarded most and why?
[Delhi 2015 (C)] [Delhi 2017]
III. Long Answer Type Questions  −4 4 4  1 −1 1
1. Using matrix method, solve the following system of 8. Determine the product:  −7 1 3  1 −2 −2 and
  
 x + 2y − 3z = −4  5 −3 −1  2 1 3

equations:  2x + 3y + 2z = 2 [A.I. 2012] use it to solve the system of equations:
3x − 3y − 4z = 11
 x – y + z = 4, x – 2y – 2z = 9, 2x + y + 3z = 1. [A.I. 2017]

90 Mathematics–12
E:\AMIT_WORKS\Exam_Guru\EG_Mathematics-12_(working_02-06-2022)\EG_Mathematics-12_working\Open_Files\Chap_4\Chap_4
\ 16-Aug-2022 Amit Proof-5 Reader’s Sign _______________________ Date __________

9. A school wants to award its students for the value of (ii) solve these equations using matrices.
honesty, regularity and hard-work with a total cash award (iii) which values are reflected in this question?
of ` 6000. The three times award money for hard work [A.I. 2013 (C)]
added to that given for the honesty amount to ` 11000. The 13. Two schools P and Q want to award their selected students
award money given for honesty and hard-work together is on the values of discipline, politeness and punctuality. The
double the one given for regularity. Represent the above school P wants to award ` x each, ` y each and ` z each
situation algebraically and find the award money for each for the three respectively values to its 3, 2 and 1 students
value, using matrix method. with a total award money of ` 1000. School Q wants
10. The management committee of a residential colony to spend ` 1,500 to award its 4, 1 and 3 students on the
decided to award some of its members (say x) for honesty, respective values (by giving the same award money for
some (say y) for helping others and some others (say z) the three values as before) If the total amount of awards
for supervising the workers to keep the colony neat and for one prize on each value is ` 600, using matrices, find
clean. The sum of all the awardees is 12. Three times the the award money for each value. [Delhi 2014]
sum of awardees for cooperation and supervision added 14. Two schools A and B want to award their selected students
to two times the number of awardees for honesty is 33. on the values of sincerity, truthfulness and helpfulness. The
If the sum of the number of awardee for honesty and school A wants to award ` x each, ` y each and ` z each,
supervision is twice the number of awarded for helping for the three respectively values to 3, 2 and 1 students
others, using matrix method, find the number of awardee respectively with a total award money of ` 1,600. School
of each category.  [A.I. 2013] B wants to spend ` 2,300 to award its 4, 1 and 3 students
11. Two institutions decided to award their employees for on the respective values (by giving the same award money
the three values of resourcefulness, competence and to the three values as before).
determination in the form of prizes at the rate of ` x, ` y If the total amount of award for one prize on each value is
and ` z respectively per person. The first institution decided ` 900, using matrices find the award money for each value.
to award respectively 4, 3 and 2 employees with a total  [A.I. 2014]
prize money of ` 37000 and the second institution decided 15. A total amount of ` 7,000 is deposited in three different
to award respectively 5, 3 and 4 employees with a total saving bank accounts with annual interest 5%, 8% and
prize money of ` 47000. If all the three prizes per person 1
8 % respectively. The total annual interest from these
together amount to ` 12000. Then using matrix method, 2
three accounts is ` 550. Equal amounts have been deposited
find the values of x, y and z.[Delhi 2013 (C)]
in the 5% and 8% saving account. Find the amount
12. Two factories decided to award their employees deposited in each of the three accounts with the help of
for three values of (a) adaptable to new techniques matrices.[Delhi 2014 (C)]
(b) careful and alert in difficult situations and (c) keeping
16. Two schools P and Q want to award their selected students
calm in tense situations at the rate of ` x, ` y and ` z per
for values of sincerity, truthfulness and hard work at the rate
person respectively. The first factory decided to honour
of ` x, ` y, and ` z for each respective value per student.
respectively 2, 4 and 3 employees with a total prize
School P awards its 2, 3 and 4 students on the above
money of ` 29,000. The second factory decided to honour respective values with a total prize money of ` 4,600.
respectively 5, 2 and 3 employees with the prize money School Q wants to award is 3, 2 and 3 students on the
of ` 30,500. If the three prizes per person together cost respective values with a total award money of ` 4,100.
` 9,500. Then If the total amount of award money for one prize on each
(i) represent the above situation by matrix equations and value is ` 1500, using matrices find out the award money
form linear equations for each value.  [A.I. 2014 (C)]

Answers 4.5
I. 1. (a) x = – 3, y = 2, z = – 1 3. (i) Given system of equations is consistent with unique solution.
II. 1. x = 3, y = –1; Hint: | A | = –29 ≠ 0.

 2 −5
Hint: A
−1
= −1  (ii) Given system of equations is consistent with unique solution.
11  −3 2
Hint: | A | = –24 ≠ 0

x = 1  −1 −5 −3 (iii) G iven system of equations is consistent with infinite
2. y = 1 Hint: P −1 = −1  −5 2 3 solutions.
9  
z = 1  2 1 −3 Hint: | A | = 0 and (Adj A)B = 0.

 −1 −5 2 4. (i) G iven system of equations is consistent with infinite
−1 A = P( )−1 ′
= −1  −5 2
9
1 solutions.
 −3 3 −3 Hint: | A | = 0 and (Adj A)B = 0.

Determinants 91
E:\AMIT_WORKS\Exam_Guru\EG_Mathematics-12_(working_02-06-2022)\EG_Mathematics-12_working\Open_Files\Chap_4\Chap_4
\ 16-Aug-2022 Amit Proof-5 Reader’s Sign _______________________ Date __________

(ii) Given system of equations is inconsistent with no solution. 8. x = 3, y = –2, z = –1


Hint: | A | = 0 and (Adj A) B ≠ 0.  −4 4 4  1 −1 1
5. Ans. x = ` 100, y = ` 200, z = ` 300  1 3  1 −2 −2
and BA =  −7
Hint: Equations are  5 −3 −1  2 1 3
3x + y + 2z = 1100
= 8I
x + 2y + 3z = 1400
1
x + y + z = 600 ⇒ A–1 = B , Now solve.
8
 −1 1 −1 9. x = ` 500, y = ` 2000 and z = ` 3500.
1
A = −  2
−1
1 −7 
3 x + y + z = 6000
 1 −2 5 
Hint: Equations are: x + 0y + 3z = 11000

 −6 17 13 x − 2y + z = 0
III. 1. x = 3, y = – 2, z = 1; Hint: A −1 = 1  14 5 −8
67    6 −3 3
 −15 9 −1
and A −1 = 1  2 0 −2
6 
 0 1 −2  −2 3 −1
2. x = 1, y = 2, z = 3 Hint: A −1
=  −2 9 −23 x = 3
 −1 5 −13 
10. y = 4
x = 2  75 150 75 z = 5   x + y + z = 12
 1 
3. y = 3 Hint: A = −1
110 −100 30

Hint: Equations are: 2x + 3y + 3z = 33
1200 
z = 5  72 0 −24  x − 2y + z = 0

x = 3  −2 0 2  9 −3 0
1
=  5 −2 −1

4. y = 1 Hint: A −1 1
and A =  1 0 −1
−1
4 3 
z = 2  1 2 −1  −7 3 1
x=3  x = ` 4000
 
5. y = − 2 Hint: PA = 8 I 11. y = ` 5000
z = − 1  z = ` 3000 

⇒ PAA–1 = 8IA–1 ⇒ PI = 8A–1 x + y + z = 12000



1 Hint: Equations are: 4x + 3y + 2z = 37000

⇒ 8A–1 = P ⇒ A–1 = P. 5x + 3y + 4z = 47000
8
 −4 4 4  6 −1 −1
1
\ A −1 =  −7 1 3 and A −1 = −1  −6 −1 2
8 3  
 5 −3 −1  −3 2 −1
 1 −1 2  2 0 1 x = ` 2500
6. x = 0, y = 5, z = 3 and 0 2 −3  −3 2 −3 
   12. y = ` 3000
 3 −2 4  4 1 −2 z = ` 4000
x + y + z = 9, 500
1 0 0 
PB = 0 1 0 Hint: Equations are: 2x + 4y + 3z = 29, 000

 
0 0 1 5x + 2y + 3z = 30, 500


⇒ PB = I ⇒ B = P–1  −6 1 1

−1
and A =  −9 2 1
−1
 1 0 3
 16 −3 −2
⇒ −1 2 −2
 = (P–1)′
  x = ` 100
 2 −3 4 
13. y = ` 200
7. x = 1, y = 2, z = –3 z = ` 300 
 −4 4 4 3x + 2y + z = 1000
1  5
–1
A = − − 1 −3 Hint: Equations are: 4x + y + 3z = 1500
16
 7 −11 1 x + y + z = 600

92 Mathematics–12
E:\AMIT_WORKS\Exam_Guru\EG_Mathematics-12_(working_02-06-2022)\EG_Mathematics-12_working\Open_Files\Chap_4\Chap_4
\ 16-Aug-2022 Amit Proof-5 Reader’s Sign _______________________ Date __________

 −2 −1 5 x = ` 1125
and A −1 =
−1  −1 2 −5 15. y = ` 1125 
5  
 3 −1 −5 z = ` 4750 
Hint: Equations are:

x = ` 200
 x + y + z = 7, 000
14. y = ` 300   17 −1 1
 1 
z = ` 400  −1
10x + 16y + 17z =1,10, 000 and A = 17 −1 −7 
 8
x − y= 0  −26 2 6
3x + 2y + z = 1, 600 
 x = ` 500
Hint: Equations are: 4x + y + 3z = 2, 300

16. y = ` 400 
x + y + z = 900 
z = ` 600 
 −2 −1 5 Hint: Equations are:

and A −1 = −1  −1 2 −5
5   x + y + z = 1, 500  1 −1 1
 3 −1 −5  −1 1  
2x + 3y + 4z = 4, 600 and A =  6 0 −2
2
This is same as Q. No. 10. 3x + 2y + 3z = 4,100  −5 1 1

Case Based Questions


1. Two schools A and B want to award their selected students (ii) With reference to Q1, inverse of matrix is given by
on the values of sincerity, truthfulness and helpfulness.  2 −1 5 
The school A wants to award ` x each, ` y each and ` z 1
(a) − 1 2 −5
each for the three respective values to 3, 2 and 1 students 5 
respectively with a total award money at ` 1600. School  3 −1 −5
B wants to spend ` 2300 to award its 4, 1 and 3 students  −2 −1 5 
on the respective values (by giving the same award money 1
(b) − −1 2 −5
to the three values as before). 5
 3 −1 −5

 −2 −1 5 
1 
(c)  −1 2 −5
5
 3 −1 −5

 −2 −1 5
1
If the total amount of award for one piece on each value (d) −1 2 5
is ` 900, then answer the following questions. 5
 3 −1 5
(i) Represent the above situation by a matrix equation.
(iii) The award money for sincerity amount to
 3 2 1  x  1600 
     (a) ` 300 (b) ` 200 (c) ` 400 (d) ` 500
(a)  4 1 3  y  =  2300 (iv) The total award money for sincerity and truthfulness
1 1 1  z   900  amount to
(a) ` 300 (b) ` 400 (c) ` 500 (d) ` 300
 3 2 1  x  1600 
(v) What is the award money for helpfulness?
(b) 4 2 3  y  =  2300

     (a) ` 200 (b) ` 500 (c) ` 300 (d) ` 400
1 1 1  z   900 
 3 2 1  x  1600 
3 2 1  x  1600   4 1 3  y  =  2300
Ans. (i) (a)     
    
(c) 9 1 3  y  =  2300 1 1 1  z   900 
1 1 1  z   900 
 −2 −1 5 
1
3 2 3  x  1600  (ii) (b) − −1 2 −5 (iii) (b) ` 200
5
(d) 9 2 1  y  =  2300
 3 −1 −5
    
1 1 1  z   900  (iv) (c) ` 500 (v) (d) ` 400

Determinants 93
E:\AMIT_WORKS\Exam_Guru\EG_Mathematics-12_(working_02-06-2022)\EG_Mathematics-12_working\Open_Files\Chap_4\Chap_4
\ 16-Aug-2022 Amit Proof-5 Reader’s Sign _______________________ Date __________

2. Gaurav purchases 3 pens, 2 bags and 1 instrument box and 3. Manjit wants to donate a rectangular plot of land for
pays ` 41. From the same shop, Dheeraj purchases 2 pens, a school in his village. When he was asked to give
1 bag and 2 instrument boxes and pays ` 29, while Ankur dimensions of the plot, he told that if its length is decreased
purchases 2 pens, 2 bags and 2 instrument boxes and pays by 50 m and breadth is increased by 50 m, then its area
will remain same, but if length is decreased by 10 m and
` 44.
breadth is decreased by 20 m, then its area will decrease
by 5300 m2. Based on the information given above, answer
the following questions:

(i) The equations in terms of x and y are


(a) x – y = 50, 2x – y = 550
(b) x – y = 50, 2x + y = 550
Read the above information, answer the following (c) x + y = 50, 2x + y = 550
questions. (d) x + y = 50, 2x + y = 550l
(i) Translate the given problem into a system of equations (ii) Which of the following matrix equation is represented
(a) 3x + 2y + z = 41 (b) 3x + 2y + 2z = 41 by the given information?
2x + y + 2z = 29 2x + y + 2z = 20  1 −1  x   50  1 1  x   50
(a)     =  (b)    =  
2x + 2y + 2z = 44 2x + 2y + 2z = 44 2 1  y  550  2 1  y  550
(c) 3x + 2y + z = 41 (d) 3x + 2y + z = 41  1 1  x   50  1 1  x   −50
2x + y + 2z = 29 2x + y + z = 29 (c)     =  (d)    =  
 2 −1  y  550  2 1  y   −550
x + 2y + 2z = 44 x + 2y + 2z = 44 (iii) The value of x (length of rectangular field) is
(ii) With reference to Q1, A–1 is equal to (a) 150 m (b) 400 m (c) 200 m (d) 320 m
− 2 − 2 3 − 2 −2 3 (iv) The value of y (breadth of rectangular field) is
1  (b) 1 
(a) − 0 4 − 4 0 4 − 4 (a) 150 m. (b) 200 m. (c) 430 m. (d) 350 m.
4  4 (v) How much is the area of rectangular field?
 2 − 2 − 1  2 − 2 − 1
(a) 60000 sq.m. (b) 30000 sq.m.
2 − 2 3 − 2 2 3 (c) 30000 sq.m. (d) 3000 sq.m.
1 1
(c) −  0

4 − 4 (d) 0 4 − 4 Ans. (i) (b) x – y = 50, 2x + y = 550
4 4
 2 − 2 − 1 
 2 2 − 1  1 −1  x   50
(ii) (a)  = (iii) (c) 200 m
(iii) The cost of one pen is equal to 2 1  y  550
(a) ` 2 (b) ` 5 (c) ` 10 (d) ` 15 (iv) (a) 150 m (v) (b) 30000 sq.m.
(iv) What is the combined cost of one pen and one bag? 4. Yash scored 40 marks in a test, getting 3 marks for each
right answer and losing 1 mark for each wrong answer.
(a) ` 12 (b) ` 15 (c) ` 17 (d) ` 25
Had 4 marks been awarded for each correct answer and 2
(v) What is the combined cost of one pen and one marks been deducted for each incorrect answer, then Yash
instrument box? would have scored 50 marks. Let number of right answer
(a) ` 7 (b) ` 12 (c) ` 17 (d) ` 25 is x and number of wrong answer is y.
Ans. (i) (a) 3x + 2y + z = 41
2x + y + 2z = 29
2x + 2y + 2z = 44
− 2 − 2 3
1 
(ii) (a) − 0 4 − 4  (iii) (a) ` 2
4
 2 − 2 − 1
(iv) (c) ` 17 (v) (a) ` 7

94 Mathematics–12
E:\AMIT_WORKS\Exam_Guru\EG_Mathematics-12_(working_02-06-2022)\EG_Mathematics-12_working\Open_Files\Chap_4\Chap_4
\ 16-Aug-2022 Amit Proof-5 Reader’s Sign _______________________ Date __________

Based on the information given above, answer the 5. Three shopkeepers Sanjeev, Rohit and Deepak are using
following questions: polythene bags, handmade bags (prepared by prisoners)
(i) The equations in terms of x and y are and newspaper’s envelope as carry bags. It is found that
the shopkeepers Sanjeev, Rohit and Deepak are using
(a) 3x – y = 40, 2x – y = 25
(20, 30, 40), (30, 40, 20) and (40, 20, 30) polythene bags,
(b) x – 3y = 40, x – y = 25
handmade bags and newspaper’s envelopes respectively.
(c) 3x + y = 40, 2x + y = 25 The shopkeepers Sanjeev, Rohit and Deepak spent ` 250,
(d) x + 3y = 40, 2x + 2y = 25 ` 270 and ` 200 on these carry bags respectively.
(ii) Which of the following matrix equations represent the Using the concept of matrices and determinants, answer
above information? the following questions:
1 3  x   40  3 1  x   40 (i) What is the cost of one polythene bag?
(a)     =  (b)    =   (a) ` 1 (b) ` 2 (c) ` 3 (d) ` 5
1 2  y   25  2 1  y   25
(ii) What is the cost of one handmade bag?
1 −3  x   40  3 −1  x   40 (a) ` 1 (b) ` 2 (c) ` 3 (d) ` 5
(c)    =   (d)    =  
1 −2  y   25  2 −1  y   25 (iii) What is the cost of one newspaper envelope?
(iii) Using matrix method, the number of right answer given (a) ` 1 (b) ` 2 (c) ` 3 (d) ` 5
by Yash is (iv) Keeping in mind the social condition, which shopkeeper
(a) 5 (b) 15 (c) 20 (d) 40 is best?
(iv) The number of wrong answer given by Yash is (a) Sanjeev (b) Rohit
(c) Deepak (d) None of these.
(a) 15 (b) 20 (c) 5 (d) 25
(v) Keeping in mind the environment conditions, which
(v) How many questions were there in the test?
shopkeeper is best?
(a) 20 (b) 25 (c) 35 (d) 40
(a) Sanjeev (b) Rohit
Ans. (i) (a) 3x – y = 40, 2x – y = 25 (c) Deepak (d) None of these.
 3 −1  x   40 Ans. (i) (a) ` 1 (ii) (d) ` 5
(ii) (d)     =   (iii) (b) 15
 2 −1  y   25 (iii) (b) ` 2 (iv) (b) Rohit
(iv) (c) 5 (v) (a) 20 (v) (a) Sanjeev

Author’s Comments
Questions based on following types are very important for Exams. So, students are advised to revise them thoroughly.
1. To find inverse of a non-singular square matrix, i.e. 2 × 2 or 3 × 3 order matrix.
2. To solve a system of linear equation of two variables or three variables. (Most Important)
3. To form the three linear equations of the given word problem and solving them.
4. To find the value of a determinant without expanding the determinant.

IMPORTANT FORMULAE
1. Area of a triangle: If A(x1, y1), B(x2, y2) and C(x3, y3) are the vertices of a DABC. Then the area of the triangle ABC
x1 y1 1
1
D= x y2 1
2 2
x3 y3 1
Note: Take positive value of D if the value of D comes negative.
2. If the three points A(x1, y1), B(x2, y2) and C(x3, y3) are collinear, then
x1 y1 1
x2 y2 1 = 0
x3 y3 1

 a11 a12 a13 
3. If matrix A =  a21 a22 a23  , then
 
 a31 a32 a33 

Determinants 95
E:\AMIT_WORKS\Exam_Guru\EG_Mathematics-12_(working_02-06-2022)\EG_Mathematics-12_working\Open_Files\Chap_4\Chap_4
\ 16-Aug-2022 Amit Proof-5 Reader’s Sign _______________________ Date __________

 A11 A 21 A31 
(i) Adjoint of a square matrix A = A12 A 22 A32  , where Aij is the cofactor of ijth element of matrix A.

 
 A13 A 23 A33 
(ii) A(adj A) = (adj A)A = |A|I (iii) (adj A)′ = (adj A′)
(iv) adj (AB) = (adj B) (adj A) [Matrices A and B are square matrices of same order]
(v) |adj A| = |A|n – 1 If A is a square matrix of order n.
4. Inverse of a square matrix A −1 = 1 [ adj A ]
A
(i) AI = IA = A (ii) AA–1 = I and A–1A = I (iii) A–1I = IA–1 = A–1
5. If A and B are two square matrices of the same order then (AB)–1 = B–1A–1.
a1x + b1y + c1z = d1

6. a2 x + b2 y + c2 z = d 2 is a system of linear equations.
a x + b y + c z = d
 3 3 3 3

 a1 b1 c1   x  d1 
Take A =  a2 b2 c2  ≠ 0 and X =  y  , B =  d 2  then solution of linear equations X = A −1B
     
 a3 b3 c3   z   d3 
 a11 a12 a13 
7. Value of the matrix A =  a21 a22 a23 
 a31 a32 a33 
= a11A11 + a12A12 + a13A13
= a21A21 + a22A22 + a23A23 Expanding along the Rows
= a31A31 + a32A32 + a33A33
= a11A11 + a21A21 + a31A31
= a12A12 + a22A22 + a32A32 Expanding along the Columns
= a13A13 + a23A23 + a33A33
where A11, A12, ..., are cofactors.

COMMON ERRORS
ERRORS CORRECTIONS
(i) Wrong notation for determinants (i) Determinant: | |
a1 + α1 a2 + α 2 a3 + α 3
(ii) b1 + β1 b2 + β 2 b3 + β3 (ii) The determinant will be broken up in any number of
c1 + γ 1 c2 + γ 2 c3 + γ 3 determinant. At a time, only one row/column can be
divided, rest of the rows/columns remains the same.
a1 a2 a3 α1 α 2 α 3
= b1 b2 b3 + β1 β 2 β3
c1 c2 c3 γ1 γ 2 γ 3

(iii) Directly try to expand (iii) By using rules of determinants try to make maximum
number of zeroes in a row or column and expand.
(iv) Students commit error in calculating the value of determnant (iv) Students are advised to calculate value of determinant of
of a matrix because they try it orally. a matrix by using two sticks.
(v) (a) For finding co-factor not taking proper sign. (v) Find co-factors by using (–1)i+j and remember to take
(b) Not taking transpose of a co-factor matrix. transpose.

96 Mathematics–12
E:\AMIT_WORKS\Exam_Guru\EG_Mathematics-12_(working_02-06-2022)\EG_Mathematics-12_working\Open_Files\Chap_4\Chap_4
\ 16-Aug-2022 Amit Proof-5 Reader’s Sign _______________________ Date __________

REVISION CHART

Determinant
The determinant of a square matrix is a real number associated with the matrix. Every square matrix has a determinant. i.e. To every square
matrix [aij] of order n, we can associate a number (real or complex) called determinant of the square matrix A, where ajj = (i, j)th element of A.
It is denoted by |A| or det. A, if the matrix is denoted by some alphabet (A, B, ...), otherwise it is denoted by D (Delta.)

Minor of an Element of a Matrix Co-factor


a11 a12 a13  Co-factor of aij element of a matrix is denoted by Aij and defined as Aij = (–1)i+j Mij.
If matrix A = a21 a22 a23  then a11 a12 a13 
a31 a32 a33  Value of the determinant of the matrix A = a21 a22 a23  is denoted by D.
minor aij is denoted by Mij. a31 a32 a33 
Mij is the determinant obtained by D = a11A11 + a12A12 + a13A13 (along first row)
eliminating ith row and jth column of In a similar way the value of the determinant can be calculated along 2nd row, 3rd
the matrix A. row, 1st column, 2nd column and 3rd column.

Area of a triangle
x1 y1 1
1
If A(x1, y1), B(x2, y2) and C(x3, y3) are vertices of a DABC. Then area of the DABC = x2 y2 1
2
x3 y3 1
(i) The value of this determinant is sometimes negative, take only positive value of this determinant.
(ii) If value of this determinant is zero, then the three points A, B and C are collinear.

Adjoint of a Square Matrix


The adjoint of a square matrix is defined as the transpose of the matrix of the cofactors of the matrix. It is denoted by (adj. A).

Inverse of a non-singular Properties of Adjoint


square matrix A : If A and B are square matrix of order n × n and |A| ≠ 0 and |B| ≠ 0, then
1 (i) A(adj A) = (adj A)A = | A |I. (ii) adj(AB) = (adj B) (adj A).
−1
A = (adj A)
A (iii) | adj A | = |A|n – 1 if A is a matrix of order n × n. (iv) (adj A)′ = (adj A′)
(v) adj(adj A) = |A|n – 2 A

Solution of Consistent System of Linear Equations


Take a system of linear equations in three variables:
a1x + b1y + c1z = d1

Applications of Matrices a2 x + b2 y + c2 z = d 2
a x + b y + c z = d
To solve the system of linear equations in two  3 3 3 3

and three variables and the consistency of the This can be written in matrix form as:
system of linear equations.
 a1 b1 c1   x   d1 
 a b c   y  = d 
 2 2 2    2
a3 b3 c3   z  d 3 

Determinants 97
E:\AMIT_WORKS\Exam_Guru\EG_Mathematics-12_(working_02-06-2022)\EG_Mathematics-12_working\Open_Files\Chap_5\Chap_5
\ 16-Aug-2022 Amit Proof-4 Reader’s Sign _______________________ Date __________

5 Continuity And Differentiability


Topics Covered
5.1 Continuity and Discontinuity of a Function at a Point 5.2 Differentiability of a Function
5.3 Differentiation 5.4 Derivative of a Composite Function
5.5 Product Rule and Quotient Rule of Differentiation 5.6 Implicit Functions
5.7 Differentiation by using Trigonometrical Substitution 5.8 Logarithmic Differentiation
5.9 Differentiation of Parametric Functions 5.10 Higher Order Derivatives

C hapter map
CONTINUITY

Differentiability of a Function Direct Differentiation

Derivative of a Product Rule, Quotient Higher Order


Composite Function Rule of Differentiation Derivatives

Differentiation Logarithmic Differentiation by Implicit


of a Parametric Differentiation using Trigonometrical Functions
function Substitution

Topic 1. Continuity and Discontinuity of a Function at a Point


A function y = f (x), x ∈ D is called continuous at a point x = m ∈ D, if there is no interruption in the graph of f (x) at x = m. The graph
is unbroken at m and there is no gap or hole in the graph of the function at that point. If there is interruption or break in the graph
or there is a gap or hole in the graph of the function at the point x = m ∈ D, then the function y = f (x), x ∈ D is called discontinuous
at x = m ∈ D.

98
E:\AMIT_WORKS\Exam_Guru\EG_Mathematics-12_(working_02-06-2022)\EG_Mathematics-12_working\Open_Files\Chap_5\Chap_5
\ 16-Aug-2022 Amit Proof-4 Reader’s Sign _______________________ Date __________

How to prove that a given function is continuous or not at a


(iii) RHL = lim+ f ( x ) = lim+ (3x + 2)
given point? x→2 x→2
1. If a function y = f (x); x ∈ D, has a single definition for all 3 ( 2 + h ) + 2 = lim (8 + 3h )
the points in its domain D, then to prove its continuity at = hlim
→0 h→0
a point x = m ∈ D, work out: = 8+3×0=8+0=8
(i) f (m), (ii) single limit: lim f ( x ) and if f (m) Since f (2) = lim f ( x ) = lim f ( x ) = 8
x→m x → 2− x → 2+
= lim f ( x ) then the function is continuous at ∴ f (x) is continuous at x = 2.
x→m
x = m ∈ D. Hence, option (b) is the correct answer.
 x 2 if x ≤ 0
Example 2. The function: f ( x ) =  is
4x + 3 if x > 0
discontinuous at
(a) x = –1 (b) x = 1 (c) x = 0 (d) x = –2
 x 2 if x≤0
Solution. f (x) = 
4x + 3 if x>0
Discontinuity at x = 0:
(i) f (0) = (0)2 = 0
LHL = lim f ( x ) = lim x 2 = lim ( 0 − h )
2
(ii)
x → 0− x → 0− h→0

2. If the given function y = f (x), x ∈ D has two or three



h→0
(
= lim 02 − 2 × 0 × h + h 2 )
definitions for different values of x in the domain D, like:
= lim ( 0 + 0 + h ) = 0 = 0
2 2
 g(x) if x ≥ m h→0
y = f ( x) =  ;
φ(x) if x < m (iii) RHL = lim+ f ( x ) = lim ( 4x + 3)
x→0 x → 0+
h(x) if x > m
 = lim  4 ( 0 + h ) + 3 = lim [ 4h + 3]
y = f ( x ) =  g(x) if x = m
h→0 h→0
φ(x) if x < m
 = 4×0+3=3
Then to prove the continuity or discontinuity of the given
function at x = m, work out: Since f (0) = lim f ( x ) ≠ lim f ( x )
x → 0− x → 0+
(i) f (m), (ii) LHL = lim f ( x ) ,
h → m− \ f (x) is discontinuous function at x = 0.
(iii) RHL = lim + f ( x ) Hence, option (c) is the correct answer.
h→m
I f f ( m ) = lim f ( x ) = lim f ( x ) , t h e n f ( x ) i s  3x + x
h → m− h→m+  if x≤0
Example 3. Prove that the function: f (x) =  x
continuous at x = m.  3 if x=0
is discontinuous function at x = 0.
3x + 2 if x > 2
  3x + x
Example 1. Function f ( x ) = 8 if x = 2 at x = 2 will be a  if x ≤ 0
5x − 2 if x < 2 Solution. f (x) =  x
  3 if x = 0
(a) discontinuous function (b) continuous function
(i) f (0) = 3
(c) signum function (d) None of these
 3x + x 
3x + 2 if x>2 (ii) LHL = lim f ( x ) = lim 
 x→0 − x→0 
− x 
Solution. f (x) = 8 if x=2
5x − 2 if x<2 3 (0 − h) + 0 − h
 = lim
h→0 (0 − h)
Continuity at x = 2:
(i) f (2) = 8  3 ( −h ) + − h 
= lim  
h→0  −h 
(ii) LHL = lim f ( x ) = lim (5x − 2)


x → 2− x → 2−

= lim 5 ( 2 − h ) − 2 = lim [8 − 5h ]


= lim
h→0
(
−3h + h
−h )
= lim
−2h
h → 0 −h
( )
= lim ( 2) = 2.
h→0 h→0

= 8–5×0=8 h→ 0

Continuity And Differentiability 99


E:\AMIT_WORKS\Exam_Guru\EG_Mathematics-12_(working_02-06-2022)\EG_Mathematics-12_working\Open_Files\Chap_5\Chap_5
\ 16-Aug-2022 Amit Proof-4 Reader’s Sign _______________________ Date __________

(iii) RHL = lim f ( x ) = lim 


x→0 + x→0 
+
 3x + x 
x 
= lim
h→0
( 3h + h
h )
= lim
4h
h→0 h
( )
= lim ( 4) = 4
h→0

Since f (0) ≠ lim− f ( x ) ≠ lim+ f ( x )


 3 (0 + h) + 0 + h  x→0 x→0
= lim  
h→0  (0 + h)  \ The given function f (x) is discontinuous at x = 0.

EXERCISE 5.1
I. Multiple Choice Questions (MCQs) II. Short Answer Type Questions-I
Choose the correct answer from the given options.  cos 3x − cos x
 if x≠0
1. For what value of the constant k, the function 1. Prove that the function f ( x ) =  x2
 kx , if x < 0  −4 if x=0
 is continuous at x = 0.
f ( x) =  x is continuous at x = 0?
3, if x ≥ 0 2. Find the value of k, so that the function
(a) –2 (b) 2 (c) 3 (d) –3  x 2 − 2x − 3 
 if x ≠ − 1
2. For what value of k, the function f ( x ) =  x + 1 
 k if x = − 1
 ( x + 3)2 − 36 
 if x ≠ 3
f ( x) =  x−3 becomes continuous at x = –1.
k if x = 3
 3. Find the value of k so that the function
is continuous at x = 3?
 x3 + x 2 − 16x + 20 
(a) 7 (b) 12 (c) 5 (d) 9  if x≠2
f ( x ) =  (x − 2) 2 
 sin 3x if x ≠ 0 
 k if x=2
3. The function f ( x ) =  x is a continuous 
function at  2x + 3 if x = 0 becomes continuous at x = 2.


(a) x = 0 (b) x = 1
 x−5
(c) x = –1 (d) x = 2 


( )
3
2
−x if x<
1
2
 if x≠5  1
4. The function f ( x ) =  x − 5 is discontinuous 4. If the function: f ( x ) =  k if x = is a
 1  2
at if x=5
(a) x = 5 (b) x = –5 (c) x = –1 (d) x = –2
1

2 ( )
+ x if x>
1
2
5. The function f (x) = | x | is continuous at 1
continuous function at x = , then find the value of k.
(a) x = 0 (b) x = 1 (c) x = –1 (d) x = 2 2
5. Find the point of discontinuity of f (x), where f (x) is defined
 x + 2 if x ≤1
6. The function f ( x ) =  is discontinuous at | x | + 3 if x≤−3
 x − 2 if x >1

as : f ( x ) =  − 2x if −3 < x < 3 .
(a) x = 0 (b) x = 1 (c) x = –1 (d) x = –2  6x + 3 if
 x≥3
 x −1 2
 if x ≠1  1 − cos 2x 
7. The function f ( x ) =  x − 1 is continuous at   if x ≠ 0
 2 if x =1 6. If the function f ( x ) =  2x 2 
 k if x = 0

(a) x = 0 (b) x = 1 (c) x = –1 (d) x = 2
is continuous at x = 0, find the value of k.
8. The function f (x) = 2x – | x | is continuous at
7. Show that the function f (x) defined as:
(a) x = 1 (b) x = 0 (c) x = 3 (d) x = 2
 e3 x − e −5 x 
 , if x ≠ 0  sin x 
9. The function f(x) =  x  x + cos x if x>0
k , if x = 0 
 f ( x) = 
2 if x=0
is continuous at x = 0 for the value of k, as [CBSE 2022] 
(a) 3 (b) 5 (c) 2 (d) 8  x2

   if x<0
2
10. The function f (x) = | x – 3 |, x ∈ R, is continuous at   1 − 1 − x 
(a) x = 2 (b) x = –2 (c) x = 3 (d) x = 0 is continuous at x = 0.

100 Mathematics–12
E:\AMIT_WORKS\Exam_Guru\EG_Mathematics-12_(working_02-06-2022)\EG_Mathematics-12_working\Open_Files\Chap_5\Chap_5
\ 16-Aug-2022 Amit Proof-4 Reader’s Sign _______________________ Date __________

8. Find the value of a for which the function defined as: 



a sin
f ( x) = 
{
π(x + 1)
2 } if x≤0
  1 − cos 4x 
 

x2
 if x < 0

5. If the function f ( x ) =  a if x = 0 is
 tan x − sin x if x>0
 x3 
 x

is continuous at x = 0.    if x > 0

 16 + x − 4 
III. Long Answer Type Questions
is continuous at x = 0, find the value of a.
 k cos x  π 6. Find the value of k for which the function:
 π − 2x  if x ≠ 2
1. If f ( x ) =  is continuous function  1 + kx − 1 − kx 
 π 
x  if −1 ≤ x < 0
5 if x =  
 2 f ( x) = 
π   2x + 1
at x = , find the value of k.[CBSE 2022]   x − 1  if 0 ≤ x ≤ 1
2 

2. Prove that the function f ( x ) = 



 x sin
1
x
if x ≠ 0 () is continuous at x = 0.
7. Find the values of a and b so that the function
 0 if x = 0  24 if x≤3
is continuous at x = 0. 
f ( x ) = ( 2ax + 3b ) if 3 < x < 5 is continuous at
 5x + 4 − 4 x + 5  36 if x≥5
 if x ≠1 
3. If the function f ( x ) =  x −1 x = 3 and x = 5 respectively.
 k if x =1
  x − 4 + a if
is continuous at x = 1, find the value of k. x<4
 x−4

 (ax + b) if x > 2 8. If f ( x ) =  a + b if x = 4 is continuous at
  x−4
4. If f ( x ) =  (7x − 4) if x = 2 is a continuous function + b if x>4

(3ax − 2b) if x < 2
  x−4
at x = 2, find the values of a and b. x = 4, then find a and b.

Answers 5.1
I. 1. (d) –3 3. (a) x = 0
 kx Hint:

 if x<0
Hint: f (x) =  −x
(i) f (0) = 2 × 0 + 3 = 3

( )
 3 if x≥0
(ii) L.H.L. = lim f ( x ) = lim sin 3x
x → 0− x → 0− x
 −k if x<0
=   sin 3( 0 − h ) 
 3 if x≥0 − sin 3h
= lim 
h→ 0  (0 − h ) 
 = hlim
→0 −h
⇒ f (0) = 3, LHL = –k, RHL = 3
 sin 3h  × 3 = 1 × 3 = 3

⇒ k = –3 = lim 
h → 0  3h  
2. (b) 12 Similarly, RHL = 3
 ( x + 3) − 36 2 Since
 if x≠3
­
Hint: f (x) = 
x−3 f(0) = lim f ( x ) = lim f ( x ) = 3
 x → 0− x → 0+
 k if x=3
⇒ f (x) is continuous at x = 0.

 ( x + 3 + 6) ( x + 3 − 6) 4. (a) x = 5
 if x≠3
=  x−3
  − ( x − 5)
k if x=3  x−5 if x−5< 0

 x + 9 if x≠3  x−5
Hint: f (x) = 
if x−5≥ 0
= 
 k if x=3  x−5
 1 if x=5

⇒ f (3) = k, LHL = RHL = 12   ⇒ k = 12 

Continuity And Differentiability 101


E:\AMIT_WORKS\Exam_Guru\EG_Mathematics-12_(working_02-06-2022)\EG_Mathematics-12_working\Open_Files\Chap_5\Chap_5
\ 16-Aug-2022 Amit Proof-4 Reader’s Sign _______________________ Date __________

x<5 2. Hint: f (– 1) = k, LHL = – 4, RHL = – 4


 −1 if
 x3 + x 2 − 16x + 20
=  1 if x≥5 3. Hint: f (x) =
 1 if
 x=5 ( x − 2)2

⇒ f (x) ≠ lim− f ( x ) ≠ lim+ f ( x )
⇒ f (x) =
( x + 5)( x − 2)2  ⇒ f (x) = x + 5
x→5 x→5
( x − 2)2
⇒ f (x) is discontinuous at x = 5.
 x + 5, if x ≠ 2
5. (a) x = 0
\ f (x) = 
 k, if x = 2
Hint: f (0) = lim f ( x ) = lim f ( x ) = 0
Now, prove that f (2) = LHL = RHL = 7
x → 0− x → 0+


6. (b) x = 1
Hint: f (1) =3, LHL = 3, RHL = –1.
4. Hint: f
1
2 ()
= k (Given).
Prove that LHL = 1, RHL = 1 ⇒ k=1
7. (b) x = 1 5. Hint: f (3) = 21, LHL = –6, RHL = 21
 x + 1 if x ≠1 ⇒ f(x) is discontinuous at x = 3.
Hint: f (x) = 
 2 if x =1 6. k = 1
Now f (1) = lim ( x + 1) = lim ( x + 1) = 2 
x →1−
⇒ f (x) is continuous at x = 1.

x →1+
 x

(
 sin x + cos x , if
) x>0

7. Hint: f (x) =  2, if x=0


8. (b) x = 0 
 
2
x
Hint: f (x) = 2x – | x |
, if x<0
  1 − 1 − x 2 

f (0) = 2 × 0 – | 0 | = 0
(i) f (0) = 2
L.H.L. = lim− f ( x ) = lim  2 ( 0 − h ) − 0 − h  x2 
x→0 h→ 0 (ii) LHL = lim  
x → 0−  1 − 1 − x 2 
= lim ( −2h ) − h  = lim ( −3h ) = 0
h→ 0 h→ 0 On rationalisation, we get
R.H.L. = lim f ( x ) = lim 2x − x  x 2 1 + 1 − x 2  
x → 0+ x → 0+   
= lim  
= lim 2 ( 0 + h ) − 0 + h −
x→0  x 2 
h→ 0  
= lim 2h − h = lim h = 0
h→ 0 h→ 0 (
= lim− 1 + 1 − x
x→0
2
)

⇒ f (0) = lim f ( x ) = lim f ( x ) = 0  2
x → 0− x → 0+ = lim 1 + 1 − ( 0 − h )  = 2
h→ 0  
⇒ f (x) is continuous at x = 0.

9. (d) 8 (iii) RHL = lim
x → 0+
sin x
x (
+ cos x )
10. (c) x = 3
 sin ( 0 + h ) 
Hint:
= lim  + cos ( 0 + h )
h→ 0  0+h 
(i) f(3) = | 3 – 3 | = 0
 sin h 
= lim  + cos h  = 1 + 1 = 2
(ii) LHL = lim 3 − h − 3 = lim − h = 0 h→ 0  h 
h→ 0 h→ 0

(iii) RHL = lim 3 + h − 3 = lim h = 0 ⇒ f (0) = LHL of f (x) = RHL of f (x) = 2.



h→ 0 h→ 0
  π ( x + 1) 
⇒ f(3) = lim f ( x ) = lim f ( x ) = 0
a sin  2  if x≤0
x → 3− x → 3+ 8. Hint: f (x) =   
 −2 sin 2x sin x , if  tan x − sin x if x>0
 x≠0  x3
II. 1. Hint: f (x) =  x2
 −4, if x=0
(i) f (0) = a sin 
 π ( 0 + 1) 
 2 
 = a sin ()
π =a
2




 −4
f (x) = 
( )( )
sin 2x sin x , if
2x x
x≠0

(ii) LHL = lim−  a sin 
 π ( x + 1)  
 −4, if x=0 x→0   2  
Now prove that   π ( 0 − h + 1)  
= lim  a sin  2  
f (0) = –4, LHL = –4 and RHL = –4. h→ 0
  

102 Mathematics–12
E:\AMIT_WORKS\Exam_Guru\EG_Mathematics-12_(working_02-06-2022)\EG_Mathematics-12_working\Open_Files\Chap_5\Chap_5
\ 16-Aug-2022 Amit Proof-4 Reader’s Sign _______________________ Date __________

= a sin
2 ()
π =a
Since f (x) is continuous at x =
π
2

RHL = lim  tan x −3 sin x 


x → 0+  x 
\ f
()π
2
=LHL = RHL

k=k
 tan ( 0 + h ) − sin ( 0 + h ) 
⇒ 5 = ⇒ k = 10
2 2
= lim  
(0 + h)3
()
h→ 0  
  1 , if
 x sin x≠0
2. Hint: f (x) =  x
 tan h − sin h 
= lim    0, if x=0
h→ 0  h3
(i) f (0) = 0
  1 − cos h  

= hlim
 sin h  cos h  

→0  h3

(ii)

LHL = lim  x sin
x → 0− 
1 
x  ()

 


= lim ( 0 − h ) sin
h→ 0 
( )
1 
0 − h 

()
 tan h   sin h  2
1 
= lim 
h→ 0 

h   h 2  (1 + cos h )  = lim  h sin 1  = 0 × ( any value)
h→ 0  h 
1 1 = 0
= 1 × (1) ×
2
=

Since f (x) is continuous at x = 0


1+1 2
x→0 
+

(iii) Similarly RHL = lim  x sin
1  0
x 
= ()
\ f(0) = LHL = RHL
Since f (0) = LHL = RHL = 0
1 1 ⇒ f (x) is continuous at x = 0.

⇒ a=a= ⇒ a=   5x + 4 − 4x + 5 
2 2

x −1  if x ≠ 1
3. Hint: f (x) = 
(
 k cos x
 π − 2x
III. 1. Hint: f (x) = 
) if x≠ π
2

 k if x = 1

 5 π First simplify
if x=
 2 5x + 4 − 4x + 5 (5x + 4) − ( 4x + 5)
First simplify cos x =
2 π−x
2 ( ) x −1 ( x − 1)  5x + 4 + 4x + 5 

=
( x − 1) =
1
π−x π
Put
2
= y, x → , then y → 0
2
( x − 1)  5x + 4 + 4x + 5  5x + 4 + 4x + 5

π−y (i) f (1) = k



⇒ x=
2
(ii) LHL = lim  1 

( ) k →1  5x + 4 + 4x + 5 

cos x cos π − y
2 sin y
( )

\ = =  
π
2 −x 2 y 2y 1
= lim  
2 h → 0  5 (1 − h ) + 4 + 4 (1 − h ) + 5 
 
Given: f ( ) = 5
π 1 =1
2 =
3+3 6
  1
(iii) RHL = . Evaluate it yourself.
 k cos x  6

( )
LHL = lim−  
π−x Since f (x) is continuous at x = 1.
x→ π  2 
2  2  1
\ f (1) = LHL = RHL =

6
k sin y k sin ( 0 − h ) 1 1 1
= lim− = lim
⇒ k= = ⇒ k=
y→0 2y h→ 0 2 (0 − h ) 6 6 6
4. Hint: f(2) = 10, LHL = 6a – 2b
−k  sin h  = k × 1 = k
= lim   RHL = 2a + b
h → 0 −2  h  2 2
⇒ 6a – 2b = 10 and 2a + b = 10

k
In a similar way evaluate RHL = . Solve these equations.
2

Continuity And Differentiability 103


E:\AMIT_WORKS\Exam_Guru\EG_Mathematics-12_(working_02-06-2022)\EG_Mathematics-12_working\Open_Files\Chap_5\Chap_5
\ 16-Aug-2022 Amit Proof-4 Reader’s Sign _______________________ Date __________

 Since f (x) is continuous at x = 0


  1 − cos 4x  if x < 0
\ f (0) = LHL = RHL
 
 x2 

⇒ a=8=8 ⇒ a=8
5. Hint: f (x) =  a if x = 0
  1 + kx − 1 − kx
 x 
 , if −1 ≤ x < 0
  if x > 0 6. Hint: f (x) = 
x

( )
 16 + x − 4  2x + 1
 , if 0 ≤ x ≤1
  x −1

( )
2
 8 sin 2x if x < 0
 2x  2k
 if −1 ≤ x < 0
a if x = 0  1 + kx + 1 − kx
=  = 
  2x + 1
 x  if 0 ≤ x <1
 if x > 0  x −1

 16 + x − 4 
(i) f (0) = k (ii) LHL = k
(i) f (0) = a
(iii) RHL = lim 2x + 1 = −1
(ii) LHL = lim 8
x → 0−  2x ( )
 sin 2x 2 



x → 0+ x − 1

k = –1
  sin 2 ( 0 − h )  2  7. Hint:
= lim 8   
h→ 0   2 (0 − h )  
  (i) Continuity at x = 3 gives 6a + 3b = 24.
2
sin 2h  8 1 = 8 (ii) Continuity at x = 5 gives 10a + 3b = 36.
= lim 8  = ×( )
2

h→ 0  2h  Solve them.
(iii) RHL = 8 (Evaluate it) 8. a = 1, b = –1

Topic 2. Differentiability of a Function


Differentiability of a Function at a Point (Graphical (iii) Take a function:
Definition)
 x 2 if x≤2
A function f (x) is differentiable at a point x = c in the domain f ( x) = 
D, if there is no break, gap or hole in the graph of f (x) at x = c. 2x if x>2
Example. and draw its graph. In the
(i) Take a function: y = x + 4, x 2 graph of given function there
∈ R. Draw its graph. In the is a break at x = 2, hence it is
graph of y = x2 + 4, x ∈ R, not differentiable at x = 2.
there is no break, gap or hole
 x 2, if x≤2
at any point. Hence y = x2 + y=
4 is differentiable for all real 2x, if x>2
values of x.
y = x2 + 4, x ∈ R (iv) Take a function
 x 2 if x<2
(ii) Take a function: y = | x |, x ∈ f ( x) = 
R and draw its graph. In the 2x if x>2
graph of y = | x |, x ∈ R there and draw its graph. In the
is a break at x = 0. Hence y = graph there is a hole at x = 2,
| x |, x ∈ R is not differentiable hence the given function is not
at x = 0. differentiable at x = 2.
 x 2, if x<2
y = |x|, x ∈ R y=
2x, if x>2

104 Mathematics–12
E:\AMIT_WORKS\Exam_Guru\EG_Mathematics-12_(working_02-06-2022)\EG_Mathematics-12_working\Open_Files\Chap_5\Chap_5
\ 16-Aug-2022 Amit Proof-4 Reader’s Sign _______________________ Date __________

(i) It is differentiable in (a, b).


(v) Take a function
(ii) It is right differentiable at x = a.
 x 2 if x≤2
f ( x) =  (iii) It is left differentiable at x = b.
3x if x>2 A function f (x) is differentiable at a point x = x0 ∈ D if its left
and draw its graph. It is clear hand derivative (LHD)
from the graph that there is  f ( x0 − h ) − f ( x0 ) 
a gap in the graph at x = 2. =  lim  is equal to its right hand
 h→ 0 −h
Hence the given function is not  f ( x0 + h ) − f ( x0 ) 
differentiable at x = 2.
derivative (RHD) =  hlim h 
 x 2, if x≤2  →0 
y=
Note: (i) If a function f (x) is differentiable at x = x0, then the
3x, if x>2
function f (x) is continuous at x = x0.
(ii) If a function f (x) is continuous at x = x0, then the
Differentiability of a Function at a Point function f (x) may or may not be differentiable at
Left Hand Derivative Right Hand Derivative x = x0.
(LHD) (RHD) (iii) To prove that a function f (x) is differentiable
at a point x = x 0, prove that its LHD = RHD at
f(x) f(x)
x = x0.
y y Example 1. Show that the function f (x) = 3x2 + 5 is differentiable
at x = 2.
f ( 2 + h ) − f ( 2)
Solution. [R.H.D of f (x) at x = 2] = lim
h→0 h
LHD RHD
 3 ( 2 + h )2 + 5  −  3 ( 2 ) 2 + 5 
= lim    
h→0 h

O x=a
x
O x=a
x
= lim 
(  )
3 4 + 4h + h 2 + 5 − 17
h→0 h
Differentiability over an Interval 2
lim 17 + 12h + 3h − 17 = lim (12 + 3h )
= h→0
Open interval (a, b) h h→0

y = 12 + 3 × 0 = 12
f ( 2 − h ) − f ( 2)
[LHD of f (x) at x = 2] = lim
h→0 −h
 3 ( 2 − h )2 + 5  −  3 ( 2 ) 2 + 5 
   
= lim
h→0 −h

= lim 
(  )
3 4 − 4h + h 2 + 5 − 17
x h→0 −h
O a c b
A function f(x) is said to be differentiable over (a, b) if it is 17 − 12h + 3h 2 − 17
= lim
differentiable at each and every point c Œ (a, b). h→0 −h

Closed interval [a, b] = lim (12 − 3h ) = 12 − 0 = 12


h→0
y
Since [RHD of f (x) at x = 2] = [LHD of f (x) at x = 2]
RHD \f (x) is differentiable at x = 2 and f ′(2) = 12.
LHD  x 2 if x>5
Example 2. Prove that the function f ( x ) =  is
continuous but not differentiable at x = 5. 5x if x≤5
Solution.
(i) Continuity of f (x) at x = 5:
f (5) = (5x)x = 5 = 5 × 5 = 25
x
O a
A function f(x) is said to be differentiable over [a, b] if:
b

x →5
2
x →5
( )
RHL = lim+ f ( x ) = lim+ x = lim (5 + h )
h→ 0
2

Continuity And Differentiability 105


E:\AMIT_WORKS\Exam_Guru\EG_Mathematics-12_(working_02-06-2022)\EG_Mathematics-12_working\Open_Files\Chap_5\Chap_5
\ 16-Aug-2022 Amit Proof-4 Reader’s Sign _______________________ Date __________

= hlim (
25 + 10h + h 2 ) f (1) = (5a + 3b)
lim f ( x ) = lim+ (5ax + 3b ) = lim 5a (1 + h ) + 3b 
→0

= 25 + 10 × 0 + (0)2 = 25 + x →1 x →1 h→ 0

= (5a + 3b)
LHL = lim− f ( x ) = lim− (5x )
x →5 x →5
− −
(
lim f ( x ) = lim x 2 + 5 = lim (1 − h ) + 5

2
 )
5 (5 − h ) = lim ( 25 − 5h ) x →1 x →1 h → 0
= hlim
→0 h→0 2 2
= lim 1 + h − 2h + 5 = lim 6 − 2h + h  = 6
h→0 h→0
= 25 – 5 × 10 = 25
Hence 5a + 3b = 6 ...(1)
Since f (5) = lim+ f ( x ) = lim− f ( x ) = 25
x →5 x →5 [RHD of f (x) at x = 1]
\ f (x) is continuous at x = 5. f (1 + h ) − f (1)
= lim
h→0 h
(ii) Differentiability of f (x) at x = 5
f ( 5 + h ) − f ( 5) 5a (1 + h ) + 3b  − [5a + 3b ]
[R.H.D. of f (x) at x = 5] = lim = lim 
h→0 h h→0 h
5a + 5ah + 3b − 5a − 3b 5ah
= lim
(5 + h)2 − (5 × 5) = lim 25 + 10h + h2 − 25 = lim
h
= lim
h→0 h
= 5a
h→0
h→0 h h→0 h
f (1 − h ) − f (1)
[LHD of f (x) at x = 1] = lim
= hlim
→0
(10 + h) = (10 + 0) = 10 h→0 −h
[L.H.D. of f (x) at x = 5] (1 − h )2 + 5 − (5a + 3b )
f ( 5 − h ) − f ( 5) 5 ( 5 − h ) − ( 5 × 5) = lim  
= lim = lim h→0 −h
h→0 −h h→0 −h
1 − 2h + h 2 + 5 − 6
25 − 5h − 25 = lim [Q 5a + 3b = 6]
= lim = lim (5) = 5 h→0 −h
h→0 −h h→0
−2h + h 2
Since [R.H.D. of f (x) at x = 5] ≠ [L.H.D. of f (x) at x = 5] = lim = lim 2 − h = 2
h→0 −h h→0
Thus f (x) is not differentiable at x = 5.
Since f (x) is differentiable at x = 1
5ax + 3b if x ≥ 1
Example 3. If the function f ( x ) =  2 is \ [RHD of f (x) at x = 1] = [LHD of f (x) at x = 1]
 x + 5 if x < 1 2
differentiable at x = 1, then find the values of a and b. ⇒ 5a = 2    ⇒ a =
5
5ax + 3b if x ≥ 1 2
Solution. f (x) =  2 Now put a = in 5a + 3b = 6
 x + 5 if x < 1 5
Since f (x) is differentiable at x = 1, therefore, it is continuous ⇒ 2 + 3b = 6     ⇒ b = 4
also at x = 1. 3
2 4
⇒ f (1) = lim f ( x ) = lim f ( x ) , \ When f (x) is differentiable at x = 1, a = , b =
5 3
x →1+ x →1−

EXERCISE 5.2
I. Multiple Choice Questions (MCQs) (a) f is discontinuous at x = 1
Choose the correct answer from the given options. (b) f is differentiable at x = 1
 x + 2, − 1 < x ≤ 3 (c) f is continuous but not differentiable at x = 1

1. If f (x) =  5 , x=3 then at x = 3, f ′(x) = (d) None of these.
 8 − x, x>3

II. Short Answer Type Questions-I
(a) 1 (b) – 1
(c) 0 (d) Does not exist. 5ax + 3b if x ≥ 1
1. I f t h e f u n c t i o n f ( x ) =  2 is
2. Let f (x) = 
 x + 1, when x < 2
, then f ′ (2) =  x + 5 if x < 1
2 x − 1 when x ≥ 2 differentiable at x = 1, find the values of a and b.
(a) 0 (b) 1 2. For what value of λ, the function defined by:
(c) 2 (d) Does not exist.
 x , 0 ≤ x ≤1
3. If f (x) =  , then f ( x) = 
(
λ x 2 + 2 ) if x≤0
is continuous at x = 0?
 2 x − 1, 1< x  4x + 6 if x>0

106 Mathematics–12
E:\AMIT_WORKS\Exam_Guru\EG_Mathematics-12_(working_02-06-2022)\EG_Mathematics-12_working\Open_Files\Chap_5\Chap_5
\ 16-Aug-2022 Amit Proof-4 Reader’s Sign _______________________ Date __________

3. F i n d t h e v a l u e s o f a a n d b i f t h e f u n c t i o n : x = 1, find the values of a and b.


2
 x if x ≤ 1
f ( x) =  is differentiable at x = 1. 3ax 2 + 2bx − 14 if x > 1
ax + b if x > 1 6. If f ( x ) =  3
is differentiable at
4. Find the values of a and b if the function:  5ax − 4bx if x ≤ 1
x = 1, find the values of a and b.
 x 2 + 3x + a if x ≤1
f ( x) = 
is differentiable at x = 1.
 bx + 2 if x >1  3ax 2 + 5bx if x ≥ 1
7. If a function defined as f ( x ) = 
4ax + b + 10 if x ≥1 5ax − 3b + 18 if x < 1
5. If f ( x ) =  is differentiable at
2
 4ax − bx if x <1 is differentiable at x = 1, find the values of a and b.

Answers 5.2
f ( x) − f (3) f (3 − h) − f (3) (1 − h) − 1 −h
I. 1. (d) L f ′(3) = lim = lim ,h>0 = lim = lim = lim (1) = 1
x → 3− x−3 h→0 −h h→0 −h h→0 − h h →0

= lim (3 − h + 2) − 5 [ f (3) = 5] f (1 + h) − f (1)


h→0 −h R f ′(1) = lim
h→0 h
5−h−5 −h
= lim = lim = lim (1) = 1 2(1 + h) − 1 − 1
−h
h→0 h→0 − h h→0 = lim
f ( x) − f (3) f (3 + h) − f (3) h→0 h
R f ′(3) = lim = lim
x → 3+ x−3 h→0 h 2 + 2h − 1 − 1
= lim
8 − (3 + h) − 5 8−3−h−5 h→0 h
= lim = lim
h→0 h h → 0 h 2h
= lim = lim (2) = 2
−h h →0 h h →0
= lim = lim (− 1) = – 1
h→0 h h→0
As L f ′(1) ≠ R f ′(1), so f (x) is not differentiable at x = 1.
As R f ′(3) ≠ L f ′(3), so f (x) is not differentiable at x = 3. Hence Hence, (c) is the correct answer.
(d) is the correct answer.
II. 1. Hint: It is given that f (x) is differentiable at x = 1
f (2 + h) − f (2)
2. (d) RHD = R f ′ (2) = lim ∴ f (x) is continuous also at x = 1
h→0 h
(i) Continuity of the f (x) at x = 1 gives: 5a + 3b = 6.
{2(2 + h) − 1} − (2 ⋅ 2 − 1)
= lim (ii) Differentiability of the f (x) at x = 1 gives 5a = 2.
h→0 h
2. l = 3
4 + 2h − 1 − 3 2h
= lim = lim = lim 2 = 2 3. a = 2, b = –1
h→0 h h →0 h h →0
Hint: (i) Continuity of f (x) at x = 1 gives: a + b = 1
f (2 − h) − f (2)
LHD = L f ′ (2) = lim (ii) Differentiability of f (x) at x = 1 gives:
h→0 −h
[RHD of f (x) at x = 1] = a
2 − h +1− 3 ( − h)
= lim = lim = lim (1) = 1 [LHD of f (x) at x = 1] = 2
h→0 −h h→0 − h h →0
⇒ a=2
As LHD ≠ RHD, so f ′ (2) does not exist.
Hence, (d) is the correct answer. 4. a = 3, b = 5
3. (c) f (1 – 0) = lim f ( x)  lim f (1  h) Hint: (i) Continuity of f (x) at x = 1 gives a – b = –2
x  1 h0
(ii) Differentiability of f (x) at x = 1 gives b = 5
= lim (1 − h) = 1 – 0 = 1 5. a = –2, b = –4
h→0
Hint: (i) Continuity of f (x) at x = 1 gives: a + 2b = –10
f (1 + 0) = lim f ( x)  lim f (1  h)
x  1 h0 (ii) Differentiability of f (x) at x = 1 gives:
= lim 2(1 + h) – 1 = lim 1 + 2h 2a – b = 0.
h→0 h→0
6. a = 2, b = 3
= 1 + 2(0) = 1
Hint: (i) Continuity of f (x) at x = 1, gives a – 3b = –7
Now, f (1 – 0) = f (1 + 0) = 1
(ii) Differentiability of f (x) at x = 1 gives:
and f (1) = 1   ∴ lim f ( x)  1  f (1)
x 1 3a – 2b = 0
so f (x) is continuous at x = 1 7. a = –5, b = 1
f (1 − h) − f (1) Hint: (i) Continuity of f (x) at x = 1, gives a – 4b = –9
Now, L f ′(1) = lim
h→0 −h (ii) Differentiability of f (x) at x = 1 gives: a + 5b = 0

Continuity And Differentiability 107


E:\AMIT_WORKS\Exam_Guru\EG_Mathematics-12_(working_02-06-2022)\EG_Mathematics-12_working\Open_Files\Chap_5\Chap_5
\ 16-Aug-2022 Amit Proof-4 Reader’s Sign _______________________ Date __________

Topic 3. Differentiation
Derivative of a function f (x) is defined as: (iii) y = tan–1 x ⇒ x = tan y
f ( x + h) − f ( x) On differentiating both sides w.r.t. y, we get:
     f ′ ( x ) = lim .
h→0 h dx dy 1 1
⇒ = sec2 y ⇒ = =
Here the derivative of the function exists if this limit exists dy dx sec 2 y 1 + tan 2 y
d
finitely. The derivative of f (x) is denoted by f ′(x) or  f ( x ) or if dy 1
dx  ⇒ =
dy dx 1 + x2
y = f (x) by or y′. The process of finding derivative of a function
dx
is called differentiation. d tan −1 x = 1 , x ∈ R
dx 1 + x2
⇒ ( )
Standard Formulae for Differentiation
dy (iv) y = cot–1 x ⇒ x = cot y
y = x n
⇒ = nx n −1 n dy n −1
1. 2. y = αx ⇒ = (α n x ) On differentiating both sides w.r.t. y, we get:
dx dx
dy dy dx
3. y = e x ⇒ = ex 4. y = a x ⇒ = a x log a ⇒ = –cosec2 y
dx dx dy
dy 1 dy = −1 −1 −1
5. y = log e x ⇒ = ⇒ 2 = =
dx x dx cosec y 1 + cot y 1 + x 2
2

6. y = log a x ⇒
dy
=
1
dx x log e a

d cot −1 x
dx (
= − 2 ,x∈R
1
1+ x
)
dy (v) y = sec–1 x ⇒ x = sec y
dy
7. y = k ⇒ =0 8. y = sin x ⇒ = cos x On differentiating both sides w.r.t. y, we get:
dx dx
dy dx
9. y = cos x ⇒ = − sin x ⇒ = sec y tan y
dx dy
dy dy 1
=
1
10. y = tan x ⇒ = sec 2 x ⇒
dx
=
dx sec y sec y − 1 x x 2 − 1
2

dy
11. y = cot x ⇒
dx
= − cosec 2 x

dx (
d sec −1 x
= )
1
x x2 − 1
, x ∈ R – (–1, 1)
dy
12. y = sec x ⇒ = sec x tan x
dx (vi) y = cosec–1 x  ⇒ x = cosec y
dy On differentiating both sides w.r.t. y, we get:
13. y = cosec x ⇒ = − cosec x cot x
dx dx
⇒ = – cosec y cot y
We have not derived the standard formulae of differentiation for dy
inverse trigonometric functions. Let us do that: dy −1 1
(i) y = sin–1 x ⇒ x = sin y ⇒ = =
dx cosecy cosec y − 1 x x 2 − 1
2
On differentiating both sides w.r.t. y, we get:


dx
dy
= cos y ⇒
dy
=
dx cos y
1 ⇒
d
dx (
cosec −1x = )
−1
x x2 − 1
, x ∈ R – (–1, 1)

dy 1 1 Example 1. If f (x) = ax2 + bx + 5 and f ′(4) = 15 and f ′(2) = 11,


⇒ = =
dx 1 − sin 2 y 1 − x2 then values of a and b will be
(a) a = 2, b = 7 (b) a = 1, b = 2

d
dx (
sin −1 x = ) 1
1 − x2
= –1 < x < 1 (c) a = 1, b = 7 (d) a = 3, b = –3
Solution. f (x) = ax2 + bx + 5 ⇒ f ′(x) = a(2x) + b(1) + 0
(ii) y = cos–1 x ⇒ x = cos y
⇒ f ′(x) = 2ax + b ⇒ f ′(4) = 8a + b = 15 ...(1)
On differentiating both sides w.r.t. y, we get:
f ′(2) = 4a + b = 11 ...(2)
dx dy −1
⇒ = – sin y ⇒ = On solving (1) and (2), we get: a = 1, b = 7.
dy dx sin y
Hence, option (c) is the correct answer.
dy −1
=
−1
⇒ = 1 dy
dx 1 − cos 2 y 1 − x2 Example 2. If y = x + , then : 2x + y equals
x dx

d
dx (
cos −1 x = )
−1
1 − x2
, –1 ≤ x ≤ 1
(a) 2 x (b)
x
(c) x x (d) 3 x
2

108 Mathematics–12
E:\AMIT_WORKS\Exam_Guru\EG_Mathematics-12_(working_02-06-2022)\EG_Mathematics-12_working\Open_Files\Chap_5\Chap_5
\ 16-Aug-2022 Amit Proof-4 Reader’s Sign _______________________ Date __________

1
1 −1 dy 1
Solution. y= x+ = x 2 + x 2 ...(1) ⇒ 2x = x− ...(2)
x dx x
Now (1) + (2) gives:
−1 −3
dy 1 2 1 2 
⇒ dx =  x  −  x  dy 1 1
2  2  ⇒ 2x +y= x− + x+
dx x x
dy 1 1
− dy
⇒ dx = 2 x 2x x ⇒ 2x +y = 2 x
dx
dy 1  1  Hence, option (a) is the correct answer.
⇒ dx = 2x 
x− 
x

EXERCISE 5.3
I. Multiple Choice Questions (MCQs) 3. If y = a sin x + b cos x, then
Choose the correct answer from the given options. 2
dy
2 (a) y 2 +   = a 2 + b2
 1  dy  dx 
1. If f ( x ) =  x −  , then dx is given by
 x 2
 dy  2 2
1 1 1 1 (b) y +   = a + b
(a) 1 − 2 (b) 1 + 2 (c) 1 − x 3 (d) 1 + 3 dx
x x x
2
dy
2. If f (x) = ax2 + bx + 11, and f ′(4) = 61 and f ′(2) = 33, the (c) y −   = a 2 + b 2
 dx 
values of a and b will be
(a) a = 7, b = –3 (b) a = 5, b = –3 dy
2

(c) a = 7, b = 5 (d) a = 3, b = 2 (d) y 2 −   = a 2 − b2


 dx 

Answers 5.3
1 dy
1. (a) 1 − 2 2. (c) a = 7, b = 5 Hint: y = a sin x + b cos x. Find
and put these values of y
x dx
2
dy 
3. (a) y 2 +  = a 2 + b2 dy dy
2
 dx  and in y 2 +   , you will get a2 + b2.
dx  dx 

Topic 4. Derivative of a Composite Function


A function of function is called composite function. Composite dy − f ′ ( x) −f ′ ( x )
1
functions are differentiated with the help of a rule called Chain 4. y = ⇒ = =
f ( x) dx 2f ( x ) f ( x ) 3
Rule. 2  f ( x ) 2
Chain Rule is the main rule for differentiation. The one who dy
Example 1. If y = (3x2 + 4)5, then is equal to
knows chain rule, knows differentiation. dx
2 3
(a) 20x(x + 4) (b) 30x(3x2 + 5)4
Chain Rule (c) 30x(3x3 + 5)4 (d) 30x(3x2 + 4)4
If y is a function of u and u is a function of x such that y = f (u) and Solution. 2
y = (3x + 4) 5
u = g(x) then y = f [ g(x)]. Now the chain rule to differentiate it is:
dy
( ) ( )
4 d
dy dy du ⇒ = 5 3x 2 + 4 3x 2 + 4 = 5(3x2 + 4)4 × 6x
= × dx dx
dx du dx = 30x(3x2 + 4)4
Differentiation of Algebraic Functions by Chain Hence, option (d) is the correct answer.
Rule 1 dy
Example 2. If y = 3 , then dx is given by
n
1. y =  f ( x ) ⇒
dy
dx
= n  f ( x )
n −1
 f ′ ( x ) 3
2x + 5 ( )
dy − n f ′ ( x ) (a) −18x 2
(b) −18x 2
1
2. y = n ⇒ dx =
(3x + 5) ( 2x + 5 )
3 3 3 4
n +1
 f ( x )  f ( x )
f ′ ( x) (c) 18x 2 (d) 18x 2
dy
f ( x) ⇒
3. y = =
(4x + 3) (3x + 5)
2 3 4
2
dx 2 f ( x )

Continuity And Differentiability 109


E:\AMIT_WORKS\Exam_Guru\EG_Mathematics-12_(working_02-06-2022)\EG_Mathematics-12_working\Open_Files\Chap_5\Chap_5
\ 16-Aug-2022 Amit Proof-4 Reader’s Sign _______________________ Date __________

f ′ ( x)
Solution. y =
1
=
d
(
3
dy −3 dx 2x + 5
=
)
−18x 2 7. y = sin–1 f (x) ⇒
dy
dx
=
3  ⇒
2
1 −  f ( x )
(2x + 5) ( ) ( )
3 dx 4 4
2x3 + 5 2x3 + 5
dy −f ′ ( x )
Hence, option (b) is the correct answer. 8. y = cos–1 f (x) ⇒ =
dx 2
1 −  f ( x )
Differentiation of Exponential and Logarithmic
Functions by Chain Rule dy f ′ ( x)
9. y = tan–1 f (x) ⇒ =
dy  f ( x ) 
f ( x) dx 1 +  f ( x ) 2
1. y = e ⇒ = e  f ′ ( x )  
dx   
dy − f ′ ( x)
dy  f ( x ) 
2. y = a ( ) ⇒ y = cot–1 f (x) =
f x 10.
 [ log a ]  f ′ ( x )
= a ⇒
dx  dx 1 +  f ( x ) 2
 
dy f ′ ( x ) f ′ ( x)
3. y = log  f ( x ) ⇒ = y = sec–1 f (x)
dy
dx f ( x) 11. ⇒
dx
=
2
f ( x )  f ( x ) − 1
Logarithmic Formulae
  dy − f ′ ( x)
1. log(xy) = log x + log y 2. log  x  = log x − log y 12. y = cosec–1 f (x) ⇒
dx
=
2
 y f ( x )  f ( x ) − 1
 log e x  dy
3. log(x)m = m log x 4. log y x =  Example 3. If y = sin(4x2 + 5), then dx equals
 log e y 
loge f ( x ) (a) 6x cos (4x2 + 5) (b) 8x cos (4x2 + 5)
6. a a ( ) = f ( x )
log f x
5. e = f ( x)
(c) 3x cos (4x2 + 5) (d) 9x cos (4x2 + 5)
Differentiation of Trigonometric and Inverse Solution. 2
y = sin(4x + 5)
Trigonometric Functions by Chain Rule
1. y = sinm f (x) ⇒
dy
dx
= cos 4x + 5 
2
(d
dx ) (
4x 2 + 5 )
dy

dx
=  m sin (m − 1) f ( x ) cos f ( x )  f ′ ( x )
  = cos ( 4x + 5) (8x ) = 8x cos(4x
2 2
+ 5)
m
 
2. y = cos f (x) Hence, option (b) is the correct answer.
dy
⇒ =  m cos
(m − 1)
f ( x )  − sin f ( x )  f ′ ( x ) 1 − sin x ,
dx Example 4. If y=
1 + sin x
3. y = tanm f (x)

dy
dx
=  m tan
(m − 1)
f ( x ) sec 2 f ( x ) f ′ ( x )
dy
 dx 
π x
then 2   + sec 2 − equals
4 2 ( )
4. y = cotm f (x) (a) 5 (b) 4 (c) 2 (d) 0


dy
dx
=  m cot
(m − 1)
f ( x )  − cosec 2 f ( x )  f ′ ( x )
1 − sin x
1 − cos π − x
2 π x ( ) ( )
( )
Solution. y= = = tan −
5. y = secm f (x) 1 + sin x π 4 2
1 + cos − x
dy  2
⇒ = m sec(m − 1) f ( x ) sec f ( x ) tan f ( x ) ×  f ′ ( x )
6.
dx 
y = cosecm f (x) ⇒
dy
dx
π x −1
( )
= sec 2 −    ⇒ 2
4 2  2 
 dy  + sec 2 π − x = 0
 dx  4 2 ( )
dy 
⇒ = m cosec(m − 1) f ( x )  − cosec f ( x ) cot f ( x ) ×  f ′ ( x ) Hence, option (d) is the correct answer.
dx 
EXERCISE 5.4
Multiple Choice Questions (MCQs) 70x −70x
Choose the correct answer from the given options. (a) (b)
(15x + 8) (7x + 8)
6 2 6
2
5
dy
(
1. If y = 3x 2 + 5 ) 3 then dx is equal to −50x
( 5x )
6
2 2 (c) 2
+9 (d) None of these
(
(a) 5x 3x + 5 2
) 3 (
(b) 10x 3x + 5 2
) 3

dy
2 2 3. If y = 4x3 + x 2 + 8 then dx is equal to
(
(c) 5x 3x − 5 2
) 3 (
(d) 10x 3x − 5 2
) 3

5x ( 2x + 1) x (2x + 5)
1 dy (a) (b)
2. If y = , then equals 3 2
2x + 2x + 2 3x 3 + 2x 2 + 3
(7x + 8)
5
2 dx

110 Mathematics–12
E:\AMIT_WORKS\Exam_Guru\EG_Mathematics-12_(working_02-06-2022)\EG_Mathematics-12_working\Open_Files\Chap_5\Chap_5
\ 16-Aug-2022 Amit Proof-4 Reader’s Sign _______________________ Date __________

3x (2x + 1) 9. Differential of log [log (log x5)] w.r.t. x is


(c) (d) None of these 5 5
4x3 + 3x 2 + 8 (a) 5 5
(b)
1 dy x log( x ) log (log x ) x log (log x 5 )
4. If y = , then = 5x 4
5x4
3 2
x + 3x + 4x + 5 dx (c) (d)
log x 5 log (log x 5 )
(a)
(
− 2x 2 + 4x + 3 ) −3x 2 − 6x + 4
(b) 2(x 2 − 3x 2 + 4x + 5)
log ( x 5 ) log (log x 5 )
[CBSE 2022]
(
2 x 3 + 2x 2 + 3x + 5 ) 10. If y = e5 log x + 52 log5 x + 79 log7 x, then dy equals
(
− 3x + 6x + 42
) 6
(a) 5x + 2x + 9x 7
dx
(b) 7x + 3x + 2x8
2
(c) 3 (d) None of these
2(x3 + 3x 2 + 4x + 5) 2 (c) 5x3+ 3x + 7x8 (d) 5x4+ 2x + 9x8
dy II. Short Answer Type Questions-I
5. If y = x + x 2 + a 2 , then equals
dx dy
y 2y 1. If y = esin x + cos x, then find .
dx
(a) x + a 2
2 (b) 2x 2 + a 2
3x 2 + tan x dy
y 2. If y = 7 , then find .
(c) dx
x − a 2
2 (d) None of these

dy
n
−1 2
( )
3. If y = sin x , then prove
2xdy
= .
6. If y =  x + x + a  , then dx
1 − x4
2 2
equals
  dx
−1 dy 1
ny yn 4. If y = tan x, then show = .
dx 2 x (1 + x)
(a) x 2 + a 2 (b)
x2 + a2
ny 5. If y = cos
−1
( )
x x , then find
dy
dx
.
(c) x 2 − a 2 (d) None of these
3
2 dy
7. If y = 6. If y = sin 2 5x + 7, then show that .
x +1 + x − 1 , then dx
dy y dy  15x   12 
(a) x2 − 1   = (b) x 2 − 1   = 2y =  
2 2
 sin 5x + 7 cos 5x + 7  .
 dx  2  dx   2 5x 2 + 7   
dy
(c) x2 + 1   = y (d) None of these dy
 dx  7. If y = sec x , then find .
dx
1− x
8. If y = , then 8. If y = cos(log x + ex), then show that
1+ x
dy x 1 x
= − sin log x + e  ×  + e  .
(
(a) 1 − x 2
dy
dx
+ y = 0.) ( dx )
(b) 1 + x 2 dy + y = 0.
dx x 
1  dy  sec x − 1 dy
(c) 2   + y = 0. (d) None of these 9. If y = , then what is the value of ?
x dx sec x + 1 dx

Answers 5.4
2 ny
(
I. 1. (b) 10x 3x 2 + 5 ) 3 2. (b)
−70x
(7x 2 + 8)6
6. (a)
x + a2
2

3. (c)
3x ( 2x + 1)
4. (c)
(
− 3x 2 + 6x + 4 ) Hint:
y =  x + x 2 + a 2 
 
3
4x3 + 3x 2 + 8
(
2 x3 + 3x 2 + 4x + 5 ) 2


dy
= n  x + x + a 
2 2
n −1 
1 +
2x 

y dx  
5. (a)  2 x2 + a2 
x2 + a2
n −1 
dy x + x2 + a2 
= n  x + x + a  
2 2
Hint:
y = x + x2 + a2
⇒ 
dx    x 2 + a 2 

dy 2x x x2 + a2

⇒ = 1+ = + =
ny
dx 2 x2 + a2 x2 + a2 x + a2
2

dy y dy y


dx
= 7. (a) x2 − 1   =
2
x +a 2  dx  2

Continuity And Differentiability 111


E:\AMIT_WORKS\Exam_Guru\EG_Mathematics-12_(working_02-06-2022)\EG_Mathematics-12_working\Open_Files\Chap_5\Chap_5
\ 16-Aug-2022 Amit Proof-4 Reader’s Sign _______________________ Date __________

Hint:
y= x +1 + x −1 dy

⇒ (1 – x2) +y=0
dx
dy 1 + 1

⇒ =
dx 2 x +1 2 x −1 5
9. (a)
x log( x ) log (log x 5 )
5

1  x +1 + x −1
=  
2 x2 − 1  10. (d) 5x4 + 2x + 9x8
dy y
x2 − 1   =
5 2 9

⇒ Hint:
y= e
log x
+ 5log5 x + 7log7 x
 dx  2

⇒ y = x5 + x2 + x9
(
8. (a) 1 − x
dy
dx
2
+y=0 ) II. 1. [esin x + cos x] (cos x – sin x)

( )
1
Hint:
y = 1− x 2  3x2 + tan x 2 
1+ x 2. 7 (log 7) × (6x + sec x) 
 2 3x 2 + tan x 
dy 1 + x  −1 

⇒ =   −3 x
dx 1 − x  (1 + x )2  5.
 
2 1 − x3
dy − 1− x

⇒ = sec x tan x
dx (1 − x )(1 + x ) 1 + x 7.
4 x
dy 1 − x = −y 1 sec2 x

⇒ (1 – x2) = − 9.
dx 1+ x 2 2

Topic 5. Product Rule and Quotient Rule of Differentiation


Product Rule 2 2
(a) 2x (x + 2) cos(x + 22) − sin (x + 2)
dy
1. If y = f (x) g(x), then = f ′ ( x) g ( x) + f ( x) g′ ( x) ( x + 2)
dx
2. If y = f (x) g(x) f(x), 3x (x + 3) cos(x + 2) − sin (x + 2)
(b)
(x + 2) 2
then dy = f ′(x)g (x)f(x) + f (x) g′(x) f(x) + f(x) g(x) f′(x)
dx 2 2
Example 1. If y = (3x2 + 2) sin x2, then dy is equal to (c) 3x (x + 3) cos (x + 22) + sin (x + 2)
dx ( x + 2)
2 2 2
(a) 6x cos (3x + 5) + 2 sin x 5x (x 2 + 2) cos2 x + sin 2 x
(b) 3x sin x2 + 2x2 (3x2+ 5) (d)
(x + 2)2
(c) 5x cos x2 (3x2 + 5) + 2 sin x2
(d) 6x sin x2 + 2x (3x2+ 2) cos x2
Solution. y=
(
sin x 2 + 2 )
Solution. y = (3x2 + 2) sin x2 ( x + 2)

dx  dx (  ) (
dy =  d 3x 2 + 2  sin x 2 + 3x 2 + 2 d sin x 2
dx ) ( ) dy  ( )  ( ) dx ( )
d sin x 2 + 2  − sin x 2 + 2  d x + 2
( x + 2) dx
⇒ =
= 6x sin x2 + (3x2 + 2)(cos x) 2x dx ( x + 2 )2
= 6x sin x2 + 2x(3x2 + 2)cos x2.
Hence, option (d) is the correct answer. dy ( x + 2)  2x {cos ( x 2 + 2)} − sin ( x 2 + 2)
⇒ =
Quotient Rule dx ( x + 2 )2
f ( x) dy g ( x ) f ′ ( x ) − f ( x ) g ′ ( x )
If y =
g ( x)
then
dx
=
 g ( x )
2
=
( )
2x ( x + 2) cos x 2 + 2 − sin x 2 + 2( )
( x + 2) 2

Example 2. If y =
(
sin x 2 + 2 ) , then dy will be given by
( x + 2) dx Hence, option (a) is the correct answer.

112 Mathematics–12
E:\AMIT_WORKS\Exam_Guru\EG_Mathematics-12_(working_02-06-2022)\EG_Mathematics-12_working\Open_Files\Chap_5\Chap_5
\ 16-Aug-2022 Amit Proof-4 Reader’s Sign _______________________ Date __________

EXERCISE 5.5
I. Multiple Choice Questions (MCQs) II. Short Answer Type Questions-I
Choose the correct answer from the given options.
2
1. If y = a log  x + x 2 + a 2  + x dy
dy x 2 + a 2 , then find
1. If y = (3x 2+ 7) 2 (5x 2 − 4)3, then equals 2   2 dx
dx
(a) 6x (3x2 + 7) (5x2 – 4)2 (25x2 + 27) 2
2. If y = − a log  x + x 2 − a 2  + x x 2 − a 2 , then
2
(b) 6x (3x – 7) (5x + 4) (25x – 27) 2 2
2   2
(c) 3x (6x2 + 7) (3x2 – 4)2 (8x2 + 15)
(d) None of these. find dy
dx
dy
2. If y = (cos x3) (sin 2 x5), then equals
dx  a + b cos x dy
3. If y = sin − 1  , then show that
(a) x2 sin x2 [ –3sin x3sin x5)  b + a cos x  dx
(b) x2 sin x5 [ –3 sin x3sin x5 + 10 x2 cos x3cos x5]
(c) x2 sin x5 [ 3 sin x3sin x5 – 10 x2 cos x3cos x5]  − b2 − a 2 
  =  b + a cos x 
(d) None of these.  

3. If y =
2 a 2 ()
a 2 sin − 1 x + x a 2 − x 2 , then dy
dx
is equal to
4. If f ( x) =
x +1
x 2 + 1, g ( x ) =  2 
 x + 1
(a) a2 + x2 (b) a2 – x2
(c) a 2 − x 2 (d) a2 + x2 and h(x) = (2x – 3), then find f ′[h′{g′(x)}].

Answers 5.5
2 ­2 2 2
I. 1. (a) 6x(3x + 7)(5x – 4) ­(25x + 27)
II. 1. x 2 + a 2 2. x2 − a2
2 5 3 5 ­2 3 5
2. (b) x sin x [–3 sin x sin x + 10x cos x cos x ] 2
4.
3. (c) 2 2 5
a −x

Topic 6. Implicit Functions


A relation between x and y in which y cannot be expressed easily
⇒ dy = 1 − 2x − y
in terms of x, is called implicit function x2 + y2 + x2y + 7 = 0, dx 3y 2 + x − 1
x3 + sin(xy) = 2 etc are some examples of implicit functions.
Example 2. If y = x + x + x + ............... , then prove that
Differentiation of Implicit Functions
dy = 1 .
To differentiate implicit function, remember: dx 2y − 1


d n
dx ( )
y = ny n −1
dy
dx
and
d n
dy
x = nx n −1
dx
dy ( ) Solution. y= x + x + x + ...............

dy
Example 1. If x2 + y3 + xy = x + y, then find w.r.t. x
dx ⇒ y2 = x + x + x + x + ........... = x + y
Solution. x2 + y3 + xy = x + y
dy dy
On differentiating w.r.t. x, we get ⇒ 2y = 1+
dx dx
dy dy dy
2x + 3y2 +x +y×1= 1+ dy
dx dx dx ⇒
dx
( 2y − 1) = 1
dy dy 1
⇒ (3y2 + x – 1) = 1 – 2x – y ⇒ =
dx dx 2y − 1

Continuity And Differentiability 113


E:\AMIT_WORKS\Exam_Guru\EG_Mathematics-12_(working_02-06-2022)\EG_Mathematics-12_working\Open_Files\Chap_5\Chap_5
\ 16-Aug-2022 Amit Proof-4 Reader’s Sign _______________________ Date __________

EXERCISE 5.6
I. Multiple Choice Questions (MCQs)
2. If sin(xy) – cos(x – y) = y2, then find dy
Choose the correct answer from the given options. dx

1. If x3 + 8xy + y3 = 64, then


dy
= 3. If xy + y2 = y + tan x, then find dy
dx dx

8x + 3 y 2 2 4. If 1 − x 2 + 1 − y 2 = a ( x − y ) , then prove that


(a) (b) 8 y + 3 x
3x 2 + 8 y 3 y + 8x2 dy 1 − y2
=
3x 2 + 8 y dx 1 − x2
(c) – (d) None of these dx cos a
8x + 3 y 2 5. If sin y = x cos (a + y), then prove that =
dy cos 2 (a + y)
dx
2. If y = x sin y, then = [CBSE 2022]
dy
cos2(a + y)
1 − sin y 1 − x sin y 6. If cos y = x cos (a + y), then prove that dy =
(a) (b) dx sin a
x cos y x cos y
1 − x cos y 1 − x cos y 7. If y = sin x + sin x + sin x + ... , then prove that
(c) (d)
sin y x sin y dy  cos x 
=
II. Short Answer Type Questions-I dx  2y − 1

 y
1. Find the value of dy at x = 1, 8. If log x 2 + y 2 = tan − 1 , then prove that
dx  x
π 2
y= if sin y + cos (xy) = k. dy  x + y 
4 = 
dx  x − y 
dy dx
2. If 5x2 + 3y2 + xy = 7, then find × .
dx dy 9. If y 1 − x 2 − x 1 − y 2 = 1 , then prove that
III. Long Answer Type Questions
dy 1 − y2
1. If x3 + x2y + xy2 + y3 = 81, then find dy .
dx
=
dx 1 − x2

Answers 5.6
3 3
I. 1. (c) We have, x + 8xy + y = 64 dy sin y
Differentiating both sides w.r.t. x, we get
⇒ =
dx 1 − x cos y
 dy  dy dx 1 − x cos y
3x2 + 8 1y + x  + 3 y 2 =0 or =
 dx  dx dy sin y
dy dy Hence, (c) is the correct answer.

⇒ 3x 2 + 8 y + 8 x + 3y2 =0
dx dx II. 1. Hint: sin2 y + cos(xy) = k
dy dy dy

⇒ 3x 2 + 8 y + (8 x + 3 y 2 ) =0 ⇒ 2 sin y cos y
− sin ( xy )  x + y  = 0
dx dx  dx 
dy  3x 2 + 8 y  π

\ = –  dy
2 Find and put x = 1 and y =
dx  8x + 3 y  dx 4
Hence, (c) is the correct answer.
2. 1
2. (c) y = x sin y
Differentiating both sides w.r.t. x, we get III. 1. Hint: x3 + x2y + xy2 + y3 = 81

dy dy dy dy
dy
⇒ 3x2 + 2xy + x2 + y2 + 2xy + 3y2 =0
= x cos y + sin y dx dx dx
dx dx
dy
Now find .
dy dx

⇒ (1 − x cos y ) = sin y
dx 2. Hint: sin(xy) – cos(x – y) = y2

114 Mathematics–12
E:\AMIT_WORKS\Exam_Guru\EG_Mathematics-12_(working_02-06-2022)\EG_Mathematics-12_working\Open_Files\Chap_5\Chap_5
\ 16-Aug-2022 Amit Proof-4 Reader’s Sign _______________________ Date __________

 dy dy dx = cos a
⇒ cos ( xy )  x +
y  + sin ( x − y ) 1 − 

 dx   dx  dy cos 2 (a + y)
dy cos y
= 2y 6. Hint: x =
dx cos (a + y)
dy Now differentiate w.r.t. y.
Now find .
dx
3. Hint: xy + y2 = y + tan x 7. Hint: y = sin x + sin x + sin x + ......

dy dy dy
⇒ y2 = (sin x) + y.

⇒ x + y + 2y = + sec2 x dy dy dy cos x
dx dx dx
⇒ 2y = cos x +    ⇒ =
dx dx dx 2y − 1
dy
Now find .
dx  y
8. Hint: log x 2 + y 2 = tan −1
 x
4. Hint: 1 − x2 + 1 − y 2 = a ( x − y )

Put x = sin q and y = sin f.



2
+( )
1 log x 2 y 2 = tan −1  y 
 x

1 − sin 2 θ + 1 − sin 2 φ = a (sin θ − sin φ ) dy


x −y
1  2x + 2y dy  = 1 × dx

⇒ (cos q + cos f) = a(sin q – sin f)





(
2 x 2 + y 2 ) dx  y2
1+ 2
x2
x

 θ+φ θ − φ  θ+φ θ − φ dy
⇒ 2 cos
cos = a 2 cos sin x+ y
dx =  x   1 dy − y 
2
 2 2   2 2 

  
2
x +y 2
 x + y 2   x dx x 2 
2


⇒ θ−φ
cot =a
2 dy
Now simplify it and find .

⇒ q – f = 2 cot–1 a dx
⇒ sin–1 x – sin–1 y = 2 cot–1 a

9. Hint: y 1 − x 2 − x 1 − y 2 = 1
1 1 dy

⇒ − =0 Put x = sin q and y = sin f
1− x 2
1 − y dx
2
sin f cos q – sin q cos f = 1
5. Hint: sin y = x cos (a + y)

⇒ sin(f – q) = sin π
sin y 2

⇒ x=
cos (a + y) π

⇒    f – q =
Differentiate w.r.t. y 2
dx = cos ( a + y ) cos y − sin y {− sin ( a + y ) . 1}
⇒ sin −1 y − sin −1 x = π

⇒ 2
dy cos 2 ( a + y )
1 dy
dx = cos (a + y) cos y + sin (a + y) sin y
⇒ − 1 =0


dy cos 2 ( a + y ) 1 − y 2 dx 1 − x2

dx = cos ( a + y − y )
⇒ dy
=
1 − y2


dy cos 2 (a + y) dx 1 − x2

Topic 7. Differentiation by using Trigonometrical Substitution


Before starting differentiation of inverse trigonometric functions, tan A − tan B
revise and commit to your memory the following formulae. 6. tan ( A − B) =
1 + tan A tan B
1. sin (A + B) = sin A cos B + cos A sin B.
2 tan θ
2. sin (A – B) = sin A cos B – cos A sin B. 7. sin 2θ = 2 sin θ cos θ =
1 + tan 2 θ
3. cos (A + B) = cos A cos B – sin A sin B.
8. cos 2q = cos2q – sin2q = 1 – 2 sin2q = 2 cos2q – 1
4. cos (A – B) = cos A cos B + sin A sin B.
1 − tan 2 θ
5. tan (A + B) = tan A + tan B =
1 + tan 2 θ
1 − tan A tan B

Continuity And Differentiability 115


E:\AMIT_WORKS\Exam_Guru\EG_Mathematics-12_(working_02-06-2022)\EG_Mathematics-12_working\Open_Files\Chap_5\Chap_5
\ 16-Aug-2022 Amit Proof-4 Reader’s Sign _______________________ Date __________

2 tan θ y = sin −1 sin 2 x  = 2 x


9. tan 2θ = 10. 1 + cos θ = 2 cos 2 θ ⇒
1 − tan 2 θ 2
dy 1
⇒ =
11. 1 − cos θ = 2 sin 2 θ 12. sin 3q = 3 sin q – 4 sin3 q dx x
2
13. cos 3q = 4 cos3 q – 3 cos q Example 2. If y = tan −1 ( 1 + sin x
cos x )
, then find
dy
dx
.

( )
3 tan θ − tan 3 θ
14. tan 3θ = 1 + sin x
1 − 3 tan 2 θ Solution. y = tan −1
cos x
−1  x + y 
15. tan 
 1 − xy 
= tan −1 x + tan −1 y 1 − cos π + x
−1  2 ( ) 
( ) 
⇒ y = tan 
π
 x− y   sin + x
16. tan
−1 −1 −1
 1 + xy  = tan x − tan y  2

 2 sin ( π + x )
2 

  4 2
17. 2 tan −1 x = tan −1  2x 2 
−1

 2 sin ( + ) cos ( + ) 
= tan 
1 − x  π x π π 
 4 2 4 2 

 ( 4 2) 
18. 2 tan −1 x = sin −1  2x 2   sin π + x 
 = tan  tan ( 4 + 2 )
1+ x 
⇒ y = tan −1  π x 
−1

 cos ( + ) 
 π x
−1  1 − x2 
−1  
19. 2 tan x = cos 
 1 + x 2   4 2 

20. sin x = cos ( )


π
2
−x ⇒ y=
π x
+   ⇒ dy = 0 + 1   ⇒ dy = 1
4 2 dx 2 dx 2

21. sin x = − cos π + x


2 ( ) 22. cos x = sin π + x
2 ( )  3x − x3  −1
Example 3. If y = tan −1  2 ,
 1 − 3x  3
<x<
1
3
, then find
dy
dx
.

23. cos x = sin π − x


2 ( ) 24.
1 − tan x
1 + tan x
= tan
π
4( )
−x Solution.
 3x − x3 
y = tan −1  2
 1 − 3x 
25.
1 + tan x
1 − tan x
= tan
π
4 ( )
+x Put x = tan q   ⇒ q = tan–1 x
 3 tan θ − tan 3 θ 
⇒ y = tan −1  2 
 2 tan x   1 − 3 tan θ 
Example 1. If y = sin −1  , then find dy .
 1 + tan 2 x  dx
⇒ y = tan–1[tan 3q] = 3q = 3 tan–1 x
 2 tan x  dy 3
Solution. y = sin −1  ⇒ =
 1 + tan 2 x  dx 1 + x2

EXERCISE 5.7
I. Multiple Choice Questions (MCQs)  sin x + cos x  dy
3. If y = tan −1   , then is
Choose the correct answer from the given options.  cos x − sin x  dx
1. If f (x) = x tan–1 x, then f ′ (1) =
1 p
π (a) (b) (c) 0 (d) 1
(a) 1 + (b) 1 + π 2 4
4 2 4
II. Short Answer Type Questions-I
1 π
(c) − (d) 2  1 − cos x  dy
2 4 1. If y = tan − 1  , then find
 sin x  dx
 x + 1 −1  x − 1  dy
2. If y = sec −1  then =
 x − 1  + sin  x + 1  , dx −1 1 − cos x dy
    2. If y = tan , then find
1 + cos x dx
1
(a) 0 (b)
x +1  cos x + sin x  dy
3. If y = tan − 1  , then find
(c) 1 (d) None of these  cos x − sin x  dx

116 Mathematics–12
E:\AMIT_WORKS\Exam_Guru\EG_Mathematics-12_(working_02-06-2022)\EG_Mathematics-12_working\Open_Files\Chap_5\Chap_5
\ 16-Aug-2022 Amit Proof-4 Reader’s Sign _______________________ Date __________

−1 dy  5 cos x − 12 sin x  dy
4. If y = cot (cosec x + cot x), then find 8. If y = cos −1   , then find dx
dx  13
III. Long Answer Type Questions
 1 + cos x − 1 − cos x 
−1 1 − x dy 9. If y = tan −1   , then find dy
1. If y = cos  1 + x  , then find dx  1 + cos x + 1 − cos x  dx
 
 a−x dy
2. If y = tan − 1   , then find dx  1 + sin x 2 − 1 − sin x 2  dy
 a + x  10. If y = tan −1
  , then find
 1 + sin x 2 + 1 − sin x 2  dx
−1  2
x +1 
dy
3. If y = tan  x  , then find dx
 1 − 4   1 + x 2 −1 dy
11. If y = tan −1   , then find
2x x dx
− 1 1 − 3  dy  
4. If y = cos  2x  , then find dx
 1 + 3 
 1+ x + 1− x  dy
12. If y = tan −1   , then find
 2 × 5x  dy  1+ x − 1− x  dx
5. If y = sin − 1  2x  , then find dx
 1 + 5 
dy
13. If y = sin  x 1 − x − x 1 − x  , then find
−1 2
−1  a − x dy   dx
6. If y = tan   , then find dx
 1 + ax 
 1 − cos x + 1 + cos x  dy
 ax + b  dy 14. If y = sin −1   , then find dx
7. If y = tan − 1  , then find 2
 a − bx  dx  

Answers 5.7
–1
I. 1. (b) f (x) = x tan x  π 
Differentiating w.r.t. x, we get  1 + tan x   tan + tan x 

⇒ y = tan −1  −1 4
1  = tan  
f ′ (x) = x × + tan −1 x  1 − tan x   1 − tan π tan x 
1 + x2  4 
x −1 π  π

⇒ f ′ (x) = + tan −1 x
⇒ y = tan tan  + x  = + x
1 + x2 4  4
Put x = 1, then dy
1
\ = 1
1 π 1 π dx
f ′ (x) = + tan −1 (1) = + = +
1 + (1) 2 1+1 4 2 4 Hence, (d) is the correct answer.
Hence, (b) is the correct answer. −1 1 − cos x  x x
II. 1. Hint: y = tan  = tan −1  tan  =
 x + 1 −1  x − 1   sin x   2  2
2. (a) If y = sec−1 
 x − 1  + sin  x + 1 

x −1
−1 
 

−1  x − 1 

π
2. Hint:
1 + cos x 2 ( )
y = tan −1 1 − cos x = tan −1 tan x = x
2

( )
= cos 
 x + 1  + sin  x + 1  = 2 3. Hint: y = tan −1
cos x + sin x
   
cos x − sin x

( ) ( )
 −1 −1 π
−1 1 + tan x −1  π 
Q sin x + cos x = 2  = tan 1 − tan x = tan  tan 4 + x 
  
dy

∴ = d  π  = 0 = π+x
dx dx  2  4
Hence, (a) is the correct answer. 4. Hint: y = cot–1(cosec x + cot x)
 sin x + cos x  −1 1 + cos x 
3. (d) y = tan −1   = cot 
 cos x − sin x   sin x 


 sin x cos x 

−1 cos x
= tan 
+
cos x 

⇒ ( )
x
y = cot −1 cot =
2
x
2

 cos x

sin x 
 cos x cos x 
 III. 1. Hint: y = cos −1
1− x
1+ x ( )
Continuity And Differentiability 117
E:\AMIT_WORKS\Exam_Guru\EG_Mathematics-12_(working_02-06-2022)\EG_Mathematics-12_working\Open_Files\Chap_5\Chap_5
\ 16-Aug-2022 Amit Proof-4 Reader’s Sign _______________________ Date __________

Put x = tan2 q
2 
⇒ q = tan −1 x

−1
y = tan x + tan
−1 b
a ()
−1 1 −

⇒ y = cos  tan 2 θ  = cos −1 cos 2θ Now find
dy
.
1 + tan θ  dx

⇒ y = 2θ = 2 tan −1 x 8. Hint: Put 5 = cos α and 12 = sin α
13 13
Now differentiate. −1  1 + cos x − 1 − cos x 
9. Hint: y = tan  
a−x  1 + cos x + 1 − cos x 
2. Hint: y = tan −1
a+x  2 cos x − 2 sin x 
−1  2 2

Put x = a cos θ ⇒ θ = cos x−1
⇒ y = tan 
x + 2 sin x 
a  2 cos
2 2 

⇒ y = tan −1 1 − cos θ = θ = 1 cos −1 x
1 + cos θ 2 2 a ()

1 − tan x 
y = tan −1  2
Now differentiate. x
x +1 
1 + tan 2 
−1  2

( )
3. Hint: y = tan  x  π x 
1 − 4  = tan −1  tan − 
 4 2 
  dy −1
x
−1  2 × 2 
π−x =

⇒ y= tan
⇒ y= ⇒
4 2 dx 2

( )
x 2
1 − 2 
−1
 1 + sin x 2 − 1 − sin x 2 
Put 2x = tan q 10. Hint: y = tan  
 1 + sin x 2 + 1 − sin x 2 
 2 tan θ 

⇒ y = tan −1  Write
1 − tan 2 θ 
x2 x2
–1 1 + sin x 2 = cos + sin and
⇒ y = tan [tan 2q] = 2q 2 2
⇒ y = 2tan–1(2x) 2 2
1 − sin x 2 = cos x − sin x
Now differentiate. 2 2
2x   2 
 x
4. Hint: y = cos −1 1 − 32x  −1 
sin  x 2 dy
1 + 3  y = tan  2 = ⇒ =x
2 2
x dx
Put 3x = tan θ  cos 
 2
 2  11. Hint: Put x = tan θ.

⇒ y = cos −1 1 − tan 2 θ  = cos −1 cos 2θ
1 + tan θ  −1  1 + x + 1 − x 
12. Hint: y = tan  
⇒ y = 2q = 2 tan (3 ) –1 x
 1+ x − 1− x 
Now differentiate. Rationalise Dr.
 2 × 5x   2
5. Hint: y = sin −1  2x  y = tan −1 1 + x + 1 − x + 2 1 − x 
1 + 5   (1 + x ) − (1 − x ) 
Put 5x = tan q ⇒ q = tan–1(5x)

−1 1 + 1 − x
2
 2 tan θ 
⇒ y = tan  

⇒ y = sin −1  = sin–1(sin 2q) x
1 + tan 2 θ   
⇒ y = 2q = 2 tan–1(5x) Put x = cos q
Now differentiate. dy
Simplify and find .
  dx
6. Hint: y = tan −1  a − x 
 1 + ax  13. Hint: y = sin −1  x 1 − x − x 1 − x 2 
 
⇒ y = tan −1 a − tan −1 x
 
( x)
2
Now differentiate. y = sin − 1  x 1 − − x 1 − x2 
7. Hint: y = tan −1
ax + b
a − bx ( ) 
Now put x = sin θ and x = sin φ

Divide Nr. and Dr. by a


2 
  y = sin −1 sin θ 1 − sin 2 φ − sin φ 1 − sin θ 
x+ b   
−1  a
y = tan 
()

b 
–1
y = sin [sin q cos f – cos q sin f]
1 − x 
 a  y = sin–1[sin(q – f)] = q – f

118 Mathematics–12
E:\AMIT_WORKS\Exam_Guru\EG_Mathematics-12_(working_02-06-2022)\EG_Mathematics-12_working\Open_Files\Chap_5\Chap_5
\ 16-Aug-2022 Amit Proof-4 Reader’s Sign _______________________ Date __________


dy
y = sin −1 x − sin −1 x


−1 

 2 ( ) ( )
x  1  + cos x  1  
y = sin  sin 
 2  2  2  

( )
Now find
dx  x π  x π
= sin −1 sin +  = +
 2 4  2 4
 
14. Hint: y = sin −1  1 − cos x + 1 + cos x  Find
dy
.
 2  dx

Topic 8. Logarithmic Differentiation


Function of the type y = [ f (x)]g (x) are differentiated by taking log Solution. y = (x)tan x + (tan x)x
on both the sides Let xtan x = u and (tan x)x = v. Now the above equation
becomes:
log y = log [ f (x)]g (x)
y= u+v
⇒ log y = g(x) log [f (x)] dy du dv
⇒ = +
dx dx dx
1 dy  g ( x) f ′ ( x) 
⇒ = g ′ ( x ) log  f ( x ) +   Note: If y = u + v, then log y ≠ log u + log v
y dx  f ( x) 
y = xtan x + (tan x)x
dy  g ( x) f ′ ( x)  du dv
⇒ = y  g ′ ( x ) log  f ( x ) + Now let us find and
dx  f ( x )  dx dx
Take: u = xtan x   ⇒ log u = tan x log x
dy g ( x)  g ( x) f ′ ( x) 
⇒ =  f ( x )  g ′ ( x ) log  f ( x ) +
dx  f ( x )  log
1 du
= sec 2 x log x +
tan x
u dx x
Example 1. If y = (sin x + cos x ) (3x2 +5) , then find dy . du tan x  tan x 
dx ⇒ = x sec 2 x log x +
dx  x 
Solution. y = (sin x + cos x ) ( 3x 2 + 5 ) Take v = (tan x)x  ⇒ log v = x log tan x
On taking log on both sides, we get 1 dv x sec 2 x
⇒ = log tan x +
log y = (3x2 + 5) log(sin x + cos x) v dx tan x
1 dy
⇒ = (6x) log (sin x + cos x) + (3x2 + 5) × 2
dv = ( tan x ) x log tan x + x sec x 
( )
y dx ⇒ 

cos x − sin x dx  tan x 
2

sin x + cos x
dy 2
du dv dy du dv
⇒ = (sin x + cos x)3x + 5 On putting the values of and in = + ,
dx dx dx dx dx dx
  2
( )(
cos x − sin x 
6x log (sin x + cos x ) + 3x + 5 sin x + cos x  ) We get
dy
= x tan x sec 2 x log x +
tan x 
+
dx  x 
dy
Example 2. If y = xsin x × (tan x)x, then find .
dx  2 
Solution. y = xsin x× (tan x)x  ( tan x )x log tan x + x tan
sec x
x 
 
⇒ log y = log(xsin x) + log(tan x)x
⇒ log y = sin x log x + x log tan x x.... dy
Example 4. If y = x x , then find .
dx

1 dy
y dx (
= cos x log x +
sin x 
x  )
+  log tan x + x
sec 2 x 
tan x 
Solution. y= x
x x....
⇒ y = xx
x....
= x y ⇒ log y = y log x
dy
⇒ = xsin x (tan x)x 1 dy y dy dy  1  y
dx ⇒ = + log x ⇒ − log x =
y dx x dx dx  y  x
 cos x log x + sin x + log tan x + 2x cosec 2x 
 x 
dy  1− y log x  y dy y2
Example 3. Differentiate y = (x)tan x + (tan x)x, then find
dy
.

dx  y  = x ⇒
dx
=
x (1− y log x )
dx

Continuity And Differentiability 119


E:\AMIT_WORKS\Exam_Guru\EG_Mathematics-12_(working_02-06-2022)\EG_Mathematics-12_working\Open_Files\Chap_5\Chap_5
\ 16-Aug-2022 Amit Proof-4 Reader’s Sign _______________________ Date __________

EXERCISE 5.8
I. Multiple Choice Questions (MCQs) dy
3. If yx = xy, then find .
Choose the correct answer from the given options. dx
dy
1. If xx = yy, then dy = 4. If y = (log x)(log x), then find .
dx dx
x
(a) − y (b) − 5. If y = sin (xx), then find
dy
.
x y dx
x 1 + log x III. Long Answer Type Questions
(c) 1 + log   (d)
 y 1 + log y
log x
1. If (x)y = ex – y, then prove that dy = .
y x dy dx (1 + log x )2
2. If x × y = 1, then is equal to
dx
x x dy
y ( x log y − y ) y ( x log y − y ) 2. If y = x x or y = x (x ), then find .
(a) (b) dx
x ( y log x − y ) x ( y log x + y )
dy y
y ( x log y + y ) y ( x log y + y ) 3. If xmyn = (x + y)m + n, then prove that = .
(c) (d) − dx x
x ( y log x − x) x ( y log x + x)
dy
4. If (cos x)y = (sin y)x, then find .
dy dx
3. If y = (sin x)tan x, then is equal to
dx dy
5. If y = (tan x)tan x ÷ (sin x)sin x, then find .
(a) (sin x)tan x (1 + sec2 x log sin x) dx
(b) tan x (sin x)tan x – 1 cos x dy
(c) (sin x)tan x · sec2 x · log sin x 6. If y = (x)cot x + (sin x)x, then find .
dx
(d) tan x · (sin x)tan x – 1
dy
7. If y = (sin x ) + sin −1 x, then find
x
.
II. Short Answer Type Questions-II dx
(x − 3) (x − 4) (x + 1) dy x ..... dy
1. If y = , then find . 8. If y = x x , then find .
(x 2 + 2) (x3 + 5) dx dx
dy (log x )(log x ) ...... dy
2. If y = (x + 3)2(x + 4)3(x + 5)4, then find . 9. If y = ( log x ) , then find .
dx dx

Answers 5.8
x y
I. 1. (d) x =y
x  dy  y 
Taking logarithms at both sides, we get  + log x  +  + log y  = 0
 y  dx  x 
x log x = y log y
Differentiating both sides w.r.t. x, we get ⇒ 1 ( x + y log x ) dy + 1 ( y + x log y ) = 0

y dx x
1  1  dy
 x × + log x ⋅ 1 =  y ⋅ + log y ⋅ 1

1
( x + y log x)
dy 1
= − ( y + x log y )
 x   y  dx y dx x

dy dy y ( x log y + y )

⇒ (1 + log x) = (1 + log y)
⇒ = −
dx dx x ( y log x + x)
dy 1 + log x Hence, (d) is the correct answer.

⇒ =
dx 1 + log y 3. (d) y = (sin x)tan x
Hence, (d) is the correct answer. Taking logarithms of both sides
2. (d) xy · yx = 1 log y = tan x · log sin x
Taking logarithms of both sides w.r.t. x Differentiating both sides w.r.t. x, we get
y log x + x log y = 0 [ log 1 = 0] 1 dy 2
⋅ = tan x ⋅ cot x + log sin x ⋅ sec x
Differentiating both sides w.r.t. x, we get y dx
1 dy 1 dy dy
y ⋅ + (log x) + x⋅ + log y ⋅ 1 = 0
⇒ = y (1 + log sin x sec2 x)
x dx y dx dx

120 Mathematics–12
E:\AMIT_WORKS\Exam_Guru\EG_Mathematics-12_(working_02-06-2022)\EG_Mathematics-12_working\Open_Files\Chap_5\Chap_5
\ 16-Aug-2022 Amit Proof-4 Reader’s Sign _______________________ Date __________

dy dy xx  x − 1
+ dv (log x) 
tan x
or = (sin x) (1 + sec2 x log sin x) = x x
dx dx  dx 
Hence, (a) is the correct answer.
Now put the value of dv .
dy (x − 3) (x − 4) (x + 1) dx
II. 1. = 1 3.
dy
=
y
dx 2 (x 2 + 2) (x3 + 5) dx x
Hint: xmyn = (x + y)m + n
 1 3 2 
− 2 x − 3x 
1 1 2
× + + ⇒ m log x + n log y = (m + n) log (x + y)
 x − 3 x − 4 x + 1 x + 2 x + 5
Hint: Take log on both sides and then differentiate.
⇒ m +  n  dy =  m + n  1 + dy 
x  y  dx  x + y   dx 
dy 2 3 4 
2. = (x + 3)2 (x + 4)3(x + 5)4 ×  + +
dx  x + 3 x + 4 x + 5  m +  n  dy = m + n +  m + n   dy 


Hint: Take log on both sides and then differentiate.
x  y  dx x + y  x + y   dx 
dy y ( y − x log y ) Work out dy . It will come .
y
3. =
dx x ( x − y log x ) dx x
Hint: yx = xy
dy log (sin y) + y tan x
4. =
⇒ x log y = y log x
dx log (cos x) − x cot y
x dy y dy Hint: (cos x)y = (sin y)x
⇒ log y +
= + log x
y dx x dx ⇒ y log cos x = x log sin y
dy dy
Now find . ⇒ −y tan x + log cos x = x cot y + log sin y
dx dx
dy
dy 1 + log ( log x )  Find .
4. = (log x)(log x)   dx
dx  x  (tan x) tan x
dy
5. = [sec2 x log(tan x) + sec2 x
Hint:
y = ( log x )log x dx (sin x)sin x
– (cos x) log(sin x) – cos x]
⇒ log y = (log x) [log(log x)]

Hint: y = [(tan x)tan x] ÷ [(sin x)sin x]
1 dy =  1 log ( log x ) + log x 


y dx x{ } x log x  ( tan x )tan x
 
⇒ y=
(sin x )sin x
dy log x 1 + log ( log x ) 

⇒ = ( log x )  

⇒ log y = tan x log(tan x) – sin x log(sin x)

( )
dx  x  1 dy tan x sec2 x
dy
⇒ = sec2 x log ( tan x ) +
5. = xx cos (xx) (1 + log x) y dx tan x


dx
Hint: y = sin (xx) sin x( )
− cos x log (sin x ) − sin x cos x
Put v = xx  ⇒ y = sin v dy (tan x) tan x
= [sec2 x log(tan x) + sec2 x
⇒ dv = xx(1 + log x) dx (sin x)sin x
dx – (cos x) log(sin x) – cos x]
We have y = sin v, on differentiating we get dy cot x
dy 6. = (x)cot x  − (cosec 2 x) log x 
= (cos v) dv ...(1) dx  x 
dx dx + (sin x)x [log(sin x) + x cot x]
Put the value of v = x and dv = xx(1 + log x) in eq. (1)
x
Hint: y = (x)cot x + (sin x)x
dx
⇒ y=u+v
dy log x dy du + dv
III. 1. = ⇒ = ...(1)
dx (1 + log x )2 dx dx dx
cot x
Hint: (x)y = ex – y Now u=x and v = (sin x)x
x ⇒ log u = (cot x)(log x) and log v = x log(sin x)
⇒ y log x = x – y
⇒ y=
1 + log x Now find du and dv and put their values in equation (1).
dy log x dx dx

⇒ = dy 1
dx (1 + log x )2 7. = (sin x)x [log (sin x) + x(cot x)] +
dx 2 x 1− x
dy x
Hint: y = (sin x)x + sin −1 x  ⇒y=u+v
2. = x x  x x −1 + x x log x (1 + log x )
dx dy du + dv
x ⇒ = ...(1)
Hint:
y = x x = x v where v = xx dx dx dx
Now u = (sin x)x and v = sin −1 x
dv = xx(1 + log x)
dx
⇒ log u = x log(sin x) and v = sin −1 x

Continuity And Differentiability 121


E:\AMIT_WORKS\Exam_Guru\EG_Mathematics-12_(working_02-06-2022)\EG_Mathematics-12_working\Open_Files\Chap_5\Chap_5
\ 16-Aug-2022 Amit Proof-4 Reader’s Sign _______________________ Date __________

Differentiate and find du and dv and put their values in 9.


dy
=
y2
equation (1). dx dx dx (x log x) [1 − y log (log x) ]
dy y2 ......................
8. = (log x )(log x )
dx x(1 − y log x) Hint:
y = ( log x )
 x x.... 
x x....
x



⇒ y = (log x)y
Hint: y = x
⇒y=x = xy ⇒ y = xy
⇒ log y = y log x

⇒ log y = y log(log x)
1 dy dy y dy  1  y dy

⇒ = log x + ⇒ − log x = Now differentiate and find .
y dx dx x dx  y  x dx

Topic 9. Differentiation of Parametric Functions


If y = f (θ) and x = g(θ) be two functions of same independent
 2 tan θ  = tan −1 tan 2θ = 2θ
variable θ, then such functions are called parametric functions ⇒ y = tan −1  ( )
1 − tan 2 θ 
and θ is called a parameter.
y = f (q) dy 2
⇒ y = 2 tan–1 p ⇒ =
dy dx dp 1 + p 2
⇒ = f ′(q) and x = g(q)  ⇒ = g ′ (θ )
dθ dθ
dy  2 
 dy   2
 dθ f ′ (θ )
dy dp  1 + p  dy
dy Now = = = 1   ⇒ =1

( )
⇒ = = dx dx  2  dx
dx dx g ′ (θ ) dp  1 + p 2 

 2p  −1  2p  dy
Example 1. If x = sin −1  2  and y = tan 
, then Example 2. If y = 3 cos q – 2 cos3 q and x = 3 sin q – 2 sin3 q, then find
1+ p   1 − p 2  .
dx
dy Solution. y = 3 cos q – 2 cos3 q and x = 3 sin q – 2 sin3 q
prove that = 1.
dx Take y = 3 cos q – 2 cos3 q
 2p  −1  2p  dy
Solution. x = sin −1  2  and y = tan  ⇒ = –3 sin q + 6 cos2 q sin q
1+ p   1 − p 2  dθ
 2p  dy
Take x = sin −1  and put p = tan q ⇒ = 3 sin q[2 cos2 q – 1] = 3 sin q cos 2q
 1 + p 2  dθ
⇒ q = tan–1 p Take x= 3 sin q – 2 sin3 q
dx
⇒ x = sin −1  2 tan θ2  = sin −1 [sin 2θ ] = 2θ ⇒ = 3 cos q – 6 sin2 q cos q
1 + tan θ  dθ
= 3 cos q[1 – 2 sin2 q] = 3 cos q cos 2q
dx 2  dy 
⇒ x = 2 tan–1 p ⇒ =
dp 1 + p 2 dy
⇒ =  d θ  = 3 sin θ cos 2θ = tan θ
 2p  dx  dx  3 cos θ cos 2θ
Take y = tan −1  and put p = tan q  dθ 
 1 − p 2  dy
⇒ = tan q
⇒ q = tan–1 p dx

EXERCISE 5.9
I. Multiple Choice Questions (MCQs) 1 t dy
2. If cos x = and sin y = , then =
Choose the correct answer from the given options. 1 + t2 1+ t 2 dx
dy 1− t 1
1. If x = a (t – sin t) and y = a (1 – cos t), then =
dx (a) –1 (b) 1 (c) (d)
1+ t 2
1 + t2
t t
(a) tan (b) − tan   II. Short Answer Type Questions-II
2 2
 y = 3 sin θ − 2 sin 3 θ and dy
t t 1. If  , then find
(c) cot   (d) − cot   3 dx
2 2  x = 3 cos θ − 2 cos θ

122 Mathematics–12
E:\AMIT_WORKS\Exam_Guru\EG_Mathematics-12_(working_02-06-2022)\EG_Mathematics-12_working\Open_Files\Chap_5\Chap_5
\ 16-Aug-2022 Amit Proof-4 Reader’s Sign _______________________ Date __________


2. If  {
 x = a cos θ + log tan
θ
2 ( )} and, then find
dy
dx
at θ =
π
4

x = a
5. If 
sin −1 t
and
, then show that
dy −y
dx
=
x
.
 y = a sin θ  y = a cos t
−1
 
 x = a (cos θ + θ sin θ) and dy
3. If  , then find .  x = a (sin θ − cos θ) eθ and dy π
 y = a (sin θ − θ cos θ) dx 6. If  , then find at θ =
θ dx 4
 x = a (θ − sin θ) and  y = a (sin θ + cos θ) e
dy π
4. If  then find at θ = .
 y = a (1 + cos θ) dx 3

Answers 5.9
I. 1. (c) x = a (t – sin t) 1
dy
dx 1 + t2

∴ = a (1 − cos t )
\
dy
= dt = =1
dt dx dx 1
dy dt 1 + t 2
and y = a (1 − cos t ) ⇒ = a sin t
dt Hence, (b) is the correct answer.
dy
dy a sin t 2 sin t / 2 cos t / 2 dy

∴ = dt = = II. 1. = cot θ
dx a (1 − cos t ) dx
dx 2 sin 2 t / 2
dt Hint: dy = 3 cos θ cos 2θ and


= cos t / 2 = cot t dx = 3 sin θ cos 2θ
sin t / 2 2

Hence, (c) is the correct answer.
1  1  2. dy =1
2. (b) cos x = , ∴ x = cos  −1  dx at θ = π
4
1 + t2  1 + t2 
  2 

Put t = tan q, then Hint: dy = a cos θ and dx = a  cos θ 

dθ dθ  sin θ 
 1 
x = cos −1   dy
3. = tan q
 1 + tan 2 θ  dx
 
dx a θ cos θ, dy a sin
 1    Hint:
= = θ θ
= cos −1   = cos −1  1  dθ dθ
 sec2 θ   sec θ 
  dy
4. = − 3

⇒ x = cos–1 (cos q) = q = tan–1 t dx at θ = π
3
dx 1

⇒ = dy
dt 1 + t2 Hint:
= − 2a sin θ cos θ ; dx = 2a sin 2 θ
dθ 2 2 dθ 2
t dy y
Again, sin y = 5. = −
1+ t 2 dx x
−1 t −1 t
 t  Hint: xy = a sin
× a cos

⇒ y = sin −1    sin −1 t + cos −1 t 
 1 + t2   
   2 
=a
 tan α 
Put t = tan a, then y = sin −1 
⇒ xy = a

( π4 )
 1 + tan 2 α 
 
Now differentiate.
 tan α 
−1 dy

⇒ y = sin   6. =1
 sec α  dx at θ = π
4
= sin–1 (sin a) = a = tan–1 t
dx = 2a (sin θ) eθ
Hint:
dy 1 dθ

⇒ =
dt 1 + t2 dy
= 2a (cos θ) eθ
and

Continuity And Differentiability 123


E:\AMIT_WORKS\Exam_Guru\EG_Mathematics-12_(working_02-06-2022)\EG_Mathematics-12_working\Open_Files\Chap_5\Chap_5
\ 16-Aug-2022 Amit Proof-4 Reader’s Sign _______________________ Date __________

Topic 10. Higher Order Derivatives


Let y = f (x) be a function. dy
On putting the value of (y – b) in ( x − a ) + ( y − b ) = 0, we
dx
dy get
Then first order derivative = = y′ = y1 = f ′ ( x )
dx   dy  2 
1 +  dx   dy
d 2y (x – a) =  
Second order derivative = = y′′ = y2 = f ′′ ( x ) 2
dx 2  d y  dx
 dx 2 
d 3y
Third order derivative = = y′′′ = y3 = f ′′′ ( x ) Now on putting the values of (x – a) and (y – b) in (x – a)2 +
dx3 (y – b)2 = c2, we get
Example 1. If (x – a) 2 + (y – b) 2 = c 2 , then prove that 2 2
  dy  2    dy  2 
3 1 +
  dx   dy 2 1 + 
  dy  2 2

    +   dx   = c2
1 +  dx    d y   dx   d y 
2 2
  = c.  dx 2   dx 2 
d 2y
2
dx 2   dy  2 
1 +  dx     dy  2 
Solution. (x – a)2 + (y – b)2 = c2 2
⇒  2
 1 +    = c
 d y   dx 
On differentiating w.r.t. x, we get
 dx 2 
⇒ ( x − a ) + ( y − b) dy
dx
= 0
  dy  2 
3

1 +  dx  
Again differentiating w.r.t. x, we get  
⇒ 2 = c2
2
 d y
2 2
d y  dy 
1 + ( y − b) + = 0  dx 2 
dx 2  dx 
3
  dy  2    dy  2  2
1 +  dx   1 +  dx  
⇒ (y – b) = –  2
 ⇒   =c
 d y  d 2y
 dx 2 
dx 2

EXERCISE 5.10
I. Multiple Choice Questions (MCQs) 5
(c) (d) None of these.
Choose the correct answer from the given options. 16 t 6
d2y 1 d 2u du
1. If x = at2, y = 2 at, then = 4. u = v3 log , show that v 2 − 2 + 3v 2 =
dx 2 v dv dv
1
(a) − (b) − 1 (a) 1 (b) 2
t2 t3 (c) 0 (d) None of these.
1 1
(c) (d) − II. Long Answer Type Questions
2at 3 2at 3
d 2y
2 d y 2 1. If y = x sin x, then find .
2. If x = 3
, y = t – 1, then = dx 2
t 2
dx 2
d 2y
15 2. If y = sin–1 x, then find .
(a) 15t2 (b) 16t2 (c) (d) 15 t 7 dx 2
16t 3 16 d 2y
d y 2 3. If y = a sin x + b cos x, then prove that + y = 0.
3. Let y = t10 + 1 and x = t8 + 1, then = dx 2
dx 2
 d 2y 
5 8
4. If y = x + tan x, then prove that cos2 x  2  − 2y + 2x = 0.
(a) t (b) 20t  dx 
2

124 Mathematics–12
E:\AMIT_WORKS\Exam_Guru\EG_Mathematics-12_(working_02-06-2022)\EG_Mathematics-12_working\Open_Files\Chap_5\Chap_5
\ 16-Aug-2022 Amit Proof-4 Reader’s Sign _______________________ Date __________

5. If y = tan x + sec x, then prove that  d 2y 


 d y 2 (  dx 
) dy
1 − x 2  2  − x   + m2 y = 0
 dx 
(1 − sin x)2  2  = cos x.

 dx  21. If y = cos(cos x), then prove that
6. If y = cosec x + cot x, then prove that  d 2y   dy 
 dx 2  − ( cot x )  dx  + y sin x = 0
2

 d 2y 
(1 − cos x)2  2  = sin x.

 dx  22. If y = (sin x + cos x)ex, then prove that
2
7. If y = a sin mx – b cos mx, then prove that d y2 + m 2 y = 0 d 2y dy
2 − 2   + 2y = 0
dx dx  dx 
 d 2y   dy  −1
8. If y = tan–1 x, then prove that (1 + x 2)  2  + 2x
 dx 
=0 23. If y = sin x , then prove that
 dx  1 − x2
 d 2y   dy   d 2y  dy
9. If y = sin–1 x, then prove that (1 − x 2)  2  − x
 dx 
=0 (1 − x 2)  2  − 3x   − y = 0

 dx   dx   dx 
10. If y = cosec–1 x, then prove that  d 2y  dy
24. If y = (sin x)ex, then show that  2  − 2   + 2y = 0
 d y 2
dy  dx   dx 
x (x 2 − 1)  2  + (2x 2 − 1)   = 0

 dx   dx 
25. If xy = aex + be–x + x2, then prove that
11. If y = (tan–1 x)2 , then prove that  d 2y   dy  2
x  2  + 2   − xy + x − 2 = 0
 d 2y   dx  dx
dy
(x 2 + 1) 2  2  + 2x (x 2 + 1)   = 2

 dx   dx  d 2y  dy 
26. If y = (ax + b)e3x, then prove that 2 − 6  dx  + 9y = 0
12. If y = (cot–1 x)2, then show that dx
 d 2y  dy  y = θ sin θ + cos θ and d 2y
( x 2 + 1) 2  2  + 2x ( x 2 + 1)   = 2
27. If  , then find .
 dx   dx   x = θ cos θ − sin θ dx 2
2
d 2 y  1   dy  y
13. If y = (x)x, then prove that 2 −   dx  − x = 0  y = 3 sin θ − 2 sin 3 θ and d 2y π
dx  y 28. If  , then find 2 at θ = 2
3 dx
14. If y = Peax + Qebx, then prove that  x = 3 cos θ − 2 cos θ
 −1  2t 
d 2y  dy  =0  y = sin  and
2 − (a + b)  dx  + aby   1 + t 2  d 2y
dx 29. If  , then find .
4
 x = cos −1  1 − t 
2 dx 2
15. If y =  x + x 2 + 1 , then prove that
    1 + t 2 

2
 d y dy
(1 + x 2)  2  + x   − 16y = 0  x = a cos θ + b sin θ
 dx   dx  30. If  , then prove that
 y = a sin θ − b cos θ
16. If y = sin(log x), then prove that
 d 2y  dy
y2  2  − x   + y = 0
2
2d
y dy
x  2  + x  + y = 0
 dx   dx 
 dx   dx 
17. If y = a cos(log x) + b sin(log x), then prove that  x = cos θ
2 31. If  , then prove that
y 2d
dy
x  2  + x  + y = 0
3
 y = sin θ
 dx   dx 
2
 d2y dy
n n
y  2  +   = 3 sin2 q (5 cos2 q – 1)
18. If y = P  x + x 2 − 1 + Q  x − x 2 − 1 , then prove that
 dx   dx 
   
 d 2y  d 2y
(
 dx 
)dy
x2 − 1  2  + x   − n2 y = 0
 dx 
32. If xmyn = (x + y)m + n, then prove that
dx 2
=0
−1x
19. If y = e m cos , then prove that d 2y
33. If ey(x + 1) = 1, then prove that = 0
dx 2
 d 2y 
(
 dx 
) dy
1 − x 2  2  − x   − m2 y = 0
 dx  34. If y = ex(tan–1 x), then prove that
 d2y
 sin −1 y 
20. If x = sin 
 m 
, then prove that ( )
1 + x2  2  − 2 1 − x + x2
 dx 
dy
dx (
+ (1 − x ) y = 0
2
)
Continuity And Differentiability 125
E:\AMIT_WORKS\Exam_Guru\EG_Mathematics-12_(working_02-06-2022)\EG_Mathematics-12_working\Open_Files\Chap_5\Chap_5
\ 16-Aug-2022 Amit Proof-4 Reader’s Sign _______________________ Date __________

Answers 5.10
dx d 2u
I. 1. (d) x = at2, ∴ = 2at
⇒ = − 5v − 6v log v
dt dv 2
dy d 2u du
and y = 2at, ∴ = 2a
dt
\ v 2
−2 = − 5v 2 − 6v 2 log v + 2v 2 + 6v 2 log v
dv dv
dy
dy 2a 1 d 2u du
Now = dt = =
⇒ v 2
−2 + 3v 2 = 0
dx dx 2at t dv dv
dt Hence, (c) is the correct answer.
2
d y d2y d2y x

∴ = d  dy  × dt = d 1 1
 × II. 1. = 2 cos x − x sin x 2. = 3
dx 2 dt  dx  dx dt  t  2at dx 2 dx 2
(1 − x 2) 2
1 1 1 3. Hint: y = a sin x + b cos x
×= − =−
t 2 2at 2at 3 dy

⇒ = a cos x – b sin x
Hence, (d) is the correct answer. dx
2 dx − 4 d 2y
2. (d) x= 2, ∴ = 3
⇒ = –a sin x – b cos x = –y
t dt t dx 2
4. Hint: y = x + tan x
dy
Also y = t3 – 1, ∴ = 3t 2
dt dy d 2y

⇒ = 1 + sec2 x ⇒ = 2 sec2 x tan x
dx dx 2
dy
dy 3t 2 3 2
Now,
dx
= dt =
dx  − 4 
= − t5
4

⇒ (cos x) ddxy
2
2
= 2 tan x = 2y – 2x
dt  t 3  2

d y 2
d  dy  dt 3 1 15 7

⇒ (cos x) ddxy
2
2
– 2y + 2x = 0
=  × = − (5t 4 ) × = t
dx 2 dt  dx  dx 4 −4 16 5. Hint: y = tan x + sec x
 3  dy
 t 
⇒ = sec2 x + sec x tan x
Hence, (d) is the correct answer. dx
3. (c) We have y = t10 + 1 and x = t8 + 1 dy

⇒ = sec x(sec x + tan x)
dx
Now, t8 = x – 1 ⇒ (t2)4 = (x – 1)


\
\
t2 = (x – 1)1/4
y = t10 + 1 = (t2)5 + 1 = (x – 1)5/4 + 1


dy
dx
= sec x
cos x (
1 + sin x 1 + sin x
=
cos 2 x )
dy 1
Now differentiating both sides w.r.t. x we get
⇒ =
dx 1 − sin x
dy 5
= ( x − 1)1/ 4 d 2y cos x
dx 4
⇒ 2 =
Again differentiating both sides w.r.t. x, we get
dx (1 − sin x )2
6. Hint: y= cosec x + cot x
d2y 5 5
= ( x − 1) −3/ 4 = dy
dx 2
16 16( x − 1)3/ 4
⇒ = –cosec x cot x – cosec2 x
dx
d2y 5 5 dy

\ = = [ x – 1 = t8]
⇒ = –cosec x(cot x + cosec x)
dx 2 16(t )8 3/ 4
16t 6 dx
Hence, (c) is the correct answer.
1


dy
dx
= −cosec x (1 + cos x
sin x )
4. (c) u = v3 log = v3 (log 1 – log v) dy 1 + cos x = −1
v
⇒ = −
dx sin 2 x 1 − cos x
⇒ u = – v3 log v
du d 2y sin x
1
⇒ =
⇒ = −  v3 ⋅ + log v ⋅ 3v 2  = − v 2 − 3v 2 log v dx 2 (1 − cos x )2
dv  v 
7. Hint: y = a sin mx – b cos mx
d 2u 1
and = − 2v − 3 v 2 ⋅ + ( log v ) 2v  dy
dv 2  v 
⇒ = am cos mx + bm sin mx
dx
2
d u d 2y

⇒ = − 2v − 3v − 6v log v
⇒ = –am2 sin mx + bm2 cos mx
dv 2 dx 2

126 Mathematics–12
E:\AMIT_WORKS\Exam_Guru\EG_Mathematics-12_(working_02-06-2022)\EG_Mathematics-12_working\Open_Files\Chap_5\Chap_5
\ 16-Aug-2022 Amit Proof-4 Reader’s Sign _______________________ Date __________

d 2y dy

⇒ = –m2(a sin mx – b cos mx) 14. Hint: y = Peax + Qebx ⇒ = Paeax + Qbebx
dx 2 dx
d 2y
d 2y d 2y
⇒ = Pa2eax + Qb2ebx...(1)
⇒ = –m2y ⇒ + m2y = 0 dx 2
dx 2 dx 2
 dy = Paeax + Qbebx
8. Hint: y = tan–1 x 
dy 1 dy

⇒  dx
⇒ = ⇒ (1 + x2) =1  y = Peax + Qebx
dx 1 + x2 dx
2
Solve these two equations for Peax and Qebx. It gives:

⇒ (1 + x ) ddxy + 2x  dy
2
2
 =0
dx  ax
 dy − by 
 dx  bx
 dy − ay 
 dx 
Pe =  and Qe = − 
9. Hint: y = sin–1 x a−b  a−b 
   
dy 1 dy

⇒ = ⇒ 1 − x2 =1 Put these values of Peax and Qebx in equation (1)
dx 1 − x2 dx
d 2y
d 2y dy = Pa2eax + Qb2ebx

⇒ 1− x2
+ −2x   = 0 dx 2
dx 2 2 1 − x 2  dx 
d 2y
2
⇒ = a2(Peax) + b­2(Qebx)

⇒ (1 − x ) ddxy − x  dy
2  =0
dx 
2
dx 2
 dy − by   dy ay 
10. Hint: y = cosec–1 x d 2y 2  dx  2  dx − 

⇒ = a  a−b  − b  a −b 
dy −1 dy 2 dx 2
⇒ = ⇒ x x − 1 = −1    
dx x x2 − 1 dx
d 2y 1  2 dy dy
 d 2y    dy
⇒ = a − a 2by − b 2 + ab 2 y 

⇒ x x 2 − 1  2  +  x 2 − 1 + x × 2x  =0 dx 2 a − b  dx dx 
 dx   2 x − 1  dx
2
d 2y  dy 

⇒ 2 − ( a + b )  dx  + aby = 0
2

⇒ (
x x2 − 1 ) ddxy + (2x − 1) dy
2 dx
=0 2 dx
4
11. Hint: y = (tan–1 x)2 15. Hint: y =  x + x 2 + 1
 
dy 2 tan −1 x dy

⇒ =  ⇒ ( x 2 + 1) = 2 tan −1 x 3
dx x2 + 1 dx

dy
= 4  x + x 2 + 1 1 + 2x 
2
dx    2 x 2 + 1 

⇒ ( x + 1) ddxy + 2x  dy
2
2
= 2
dx  1 + x 2
dy 4
1
= 4  x + x + 1 ×
2


2 dx   x2 + 1
( x + 1) d y
(
dy
)
2 2 2

⇒ 2 + 2x x + 1 dx = 2
dx dy

⇒ x2 + 1 = 4y
12. Hint: y = (cot–1 x)2 dx
−2 cot −1 x d 2 y  2x  dy

dy
⇒ =
dx 1 + x2
⇒ x2 + 1
dy
dx
= −2 cot −1 x ( )
⇒ x2 + 1 +
dx 2  2 x 2 + 1  dx
=4
dy
dx
2

⇒ ( x + 1) ddxy + 2x  dy
2
2
= 2
dx  x + 1 2

2
2
( x + 1) ddxy + x dy
2 =4 x2 + 1
dy
dx dx
2
( x + 1)  ddxy  + 2x ( x + 1)  dy  =2
2 2
( x + 1) ddxy + x dy
2 2


⇒ 2
= 4 × 4y
2 dx  2 dx
x
13. Hint: y = x 2


⇒ log y = x log x ⇒
1 dy
= 1 + log x

⇒ ( x + 1) ddxy + x dy
2
2 dx
− 16y = 0
y dx
dy 16. Hint: y = sin(log x)

⇒ = y(1 + log x)
dx dy cos ( log x )  dy  = cos log x

⇒ = ⇒ x ( )
() dx x  dx 
2
d y dy

⇒ 2 = dx
(1 + log x ) + y 1x
dx
 d 2 y  dy sin ( log x )
2
⇒ x 2  + =−
d y dy  dy  1 y  dx  dx x

⇒ = +
dx 2 dx  dx  y x
2  d 2y dy
d 2 y 1  dy  y
⇒ x2  2  + x + y = 0

⇒ − − =0  dx  dx
dx 2 y  dx  x

Continuity And Differentiability 127


E:\AMIT_WORKS\Exam_Guru\EG_Mathematics-12_(working_02-06-2022)\EG_Mathematics-12_working\Open_Files\Chap_5\Chap_5
\ 16-Aug-2022 Amit Proof-4 Reader’s Sign _______________________ Date __________

17. Hint:
dy
y = a cos(log x) + b sin(log x)
−a sin ( log x ) b cos ( log x )

 d 2y
1 − x 2  2  + −2x   =
dy −m 2 sin m sin −1 x ( )

⇒ = +  dx  2 1 − x 2  dx  1 − x2
dx x x
2

⇒ x  dy 
 dx  = –a sin(log x) + b cos(log x)

⇒ (1 − x )  ddxy  − x  dy
2
2
 +m y=0
dx 
2

 d 2 y  dy −a cos ( log x ) b sin ( log x ) dy



⇒ x 2  + = − 21. Hint: y = cos(cos x) ⇒ = [sin(cos x)]sin x
 dx  dx x x dx
d 2y
2 d 2y dy
⇒ = [cos(cos x)(– sin x)]sin x + [sin(cos x)]cos x

⇒ x  2  + x  + y = 0 dx 2
 dx   dx 

18. Hint:
n
y = P  x + x − 1 + Q  x − x − 1
2 2
n

d 2y
dx 2
2
( ) cos x  dy 
= − sin x cos ( cos x ) + sin x  dx 
   



dy n −1 
= Pn  x + x 2 − 1 1 + 2x 



d 2y
dx 2
= − sin x ( y ) + cot x
2 dy
dx ( )
dx  
 2 x2 − 1  d 2y dy

⇒ − ( cot x )   + y sin 2 x = 0
n n dx 2  dx 
dy Pn  x + x 2 − 1 Qn  x − x 2 − 1 22. Hint: y = (sin x + cos x)ex
   

⇒ = −
dx 2
x −1 x2 − 1 dy

⇒ = (sin x + cos x)ex + (cos x – sin x)ex
n n dx
dy

⇒ x 2 − 1   = Pn  x + x 2 − 1  −Qn  x − x 2 − 1
  dy
 dx   
⇒ = y + (cos x – sin x)ex
dx
 d y 2
2x  dy  d 2y dy

⇒ x2 − 1  2  +
⇒ = + (–sin x –cos x)ex + (cos x – sin x)ex
 dx  2 x 2 − 1  dx  dx 2 dx
n n
d 2y dy  dy − y 
Pn 2  x + x 2 − 1  Qn 2  x − x 2 − 1
⇒ = –y+
=   +   dx 2 dx  dx 
x2 − 1 x2 − 1 d 2y dy

⇒ − 2   + 2y = 0
2 dx 2  dx 

⇒ ( x − 1)  ddxy  + x  dy
2
2
 −n y=0
dx 
2
sin −1 x
23. Hint: y= ⇒ 1 − x 2 ( y ) = sin −1 x
m cos −1 x 1 − x2
19. Hint: y= e
dy −2x 1
dy −m em cos −1 x
⇒ 1 − x2   + ( y) =

⇒ =  dx  2
dx 2 1− x 1 − x2
1 − x2


dy
1 − x2 ( ) −1
= −mem cos x

⇒ (1 − x ) dy
2
dx
− xy = 1
dx 2
On differentiating, we get
⇒ (1 − x ) ddxy − (2x)  dy
2
2
 − x  dy  = 0
dx   dx 
 d y 2 2
dy
1 − x 2  2  + −2x   = m em cos x
−1 2

 dx  2 1 − x 2  dx  1− x 2
⇒ (1 − x ) ddxy − 3x  dy
2
2
 =0
dx 

24. Hint: y = (sin x)ex
 d 2y 

⇒ ( )
 dx 
dy
1 − x 2  2  − x   − m2 y = 0
 dx 

dy
dx
= (cos x)ex + (sin x)ex
dy
 sin −1 y 
⇒ = (cos x)ex + y
20. Hint: x = sin  dx
 m 
d 2y dy

⇒ = (cos x)ex – (sin x)ex +
sin y−1
dx 2 dx

⇒ = sin­–1 x ⇒ sin–1 y = m sin–1 x
m d 2y  dy  dy
–1
⇒ =  − y − y +

⇒ y = sin[m sin x] dx 2  dx  dx



dy
=
m cos m sin −1 x ( )

d 2y
dx 2
dy
− 2   + 2y = 0
 dx 
dx 1 − x2 25. Hint: xy = aex + be–x + x2



dy
1 − x 2   = m cos m sin −1 x
 dx  ( )

dy
x   + y = ae x − be −x + 2x
 dx 

128 Mathematics–12
E:\AMIT_WORKS\Exam_Guru\EG_Mathematics-12_(working_02-06-2022)\EG_Mathematics-12_working\Open_Files\Chap_5\Chap_5
\ 16-Aug-2022 Amit Proof-4 Reader’s Sign _______________________ Date __________

d 2y


 d 2 y  dy dy
x 2  +
 dx  dx dx
+ = ae x + be −x + 2

dx 2
(
= − cosec2 θ

dx )
1
 d 2y  dy = −cosec2 θ ×

⇒ x  2  + 2   = xy − x 2 + 2 3 sin θ cos 2θ
 dx   dx 
cosec3 θ 
= − =1


 d 2y  dy
x  2  + 2   − xy + x 2 − 2 = 0 3 cos 2θ  3
θ= π
 dx   dx  2

26. Hint: y = (ax + b)e3x  −1  2t 


 y = sin  1 + t 2 
dy 
29. Hint: 

⇒ = 3(ax + b)e3x + ae3x 2
dx  x = cos −1  1 − t 
 
 1 + t 2 
dy

⇒ = 3y + ae3x Put t = tan q
dx
d 2y  −1  2 tan θ 
 y = sin  1 + tan 2 θ  = sin (sin 2θ ) = 2θ
 dy  + 3ae3x −1

⇒ = 3
dx 2  dx 
⇒ 
 2
2
d y  dy  + 3  dy − 3y   x = cos −1  1 − tan θ  = cos −1 ( cos 2θ ) = 2θ

⇒ = 3  
 1 + tan θ 
2
dx 2  dx   dx 

d 2y  dy  + 3  dy  − 9y
⇒  y = 2θ = 2 tan −1 t

⇒ = 3  −1
dx 2  dx   dx   x = 2θ = 2 tan t
d 2y  dy   dy = 2

⇒ 2 − 6  dx  + 9y = 0  dt 1 + t 2
dx
⇒ 
 y = θ sin θ + cos θ  dx = 2 2
27. Hint:   dt 1 + t
 x = θ cos θ − sin θ
dy d 2y
 dy = sin θ + θ cos θ − sin θ = θ cos θ
⇒ =1 ⇒ =0
dx dx 2
 dθ

⇒   dy 
 dx = cos θ − θ sin θ − cos θ = −θ sin θ dy  d θ  a cos θ + b sin θ  −x 
 dθ 30. Hint: = = =
dx  dx  −a sin θ + b cos θ  y 
dy dy / d θ θ cos θ  dθ 

⇒ = = = − cot θ
dx dx / d θ −θ sin θ
  dy  
d 2y  y − x  dx  


dx 2
(
= cosec2 θ

dx ) d 2y
⇒ 2 =−
dx y2

 

( )
d 2y 1  

⇒ = cosec2 θ
dx 2 −θ sin θ  d 2y dy
⇒ y2  2  − x   + y = 0
2
d y cosec θ 3  dx   dx 

⇒ = −
dx 2 θ
 x = cos θ
 y = 3 sin θ − 2 sin 3 θ 31. Hint:  3
28. Hint:   y = sin θ
3
 x = 3 cos θ − 2 cos θ
 dx = − sin θ
 dy = 3 cos θ − 6 sin 2 θ cos θ  dθ
 dθ
⇒ 

⇒   dy = 3 sin 2 θ cos θ
 dx = −3 sin θ + 6 cos 2 θ sin θ  dθ
 dθ dy

⇒ = –3 sin q cos q
dx
(
 dy = 3 cos θ 1 − 2 sin 2 θ
 dθ ) d 2 y = −3  − sin 2 θ + cos 2 θ  d θ

⇒ 
⇒   dx
(
 dx = 3 sin θ 2 cos 2 θ − 1 ) dx 2

( )
 dθ d y2
−1

⇒ = 3 sin 2 θ − cos 2 θ 
 dy = 3 cos θ cos 2θ dx 2 sin θ
 dθ

⇒  d 2y  cos 2 θ − sin 2 θ 

⇒ 2 = 3
 dx = 3 sin θ cos 2θ dx  sin θ 

 dθ
 d 2 y   dy  2
dy dy / d θ 3 cos θ cos 2θ
⇒ y 2  +

⇒ = = = cot θ  dx   dx 
dx dx / d θ 3 sin θ cos 2θ

Continuity And Differentiability 129


E:\AMIT_WORKS\Exam_Guru\EG_Mathematics-12_(working_02-06-2022)\EG_Mathematics-12_working\Open_Files\Chap_5\Chap_5
\ 16-Aug-2022 Amit Proof-4 Reader’s Sign _______________________ Date __________

 2 2  dy −1 × x + 1 = −1
= 3 sin 3 θ  cos θ − sin θ  + [ −3 sin θ cos θ ]
2

⇒ =
 sin θ  dx ( x + 1)2 1 x + 1
= 3 sin2 q[cos2 q – sin2 q] + 9 sin2 q cos2 q d 2y 1  dy  −1

⇒ = 2 and  dx  = x + 1
= 3 sin2 q cos2 q – 3 sin4 q + 9 sin2 q cos2 q dx 2 ( )
x + 1 ( )
= 12 sin2 q cos2 q – 3 sin4 q 2
d 2y  dy 
= 3 sin2 q[4 cos2 q – sin2 q]
⇒ =
dx 2  dx 
= 3 sin2 q[5 cos2 q – 1] 34. Hint: y = ex(tan–1 x)
32. Hint: xmyn = (x + y)m + n
dy e x + e x tan −1 x
⇒ m log x + n log y = (m + n) log(x + y)
⇒ =
dx 1 + x2
m + n dy = m + n 1 + dy 

⇒ dy ex + y
x y dx x + y  dx 
⇒ = ...(1)
dx 1 + x2



dy  my − nx  my − nx
=
dx  y ( x + y )  n ( x + y )

⇒ (1 + x2
dy
dx
)
= ex + (1 + x2)y
2


⇒ dy y
=
⇒ (1 + x ) ddxy + 2x dy
2
2
dx
dx x

2
dy
x  − y
= ex +
dy
dx
( )
1 + x 2 + y ( 2x )
d y  dx  1 dy y

⇒ 2 = =  −  2
dx x 2 x  dx x 
⇒ (1 + x ) ddxy + (2x − 1 − x ) dy
2
2 dx
2
= ex + y(2x)
d 2y 1  y y  2

⇒ = −
dx 2 x  x x 
=0

⇒ (1 + x ) ddxy − ( x
2
2
2
− 2x + 1 ) dy
dx
1
33. Hint: ey(x + 1) = 1 ⇒ ey =
x +1
...(1)
( 2  dy 
)
= 1 + x  dx − y + 2xy  [Using (1)]
dy −1 2

⇒ ey
dx
=
( + 1)2
x

(1 + x ) ddxy − (2x
2
2
2
− 2x + 2 ) dy
dx
+ (1 − x )
2
y=0

1 dy −1 2


( x + 1) dx =
( x + 1)2
 [Using (1)]

(1 + x ) ddxy − 2( x
2
2
2
− x +1 ) dy
dx
+ (1 − x )
2
y=0

Case Based Questions


1. A potter made a mud vessel, where the shape of the pot is (i) When x > 4, what will be the height in terms of x?
based on
(a) x – 2 (b) x – 3
f(x) =| x − 3 | + | x − 2 | , where f (x) represents the height
of the pot. (c) 2x – 5 (d) 5 – 2x
(ii) For x = 3, we have f (3) equal to
(a) 1 (b) 2
(c) 3 (d) 4
dy
(iii) What is at x = 3 ?
dx
(a) 2 (b) – 2
(c) Function is not differentiable
(d) 1
(iv) When the x value lies between (2, 3), then the function
is
(a) 2x – 5 (b) 5 – 2x
(c) 1 (d) 5

130 Mathematics–12
E:\AMIT_WORKS\Exam_Guru\EG_Mathematics-12_(working_02-06-2022)\EG_Mathematics-12_working\Open_Files\Chap_5\Chap_5
\ 16-Aug-2022 Amit Proof-4 Reader’s Sign _______________________ Date __________

(v) If the potter is trying to make a pot using the function (i) The derivative of log (log x) is
f(x) = [x], will he get a pot or not? Why? 1 1
(a) (b)
(a) Yes, because it is a continuous function log x x
(b) Yes, because it is not continuous function
1 x
(c) No, because it is a continuous function (c) (d)
x log x log x
(d) No, because it is not continuous function
(ii) The derivative of sin (log x) is
Ans. (i) (c) 2x – 5 (ii) (a) 1 cos (log x)
(a) cos (log x) (b)
(iii) (c) Function is not differentiable x
(iv) (c) 1 (log x) sin x
(c) sin (d)
(v) (d) No, because it is not continuous function x log x

2. Mrs Pallavi Jain, a teacher of Blue Marble School, is (iii) The derivative of cot (4x2 + 3) at x = 2 is
teaching chain rule to his students in the classroom. Let 2 2
f be a real valued function which is a composite of two (a) −8 2 cos ec (11) (b) −16 cosec (4 2 )
functions u and v; i.e., f = vou. Suppose t = u(x) and if both
(c) 1 (d) 2
dt dv df dv dt
(iv) The derivative of tan [sin(cosx)] is
and exist, we have = .
dx dt dx dt dx
(a) cos x sin (cos x). sec2 (sin (cos x)]
(b) –sin x .cos (cos x). sec2 {sin (cos x)}

(c) cos x sin (sin x). tan (sin (cos x)]
(d) None of these
π2
(v) The derivative of sec [tan ( x )] at x = is
16
π 4
(a) sec 2 (1) (b) tan 2 (1)
4 π
4 4
(c) π sec [tan (1)] (d) sec (1). tan (1)
π
1
Ans. (i) (c) (ii) (b) cos (log x)
x log x x


Chain rule may be extended as follows. Suppose f is a real 2
(iii) (a) −8 2 cosec (11)
valued function which is a composite of three functions u,
2
v and w, i.e., f = (w.u.).v (iv) (b) −sin x.cos (cos x). sec {sin (cos x)}
df d ( wou ) dt dw ds dt 4

If t = v (x) and s = u (t), then = . = . . (v) (d) sec (1). tan (1)
dx dt dx ds dt dx π

Author’s Comments
Questions based on following types are very important for Exams. So, students are advised to revise them thoroughly.
1. To prove continuity and differentiability of a given function at a point. (Most Important)
2. Differentiation of implicit function, differentiation using logarithms, differentiable and second order derivatives. (Most Important)
3. Find the values of a and b when the given function is continuous.
4. Find the value of k, when the function is continuous.
5. Derivatives of parametric functions.

Continuity And Differentiability 131


E:\AMIT_WORKS\Exam_Guru\EG_Mathematics-12_(working_02-06-2022)\EG_Mathematics-12_working\Open_Files\Chap_5\Chap_5
\ 16-Aug-2022 Amit Proof-4 Reader’s Sign _______________________ Date __________

IMPORTANT FORMULAE
1. Continuity of a function f (x) at a point x = a

A function y = f (x) is continuous at x = a, if f ( a ) = lim f ( x ) = lim f ( x ) = n .


x→ a − x→ a+

Here lim f ( x ) is called left hand limit and lim f ( x ) is called right hand limit of f (x).
x→ a − x→ a+

2. Differentiability of a function f (x) at x = x0

A function f (x) is differentiable at x = x0, if its left hand derivative

 f ( x0 − h ) − f ( x0 ) 
(LHD) = lim   is equal to its right hand derivative
h→0  −h 

 f ( x0 + h ) − f ( x0 ) 
(RHD) = lim  
h→0  h 

3. Standard formulae for direct differentiation:

(i)
d n
dx ( )
x = nx n −1 (ii)
d
dx ( )
αx n = α  nx n −1 (iii)
d x
dx ( )
e = ex

(iv)
d x
dx ( )
a = a x log a (v)
d
dx
(loge x ) = 1x d 1 1
(vi) dx ( log a x ) = x × log a

d d d
(vii)
dx
(k ) = 0 (viii)
dx
(sin x ) = cos x (ix)
dx
(cos x ) = − sin x

d d d
(x)
dx
( tan x ) = sec2 x (xi)
dx
(cot x ) = − cosec2 x (xii)
dx
(sec x ) = sec x tan x

(xiii)
d
dx
(cosec x ) = − cosec x cot x (xiv)
d
dx
(
sin −1 x = ) 1
1 − x2
d
(
−1
(xv) dx cos x =
−1
1 − x2
)

dx (
(xvi) d tan −1 x = 1 2
1+ x
) (xvii)
d
dx (
cot −1 x = ) −1
1 + x2
(xviii)
d
dx
(
sec −1 x =) 1
x x2 − 1

(xix)
d
dx
(
cosec −1 x = )
−1
x x2 − 1

4. Differentiation of algebraic functions by chain rule:


n dy n −1 1 dy −nf ′ ( x )
(i) If y =  f ( x ) ⇒ = n  f ( x ) f ′ ( x ) (ii) If y = n ⇒ dx =
dx  f ( x )  f ( x )
n +1

dy f ′ ( x) 1 dy f ′ ( x)
(iii) If y = f ( x) ⇒ = (iv) If y = ⇒ =−
dx 2 f ( x ) f ( x) dx 2  f ( x )
3/ 2

132 Mathematics–12
E:\AMIT_WORKS\Exam_Guru\EG_Mathematics-12_(working_02-06-2022)\EG_Mathematics-12_working\Open_Files\Chap_5\Chap_5
\ 16-Aug-2022 Amit Proof-4 Reader’s Sign _______________________ Date __________

5. Differentiation of trigonometric and inverse trigonometric functions by chain rule


dy 
(i) y = sinm f (x) ⇒ = m sin m −1 f ( x ) cos f ( x ) f ′ ( x )
dx 

dy 
(ii) y = cosm f (x) ⇒ = −m cos m −1 f ( x ) sin f ( x ) f ′ ( x )
dx 

dy 
(iii) y = tanm f (x) ⇒ = m tan m −1 f ( x ) sec 2 f ( x ) f ′ ( x )
dx 

dy 
(iv) y = cotm f (x) ⇒ = −m cot m −1 f ( x ) cosec 2 f ( x ) f ′ ( x )
dx 

dy 
(v) y = secm f (x) ⇒ = m sec m −1 f ( x ) sec f ( x ) tan f ( x ) f ′ ( x )
dx 

dy 
(vi) y = cosecm f (x) ⇒ = −m cosec m −1 f ( x ) cosec f ( x ) cot f ( x ) f ′ ( x )
dx 

dy f ′ ( x) dy −f ′ ( x )
(vii) y = sin–1 f (x) ⇒ = (viii) y = cos −1 f ( x ) ⇒ =
dx 2 dx
1 −  f ( x )
2
1 −  f ( x )

dy f ′ ( x) dy −f ′ ( x )
(ix) y = tan −1 f ( x ) ⇒ (x) y = cot f ( x ) ⇒
−1
= =
dx 1 +  f ( x ) 2 dx 1 +  f ( x ) 2
   

dy f ′ ( x) dy −f ′ ( x )
(xi) y = sec −1 f ( x ) ⇒ = (xii) y = cosec −1 f ( x ) ⇒ =
dx 2 dx 2
f ( x )  f ( x ) − 1 f ( x )  f ( x ) − 1

6. Product rule:

dy
(i) y = f ( x ) g ( x ) ⇒ = f ′ ( x) g ( x) + f ( x) g′ ( x)
dx

dy
(ii) y = f ( x ) g ( x ) φ ( x ) ⇒ = f ′ ( x ) g ( x ) φ ( x ) + f ( x ) g ′ ( x ) φ ( x ) + f ( x ) g ( x ) φ′ ( x )
dx

f ( x) dy g ( x ) f ′ ( x ) − f ( x ) g ′ ( x )
7. Quotient rule: y = ⇒ =
g ( x) dx 2
 g ( x )

8. Revise following trigonometric and inverse trigonometric formulae


(i) sin(A + B) = sin A cos B + cos A sin B (ii) sin(A – B) = sin A cos B – cos A sin B

(iii) cos(A + B) = cos A cos B – sin A sin B (iv) cos(A – B) = cos A cos B + sin A sin B

tan A + tan B tan A − tan B


(v) tan ( A + B) = (vi) tan ( A − B) =
1 − tan A tan B 1 + tan A tan B

2 tan θ
(vii) sin 2θ = 2 sin θ cos θ =
1 + tan 2 θ

Continuity And Differentiability 133


E:\AMIT_WORKS\Exam_Guru\EG_Mathematics-12_(working_02-06-2022)\EG_Mathematics-12_working\Open_Files\Chap_5\Chap_5
\ 16-Aug-2022 Amit Proof-4 Reader’s Sign _______________________ Date __________

2 2 2 2 1 − tan 2 θ
(viii) cos 2θ = cos θ − sin θ = 2 cos θ − 1 = 1 − 2 sin θ =
1 + tan 2 θ

2 tan θ (x) 1 + cos θ = 2 cos 2 θ


(ix) tan 2θ =
1 − tan 2 θ 2

(xi) 1 − cos θ = 2 sin 2 θ (xii) sin 3q = 3 sin q – 4 sin3 q


2

(xiii) cos 3q = 4 cos3 q – 3 cos q 3 tan θ − tan 3 θ


(xiv) tan 3θ =
1 − 3 tan 2 θ

 x+ y  x− y
(xv) tan −1  = tan −1 x + tan −1 y (xvi) tan −1  = tan −1 x − tan −1 y
 1 − xy   1 + xy 

(xvii) 2 tan −1 x = tan −1  2x 2  (xviii) 2 tan −1 x = sin −1  2x 2 


1− x  1+ x 

 1 − x2 
(xix) 2 tan −1 x = cos −1 
 1 + x 2 
(xx) sin x = cos ( )
π
2
−x

(xxi) sin x = − cos ( )


π
2
+x (xxii) cos x = sin ( )
π
2
+x

(xxiii) cos x = sin ( )


π
2
−x (xiv)
1 − tan x
1 + tan x
π
= tan − x
4 ( )
(xxv)
1 + tan x
1 − tan x
π
= tan + x
4 ( )
9. Logarithmic Differentiation
y = [ f (x)]g(x) ⇒ log y = g(x) log [ f (x)]

1 dy  g ( x) f ′ ( x) 
⇒ = g ′ ( x ) log  f ( x ) +  
y dx  f ( x) 

dy   g ( x) f ′ ( x) 
⇒ = y  g ′ ( x ) log { f ( x )} +  
dx   f ( x) 

10. Differentiation of Parametric Function


dy dx dy  dy/d θ  f ′ (θ )
If y = f (q) and x = g(q) then = f ′ (θ ) and = g ′ (θ ) ⇒ = =
dθ dθ dx  dx/d θ  g ′ (θ )

134 Mathematics–12
E:\AMIT_WORKS\Exam_Guru\EG_Mathematics-12_(working_02-06-2022)\EG_Mathematics-12_working\Open_Files\Chap_5\Chap_5
\ 16-Aug-2022 Amit Proof-4 Reader’s Sign _______________________ Date __________

COMMON ERRORS
ERRORS CORRECTIONS

(i) Students commit mistake in applying Chain Rule


(i) For example, Differentiate: y = sin2 x2 and find dy Generally,
dx
they do it as: dy = 2 cos x 2 x 2
dx ( )
( )( )
in place of dy = 2 sin x 2 cos x 2 ( 2x ) .
dx
(ii) In continuity, children find LHL = RHL, but forget to prove (ii) Def. of continuity to be drilled.
that equal of f (a).

(iii) Derivative of implicit function like xy + y2 + x2 = 100, they (iii) Practice product rule in various forms.
don’t consider xy as a product of 2 functions.

(iv) While differentiating Inverse trigonometric functions, (iv) Simple way


usually students differentiate directly. They do not simplify  1 + cos x 
and differentiate, thus not obtaining the answer on the most y = tan −1 
 1 − cos x 
simplified manner.

For e.g., y = tan 


−1  1 + cos x 

 1 − cos x 

 2 cos 2 x / 2 
= tan −1  2
 2 sin x / 2 
−1 x
 = tan cot 2 ( )
dy 1 d  1 + cos x  π x
( )
= . = −
dx 1 + 1 + cos x dx  1 − cos x  2 2
1 − cos x
dy 1
=−
which becomes complicated. dx 2
Sometimes, students use this method but forget to
differentiate. They stop after just simplification. Do not
forget to differentiate.

(v) Logarithmic differentiation: (v) log(a + b) ≠ log a + log b. So, we should take log separately
If y = xsin x + cos xsin x students usually write as shown: y = xsin x + cos xsin x
log y = log xsin x + log cos xsin x. Let u = xsin x; v = cos xsin x
dy du dv
⇒ y = u + v, = +
dx dx dx
Calculate du , dv separately.
dx dx

(vi) Parametric differentiation: 2nd order derivative is usually


calculated incorrect. (vi) dy = t
dx
Let y = at2, x = 2at
d 2y dt
dy dy / dt 2at 2 = 1. dx
= = =t dx
dx dx / dt 2a
d 2y
• = 1 (which is wrong)
dx 2
Usually students forget to multiply with dt/dx
d 2 y d 2 y / dt 2
• = which is also wrong
dx 2 dx 2 / dt 2

Continuity And Differentiability 135


E:\AMIT_WORKS\Exam_Guru\EG_Mathematics-12_(working_02-06-2022)\EG_Mathematics-12_working\Open_Files\Chap_5\Chap_5
\ 16-Aug-2022 Amit Proof-4 Reader’s Sign _______________________ Date __________

REVISION CHART

Continuity and Discontinuity of a Function at a Point


A functrion y = f(x), x ∈ D is called continuous at a point x = m ∈ D, if there is no interruption in the graph of f(x) at x = m. If
there is interruption or break in the graph or there is a gap or hole in the graph of the function at the point x = m ∈ D, then the
function y = f(x), x ∈ D is called discontinuous at x = m ∈ D.

Differentiation of a Function at a Point

Graphical Method Limit of the Function method

Implicit Functions Inverse Trigonometric Functions

Direct
Differentiation of Parametric Differentiation of
Logarithmic Differentiation
Functions Different Type of
Functions

Higher Order Derivatives

136 Mathematics–12
E:\AMIT_WORKS\Exam_Guru\EG_Mathematics-12_(working_02-06-2022)\EG_Mathematics-12_working\Open_Files\Chap_6\Chap_6
\ 17-Aug-2022 Ved_Goswami Proof-5 Reader’s Sign _______________________ Date __________

Topics Covered
6 Application of Derivatives

6.1 Rate of Change of Bodies 6.2 Increasing and Decreasing Functions


6.3 Local Maximum and Local Minimum Values of a Function
6.4 Absolute Maximum and Absolute Minimum Values of a Function

C hapter map
APPLICATION OF DERIVATIVES
y = f(x)

Rate of Change of Bodies Increasing and Decreasing Functions Maximum and Minimum

Increasing if x1 < x2 implies


f(x1) ≤ f(x2) or
if f'(x) ≥ 0 for all x1, x2 in (a, b)

Decreasing if x1 < x2 implies


f(x1) ≥ f(x2) or
if f'(x) ≤ 0 for all x1, x2 in (a, b)

First Derivative Test Second Derivative Test

Local Maximum and Local Minimum


Absolute Maximum and Word Problems on
Absolute Minimum Maxima and Minima

INTRODUCTION
In this chapter we shall learn how the derivatives can be used (i) to determine the rate of change of quantities; (ii) to find the interval
on which a function is increasing or decreasing. (iii) to find turning points on the graph of a function which in turn will help us to
locate at which largest or smallest value (locally) of a function occurs.

Topic 1. Rate of Change of Bodies


(i) Whenever one quantity y varies with respect to (ii) If two variables x and y are varying with respect to another
another quantity x, satisfying some rule, y = f (x), variable t, i.e., if x = f(t) and y = g(t), then we can work out
dy dy
then or f ′(x) represents the rate of change of y with the formula:
dx dx
dy dy
with respect to x and or f ′(x 0 ) represents dy dt
dx x = x0 = dx
dx
the rate of change of y with respect to x at x = x 0. dt

137
E:\AMIT_WORKS\Exam_Guru\EG_Mathematics-12_(working_02-06-2022)\EG_Mathematics-12_working\Open_Files\Chap_6\Chap_6
\ 17-Aug-2022 Ved_Goswami Proof-5 Reader’s Sign _______________________ Date __________

Thus, the rate of change of y with respect to x can be


calculated using rate of change of y and that of x both with Now putting dA = 12 cm 2/sec and dP = 4 cm 2/sec in the
dt dt
respect to t. dA
x
Under this heading we shall discuss the rate of change of equation: ddtP = we get
2
perimeters and areas of square and rectangle with respect to dt
another quantity. 12 x
= ⇒ x = 6 cm
Example 1. The side of a variable square is increasing at the rate 4 2
of 2 cm/sec, find at what rate the perimeter and the area of the Hence, length of the side at that time is 6 cm.
dP
square are increasing with respect to time when the length of the On putting the value of in the equation dP = 4 dx ,
side of the square is 25 cm. dt dt dt
we get
Solution. Let the length of the side of the square at any time
dx dx
be = x. Then we are given: 4= 4 ⇒ = 1 cm/sec
dt dt
dx \ The side of the variable square is increasing at that time at
= 2 cm/sec and x = 25 cm
dt the rate of 1 cm/sec.
Perimeter of the square: P = 4x
The rate of change of perimeter of the square with respect to Questions Based on Variable Rectangle
dP Perimeter of the rectangle P = 2(l + b)
t, will be: , which can be calculated as follows: Area of the rectangle = A = lb
dt
dP dx
P = 4x ⇒ =4 2
Diagonal of the rectangle = D = l + b
2
dt dt
dP Example 3. The length of a variable rectangle is decreasing
⇒ = 4 × 2 cm/sec = 8 cm/sec  dx = 2 cm/sec 
dt  dt  at the rate of 1 cm/sec and breadth is increasing at the rate of
Now let A be the area of the square at any time t, then 2 cm/sec. Find at what rate the perimeter and area of the rectangle
A = x2 cm2. are changing when length and breadth of the rectangle are 30 cm
and 25 cm respectively.
The rate of change of area A with respect to time t is: dA ,
dt Solution. If l and b be the length and breadth of the rectangle,
which can be calculated as follows:
then we are given:
A = x2 ⇒ dA = 2x dx dl
dt dt = –1 cm/sec,
dA  2 2 dt
⇒ = 2 × 25 × 2 cm /sec = 100 cm /sec
dt  x =25 cm db = 2 cm/sec, l = 30 cm and b = 25 cm

 dx = 2 cm/sec  dt
( )

 dt  dP dl db
Perimeter P = 2(l + b) ⇒ =2 +
Hence, (i) rate of change of perimeter of the square w.r.t. dt dt dt
time t when x = 25 cm is 8 cm/sec, and (ii) rate of change of area dP
⇒ = 2(–1 + 2) cm/sec = 2 cm/sec
of the square with respect to time t when x = 25 cm is 100 cm2/sec. dt
⇒ Perimeter of the rectangle is increasing at the rate of
Example 2. At some particular time the area and perimeter of
2 cm/sec

( )
a variable square are increasing at the rate of 12 cm2/sec and 4
dA db dl
cm/sec respectively. Find what is the length of the side of the Area: A = lb ⇒ = l +b
dt dt dt
square at that time? Also find at what rate is the side of the square dA 2
⇒ = [30 × 2 + 25 (–1)] cm /sec
increasing at that time? dt
Solution. Let x is the length of the side of the variable square at = (60 – 25) cm2/sec = 35 cm2/sec
any time. P and A be the perimeter and area of the variable square ⇒Area of the variable rectangle is increasing at the rate of
at that time. Now we are given: 35 cm2/sec.
dP dA Questions Based on Variable Circle
= 4 cm/sec, = 12 cm 2/sec
dt dt Example 4. Find the radius and rate of change of radius of a
dx
and we want to find out: x and at that time variable circle, when its area and circumference are increasing
dt
P = 4x and A = x2 at the rate of 20 cm2/sec and 5 cm/sec respectively.
dP Solution. Let r be the radius of the variable circle at some
⇒ = 4 dx and dA = 2x dx particular time, then we are given:
dt dt dt dt
dC dA
dA dx dA = 5 cm/sec and = 20 cm 2/sec and we want to find
2x x dt dt
dt dt dt
⇒ dP = 4 dx ⇒ dP = dr
2 out: r and at that time.
dt dt dt dt

138 Mathematics–12
E:\AMIT_WORKS\Exam_Guru\EG_Mathematics-12_(working_02-06-2022)\EG_Mathematics-12_working\Open_Files\Chap_6\Chap_6
\ 17-Aug-2022 Ved_Goswami Proof-5 Reader’s Sign _______________________ Date __________

Now, circumference of the variable circle dA 2πr dr


C = 2pr ⇒ dC = 2π dr ⇒ dt = dt = r ⇒
20
cm = r ⇒ r = 4 cm
dt dt dC 2π dr 5
dt dt
Area of the variable circle:
dC dr dr
Also = 2π ⇒ 5 cm/sec = 2π
A = pr2 ⇒ dA = 2πr dr dt dt dt
dt dt
dr 5
cm/sec
⇒ =
dt 2π

EXERCISE 6.1
I. Multiple Choice Questions (MCQs) seconds. Find out at what rate the perimeter and area of
Choose the correct answer from the given options. the rectangle are changing with respect to time, when 10
seconds has elapsed.
1. The rate of change of the area of a circle with respect to
3. The length x cm of a rectangle is decreasing at the rate of
its radius r at r = 6 cm is
5 cm/min and width y cm is increasing at the rate of
(a) 10p cm (b) 12p cm
4 cm/min. Find the rate of change of perimeter and area of
(c) 8p cm (d) 11p cm the rectangle, when x = 8 cm and y = 6 cm.
2. The total revenue in rupees received from the sale of x [A.I. 2009]
units of a product is given by R(x) = 3x2 + 36x + 5. The 4. A stone is dropped into a quiet lake and waves move at a
marginal revenue, when x = 15 is fixed speed of 8 cm/sec. At the instant when the radius of
(a) 116 (b) 96 (c) 90 (d) 126 the circular wave is 15 cm, how fast is the enclosed area
3. A point on the parabola y2 = 18x at which the ordinate increasing?
increases at twice the rate of the abscissa is : III. Long Answer Type Questions
(a) (2, 4) (b) (2, – 4) 1. The length of a variable rectangle is always two times of
 9 9 9 9 its breadth. Find out the rate with which the perimeter, area
(c)  − ,  (d)  , 
 8 2 8 2 and diagonal of the rectangle are changing, (i) with respect
4. The sides of an equilateral triangle are increasing at the to its breadth, when breadth is 10 cm. (ii) with respect to
rate of 2 cm/sec. The rate at which the area increases, when its length when length is 20 cm, (iii) with respect to its
the side is 10 cm is diagonal, when diagonal is 25 cm.
(a) 3 cm 2 / sec (b) 10 cm2/sec 2. The area of an expanding rectangle is increasing at the
10 rate of 48 cm2/sec. The length of the rectangle is always
(c) 10 3 cm 2 / sec (d) cm 2 / sec
3 equal to the square of the breadth. At what rate, the length
II. Short Answer Type Questions-I of the rectangle is increasing at the instant when breadth
1. The side of a square is increasing at the rate of 2 cm/sec, is 4.5 cm?
find out at what rate the perimeter, area and diagonal of 3. The two sides of an isosceles triangle with fixed base are
the square are increasing with respect to time when length
decreasing at the rate of 3 cm/sec. How fast is the area of
of the square is 25 cm? [Delhi 2014]
the triangle decreasing when two equal sides are equal to
2. The length and breadth of a variable rectangle are : l =
(4t + 5) cm, b = (2t + 1) cm respectively. Here t is time in the base? [A.I. 2014 (C), Delhi 2015]

Answers 6.1
I. 1. (b) Let A be the area of the circle at any time t.
 dR 
dA
∴   = 6(15) + 36 = 126
∴ A = pr2 ⇒ = π × 2r = 2πr cm  dx  x = 15
dr
Hence, (d) is the correct answer.
 dA 

∴   = 2π × 6 = 12π cm dy dx
 dr r = 6 cm 3. (b) y2 = 18x ⇒ 2y = 18
dt dt
Hence, (b) is the correct answer.
 dx  dx  dy dx 
2. (d) Total revenue
⇒ 2 y  2  = 18  dx = 2 dt (Given ) 
 dt  dt  
R(x) = 3x2 + 36x + 5
dR d dx

∴ MR = = ( 3x 2 + 36 x + 5 ) = 6 x + 36
⇒ (4 y − 18) = 0 ⇒ 4y – 18 = 0
dx dx dt

Application of Derivatives 139


E:\AMIT_WORKS\Exam_Guru\EG_Mathematics-12_(working_02-06-2022)\EG_Mathematics-12_working\Open_Files\Chap_6\Chap_6
\ 17-Aug-2022 Ved_Goswami Proof-5 Reader’s Sign _______________________ Date __________

18 9 Hint : Let b = x, l = 2x find D = 5x , P and A




⇒ y= =
4 2
 
Putting y =
9
in y 2 = 18x , we get (ii) dP = 3, dA = 20 cm; dD =  5 
dx dx dx  2 
2
2
9 81 9 x x2
  = 18x ⇒ x = 4 × 18 = 8 Hint: Let l = x, b = , P = 3x, A =
2 2
2
 
9 9
Thus, the required point is  ,  . and D = 5 x
8 2 2
Hence, (d) is the correct answer.
(iii) dP = 6 ; dA = 20 cm; dD = 1
4. (c) Let the side of the equilateral triangle be x cm. dx 5 dx dx
3 3 2
Area = (side) 2 = x Hint : Let D = x, then b = x and l = 2x

4 4 5 5
dA 3 dx dl = 64 cm/sec

\ = ⋅2x⋅ 2.
dt 4 dt dt 9
dx 3
Now, = 2 cm /sec and x = 10 cm Hint: Let length b 2 = l ⇒ b = l , A = l 2 .

dt
dA 3 dA

\ = ⋅ 2(10) ⋅ 2 cm 2 /sec =10 3cm 2 /sec Now find .
dt 4 dl
Hence, (c) is the correct answer. 2
3. dA = 3l cm 2/sec Hint: A = l x2 − l
dP = 8 cm/ sec; dA = 100 cm 2/ sec; dt 2 4
II. 1.
dt dt
dD = 2 2 cm/sec.
dt

2. dP = 12 cm/sec; dA = 174 cm 2/sec


dt dt

3. dP = − 2 cm/min; dA = 2 cm 2/min
dt dt

4. dA = 204 π cm 2/ sec.
dt
dP = 6; dA = 40 cm and dD = 5
III. 1. (i)
dx dx dx

(A) RELATED RATES and S = 6x2 ⇒ dS = 12x dx


dt dt
Cube dV
If x is length of an edge of a cube, then: 3x 2 dx dV
dt dt x 50 x
Now, dS = dx ⇒ ddtS = ⇒ cm =
12x 4 10 4
S = 6x2­, V = x3, D = 3x dt dt dt
Example 1. If x is the length of the edge ⇒ x = 20 cm
of a variable cube, find x and dx when On putting x = 20 cm and dS = 10 cm2/sec in dS = 12x dx ,
dt dt dt dt
we get
the volume and surface area of the cube are increasing
dx dx 1
at the rate of 50 cm 3 /sec and 10 cm 2 /sec respectively. 10 cm2/sec = 12 × 20 cm × ⇒ = cm/sec
dt dt 24
dV \ x = 20 cm and dx = 1 cm/sec
Solution. In a variable cube, we are given = 50 cm3/sec,
dt dt 24
dS = 10 cm2/sec.
dt Cuboid
If x is the length of the cube at some time then we have to find If l, b and h are the length, breadth and height of
dx a cuboid then:
out: and x. S = 2(lb + bh + hl), V = lbh,
dt
V = x3 ⇒ dV = 3x 2 dx D= l 2 + b2 + h2
dt dt

140 Mathematics–12
E:\AMIT_WORKS\Exam_Guru\EG_Mathematics-12_(working_02-06-2022)\EG_Mathematics-12_working\Open_Files\Chap_6\Chap_6
\ 17-Aug-2022 Ved_Goswami Proof-5 Reader’s Sign _______________________ Date __________

Example 2. If height, breadth and length of a variable cuboid


are x, 2x and 3x respectively, find the rate with which its surface Solution. In a cylinder, it is given: dr = 2 cm/sec , dh = 3 cm/sec ,
h = 20 cm, r = 5 cm. dt dt
area, volume and diagonal are increasing with respect to x, when
x = 10 cm.
To find: dS and dV
Solution. Given that: h = x, b = 2x and l = 3x. dt dt
\ Surface area of the variable cuboid: S = (2prh + 2pr2)


S = 2(lb + bh + hl) = 2[3x × 2x + 2x × x + x × 3x]
S = 2[6x2 + 2x2 + 3x2] = 22x2
\
dS
dt

=  2π r
 (dh
dt
+h
dr
dt ) dr 
+ 2π ( 2r ) 
dt 
⇒ S = 22 x2 dS
⇒ = 2p[(5 × 3) + (20 × 2) + 2(5 × 2)] cm2/sec
dS dt
⇒ = 44x
dx = 2p[15 + 40 + 20] cm2/sec = 150p cm2/sec
⇒ dS  = 44 × 10 cm = 440 cm V = pr­2h
dx  x =10 cm
dV = π  r 2 dh + h 2r dr 
Volume of the variable cuboid: V = lbh ⇒
dt  dt ( ) dt 

⇒ V = (3x)(2x)(x) = 6x3 ⇒ V = 6x3
dV
dV ⇒ = p[(5)2 × 3 + 20 × (2 × 5) × 2] cm3/sec
⇒ = 18x2 dt
dx
dV

dV 
= 18(10)2 cm2 = 1800 cm2 ⇒ = p[75 + 400]cm3/sec = 475p cm3/sec.
dx  x =10 cm dt

Diagonal of the variable cube: D = l 2 + b 2 + h 2 Cone


Volume of a variable cone increases or decreases in such
⇒ D= 9x 2 + 4x 2 + x 2 cm = 14 x a way that ratio between its radius, height and slant height
is always same. In questions based on variable cones, a
dD
⇒ = 14 relation between radius, height and slant height is given.
dx
Such questions are solved either by changing radius into height
Sphere or height into radius. For example, if a variable cone is such that
4 3 its radius is always 3 times of its height, then r = 3h.
If r is the radius of a sphere then: S = 4pr2 and V = πr .

()
3
1 2 1 2 r 1 3
Example 3. The radius of a spherical balloon is increasing at the V = 3 πr h = 3 πr 3 = 9 πr
rate of 2 cm/sec when it is being inflated. Find rates with which
its surface area and volume are increasing at the time when its
radius is 25 cm. or V=
1 2
3
1
3 ( )
πr h = π 9h 2 h = 3πh3
dr dV 1 2 dr
Solution. It is given that: = 2 cm/sec, r = 25 cm dV dh
dt Then = πr = 9πh 2
or
dt 3 dt dt dt
dS dV
To find
dt
and
dt Also r = 3h ⇒ dr = 3 dh
dt dt
dS dr
S = 4pr2 ⇒ = 8πr Example 5. Falling sand forms a conical heap on the ground in
dt dt
such a way that its radius is increasing at the rate of 1 cm/sec. If
dS
⇒ = 8p × 25 × 2 cm2/sec = 400 p cm2/sec the radius of cone so formed is always 5 times of its height, find
dt
the rate with which sand is falling on the ground when the radius
4 3 of the cone is 20 cm.
V = 3 πr
dr
dV = 4πr 2 dr = 4p (25)2 × 2 cm3/sec Solution. In a variable cone: r = 5h, = 1 cm/sec, r = 20 cm.
⇒ dt
dt dt To find: d V
= 5000 p cm3/sec dt

Cylinder
1
Volume of a cone: V = πr 2h ⇒ V = πr 2
3
1
3
r 1
= πr 3
5 15 ()
If r and h are the radius and height of a cylinder, dV 1 2 dr 1
= π × ( 20) × 1 cm3/sec
2
⇒ = πr
then: dt 5 dt 5
S = 2prh + 2pr2 and V = pr2h dV
⇒ = 80 p cm3/sec
Example 4. If the radius and height of a variable dt
right circular cylinder are increasing at the rate of Example 6. An inverted conical vessel whose height is 10 m and
2 cm/sec and 3 cm/sec respectively, find at what radius 5 m is filled with water at the rate of 8 cubic metre/minute.
rates its total surface area and volume increasing Find the rate at which the level of the water in the vessel rising
when its height is 20 cm and radius 5 cm. when the depth of the water is 4 m.

Application of Derivatives 141


E:\AMIT_WORKS\Exam_Guru\EG_Mathematics-12_(working_02-06-2022)\EG_Mathematics-12_working\Open_Files\Chap_6\Chap_6
\ 22-Aug-2022 Ved_Goswami Proof-5 Reader’s Sign _______________________ Date __________

h 10 Volume of water in the tank


Solution. In a cone, it is given: = =2
r 5
V = 1 πr 2h
3
⇒ h = 2r and dV = 8 m3/min, h = 4 m
dt
()
2
1 l  3l  1 l 2 3l
()
2 ⇒ V= π = π ×
To find
dh 1 1 h
, V = πr 2h = π
1
h = πh3 3 2  2  3 4 2
dt 3 3 2 12
1
dV 1 dh ⇒ V= πl 3 ⇒ dV = 3 πl 2 dl
⇒ = πh 2 8 3 dt 8 3 dt
dt 4 dt
1 2 2 dh dh 2 3 2 dl 48π dl
⇒ 8 m3/min = π ( 4) m ⇒ = m/min. ⇒ 1= π × ( 4) × =
4 dt dt π 8 3 dt 8 3 dt
\ The level of the water in the conical vessel is increasing at
6π dl
the rate of
2
m/min. ⇒ 1= ⇒ dl = 3 cm/sec
π 3 dt dt 6π
Example 7. Water is dripping out at the rate of 1 cm3/sec through ⇒ Slant height of water is decreasing at the rate of
a tiny hole at the vertex of an inverted conical vessel whose axis 3 cm/sec.
is vertical. When the slant height of the water in the vessel is 6π
4 cm, find the rate of decrease of (i) slant height of the water and
(ii) Let A be the area of water surface, i.e.,
(ii) the area of the water surface, given that the vertical angle of
2
the vertex is 60°. [V. Imp.]
A = πr 2 = πl   r = l 
Solution. (i) Semi vertical angle = q = 30° 4  2 
dV = 1 cm3/sec, l = 4 cm
dA 2π dl 2π 3
dt ⇒ = l = ×4×
dt 4 dt 4 6π
r
In DABC, = sin 30°
l  dl 3
l  l = 4 cm, dt = 6π 
⇒ r=  
2
3
h = cm 2/ sec
= cos 30° ⇒ h = 3l 3
l 2

EXERCISE 6 (A)
I. Very Short Answer Type Questions 6. The volume V cm 3 of water in a leaking vessel is
1. If x is the side of variable cube, find at what rates its surface given in terms of time in seconds by the formula
area, volume and diagonal are increasing with respect to V = 100 – 4t + t2. Find the rate at which the water is running
x, when x = 20 cm. out of the vessel when t = 10 sec.
II. Short Answer Type Questions-I
2. If x is the length of the side of a variable cube and
dx = 1 cm/sec, find the rates with which its surface 1. The height of a closed variable right circular cylinder is
dt always 5 times of its radius. Find the radius and rate of
area, volume and diagonal are changing w.r.t. time, when change of radius of the cylinder when volume and total
x = 10 cm. surface area of the cylinder are changing at the rate of
400 π cm3/sec and 50 π cm2/sec respectively.
3. Find the rates with which the volume and surface area of a 2. If length, breadth and height of a cuboid are changing at
spherical balloon are increasing with respect to the radius, the rate of –1 cm/sec, 2 cm/sec and 1 cm/sec respectively,
find the rates of change of surface area and volume of the
when radius of the balloon is 20 cm.
cuboid when its length, breadth and height are 20 cm, 15
4. The radius of a spherical balloon is increasing at the rate cm and 10 cm respectively.
of 2 cm/sec when it is being inflated. Find rates with this 3. Find the radius of the variable spherical balloon at the time
when its volume and surface area are increasing at the rate
its surface area and volume are increasing at the time when
of 200 p cm3/sec and 40 p cm2/sec. Also find the rate with
its radius is 25 cm. which the radius of the balloon is increasing at that time.
5. The volume of a spherical balloon is increasing at the rate 4. An oil tank is being filled. The oil volume V in litres after
of 20π cm3/sec. Find the rate of change of the surface area t minutes is given by: V = 3t + t2­. How fast is the volume
at the instant when the radius is 10 cm. increasing when there is 10 litres of oil in the tank?

142 Mathematics–12
E:\AMIT_WORKS\Exam_Guru\EG_Mathematics-12_(working_02-06-2022)\EG_Mathematics-12_working\Open_Files\Chap_6\Chap_6
\ 17-Aug-2022 Ved_Goswami Proof-5 Reader’s Sign _______________________ Date __________

Cone the height of the cone is always one-sixth of the radius of


III. Short Answer Type Questions-II the base. How fast is the height of the sand cone increasing
1. Water is leaking from a conical funnel at the rate of 5 cm3/ when the height of the cone is 4 cm? [Delhi 2011]
sec. If the radius of the base of the funnel is 5 cm and height 2. Water tank has the shape of an inverted circular cone with
10 cm, find the rate at which the water level is dropping its axis vertical and vertex is lower most. Its semi-vertical
when it is 2.5 cm from the top. angle is tan–1 (0.5). Water is poured into it at a constant rate
IV. Long Answer Type Questions of 5 m3/hr. Find the rate at which level of the water rising
1. Sand is falling from a pipe at the rate of 12 cm3/sec. The at the instant when the depth of the water in the tank is 4
falling sand forms a cone on the ground in such a way that metres.

Answers 6 (A)
dS dV dD 5. dS = 4 p cm2/sec
I. 1. = 240 cm ; = 1200 cm 2 ; = 3
dx dx dx dt
Hint: It is a cube.  dV 
S = 6x2 Hint: It is a sphere. Use:  dt  = r

 dS  2
dS  dt 
⇒ = 12x cm = 12 × 20 cm = 240 cm.
dx
V = x3 Put dV = 20π cm3/ sec and r = 10 cm .
dV dt
⇒ = 3x2 cm2 = 3(20)2 cm2
dx 6. d V = 16 cm3/sec.
dD dt
D = 3x ⇒ = 3
dx Hint: V = 100 – 4t + t2
dS dV = (–4 + 2t) cm3/sec
2. = 120 cm2/sec; dV = 300 cm3/sec ⇒
dt dt dt
dD = 3 cm/sec Now put t = 10 sec.
dt
II. 1. r = 12.8 cm and dr = 125 cm/sec
Hint: It is a cube dt 768
S = 6x2 Hint: It is a cylinder.

dS
dt
= 12x
dt ( )
dx cm 2/sec
h = 5r, dV = 400 p cm3/sec
dt
V = x3 dS = 50 p cm2/sec, r = ?, dh = ?
( )
dt
dV dx dt

⇒ = 3x 2 cm3/sec
dt dt
V = pr2h = pr2(5r) = 5pr3
dD = 3 cm/sec dV = 15pr2 dh
D = 3x ⇒

dt dt dt
Now put x = 10 cm and dx S = 2prh + 2pr2 ⇒ S = 2pr(5r) + 2pr2
= 1 cm/sec
dt
⇒ S = 10pr2 + 2pr2 = 12pr2
dV
3. dS = 160 p cm, = 1600 p cm2. 15πr 2 dr
dr dr

⇒ dS = 24pr dh ⇒ dV/dt = dt = 15 r
Hint: It is a sphere. dt dt dS/dt 24πr dr 24
S = 4pr2 ⇒ dS = 8pr cm 400π 15 r
dt
dr
⇒ = ⇒ 15r = 24 × 8
4 πr 3 dV 50π 24
V= ⇒ = 4pr2 cm2
3 dr 64 cm dh

⇒ r= . Now find dt .
Now put r = 20 cm. 5
4. dS = 400 p cm2/sec; dV = 5000 p cm3/sec 2. dS = 140 cm2/sec; dV = 550 cm3/sec
dt dt dt dt
Hint: It is a sphere. Hint: It is a cuboid.
S = 4pr2 S = 2[lb + bh + hl]
⇒ dS = 8πr dr cm 2/ sec
dt dt

dt  dt ( ) ( ) ( )
dS = 2 l db + b dl + b dh
dt dt

( ) ( ) ( )
4 πr 3 dr
V=  ⇒ dV = 4πr 2 cm3/sec + h db + h dl + l dh  cm 2/sec

3 dt dt dt dt dt 
dh
Now put r = 25 cm and dt = 2 cm/sec V = lbh

Application of Derivatives 143


E:\AMIT_WORKS\Exam_Guru\EG_Mathematics-12_(working_02-06-2022)\EG_Mathematics-12_working\Open_Files\Chap_6\Chap_6
\ 17-Aug-2022 Ved_Goswami Proof-5 Reader’s Sign _______________________ Date __________

dV =  dl (bh ) + db (lh ) + dh (lb ) cm3/ sec Also h = (10 – 2.5) = 7.5 cm



⇒  dt  1 2 1 2
dt dt dt V = πr h = πr ( 2r )
dl 3 3
Now put l = 20 cm, b = 15 cm, h = 10 cm, = –1 cm/sec

db
dt
= 2 cm/sec and
dh
dt
= 1 cm/sec
dt

⇒ V=
2 πr 3
3

dV
dt
= 2πr 2
dr
dt ( )
dV = 5 cm3/sec
3. r = 10 cm and dr = 1 cm/sec Now put , r = 5 cm and get dh
dt 2 dt dt
Hint: It is a sphere.
IV. 1. dh = 1 cm/sec
V = 4 πr 3 ⇒ dV = 4πr 2 dr cm3/sec dt 48π
3 dt dt
Hint: It is a cone.
dS = 8πr dr cm 2/ sec 1
S = 4pr2 ⇒ V = πr 2h, r = 6h
dt dt 3
 dV   4πr 2 dr 
 dt  =  dt  = r
⇒ V=
1
3 ( )
π 36h 2 h = 12πh3
 dS  8 πr dr  2
 dt   dt  dV 2 dh

⇒ = 36πh
dt dt
Now put dV = 200 π cm3/sec
dt dV
Now put = 12 cm3/sec, h = 4 cm and get dh
dS dt dt
= 40 p cm2/sec
dt
2. dh 5 35
= m/h = m/h
4. dV = 7 litres/min. dt 4π 88
dt
Hint: It is a cone.
Hint: V = 10, V = t2 + 3t
h r
⇒ t2 + 3t – 10 = 0 ⇒ t = 2 min. = 0.5 ⇒ h=
r 2
dV 

dt  t =2 min
= 2t + 3 = 7 litres/min.

3 3 ( )
V = 1 πr 2h = 1 π 4h 2 h
dh = 16 cm/sec
III. 1. 4 πh3 dV = 4πh 2 dh
dt 45π
⇒ V= ⇒
3 dt dt
Hint: It is a cone.
h dV
= 10 ⇒ h = 2r Now put = 5 m3/hr, h = 4 m and get dh
r 5 dt dt

(B) SHADOW FORMATION, SLIDING OF LADDER AND MOTION OF A


PARTICLE ALONG A CURVE
Example 1. A man of height 2 m is running away from a lamp 5 x+ y
post of height 5 m at the rate of 1 m/sec. Find at what rate the ⇒ = ⇒ 5x = 2x + 2y
2 x
dy
shadow of the man is lengthening. Also ⇒ 3x = 2y ⇒ 3 dx = 2
find with what rate tip of the shadow is dt dt
moving away with respect to the lamp-post. dx = 2 ×1 dx 2
⇒ 3 ⇒ = m/sec
Solution. Let AB be the lamp-post of dt dt 3
height 5 m, CD be the running man of ⇒ The length of the shadow of the man is increasing at the
height 2m. DE = x be the length of the 2
rate of m/sec .
shadow at any time, y be the distance of 3
the man from the pole. Now the rate with which tip of the shadow of the man is
dx moving away from the pole
\ = rate of lengthening the shadow of the man
dt

dy
dt
= rate with which man is running away from the
=
dx dy
+
dt dt
2
3 ( ) 5
= + 1 m/sec = m/sec
3
pole = 1 m/sec
Example 2. A ladder 13 m long is leaning against a vertical wall.
AB BE
DABE ~ DCDE ⇒ = The foot of the ladder is dragged away from the wall at a rate of
CD DE
 [Corresponding sides of similar Ds are proportional] 3 m/sec. How fast the angle q between the ladder and the ground
2

144 Mathematics–12
E:\AMIT_WORKS\Exam_Guru\EG_Mathematics-12_(working_02-06-2022)\EG_Mathematics-12_working\Open_Files\Chap_6\Chap_6
\ 17-Aug-2022 Ved_Goswami Proof-5 Reader’s Sign _______________________ Date __________

is changing when the foot of the ladder is 12 m away from the dθ −507 −3
⇒ = = = − 0.3 radian/sec.
foot of the wall. dt 1690 10
3
Solution. Let AB be the ladder, x = 12 m Example 3. A particle moves along the curve: y = 2x + 1 . Find
be the distance of the foot of the ladder 3
the point on the curve at which y-coordinate is changing twice
and foot of the wall and y be the height
as fast as x-coordinate.
of the upper end of the ladder. Here
Solution. Let the required point be P(x, y) on the curve
dx 3
= m/sec is the speed with which 2x3
dt 2 y= +1 .
lower end of the ladder is being dragged. 3
dy Given condition: rate of change of y-coordinate = 2(rate of
is the speed with which upper end of change of x-coordinate).
dt
the ladder is coming down. dy dx
⇒ = 2
dt dt
Since ABC is right triangle
y 2x3 dy dx
\ x + y = 169 and tan q = = 5
2 2 Now y=
3
+1 ⇒
dt
= 2x 2
dt
x 12
dy dy
⇒ 2x dx + 2y
dy
= 0 ⇒
dy −18
= m/sec On putting = 2x 2 dx in dx
= 2 , we get
dt dt dt 5 dt dt dt dt
dy
y x − y dx 2x 2
dx
= 2
dx
⇒ x2 = 1 ⇒ x = ±1

2 dt dt dt dt
Now tan q = ⇒ sec θ =
x dt x2 3
On putting x = ±1, in y = 2x + 1 ,
 x dy − y dx  3
dy
dθ  dt dt  x − y dx
⇒ = = 2 dt dt y = 5
, when x = 1
dt (
x 2 sec 2 θ ) (
x 1 + tan 2 θ ) 
we get 
3
1
( ) ()
y = , when x = −1
12 −18 − 5 3 −216 − 15  3
dθ 5 2 = 5
( ) ( )
2
( )
⇒ = 169 5 1
dt 25 ⇒ The required points are: 1, and −1, .
( ) 144
144 1 + 144 ×
144 3 3

EXERCISE 6 (B)
I. Long Answer Type Questions road. How fast is his shadow increasing if he is walking
1. A man of height 2 metres walks at a uniform speed of 5 away from the light pole at a rate of c metres/sec?
km/hour away from a lamp post, which is 6 metres high. 6. A particle moves along a curve: y = 5x3 + 4x + 5. Find the
Find the rate at which the length of his shadow increasing. point on the curve at which the y-coordinate is changing
2. A particle moves along a curve : 6y = x3 + 2. Find the points 19 times as fast as x-coordinate.
on the curve at which the y-coordinate is changing 8 times 7. The ends of a rod AB which is 5 metres long moves along
as fast as x-coordinates. [V. Imp.] the two axes OX and OY which are at right angles. If A
1
3. At what points on the ellipse 16x2 + 9y2 = 400, does the moves at a constant speed of m/sec. What is the speed
2
ordinate decreasing at the same rate at which abscissa is of B which is 4 metres from O?
increasing? 8. A particle is moving according to the equation S = 2t3 – 9t2
2
4. Find the point on the curve y = 8x for which the abscissa + 12t + 1, where S is the displacement in cm, and t is the
and ordinate changes at the same rate. time in seconds find (i) velocity and acceleration of the
5. A point source of light along straight road is at a height of particle at t = 2 sec. (ii) The time when the particle stops
‘a’ metres. A boy ‘b’ metres in height is walking along the momentarily and (iii) the distance between the two points.

Answers 6 (B)
1. Rate of increase of shadow = dx = 2.5 km/hour 
2. (4, 11) and  − 4,
− 31

dt  3 
Hint: See the solution of Example 1.
Hint: See the solution of Example 3.

Application of Derivatives 145


E:\AMIT_WORKS\Exam_Guru\EG_Mathematics-12_(working_02-06-2022)\EG_Mathematics-12_working\Open_Files\Chap_6\Chap_6
\ 17-Aug-2022 Ved_Goswami Proof-5 Reader’s Sign _______________________ Date __________

3. ( )
3, 16 and  − 3,
3 
− 16 
3 
 \ 19

dx
dt ( )
= 15x 2 + 4
dx
dt ( )
Hint: It is an ellipse. ⇒ 15x2 + 4 = 19
16x2 + 9y2 = 400 ⇒ 15x2 – 15 = 0
dy
⇒ 32x dx + 18y = 0 ⇒ x2 = 1
dt dt
⇒ x = ±1. Now find y.
dy dx
⇒ 18y = −32x dy − 3
dt dt 7. = m/sec
dt 8

⇒ 18y = –32 x  dy = dx 
 dt dt  dx 1 m/sec
Hint:
=
⇒ y = −16 x dt 2
9
x2 + y2 = 25
Put it in 16x2 + 9y2 = 400
16x 2 + 9 × 256 x 2 = 400

dy
2x dx + 2y = 0
81 dt dt
⇒ x = ±3 and y = ± 16

dy
x dx + y = 0
3 dt dt


( )
3, 16 and  − 3,
3 
− 16 
3 
 Put x = 3 m,
dx
dt
1
= m/sec and y = 4 m in
2
4. (2, 4) dy dy
x dx + y = 0 to get .
Hint:
y = 8x 2 dt dt dt

⇒ 2y

dy
= 8
dx 8. (i) dS = 0 cm/sec;
dt dt dt
dy dx dv = d 2s
Since = \ 2y = 8 ⇒ y=4 acc. = = 6 cm/sec2
dt dt dt dt 2
Now find x. Hint: S = 2t3 – 9t2 +12t + 1
dS = 6t 2 − 18t + 12
dx =  bc  ⇒Velocity =
5. m/sec dt
dt  a − b 
dS 
dy = dt  = 24 – 36 + 12 = 0 cm/sec
 t =2
Hint:
= c m/sec
dt (ii) t = 1 sec, 2 sec
Now see the solution of Example 1. Hint: 6(t2 –3t + 2) = 0
6. (1, 14) and (– 1, – 4) ⇒ t = 1 sec, 2 sec
Hint: y = 5x3 + 4x + 5 ⇒ Particle stops momentarily at t = 1 sec and t = 2 sec.

dy
dt (
= 15x 2 + 4
dx
dt ) (iii) distance between two points = 1 cm.
Hint: St = 1 = (2 – 9 + 12 + 1) cm = 6 cm
 dy 
Now   = 19
dt
dx
dt ( ) St = 2 = (16 – 36 + 24 + 1) cm = 5 cm
St = 2 – St = 1 = 6 – 5 = 1 cm.

Topic 2. Increasing and Decreasing Functions


Increasing Function Finding the intervals in which a function is
(i) A function y = f (x) is said to be increasing on an interval increasing or decreasing
[a, b] if f ′(x) ≥ 0 on the interval [a, b]. Working Rule
(ii) A function y = f (x) is said to be strictly increasing on an (i) Take the function y = f (x).
interval [a, b] if f ′(x) > 0 on the interval [a, b]. dy
(ii) Work out = f ′ ( x)
dx
Decreasing Function
(iii) To find the intervals in which f (x) increases, solve
(i) A function y = f (x) is said to be decreasing on an interval f ′(x) ≥ 0 and to find the intervals in which f (x) strictly
[a, b] if f ′(x) ≤ 0 on the interval [a, b]. increases, solve f ′(x) > 0.
(ii) A function y = f (x) is said to be strictly decreasing on the (iv) To find the intervals in which f (x) decreases, solve
interval [a, b] if f ′(x)< 0 on the interval [a, b]. f ′(x) ≤ 0 and to find the intervals in which f (x) strictly
decreases solve f ′(x) < 0.
146 Mathematics–12
E:\AMIT_WORKS\Exam_Guru\EG_Mathematics-12_(working_02-06-2022)\EG_Mathematics-12_working\Open_Files\Chap_6\Chap_6
\ 17-Aug-2022 Ved_Goswami Proof-5 Reader’s Sign _______________________ Date __________

Example 1. Find the intervals in which the function: If a > b and (x – a) (x – b) < 0 ⇒ b < x < a]

{ } { ( )}
3 2
f (x) = x – 4x + 4x + 5 is 2 2
⇒ f (x) is S↓ in the interval: < x < 2 or x ∈ , 2
(i) strictly increasing or S↑ 3 3
(ii) strictly decreasing or S↓ Third Method:
f ′(x) = (3x – 2)(x – 2).
Solution. f (x) = x3 – 4x2 + 4x + 5
⇒ f ′(x) = 3x2 – 8x + 4 = (x – 2)(3x – 2) Take (3x – 2)(x – 2) = 0 ⇒ x = 2 ,2
3
To find the x-coordinates of the critical point, solve:
Plot the points x = 2 and x = 2 on a number line.
(x – 2) (3x – 2) = 0 3
⇒ x = 2, 2 Decide the sign of f ′(x) below x = 2
3 3

First Method: x = 2 , 2
3
Intervals in which f (x) is S↑ or S↓ It can be done by putting x = 0, a number just smaller

than x = 2 . f ′(0) = (–) (–) = +, so it gives + sign.



x < 3

2
or x ∈ −∞,
2
3 ( ) 3

2
 3
2
 < x < 2 or x ∈ , 2
3 ( ) On the number line put + sign before x = 2 put – sign between
2 and 2 and + sign next to 2.
3

 3
 x > 2 or x ∈( 2, ∞ )
( )
⇒ f (x) is strictly increasing in −∞, 2 ∪ ( 2, ∞ ) and strictly
3
( )
∴ f (x) is S↑ in the intervals:
2
decreasing in ,2 .
{ }
x<
2 ∪ {x > 2}
3
or { ( x ∈ − ∝,
2
3 )}∪ {x ∈ (2, ∞)}
3

The monotonicity of a function on a given values


and f (x) is S↓ in the interval:
2
3 { 2
< x < 2 or x ∈ , 2
3 } { ( )} internal of x
Example 2. For all real values of x, the function: f (x) = x3 + 3x
Decision Making Table + 5 is
Intervals x f ′(x) = (3x – 2) Sign Result (a) strictly decreasing (b) strictly increasing
(x –2) (c) increasing (d) decreasing
2
x < or x ∈ −∞,
3
2
3 ( ) 0 (–) (–) + S↑ Solution.

3
f (x) = x + 3x + 5
f ′(x) = 3x2 + 3 = 3(x2 + 1)
2
3
2
< x < 2 or x ∈ , 2
3 ( ) 1 (+) (–) – S↓ Since f ′(x) = 3(x2 + 1) > 0 for all real values of x,
\ f (x) is strictly increasing function for all values of x.
x > 2 or x ∈ (2, ∞) 3 (+) (+) + S↑ Hence, option (b) is the correct answer.
Second Method: f (x) = x – 4x + 4x + 5 3 2 Example 3. For all real values of x, the function: f (x) = 3x3 +
⇒ f ′(x) = 3x2 – 8x + 4 = (x – 2) (3x – 2) 6x2 + 4x + 8
For S↑ intervals, solve: (a) increases (b) strictly increasing
(x – 2) (3x – 2) > 0
2
2
⇒ 3 ( x − 2) x − > 0
3 ( ) (c) decreases
3
Solution. f (x) = 3x + 6x + 4x + 8 2
(d) strictly increasing

⇒ x > 2 or x <
3 ⇒ f ′(x) = 9x2 + 12x + 4 = (3x + 2)2
[If a > b and (x – a) (x – b) > 0 ⇒ x > a or x < b] Since (3x + 2)2 ≥ 0 for all real values of x,
⇒ f (x) is S↑ in the intervals
\ f (x) increases for all real values of x.

{ }
x<
2
3
∪{x > 2} or { ( x ∈ − ∞,
2
3 )} ∪ { x ∈ (2, ∞)} Hence, option (a) is the correct answer.
Example 4. For all real values of x, the function: f (x) = 3x + sin x is
For S↓ intervals, solve
(a) strictly increasing (b) strictly decreasing
(3x – 2) (x – 2) < 0 ⇒ 2<x<2 (c) increasing (d) decreasing
3

Application of Derivatives 147


E:\AMIT_WORKS\Exam_Guru\EG_Mathematics-12_(working_02-06-2022)\EG_Mathematics-12_working\Open_Files\Chap_6\Chap_6
\ 17-Aug-2022 Ved_Goswami Proof-5 Reader’s Sign _______________________ Date __________

Solution. f (x) = 3x + sin x ⇒ f ′(x) = 3 + cos x Solution. f (x) = x3 + 18x2 + 144x + 5


Since (3 + cos x) > 0 for all real values of x, ⇒ f ′(x) = 3x2 + 36x + 144
\f (x) is a strictly increasing function for all real values of x. ⇒ f ′(x) = 3[x2 + 12x + 48]
Hence, option (a) is the correct answer. ⇒ f ′(x) = 3[(x + 6)2 + 12]
Example 5. For all real values of x, the function: f (x) = x3 + 18x2
+ 144x + 5 is ⇒ f ′(x) = 3(x + 6)2 + 36 > 0 for all real values of x
(a) strictly increasing (b) decreasing \ f (x) is a strictly increasing function.
(c) increasing (d) strictly decreasing Hence, option (a) is the correct answer.

EXERCISE 6.2
I. Multiple Choice Questions (MCQs) 4. Find the intervals in which the function: f (x) = (x – 1)3
Choose the correct answer from the given options. (x – 2)2 is
1. The function given by f(x) = 3x + 17 is (i) increasing (ii) decreasing
(a) increasing on R (b) decreasing on R [A.I. 2011 (C)]
(c) strictly increasing on R (d) strictly decreasing on R 5. Show that the function: f (x) = x3 – 3x2 + 4x, x ∈ R, strictly
increasing (S↑) on R. [A.I. 2011 (C)]
2. The intervals in which the function f(x) = ex is strictly
increasing are 6. Find the intervals in which the function: f (x) = x4 – 8x3 +
22x2 – 24x + 21
(a) Only ∀ x ∈ R (b) Only ∀ x ∈ N
(i) strictly increases (S↑) (ii) strictly decreases (S↓)
(c) Only ∀ x ∈ Q (d) None of these
2 –x
[A.I. 2012 (C), Delhi 2014]
3. The function y = x e is decreasing in the interval
7. Find the intervals in which the function:
(a) (0, 2) (b) (2, ∞)
3x 4 4 x 3 36x
(c) (– ∞, 0) (d) (– ∞, 0) ∪ (2, ∞) f ( x) =
− − 3x 2 + + 11
10 5 5
II. Short Answer Type Questions-I (i) strictly increases (S↑) (ii) strictly decreases (S↓)
1. Show that the function f (x) = x3 – 6x2 + 12x + 8 increases [Delhi 2014 (C)]
(↑) for all real values of x. 8. Prove that the function : f (x) = x2 – x + 1 is neither increases
2. Find the intervals in which the function is given by: f (x) = or decreases in (– 1, 1). Hence, find the intervals in which
x2 – 4x + 6 is (a) strictly increasing, (b) strictly decreasing. f (x):
(i) strictly increases (S↑) (ii) strictly decreases (S↓)
3. Find the intervals in which the function is given by f (x) =
[Delhi 2014 (C)]
2x2 – 3x is (a) strictly increasing, (b) strictly decreasing.
9. Find the intervals in which the function:
4. Find the intervals for each of the following functions (a)
1
strictly increasing or (b) strictly decreasing. f ( x ) = x3 + 3 , x ≠ 0
x
(i) f (x) = xex (ii) f (x) = xe–x (i) strictly increases (S↑) (ii) strictly decreases (S↓)
(iii) f (x) = x2ex (iv) f (x) = x2e–x [Delhi 2014]
10. Find the intervals in which the function: f (x) = (x – 1)
III. Short Answer Type Questions-II
(x – 2)3
1. Find the intervals in which the function: f (x) = 2x3 – 8x2
(i) strictly increases (S↑) (ii) strictly decreases (S↓)
+10x + 5 is
[A.I. 2014 (C)]
(i) strictly increasing or S↑
11. Find the intervals in which the function: f (x) = (x + 2)e–x
(ii) strictly decreasing or S↓[A.I. 2011 (C)]
(i) strictly increases (S↑) (ii) strictly decreases (S↓)
2. Find the intervals in which the function: f (x) = (x – 1)
[Delhi 2016 (C)]
(x – 2)2 is
x 2
(i) increasing or ↑ (ii) decreasing or ↓ 12. Find the intervals in which the function: f ( x ) = + , x ≠ 0
2 x
3. Find the intervals in which the function f (x) = 2x3 + 9x2 + (i) strictly increases (S↑) (ii) strictly decreases (S↓)
12x + 20 is 13. Find the intervals in which the function:
(i) strictly increasing or S↑
(ii) strictly decreasing or S↓
π
4( )
on [ 0, π ]
f ( x ) = sin x +

[Delhi 2011 (C), A.I. 2011 (C)] (i) strictly increases (S↑) (ii) strictly decreases (S↓)

148 Mathematics–12
E:\AMIT_WORKS\Exam_Guru\EG_Mathematics-12_(working_02-06-2022)\EG_Mathematics-12_working\Open_Files\Chap_6\Chap_6
\ 17-Aug-2022 Ved_Goswami Proof-5 Reader’s Sign _______________________ Date __________

14. Find the intervals in which the function: f (x) = sin x 2. Find the intervals in which the function: f(x) = 20 – 9x +
– cos x on [0, 2p] 6x2 – x3.
(i) strictly increases (S↑) (ii) strictly decreases (S↓) (i) strictly increasing or S↑
15. Find the intervals in which the function: f (x) = sin x (ii) strictly decreasing or S↓[A.I. 2010]
+ cos x on [0, 2p] 3. Show that the function f given by: f (x) = x3 – 3x2 + 4x,
(i) strictly increases (S↑) (ii) strictly decreases (S↓) x ∈ R is strictly increasing on R.
16. Find the intervals in which the function: 4. Prove that the function given by: f (x) = x3 – 3x2 + 3x – 100
π is increasing in R.
f ( x ) = sin 4 x + cos 4 x on 0, 
 2  5. Show that the function: f (x) = x3 – 6x2 + 12x –18 is an
(i) strictly increases (S↑) (ii) strictly decreases (S↓) increasing function on R.
 4 sin x  6. Show that each of the following functions is an increasing
17. Prove that the function: f ( x ) =  − x  is an
 2 + cos x  function on R.

( )
increasing function of x in 0, π .[A.I. 2011]
2
(i) f (x) = 4x3 – 18x2 + 27x + 5
(ii) f (x) = 3x3 + 6x2 + 4x + 8
3 2
18. Show that the function f (x) = 3x + 6x + 4x + 8 is 7. Show that each of the following functions, is strictly
increasing function for all real values of x. increasing function on R.
19. Show that the function f (x) = 3x + sin x is strictly increasing (i) f (x) = 4x3 – 30x2 + 87x + 15
function for all real values of x. (ii) f (x) = x3 – 6x2 + 15x + 3
20. Show that the function: f ( x ) = log ( x + 1) − 2x
x+2 ( )
, x > −1 (iii) f (x) = log x, x > 0
(iv) f (x) = x + cot–1 x
strictly increases (S↑) for all real values of x.
(v) f (x) = x + tan–1 x
IV. Long Answer Type Questions
1. Find the intervals in which f (x) = [x(x – 2)]2 8. Show that each of the following functions, is an increasing
function on R.
(i) increases (↑) (ii) decreases (↓).
(i) f (x) = x + sin x
Also find the points on the curve f (x) = [x(x – 2)]2 where
the tangent is parallel to x-axis. [Delhi 2010, AI 2014] (ii) f (x) = x – sin x

Answers 6.2
I. Multiple Choice Questions (MCQs) (b) f(x) S↓ in the interval f ′(x) < 0
1. (c) Strictly increasing on R ⇒ 2x – 4 < 0 ⇒ x<2
Hint: f ′(x) = 3 > 0 ∀ x ∈ R, or x ∈ (–∞, 2)
\ f (x) is S↑ ∀ x ∈ R.
2. (a) ∀ x ∈ R
( )
3. (a) f (x) S ↓ in −∞, 3
4
Reason: f (x) = 2x2 – 3x
Hint: f (x) = ex ⇒ f ′(x) = ex > 0 ∀ x ∈ R
⇒ f ′(x) = 4x – 3
\ f (x) is S↑ ∀ x ∈ R.
f (x) S↓ in the interval f ′(x) < 0
3. (d)
3
⇒ 4x – 3 < 0 ⇒ x<
II. 1. Hint: f (x) = x3 – 6x2 + 12x + 8 4


f ′(x) = 3x2 – 12x + 12
f ′(x) = 3[x2 – 4x + 4]
or x∈( ) −∞, 3
4

\ f ′(x) is ↑ ∀ x ∈ R
= 3(x – 2)2 ≥ 0 (b) f (x) S ↑ in 3 ,∞
4( )
2. f (x) is S↑ in (2, ∞,) Reason: f (x) S↑ in the interval f ′(x) > 0
f (x) is S↓ in (–∞, 2) ⇒ 4x – 3 > 0 ⇒ x> 3
4
Reason: f (x) = x2 – 4x + 6 ⇒ f ′(x) = 2x – 4
(a) f(x) S↑ in the interval f ′(x) > 0
or x ∈ 3 ,∞
4 ( )
⇒ 2x – 4 > 0 ⇒ x>2 4. (i) f (x) S ↑ in (–1, ∞) and S ↓ in (–∞, –1)
or x ∈ (2, ∞) Reason: f (x) = xex ⇒ f ′(x) = (x + 1)ex

Application of Derivatives 149


E:\AMIT_WORKS\Exam_Guru\EG_Mathematics-12_(working_02-06-2022)\EG_Mathematics-12_working\Open_Files\Chap_6\Chap_6
\ 17-Aug-2022 Ved_Goswami Proof-5 Reader’s Sign _______________________ Date __________

(a) f (x) S ↑ in the interval f ′(x) > 0 ⇒ (x + 1)ex > 0


⇒ x > –1 or x ∈ (–1, ∞)

⇒ x ∈ (–∞, 1) ∪ 5 ,∞
3 ( )
(b) f (x) S ↓ in the interval f ′(x) < 0 f (x) is S↓ in the interval: f ′(x) < 0
⇒ (x + 1)ex < 0 ⇒ x < –1
⇒ 2(x – 1)(3x – 5) < 0
⇒ x ∈ (–∞, –1)
(ii) f (x) S ↑ in (–∞, 1) and S ↓ in (1, ∞)
–x –x
⇒ 1<x<

5
3
or x ∈ 1, 5
3 ( )
Reason: f (x) = xe
⇒ f ′(x) = (–x + 1)e
(a) f (x) S ↑ in the interval f ′(x) > 0
–x
3 { }
2. f (x) is ↑ in the intervals: x ≤ 4 ∪ { x ≥ 2}

{ ( }
⇒ (1 – x)e > 0 ⇒ 1–x>0
⇒ x< 1 or x ∈ (–∞, 1) or x ∈ −∞, 4  ∪ { x ∈[ 2, ∞ )}
3 
(b) f (x) S ↓ in the interval f ′(x) < 0
f (x) is ↓ in the interval

⇒ (1 – x)e–x < 0 ⇒ 1–x<0




x–1> 0
x ∈ (1, ∞)
⇒ x>1
4≤ ≤2
3
x { 4
} {
or x ∈  , 2
 3  }
Hint: f (x) = (x – 1)(x – 2)2

(iii) f (x) S ↑ in (–∞, –2) ∪ (0, ∞) and S ↓ in (–2, 0)

⇒ f ′(x) = (x – 2)(3x – 4)
Reason: f (x) = x2ex
⇒ f ′(x) = x(x + 2)ex
(a) f (x) S ↑ in the interval f ′(x) > 0 (a) f (x) ↑ in the interval f ′(x) ≥ 0
or x(x + 2)ex > 0 ⇒ x(x + 2) > 0
⇒ (x – 2)(3x – 4) ≥ 0
⇒ x < –2, x > 0 ⇒ x ∈ (–∞, –2) ∪ (0, ∞) 4
⇒ x ≥ 2
or x ≤
3

{ ( )}
(b) f (x) S ↓ in the interval f ′(x) < 0
4
x(x + 2)ex < 0 ⇒ –2 < x < 0 or x ∈ −∞, ∪ { x ∈( 2, ∞ )}
3
⇒ x ∈ (–2, 0)
(b) f (x) is ↓ in the interval: f ′(x) ≤ 0
(iv) f (x) S ↑ in (0, 2); S ↓ in (–∞, 0) ∪ (2, ∞)

⇒ (x – 2)(3x – 4) ≤ 0
Reason: f (x) = x2e–x
⇒ f ′(x) = –x(x – 2)e–x
(a) f (x) S ↑ in the interval: f ′(x) > 0
⇒ – x(x – 2)e–x > 0
⇒ x(x – 2) < 0
⇒ 4≤x≤2

3
or { x ∈  4 , 2
 3  }
3. f (x) is S↑ in the intervals: {x < –2} ∪ {x > –1}

⇒ 0<x<2

⇒ x ∈ (0, 2) or {x ∈ (–∞, –2)} ∪ {x ∈ (–1, ∞)}

(b) f (x) S ↓ in the interval f ′(x) < 0 f (x) is S↓ in the interval: {–2 < x < –1}
⇒ –x(x – 2)e–x < 0
⇒ x(x – 2) > 0 or {x ∈ (–2, –1)}

⇒ x>2 or x < 0 Hint: f (x) = 2x3 + 9x2 + 12x + 20


⇒ x ∈ (–∞, 0) ∪ (2, ∞)
⇒ f ′(x) = 6x2 + 18x + 12 = 6(x + 1)(x + 2)

{ }
III. 1. f (x) S↑ in the interval: x > 5 ∪ { x < 1}
3
(a) f (x) is S↑ in the interval: f ′(x) > 0

⇒ 6(x + 1)(x + 2) > 0 ⇒ {x > –1} ∪ {x < –2}
5
{ ( )}
or { x ∈ (−∞,1)} ∪ x ∈ , ∞
3 or {x ∈ (–∞, –2)} ∪ {x ∈ (–1, ∞)}

f (x) is S↓ in the interval:


(b) f (x) is S↓ in the interval: f '(x) < 0


{ } { ( )}
1 < x < 5 or x ∈ 1, 5
3 3

⇒ 6(x + 1)(x + 2) < 0
or {x ∈ (–2, –1)}
⇒ {–2 < x < – 1}

Reason: f (x) = 2x3 – 8x2 + 10x + 5


4. f (x) is ↑ in the intervals:





f ′(x) = 6x2 – 16x + 10
f ′(x) = 2[3x2 –8x + 5]
5
{ } {
x ≤ 8 ∪ x ≥ 2}


⇒ f ′(x) = 2(x – 1)(3x – 5)
f (x) is S↓ in the interval: f ′(x) > 0

or { ( }
x ∈ − ∞, 8  ∪ { x ∈[2, ∞)}
5 
f (x) is ↓ in the interval:

} { }

⇒ 2(x – 1)(3x – 5) > 0

⇒ x<1
or x >
5
5
{
8 ≤ x ≤ 2 or x ∈  8 , 2
 5 
3
150 Mathematics–12
E:\AMIT_WORKS\Exam_Guru\EG_Mathematics-12_(working_02-06-2022)\EG_Mathematics-12_working\Open_Files\Chap_6\Chap_6
\ 17-Aug-2022 Ved_Goswami Proof-5 Reader’s Sign _______________________ Date __________

Hint: f (x) = (x – 1)3­ (x – 2)2


1 6x3 − 12x 2 − 30x + 36

⇒ f ′(x) =

⇒ 2
f ′(x) = (x – 2)(x – 1) (5x – 8) 5 

f (x) is ↑ in the interval: f ′(x) ≥ 0


6  x3 − 2x 2 − 5x + 6

⇒ f ′(x) =
5 
⇒ (x – 2)(x – 1)2(5x – 8) ≥ 0

6 x −1 x + 2 x − 3
⇒ {x ≥ 2} ∪ x ≤

8
5 { }
⇒ f ′(x) =
5
( )( )( )
Solve: (x – 1)(x + 2)(x – 3) = 0
or { ( } 8
x ∈ −∞,  ∪ { x ∈[ 2, ∞ )}
5 

⇒ x = –2, 1, 3

f (x) is ↓ in the interval: f ′(x) ≤ 0



⇒ (x – 2)(x – 1)2(5x – 8) ≤ 0
Put x = –3 in f ′(x). It gives negative sign. Therefore, in between


⇒ { 5 } {
8 ≤ x ≤ 2 or x ∈  8 , 2
 5  } –2 and 1, f ′(x) is positive. In between 1 and 3, f ′(x) is negative.
For x > 3, f ′(x) is positive.
5. Hint: f (x) = x3 – 3x2 + 4x, x ∈ R \ f (x) S↑ in the interval f ′(x) > 0


⇒ 2
f ′(x) = 3x – 6x 4 ⇒ {–2 < x < 1} ∪ {x > 3}

2
= [3(x – 1) + 1] > 0 ∀ x ∈ R or {x ∈ (–2, 1)} ∪ {x ∈ (3, ∞)}
\ f (x) is S ↑ ∀ x ∈ R
and f (x) S↓ in the interval f ′(x) < 0
6. f (x) is S↑ in the intervals: {1 < x < 2} ∪ {x > 3} ⇒ {x < –2} ∪ {1 < x < 3}

or {x ∈ (1, 2)} ∪ {x ∈ (3, ∞)} or {x ∈ (–∞, –2)} ∪ {x ∈ (1, 3)}
f (x) is S↓ in the intervals: {x < 1} ∪ {2 < x < 3}

or {x ∈ (–∞, 1)} ∪ {x ∈ (2, 3)}
{ } { ( )}
8. f (x) is S↑ in the interval: x > 1 or x ∈ 1 , ∞
2 2

Hint: f (x) = x4 – 8x3 + 22x2 – 24x + 21


and f (x) is S↓ in the interval:




f ′(x) = 4x3 – 24x2 + 44x – 24
f ′(x) = 4(x – 1)(x – 2)(x – 3) 2
{ } { (
x < 1 or x ∈ − ∞, 1
2 )}
f (x) is S↑ in the interval: f ′(x) > 0
Since the behaviour of the function f (x) in the interval (– 1, 1)
and f (x) S↓ in the interval: f ′(x) < 0 is not same.
f ′(x) = 4(x – 1)(x – 2)(x – 3) = 0 ∴ f (x) neither strictly increases nor strictly decreases in the interval
(– 1, 1).
gives x = 1, 2, 3
Hint: f (x) = x2 – x + 1 in (–1, 1)


⇒ f ′(x) = 2x – 1
If you put x = 0 in f ′(x) = 4(x – 1)(x – 2)(x – 3), we get a negative f (x) is S↑ in the interval: f ′(x) > 0

{ }
number. So, for x < 1, f ′(x) is negative, for 1 < x < 2, f ′(x) is
positive, for 2 < x < 3, f ′(x) is negative and for x > 3, f ′(x) is
⇒ 2x – 1 > 0 ⇒ x> 1
2

{ ( )}
positive.
(a) f (x) is S↑ in the interval: {1 < x < 2} ∪ {x > 3}
or x ∈ 1 ,∞
2
or {x ∈ (1, 2)} ∪ {x ∈ (3, ∞)} f (x) is S↓ in the interval: f ′(x) < 0

{ }
(b) f (x) is S↓ in the interval: {x < 1} or {2 < x < 3}

⇒ 2x – 1 < 0 ⇒ x< 1
or {x ∈ (–∞, 1)} ∪ {x ∈ (2, 3)} 2
7. f (x) is S↑ in the intervals: {–2 < x < 1} ∪ {x > 3}
or {x ∈ (–2, 1)} ∪ {x ∈ (3, ∞)}
or { ( )}
x ∈ −∞, 1
2
f (x) is S↓ in the intervals: {x < –2} ∪ {1 < x < 3} 9. f (x) is S↑ in the intervals: {x < –1} ∪ {x > 1}
or {x ∈ (–∞, –2)} ∪ {x ∈ (1, 3)} or {x ∈ (–∞, –1)} ∪ {x ∈ (1, ∞)}
3x 4 4x3 2 36x and f (x) is S↓ in the interval:
Hint: f (x) = 10 − 5 − 3x + 5 + 11

{–1 < x < 1} or {x ∈ (–1, 1)}
3 2
6x − 12x − 6x + 36 1

⇒ f ′(x) = Hint: f (x) = x3 + 3 , x ≠ 0

5 5 5 x

Application of Derivatives 151


E:\AMIT_WORKS\Exam_Guru\EG_Mathematics-12_(working_02-06-2022)\EG_Mathematics-12_working\Open_Files\Chap_6\Chap_6
\ 17-Aug-2022 Ved_Goswami Proof-5 Reader’s Sign _______________________ Date __________

1 f (x) is S↑ in the interval f ′(x) > 0





⇒ f ′(x) = 3x2 – 3x–4 = 3  x 2 − 4  1 x+2 x−2 >0
 x  ⇒ ( )( )
2x 2

⇒ f ′(x) = 34  x 6 − 1
x   ⇒ {x > 2} ∪ {x < –2}
or {x ∈ (2, ∞)} ∪ {x ∈ (–∞, –2)}
=
3
4 ( )(
( x − 1)( x + 1) × x 2 + x + 1 x 2 − x + 1 ) f (x) is S↓ in the interval: f ′(x) < 0

(a) f (x) S ↑ in the interval f ′(x) > 0 1 x+2 x−2 <0
⇒ ( )( )
3 2x 2
⇒ (x – 1)(x + 1)(x2 + x + 1)(x2 – x + 1) > 0
4 ⇒ {–2 < x < 2} or {x ∈ (–2, 2)}
⇒ {x > 1} ∪ {x < – 1} 13. f (x) is S↑ in the interval:

{ } { ( )}
or {x ∈ (1, ∞)} ∪ {x ∈ (–∞, –1)}
0 < x < π or x ∈ 0, π
f (x) S ↓ in the interval f ′(x) < 0
4 4

⇒ 3(x – 1)(x + 1)(x2 + x + 1)(x2 – x + 1) < 0
f (x) is S↓ in the interval:

⇒ {–1 < x < 1} or{x ∈ (–1, 1)}
} { ( )}

10. f (x) is S↑ in the interval:

π
4 { π
< x < π or x ∈ , π
4

4
{ } { ( )}
x ≥ 5 or x ∈ 5 , ∞
4 Hint: f (x) = sin x +

π
4 ( )
on [0, p]

( )
f (x) is S↓ in the intervals: π
f ′(x) = cos x +
{ } { ( )}


4
x < 5 or x ∈ − ∞, 5
4
( )
4
π
Solve: cos x + =0
Hint: f (x) = (x – 1)(x – 2)
3 4

f ′(x) = (x – 2)2(4x – 5) π 3π π , 5π

⇒ ⇒ x + π = ,
⇒ x=
4 2 2 4 4
f (x) is S↑ in the interval f ′(x) > 0

⇒ (x – 2)2(4x – 5) > 0 π π  5π > π 
Intervals: 0 < x < and < x < p Since 
4 4  4
⇒ { }
x> 5
4
or { ( )}
x ∈ 5 ,∞
4 Decision Making Table

f (x) is S↓ in the interval f ′(x) < 0


Intervals x ( )
f ′ ( x ) = cos x + π
4
sign Result

⇒ { }
x< 5
4
or { ( )}
x ∈ −∞, 5
4
0< x< π 30° cos (75°) + S↑
4
11. f (x) is S↑ in the interval:
{x < –1} or {x ∈ (–∞, –1)} π
<x<π 90° cos (135°) – S↓
4
f (x) is S↓ in the interval:
{x > –1} or {x ∈ (–1, ∞)}
12. f (x) is S↑ in the intervals:
\ f (x) is S↑ in 0 < x <{π or x ∈ 0, π
4 4 } { ( )}
{x < –2} ∪ {x > 2}
or {x ∈ (–∞, –2)} ∪ {x ∈ (2, ∞)}, x ≠ 0
f (x) is S↓ in
{ 4 } { ( )}
π < x < π or x ∈ π , π
4
14. f (x) is S↑ in the intervals:
f (x) is S↓ in the interval:
{–2 < x < 2} or {x ∈ (–2, 2)}

{ } {
0 < x < 3π ∪ 7π < x < 2π or
4 4 }
1  x2 
{ ( )} { ( )}
1 2
Hint: f ′(x) = 2 − 2 = 2  2 − 2

x x   x ∈ 0, 3π ∪ x ∈ 7π , 2 π
4 4
1  x 2 − 4

⇒ f ′(x) = f (x) is S↓ in the interval:

2x 2  


⇒ f ′(x) =
1 x+2 x−2
2x 2
( )( )
4
{ 4 4 4} { (
3π < x < 7π or x ∈ 3π , 7π
)}
152 Mathematics–12
E:\AMIT_WORKS\Exam_Guru\EG_Mathematics-12_(working_02-06-2022)\EG_Mathematics-12_working\Open_Files\Chap_6\Chap_6
\ 17-Aug-2022 Ved_Goswami Proof-5 Reader’s Sign _______________________ Date __________

Hint: f (x) = sin x – cos x on [0, 2p]


π π π

( )
Intervals 0 < x < , <x<
⇒ f ′(x) = cos x + sin x = 2 sin x + π 4 4 2
4 Now make the decision table and decide the intervals in which
Now solve: cos x + sin x = 0 f (x) is S↑ or S↓.
3π , 7π
⇒ tan x = –1 ⇒ x= 17. f (x) is S↑ in the interval:
4 4

Intervals: 0 < x <



4
, and

4
<x<

4
,

{
0 < x < π or x ∈ 0, π
2 2 } { ( )}
7π < x < 2π Hint: f ′ ( x ) = cos x (4 − cos2 x)
4 (2 + cos x)
Decision Making Table
( )
In first quadrant 0, π , cos x, (4 – cos x) and (2 + cos x)2 are positive:

( )
2
π
Intervals x f ′ ( x ) = 2 sin x +
4
sign Result \ f ′ ( x ) > 0 in 0, π
2( )
0< x<

4
π
2
2 sin ( )

4
+ S↑
{ } { ( )}
\ f ( x ) is S ↑ in 0 < x < π or x ∈ 0, π
2 2
2 sin ( 5π )
3π 7π
<x< p – S↓ 18. Hint: f ′(x) = (9x2 + 12x + 4)
4 4 4
⇒ f ′(x) = (3x + 2)2 ≥ 0

< x < 2π 330° 2 sin (375°) + S↑ \ f ′(x) is ↑ ∀ x ∈ R
4
19. Hint: f ′(x) = (3 + cos x) > 0 ∀ x ∈ R
\ f (x) is S↑ in the interval

\ f (x) is S ↑ ∀ x ∈ R

4{ 4} {
0 < x < 3π ∪ 7π < x < 2π } 20. Hint : f ′ ( x ) = x2
(x + 1) (x + 2) 2

or { ( )} { (
x ∈ 0, 3π
4
∪ x ∈ 7π , 2π
4 )} \ f ′(x) > 0 ∀ x ∈ R except x > – 1
\ f (x) is S ↑ ∀ x ∈ R except x > –1.
f (x) is S↓ in the interval
IV. 1. f (x) is ↑ in the intervals:

< x <
4

4 { } or { (
x ∈ 3π , 7π
4 4 )} {0 ≤ x ≤ 1} ∪ {2 ≤ x < ∞}
or {x ∈ [0, 1]} ∪ {x ∈ [2, ∞)}
15. f (x) is S↑ in the intervals: f (x) is ↓ in the intervals:


{ } {
0 < x < π ∪ 5π < x < 2π
4 4 } {x ≤ 0} ∪ {1 ≤ x ≤ 2}
or {x ∈ (–∞, 0]} ∪ {x ∈ [1, 2]}

{ ( )} { ( )}
Points at which tangents are parallel to x-axis are: A(0, 0),
or x ∈ 0, π ∪ x ∈ 5 π , 2π and B(1, 1), C(2, 0)
4 4
2. f (x) is S↑ in the interval:
f (x) is S↓ in the interval:

} { ( )}
{1 < x < 3}

4
{
π < x < 5π or x ∈ π , 5π
4 4 4 or {x ∈ (1, 3)} and
f (x) is S↓ in the intervals:

Hint: See the Solution of Q.No. 14.
{x < 1} ∪ {x > 3}
16. f (x) is S↓ in the interval:
or {x ∈ (–∞, 1)} ∪ {x ∈ (3, ∞)}


{ 4 } { ( )}
0 < x < π or x ∈ 0, π and
4
3. Hint: f (x) = x3 – 3x2 + 4x
⇒ f ′(x) = 3x2 – 6x + 4 = 3[(x – 1)2 – 1] + 4
f (x) is S↑ in the interval:
= 3(x – 1)2 + 1 > 0 ∀ x ∈ R


π< <π
4
x
2{ π π
or x ∈ ,
4 2 } { ( )} \ f (x) is S ↑ ∀ x ∈ R.
4. Hint: f ′(x) = 3[x2 – 2x + 1] = 3(x – 1)2 ≥ 0
 π \ f (x) is ↑ ∀ x ∈ R.
Hint: f (x) = sin4 x + cos4 x on 0, 
 2 5. Hint: f ′(x) = 3(x2 – 4x + 4) = 3(x – 2)2 ≥ 0
⇒ f ′(x) = –sin 4x
Solve: sin 4x = 0 ⇒ 4x = 0, p, 2p \ f (x) is ↑ ∀ x ∈ R.
π π 6. (i) Hint: f ′(x) = 3[4x2 – 12x + 9] = 3[(2x – 3)2] ≥ 0
⇒ x = 0, ,
4 2 \ f (x) is ↑ ∀ x ∈ R.

Application of Derivatives 153


E:\AMIT_WORKS\Exam_Guru\EG_Mathematics-12_(working_02-06-2022)\EG_Mathematics-12_working\Open_Files\Chap_6\Chap_6
\ 19-Aug-2022 Ved_Goswami Proof-5 Reader’s Sign _______________________ Date __________

(ii) Hint: f ′(x) = 9x2 + 12x + 4 = (3x + 2)2 ≥ 0


x2
\ f (x) is ↑ ∀ x ∈ R. (iv) Hint: f ′(x) = >0
1+ x 2
7. (i) Hint: f ′(x) = 12x2 – 60x + 87 \ f (x) is S ↑ ∀ x ∈ R.

( )
2
= 12 x − 5 + 12 > 0 x2 + 2
2 (v) Hint: f ′(x) = >0
x2 + 1
\ f (x) is S ↑ ∀ x ∈ R. \ f (x) is S ↑ ∀ x ∈ R.
(ii) Hint: f ′(x) = 3x2 – 12x + 15 = 3(x – 2)2 + 3 > 0
\ f (x) is S ↑ ∀ x ∈ R.
(
8. (i) Hint: f ′(x) = 1 + cos x = 2 cos 2 x ≥ 0
2 )
\ f (x) is ↑ ∀ x ∈ R.
1
(iii) Hint: f ′(x) =
x
>0 Q x>0 (ii) Hint: f ′(x) = 1 – cos x = 2 sin 2 x ≥ 0
2
\ f (x) is S ↑ ∀ x ∈ R. \ f (x) is ↑ ∀ x ∈ R.

Topic 3. Local Maximum and Local Minimum Values of a Function


Take a function y = x3 – 3x and draw its graph. y = x3 – 3x at points A and B are zero. So to get coordinates of
In the graph, observe the point A(x1, y1) we find that at the point A and B, solve:
point A, the function y = x3 – 3x has the maximum value in its dy
= 3x2 – 3 = 0 ⇒ 3(x + 1)(x – 1) = 0 ⇒ x = –1, 1
neighbourhood. If we move to left or right side of the point A dx
on the graph, the value of the function reduces. Therefore the Now we want to decide which value of x; – 1 or 1, maximises
3 or minimises the value of the function y = x3 –3x. For this we use
function y = x – 3x has the maximum value at A and this value
the second derivative test.
of the function is called local maximum value of the function
2
y = x3 – 3x and value of x1 at A is called point of local maxima. So, find d y2 = 6x.
dx
d 2y
Now put the values of x in = 6x, which we have got
dx 2
dy
after solving = 0 (x = – 1, 1).
dx
d 2y
The value of x out of x = – 1, 1, makes negative will
dx 2
3
maximise the value of y = x – 3x and value of x out of x = – 1,
d 2y
1 makes positive, will minimise the value of the function
dx 2
y = x3 – 3x.
At point A(x1, y1), x1 is the point of local maxima and y1 is
d 2y 
the local maximum value of the function y = x3 – 3x. 2  = 6x ]x = −1 = −6 ( negative )
dx  x =1
Now observe the point B on the graph of the function y =
∴ x = – 1 is the point of local maxima and y] x = –1
x3 – 3x. If we move to the left or right side of the point B on the 3
= [x – 3x]x = –1 = 2 is the local maximum value of the given
graph, the value of the graph increases, therefore the value of the
function.
function y = x3 – 3x at point B is minimum in its neighbourhood
d 2y 
and hence this value of the function is called local minimum 2  = [ 6x ]x =1 = 6 ( positive )
dx  x =1
value of the function y = x3 – 3x and value of x2 at B(x2, y2), is
∴ x = 1 is the point of local minima and y] x = –1
called point of local minima. On point B(x2, y2), x2 is the point
= [x3 – 3x]x = 1 = –2 is the local minimum value of the given
of local minima and y2 is the local minimum value of the function function y = x3 – 3x.
y = x3 – 3x. Hence,
How to get points A and B? (i) Point of local maxima : x = – 1
3
If we observe the graph of the function y = x – 3x, the points A and B (ii) Local maximum value : y = 2
are located at such position that the tangent through them are parallel (iii) Point of local minima : x = 1
to x-axis. Hence the slopes of the tangents on the curve of the function (iv) Local minimum value : y = – 2

154 Mathematics–12
E:\AMIT_WORKS\Exam_Guru\EG_Mathematics-12_(working_02-06-2022)\EG_Mathematics-12_working\Open_Files\Chap_6\Chap_6
\ 19-Aug-2022 Ved_Goswami Proof-5 Reader’s Sign _______________________ Date __________

Local Maximum and Minimum Write these numbers as shown in the above figure.
y Now put x = – 2, 0 and 3 turn by turn in
f ′(x) = 6(x + 1)(x – 2)
Local maximum
Decision Making Table

x f ′(x) = Sign Result


6(x + 1)(x – 2)
f(x) –2 (–)(–) + When x changes from –2 to 0,
f ′(x) changes (+ to –) \ x =
–1 is the local maxima.
Local minimum 0 (+)(–) – When x changes from 0 to 3,
x f ′(x) changes (– to +) \ x = 2
is the point of local minima.
3 (+)(+) +
First Derivative Test
Local f ' (a) = 0 f ' (a–) > 0 f ' (a+) < 0 Point of local maxima : x = – 1
Maximum Local maximum value : f (– 1) = 11
Local f ' (b) = 0 f ' (b–) < 0 f ' (b+) > 0
Point of local minima : x = 2
Minimum
Saddle f ' (c) = 0 f ' (c–) > 0 and f ' (c+) > 0 Local minimum value : f (2) = – 16
Point or f ' (c) < 0 and f ' (c) < 0 Example 2. If f (x) = (x – 4)3 (x + 1)2 find a point of local maxima,
Second Derivative Test point of local minima and the point of inflexion.
Solution. f (x) = (x – 4)3 (x + 1)2
Local f ' (a) = 0 f ' ' (a) < 0
Maximum ⇒ f ′(x) = 3(x – 4)2(x + 1)2 + 2(x + 1)(x – 4)3
Local f ' (b) = 0 f " (b) > 0 ⇒ f ′(x) = (x – 4)2(x + 1)[3(x + 1) + 2(x – 4)]
Minimum
⇒ f ′(x) = 5(x – 4)2 (x + 1)(x – 1)
Saddle f ' (c) = 0 f " (c) = 0
Point To get critical points or turning points, solve
Example 1. If f (x) = 2x3 – 3x2 – 12x + 4, find the point of local 5(x – 4)2 (x + 1)(x – 1) = 0 ⇒ x = –1, 1, 4.
maxima, point of local minima, local maximum value and local
minimum value of the function. Use first derivative test. x=–1 x=1 x=4
Solution. f (x) = 2x3 – 3x2 – 12x + 4
⇒ f ′(x) = 6x2 – 6x – 12 = 6(x – 2)(x + 1) –2 0 2 5
To find turning points, solve f ′(x) = 0
\ 6(x + 1)(x – 2) = 0 ⇒ x = –1, 2 Decision Making Table
For making decision of the point of local maxima or point of x f ′(x) = 5 Sign Result
local minima, we are going to use first derivative test. (x – 4)2
f ′(x) = 6(x + 1)(x – 2) and x = –1, 2 (x + 1)
(x – 1)
x=–1 x=2 –2 (+)(–)(–) + (Changes + ⇒ x = –1 is
to –) the point of
–2 0 3 L. maxima.
0 (+)(+)(–) – (Changes – ⇒ x = 1 is
We have two points x = – 1, 2. First take x = – 1 and take a to +) the point of
number just less than – 1. (We have taken it x = – 2). Also take L. minima.
a number just greater than – 1 but less than 2. (We have taken 2 (+)(+)(+) + (No change ⇒ x = 4 is
it x = 0).
in sign) the point of
Now take x = 2 and choose a number just less than 2 but inflexion.
greater than – 1, (We have taken it x = 0) and choose a number
5 (+)(+)(+) +
just greater than 2. (We have taken it 3).

Application of Derivatives 155


E:\AMIT_WORKS\Exam_Guru\EG_Mathematics-12_(working_02-06-2022)\EG_Mathematics-12_working\Open_Files\Chap_6\Chap_6
\ 19-Aug-2022 Ved_Goswami Proof-5 Reader’s Sign _______________________ Date __________

y Gradient = 0
Point of horizontal inflection
Positive Negative
Positive f ' (c–) > 0 f ' (c+) > 0
Gradient Gradient Positive
– + Gradient Gradient
f ' (a ) > 0 f ' (a ) < 0
Saddle
Point
gradient = 0
gradient = 0
Local minimum
Local maximum

y = f (x)
Negative Gradient Positive Gradient

f ' (b ) < 0 f ' (b+) > 0
x
x=a x=b x=c

Local Maximum and Local Minimum Value of Functions Not Having Local Maximum and Local
Modulus Function Minimum Values
Example 1. Find the local maximum or local minimum value, if
Example 1. Show that the following functions do not have local
any, of the following functions:
maxima or local minima points and values.
(i) f (x) = | x – 3 | + 5 (ii) f (x) = 7 – | x + 4 |
Solution. (i) f (x) = | x – 3 | + 5 (i) f (x) = ex
f (x) = – (x – 3) + 5 when x < 3 (ii) f (x) = x3 + 18x2 + 144x + 5
⇒ f (x) = – x + 8 when x < 3
Solution. (i) f (x) = ex
⇒ f ′(x) = –1 < 0 when x < 3
Now f (x) = x + 2 when x > 3 ⇒ f ′x) = 1 > 0 ⇒ f ′(x) = ex. To find points of local maxima or local minima,
when x > 3 solve: f ′(x) = 0
Therefore, sign of f ′(x) changes negative to positive as x ⇒ ex = 0. No solution.
changes from x < 3 to x > 3.
\ x = 3 is the point of local minima. \ Here f (x) = ex is a differentiable function and f ′(x) = 0 has

Thus, local minimum value: f (3) = | 3 – 3 | + 5 = 5 no solution.

Hence, there is no local maxima. \ f (x) = ex has no local maximum or local minimum values.
(ii) f (x) = 7 – | x + 4 |
(ii) f (x) = x3 + 18x2 + 144x + 5
⇒ f (x) = 7 – [– (x + 4)] for x + 4 < 0
⇒ f ′(x) = 3x2 + 36x + 144 = 3[x2 + 12x + 48]
⇒ f (x) = x + 11 for x < –4
⇒ f ′(x) = 1 > 0 for x < – 4 To find the points of local maxima or local minima,

Now f (x) = 7 – (x + 4) for x + 4 > 0 solve f ′(x) = 0.


⇒ f (x) = 3 – x for x > – 4 \ 3(x2 + 12x + 48) = 0
⇒ f ′(x) = –1 < 0 for x > – 4 ⇒ x2 + 12x + 48 = 0
\ f ′(x) changes sign from +ve to –ve when x changes from
Here: D = b2 – 4ac = (12)2 – 4 × 1 × 48
x < – 4 to x > – 4.
⇒ D = 144 – 192 = – 48 < 0
Thus, x = –4 is the point of local maxima.
Hence, local maximum value: f (–4) = 7 – | – 4 + 4 | = 7. \ f ′(x) = 0 has no solution.

156 Mathematics–12
E:\AMIT_WORKS\Exam_Guru\EG_Mathematics-12_(working_02-06-2022)\EG_Mathematics-12_working\Open_Files\Chap_6\Chap_6
\ 17-Aug-2022 Ved_Goswami Proof-5 Reader’s Sign _______________________ Date __________

EXERCISE 6.3
I. Multiple Choice Questions (MCQs) 6. Find the point of local maxima, point of local minima, local
Choose the correct answer from the given options. maximum value and local minimum value of the function:
1 f (x) = sin x + cos x in (0, 2p)
1. The maximum value of [ x ( x − 1) + 1 ]3 , 0 ≤ x ≤ 1 is 7. Find the point of local maxima, point of local minima, local
1 maximum value and local minimum value of the function:
 1 3 1 f (x) = sin x – cos x in (0, 2p)
(a)   (b) (c) 1 (d) 0
3 2 8. Find the point of local maxima; point of local minima, local
1 − x + x2 maximum value and local minimum value of the function:
2. For all real values of x, the minimum value of is
f (x) = 2 cos 2x – cos 4x in (0, p)
( )
2
1 + x + x2
1 9. Find the point of local maxima, point of local minima, local
(a) 0 (b) 1 (c) 3 (d) maximum value and local minimum value of the function:
3
f (x) = – x + 2 sin x in (0, 2p)
log x
3. The maximum value of is 10. Find the point of local maxima, point of local minima, local
x maximum value and local minimum value of the function:
2 1
(a) 1 (b) (c) e (d) π .
e e     f ( x ) = sin 4 x + cos 4 x on 0,
 2 
II. Short Answer Type Questions-II
1. It is given that at x = 1, function f (x) = x 4 – 62x 2
Local Maximum and Local Minimum Value of
+ ax + 9 attains the maximum value at the interval Modulus Function
[0, 2], find the value of a. IV. Very Short Answer Type Questions
2. The function f (x) = a log x + bx2 + x has extreme value at 1. Find the local minimum value of the function: f (x) = 3 +
x = 1 and x = 2, find the values of a and b.[Delhi 2009] | x |, x ∈ R.
2. Find the local maximum or local minimum value of the
III. Long Answer Type Questions
function: f (x) = 5 – | 2x – 7 |, if any.
1. If f (x) = x4 – 62 x2 + 120x + 9, find the points of local
maxima, local minima, local maximum value and local Functions Not Having Local Maximum and Local
minimum value of the function. Minimum Values
x 2
2. If f ( x ) = + , x ≠ 0 , find the point of local maxima, Prove that the following functions do not have local maxima or
2 x local minima.
point of local minima, local maximum value and local
minimum value of f (x). 3. g(x) = log x 4. f (x) = 4x + 5
5. f (x) = 3 – 5x 6. f (x) = x3 + 3x + 5
3. If f ( x ) = − 3 x 4 − 8x3 − 45 x 2 + 105 , find the point of local
4 2 7. f (x) = e1/x 8. f (x) = (x + 1)2/3
maxima, point of local minima, local maximum value and
V. Short Answer Type Questions
local minimum value of the function.
Prove that the following functions do not have local maxima or
4. If f (x) = (x – 1)(x + 2)2, find point of local maxima, point local minima.
of local minima, local maximum value and local minimum
1. h(x) = x3 + x2 + x + 1 2. f (x) = x3 – 6x2 + 15x – 8
value of the function f (x). Use first derivative test.
5. If f (x) = (x – 3)3, find the point of local maxima, point of 3. f (x) = 4x3 – 18x2 + 30x + 5 4. f (x) = x3 + 18x2 + 144x + 5
local minima and point of inflexion. 5. f (x) = x3 – 6x2 + 15x + 3

Answers 6.3
1 \ The maximum value of f is 1.
I. 1. (c) y = [ x ( x − 1) +1]3 Hence (c) is the correct answer.
1
dy 1 − x + x2
= 1 [ x ( x − 1) +1]3 × d ( x 2 − x +1)
−1

\ 2. (d) Let y =
dx 3 dx 1 + x + x2
2
1 − 2x −1
= [ x ( x − 1) + 1] 3 × (2 x − 1) = dy (1 + x + x 2 )(−1 + 2 x) − (1 − x + x 2 )(1 + 2 x)
3 2 =
3[ x( x − 1) + 1] 3 dx (1 + x + x 2 ) 2
dy 1
Now, for max. or min., = 0 ⇒ 2 x − 1= 0 ⇒ x = dy 2 ( x 2 −1)
dx 2
⇒ =
1 dx (1 + x + x 2 ) 2
 3 3
Now f (0) = 1, f  1  =   , f (1) = 1 For max. or min.,
dy
=0
2 4 dx

Application of Derivatives 157


E:\AMIT_WORKS\Exam_Guru\EG_Mathematics-12_(working_02-06-2022)\EG_Mathematics-12_working\Open_Files\Chap_6\Chap_6
\ 17-Aug-2022 Ved_Goswami Proof-5 Reader’s Sign _______________________ Date __________

Local Max. Value = f (x = 1) = 68


2 ( x 2 −1)

⇒     = 0 ⇒ x2 − 1 = 0 ⇒ x = ± 1 Point of Local Min : x = 5 and
2 2
(1 + x + x )
Local Min. Value = f (x = 5) = – 316
dy
When x is slightly < 1, < 0 and when x is slightly > 1, 2. Point of Local Max. : x = – 2 and
dx
dy Local maximum Value = f (– 2) = – 2
> 0.
dx Point of local minima : x = 2
dy Local Min. Value = f (x = 2) = 2

\ changes sign from – ve to + ve.
dx 3. Point of Local Max. : x = – 5 and
Hence, y is minimum at x = 1. 295
Local Max. Value = f (x = – 5) =
4
1 − x + x2 1 − 1 + (1) 2 1
Minimum value of 2 = = Point of Local Min. : x = – 3 and
1+ x + x 1 + 1 + (1) 2 3
231
Hence, (d) is the correct answer. Local Min. Value = f (x = – 3) = .
4
log x Point of Local Max. : x = 0 and
3. (d) f (x) =
x Local Max. Value = f (x = 0) = 105.
1 4. Point of Local Max. : x = – 2 and
× x − (log x) × 1
1 − log x Local Max. Value = f (x = – 2) = 0

\ f ′(x) = x =
x2 x2 Point of Local Min. : x = 0 and
For max. or min., f ′(x) = 0 Local Min. Value = f (x = 0) = – 4
1 − log x 5. No point of local max. and No point of local min.

\ = 0 ⇒ 1 – log x = 0 There is only point of inflexion at x = 3.
x2

⇒ loge x = 1 = loge e ⇒ x = e ∈ (0, ∞) π
6. Point of Local Max. : x = and
4

()
 1
x 2  −  − (1 − log x ) 2 x Local Max. Value = f π = 2.
f ′′(x) =  x 4
x4 5π
Point of Local Min. : x = and
− 3 x + 2 x log x − 3 + 2 log x 4

( )
= =
x4 x3 Local Min. Value = f 5π = − 2 .
4
−3 + 2 log e −3 + 2(1) − 1
Now, f ′′(e) = = = 3 <0 7. Point of Local Max. : x = 3π and
e3 e3 e 4
Thus, f (x) is maximum at x = e and maximum value

=
log e 1
=
Local Max. Value = f ( )
3
4
π = 2

e e Point of Local Min. : x = 7π and


4
Hence, (d) is correct answer.
II. 1. a = 120 Local Min. Value = f ( )

4
=− 2
Hint: f (x) = x4 – 62x2 + ax + 9 π
8. Point of Local Max. : x = and
⇒ f ′(x) = 4x3 – 124x + a 6


f (x) maximises at x = 1
\ f ′(x = 1) = 0
()
Local Max. Value = f π = 3
6 2
⇒ 4 – 124 + a = 0 ⇒ a = 120 Point of Local Min. : x = 0 and
−2 −1 Local Min. Value f (0) = 1
2. a = ;b = .

3 6 Point of Local Max. : x = and
Hint: f (x) = a log x + bx2 + x 6

⇒ f ′(x) = a + 2bx + 1
x
Local Max. Value = f ( )
5π = 3
6 2
f(x) maximises at x = 1 and x = 2 π
9. Point of Local Max. : x = and
\ f ′(1) = 0 and f ′(2) = 0 3

⇒ a + 2b = –1 and a + 4b = −1
2
Local Max. Value = f ()(
π
3
= 3−
π
3 )
Solve for a and b. 5π
Point of Local Min. : x = and
III. 1. Point of Local Min. : x = – 6 and 3
Local Min. Value = f (x = – 6) = – 1647
Point of Local Max. : x = 1 and 3( ) 
 3

Local Min. Value = f 5π = −  5π + 3 
2 

158 Mathematics–12
E:\AMIT_WORKS\Exam_Guru\EG_Mathematics-12_(working_02-06-2022)\EG_Mathematics-12_working\Open_Files\Chap_6\Chap_6
\ 17-Aug-2022 Ved_Goswami Proof-5 Reader’s Sign _______________________ Date __________

10. Point of Local Max. : x = 0 and


⇒ f ′(x) = –2 < 0 if x > 7
2
Local Max. Value = f (0) = 1
Hence, sign of f ′(x) changes from (+) to (–) as x changes from
π
Point of Local Min. : x =
4
and x < 7 to x > 7
2 2
Local Min. Value = f
π =1
4 2 () 7
\ x = is the point of local maxima and
2

() ( )
π
Point of Local Max : x = and local maximum value: f 7 = 5 − 2 × 7 − 7 = 5
2
2 2
Local Max. Value = f π = 1.
2 () 1
Hint: f ′ ( x ) = . Here f (x) is differentiable and
3. 
x
IV. 1. Local minimum value = f (0) = 3. f ′(x) ≠ 0.
Hint: f (x) = 3 + | x |, ⇒ f (x) = 3 –x for x < 0 \ f (x) = log x has no local maxima or local minima.
⇒ f ′(x) = –1 < 0 for x < 0 and f (x) = 3 + x for x > 0
4. 
Hint: f ′(x) = 4 ≠ 0. Hence f (x) is differentiable but f ′(x) ≠ 0.
⇒   f ′(x) = 1 > 0 for x > 0. So sign of f ′(x) changes from (–) to
(+) when x changes from x < 0 to x > 0. 5. Hint: f ′(x) = –5 ≠ 0.
\ x = 0 is the point of local minima and local minimum value: 6. f ′(x) = 3x2 + 3 ⇒ (x2 + 1) = 0 has no solution.
   f (0) = 3 + | x | = 3 + 0 = 3. 1/ x
7. f ′ ( x ) = −e 2 = 0 has no solution.
2. (i) Point of local maxima: x = 7 x
2
(ii) No point of local minima. 8. Hint: f ′ ( x ) = 2 ( x + 1)−1/ 3 = 2
2 =0
has no solution.
3
(iii) Local maximum value: f 7 = 5
2 () 3( x + 1) 3
V. 1. Hint: f ′(x) = 3x2 + 2x + 1 = 0 has no solution because
(iv) No local minimum value. D = b2 – 4ac < 0.
Hint: f (x) = 5 – [– (2x –7)] if x < 7 Hence f (x) is differentiable and f ′(x) = 0 has no solution.
2
⇒ f (x) = 5 + (2x – 7) if x < 7 2. Hint: f ′(x) = 3x2 – 12x + 15 = 3(x2 – 4x – 5) = 0 has no solution
2 because D < 0. Hence f (x) is differentiable but f ′(x) = 0 has no
⇒ f (x) = 2x – 2 if x < 7 solution.
2 3. f ′(x) = 12x2 – 36x + 30 = 6(2x2 – 6x + 5) = 0 has no solution
⇒ f ′(x) = 2 > 0 if x < 7 because D < 0.
2
4. f ′(x) = 3(x2 + 12x + 48) = 0 has no solution.
f (x) = 5 – (2x – 7) if x > 7 = 12 – 2x if x > 7 5. f ′(x) = 3(x2 – 4x + 5) = 0 has no solution.
2 2

Topic 4. Absolute Maximum and Absolute Minimum Values of a Function


Absolute maximum and absolute minimum values of a function: of the function on f (x) from A to G (including A and G) will be
y = f (x) on [a, b] are the maximum and minimum values of the the absolute maximum value of f (x) and any minimum value of
given function within the values x = a and x = b, including x = a the function f (x) from A to G (including A and G) will be absolute
and x = b. minimum value.
Note: Sometimes, absolute maximum value and the local
maximum value coincide.
This discussion shows that the absolute maximum and
absolute minimum values depend upto some extent on the values
of a and b.
If the values of a and b are changed then values of absolute
maxima and absolute minima may also change.

How to work out Absolute Maximum and Absolute


Minimum Values of a Function.
(i) Take the given function: y = f (x) on [a, b].
To understand the concept of absolute maximum and absolute dy
minimum, draw the graph of y = f (x) and x = a and x = b. (ii) Work out = f ′ ( x ) and solve f ′(x) = 0. Assume that it
dx
Now take the whole curve ABCDEFG. Any maximum value gives x = c, d.

Application of Derivatives 159


E:\AMIT_WORKS\Exam_Guru\EG_Mathematics-12_(working_02-06-2022)\EG_Mathematics-12_working\Open_Files\Chap_6\Chap_6
\ 17-Aug-2022 Ved_Goswami Proof-5 Reader’s Sign _______________________ Date __________

(iii) Out of c and d choose those values which lie between a To find the critical points, solve:
and b. Assume that only c lies between a and b. f ′(x) = 0 \ (x – 1)(x – 5) = 0
∴ Take x = a, b, c
⇒ x = 1, 5 out of these two values, only x = 1 lies
(iv) Work out f (a), f (b) and f (c).
between 0 and 4.
(v) Out of these three values, choose the maximum value. That
gives absolute maximum value. Now choose the minimum \ Take x = 0, 1, 4 and work out f (0), f (1) and f (4).
value out of the above three given values. This value is
f (0) = 8; f (1) = 31 and f ( 4) = 4
absolute minimum value. 3 3
Example 1. If f ( x ) = 1 x3 − 3x 2 + 5x + 8 on [0, 4], find the 4
3 Here is the minimum value and 31 is the maximum value.
point of absolute maxima, the point of absolute minima, absolute 3 3
maximum value and absolute minimum value of the above 4
∴ x = 4 is the point of absolute minima and f (4) = is the
function. 3
absolute minimum value of the function.
Solution. f (x) = 1 x3 − 3x 2 + 5x + 8 on [0, 4]
3 x = 1 is the point of absolute maxima and f (1) = 31 is the
⇒ f ′(x) = x2 – 6x + 5 = (x – 1)(x – 5) 3
absolute maximum value.

EXERCISE 6.4
I. Short Answer Type Questions-II 2. Find the absolute maximum and the absolute minimum
1. Find the absolute maximum and the absolute minimum values of the function:
values of the function: 4 1
f ( x ) = x − 4 on [4, 29]. f ( x ) = 12x 3 − 6x 3 on [ −1, 1]

2. Find the absolute maximum and the absolute minimum 3. Find the absolute maximum and the absolute minimum
value of the function: value of the function:
f ( x ) = x 32 − x 2 on [ −4, 4]. f (x) = sin x + cos x on [0, p].


II. Long Answer Type Questions 4. Find the absolute maximum and absolute minimum value
1. Find absolute maximum and the absolute minimum value of the function:
of the function. π
f ( x ) = sin 4 x + cos 4 x on 0, 
f (x) = 2x3 – 15x2 + 36x + 1 on [1, 5]  2 


Answers 6.4
I. 1. Absolute maximum value = f (29) = 5.
Absolute minimum value = f (4) = 0
3. Absolute maximum value = f π = 2 and
4 ()
2. Absolute maximum value = f (4) = 16 and
Absolute minimum value = f (– 4) = – 16 Absolute minimum value = f (π) = – 1
II. 1. Absolute maximum value = f (5) = 56
Absolute minimum value = f (1) = 24.
4. Absolute maximum value = f ( 0) = f
π =1
2 ()
2. f (1) = 18 is the absolute maximum value.

8 4()
f 1 = −9 is the absolute minimum value.
and absolute minimum value = f π = 1
4 2 ()
WORD PROBLEMS ON MAXIMA AND MINIMA
I. Short Answer Type Questions-II 2. Show that the rectangle of maximum area that can be
1. Find two numbers whose sum is 12 and product is inscribed in a given circle of diameter d is a square.
maximum. [A.I. 2011, 2013]
2. Show that among the rectangles of given perimeter, the 3. Show that the rectangle of maximum perimeter that can
be inscribed in a given circle of diameter d is a square.
square has the maximum area. [V. Imp.] [Delhi, 2011]
4. A rectangle is inscribed in a semicircle of radius r with
II. Long Answer Type Questions one of its side on the diameter of the semicircle. Find the
1. Show that among all the rectangles with given area square dimensions of the rectangle so that its area is maximum.
has the least perimeter. [Delhi 2011] Find the maximum area also.

160 Mathematics–12
E:\AMIT_WORKS\Exam_Guru\EG_Mathematics-12_(working_02-06-2022)\EG_Mathematics-12_working\Open_Files\Chap_6\Chap_6
\ 17-Aug-2022 Ved_Goswami Proof-5 Reader’s Sign _______________________ Date __________

5. Find the dimensions of a rectangle of maximum perimeter is 16 metres, find the dimensions of the window so that
that can be inscribed in a semicircle of radius r. Find the maximum amount of light may enter in the room.
maximum perimeter also. 20. The sum of the surface areas of a rectangular box
6. If the length of three sides of a trapezium other than the with sides 2x cm, x cm and x cm and a sphere of
base are equal to 10 cm, then find the maximum area of 2
the trapezium. [A.I. 2014 (C), Delhi 2013 (C)] radius r cm is given to be constant. Prove that the
sum of their volumes is minimum if 3x = 7r. Also
7. Prove that the area of a right angled triangle of given find the minimum value of the sum of the volumes.
hypotenuse is maximum when the triangle is isosceles.
21. The sum of the surfaces of a sphere and a cube is given.
[Delhi 2012 (C)]
Show that when the sum of the two volumes of the cube
8. Prove that the perimeter of a right angled triangle of given and sphere is least, the diameter of the sphere is equal to
hypotenuse is maximum when the triangle is isosceles. edge of the cube.
 [V. Imp.] 22. An open rectangular box with a square base and given
9. Two sides of a triangle have lengths a cm and b cm and surface area is made in such a way that its volume is
angles between them is θ. What value of θ will maximise maximum. Show that its height will be half of its width.
the area of the triangle? Find the maximum area of the 23. An open tank with square base and vertical sides is to
triangle. be constructed from a metal sheet so as to hold a given
10. If the sum of a side and the hypotenuse of a right angled quantity of water. Show that material will be least when
triangle be given. Show that the area of the triangle will the depth of the tank is half of its width.
be maximum when the angle between the given side and 24. 48 cm2 metal sheet is to be used in construction of an open
hypotenuse is 60°. [A.I. 2014, Foreign 2005] rectangular tank with square base, find the dimensions of
11. The perimeter of a triangle is 8 cm. If one of the sides is 3 the tank so that the tank has the maximum capacity.
cm, what are the other two sides for maximum area of the 25. An open rectangular tank with square base and capacity 108
triangle? cm3 is to be made out of a metal sheet. Find the dimensions
12. Let AP and BQ be two vertical poles at points A of the tank, so that minimum sheet is used.
and B respectively. If AP = 16 m, BQ = 22 m and 26. An open rectangular box with square base is to be made
AB = 20 m, then find the distance of a point R on AB from out of a given quantity of sheet of area a2. Show that the
the point A such that (RP)2 + (RQ)2 is minimum. a3
maximum volume of the box will be .
13. A wire of length l is cut into two pieces. One of the pieces 6 3
is to be bent into a square and the other into a circle. What [A.I. 2012 (C)]
should be the length of the two pieces, so that the combined 27. A square tank of capacity 250 m3 has to be dug out. The
area of the square and the circle is minimum? cost of the land is ` 50/m2. The cost of digging increases
14. A wire of length 28 cm is to be cut into two pieces. One of with depth and digging for the whole tank is 400 (depth)2
the pieces is to be bent into a square and other pieces into rupees. Find the dimensions of the tank for least cost.
a circle. What should be the lengths of two pieces so that 28. A tank with rectangular base and rectangular sides open
the combined areas of the square and circle is minimum? at the top is to be constructed so that its depth is 2m and
15. A wire of length 36 cm is cut into two pieces. One piece volume is 8m3. If building of the tank cost ` 70/m2 for the
is bent into a square and other piece into an equilateral base and ` 45/m2 for the sides. What is the cost of least
triangle. Find the lengths of each piece so that the sum of expensive tank? [Delhi 2015 (C)]
the areas of the square and triangle is minimum. 29. Prove that surface area of a solid cuboid with square base
16. A wire of length l cm is cut into two pieces. One piece is and given volume is minimum, when it is a cube.
bent into a circle and other piece is bent into an equilateral [Delhi 2015]
triangle. Find the lengths of the two pieces when sum of 30. A rectangular closed box with square base and given
the two areas of two figures is minimum. surface area is made in such a way that its volume is
17. Sum of the perimeters of a circle and a square is K, where maximum. Show that it will be a cube.
K is some constant. Prove that the sum of their areas is 31. A closed rectangular box with square base and given
least when the side of the square is double of the radius of volume 343 cm3 is made in such a way that its surface
the circle. [Delhi 2012, 2014 (C)] area is minimum. Find the dimensions of the box.
18. A window is in the form of a rectangle surmounted by a 32. A closed rectangular box with square base and given
semicircular opening. The total perimeter of the window surface area 726 cm2 is made in such a way that its volume
is P metres. Find the dimensions of the window when the is maximum. Find the dimension of the box.
window admits maximum light through the whole opening. 33. A metal square piece of tin of given side l cm, is to be made
19. A rectangular window is surmounted by an equilateral into a box without a top, by cutting equal square pieces
triangle. Given that the perimeter of the whole window from each corner and folding up the flaps to form an open

Application of Derivatives 161


E:\AMIT_WORKS\Exam_Guru\EG_Mathematics-12_(working_02-06-2022)\EG_Mathematics-12_working\Open_Files\Chap_6\Chap_6
\ 17-Aug-2022 Ved_Goswami Proof-5 Reader’s Sign _______________________ Date __________

rectangular box with square base. Prove that metal square i.e., with rectangular base and semi-circular ends. Show
pieces will be one-sixth of the side of the metal square that the ratio of the length of the cylinder to the diameter of
piece, when the volume of the box is maximum. its semi-circular ends is π : (π + 2), when the total surface
34. A rectangular sheet of metal of length ‘a’ units and breadth area is minimum. [HOTS]
‘b’ units is to be made into a box without a top by cutting 47. Show that the semi-vertical angle of a cone of maximum
equal square pieces from each corner and folding up the
flaps to form an open box. Show that when the volume of
volume and given slant height is tan −1 ( 2 ).
the box is maximum, the depth of the box will be equal to [V. Imp.] [A.I. 2011, Delhi 2014]
48. Show that the right circular cone of least curved surface
(a + b) − a 2 + b 2 − ab
. [HOTS] area and given volume has an altitude equal to 2 times
6
the radius of the base and semi-vertical angle is equal to
35. A square piece of tin of side 18 cm is to be made into a
 1 
box without a top, by cutting equal square pieces from tan −1  .
 2 
each corner and folding up the flaps to form the box, what
should be the size of the square to be cut off so that the 49. Show that the semi-vertical angle of a right circular cone
volume of the box is maximum. of given total surface area and the maximum volume is
36. A rectangular sheet of tin 45 cm long and 24 cm broad is
to be made into a box without a top by cutting off equal
1
sin −1 . 
3 () [HOTS]

square pieces from each corner and folding up the flaps. 50. Show that semi-vertical angle of a cone of given curved
What should be the side of each square to be removed from −1  1 
surface area and maximum volume is sin   .
each corner of the sheet so that the volume of the box is  3
maximum. 51. Show that the height of a right circular cylinder of
37. An open topped box is to be constructed by removing equal maximum volume that can be inscribed in a given right
square pieces from each corner of a 3 metres by 8 metres circular cone of height H is H . [V. Imp.]
rectangular sheet of aluminium and folding of the flaps. 3
Find the volume of largest box. 52. Show that the volume of the greatest cylinder which can
be inscribed in a cone of height H and semi-vertical angle
38. Show that the height of an open cylindrical can of given
surface area and greatest volume is equal to radius of the 4πΗ 3 tan 2 α
α is .
base. 27
39. Show that the height of an open cylindrical can of given 53. Prove that the radius of a right circular cylinder of
volume and least surface area is equal to the radius of the maximum curved surface area which can be inscribed in
base.[Delhi 2011 (C)] a given cone is half of the radius of the cone.
40. Show that a closed right circular cylinder of given surface  [V. Imp.] [A.I. 2012, 2013 (C)]
area and the maximum volume is such that its height is 54. Prove that the volume of the largest cone that can be inscribed
equal to diameter of the base. [Delhi 2012]
in a sphere of radius R is 8 of the volume of the sphere.
41. Show that a closed right circular cylinder of given volume 27
is made in such a way that its surface area is minimum.  [V. Imp.], [A.I. 2013 (C), 2014]
Show that its height will be equal to diameter of the base. 55. Show that the altitude of the right circular cone of
42. A closed circular cylinder has volume 2156 cm3. What will maximum volume that can be inscribed in a sphere of
be the radius of its base and height so that its total surface radius R is 4R . [Delhi 2013 (C)]
 22  3
area is minimum. Take π = 7  . [Delhi 2014] 56. Show that the height of a right circular cylinder of
 
43. A closed circular cylinder has surface area 924 cm2, what maximum volume that can be inscribed in a sphere of
will be its radius and height if its volume is maximum? 2r
radius r is . Also find the maximum volume.
 22  3
Find the maximum volume also.  Take π = 7  . [Delhi 2012 (C), 2013, A.I. 2012 (C), 2014]
 
44. A closed right circular cylinder of maximum volume is to 57. Show that volume of the greatest cylinder which can be
be made so that sum of its radius and height is equal to K, inscribed in a given cone of height H and semi-vertical
a constant. Prove that the ratio of its radius and height is 3
angle 30° is 4πH .
2 : 1. Find the maximum volume also. 81
45. A closed circular cylinder of maximum volume is made in 58. A cylinder is inscribed in a sphere of radius a,
such a way that the sum of its circumference and height is show that, if the volume of the cylinder is V, then
equal to K, a constant. What will be the ratio of its height V = 2πr 2 a 2 − r 2 , where r is the radius of the base of the
and radius? Find the maximum volume also. cylinder. Hence find the height of the cylinder when V is
46. A given quantity of metal is to be cast into half cylinder maximum and also find the maximum volume.

162 Mathematics–12
E:\AMIT_WORKS\Exam_Guru\EG_Mathematics-12_(working_02-06-2022)\EG_Mathematics-12_working\Open_Files\Chap_6\Chap_6
\ 17-Aug-2022 Ved_Goswami Proof-5 Reader’s Sign _______________________ Date __________

59. Find the maximum area of an isosceles triangle volumes is minimum if x is equal to three times the radius
2
y2 of the sphere. [A.I. 2015 (C)]
inscribed in the ellipse x 2 + 2 = 1 with the vertex at
a b 64. A manufacturer can sell x items at a price of ` (250 – x)
one end of the major axis. [HOTS] each. The cost of producing x items is ` (2x2 – 50x + 12).
Determine the number of items that can be sold so that he
60. Find the area of the greatest rectangle that can be inscribed
can make maximum profit. Find maximum profit also.
y2
( )
2
is an ellipse x 2 + 2 = 1 .[A.I. 2013]
a b 65. A manufacturer can sell x items at a price of ` 5 − x

( )
100
61. AB is the diameter of a circle and C is any point on the x
each. The cost price of x items is ` + 500 .
circle. Show that area of the ∆ABC is maximum when it 5
is isosceles. [A.I. 2014 (C)] Find the number of items he should sell to earn maximum
62. A point on the hypotenuse of a right triangle is at a distance profit.
of a and b from the sides of the triangle. Show that the 66. Find a point on the parabola y2 = 2x, which is closest to a
3 point (1, 4). [Delhi 2011 (C)]
 2 2 2
minimum length of the hypotenuse is  a 3 + b 3  . 67. An apache helicopter of enemy is flying on the
  curve : y = x 2 + 7. A soldier is placed at a point
 [HOTS] [Delhi 2015 (C)] (3, 7), wants to shoot down the helicopter when it is nearest
to him. Find the nearest distance.
63. The sum of the surfaces of a sphere and a cuboid with
x 68. Find the shortest distance of the point A(0, c) from the
sides , x and 2x, is constant. Show that the sum of their parabola y = x2 where 0 ≤ c ≤ 5.
3
Answers
Hint: If a rectangle is inscribed in a circle of diameter d. The

I. 1. 6, 6
diagonal of the rectangle passes through the centre of the circle.
Hint: Let the numbers be x and (12 – x).
2 2 2 2 2 2
Then their product = P = x(12 – x) = 12x – x2 Area = x d − x ⇒ (area) = A = x (d − x )
Find dP .
dx Now maximise A and prove that: l = b = d
2
Then solve: dP = 0 ⇒ x = 6, and 12 – x = 6.
dx
d 2P = −2
Find (negative)
dx 2
∴ x = 6 maximise P.

2. l = b = P
4
Hint: Let P be the fixed perimeter and x be one side of the

P d , P = 2  x + d 2 − x2 
rectangle. Then other side will be  − x . 3. l = b =
2  
2 
d .
If A is the area of the rectangle A = x ( )
P
2
−x .
Now maximise P. It will give x =
2

P
Now Maximise A and prove that: l = b =
4
II. 1. l = b = A
Hint: Rectangle: A = fixed area, x = one side.
A
∴ Other side = .
x
P be the perimeter of the rectangle.
Then P = 2 x + A .
x ( ) 4. l = 2r and b =
r . A = r2
2
Now minimise P and prove that l = b = A .
l = 2x, b =
r 2 − x 2 , Area = 2x r 2 − x 2
d
2. l = b =
2 A = (area)2 = 4(x2r2 – x4)

Application of Derivatives 163


E:\AMIT_WORKS\Exam_Guru\EG_Mathematics-12_(working_02-06-2022)\EG_Mathematics-12_working\Open_Files\Chap_6\Chap_6
\ 17-Aug-2022 Ved_Goswami Proof-5 Reader’s Sign _______________________ Date __________

r 9. θ = 90°
Now maximise A. It will give x = .
2
1
Hint: ∆ = ab sin θ
2

Find d ∆ and maximise ∆.



It will give you θ = 90°.

5. l = 4r , b = r , Pmax = 2 5r 10. θ = 60° Hint: BC = a, AB + BC = S


5 5
⇒ AB = (S – a)
Hint : Take fig. of Q. No. 4.

⇒ AC = (S − a) 2 − a 2 = S2 − 2aS
l = 2x, b = r 2 − x 2

Area of ∆ABC
\ P = 2  2x + r 2 − x 2  Maximise.
= (BC) (AC) = a S2 − 2aS
  1
2 r 2 2
It will give x = .
5 2
⇒ A = (area) 2 = a (S2 − 2aS)
6. A max = 75 3 cm 2 4
S
⇒ A = (Sa 2 − 2a 3)
4

Maximise A. Find

da ( ) S
It gives: a = .
3

11. a = 3 cm, b = 2.5 cm and c = 2.5 cm


Hint: Three sides of the ∆ are a, b and c.
a = 3 cm, b + c = (8 – 3) = 5 cm.

Hint: AB = (2x + 10) cm, DE = 100 − x 2 cm
Let b = x cm . Then c = (5 – x) cm
2
100 − x Let S be the half perimeter of the D.
Area = A =
2
[10 + (2x + 10)]

∴ S = 4 cm

⇒ A = 100 − x 2 [ x + 10]
Area of the ∆ = S(S − a) (S − b) (S − c)
Maximise A. It will give x = 5 cm.
h ; AC = h = 4 × 1 × (4 − x) (x − 1) = 4(x − 1) (4 − x)
7. BC = x =    
2 2 (Area)2 = A = 4(– x2 + 5x – 4). Maximise A.
x h2 − x2 It gives: x = 2.5 cm
Hint: Area =
2
2
12. AR = 10 metres
⇒ (Area) 2 = A = x (h 2 − x 2)
4
1
⇒ A = (h 2 x 2 − x 4).
4
Now maximise A.
It gives x = h .
2
h and AC = h
8. BC = x =
2 2
Hint: S = (RP)2 + (RQ)2

Hint: BC = x
⇒ S = (x2 + 256) + (20 – x)2 + 484

2 2
AC = h −x
⇒ S = 2x2 – 40x + 1140

⇒ P = h + x + h2 − x2 Minimise S. It gives x = 10 m.
Maximise P.
13. x = πl ; (l − x) = 4l
It gives x = h . π+4 π+4
2

164 Mathematics–12
E:\AMIT_WORKS\Exam_Guru\EG_Mathematics-12_(working_02-06-2022)\EG_Mathematics-12_working\Open_Files\Chap_6\Chap_6
\ 17-Aug-2022 Ved_Goswami Proof-5 Reader’s Sign _______________________ Date __________

Hint: 2
S = x + 3 (36 − x) 2
16 36

Minimise S : it will give you x = 144 3 .


9+4 3

 3πl 
16. Length of first piece = x =  cm
 3π + 9 

2πr = x ⇒ r = x Length of second piece = l − x =


 9l 
cm
2π  3π + 9 
l − x
4 (side) = (l – x) ⇒ side = 
 4  Hint:
2
S = x + 1 (l − x) 2
4 π 16

Minimise S. It gives: x = πl .
π+4

14. Length of first piece = 28π cm


π+4
Length of second piece = 112 cm
π+4 2
S = x + 3 (l − x )
2
Hint: 4π 36

Maximise S and on solving dS = 0 ,


dx

 
You will get x =  3πl  cm
 3π + 9 

17. x =  K  ; r = K ∴ x = 2r
 π + 4 2 (π + 4)

2πr = x ⇒ r = x cm, side =  28 − x  cm


2π  4 
 2 
⇒ S =  x + 1 ( 28 − x )2  cm2
 4 π 16 
Now minimise S : It gives x = 28π cm
π+4 ( )
 144 3 
15. Length of first piece =
 9 +4 3  cm
Hint: 4x + 2πr = K ⇒ r = K − 4x
 324  2π
Length of second piece =
 9 + 4 3  cm
If length of the wire is l cm, then length of first piece = S = x 2 + 1 (K − 4x) 2

 9l   
and length of second piece =  4 3l  cm
 9 + 4 3  cm  9 + 4 3 dS = 0
Now solve to minimise S.
dx
Hint:
K
It gives x = K and r =
π+4 2(π + 4)

2P metres, b = y =  P  metres and


18. l = x =  π + 4 
π+4

r= x =
P metres
2 π+4

Application of Derivatives 165


E:\AMIT_WORKS\Exam_Guru\EG_Mathematics-12_(working_02-06-2022)\EG_Mathematics-12_working\Open_Files\Chap_6\Chap_6
\ 17-Aug-2022 Ved_Goswami Proof-5 Reader’s Sign _______________________ Date __________

πx 1
Hint: P = 2y + x +
2 2
2 Hint: S = 6x2 + 4pr2 ⇒ r =  S − 6x 
 4π 
⇒ y = P − x − πx
2 2
3
2 2 4 
V = x + 4 πr 3 ⇒ V = x3 + 4 π  S − 6x 
3
2 2 3 3  4π 

A = Px − x − πx
   2 2 8 Solve: dV = 0 (for min. value of V)
dx
For Max. of A, solve
2
dA
= 0. 22. Hint: S = 4xh + x2 ⇒ h = S− x
dx 4x
 S − x2 
V = x 2h = x 2 
It gives x = 2P metres  4x 
π+4

 16  4
1
(
= Sx − x 3 )
l=x= m,
 6 − 3  Solve:
dV
=0
19. dx
8 3− 3 
b= y=
(
m
) (for max. value of V)
 6 − 3  It gives S = 3x2
23. Hint: V = x 2h ⇒ h = V2
Hint: 2y + 3x = 16 x
 V
  S = x2 + 4xh = x 2 + 4x  2 
y = 16 − 3x and A = xy +
3 2 x 

⇒ (x)
2 4
⇒ S= x +2 4 V

   (2 4 )
A = x 16 − 3x + 3 x 2
x
Now minimise S
dA = 0 3
For maximum value of A, solve Solve dS = 0 ⇒ V = x
dx dx 2
 16  24. x = 4 cm, h = 2 cm
It gives: x =   metres.
6− 3 2
Hint: x2 + 4xh = 48 ⇒ h =  48 − x 
20. Hint: Surface area of the rectangular box = 7x2  4x 
Surface area of the sphere = 4πr2    V = x2h
 2 3
S = 7x2 + 4πr2 = x 2  48 − x  = 48x − x
 4x  4
   
Maximise V, solve dV = 0 ⇒ x = 4 cm
dx
48 − x 2
and put x = 4 in h =
4x
25. x = 6 cm, h = 3 cm
Hint: x2h = 108 cm3
3
 S − 7x 2  2 ⇒ h = 108 cm
V = x3 + 4 πr 3 = x3 + 4 π  x2
3 3  4π 
  
S = x 2 + 4x  108 
Solve: dV = 0 (for min. value of V)  x 2 
dx   
432
( )
2
343 + 36π x 2    = x + Minimise S.
It gives S = x
49
Solve: dS = 0 ⇒ x = 6 cm and h = 108
( )
= 3 cm
343 + 36π x 2 = x 2 + πr 2 ⇒ 3x = 7r dx x2

⇒ 7 4 2 2
49 26. Hint: x + 4xh = a
21. a2 − x2

⇒ h=
4x
 2 2
   V = x2h = x 2  a − x 

 4x 

= 1  a 2 x − x3 
   4

166 Mathematics–12
E:\AMIT_WORKS\Exam_Guru\EG_Mathematics-12_(working_02-06-2022)\EG_Mathematics-12_working\Open_Files\Chap_6\Chap_6
\ 17-Aug-2022 Ved_Goswami Proof-5 Reader’s Sign _______________________ Date __________

For maximum volume, It gives: V = x3


2 3
Solve:
dV = 0 ⇒ x = a ; V = a x − x Put it in h = V2 ⇒ h = x.
dx 3 4 x

a a3 30. Hint: S = 2x2 + 4 xh


Put x = in it ⇒ Vmax =
3 6 3 2

⇒ h = S − 2x
4x
Height of the tank = x = 5 metre, 1
27. 2 V = x 2h = Sx − 2x3 
Length of the square base = 10 metre    4

Hint: Height = x metres, Volume = 250 m3


Solve dV = 0 ⇒ S = 6x 2
dx
250

∴ base area =
x S − 2x 2
Put it in h = = x.
4x
Cost of land = ` 250 × 50 = ` 12500
x x 31. x = 7 cm and h = 7 cm
Cost of digging = ` 400x2
Hint: V = 343 cm3.
Total cost
  x2h = 343
C = ` 12500 + 400x 2 
 x  ⇒ h = 343 cm
Minimise C: x2
S = 2x2 + 4xh
Solve: dC = 0 ⇒ x = 5 m
dx 2
= 2x 2 + 1372 .

x
( )
250 m 2 = 100 m 2    x

Now solve dS = 0 ⇒ x = 7 cm.



⇒ Side of the square tank = 10 m. dx
28. Cost = ` 1000 32. x = 11 cm and h = 11 cm
Hint: Height = 2m,
Hint: S = 726

Volume = 8m3.
⇒ 2x2 + 4xh = 726

Length of the base
2
= x metre
⇒ h = 726 − 2x
4 4x
Breadth = metres
x 3
Four walls area V = 726x − 2x
4
( )
  
=4 x+ 4
x Solve: dV = 0
dx
Cost of base construction = ` 4 × 70 = ` 280
Cost of construction of four walls
⇒ x = 11 cm

( )
= ` 4 x + 4 × 45 = 180 x + 4
x x ( ) Put it in h =
726 − 2x 2
4x
⇒ h = 11 cm.

∴ Total cost = C = 280 + 180 x + 4


x ( ) l
33. x = cm
6
dC = 0
Minimise C and solve ⇒ x = 2 metres.
dx
Put x = 2 in C = 280 + 180 x + 4
x ( )
V
29. Hint: V = x 2h ⇒ h = 2
x
2
  S = 2x + 4xh
= 2x 2 + 4V
  x
dS
Find
dx
dS = 0 l = (l – 2x) cm, b = (l – 2x) cm,
and solve
dx
h = x cm, V = lbh

Application of Derivatives 167


E:\AMIT_WORKS\Exam_Guru\EG_Mathematics-12_(working_02-06-2022)\EG_Mathematics-12_working\Open_Files\Chap_6\Chap_6
\ 17-Aug-2022 Ved_Goswami Proof-5 Reader’s Sign _______________________ Date __________

V = (l – 2x)2x
( )


37. Vmax = 200 m3

⇒ V = 4x3 – 4lx2 + l2x 27
dV l l
Solve = 0 ⇒ x = and
dx 2 6

34.

Hint: l = 8 – 2x = 2(4 – x) cm, b = (3 – 2x) cm


h = x cm, V = 4x3 – 22x2 + 24x
dV dV 2
Find . Solve = 0 ⇒ x = 3 cm, cm
dx dx 3
200 m3
Vmax = [4x3 − 22x 2+ 24x] 2 =
x=
3
27
Hint: l = (a – 2x) cm, b = (b – 2x) cm
38. Hint: Radius = r
h = x cm, V = lbh = (a – 2x) (b – 2x) x Height = h

⇒ V = 4x3 – 2(a + b)x2 + abx   S = pr2 + 2prh
2
2
dV = 0 ⇒ x = (a + b) ± (a + b) − 3ab
⇒ h = S − πr
Solve 2πr
dx 6
   V = pr2h
35. x = 3 cm  2
= πr 2  S − πr  = 1 [Sr − πr 3]
Hint: l = 2(9 – x) cm, b = 2(9 – x) cm
     2πr  2
dV dV = 0 ⇒ S = 3πr 2
Find and solve
dr dr
S − πr 2
Now find h using h =
2πr
39. Hint: Radius = r, Height = h
V = πr 2h ⇒ h = V2
πr
S = pr2 + 2prh

S = πr 2 + 2πr ×  V2 
 πr 
h = x cm, V = lbh
2V

⇒ V = 4[x3 – 18x2 + 81x] S = πr 2 +
r
dV = 0
Solve
dx
⇒ x = 9 cm and 3 cm. Now find dS and solve dS = 0 ⇒ V = πr 3
dr dr
36. x = 5 cm V
Find h using h =
πr 2
Hint: l = (45 – 2x) cm, b = (24 – 2x) cm, h = x cm

40. Hint: Height = h, Radius = r
S = 2pr2 + 2prh
2
⇒ h = S − 2πr
2πr
V = pr2h
 S − 2πr 2 
= πr 2  
V = lbh = 2[2x3 – 69x2 + 540x]  2πr 

Find dV ; Solve dV = 0 ⇒ x = 18 cm, 5 cm = 1 [Sr − 2πr 3]


dx dx 2

168 Mathematics–12
E:\AMIT_WORKS\Exam_Guru\EG_Mathematics-12_(working_02-06-2022)\EG_Mathematics-12_working\Open_Files\Chap_6\Chap_6
\ 17-Aug-2022 Ved_Goswami Proof-5 Reader’s Sign _______________________ Date __________

dV 2K ,
Now find and solve dV = 0 ⇒ S = 6πr2 ⇒   r = 0; r =

dr dr 3
Find h using h = K – r
S − 2πr 2
Find h using h =
2πr Find Vmax and show that r = 2 : 1 .
h
41. Hint: Radius = r 3
Height = h 45. r = K ; h = K ; h = π and Vmax = K
3π 3 r 1 27π
V = πr 2h ⇒ h = V2 Hint: Height = h cm
πr Radius = r cm

Now S = 2pr2 + 2prh and h + 2πr = K
⇒ h = K – 2πr
= 2πr 2 + 2V V = πr2h = π(Kr2 – 2πr3)
r
Find dV and
Solve dS = 0 ⇒ V = 2πr 3. dr
dr
Solve dV = 0
V = 2r. dr
Now find h using h =
πr 2
⇒ r = 0, K . Find h using h = K – 2pr
42. r = 7 cm, h = 14 cm 3π
Hint: Height = h cm ⇒ h = K . Find h and Vmax.
3 r
Radius = r cm 46. Hint: Let r, h, V and S be the radius, length, volume (fixed) and
V = 2156 cm3 (given) surface area of the half cylinder.
∴ πr2h = 2156
V = 1 πr 2h ⇒ h = 2V2
⇒ h = 2156 2 πr
πr 2 S = πr 2 + 2rh + πrh
( )

S = 2πr 2 + 2πrh = 2πr 2 + 2 × 2156 2(π + 2) V
r = πr 2 +
πr
dS dS = 0 ⇒ r = 7 cm dS
Now find and solve Find and solve
dr dr dr
2156 , use π = 22
Find h using h = dS = 0 ⇒ V =  π 2r 3 
πr 2 7  π + 2 
dr
3
43. r = 7 cm, h = 14 cm, Vmax = 2156 cm
2V ⇒ h =  π 
Hint: Height = h cm, Radius = r cm Now find h using h =
πr 2 2r  π + 2 
   S = 924 cm2 = 2πr2 + 2πrh = 924
2 47. Hint: Let l, x and V be the slant height, height and volume of
h = 924 − 2πr
( )

⇒   2
1
2πr the cone. Then radius = l 2 − x 2 and V = π l 2 − x 2 x
3
V = πr h = 1 [924r − 2πr 3]
2

  2 = 1 π (l 2 x − x3)
3
Find dV and
dr Find dV and solve
dV = 0 ⇒ r = 7 cm dx
Solve dV = 0 ⇒ = l
dr x
924 − 2πr 2 dx 3
Find h using h = , use π = 22 If θ is the semi-vertical angle, then
2πr 7
3 l 2 − x2
44. r = 2K ; h = K ; Vmax = 4πK ; r = 2 tan θ =
x
= 2
3 3 27 h 1
Hint: Height = h cm, 48. 
Let h, x and l be the height, radius and slant height of the cone,
so that l2 = h2 + x2.
Radius = r cm
Let V be the fixed volume.
r+h= K⇒h=K–r
V = πr2h = π(r2K – r3) V = 1 πx 2h ⇒ h = 3V2
  3 πx
Find dV and (Curved surface area)2
dr
2
dV = 0
solve S = (πxl) 2 =  πx h 2 + x 2 
dr    

Application of Derivatives 169


E:\AMIT_WORKS\Exam_Guru\EG_Mathematics-12_(working_02-06-2022)\EG_Mathematics-12_working\Open_Files\Chap_6\Chap_6
\ 17-Aug-2022 Ved_Goswami Proof-5 Reader’s Sign _______________________ Date __________

2 AM = ΜC

⇒ S = π 2 x 4 + 9V2

EF FC
x
dS dS = 0 ⇒ 3V = 2 πx3 ⇒ H R
Find and solved =
dx dx h R−x
3 ⇒ h = H (R − x)
Now h = 3V2 = 2π2x = 2 x;tan θ = x = 1 R
πx πx h 2 Let V be the volume of the cylinder.
49. Hint: Let h, r and l be the height, radius and slant height of the
cone, such that l2 = h2 + r2.
\ V = πx 2h = πH Rx 2 − x3
R ( )
Let S be the total surface area (fixed) dV dV = 0 ⇒ x = 2R
Find and solve
\ S = prl + pr2 = pr(l + r) dx dx 3

⇒ l= S −r
πr ( ) H
Now h = (R − x) =
R
H
R
R− 2R
3 (
= H.
3 )
(Volume of the cone)2 52. Hint: Same fig. as in Q. No. 51.
2 2R
We have proved x =
= V =  1 πr 2h  = 1 π 2r 4h 2 3
   3  9 R
Also = tan α ⇒ R = H tan α.

( )
2 4  2  H
= 1 π 2r 4(l 2 − r 2) = π r  S − r − r 2  2H tan α
9 9  πr  \ x= and h = H .
  3 3
2 4  2  Put these values in V = πx2h.
⇒ V = π r  S2 2 + r 2 − 2Sr − r 2 
9 π r πr  4πH3 tan 2 α
It gives V = .
S 2 4
27
⇒ V = Sr − 2π r  . 53. Hint: Let R and H be the fixed radius and height of the cone.
9
i.e., AM = H and MC = R.
dV = 0 ⇒ S = 4πr 2
Find dV and solve Let h and x be the height and
dr dr radius of the cylinder i.e., DG
  = PM = EF = h and MF = x.
r  r  πr 2 DAMC ~ DEFC
sin θ = =  2  =
l S − πr S − πr 2 H= R
  ⇒
  πr h R−x
Now put S = 4πr2. H
   h = (R − x)
50. Let h, r and l be the height, radius and slant height of the cone, R
Let C be the curved surface
such that : l2 = h2 + r2 area of the cylinder
Let S be the fixed curved surface area.
2
\ C = 2πxh = 2πx H (R − x) = 2πH (Rx − x 2)
R R

\ S = πrl ⇒ l 2 = S2 2 dC dC = 0 ⇒ x = R
πr Find and solve
1 2 4 2 dx dx 2
   V = (Volume)2 = π r h 54. Hint: Two dimensional cone ABC has been inscribed in a fixed
9
sphere of radius R and centre O. AO = BO = CO = R and OM = x.
2 4

⇒ V = π r (l 2 − r 2) ⇒ Radius of the variable cone = R 2 − x 2.
9
If V is the volume of the cone, then,
2 4  2 

⇒ V = π r  S2 2 − r 2  1
(
  V = π R − x ( R + x )
2 2
)
9 π r  3
⇒ V = 1 S2r 2 − π 2r 6  .
9
dV dV = 0.
Find and solve
dr dr
  S = 3πr 2

Now sin θ = r = r = 1
l S 3
πr
51. Hint: ABC is fixed cone in two dimensions. DEFG is the variable
cylinder inscribed in the fixed cone. Let R and H be the radius
dV = 0 ⇒ x = R .
and height of the fixed cone. AM = H and MC = R. Now find dV and solve
dx dx 3
Let h and x be the height and radius of the variable cylinder. i.e.
DG = PM = EF = h and MF = MG = x. Now find volume of the sphere and cone and find
Volume of cone
  DAMC ~ DEFC = 8
Volume of sphere 27

170 Mathematics–12
E:\AMIT_WORKS\Exam_Guru\EG_Mathematics-12_(working_02-06-2022)\EG_Mathematics-12_working\Open_Files\Chap_6\Chap_6
\ 17-Aug-2022 Ved_Goswami Proof-5 Reader’s Sign _______________________ Date __________

55. Hint: Fig. same is in Q. No. 54.


R R 4R
Prove x = , Now h = R + x = R + = .
3 3 3
56. Hint: AC = 2r and BC = x
Radius of the cylinder
= 1 4r 2 − x 2
    2
Volume of the cylinder
2
1 4 2
=V=π r − x 2  x.
     2  Hint: An isosceles ∆ABC has been inscribed in an ellipse. The
vertices of ∆ABC are:

⇒ V = π (4r 2 x − x3) A (a, 0), B (a cos θ, b sin θ) and
4
dV 2r C (a cos θ, – b sin θ).
dV
Find and solve dx = 0 ⇒ x = . OM = a cos θ and MB = b sin θ.
dx 3
∴ MA = OA – OM = (a – a cos θ)
r3 r3 = a (1 – cos θ) and BC = b sin θ – (– b sin θ)
Vmax = π [4r 2 x − x3] = 4π ⇒ Vmax = 4π
4 3 3 3 3 = 2b sin θ
  1
57. Hint: Same as Q. No. 52.      ∆ = 2 BC × AM = ab sin θ (1 − cos θ )
D = ab(sin q – sin q cos q)
4πH3 tan 2 α
We have proved V =

4πH3
27 ( 1
= ab sin θ − sin 2 θ
2 )
Now put α = 30°. It gives V = d ∆ = ab [cos θ − cos 2θ]. d∆ = 0
81 ⇒ Solved:
dθ dθ
3 θ θ
58. Height of the cylinder = 2a ⇒ cos θ − cos 2θ = 0 ⇒ 2 sin sin = 0
3 2 2
3θ = 0 or sin θ = 0
3 ⇒ sin
Vmax of the cylinder = 4πa 2 2
3 3 ⇒ 3θ θ
= 0, π or sin = 0, π
Hint: ABCD is a variable cylinder with radius r and height =
2 2
π , θ = 0, 2π ⇒ θ = 2π .
⇒ θ = 0, θ = 2
2 a2 − r 2 . 3 3
2
Find d ∆2 and it becomes negative for θ = 2π .
dθ 3
Max. ∆ = ab [sin θ − sin θ cos θ ]θ = 2π
3
3 3 ab
= ab sin 2π − sin 2π cos 2π  =
 3 3 3  4
60. Area of the greatest rectangle = 2ab

Here a is the radius of the sphere.


V = (volume of the cylinder)2
= 4p2r4(a2 – r2) ⇒ V = 4p2(a2r4 – r6)
dV dV = 0 2a
Now find and solve ⇒ r=
dr dr 3
Height of the cylinder = 2 a 2 − r 2
2 2
2a 2a Hint: GAEDHCFBG is the ellipse x 2 + y2 = 1 and ABCD is a
Put r = in it. It gives height = . rectangle with vertices shown in theafig. bl and b are length and
3 3
breadth of the rectangle.
2a
Vmax = 2πr 2 (a 2 − r 2). Put r = . ∴ l = DA = 2a cos θ, b = AB = 2b sin θ
3
A = lb = 2ab sin 2θ.
4πa 3 . dA
It gives Vmax =
3 3 Find and solve dA = 0
dθ dθ
3 3 ab π
59. A max = ⇒    θ = and A max = 2ab.
4 4

Application of Derivatives 171


E:\AMIT_WORKS\Exam_Guru\EG_Mathematics-12_(working_02-06-2022)\EG_Mathematics-12_working\Open_Files\Chap_6\Chap_6
\ 17-Aug-2022 Ved_Goswami Proof-5 Reader’s Sign _______________________ Date __________

61. Let ∠A = θ; AB = 2r 2
Now AC = 2r cos θ and BC = 2r sin θ
⇒ P = −x + 24x − 500
100 5
Area = A = r2 sin 2θ
Find dA and solve
dA = 0 Find dP and solve dP = 0 ⇒ x = 240.
dθ dθ dx dx

⇒θ= . π 66. P = (2, 2)


4
Now find AC and BC. We get AC = BC = 2r.
62. Hint: Let AM = x, PL = a and PM = b
∆CPL ~ ∆PAM
PL CL
\ =
AM PM
⇒ CL = PL × PM = ab
AM x
Now AB = x + a
and BC = b + CL
Hint: PA = (x − 1) 2 + (y − 4) 2
⇒ BC = b + ab
x ⇒ D = (PA)2 = (x – 1)2 + (y – 4)2

( )
2
⇒ l = (x + a) 2 + b + ab
2 2
 y2 
D =  − 1 + ( y − 4)
2
x

1 2  2 
dl dl
Find and solve = 0 ⇒ x = a 3b 3
dx dx dD dD = 0
Find and solve ⇒y=2
 Find l using x = dy dy
3 Now find x.
( )  2
2 2 2
(x + a) + b + ab
2
=  a3 + b3 67. D = 5 units
x  

63.

1
 2 2
Hint: S = 4πr + 6x ⇒ r = S − 6x
2 2
 4π 
3
V = Vcuboid + Vsphere = 2x + 4 πr 3
   3 3
dV dV = 0 PA = (x − 3) 2 + (y − 7) 2
Find and solve
dx dx
⇒ 9S = x2 (4p + 54). Put the value of S in D = PA2 = (x –3)2 + (y –7)2
1 = (x – 3)2 + (x2 + 7 – 7)2 = (x – 3)2 + x4
S 6 2  2
r =  − x  ⇒ x = 3r Find dD and solve
dD = 0 ⇒ x = 1.
4π 
     dx dx
64. x = 50 and Pmax. = ` 7, 488 . Find distance when x = 1

Hint: P = x(250 – x) – (2x2 – 50x + 12) Distance = (x − 3) 2 + x 4 = 5.


⇒ P = –3x2 + 300x – 12
dP dP = 0 4c − 1
Find and solve 68. Shortest distance =
dx dx 2
⇒ x = 50. Pmax. = ` 7488. Hint: AP =
(x − 0) 2 + (y − c) 2
65. x = 240
= x 2 + (x 2 − c) 2
( )
 2 
Hint: P =  5x − x  − x + 500 (Distance)2 = D = x2 + (x2 – c)2
 100  5

172 Mathematics–12
E:\AMIT_WORKS\Exam_Guru\EG_Mathematics-12_(working_02-06-2022)\EG_Mathematics-12_working\Open_Files\Chap_6\Chap_6
\ 17-Aug-2022 Ved_Goswami Proof-5 Reader’s Sign _______________________ Date __________

dD
Find and
dx
dD = 0
Solve
dx
2c − 1
⇒ x=0 or x =
2
⇒ c≥ 1
2

( )
2
2c − 1 + 2c − 1 − c
Distance =
2 2

= 4c − 1
2

Case Based Questions


1. The relation between the height of the plant (y in cm) with respect 2. P(x) = –5x2 + 125x + 37500 is the total profit function of
to exposure to sunlight is governed by the following equation a company, where x is the production of the company.
1 2
y = 4x – x where x is the number of days exposed to
2
sunlight.

(i) What will be the production when the profit is


maximum?
(i) The rate of growth of the plant with respect to sunlight
is ______. (a) 37500 (b) 12.5 (c) –12.5 (d) –37500
1 2 (ii) What will be the maximum profit?
(a) 4 x − x (b) 4 – x (a) ` 38,28,125 (b) ` 38281.25
2
1 2 (c) ` 39,000 (d) None
(c) x – 4 x (d) x −
2 (iii) In which interval the profit is strictly increasing?
(ii) What is the number of days it will take for the plant to (a) (12.5, ∞)
grow to the maximum height?
(b) for all real numbers
(a) 4 (b) 6 (c) 7 (d) 10 (c) for all positive real numbers
(iii) What is the maximum height of the plant? (d) (0, 12.5)
(a) 12 cm (b) 10 cm (iv) When the production is 2 units, what will be the profit
(c) 8 cm (d) 6 cm of the company?
(iv) What will be the height of the plant after 2 days? (a) ` 37500 (b) ` 37,730
(c) ` 37,770 (d) None
(a) 4 cm (b) 6 cm
(v) What will be the production of the company when the
(c) 8 cm (d) 10 cm profit is ` 38250?

(v) If the height of the plant is 7/2 cm, the number of days (a) 15
it has been exposed to the sunlight is ______.
(b) 30
(a) 2 (b) 3 (c) 2
(c) 4 (d) 1 (d) data is not sufficient to find
Ans. (i) (b) 4 – x (ii) (a) 4 (iii) (c) 8 cm Ans. (i) (b) 12.5 (ii) (b) ` 38281.24
(iv) (b) 6 cm (v) (d) 1 (iii) (d) (0 , 12.5) (iv) (b) ` 37,730 (v) (a) 15

Application of Derivatives 173


E:\AMIT_WORKS\Exam_Guru\EG_Mathematics-12_(working_02-06-2022)\EG_Mathematics-12_working\Open_Files\Chap_6\Chap_6
\ 17-Aug-2022 Ved_Goswami Proof-5 Reader’s Sign _______________________ Date __________

3. Rohan, a student of class XII, visited his uncles, flat with (i) Let the side of the square tank be x m, then the cost of
his father. He observed that the window of the house was land is:
in the form of a rectangle surmounted by a semicircular (a) ` 400 xxx (b) ` 100 x2
opening having perimeter 10 m as shown in the figure.
(c) ` 50 x2 (d) ` 200 x2
Based on the above information, give the answer of the
following questions. (ii) Total cost of land and digging is:
(a) ` (100 x2 + 3200 h2) (b) ` (100 x2 + 3200)
(c) ` (x2 + 3200 h2) (d) None of these
(iii) Height (h) of tank in terms of x and volume is:
x 4x
(a) 1000 x (b) 1000
1000 x2
y (c) 2 (d)
x 500
(iv) Village Panchayat wants minimum cost for making
tank, then h should be:
(a) 5 mx (b) 2.5 m (c) 3 m (d) 5.5 m
x
(v) For minimum cost, x should be :
(i) If x and y represent the length and breadth of the (a) 10 mx z(b) 15 m (c) 20 m (d) 30 m
rectangular region, then relation between x and y can
be represented as: Ans. (i) (b) `100x2 (ii) (a) `(100 x2 + 3200h2)
1000
(a) x + y + π = 10 (b) x + 2y + πx = 10 (iii) (c)
x2
(iv) (b) 2.5 m
2 2
π (v) (c) 20 m
(c) 2x + 2y = 10 (d) x + 2y + = 10
2 5. Shreya got a rectangular parallelopiped shaped box and
(ii) The area (A) of the window can be given by: spherical ball inside it as return gift. Sides of the box are
3 2
x 2 πx 2 x, 2x and x/3, while radius of the ball is r.
(a) x − x − x (b) 5x − −
8 2 2 8
πx3 3x 2 x 2 πx 2
(c) x + − (d) 5x + −
8 8 2 8
(iii) Rohan is interested in maximising the area of the whole
window, for this the value of x should be:
10 20 20 10
(a) 2 − π (b) 4 − π (c) 4 + π (d) 2 + π
(iv) Maximum area of the window is: Based on the given information, answer the following
30 30 50 50 questions:
(a) (b) 4 + π (c) 4 − π (d)
4−π 4+π (i) If S represents the sum of volume of parallelopiped and
(v) For maximum value of A, the breadth of rectangular sphere, then S can be written as:
part of the window is: 4x3 2 2 2
20 (a) + πr x (b) 2x + 4 πr 2
10 10 20 3 2 3 3
(a) (b) (c) (d)
4+π 4−π 4+π 4−π
2x3 4 3 2 4
x (c) + πr (d) x + πr
Ans. (i) (b) x + 2y + π = 10 3 3 3 3
2
2 2 (ii) If sum of the surface area of box and ball are given to
(ii) (b) A = 5x − x − πx be constant k2, then, x is equal to:
2 8
20 50 (a) k 2 − 4πr 2 (b) k 2 − 4πr
(iii) (c) (iv) (d)
4+π 4+π 6 6
10
(v) (a) (c) k 2 − 4π (d) None of these
4+π
4. During rainy season, a lot of water is wasted. To prevent 6
(iii) The radius of the ball, when S is minimum, is:
it, village Panchayat decides to dug out a square tank of
capacity 1000 cubic metres. The cost of land is `100 per k2 k2
m2. The cost of digging increases with the depth and for (a) 54 + π (b) 54 + 4π
the whole tank, it is ` (32000 × h2), where h metres is the
depth of the tank. Based on the above information, answer k2 k2
the following questions: (c) 64 + 3π (d) 4π + 3

174 Mathematics–12
E:\AMIT_WORKS\Exam_Guru\EG_Mathematics-12_(working_02-06-2022)\EG_Mathematics-12_working\Open_Files\Chap_6\Chap_6
\ 19-Aug-2022 Ved_Goswami Proof-5 Reader’s Sign _______________________ Date __________

(iv) Relation between length of the box and radius of the 7. As we know good planning can save energy, time, and
ball can be represented as: money. A farmer wants to construct a circular well and
r 3r square garden in his field. He wants to keep the sum of
(a) x = 2r (b) x = (c) x = (d) x = 3r
2 2 their perimeters 600 m:
(v) Minimum value of S is:
(a) k2 (b) k2
2/3
2(3π + 54) (3π + 54)3/ 2
3
(c) k (d) None of these
3(4π + 54)1/ 2
3
Ans. (i) (c) 2x + 4 πr 3 (ii) (a) k 2 − 4πr 2
3 3 6
(iii) (b) k 2 (iv) (d) x = 3r
54 + 4π (i) If the radius of the circular garden be r m and the side
k3 of the square garden be x m then sum of area S is:
(v) (c)
3 ( 4π + 54)
1/ 2
2
600 + 2πr 
6. A gardener wants to construct a rectangular bed of garden (a) S = πr 2 +  
in a circular patch of land. He takes the maximum perimeter  4
of the rectangular region as possible. (See the images given 300 + πr 
2

here for calculations) (b) S = πr 2 +  


 4
2
600 + 2πr 
D C (c) S = 2πr 2 +  
a  4
2
y
O  600 + πr 
a (d) S = πr +  
 4
A B (ii) Radius of circular well is:
600 300
(a) r = (b) r =
(i) The perimeter (P) of rectangle is: π+4 π+4
(a) 4 x + 4 a 2 − x 2 (b) x + a 2 − x 2 300 150
(c) r = (d) r =
(c) 4 x + a 2 − x 2 (d) x + 4 a 2 − x 2 2π + 4 π+2
(ii) To find critical points put (iii) For the given condition
dP dP d 2S d 2S
(a) > 0 (b) <0 (a) = 0 (b) <0
dx dx dr 2 dr 2
dP d 2S
(c) = 0 (d) None of these (c) > 0 (d) None of these
dx dr 2
(iii) Value of y is: (iv) The relationship between the side of the square garden
a a and the radius of the circular garden
(a) (b) (c) 2a (d) 2a
2 2 1
(iv) P is maximum when the rectangle is: (a) a = r (b) 2a = r2 (c) a = r (d) a = 2r
2
(a) Square (b) Parallelogram
(v) The number which exceeds its square by the greatest
(c) Rectangle (d) Trapezium possible number:
(v) If a rectangle of the maximum perimeter which can be
inscribed in a circle of radius 10 cm is a square then 1
(a) (b) 2 (c) 1 (d) 0
the sides of the region: 2 2
600 + 2πr 
(a) 10 8 cm (b) 2 10 cm Ans. (i) (a) S = πr 2 +  
 4
(c) 20 2 cm (d) 10 2 cm 300
(ii) (b) r =
dP π+4
Ans. (i) (a) 4 x + 4 a 2 − x 2 (ii) (c) =0
dx d 2S
a (iii) (c) > 0 (iv) (d) a = 2r
(iii) (b) (iv) (a) Square dr 2
2 1
(v) (d) 10 2 (v) (a)
2

Application of Derivatives 175


E:\AMIT_WORKS\Exam_Guru\EG_Mathematics-12_(working_02-06-2022)\EG_Mathematics-12_working\Open_Files\Chap_6\Chap_6
\ 17-Aug-2022 Ved_Goswami Proof-5 Reader’s Sign _______________________ Date __________

Author’s Comments
Questions based on following types are very important for Exams. So, students are advised to revise them thoroughly.
1. Questions based on increasing and decreasing or strictly increasing/decreasing function.
2. Questions based on tangents and normals. (Most Important)
3. Word Problems on Maxima and Minima.

IMPORTANT FORMULAE
1. Rectangle: If P is the perimeter, A is the area, D is the diagonal, l is the length and b is the breadth of a rectangle, then
P = 2(l + b) cm
A = lb cm2
D = l 2 + b 2 cm
2. Square: If x is the length of the side of the square, P is the perimeter, A is the area and D is the diagonal of the square, then
P = 4x cm
A = x2 cm2
D = 2x cm
3. Circle: C is the circumference, A is the area and r is the radius of the variable circle, then
C = 2pr cm
A = pr2 cm2
4. Cuboid: V = Volume, S = surface area, D = diagonal and l, b, h are length, breadth and height of a cuboid, then
V = lbh cm3
S = 2[lb + bh + hl] cm2
D = l 2 + b 2 + h 2 cm
5. Cube: V = Volume, S = Surface area and x is the side of the cube.
V = x3 cm3
S = 6x2 cm2
D = 3x cm
6. Sphere: V = Volume, S = Surface area and r is the radius of the sphere
V = 4 πr 3 cm3
3
S = 4pr2 cm2
dy
7. (a) For the values of x for which = f ′ ( x ) ≥ 0, f (x) increases.
dx
(b) For the values of x for which dy = f ′ ( x ) ≤ 0, f (x) decreases.
dx
dy
(c) For the values of x for which = f ′ ( x ) > 0, f (x) strictly increases.
dx
dy
(d) For the values of x for which = f ′ ( x ) < 0, f (x) strictly decreases.
dx

COMMON ERRORS
ERRORS CORRECTIONS
(i) Ignore –ve sign while finding increasing or decreasing (i) Students have to check increasing/decreasing using (1) and
interval. not (2).
E.g y = –2x3 – 3x2 + 36x – 1
dy
dx (
= 0 ⇒ − 6 x2 + x − 6 = 0 ) ...(1)

⇒ x2 + x – 6 = 0 ...(2)
While checking increasing/decreasing, students ignore –ve
sign as in (ii)

176 Mathematics–12
E:\AMIT_WORKS\Exam_Guru\EG_Mathematics-12_(working_02-06-2022)\EG_Mathematics-12_working\Open_Files\Chap_6\Chap_6
\ 17-Aug-2022 Ved_Goswami Proof-5 Reader’s Sign _______________________ Date __________

REVISION CHART

Increasing and Decreasing


Functions
(a) The function y = f (x) is
called increasing function
for all x values which satisfy Application of
f ′(x) ≥ 0. Derivatives
(b) The function y = f (x) is The two main
called decreasing function applications are Rate of Change of Bodies
for all x values which satisfy using derivatives
f ′(x) ≤ 0. This point reminds us about the application/
to determine interpretation of derivatives.
(c) The function y = f (x) is called information about
strictly increasing function graphs of functions
for all x values which satisfy and optimization
f ′(x) > 0. problems.
(d) The function y = f (x) is called
strictly decreasing function
for all x values which satisfy
f ′(x) < 0.

Local Maximum and Local Minimum Values of a Function

(A) Based on second derivative test: Let f (x) be a function (B) Based on first derivative test to decide points of local
defined on an interval I and x = c ∈ I. Let f (x) be twice maxima or points of local minima:
differentiable. Then, (i) Let f ′(x) = 0 at x = c. If f ′(x) > 0 at every point
(i) x = c is the point of local maxima if f ′(x) = 0 and sufficiently close to and to the left of x = c and
f ″(x) < 0. Then value f (c) is local maximum value f ′(x) < 0 at every point sufficiently close to and
of f (x). to the right of x = c, then x = c is the point of local
(ii) x = c is the point of local minima if f ′(x) = 0 and maxima.
f ″(x) > 0. In this case f (c) is local minimum value (ii) Let f ′(x) = 0 at x = c. If f ′(x) < 0 at every point
of f (x). sufficiently close to and to the left of x = c and
(iii) This second derivative test fails when f ′(x) = 0 f ′(x) > 0 at every point sufficiently close to and
and f ″(x) = 0. to the right of x = c, then x = c is point of local
minima.
(iii) If f ′(x) does not change sign as x increases
through c, then c is neither a point of local
maxima nor a point of local minima. In fact such
a point is called point of inflexion or inflection.

Application of Derivatives 177


E:\AMIT_WORKS\Exam_Guru\EG_Mathematics-12_(working_02-06-2022)\EG_Mathematics-12_working\Open_Files\Chap_7\Chap_7
\ 17-Aug-2022 Amit Proof 5 Reader’s Sign _______________________ Date __________

7 Integrals
Topics Covered
I. INDEFINITE INTEGRALS
7.1 Integration of Simple Algebraic Functions and Simple 7.2 Integration of Simple Trigonometric Functions
Exponential Functions
7.3 Integration by Substitution 7.4 Integration Using Standard Formulae
7.5 Directly using Formulae & Special Types of Integrals 7.6 Integration Using Partial Fractions
7.7 Integration By Parts 7.8 Repeating After Twice Integration
7.9 Integrals of Exponential Forms 7.10 Three More Formulae for Integration
II. DEFINITE INTEGRALS
7.11 Direct Evaluation of Definite Integrals 7.12 Properties of Definite Integrals
7.13 Odd and Even Functions

C hapter map
I. INDEFINITE INTEGRALS

Integration of Simple Integration by Substitution Integration of Simple


Algebraic Functions Exponential Functions

Integration of Simple
Integration using Integration using Trigonometric Functions
Standard Formulae Partial Fractions

Integration Three More


by Parts Formulae for Integration

II. DEFINITE INTEGRALS

Direct Evaluation of Properties of Definite


Definite Integrals Integrals

I. INDEFINITE INTEGRALS
Indefinite integral is the inverse-process of differentiation. If derivative of a function F(x) is f (x), then integral of function f (x) is F(x).
For example derivative of x5 is 5x4 then integral of 5x4 will be x5. Let us introduce a new symbol ∫ f ( x ) dx . This symbol represents
integration of f (x) with respect to x. If derivative of the function F(x) is another function f (x) then symbolically we write it as:

∫ f ( x ) dx = F(x), F(x) is also called anti-derivative of f (x).

178
E:\AMIT_WORKS\Exam_Guru\EG_Mathematics-12_(working_02-06-2022)\EG_Mathematics-12_working\Open_Files\Chap_7\Chap_7
\ 17-Aug-2022 Amit Proof-5 Reader’s Sign _______________________ Date __________

Constant of Integration
From the above definition of indefinite integral, we can write:
3
( )
unique. Actually ∫ 4x dx = x + (some unknown constant)
4

= x4 + C. It is true for all functions. Hence, when we integrate


(i)
d 4
dx
( )
x = 4x 3 ⇒ ∫ 4x dx = x
3 4
( ) a function, we must put ‘+ C’ with its integral. Here C is any
arbitrary constant called constant of integration.
(ii)
d 4
dx
(
x + 7 = 4x 3 ) ⇒ ∫ ( 4x ) dx = x
3 4
+7 In ∫f
(i)
( x ) dx = F ( x ) + C
the function f (x) to be integrated is called integrand.
(
(iii) d x 4 + k = 4x3 ) ∫ ( 4x ) dx = x (ii) F(x) is called indefinite integral of f (x).
3 4
⇒ +k
dx (iii) C is called constant of integration and
From the above examples, we conclude that integral of 4x3 is not (iv) x is called variable of integration.

Topic 1. Integration of Simple Algebraic Functions and Simple Exponential Function


Simple Algebraic Functions
∫ 7x
5
Example 3. Evaluate: dx
Formulae
 x5+1 
∫ 7x dx = 7  5 + 1 + C
5
Solution. I=
 x n +1  d  x n +1  
(i) ∫ x dx =  n + 1 + C, n ≠ − 1
n
Q    + C = x n
dx  n + 1  ⇒ I = 7 x6 + C
6
x3
Example 1. ∫ x + 1 dx is equal to
(iii)
∫  f ( x ) + g(x) + φ (x) dx
x 2 x3 = ∫ f ( x ) dx + ∫ g(x) dx + ∫ φ(x) dx + C
(a) x + + − log 1 − x + C
2 3 1

∫  x
5
x 2 x3 Example 4. Evaluate: + 3x 2 + 5x 2  dx
(b) x + − − log 1 − x + C 
2 3 1
 5
I = ∫  x + 3x + 5x 2  dx
2
x 2 x3 Solution.
(c) x − − − log 1 + x + C  
2 3 1

∫ x dx + ∫ 3x dx + ∫ 5x 2 dx
5 2
x 2 x3 =
(d) x − + − log 1 + x + C
2 3  3
x6  x3  x2
x 3
I= + 3  + 5  + C
Solution. Let I = ∫ x + 1 dx 6  3  3
 2
1 6 10 3/ 2
\ I=

∫  x
2
− x +1−
1 
dx =x +x +
3
x +C ( )
x + 1 6 3
x3 x 2
= −
3 2
+ x − log x + 1 + C (iv) ∫ k dx = kx + C, Here k is constant
x 2 x3 d
=x− + − log x + 1 + C Q
dx
[ kx + C] = k
2 3
Hence, the correct option is (d). Example 5. Evaluate: ∫ 4 dx
∫ 4 dx
3
Solution. I= = 4x + C
Example 2. Evaluate: ∫ x 2 dx
 3 +1 
x2  2
5 (ax + b) n + 1
∫ (ax + b) dx =
n
Solution. I = ∫ x dx =  3/ 2
+ C = x2 + C (v) + C, n ≠ − 1
 3 + 1 5 a(n + 1)
2 
d  ( ax + b )
n +1 
+ C = ( ax + b )
n
Q 
 xn + 1  dx  a ( n + 1) 
∫ ax dx = a  n + 1 + C, n ≠ − 1
n
(ii) 

∫ (3x + 5)
7
Example 6. Evaluate: dx
d   x n +1  
Q  a  + C = ax n ( 3x + 5)
7 +1

∫ (3x + 5) dx = 3
7
dx   n + 1  Solution. I= +C
(7 + 1)
Integrals 179
E:\AMIT_WORKS\Exam_Guru\EG_Mathematics-12_(working_02-06-2022)\EG_Mathematics-12_working\Open_Files\Chap_7\Chap_7
\ 17-Aug-2022 Amit Proof 5 Reader’s Sign _______________________ Date __________

(3x + 5) + C = 1 (3x + 5)8 + C ∫ ( x )


8 66 
I=  2
+ 10x + 27 + dx
= x − 4 
24 24
dx 3
Example 7. Evaluate: ∫ =x + 5x 2 + 27x + 66 log x − 4 + C
8x + 3 + 8x + 1 3
dx
Solution. I= ∫
8x + 3 + 8x + 1 dx −1
Rationalise denominators
(vi) ∫ x n = (n − 1) x n − 1 + C, n ≠ 1
( 8x + 3 − 8x + 1 dx )  
I= ∫ Q
d  −1
+ C  = 1n

(
8x + 3 − 8x − 1
8x + 3 − 8x + 1 dx )  ( )
dx  ( n − 1) x n −1  x

= ∫ 2 1
 3/ 2 
Example 11. Evaluate: ∫ x 4 dx
⇒ I= 1  ( 8x + 3)
3/ 2

( 8x + 1) 
+C
2  3 ×8 3 ×8   1 −1 −1
 2 2 
Solution. I= ∫  x 4  dx = ( 4 − 1) x 4−1 + C = 3x3 + C
1
= (8x + 3) − (8x + 1)  + C
3/ 2 3/ 2
dx −1
24 (vii) ∫ (ax + b)n = a (n − 1) (ax + b)n − 1 + C, n ≠ 1
(
Example 8. Evaluate: ∫ x 2 − 1  x + 13  dx
x  x  )    
Q d   −1 1
Solution. I= ∫ ( 2 1
x 
1
x  )
x −  x + 3  dx dx   a ( n − 1) ( ax + b ) 

n −1  + C =
  ( ax + b )
n

 1 1 dx
⇒ I= ∫  x
3
+ − 1 − 4  dx Example 12. Evaluate: ∫ ( 4x + 3)5
x x 
4
dx −1
=x + log x − x + 1 3 + C Solution. I= ∫ ( 4x + 3)5 = 4 (5 − 1) ( 4x + 3)5−1 + C
4 3x
(x 2
)
+ 2 (3x + 5)  −1   −1 
Example 9. Evaluate: ∫ x 2 dx I=  4 +C =  4 +C
 4 × 4 ( 4x + 3)  16 ( 4x + 3) 
(x 2
)
+ 2 (3x + 5)
Solution. I= ∫ x2
dx
(viii) ∫
dx
= log | x | + C Q d log x + C = 1
x dx x
 3x3 + 5x 2 + 6x + 10 
⇒ I= ∫  x2  dx Example 13. Evaluate: ∫
dx
x
 6 + 10  dx
=∫  3x + 5 +
 x x2 
 Solution. I= ∫
dx
x
= log x + C
2
⇒ I = 3x + 5x + 6 log x − 10 + C  dx 
2 x 1
(ix) ∫  ax + b  = a log | ax + b | + C
 ( x − 3) ( x + 2) ( x + 7 ) 
Example 10. Evaluate: ∫  ( x − 4)  dx
 Q
d 1
log ax + b + C =
1
dx  a  ax + b
 ( x − 3) ( x + 2) ( x + 7 ) 
Solution. I= ∫  ( x − 4)  dx dx
 Example 14. Evaluate: ∫ (7x + 9)
 ( x − 3) x 2 + 9x + 14  ( ) dx 1
⇒ I= ∫ 
x−4
 dx
 Solution. I= ∫ (7x + 9) = 7 log 7x + 9 + C
 
 x3 + 9x 2 + 14x − 3x 2 − 27x − 42  1
=∫  x−4  dx = log 7x + 9 + C
7
 3 2  (3x − 7)2 dx
I = ∫  x + 6x − 13x − 42  dx Example 15. Evaluate: ∫ 3x + 2
 ( x − 4) 
3 2
(3x − 7)2 dx
\ x + 6x − 13x − 42 = x 2 + 10x + 27 + 66
x−4 x−4 ( ) Solution. I= ∫ 3x + 2

180 Mathematics–12
E:\AMIT_WORKS\Exam_Guru\EG_Mathematics-12_(working_02-06-2022)\EG_Mathematics-12_working\Open_Files\Chap_7\Chap_7
\ 17-Aug-2022 Amit Proof-5 Reader’s Sign _______________________ Date __________

 9x 2 − 42x + 49  Example 18. Evaluate: ∫ 5 xdx


⇒ I= ∫  3x + 2  dx
5x
∫ 5 dx = log 5 + C
x
2 Solution. I=
9x − 42x + 49 81
\ = (3x − 16) +
3x + 2 3x + 2
5x

( )
⇒ I= +C
 81  log 5
\ I= ∫ (3x − 16) + 3x + 2  dx
a mx + n d  a mx + n 
3x 2 81 (iv) ∫ a dx =
mx + n
+C Q + C = a mx + n
= − 16x + log 3x + 2 + C m log a dx  m log a 
2 3
2 ( 2x + 5)dx
⇒ I = 3x − 16x + 27 log 3x + 2 + C Example 19. Evaluate: ∫7
2
( 2x + 5)
( 2x + 5)dx = 7
 dx  2 ax + b
(x) ∫ 
Solution. I= ∫7 2 log 7
+C
= a
+C
 ax + b  ( 2x+ 5)
⇒ I= 7 +C
  2 log 7
Q d  2 ax + b + C = 1
dx  a  ax + b  5 x + 23x 
Example 20. Evaluate: ∫  dx
 7 2x 
dx
Example 16. Evaluate: ∫  5 x + 23x   5 x 23x 
5x − 3 Solution. I= ∫  7 2x  dx = ∫  2x + 2x  dx
 7 7 
dx 2 5x − 3
Solution. I= ∫ 5x − 3
=
5
+C  ( 5) x (8)x  dx
I= ∫  ( 49)x ( 49)x 
 +
Simple Exponential Functions  

( ) ( )
 5 x x
8
(i) ∫e
x
dx = e x + C Q d e x + C = e x =∫  49 + 49  dx
dx  

( ) ( )
x x
1 5 8
(ii) ∫ e mx + n
dx = e mx + n + C 49 49
m
( ) ( )
= + +C
log 5 log 8
Q d  1 e mx + n + C = e mx + n 49 49
dx  m 
∫ (2 )
x 2
Example 21. Evaluate: + 3x dx
∫e
4x + 7
Example 17. Evaluate: dx

() ∫ (2 )
x 2
1 4x + 7 Solution. I= + 3x dx
∫ e dx =
4x + 7
Solution. I= e +C
4

()
( x) ( x)
∫ ( 2) + (3) + 2 × 2 × 3  dx
2x 2x
1 4x + 7 =
⇒ I= e +C
4
∫ 4 + 9 + 2 × 6  dx
x x x
=
ax  x 
∫ a dx = log a + C Q d  a + C = a x
x
(iii) 4x 9x 2.6 x
dx  log a  = + + +C
log 4 log 9 log 6

EXERCISE 7.1
I. Multiple Choice Questions (MCQs)  1 
3. The antiderivative of  x +
1.
d
dx ∫
(
f ( x)dx = )   equals
x
1 1 1 1
2 2 1
(a) f ′ (x) (b) (f(x))2 (a) x 2 + 2 x 2 + C (b) x 3 + x 2 + C
2 3 3 2
(c) f (x) (d) one of these 2 32 1
3 2 1
(c) x + 2x 2 + C (d) x 3 + x 2 + C
2. One value of ∫ f ′(x) dx is 3 2 2
1 4. ∫ (1 + 5x + 10x2 + 10x3 + 5x4 + x5) dx =
(a) f ′ (x) (b) (f(x))2
2 (a) 5 + 20x + 30x2 + 20x3 + 5x4
1 5 x 2 10 x 2 10 x 4
(c) (d) f(x) (b) x + + + + x5 + 5 x6
f ( x) 2 3 4

Integrals 181
E:\AMIT_WORKS\Exam_Guru\EG_Mathematics-12_(working_02-06-2022)\EG_Mathematics-12_working\Open_Files\Chap_7\Chap_7
\ 17-Aug-2022 Amit Proof 5 Reader’s Sign _______________________ Date __________

(1 + x)6 dx
(c)
3!
2. ∫ 4x + 3 + 4x − 5
(d) None of these dx
3. ∫ 8 − 5x − 7 − 5x
d 3
5. If f ( x) = 4 x 3 − 4 such that f(2) = 0, then f (x) is
dx
1 129
x
1 129
4. ∫ ( ) x+
1  2
x 
x +
1 
 dx
x
(a) x 4 + (b) x 3 + +
( )

x3 8 x4 8  1  1
5. ∫  x + 
x
x+
x
dx
1 129 4 1 129
3
(c) x + 3 + (d) x + 4 +
x 8
II. Very Short Answer Type Questions
x 8
6. ∫ ( ) x+
1  2 1
 x − 3  dx
x  x

Evaluate each of the following Integrals IV. Long Answer Type Questions
 3  1. ∫ (x + 1) (x + 2) (x + 3) dx
∫ (ax ∫ 5x 2 + x + 7 dx
2
1. + bx + c) dx 2.

3. ∫ (7x + 5) dx
5
4. dx
∫ x7
2. ∫x
2
 2
x

x (
 1 − x 2  x3 + 1 dx
)
dx dx (x + 1) (3x 2 − 1)
5. ∫ (4x + 7)5 6. ∫ 7x − 3
3. ∫ x2
dx

dx  (3x + 2) (2x − 3) 2 
∫ 9x + 5 ∫ (3x
3
7. 8. + 5) 2 dx 4. ∫  x2
 dx

2
2  4 1   1 
9. ∫ x  x + x + x + 3 dx 10. ∫  x − x  dx 5.
 (2x + 1) 2 
∫  3x + 2  dx
∫e ∫7
(4x + 3) (2x − 5)
11. dx 12. dx
 (x + 1) (x + 2) (x + 3) 
6. ∫  (x + 4)  dx

∫ (3 ∫ (5
x
13. × 2 x) dx 14. x
× 32x) dx
 x 4 + x 2 + 1
8 + 3 x x a +b x x
7. ∫  x 2 + x + 1  dx
15. ∫  5 x 
dx 16. ∫  a xb x 
dx
 x4 + 7 
8. ∫  x + 1 
dx
∫ (e )
3 log x
17. + e 2 log x dx

( )  dx
 3 x + 5 2x 2
III. Short Answer Type Questions-I 9. ∫ 
 22x 
1. ∫ 5 x + 7x3 x + x x  dx  

Answers 7.1
I. 1. (c) 2. (d) 3 x4 x− 3

∫  4 x
3
f (x) = 4
dx = 4 ⋅ − 3 ⋅
− +C
 1  1

1
x  4 −3
3. (c) ∫  x +  dx = ∫ x dx + ∫ x dx
2 2
1 1
 x ⇒ f (x) = x 4 + 3 + C \ f (2) = x 4 + 3 + C
3 1 x x
x 2
x 2 3 2 1
1 129
= + = x 2 + 2x 2 + C Now, f (2) = 0 ⇒ 16 + + C = 0 ⇒ C = −
3 1 3 8 8
2 2 4 1 129
Thus, f (x) = x + 3 −
4. (c) I = ∫ (1 + 5 x + 10 x 2 + 10 x3 + 5 x 4 + x5 ) dx x 8
5
ax3 bx 2 cx x 2 + 7x + C
(1 + x)6 (1 + x)6 (1 + x)6 II. 1. + + +D 2. 2x 2 +
= ∫ (1 + x)5 dx = = = 3 2 2
6 3 ⋅ 2 ⋅1 3! 6
−1
d 3 3. (7x + 5) + C 4. +C
5. (a) We have 3
f ( x) = 4 x − 4 42 6x 6
dx x
−1 2 7x − 3
5. +C 6. +C
Integrating both sides, we get 16(4x + 7) 4 7

182 Mathematics–12
E:\AMIT_WORKS\Exam_Guru\EG_Mathematics-12_(working_02-06-2022)\EG_Mathematics-12_working\Open_Files\Chap_7\Chap_7
\ 17-Aug-2022 Amit Proof-5 Reader’s Sign _______________________ Date __________

7. 1 log | 9x + 5 | + C
4 2
9 IV. 1. x + 2x3 + 11x + 6x + C
4 2
9x 7 + 15x 4 + 25x + C Hint: First multiply all the 3 factors and then integrate.
8.
7 2 8
x
Hint: Expand (3x2 + 5)2 and integrate. 2. log | x | − + C
8
5 2
9. x + x + log | x | + 3x + C Hint: First multiply 3 factors and then integrate.
5 2 3x 2 + 3x − log | x | + 1 + C
Hint: First multiply and then integrate. 3.
2 x
x2
10. − 2x + log | x | + C ( x + 1)(3x 2 − 1)
2
2 Hint: = 3x + 3 − 1 − 12  Now integrate.
Hint: Expand  x − 1  and then integrate. x2 x x
 x 4. 6x 2 − 28x + 3 log | x | − 18 + C
(2x − 5) x
11. 1 e(4x + 3)  + C 12. 7 +C Hint: Expand Nr and divide by x2, then integrate.
4 2 log 7
x 2x 2 + 4x + 1 log | 3x + 2 | + C
13. 6 + C . Hint: 3x × 2x = 6x. 5.
3 9 27
log 6
x ( 2x + 1)2 4x 2 + 4x + 1
14. (45) + C . Hint: 5x × 32x = 5x × 9x = (45)x. Hint:
3x + 2
=
3x + 2
log (45)
4 4 1
( ) + ( ) +C
x x x+ +
8 3 =
3 9 9 (3x + 2)
15. 5 5
log ( 8 ) log ( 3 )
x3 + x 2 + 3x − 6 log | x + 4 | + C
6.
5 5 3
( x + 1)( x + 2)( x + 3) = x3 + 6x 2 + 11x + 6
= ( ) +( )
x x x x
8 +3 8 3 Hint:
Hint:
5 5 x 5 ( x + 4) ( x + 4)
2 6
Now integrate. = x + 2x + 3 −
( x + 4)
16. −1 1 x3 − x 2 + x + C
− +C 7.
(a x) log a (b x) log b 3 2

Hint:
ax + bx 1
= x+ x
1
Now integrate. Hint:
x4 + x2 + 1
=
(
x2 + x + 1 x2 − x + 1 )( )
a xb x b a x2 + x + 1 x2 + x + 1 ( )
4 3
17. x + x + C;
 m log x
Q e =e
log x m 
= x m 
( ) = x2 – x + 1.
4 3 
Now integrate.
log x 3 2 4 3 2
Hint: e3 log x + e2 log x = e + elog x 8. x − x + x − x + 8 log | x + 1 | + C
3 9 5 4 3 2
III. 1. 10 x 2 + 14 x 2 + 2 x 2 + C
3 9
3
5
3 Hint:
x4 + 7
=
x4 − 1 + 8( )
 x +1
2. 1 (4x + 3) 2 − (4x − 5) 2  + C x +1
48   ( x + 1)( x − 1)( x 2 + 1)
dx = + 8
x +1 x +1
Hint: ∫ 4x + 3 + 4x − 5
Rationalise Dr and integrate.
(
= ( x − 1) x 2 + 1 + 8
x +1 )
 3 3 3 2 8
3. − 2 (8 − 5x) 2 + (7 − 5x) 2  + C = x − x + x −1+
x +1
15   Now integrate.
Hint: Rationalise the Dr and then integrate.
( 32 ) ( ) 2 ( 75 )
2x 2x x
3 −1 25
x 4 + 2 x 2 + x 2 − 2x 2 + C
4. 9. 2 4
2 log ( ) 2 log ( ) log ( 75 )
4 3 2 + + +C
3 25
Hint: Multiply two factors and then integrate. 2 2 4
5 −1
2 x 2 + 2x − 2x 2 + C
(3 )
x 2
5. + 52x 32x + 54x + 2 ( 75)
x
5 Hint: =
2x
Hint: Multiply two factors and then integrate. 2 22x

() ( ) ( )
4 2 2x 2x x
6. x + 1 + x + 1 3 + C = 3 + 25 + 2 75
4 x 2 3x 2 2 4
Hint: First multiply two factors and then integrate. Now integrate.

Integrals 183
E:\AMIT_WORKS\Exam_Guru\EG_Mathematics-12_(working_02-06-2022)\EG_Mathematics-12_working\Open_Files\Chap_7\Chap_7
\ 17-Aug-2022 Amit Proof 5 Reader’s Sign _______________________ Date __________

Topic 2. Integration of Simple Trigonometric Functions


Formulae 12. cos 4x = 8 sin4 x – 8 sin2 x + 1
cos 2 x − cos 2θ
1. ∫ sin x dx = − cos x + C Example 1. ∫ cos x − cos θ
dx is equal to

d (a) 2(sin x + x cos q) + C


Q
dx
( − cos x ) + C = sin x
(b) 2(sin x – x cos q) + C
(c) 2(sin x + 2x cos q) + C
2. ∫ cos x dx = sin x + C
(d) 2(sin x – 2x cos q) + C
d
Q (sin x + C) = cos x cos 2 x − cos 2θ
dx Sol. Let I = ∫
cos x − cos θ
dx
(2 cos x − 1) − (2 cos 2 θ − 1)
2
3. ∫ tan x dx = log | sec x | + C = ∫
cos x − cos θ
dx
2 2
d 2 cos x − 1 − 2 cos θ + 1
Q log sec x + C = tan x = ∫ dx
dx  cos x − cos θ
2 cos 2 x − 2 cos 2 θ
= ∫
4. ∫ cot x dx = log | sin x | + C
cos x − cos θ
dx

cos 2 x − cos 2 θ
d
log sin x + C = cot x = 2∫ dx
Q
dx  cos x − cos θ
(cos x + cos θ) (cos x − cos θ)
= 2∫ dx
5. ∫ sec x dx = log | sec x + tan x | + C
(cos x − cos θ)
= 2 ∫ (cos x + cos θ) dx
d \ I = 2(sin x + cos q . x) + C.
Q log sec x + tan x + C = sec x
dx  Hence, correct option is (a).
dx
Example 2. ∫ is equal to
6.
∫ cosec x dx = log | cosec x − cot x | + C sin ( x − a ).sin ( x − b)
= − log | cosec x + cot x | + C sin( x − b)
(a) sin (b − a) log +C
sin ( x − a )
Q d log ( cosec x − cot x ) + C = cosec x
dx sin( x − a )
(b) cosec (b − a) log +C
or d  − log cosec x + cot x + C = cosec x sin ( x − b)
dx
sin( x − b)
(c) cosec (b − a) log +C
∫ sec x dx = tan x + C
2
7. sin ( x − a )
d sin( x − a )
Q
dx
[ tan x + C] = sec2 x (d) sin (b − a) log
sin ( x − b)
+C

dx
∫ cosec x dx = − cot x + C
2
8. Sol. Let I = ∫ sin ( x − a).sin ( x − b)
d
Q
dx
[ − cot x + C] = cosec2 x Multiplying and dividing by sin (b – a) we get,
1 sin (b − a )
I = ∫
sin (b − a ) sin ( x − a ).sin ( x − b)
dx
9. ∫ (sec x tan x) dx = sec x + C 1 sin ( x + b − x − a )
sin (b − a ) ∫ sin ( x − a ).sin ( x − b)
= dx
d
Q
dx
(sec x ) + C = sec x tan x
1 sin [( x − a ) − ( x − b)]
10. ∫ (cosec x cot x) dx = − cosec x + C
= ∫
sin (b − a ) sin ( x − a ).sin ( x − b)
dx

d sin ( x − a )cos ( x − b)
Q [ −cosec x ] + C = cosec x cot x 1 − cos ( x − a )sin ( x − b)
dx =
sin (b − a ) ∫ sin ( x − a).sin ( x − b) dx
Note: If angle of T-ratio is (ax + b) then divide the integral of
1  sin ( x − a ).cos ( x − b)
T-ratio by a.
sin (b − a ) ∫  sin ( x − a ).sin ( x − b)
= 
For example:
cos ( x − a ).sin ( x − b) 
−1
−  dx
sin ( x − a ).sin ( x − b) 
∫ sin (ax + b) dx = a
cos (ax + b) + C
1  cos ( x − b) cos ( x − a ) 
11. cos 4x = 8 cos x – 8 cos x + 1
4 2
= ∫ −
sin (b − a )  sin ( x − b) sin ( x − a ) 
dx

184 Mathematics–12
E:\AMIT_WORKS\Exam_Guru\EG_Mathematics-12_(working_02-06-2022)\EG_Mathematics-12_working\Open_Files\Chap_7\Chap_7
\ 17-Aug-2022 Amit Proof-5 Reader’s Sign _______________________ Date __________

1 I=
∫ ( 4 cos )
2
[cot ( x − b) − cot ( x − a)] dx x − 3 dx
sin (b − a ) ∫
=

=
1
sin (b − a )
[log sin ( x − b) − log sin ( x − a)] + C = (
 1 + cos 2x
∫ 4 2 − 3 dx

)
1 sin ( x − b)
=
sin (b − a)
.log
sin ( x − a )
+C =∫ [ 2 cos 2x − 1] dx = sin 2x − x + C
sin ( x − b)  sin 3x 
I = cosec (b − a ).log
sin ( x − a )
+C Example 4. Evaluate: ∫  sin 3 x  dx . [HOTS]

Hence, the correct option is (c).  3 sin x − 4 sin 3 x 


 sin 3x 
Example 3. Evaluate: ∫
cos 3x
cos x (
dx . ) [HOTS]
Solution. I= ∫  sin 3 x  dx = ∫  sin 3 x  dx

( ) ∫ 3 cosec x − 4 dx = −3 cot x − 4x + C


2
cos 3x  4 cos3 x − 3 cos x  =
Solution. I = ∫ cos x dx = ∫  cos x  dx

EXERCISE 7.2
I. Multiple Choice Questions (MCQs) Evaluate each of the following Integrals
cos 4 x x II. Very Short Answer Type Questions
1. If ∫ 2
dx = A cot x + B sin 2 x + C + D, then
sin x 2
∫ (tan 3x) dx ∫ (sin 3x) dx
2 2
1. 2.
1
∫ (cos 5x) dx
2
∫ [tan (5x + 3)] dx
2
(a) A = –2 (b) B = 3. 4.
4
 (cos 2x)  dx
(c) C = 3 (d) A = –1, B = −
1
, C = –3
5. ∫  sin 2 x cos2 x  dx 6. ∫ (sin 2 x cos2 x)
4
tan x dx dx
cos 8 x + 1 7. ∫ sec x + tan x 8. ∫ 1 + cos 2x
2. ∫ tan 2 x − cot 2 x dx = a cos 8 x + C, then
 sin 2 x − cos 2 x 
(a) a = −
1
(b) a =
1 9. ∫  sin x cos x  dx  [A.I. 2017]
16 16
 sin 2x   sin 2x 
1 1 10. ∫  sin 3 x  dx 11. ∫  cos3 x  dx
(c) a = − (d) a =
8

3. Evaluate: ∫
sin 8 x − cos8 x
dx
18
12. ∫( ) sin 2x
sin 4x
dx 13. ∫ ( cos 2x
cos x )
dx

∫( )
1 − 2 sin 2 x cos 2 x cos 2x  cos 2x 
1
14. sin x
dx 15. ∫  cos2 x  dx
(a) Sin 2x + C (b) − sin 2x + C

1
(c) sin 2x + C (d) –sin 2x + C
2
16.
 cos 2x 
∫  sin 2 x  dx 17. ∫ ( cos 2x
sin 4x )
dx

2  sin 2 x   sin 2 x 
cos 2 x + 2 sin 2 x 18. ∫  cos x  dx 19. ∫  sin 2x  dx
4. ∫ dx =
cos 2 x  cos 2 x   cos 2 x 
(a) 2 sec x + C (b) 2 tan x + C 20. ∫  sin x 
dx 21. ∫  sin 2x  dx
(c) tan x + C (d) None of these
III. Short Answer Type Questions-I
cos 2 x − 1
5. ∫ dx is equal to 1. ∫ (sin
2
x cos 2 x) dx 2. ∫ (sin
3
x cos3 x) dx
cos 2 x + 1
(a) tan x – x + C (b) x + tan x + C 1 + sin x
(c) x – tan x + C (d) – x – cot x + C
3. ∫ 1 − sin x
dx 4. ∫ 1 + cos 2x dx

6. ∫ (sin
−1
x + cos −1 x) dx = 5. ∫ 1 + sin 2x dx
1
(a)
2
πx + C (b) x(sin–1 x – cos–1 x) + C 6. ∫ 1 + 2 tan x (sec x + tan x) dx

π  cos 2x − cos 2α   cos 2x − cos 2α 


(c)
2
+ x + C (d) None of these 7. ∫  cos x + cos α 
dx 8. ∫  sin x + sin α 
dx

Integrals 185
E:\AMIT_WORKS\Exam_Guru\EG_Mathematics-12_(working_02-06-2022)\EG_Mathematics-12_working\Open_Files\Chap_7\Chap_7
\ 17-Aug-2022 Amit Proof 5 Reader’s Sign _______________________ Date __________

 cos 2x − cos 2α   cos 2x + cos 2α  dx


9. ∫  dx 10. ∫  dx 9. ∫ sin ( x + a )sin ( x − b)  [HOTS]
sin (x + α)  cos (x − α) 
dx
cos 2x + cos 2α cos 2x + cos 2α 10. ∫ cos ( x − a ) cos ( x + b)  [HOTS]
11. ∫ cos x − sin α dx 12. ∫ sin x + cos α dx
dx
11. ∫ sin ( x + a ) cos ( x − b)  [HOTS]
13. ∫ [sin 3x cos 2x ] dx 14. ∫ sin 7x 1 − cos 4x  dx
dx
12. ∫ cos ( x − a )sin ( x + b)  [HOTS]
15. ∫( )cos 4x
cos 2x
dx  cos 3x  dx
16. ∫ 
 cos 2 x  13. ∫ sin
4
x cos 4 x dx  [HOTS]

∫( )sin 3x  cos 3x 
17.
sin x
dx 18. ∫  cos3 x  dx 14. ∫
dx 15. 1 − sin 2x dx

1 + sin 2x
1 − sin 2x
sin x dx dx
IV. Long Answer Type Questions 16. ∫ 17. ∫
1+ sin x cos 3x − cos x
∫ (sin x) dx 
4
1. [A.I. 2007 (C)] 18. ∫ ( tan 3x tan 2x tan x ) dx
∫ cos 3x  dx 
( )
4
2. [Delhi 2008 (C)] sin 5x + sin 3x + sin x
dx dx
19. ∫ cos 5x + cos 3x + cos x
dx
3. ∫ sin (x + a) sin (x + b) 4. ∫ sin (x + a) cos (x + b)
5. dx
∫ cos (x − a) cos (x − b)
20.  cos 4x 
∫  cos2 x  dx 21. ∫ ( )
sin 3x dx
sin 2x

6. ∫ dx
sin (x − a) cos x
 [Delhi 2000] 22. ∫ ( cos 3x dx
sin 2x ) 23.
 cos 4x 
∫  cos2 x  dx
dx  cos 4x 
7. ∫
sin (x − a) sin (x + b)
8. ∫ [sin 4x cos 3x sin 2x ] dx 24. ∫  sin 2 x  dx

Answers 7.2
cos 4 x (1 − sin 2 x) 2 1 + sin 4 x − 2 sin 2 x [(sin 2 x + cos 2 x) 2 − 2 sin 2 x cos 2 x] (sin 4 x − cos 4 x)
I. 1. (d) I = ∫ dx = ∫ sin 2 x dx = ∫ dx = ∫ dx
sin 2 x sin 2 x (sin 2 x + cos 2 x) 2 − 2 sin 2 x cos 2 x
= ∫ (sin 4 x − cos 4 x) dx
= ∫ cosec x dx + ∫ sin 2 x dx − 2 ∫ dx
2

= ∫ (sin x + cos x)(sin x − cos x) dx


2 2 2 2

 1 − cos 2 x 
= − cot x + ∫   dx − 2 x
 2 sin 2 x
x sin 2 x
= ∫ (sin 2 x − cos 2 x) dx = ∫ − cos 2 x dx = − 2
+C

= – cot x + − 2 x + D (constant) 4. (c) tan x + C
2 4
1 3 5. (c) x – tan x + C
= − cot x − sin 2 x − x + D 1
4 2 6. (a) πx + C
1 1 2
\ A = –1, B = − and C = –3. II. 1. 3 tan 3x − x + C;
4
Hint: Use: tan2 3x = sec2 3x – 1
cos 8 x + 1 2 cos 2 4 x
2. (b) I = ∫ tan 2 x − cot 2 x ∫ sin 2 x cos 2 x dx sin 6x
dx = 2. x − + C;
− 2 12
cos 2 x sin 2 x 1 − cos 6x
2
2 cos 4 x Hint: Use : sin 2 3x =
=∫ 2 ⋅ sin 2 x cos 2 x dx 2
sin 2 x − cos 2 2 x 3. x + sin 10x + C;
2 20
cos 2 4 x
= ∫ − (cos 2
2 x − sin 2 2 x)
⋅ ( 2 sin 2 x cos 2 x) dx Hint : Use: cos 2 5x =
1 + cos 10x
tan (5x + 3) 2
cos 2 4 x 1 4. − x + C;
= ∫ − cos 4 x .sin 4 x dx = − 2 ∫ 2 cos 4 x sin 4 x dx 5
Hint: Use: tan2 (5x + 3) = sec2 (5x + 3) – 1
1 1  cos 8 x  1 5. – (tan x + cot x) + C;
2∫
=− sin 8 x dx = −  −  + C = cos 8 x + C
2 8  16 cos 2x = cos 2 x − sin 2 x
Hint: = cosec2 x – sec2 x
sin 2 x cos 2 x sin 2 x cos 2 x
4 4 4 4
(sin x + cos x)(sin x − cos x)
3. (b) I = ∫ (sin 2
x + cos 2 x) 2 − 2 sin 2 x cos 2 x
dx
Now integrate.

186 Mathematics–12
E:\AMIT_WORKS\Exam_Guru\EG_Mathematics-12_(working_02-06-2022)\EG_Mathematics-12_working\Open_Files\Chap_7\Chap_7
\ 17-Aug-2022 Amit Proof-5 Reader’s Sign _______________________ Date __________

6. (tan x – cot x) + C; − 3 cos 2x cos 6x


2. + +C;
Hint: 1 sin 2 x + cos 2 x = sec2 x + cosec2 x 64 192
2 2 =
sin x cos x sin 2 x cos 2 x
Hint: sin 3 x cos3 x = 1 sin 3 2x
OR Ans. –2 cot 2x + C 8
Hint : 2
1 = 4 = 4 cosec22x = 1 (3 sin 2x − sin 6x )
sin x cos 2 x sin 2 2x 32
3. log| sec x + tan x | + log| sec x | + C
Now integrate.
7. [sec x – tan x + x] + C; 1 + sin x = 1 + sin x = sec x + tan x
Hint:
tan x 1 − sin x cos x
Hint: = tan x (sec x − tan x ) Now integrate.
sec x + tan x
= sec x tan x – tan2 x 4. Ans. 2 sin x + C
= sec x tan x – sec2 x + 1 Hint: 1 + cos 2x = 2 cos 2 x = 2 cos x
Now integrate. Now integrate.
1 tan x + C;
8. 5. (sin x – cos x) + C;
2
1 1 1 2 Hint: 1 + sin 2x = sin 2 x + cos 2 x + 2 sin x cos x
Hint : = 2 = 2 sec x
1 + cos 2 x 2 cos x
=(sin x + cos x ) = sin x + cos x
2

Now integrate.
Now integrate.
9. –log | sin x cos x | + C.
10. – 2 cosec x + C 6. log | sec x + tan x | + log| sec x | + C
sin 2x = 2 sin x cos x = 2 cosec x cot x Hint: 1 + 2 tan x (sec x + tan x )
Hint:
sin 3 x sin 3 x
11. 2 sec x + C. = 1 + 2 sec x tan x + 2 tan 2 x

Hint:
sin 2x = 2 sin x cos x = 2 sec x tan x
cos3 x cos3 x
= (1 + tan x) + 2 sec x tan x + tan
2 2
x

12. 1 log | sec 2x + tan 2x | + C. = sec2 x + 2 sec x tan x + tan 2 x


4
Hint:
sin 2x = 2 sin x cos x = 1 (sec 2x ) = (sec x + tan x) 2 = sec x + tan x
sin 4x 4 sin x cos x cos 2x 2
Now integrate.
13. 2 sin x – log | sec x + tan x | + C
cos 2x = 2 cos 2 x − 1 = 2 cos − sec 7. 2[sin x – x cos a] + C
Hint: x x cos 2x − cos 2α
cos x cos x Hint:
14. 2 cos x + log | cosec x – cot x | + C. cos x + cos α
15. 2x – tan x + C (2 cos 2 x − 1) − (2 cos 2 α − 1)
=
16. –[2x + cot x] + C cos x + cos α

17. 1 log cosec 2x − cot 2x + C = 2(cos x – cos a)
4
18. log | sec x + tan x | – sin x + C 8. 2[cos x + x sin a] + C;
− 2 sin 2 x − sin 2 α 
Hint:
sin 2 x = 1 − cos 2 x = sec − cos
x x Hint: cos 2x − cos 2α =
cos x cos x sin x + sin α sin x + sin α
1 log sec x + C = –2[sin x – sin a]
19.
2 Now integrate.
sin 2 x = sin 2 x − 1 tan x 9. 2 cos(x – a) + C;
Hint:
sin 2x 2 sin x cos x 2
20. log | cosec x – cot x | + cos x + C Hint: cos 2x − cos 2α = −2 sin (x + α) sin (x − α)
sin (x + α) sin (x + α)
cos 2 x = 1 − sin 2 x = cosec − sin = –2sin (x – a)
Hint: x x
sin x sin x Now integrate.
21. 1 log sin x + C 10. 2 sin (x + α) + C;
2
Hint: cos 2x + cos 2α = 2 cos (x + α) cos (x − α)
2 2
Hint: cos x = cos x = 1 cot x cos (x − α) cos (x − α)
sin 2x 2 sin x cos x 2
1 sin 4x  = 2 cos (x + a)
III. 1.  x − +C
8 4  Now integrate.
11. 2[sin x + x cos a] + C
Hint: sin 2 x cos 2 x = 1 (sin 2 2x) = 1 [1 − cos 4x]
4 8 cos 2x + cos 2α
Hint:
Now integrate. cos x − sin α

Integrals 187
E:\AMIT_WORKS\Exam_Guru\EG_Mathematics-12_(working_02-06-2022)\EG_Mathematics-12_working\Open_Files\Chap_7\Chap_7
\ 19-Aug-2022 Amit Proof 5 Reader’s Sign _______________________ Date __________

=
(2 cos 2
) (
x − 1 + 1 − 2 sin 2 α ) =
1  sin {(x + a) − (x + b)} 
 sin (x + a) sin (x + b) 
cos x − sin α sin (a − b)  

=
(
2 cos 2 x − sin 2 α )  sin ( x + a ) cos ( x + b ) −  
 
cos x − sin α
= 1   cos ( x + a ) sin ( x + b )  
= 2(cos x + sin a) sin ( a − b )  sin ( x + a ) sin ( x + b ) 
 
Now integrate.  

12. 2[cos x + x cos a] + C 1
= cot ( x + b ) − cot ( x + a )
Hint: See solution of Q.No.11
sin ( a − b ) 

cos 5x cos x 
13. Ans. −  + + C; Now integrate.
 10 2 
 1   sin (x + a) 
1 4.   log  cos (x + b)  + C
Hint: sin 3x cos 2x =

2
[sin 5x + sin x ]  cos (a − b )   

14.
sin 5x sin 9x
− +C; Hint: 
1 
5 2 9 2  sin (x + a) cos (x + b) 

Hint: (sin 7x) 1 − cos 4x = 2 sin 7x sin 2x


1  cos {(x + a) − (x + b)} 
=
1 cos (a − b)  sin (x + a)cos (x + b) 
= [cos 5x − cos 9x ]
2
 cos ( x + a ) cos ( x + b ) + 
Now integrate.  
= 1  sin ( x + a ) sin ( x + b ) 
15. sin 2x − 1 log sec 2x + tan 2x + C cos ( a − b )  sin ( x + a ) cos ( x + b ) 
2  
 
cos 4x = 2 cos 2 2x − 1 = 2 cos 2 − sec 2
Hint:
x x
cos 2x cos 2x
 1  cos (x − b)
16. 4 sin x – 3 log | sec x + tan x | + C 5. 
sin (b − a )  log cos (x − a) + C
 
3
Hint: cos 23x = 4 cos x −2 3 cos x 1  sin {(x − a) − (x − b)} 
cos x cos x Hint:
 cos (x − a) cos (x − b) 
sin (b − a)  
= 4 cos x – 3 sec x
17. x + sin 2x + C. 1 log sin (x − a) + C
6.
3 cos a cos x
Hint: sin 3x = 3 sin x − 4 sin x

sin x sin x
4 (1 − cos 2x ) Hint : 
1 
= 3 – 4 sin­2­ x = 3 −  sin (x − a) cos x 
2  
= 3 – 2 + 2 cos 2x = 2 cos 2x + 1    cos {x − (x − a)} 
= 1   sin (x − a) cos x 
18. 4x – 3 tan x + C  cos a   

1 12x − 8 sin 2x + sin 4x + C
IV. 1.
32
[ ] sin (x − a)
1
7. log +C
Hint : sin 4 x = 1 [3 − 4 cos 2x + cos 4x ] sin (a + b) sin (x + b)
8
sin 5x sin 9x sin x sin 3x
Now integrate. 8. − + − +C
20 36 4 12
1 3x + 2 sin 6x + sin 12x  + C
2. Hint: sin 4x cos 3x sin 2x

8  3 12 
= 1 [cos 5x − cos 9x + cos x − cos 3x]
Hint: cos 4 3x = 1 [3 + 4 cos 6x + cos 12x ] 4
8
Now integrate. 1 sin ( x − b )
9. log +C
 1   sin (x + b)  sin ( a + b ) sin ( x + a )
3.   log  sin (x + a)  + C ;
 sin (a − b)   
 −1  cos ( x + b )
1 10.   log +C
Hint:

sin (x + a) sin (x + b)  (
sin a + b ) cos ( x − a)

188 Mathematics–12
E:\AMIT_WORKS\Exam_Guru\EG_Mathematics-12_(working_02-06-2022)\EG_Mathematics-12_working\Open_Files\Chap_7\Chap_7
\ 17-Aug-2022 Amit Proof-5 Reader’s Sign _______________________ Date __________

20. tan x + 2 sin 2x – 4x + C


1 sin ( x + a )
11. log +C
cos ( a + b ) cos ( x − b ) cos 4x = 2 cos 2 2x − 1
Hint:
cos 2 x cos 2 x
1  sin ( x + b ) 
12. log  
2
cos ( a + b )  cos ( x − a )  2  2 cos 2 x − 1 − 1
=
cos 2 x
13. 3x − sin 4x + sin 8x + C
128 128 1024 2  4 cos 4 x − 4 cos 2 x + 1 − 1
1 = 
Hint: sin4 x cos4 x = sin4 2x cos 2 x
16
4 2
= 1 [3 − 4 cos 4x + cos 8x ] = 8 cos x − 82cos x + 1
128 cos x

14.
−1
2

 4 ( ) ( )
log cosec π − x − cot π − x
4

 + C


= 8 cos2 x – 8 + sin2 x

= 8
1 + cos 2x  8
− + sec 2 x
1 1  2 
Hint: =
1 − sin 2x
( )
1 − cos π − 2x
2


= 4 + 4 cos 2x –8 + sec2 x
= sec2 x + 4 cos 2x –4
= 1 cosec ( π − x ) 1
21. − log sec x + tan x + 2 sin x + C
2 4 2
1 log 1 + sin 2x + C
15. sin 3x 3
2 Hint: = 3 sin x − 4 sin x
sin 2x 2 sin x cos x
1 − sin 2x = 1 − sin 2x
Hint:  2 
1 + sin 2x cos 2x = 3 sec x − 2  1 − cos x 
2  cos x 
= sec 2x – tan 2x

( )
x x π+x 3
16. 2 sin − 2 cos − 2 log cosec = sec x − 2 sec x + 2 cos x
2 2 4 2 2

sin x = (1 + sin x ) − 1 = − 1 sec x + 2 cos x


Hint: 2
1 + sin x 1 + sin x
1 log cosec x − cot x + 2 cos x + C
1 22.
= 1 + sin x − 2
1 + sin x
cos 3x 4 cos3 x − 3 cos x
Hint: =
= cos x + sin x − 1 sin 2x 2 sin x cos x
2 2
( )
1 − cos π + x
2

 2 
= 2  1 − sin x  − 3 cosec x
 sin x  2
= cos x + sin x − 1 cosec ( π + x ) 3
2 2 2 4 2 = 2 cosec x − 2 sin x − cosec x
2
17. − 1 log sec x + tan x − cosec x  + C
4 = 1 cosec x − 2 sin x
2
2 2
= sin x + cos x
Hint: 1
cos 3x − cos x −2 sin 2x sin x 23. 2 sin 2x + tan x – 4x + C
cos 4x 4 2
18.
1 log sec 3x − 1 log sec 2x
– log | sec x | + C Hint: = 8 cos x − 82cos x + 1
3 2 cos 2 x cos x
= 8 cos2 x –8 + sec2 x
Hint: tan 3x = tan(2x + x) = tan 2x + tan x
1 − tan 2x tan x = 4(1 + cos 2x) – 8 + sec2 x
⇒ tan 3x tan 2x tan x = tan 3x – tan 2x – tan x. = 4 cos 2x + sec2 x – 4
19. 1 log sec 3x + C 24. –cot x – 2 sin 2x – 4x + C
3
cos 4x 8 sin 4 x − 8 sin 2 x + 1
Hint: sin 5x + sin 3x + sin x Hint: 2
sin x
=
sin 2 x
cos 5x + cos 3x + cos x
(sin 5x + sin x ) + sin 3x = 4(1 – cos 2x) – 8 + cosec2 x
=
(cos 5x + cos x ) + cos 3x = –4 cos 2x + cosec2 x – 4

Integrals 189
E:\AMIT_WORKS\Exam_Guru\EG_Mathematics-12_(working_02-06-2022)\EG_Mathematics-12_working\Open_Files\Chap_7\Chap_7
\ 17-Aug-2022 Amit Proof 5 Reader’s Sign _______________________ Date __________

Topic 3. Integration by Substitution


x9 dx
Example 1. ∫ (4 x 2 + 1)6 dx is equal to Example 3. Evaluate: ∫x+ x

dx dx
1  1
−5
1 1
−5
Solution. I= ∫x+ x
=∫
(
x x +1 )
(a)  4 + 2  +C (b)  4 + 2  +C
5x  x 5 x
 1  dx
(c)
1
(1 + 4)− 5 + C (d)
1 1 
+ 4
−5
+C
⇒ I= ∫  x + 1 x
.

10 x 10  x 2 

Substitute: x =z dx
⇒ = 2dz
x 9 x9 x
Solution. Let I = ∫ (4 x 2 + 1)6 dx = ∫  1
6
dx
1  dx  dz
x12  4 + 2 
 x 
\ I= ∫( x +1

x) = 2∫ (z + 1)
1
= ∫  1
6
dx
x3  4 + 2  = 2log|z+1|+C⇒ I = 2 log x + 1 + C.
 x 

 1 −2 dx 1 Integration of Trigonometric Functions by


Put  4 + 2  = t  3 dx = dt  3 = − dt
 x  x x 2
Substitution
1 dt 1 1
\ I = − ∫ 6 = − × − t −5 + C
2 t
1 −5
2 5
1  1
−5
Example 4. Evaluate:
( ) ∫
sin 3x
cos x
dx
= t +C=  4 + 2  +C
∫( ) ∫
10 10  x sin 3x 3 sin x − 4 sin 3 x
Solution. I = dx = dx
cos x cos x
x3
Example 2. If ∫ 1 + x2
dx = a (1 + x 2 )3/ 2 + b 1 + x 2 + C, then
 sin 3 x 
⇒ I = 3∫ tan x dx − 4∫  dx
 cos x 
1 1
(a) a = , b = 1 (b) a = − , b = 1
3 3
⇒ I = 3 log sec x − 4∫
(1 − cos x) sin x dx
2

1 1 cos x
(c) a = − , b = − 1 (d) a = , b = − 1
3 3
⇒ I = 3 log | sec x | – 4I1
x3
Solution. Let I = ∫ 1 + x2
dx
 1 − cos 2 x 
dt
⇒ I1 = ∫  cos x  sin x dx
Put 1 + x2 = t  2x dx = dt  x dx =
2
1 (t − 1) Put cos x = z ⇒ sin x dx = – dz
2∫
\ I = dt
t
 1 − z2   z 2 − 1
I1 = ∫   ( −dz ) = ∫  dz

1 t
= ∫
1 1
dt − ∫ dt z   z 
2 t 2 t

=
1
2∫
1
t dt − ∫ t −1/ 2 dt
2
I1 = ∫ ( )z−
1
z
dz =
z2
2
− log z + C

1 2 3/ 2 1 cos 2 x
= × (t ) − 2 t + C ⇒ I1 = − log cos x + C
2 3 2 2
1 I = 3 log|sec x| – 4I1
= (1 + x 2 )3/ 2 − 1 + x 2 + C
3
But I = a (1 + x 2 )3/ 2 + b 1 + x 2 + C  cos 2 x 
⇒ I = 3 log sec x − 4  − log cos x  + C
1  2 
Comparing the like terms we get, a = and b = – 1
3
Hence, the correct option is (d).
⇒ I = 3 log |sec x| – 2 cos2 x + 4 log|cos x| + C

190 Mathematics–12
E:\AMIT_WORKS\Exam_Guru\EG_Mathematics-12_(working_02-06-2022)\EG_Mathematics-12_working\Open_Files\Chap_7\Chap_7
\ 17-Aug-2022 Amit Proof-5 Reader’s Sign _______________________ Date __________

EXERCISE 7.3
I. Multiple Choice Questions (MCQs) III. Short Answer Type Questions-I
10 x 9 + 10 x log e 10
(sin x)
−1 2
1. ∫ dx = dx
10 x + x10 1. ∫ 2. ∫ dx
sin x cos3 x 1 − x2
1 1
(a) − (b) log (10x + x10) + C
x
2 (10 + x10 ) 2
+C
 tan −1 x 
3. ∫  e
 sin −1 x 2 
4. ∫ 
( )
 dx
1 1 2  dx  2 x 1− x 
(c) +C (d) None of these [NCERT]  1+ x   
x
2 (10 + x10 ) 2 (3x + 4) dx
5. ∫ 6. ∫ x x + 2 dx
1 − cos 2 x 2x + 3
2. ∫ tan
−1
dx =
1 + cos 2 x  e 2x − 1
dx
(a) 2x + C
2
(b) x + C
2 7. ∫ x + x log x 8. ∫  e2x + 1 dx
x2  tan x   (x + 1) (x + log x) 2 
(c) 2x + C (d) +C
2 9. ∫  sin x cos x  dx 10. ∫  x  dx

e x (1 + x )
3. ∫ cos dx is equal to 10x9 + 10 x log 10   (x + 1) e x
( e x x) 11. ∫   dx 12. ∫  2 x  dx
2

(x10 + 10 x)   cos (xe ) 


(a) –cot (ex x) + C (b) tan (xex) + C
 xe − 1 + e x − 1 
∫ (sin )
3
(c) tan (ex) + C (d) cot (ex) + C 13. x cos x dx 14. ∫  e x  dx
sin x  x +e 
4. The value of
2∫
 π
sin  x − 
 4
dx is
15. ∫ ( cos 3x
sin x
dx) 16. ∫  cos23x  dx
 sin x 
 sin 3x   sin 3x 
 π
(a) x − log cos  x −  + C
17. ∫  cos2 x  dx 18. ∫  cos3 x  dx
 4
 sin 3x   sin 4x 
 π
19. ∫  cos4 x  dx 20. ∫  cos2 x  dx
(b) x + log cos  x −  + C
 4  sin 4x   sin 4x 
π
21. ∫  cos3 x  dx 22. ∫  sin 4 x  dx

(c) x − log sin  x −  + C
 4  sin 4x   sin 4x 
23. ∫  sin 4 x  dx 24. ∫  cos4 x  dx
 π
(d) x + log sin  x −  + C
 4 IV. Short Answer Type Questions-II
log ( x + 1 ) − log x  sin x dx
5. ∫ x ( x + 1)
dx is equal to 1. ∫  x 
dx 2. ∫ sin x cos3 x
{
 x + cos −1 3x 2 
}
2
1   x + 1   x sin −1 x 2  ( ) 
(a) −  log   +C 3. ∫   dx 4. ∫  dx
2   x    1 − x 4   1 − 9 x 2 
 
(b) C – [{log (x + 1)}2 – (log x)2]

  x + 1 
5. ∫ ( cos 4x
cos x )
dx [HOTS] 6. ∫ (
cos 4x
sin x )
dx  [HOTS]
(c) − log  log  +C
 x    cos 4x   cos 4x 
 7. ∫  cos2 x  dx [HOTS] 8. ∫  cos4 x  dx  [HOTS]
x + 1
(d) − log  +C
 x   cos 4x 
9. ∫  sin 4 x  dx  [HOTS]
Evaluate each of the following Integrals
V. Long Answer Type Questions
II. Very Short Answer Type Questions
 1 
   1 + sin 2x  dx x2 
1. ∫  1+ cot x  dx 2. ∫  
1. Evaluate: ∫ 1 1 dx
 x + log ( sin x )   x − cos 2 x   2 
 x − x3 
1
 ( cos 2x )  x2
3. ∫  2  dx 4. ∫ sec
4
x tan x dx 2. Evaluate: ∫ 3 dx
 (sin x + cos x )  x4 +1

Integrals 191
E:\AMIT_WORKS\Exam_Guru\EG_Mathematics-12_(working_02-06-2022)\EG_Mathematics-12_working\Open_Files\Chap_7\Chap_7
\ 17-Aug-2022 Amit Proof 5 Reader’s Sign _______________________ Date __________

Answers 7.3
( )
2. 1 sin −1 x + C. Hint: Put sin–1 x = q.
9
10 x + 10 log e 10 x 3
I. 1. (b) I= ∫
10 x + x10
dx 3
Put 10x + x10 = t, then (10x loge 10 + 10x9) dx = dt −1
3. e tan x + C . Hint: Put tan–1 x = q.
dt
( )
4. 1 sin −1 x + C . Hint: Put sin −1 x = θ .
3
\ I = ∫ = log | t | + C = log | 10 x + x10 | + C
t 3
1 − cos 2 x 2 sin 2 x  3 
2. (d) I = ∫ tan −1 dx = ∫ tan −1 dx 5. 1 (2x + 3) 2 − 2x + 3] + C
1 + cos 2 x 2 cos 2 x 2 
x2
= ∫ tan −1 tan x dx = ∫ x dx = +C  x  + 4 1 
2 Hint: 3x + 4 = 3 
2x + 3  2x + 3   
2x + 3 
e x (1 + x )
3. (b) I= ∫ cos2
( e x x)
dx
3  ( 2x + 3) − 3  
− 4 1 
=
Put exx = t, then {(ex)x + ex (1)} dx = dt 2  2x + 3   2x + 3 

ex (x + 1)dx = dt
⇒ = 3 2x + 3 − 9
+ 4
dt 2 2 2x + 3 2x + 3
I= ∫
cos 2 t ∫
\ = sec 2 t dt = tant + C = tan (xex) + C
3 x+ − 1
= 2 3
π 2 2 2x + 3
4. (d) Put x − = t, so that dx = dt
4 Now integrate.
π 
sin  + t  dt 5 3
4  6. 2 (x + 2) 2 − 4 (x + 2) 2 + C
\ I = 2∫ 5 3
sin t
π π Hint: substitute: x + 2 = z
sin cos t + cos sin t 7. log |1 + log x| + C; Hint: Substitute: log x = z
= 2∫ 4 4 dt
sin t 8. log | ex + e–x | + C
 1 1  Hint: Divide Nr and Dr by ex and substitute:
= 2∫  cot t + dt = ∫ (cot t + 1) dt
 2 2  ex + e–x = z
= log |sin t| + t + C 9. 2 tan x + C .
 π  π Hint: Divide Nr and Dr by cos2x.
= log sin  x −  +  x −  + C 1 (x + log x)3 + C;
 4   4 10.
3
 π
= x + log sin  x −  + C Hint: Substitute: (x + log x) = z
 4
11. log | (x)10 + (10)x | + C;
log ( x + 1 ) − log x
5. (a) I= ∫ x ( x + 1)
dx Hint: Substitute: (x10 + 10x) = z
12. tan (xex) + C; Hint: Substitute: xex = z
 1 1 −2 3
2
7
Put log (x + 1) – log x = t, then  − dx = dt 13. cos 2 x + cos 2 x + C;
 x + 1 x  3 7
dx dx Hint: Write:
⇒ − = dt ⇒ = – dt
x ( x + 1) x ( x + 1) sin 3 x cos x = (1 − cos 2 x) cos x sin x
1 and substitute: cos x = z
\ I = ∫ t ( − dt ) = − t 2 + C
2 14. 1 log x e + e x + C; Hint: Substitute: xe + ex = z
2 e
1   x + 1 
= − log    +C 15. [log | sin x | – 2 sin2 x] + C.
2   x 
cos 3x 3
II. 1. log | x + log (sin x) | Hint: = 4 cos x − 3 cos x
sin x sin x
2. log | x – cos2 x |
 1 − 4 sin 2 x 
3. log | sin x + cos x | + C =   cos x
sin x 
1 Put sin x = z
4. sec4 x + C
4 16. –[cosec x + 4 sin x] + C
III. 1. log | tan x | + 1 tan2 x + C 17. –[sec x + 4 cos x] + C
2
1 sec4 x ( )
1 + tan 2 x sec 2 x
Hint:
sin 3x
=
3 sin x − 4 sin 3 x
Hint: 3 = tan x = 2
cos x cos 2 x
sin x cos x tan x

192 Mathematics–12
E:\AMIT_WORKS\Exam_Guru\EG_Mathematics-12_(working_02-06-2022)\EG_Mathematics-12_working\Open_Files\Chap_7\Chap_7
\ 17-Aug-2022 Amit Proof-5 Reader’s Sign _______________________ Date __________

=
(4 cos 2
x −1 ) sin x 5.  − 8 sin 3 x + log
 3
sec x + tan x  + C

2
cos x
Put cos x = z cos 4x 8 cos x − 8 cos 2 x + 1
4
Hint: =
cos x cos x
18.  − 1 sec2 x − 4 log cos x  + C = 8(cos2 x – 1)cos x + sec x
 2 
= –8 sin2 x cos x + sec x
19.  − 1 sec3 x + 4 sec x  + C
 3  6.  8 cos3 x + log cosec x − cot x  + C
 3 
20. [4 log |cos x| – 4 cos2 x] + C
cos 4x 8 sin 4 x − 8 sin 2 x + 1
sin 4x  4 sin x cos x cos 2x  Hint: =
Hint: =  sin x sin x
cos 2 x  cos 2 x 
= 8 sin3 x –8 sin x + cosec x
 2 cos x − 1 2
= 8(sin2 x – 1) sin x + cosec x
= 4  sin x
 cos x  = –8 cos2 x sin x + cosec x
Put cos x = z. 7. [–4x + 2 sin 2x + tan x] + C
21. [8 cos x + 4 sec x] + C.
cos 4x 8 cos 4 x − 8 cos 2 x + 1
22. –[4 cosec x + 8 sin x] + C Hint: 2 =
cos x cos 2 x
sin 4x 4 cos 2x cos x
Hint: = = 8 cos2 x – 8 + sec2 x
sin 3 x sin 2 x
2 
= –8 sin2 x + sec2 x
 x cos x
= 4 1 − 2 sin  1 − cos 2x  + sec2 x
2
 sin x  = −8 
 2 
Put sin x = z.
23. –[ 2 cosec2 x + 8 log |sin x | ] + C 8. 8x − 7 tan x + 1 tan 3 x  + C
 3 
24. [–2 sec2 x – 8 log | cos x |] + C
 cos 2x sin x  cos 4x 8 cos 4 x − 8 cos 2 x + 1
sin 4x Hint: =
Hint: = 4 4
cos x cos 4 x
cos 4 x  cos3 x 
2 = 8 – 8 sec2 x + sec4 x
 
= 4  2 cos 3x − 1 sin x = 8 – 7 sec2 x – sec2 x + sec4 x
 cos x 
Put cos x = z. = 8 – 7 sec2 x + sec2 x (–1 + sec2 x)
IV. 1. − 2 cos x + C; = 8 – 7 sec2 x + sec2 x tan2 x
Hint: Substitute: x=z 9. 8x − 1 cot 3 x + 7 cot x  + C
2  3 
tan x + log | tan x | + C
2. ;
2 cos 4x 8 sin 4 x − 8 sin 2 x + 1
Hint: Divide Nr and Dr of 1 4 Hint: =
3 by cos x sin 4 x
4
sin x
It gives: sin x cos x
= 8 – 8 cosec2 x + cosec4 x
2 2
1 sec4 x (1 + tan x) sec x = 8 + (cot2 x – 7)cosec2 x
3 = tan x = tan x
sin x cos x Put cot x = z.
Then, substitute tan x = z 5 2 1 1 1 1
V. 1. x + 6 x 6 + 3 x 3 + 2x 2 + 3x 3 + 6x 6 + 6 log x 6 − 1 + C;
3. 1 (sin − 1 x 2) 2 + C; 5 2
4 Hint: LCM of 2, 2, 3 is 6. ∴ Substitute x = z6.
Hint: Substitute: sin–1 x2 = z

(
4. −2 1 − 9x 2 + 1 cos −1 3x + C
3
) 3 (
2. 4  x3/ 4 − log x3/ 4 + 1  .
 )
9 9

Topic 4. Integration Using Standard Formulae


sin x dt 1 dt
Example 1. ∫ 3 + 4 cos2 x dx = . \ I = − ∫
3 + 4t 2
= − ∫3
4 + t2
sin x 4
1 dt
Solution. Let I = ∫ 3 + 4 cos2 x dx      =− ∫
4  3 2
Put cos x = t   + t
2

\ – sin x dx = dt  sin x dx = – dt 2

Integrals 193
E:\AMIT_WORKS\Exam_Guru\EG_Mathematics-12_(working_02-06-2022)\EG_Mathematics-12_working\Open_Files\Chap_7\Chap_7
\ 17-Aug-2022 Amit Proof 5 Reader’s Sign _______________________ Date __________

1 1  t 
= − × tan − 1  +C  1  1 a+ x
4 3 2  3 2  (v) ∫  a 2 − x 2  dx = 2a log a−x
+C
1  2t 
= − tan − 1   + C
2 3  3
dx
= −
1 −1 2 cos x 
Example 6. Evaluate: ∫ 9 − x2
tan   +C
2 3 3 
Hence, dx dx
1 2
Solution. ∫ = ∫ (3)2 − (x)2
I= − tan −1 
cos x + C . 9 − x2
 
2 3 3

f ′ ( x ) dx 1 3+ x 1 3+ x
= log + C = log +C
(i) ∫ = log f ( x ) + C 2×3 3− x 6 3− x
f ( x)

Example 2. Evaluate:
 3 + cos x 
∫  3x + sin x  dx
(vi)
dx
∫ a2 + x2 =
1
a
tan −1
x
a
+C ()
(3 + cos x) dx (3x + sin x )′ dx
Solution. I= ∫
3x + sin x
=∫
(3x + sin x) dx
Example 7. Evaluate: ∫ 9 + x2
⇒ I = log| 3x + sin x | + C

(ii) ∫
f ′ ( x ) dx
f ( x)
= 2 f ( x) + C Solution. I=
dx dx
∫ 9 + x 2 = ∫ (3)2 + x 2 = 3 tan
1 −1
()
x
3
+C

dx
(14x + e ) dx x (vii) ∫ 2
x +a 2
= log x + x2 + a2 + C
Example 3. Evaluate: ∫
7x 2 + e x

Solution.
(14x + e ) dx = (7x x 2
)′
+ e x dx
Example 8. Evaluate: ∫
dx
I=
∫ 7x + e 2 x ∫ 2
7x + e x x2 + 9

⇒ I = 2 7x 2 + e x + C dx
Solution. I= ∫ 2
x +9
= log x + x2 + 9 + C
n +1
 f ( x )
f ′ ( x ) dx = 
n
(iii) ∫  f ( x ) n +1
+ C , n ≠ −1
(viii) ∫
dx
= log x + x2 − a2 + C
x − a2
2

∫ (x ) ( x ) dx
3 5 2
Example 4. Evaluate: +4
dx
1 Example 9. Evaluate: ∫
∫ (x + 4) (x ) dx = 3∫
3 5 2
Solution. I= (x3 + 4)5 (3x 2) dx x2 − 9

dx
1
( ) ( x + 4)′ dx = 181 ( x + 4) + C
5 6
⇒ I=
3∫
x3 + 4 3 3 Solution. I= ∫ 2
x −9
= log x + x2 − 9 + C

(iv)
 1 
∫  x 2 − a 2  dx = 2a log
1 x−a
x+a
+C (ix) ∫
dx
2
a −x 2
= sin −1 () x
a
+C

 1 
Example 5. Evaluate: ∫  x 2 − 5  dx Example 10. Evaluate: ∫
dx
9 − x2
 
 1  1
Solution. I = ∫ 2 dx = ∫  
2 dx
dx dx

 x − 5 x − 5
2
( )  Solution. I= ∫ 9− x 2
=∫
(3) 2 − x 2
 

()
1 x− 5
= log +C x
2 5 x+ 5 =sin −1 +C
3

194 Mathematics–12
E:\AMIT_WORKS\Exam_Guru\EG_Mathematics-12_(working_02-06-2022)\EG_Mathematics-12_working\Open_Files\Chap_7\Chap_7
\ 17-Aug-2022 Amit Proof-5 Reader’s Sign _______________________ Date __________

EXERCISE 7.4
I. Multiple Choice Questions (MCQs) 1  1 
dx (c) tan −1  cot 2 x + C
1. ∫ 2 equals 2  2 
x + 2x + 2
(d) None of these
(a) x tan–1 (x + 1) + C (b) tan–1 (x + 1) + C
(c) (x + 1)tan–1 x + C (d) tan–1 x + C [NCERT]
dx Evaluate each of the following Integrals
2. ∫ equals
9x − 4x2 II. Very Short Answer Type Questions
1  9x − 8  1 − 1  8x − 9  dx (x 4 + 5)dx
(a) sin −1   + C (b) sin   +C
9  8 2 9 
1. ∫ x (x − 9)
2. ∫ x2 − 5

(c)
1  9x − 8 1  9x − 8  sec 2 x e x dx
sin − 1  + C (d) sin −1   +C
3  8  2  9  3. ∫ 7 − 3 tan 2 x dx 4. ∫ 9 − 16e2x
 [NCERT]
x x dx sec x tan x dx
3. ∫ 2
sin x
dx equals
5. ∫ 7x5 − 9 6. ∫ 8 + 3 sec 2 x
a + b 2 cos 2 x
dx 3x dx
(a) log (a + b cos x) + C (b)
2 2 21  a cos x 
tan −1  +C
7. ∫ e x + e−x 8.
∫ 5 + 32x
ab  b 
III. Short Answer Type Questions-I
1  b cos x  1  a cos x
(c) cot −1   + C (d) cot −1   + C
ab  a  ab  b x3 + x dx
1. ∫ x4 − 4
dx 2. ∫ x. (3x + 4)
sin x
4. ∫ 3 + 4 cos x
dx equals
2
7 x dx dx
(a) log (3 + 4 cos2 x) + C
3. ∫ 3 + 2 × (49) x 4. ∫3 x + 5x5 / 2
−1  cos x  dx
(b)
2 3
tan −1 
 3 
+C 5. ∫ x 2 + 4x + 8  [Delhi 2017]

−1  2 cos x  (5 sin 2x − cos x ) dx


(c)
2 3
tan −1 
 3 
 +C
6. ∫ 9 sin 2 x + 4
1  2 cos x  x dx
(d) tan −1 
  +C
7. ∫ 5x3 − 3
2 3 3 
(sin 2x) dx dx
dx 8. ∫ 9 − 16 cos4 x 9. ∫
5. ∫ sin 4 x + cos4 x is equal to [CBSE 2014] x 5x + 7
x
(a)
1  1
tan −1 

tan 2 x + C
10. ∫ a − x3 3
dx
2  2 

(b)
 1  11. (x 3
+ x 2 + 1 dx )
2 tan −1 
 2
sin 2 x + C
 ∫ x −12

Answers 7.4
dx dx dx dx
I. 1. (b) I= ∫x 2
+ 2x + 2
=∫ 2
( x + 2 x + 1) + 1
= ∫ ( x + 1) 2
+1
= ∫ 81  2 81 
−  4 x − 9 x + 
Put x + 1 = t, then dx = dt 16 16
dx
dt 1 t = ∫ 2 2
\ I = ∫ 2 2 = tan − 1   + C = tan −1 (t ) + C  9  9
t +1 1  1   −  2 x − 
4 4
9 1
Hence I = tan–1 (x + 1) + C Put 2x − = t, then 2dx = dt ⇒ dx = dt
4 1 dt 2
dx dx I= ∫
2. (b) I= ∫ =∫ 2  9
2
2
  − t
2 2
9x − 4x − ( 4x − 9x )
4

Integrals 195
E:\AMIT_WORKS\Exam_Guru\EG_Mathematics-12_(working_02-06-2022)\EG_Mathematics-12_working\Open_Files\Chap_7\Chap_7
\ 17-Aug-2022 Amit Proof 5 Reader’s Sign _______________________ Date __________

  x −3
  Form ∫
dx
= sin − 1
x
+ C II. 1. 1 log + C;
 2
a −x 2 a  3 x +3
Hint: Put x=z
 
1 −1  t  1  4t  3
\ I = sin  + C = sin − 1   + C 2. x + 5x + 3 5 log x − 5 + C;
2 9  2  9  3 x+ 5
 
4  4 4
 9 Hint: x + 5 = (x − 25) + 30
4 2x −  x2 − 5 x2 − 5
1 − 1  4 2 30
= sin +C = x + 5 + 2
2 9 x −5
1  8x − 9  1 log 7 + 3 tan x + C
= sin − 1  +C 3.
2  9  2 21 7 − 3 tan x
sin x 1 log 3 − 4e x + C
3. (c) I= ∫ 2 dx 4.
a + b 2 cos 2 x 24 3 + 4e x
1
Put b cos x = t, then – b sin x dx = dt ⇒ sin x dx = − dt 1 7 x5/ 2 − 3 + C
b 5. log
1 15 7 7 x5/ 2 + 3

1 − dt
I = ∫ 2 b 2 dt = ∫ 2 2 x x dx x3/ 2 dx
a +t b a +t Hint: =
7x5 − 9 7 ( x )5 − 9
1 t 1  b cos x 
= cot −1   = cot −1
  +C Now put x5/2 = z.
ab  a  ab a 
1  3 x
sin x 6. tan −1  sec  + C
4. (c) I = ∫ 3 + 4 cos 2
x
dx 24  8 

Put cos x = t, so that sin x dx = –dt Hint: Put sec x = z.


− dt −1 dt 7. tan–1(ex) + C
\ I = ∫
4 ∫
= 2
3 + 4t 2  3 1 ex
2
t +  x −x =
 2  e +e 1 + e2x
  Now put ex = z.
−1  t  1 2t
= tan −1   +C = − tan −1 +C  −1  3x  
3 3 2 3 3 8. 1
4×   tan    + C
2  2  5 (log 3)  5 

−1  2 cos x  Hint: Put 3x = z.


= tan −1   +C
2 3  3  2
III. 1. 1 log x 4 − 4 + 1 log x − 2 + C;
4 8 2
dx x +2
5. (a) I = ∫ sin 4
x + cos 4 x
2. 1 tan −1  3x 
dx  2  + C
= ∫ 3
(sin 2 x + cos 2 x) 2 − 2 sin 2 x cos 2 x
Hint: Put x = z.
2dx
= ∫ 2 − (2 sinx cos x)2 1  2 × 7x 
3. tan −1   +C
6 ( log 7 )  3 
2 2 sec 2 2 x
⇒ I= ∫ 2 − sin 2
2x
dx = ∫
2 sec 2 2 x − tan 2 2 x
dx
4. 2 tan −1  5 x 
  +C
2 sec 2 x 2 2
2 sec 2 x 15 3
= ∫ 2 + 2 tan dx = ∫ 2 + tan dx

( )
2
2 x − tan 2 2 x 2
2x
1 tan −1 x + 2 + C
5.
Put tan 2x = 3t, so that 2 sec2 2x dx = dt 2 2

( )
dt dt 5 log sin 2 x + − 1 tan −1 3 sin x + C
⇒ I = ∫
2 + t2 ∫ 2 2 + t2
= 6. 9 4
( ) 9 6 2
1  t  1  1  Hint:
5 sin 2x − cos x = 5 sin 2x − cos x
= tan −1   + C = tan −1  tan 2 x + C
2  2 2  2  9 sin 2 x + 4 9 sin 2 x + 4 9 sin 2 x + 4

196 Mathematics–12
E:\AMIT_WORKS\Exam_Guru\EG_Mathematics-12_(working_02-06-2022)\EG_Mathematics-12_working\Open_Files\Chap_7\Chap_7
\ 17-Aug-2022 Amit Proof-5 Reader’s Sign _______________________ Date __________

3
9. 2 log 5x + 5x + 7 + C
1 log 5x 2 − 3 + C 5
7. 3
3 15
5x 2 + 3 Hint: Put x = z.

()
Hint: Put x3/2 = z. 3/ 2
2 sin −1 x
10. +C
2 3 a
8. 1 log 3 + 4 cos x + C
24 3 − 4 cos 2 x Hint: Put x3/2 = z.
x2 x 1 x
Hint: Substitute cos2 x = z. 11. + + log x 2 − 1 + log − 1 + C
2 2 x +1

Topic 5. Directly using Formulae & Special Types of Integrals


( ) 1  x + 1
 2
 2 
How to change ax2 + bx + c into a  x + b −  b − 42ac   ? = tan −1 
 2 
+C
 2a  4a   2

We have
()
ax2 + bx + c = a  x 2 + b x + c 
 a a 
Clearly, A is true and R is the correct explanation for A.
Hence, the correct option is (a)

( )  2
b2 c  dx
= a x+
b
− 2+  ∫ Example 1. Evaluate:
x 2 + 4x + 1
 2a 4a a
dx dx
∫ ∫ Solution. I= =
( ) ( 3)
 2
 b 2 − 4ac   x 2 + 4x + 1 2
= a x+ b
2
−  (x + 2) −
 2a  4a 2  
b 1 (x + 2) − 3
= a [(x + B)2 – A], where B = ⇒ I= log +C
2a 2 3 (x + 2) + 3
 2 
and A =  b − 42ac  dx 1 A + (x ± B)
 4a  (ii) ∫ A 2 − (x ± B)2 = 2A
log
A − (x ± B)
+C

Use the following formulae directly


dx
dx 1 (x ± B) − A
Example 2. Evaluate: ∫ 4 − 2x −x 2
(i) ∫ ( x ± B)2 − A 2 = 2a log (x ± B) + A
+C
dx dx
Solution. I= ∫ 4 − 2x − x 2 = ∫ − [x 2 + 2x − 4]
Assertion-Reasoning Type Questions
dx dx
⇒ I= ∫ − [(x + 1)2 − 5] = ∫
Direction: In the following questions, a statement of
( 5)
2
− (x + 1) 2
Assertion (A) is followed by a statement of Reason (R).
1 5 + (x + 1)
Mark the correct choice as: = log +C
2 5 5 − (x + 1)
(a) Both A and R are true and R is the correct explanation
of A. dx 1  x ± B + C
∫ (x ± B)2 + A 2 = A tan
−1
(iii)  
(b) Both A and R are true but R is NOT the correct explanation A 
of A.
dx
(c) A is true but R is false. Example 3. Evaluate: ∫ x 2 − 4x + 10
(d) A is false and R is True.
dx dx
1. Assertion (A): ∫ 2
dx
=
1  x + 1
tan −1  +C
Solution. I= ∫ x 2 − 4x + 10 = ∫ (x − 2)2 + 6
x + 2x + 3 2  2 
dx 1  x 1 tan −1  x − 2  + C
Reason (R): ∫ 2 = tan −1   + C ⇒ I=  
x +a 2 a  a 6 6 
dx x 1 −1 
Sol. ∫ x 2 + a 2 = a tan
  + C
a (iv) ∫
dx
2 2
= log (x ± B) + (x ± B) 2 − A 2 + C
(x ± B) − A
This is a standard integral and hence R is true.
dx dx dx
∫ x2 + 2 x + 3 = ∫ Example 4. Evaluate: ∫
( x + 1) + ( 2 )
2 2 2
x − 6x + 1

Integrals 197
E:\AMIT_WORKS\Exam_Guru\EG_Mathematics-12_(working_02-06-2022)\EG_Mathematics-12_working\Open_Files\Chap_7\Chap_7
\ 17-Aug-2022 Amit Proof 5 Reader’s Sign _______________________ Date __________

dx dx ⇒ A = 7 ; Β = − 12
Solution. I= ∫ 2
=∫
2
⇒ 7x + 2 = A(2x + 4) +B
2
x − 6x + 1 (x − 3) − 8
 7 (2x + 4) − 12 
=log (x − 3) + (x − 3) 2 − 8 + C \ I = ∫  2 2  dx
x + 4x + 8 
 
⇒ I = log (x − 3) + x 2 − 6x + 1 + C
7 (2x + 4) dx dx
I = ∫ 2 − 12 ∫ 2
2 x + 4x + 8 x + 4x + 8
dx
(v) ∫ (x ± B) + A 2 2
= log (x ± B) + (x ± B) 2 + A 2 + C

7
= log x 2 + 4x + 8 − 6 tan − 1
2
x+2
2
+C ( )
dx
Example 5. Evaluate: ∫ (Linear polynomial in x) dx
2
x − 8x + 20 Type II: ∫ Quadratic polynomial in x
dx dx
Solution. I= ∫ =∫
2
x − 8x + 20 (x − 4) 2 + 4 ( px + q )
How to evaluate ∫ dx ?
⇒ I = log (x − 4) + (x − 4) 2 + 4 + C ax 2 + bx + c
( px + q )
I = log (x − 4) + x 2 − 8x + 20 + C We have I= ∫ dx
ax 2 + bx + c
Split px + q as given below
dx x ± B
(vi) ∫ = sin −1   +C px + q = A(ax2 + bx + c)′ + B
2
A − (x ± B) 2 A 
⇒ px + q = A(2ax + b) + B
dx Solve for A and B
Example 6. Evaluate: ∫ 5 + 4x − x 2 A ( 2ax + b ) B
\ I= ∫ dx + ∫ dx + C
2 2
dx dx ax + bx + c ax + bx + c
Solution. I= ∫ 5 + 4x − x 2
=∫
2
− [x − 4x − 5] (5x + 3) dx
dx x − 2
Example 8. Evaluate: ∫
⇒ I= ∫ = sin −1   +C x 2 + 8x + 20
9 − (x − 2) 2  3 
(5x + 3) dx
Solution. I= ∫ x 2 + 8x + 20
Some Special types of Integrals
Take: 5x + 3 = A(x2 + 8x + 20)′ + B
(Linear polynomial in x) dx
Type I. ∫ ⇒ 5x + 3 = A(2x + 8) + B ⇒ A = 5 ; B = −17
Quadratic polynomial in x
2
 5 (2x + 8) − 17 
 px + q 
How to evaluate ∫  ax 2 + bx + c  dx ? 
\ I= ∫2
2
 dx

 x + 8x + 20 
 px + q 
We have I= ∫  ax 2 + bx + c  dx 5
 (2x + 8)  dx
= ∫  dx –17 ∫ 2
Split px + q as given below 2  x + 8x + 20 
2 x + 8x + 20
 
px + q = A(ax2 + bx + c)′ + B
⇒ px + q = A(2ax + b) + B (x 2 + 8x + 20)′ dx dx
I= 5∫ − 17 ∫
Now find the values of A and B 2 2
x + 8x + 20 (x + 4) 2 + 4
 A 2ax + b 
( ) B f ′ ( x)
\ I= ∫  ( ax 2 + bx + c )  dx + ∫ ax 2 + bx + c dx Q∫ dx = 2 f ( x )
  f ( x)
(7x + 2) dx I = 5 × 2 x 2 + 8x + 20
Example 7. Evaluate: ∫ x 2 + 4x + 8 ⇒
2
 – 17 log (x + 4) + (x + 4) 2 + 4 + C
(7x + 2) dx
Solution. I= ∫ x 2 + 4x + 8
Take: ⇒ I = 5 x 2 + 8x + 20
7x + 2 = A(x2 + 4x + 8)′ + B  –17 log (x + 4) + x 2 + 8x + 20 + C

198 Mathematics–12
E:\AMIT_WORKS\Exam_Guru\EG_Mathematics-12_(working_02-06-2022)\EG_Mathematics-12_working\Open_Files\Chap_7\Chap_7
\ 17-Aug-2022 Amit Proof-5 Reader’s Sign _______________________ Date __________

Integration of Trigonometric Functions Reducible 1 1 + 2z


⇒ I= log +C
px + q 2×2 1 − 2z
 
to the form: ∫  ax 2 + bx + c  dx ⇒ I=
1
log
1 + 2 cos x
+C
4 1 − 2 cos x
 sin 2 x 
Example 9. Evaluate: ∫  sin 3x  dx Example 10. Evaluate: ∫
sin x
dx
cos 2x
 sin 2 x  sin 2 x dx sin x sin x
Solution. I= ∫  sin 3x  dx = ∫ 3 sin x − 4 sin 3 x Solution. I= ∫ cos 2x
dx = ∫ dx
2 cos 2 x − 1
Put cos x = z ⇒ sin x dx = – dz
sin x dx sin x dx
⇒ I= ∫ 3 − 4 sin 2 x = ∫ 4 cos2 x − 1 sin x dx −dz
I= ∫ 2
=∫
2 cos x − 1 2z 2 − 1
Put cos x = z
⇒ sin x dx = – dz = − 1 log 2 z + 2z 2 − 1  + C
2  
sin x dx dz
⇒ I= ∫ 4 cos2 x − 1 = ∫ 1 − 4z 2 =−
1 
log 2 cos x + 2 cos 2 x − 1  + C
2  

EXERCISE 7.5
I. Multiple Choice Questions (MCQs) x
1. ∫ 2
2x − 1
dx equals
4. ∫x 2
+ 2x + 2
dx is equal to
2x + 2x + 1
1
1 (a) log ( x 2 + 2 x + 2) − tan −1 ( x + 1) + C
(a) log (2 x 2 + 2 x + 1) − 2 tan −1 (2 x + 1) + C 2
2 1
(b) log ( x 2 − 2 x − 2) + tan −1 ( x + 1) + C
1 2
(b) log (2 x 2 + 2 x + 1) − 2 sin −1 (2 x + 1) + C
2 1
(c) log ( x 2 − 2 x − 2) − tan −1 ( x + 1) + C
1 2
(c) log (2 x 2 + 2 x + 1) − 2 cos −1 (2 x + 1) + C
2 (d) None of these
(d) None of these 2x − 1
5. ∫ 2 dx is equal to
x2 + x + 1 x + 2x + 3
2. ∫ x2 − x + 1
dx equals
3 x +1
(a) log ( x 2 + 2 x + 3) − tan −1
+C
2  2 x − 1 22
(a) x + log ( x − x + 1) +
2
tan  −1
+C
3  3  (b) (b) log ( x 2 − 2 x − 3) +
2
tan −1
x +1
+C
2  2 x − 1 3 2
(b) x + log ( x 2 + x + 1) + tan −1  +C
3  3  (c) log ( x 2 + 2 x + 3) −
2
tan −1
x +1
+C
3 2
2  2 x + 1
(c) x + log ( x 2 − x + 1) + tan −1  +C (d) None of these
3  3 
Evaluate each of the following Integrals
(d) None of these
II. Short Answer Type Questions-I
x
3. ∫ 2
x + 4x + 5
dx is equal to
1. ∫( sin 2x
cos 3x )
dx 2.
 cos 2 x 
∫  cos 3x  dx
∫( ) ( )
1
(a) log ( x 2 − 4 x + 5) + 2 tan −1 ( x + 2) + C sin x cos x
2 3.
cos 2x
dx 4. ∫ cos 2x
dx

1  sin 2 x 
(b) log ( x 2 + 4 x + 5) + 2 tan −1 ( x + 2) + C sin x
2 5. ∫ dx 6. ∫  sin 3x  dx
5 − sin 2 x
1 III. Long Answer Type Questions
(c) log ( x 2 + 4 x + 5) − 2 tan −1 ( x + 2) + C
2 x dx
1.
(d) None of these ∫ x4 + x2 + 3  [Delhi 2005]

Integrals 199
E:\AMIT_WORKS\Exam_Guru\EG_Mathematics-12_(working_02-06-2022)\EG_Mathematics-12_working\Open_Files\Chap_7\Chap_7
\ 17-Aug-2022 Amit Proof 5 Reader’s Sign _______________________ Date __________

dx (3 sin x − 2) cos x dx 
2. ∫ 9x 2 + 12x + 7  [A.I. 2009 (C)] 10. ∫ 5 − cos 2 x − 4 sin x
[Delhi 2013 (C)]

dx (5x + 3) dx 
3. ∫ x (log x)2 + 2 (log x) + 2 11. ∫ x 2 + 4x + 20
[Delhi 2009 (C), S.P. 2014]
 
dx (2x + 5) dx 
4. ∫  [A.I. 2009] 12. ∫ 7 − 6x − x 2
[Delhi 2009 (C), S.P. 2014]
9 + 8x − x 2
cos x dx dx
5. ∫ 13. ∫  [A.I. 2009]
2 sin 2 x − 6 sin x + 5 16 − 2x − 2x 2
5 x dx
6. ∫ 2 dx 14. ∫
3x + 13x − 10 52x − 2 × 5 x − 3
7. ∫
e x dx
5 − 4 e x − e 2x
15. ∫( cos 3x
cos 2x) dx [HOTS] 16. ∫( )sin 3x
cos 2x
dx  [HOTS]

∫( ) ∫( )
(5x − 2) dx sin 4x sin 4x
17. dx 18. dx
8. ∫ 1 + 2x + 3x 2  [Delhi 2013, 2014 (C)]
cos 3x sin 3x
(2 sin 2θ − cos θ) d θ   sin 2x 
9.
∫ 6 − cos 2 θ − 4 sin θ
[V. Imp.] [A.I. 2006] 19. ∫   dx
cos 2x 

Answers 7.5
2x − 1 2  2 x − 1
I. 1. (a) I = ∫ 2x 2
+ 2x + 1
dx
2
= x + log ( x − x + 1) +
3
tan −1 
 3 
+C

1 1
( 4 x + 2) − 1 − 1 3. (c) log ( x 2 + 4 x + 5) − 2 tan −1 ( x + 2) + C
=∫2 2 dx Q d (2 x 2 + 2 x + 1) = 4 x + 2 2
2x + 2x + 1  dx 
  1
1 4x + 2 dx 4. (a) log ( x 2 + 2 x + 2) − tan −1 ( x + 1) + C
= ∫ 2 dx − 2 ∫ 2 2
2 2x + 2x + 1 2x + 2x + 1
3 ( x + 1)
1 dx 5. (a) log ( x 2 + 2 x + 3) − tan −1 +C
= log (2 x + 2 x + 1) − ∫ 2 2
2

2 2 1
x +x+
1
2 II. 1. −1 log 2 cos x − 3 + C
= log (2 x 2 + 2 x + 1) − I1 2 3 2 cos x + 3
2
sin 2x 2 sin x cos x = 2 sin x
 1 Hint: =
x+ cos 3x 4 cos3 x − 3 cos x 4 cos 2 x − 3
dx dx 1 −1  2 Now put cos x = z.
=∫ 1 ∫
= 2
= tan 
1 1 
2
x +x+ 1  1  
 x +  + 1 1 + 2 sin x
2 2 4 2 2  2. log +C
2 2 1 − 2 sin x
= 2 tan–1(2x + 1)
cos 2 x cos 2 x cos x
1 Hint: = =
2 −1
Hence, from (1), I = log (2 x + 2 x + 1) − 2 tan (2 x + 1) + C cos 3x 4 cos x − 3 cos x 4 cos 2 x − 3
3
2
x2 + x + 1 =  cos x 2 
2. (a) I = ∫ x 2 − x + 1 dx  1 − 4 sin x 
Put sin x = z.
Dividing numerator by denominator, we get 2 cos x − 1 + C
3. − 1 log
 2x   (2 x − 1) + 1 2 2 2 cos x + 1
I = ∫ 1 + 2 dx = x + ∫  2 dx
 x − x + 1  x − x + 1 
Hint: sin x = sin x
dx cos 2x 2 cos 2 x − 1
= x + log ( x 2 − x + 1) + ∫ 2 Put cos x = z.
 1 3
 x −  +
2 4
4. 1 log 1 + 2 sin x + C
1 2 2 1 − 2 sin x
2 x−
2 +C
Hint: cos x = cos x
2 −1
= x + log ( x − x + 1) + tan
3 3 cos 2x 1 − 2 sin 2 x
2 Put sin x = z.

200 Mathematics–12
E:\AMIT_WORKS\Exam_Guru\EG_Mathematics-12_(working_02-06-2022)\EG_Mathematics-12_working\Open_Files\Chap_7\Chap_7
\ 17-Aug-2022 Amit Proof-5 Reader’s Sign _______________________ Date __________

5. − cos x + 4 + cos 2 x  + C
 
12. Ans. −2 7 − 6x − x 2 − sin −1 x + 3 + C
4 ( )
sin x sin x Hint: 2x + 5 = – (– 2x – 6) – 1
Hint: =
2 2
5 − sin x cos x + 4 and 7 − 6x − x 2 = 16 − (x + 3) 2
Put cos x = z.
13. 1 sin −1  2x + 1 + C
1 2 cos x − 1 + C  
6. − log 2 33 
4 2 cos x + 1

Hint:
sin 2 x
sin 3x
=
sin 2 x
=
sin x 14.
1 5x 1
log 5 ( )
− + 5 2x − 2 × 5 x − 3 + C
3 sin x − 4 sin 3 x 3 − 4 sin 2 x
sin x 15. 1 2 sin x − 1 +
= log 2 sin x + C
4 cos 2 x − 1 2 2 2 sin x + 1
Put cos x = z. 4 cos3 x − 3 cos x
Hint: cos 3x =
1 tan −1  2x + 1
2 cos 2x 1 − 2 sin 2 x
III. 1. Ans.
 11  + C;
11
=
(1 − 4 sin x) cos x
2

Hint: Substitute: x2 = z 1 − 2 sin 2 x


1  3x + 2  Put sin x = z.
2. Ans. tan −1  + C;
3 3  3 
16. −2 cos x − 1 log 2 cos x − 1 + C
( )
 2 
Hint: 9x + 12x + 7 = 9  x + 2 2 2 2 cos x + 1
2
+ 1
3 3 sin 3x 3
 Hint: = 3 sin x −2 4 sin x
cos 2x 2 cos x − 1
3. Ans. tan–1[1+ (log x)] + C
Hint: Substitute: log x = z =
(
4 cos 2 x − 1 sin x )
2 cos 2 x − 1
 x −4  + C;.
4. Ans. sin −1  Put cos x = z.
 5 
17. −2 cos x − 1 log 2 cos x − 3 + C
Hint: 9 + 8x − x 2 = 25 − (x − 4) 2 2 3 2 cos x + 3

5.
2
( 2 )
1 log sin x − 3 + sin 2 x − 3 sin x + 5 + C
3
Hint:
sin 4x
cos 3x
=
4 sin x cos x cos 2x
4 cos3 x − 3 cos x
1 log 3x − 2 4 sin x cos 2x
6. +C =
17 3x + 15 4 cos 2 x − 3
 x 
7. Ans. sin −1 e + 2 + C;
4  2 cos 2 x − 1 sin x
= 
 3 
4 cos 2 x − 3
Hint: Substitute: ex = z Put cos x = z.
5 11 tan −1  3x + 1 + ;
8. Ans. log | 1 + 2x + 3x 2 | −   C 18. 2 sin x + 1 log 2 sin x − 3 + C
6 3 2 2 
2 3 2 sin x + 3
5 11 sin 4x 4 sin x cos x cos 2x
Hint: 5x − 2 = (6x + 2) −
6 3 Hint: =
sin 3x 3 sin x − 4 sin 3 x
9. Ans. 2 log | sin2 q – 4 sin q + 5 | + 7 tan–1 (sin q – 2) + C

Hint: 2 sin 2θ − cos θ


= =
( 2
4 cos 2x cos x 4 1 − 2 sin x cos x )
6 − cos 2 θ − 4 sin θ 3 − 4 sin 2 x 3 − 4 sin 2 x
Put sin x = z.
(4 sin θ − 1) cos θ
= 19. − 2 cos 2 x − 1 + C
sin 2 θ − 4 sin θ + 5
sin 2x
10. Ans. 3 log | sin x −2 | − 4
+C Hint: = 2 sin x cos x
sin x − 2 cos 2x 2 cos 2 x − 1
Put cos x = z.
Hint: (3 sin x2 − 2) cos x = (3 sin x − 2) cos x
5 − cos x − 4 sin x sin 2 x − 4 sin x + 4 OR 1 − 2 sin 2 x + C
sin 2x
11. Ans. 5 x 2 + 4x + 20 −7 log (x + 2) + x 2 + 4x + 20 + C Hint: = 2 sin x cos x
cos 2x 1 − 2 sin 2 x
Hint: 5x + 3 = 5 (2x + 4) − 7
2 Put sin x = z.

Integrals 201
E:\AMIT_WORKS\Exam_Guru\EG_Mathematics-12_(working_02-06-2022)\EG_Mathematics-12_working\Open_Files\Chap_7\Chap_7
\ 17-Aug-2022 Amit Proof 5 Reader’s Sign _______________________ Date __________

Topic 6. Integration Using Partial Fractions


f ( x) Putting the value of B in eqn. (ii) we have
Rational Function: The fraction of the form where f (x)
g ( x) 2(2C – 1) + C = 0  4C – 2 + C = 0
3 2 2
and g(x) are polynomials and g(x) ≠ 0. x2 + 1 , x 3 + 1 are rational 5C = 2 \ C =
5
x + 2 x +1  2 1 1
functions. \ B = 2   − 1 = − and A =
 5 5 5
f ( x) 1
Proper Rational Function: is called proper rational \ ∫
g ( x) ( x + 2)( x 2 + 1)
dx
function if degree of the numerator f (x) is smaller than the degree 1 1 2
of the denominator g(x), e.g., x+2 ,
( x − 3) . 5
− x+
5 5
= ∫ dx + ∫ 2
(
( x + 1) ( x + 2) x 2 + 1 ( x + 4) )  
( x + 2) ( x + 1)
dx

f ( x)
Improper Rational Function: is called improper rational 1 1 1 x−2
g ( x)
5 ∫ ( x + 2)
= dx − ∫ 2 dx
5 ( x + 1)
function if degree of the numerator f (x) is greater than the degree
1 1 1 x 2 1
x3 + 4 x 4 + 3x + 1 .
5 ∫ ( x + 2)
= dx − ∫ 2 dx + ∫ 2 dx
of the denominator g(x), e.g., , 2 5 ( x + 1) 5 x +1
( x + 1) ( x + 2) x + 4x + 7
1 1 1 2x 2 1
f ( x)
5∫ x + 2
= dx − ∫ 2 dx + ∫ 2 dx
Partial Fractions: Suppose is a proper rational function 10 x + 1 5 x +1
g ( x)
1 1 2
and g(x) is factorised into linear, quadratic, cubic polynomials, \ I = log x + 2 − log x 2 + 1 + tan − 1 x + C
f ( x) 5 10 5
then we can express into sum or difference of simpler
g ( x) Putting the given value of I
proper fractions. These simpler fraction are called partial fractions. 1
\ a log 1 + x 2 + b tan − 1 x + log x + 2 + C
Partial fractions can be divided into four types: 5
1 1 2
= log x + 2 − log x 2 + 1 + tan − 1 x + C
Type I: When the denominator of the proper partial fraction 5 10 5
consists of linear factors without repetition. 1 2
\ a = − and b =
10 5
ax 2 ± bx ± c Hence, the correct option is (c).
(px + q) (mx + n) (dx + e) Example 2. Change into partial fractions:
dx 3x 2 + 4x + 5
Example 1. If ∫ ( x + 2)( x 2 + 1) (x − 1) (x + 2) (x − 3)
1 3 x 2 + 4x + 5
= a log 1 + x 2 + b tan − 1 x + log x + 2 + C then Solution. Take:
5 (x − 1) (x + 2) (x − 3)
1 −2 1 −2
(a) a = − ,b= (b) a = , b = A B C
10 5 10 5 = + +  ...(1)
x −1 x + 2 x − 3
1 2 1 2
(c) a = − , b = (d) a = , b =
10 5 10 5 ⇒ 3x2 + 4x + 5 =A(x + 2)(x – 3) + B(x – 1)(x – 3)
dx  + C(x – 1)(x + 2) (2)
Solution. Let I = ∫ ( x + 2)( x 2 + 1)
Put x = 1 in equation (2),
Let us resolve the given integrand into partial fractions
1 A Bx + C 3 + 4 + 5 = A(3) (–2) ⇒ A = − 2
Put = + 2
2
( x + 2)( x + 1) ( x + 2) ( x + 1) Put x = – 2 in equation (2),
1 = A(x2 + 1) + (x + 2) (Bx + C) 3
12 – 8 + 5 = B(–3) (–5) ⇒ B =
1 = Ax2 + A + Bx2 + Cx + 2Bx + 2C 5
Put x = 3 in equation (2),
1 = (A + B)x2 + (C + 2B)x + (A + 2C)
Comparing the like terms, we have 22
27 + 12 + 5 = C(2) (5) ⇒ C =
5
A+B=0 ...(i)
2B + C = 0 ...(ii) On putting the values of A, B and C in equation (1), we get
A + 2C = 1
Subtracting (i) from (iii) we get
...(iii)

3x 2 + 4x + 5
(x − 1) (x + 2) (x − 3)
=
−2
+( ) ( ) ( )
3 1
x −1 5 x + 2
+
22 1
5 x−3
2C – B = 1 \ B = 2C – 1 These partial fractions can be integrated easily.

202 Mathematics–12
E:\AMIT_WORKS\Exam_Guru\EG_Mathematics-12_(working_02-06-2022)\EG_Mathematics-12_working\Open_Files\Chap_7\Chap_7
\ 17-Aug-2022 Amit Proof-5 Reader’s Sign _______________________ Date __________

Type II: When the denominator of the fraction consists of 41


⇒ C= −
repeated and non-repeated linear factors. 6
ax 2 ± bx ± c On putting the values of A, B and C in equation (1), we get

(px ± q) (mx ± n) 2
3x 2 − 4x + 8 7 1 
= 
11x + − 41
+ 62 6 ( )
2
2x + 5x + 7 (x − 1) (x 2 + 4x + 1) 6  x −1 x + 4x + 1
Example 3. Change into partial fractions:
(x − 2) (x − 3) 2
7  1  1  11x − 41 
2x 2 + 5x + 7 = +
Solution. Take: 6  x − 1 6  x 2 + 4x + 1
(x − 2) (x − 3) 2
A B C These partial fractions can be integrated easily.
= + +  ...(1)
x − 2 x − 3 ( x − 3)2 Type IV: When the numerator and denominator of the proper
⇒ 2x2 + 5x + 7 = A(x – 3)2 + B(x – 2) (x – 3) + C(x – 2)...(2) fraction consists of purely quadratic fractions.
Put x = 2 in equation (2),
ax 2 + b
8 + 10 + 7 = A
Put x = 3 in equation (2),
⇒ A = 25
( )(
cx + d ex 2 + f
2
)
18 + 15 + 7 = C ⇒ C = 40 3x 2 + 5
Example 5. Change into partial fractions:
Equate coefficients of x on both sides of equation (2) and
2
(x 2 + 4) (x 2 + 1)
substitute the value of A, Solution. Put x2 = y in the given fraction,
2= A+B ⇒ 2 = 25 + B
3y + 5
⇒ B = – 23 we get:
(y + 4) (y + 1)
On putting the values of A, B and C in the equation (1), 3y + 5
we get Take:
(y + 4) (y + 1)
2x 2 + 5x + 7 25 23 40
2 = x−2
− + A B
(x −2) (x −3) x − 3 (x − 3) 2 = +  ...(1)
y + 4 y +1
These partial fractions can be integrated easily.
⇒ 3y + 5 = A(y + 1) + B(y + 4) ...(2)
Type III: When the denominator of the proper fraction consists
of linear factors and quadratic factors. Put y = – 4 in equation (2)

ax 2 ± bx ± c –12 + 5 = A(–3) ⇒ A = 7
3
(
( px ± q ) mx 2 ± nx ± r ) Put y = – 1 in equation (2),
2
– 3 + 5 = B(3) ⇒ B =
Example 4. Change into partial fractions: 3x 2 − 4x + 8 3
(x − 1) (x 2 + 4x + 1)
On putting the values of A and B in equation (1), we get
3x 2 − 4x + 8 A
+ 2
Bx + C 3y + 5 7 1  2 1 
Solution. Take: =  ...(1) =  +
2 x − 1
(x − 1) (x + 4x + 1) x + 4x + 1 (y + 4) (y + 1) 3  y + 4  3  y + 1
⇒ 3x2 – 4x + 8 = A(x2 + 4x + 1) + Bx(x – 1) + C(x – 1) ...(2)
3x 2 + 5 7 1  2 1 
Put x = 1 in equation (2), or =  2 +
2 2
(x + 4) (x + 1) 3  x + 4  3  x 2 + 1
7
3 – 4 + 8 = A(1 + 4 + 1) ⇒ A = These partial fractions can be integrated easily.
6
Equate coefficient of x2 on both sides of equation (2) and Note:
substitute the value of A 1. If degree of numerator of the fraction is equal to or greater
7 11 than the degree of the denominator, divide the numerator
3= A+B ⇒ B = 3 − = by the denominator and reduce the degree of the numerator
6 6
11 less than the degree of the denominator.
⇒ B= 2. Some writers say that the method given in previous
6
examples for finding values of the constants A, B and C
Equate coefficients of x on both sides of equation (2) and
is wrong. In example:
substitute the values of A and B,
3x + 4 A Β A ( x + 4) + B ( x − 1)
– 4 = 4A – B + C = + =
7 11
( x − 1) ( x + 4) x − 1 x + 4 ( x − 1) ( x + 4)
28 11
⇒ –4 = 4 × − + C ⇒ −4 = − +C
6 6 6 6 ⇒ 3x + 4 = A(x + 4) + B(x – 1).

Integrals 203
E:\AMIT_WORKS\Exam_Guru\EG_Mathematics-12_(working_02-06-2022)\EG_Mathematics-12_working\Open_Files\Chap_7\Chap_7
\ 17-Aug-2022 Amit Proof 5 Reader’s Sign _______________________ Date __________

They say that x = 1 and x = – 4 cannot be put in the above Here there is no restriction. Teachers are requested to send
A B me an example where the above substitution method gives
equation because and are not defined, we are wrong results. Then I shall change the method.
x −1 x+4
A B 3. Finally, once during board paper evaluation, it was decided
not putting x = 1 and x = – 4 in and but we are
x −1 x+4 that the values of constants A, B, C etc. can be evaluated
putting x = 1 and x = – 4 in 3x + 4 = A(x + 4) + B(x – 1). by any method, will be accepted.

EXERCISE 7.6
I. Multiple Choice Questions (MCQs)
 2 + sin x 
x (b) log  +C
1. ∫ dx equals  1 + sin x 
( x − 1) ( x − 2 )
 1 + sin x 
( x − 1)2 ( x − 2 )2 (c) log  +C
(a) log +C (b) log +C  2 + sin x 
x−2 x −1
2
(d) None of these
 x −1 
(c) log  +C (d) log |(x – 1) (x – 2)| + C ( 2x2 + 1 )
 x − 2  7. If ∫ ( x 2 − 4 ) ( x 2 − 1 ) dx
 [NCERT]
dx   x + 1  a  x − 2 b 
2. ∫ x ( x 2 + 1) equals  = log       + C
  x − 1  x + 2  
1 then, the values of a and b are respectively
(a) log | x | − log| x 2 + 1 | + C
2
1 3 3 3 −1 3
1 (a) , (b) −1, (c) 1, (d) ,
(b) log | x | + log | x 2 + 1 | + C 2 4 2 2 2 4
2
1 Evaluate each of the following Integrals
(c) − log | x | + log | x 2 + 1 | + C
2 II. Short Answer Type Questions-I
1 ( x + 1) dx
(d) log | x | + log| x 2 + 1| + C  [NCERT] (x 2 + 3) dx
2 1. ∫ (x + 2) (x − 3) 2. ∫ (x − 1) (x + 2) (x + 3)
x −1
3. ∫ dx is equal to dx
( x + 2 )( x + 3 ) 3. ∫ (x + 2) x 2
( x + 3 )4 ( x + 2 )4
(a) log +C (b) log +C (x 2 + 5) dx
( x + 2 )3 ( x + 3 )3 4. ∫ (x − 1) (x + 2)2  [Delhi 2006, 2013 (C)]
3
(c) log ( x + 2 ) + C (d) None of these (x 2 + 5) dx dx
( x + 3 )4 5. ∫ (x + 1) (x 2 + 4) 6.
∫ (x − 4) (x 2 + 4x + 7)
x2 + 1
4. ∫ x( x dx is equal to
7. (1 − x 2) dx  [Delhi 2010]

2
− 1)
x (1 − 2x)
x2 − 1 x2 − 1
(a) log +C (b) − log +C (3x + 5) dx 
x x 8. ∫ x3 − x 2 − x +1
[Delhi 2013, 2014 (C)]
x x
(c) log 2 +C (d) − log 2 +C (x 2 + x + 1) dx
x +1 x +1 9. ∫ (x + 2) (x 2 + 1)  [A.I. 2013 (C), 2015, Delhi 2013 (C)]
4x
5. ∫ 2 dx equals
( x + 1 ) ( x 2 + 3) (x 2 + x + 1) dx 
10. ∫ (x − 1)3 [A.I. 2000]
 x2 + 1  x 2 + 3
(a) log  2 +C (b) log  2 +C
 x + 3   x + 1  cos x dx
(c) tan (x + 1) (x + 3) + C
–1 2 2
11. ∫ (1 − sin x) (2 − sin x)  [Delhi 2007]

(d) 2log (x2 + 1) (x2 + 3) + C dx


cos x
12. ∫ sin x + sin 2x  [A.I. (C), Delhi 2015]
6. ∫ dx equals
(1 + sin x) (2 + sin x) 2e x dx dx
(a) log (1 + sin x) (2 + sin x) + C
13.
∫ e3x − 6e2x + 11e x − 6 14. ∫ sin x (3 + 2 cos x )
204 Mathematics–12
E:\AMIT_WORKS\Exam_Guru\EG_Mathematics-12_(working_02-06-2022)\EG_Mathematics-12_working\Open_Files\Chap_7\Chap_7
\ 17-Aug-2022 Amit Proof-5 Reader’s Sign _______________________ Date __________

dx (3 sin θ − 1) cos θ
15. ∫ x 6(log x)2 + 7 (log x) + 2 2. ∫ 5 − cos2 θ − 4 sin θ d θ
 
dx dx
16. ∫ x 17. ∫ e x (e x + 1) ( 2x + 1)
e −1 3. ∫ ( x 2 + 1) ( x 2 + 4) dx [A.I. 2015]
dx
18. ∫ 2e2x + 3e x + 1  x3 − x − 2 
4. ∫  x 2 − 1 
dx
dx
19. ∫ x ( x 4 − 1) x 4 dx
5. ∫ (x − 1) (x 2 + 1)  [S.P. 2014]
dx
20. ∫ x(x n + 1) x3 dx
3
6. ∫ x 4 + 3x 2 + 2  [A.I. 2008, 2014 (C)]
21. ∫ (tan φ +3 tan φ) d φ  [Delhi 2009 (C)]
tan φ + 1
x2 + x + 1
dx
7. ∫ ( x − 1)3
dx
22. ∫ (x 2 + 1) (x 2 + 2)   [Delhi 2010 (C)]
2x dx
III. Long Answer Type Questions
8. ∫ ( x 2 + 1) ( x 2 + 4)  [Delhi 2017]

1. (x 2 + 1) dx
∫ (x 2 + 4) (x 2 + 25) 9.
cos θ
∫ ( 4 + sin 2 θ) (5 − 4 sin 2 θ) d θ  [A.I. 2017]
 [Delhi 2013, 2014 (C), A.I. 2013 (C)]

Answers 7.6
x when t = 0, 1 = A(0 + 1) ⇒ A = 1
I. 1. (b) I = ∫ ( x − 1)( x − 2 ) dx when t = –1, 1 = B(–1)   ⇒ B = –1
x A B 1 dt 1 dt 1 1
Let = + \ I= ∫ − ∫ = log | t | − log| t + 1| + C
( x − 1)( x − 2 ) x −1 x − 2 2 t 2 t +1 2 2
⇒ x = A(x – 2) + B(x – 1) ...(1) 1 2 1 2
= log | x | − log | x + 1| + C
Putting x = 1, 2 in (1) 2 2
when x = 1, 1 = A(1 – 2) ⇒ A = –1 1 1
= × 2 log | x | − log| x 2 + 1| + C
when x = 2, 2 = B(2 – 1) ⇒ B = 2 2 2
x −1 2 1
\ = + = log | x | − log| x 2 + 1| + C
( x − 1)( x − 2 ) x − 1 x − 2 2
x −1
x 1 1 3. (a) I = ∫ dx
\ I= ∫ ( x − 1)( x − 2 ) dx = − ∫ x − 1 dx + 2 ∫ x − 2 dx ( x + 2 )( x + 3 )
x −1 A B
= − log | x − 1 | + 2 log | x − 2 | + C Let = +
( x + 2 )( x + 3 ) x + 2 x + 3
= − log | x − 1| + log | ( x − 2 ) 2 | + C
Multiplying both sides by (x + 2)(x + 3), we get
( x − 2 )2 x – 1 = A(x + 3) + B(x + 2) ...(1)
= log +C
x −1 Put x = –2, –3 in (1)
dx x when x = –2, –2–1 = A(– 2 + 3) ⇒ A = –3
2. (a) I = ∫
x( x 2 + 1) ∫ x 2 ( x 2 + 1)
= dx
when x = –3, –3–1 = B(–3 + 2) ⇒ B = 4
1 x −1 −3 4
Put x2 = t, then 2x dx = dt ⇒ x dx = dt \ = +
2 ( x + 2 )( x + 3 ) x + 2 x + 3
1 dt
\ I= ∫
2 t ( t + 1) \ I=
x −1 dx
∫ ( x + 2 )( x + 3 ) dx = − 3∫ x + 2 + 4 ∫ x + 3
dx

1 A B
Let = + = –3log | x + 2 | + 4 log | x + 3 | + C
t ( t + 1) t t +1
Multiplying both sides by t(t + 1), we get = − log| ( x + 2 )3 | + log | ( x + 3 ) 4 | + C
1 = A(t + 1) + B(t) ...(1) ( x + 3) 4
= log +C
Put t = 0, – 1 in (1) ( x + 2 )3

Integrals 205
E:\AMIT_WORKS\Exam_Guru\EG_Mathematics-12_(working_02-06-2022)\EG_Mathematics-12_working\Open_Files\Chap_7\Chap_7
\ 17-Aug-2022 Amit Proof 5 Reader’s Sign _______________________ Date __________

x2 + 1 x2 + 1 (2 x 2 + 1)
4. (a) I = ∫ x( x 2
− 1)
dx = ∫
x ( x − 1) ( x + 1 )
dx 7. (a) I= ∫ (x 2
− 4) ( x 2 − 1)
dx

x2 + 1 A B C Let x2 = t
Let = + +
x ( x − 1)( x + 1) x x − 1 x + 1 ( 2 x 2 + 1) 2t + 1
\ =
2 2
( x − 4 )( x − 1) ( t − 4 )( t − 1)
Multiplying both sides by x(x – 1)(x + 1), we get
\ x2 + 1 = A(x – 1) (x + 1) + B(x)(x + 1) + Cx(x – 1) ...(1) 2t + 1 A B
Let = +
( t − 4 )( t − 1) t − 4 t −1
Put x = 0, –1, 1 in (1)
Multiplying both sides by (t – 4)(t – 1), we get 2t + 1 = A(t – 1)
when x = 0, 0 + 1 = A(0 – 1)(0 + 1) ⇒ A = –1
+ B(t – 4)
when x = –1, 1 + 1 = C(–1)(–1 –1) ⇒ 2C = 2 ⇒ C = 1 Putting t = 4, we get 2(4) + 1 = A(4 – 1) ⇒ 3A = 9 ⇒ A = 3
when x = 1, 1 + 1 = B(1)(1 + 1) ⇒ B = 1 Putting t = 1, we get
x2 + 1 1 1 1 2(1) + 1 = B(1 – 4) ⇒ –3B = 3 ⇒ B = –1
\ =− + +
x ( x − 1)( x + 1) x x −1 x +1 2t + 1 3 1 2x2 + 1
\ = − ⇒ 2
( t − 4 )( t − 1) t − 4 t−1 ( x − 4 )( x 2 −1)
x2 + 1 dx dx dx
I = ∫ x ( x − 1)( x + 1) dx = − ∫ x ∫ x −1 ∫ x +1
+ +
3 1
= −
x2 − 4 x2 − 1
= − log | x | + log | x − 1| + log | x + 1| + C
2x2 + 1
= log
( x − 1)( x + 1)
+ C = log
x2 − 1
+C
\ I= ∫ (x 2
− 4 )( x 2 −1)
dx
x x
dx dx
= 3∫
x 2 − 22 ∫ x 2 − 1

4x 2x
5. (a) I = ∫ 2 dx = 2 ∫ 2 dx
( x + 1)( x 2 + 3) ( x + 1)( x 2 + 3 )
3  x −2 1  x −1
= log   − log  +C
Put x2 = t, then 2x dx = dt 4  x + 2  2  x + 1 
dt 1  1 1  3 1
I = 2∫ = 2 × ∫ − dt  x − 24  x +1 2
( t + 1)( t + 3 ) 2  t + 1 t + 3  = log   + log  +C
 x+ 2  x − 1 
 1 1 
= ∫ − dt = log | t + 1| − log| t + 3| + C  1 3

 t + 1 t + 3    x + 1  2  x − 2 4
+C
= log 
  x − 1  x + 2  
t +1 x +1 2  
= log + C = log 2 +C 1 3
t+3 x +3 \ a = and b =
2 4
cos x
6. (c) I = ∫ (1 + sin x) (2 + sin x) dx II. 1. Ans.
1 log | x + 2 | + 4 log | x − 3 | + C;
5 5
Put sin x = t, so that cos x dx = dt
dt
Hint: x +1
= ( ) ( )
1 1 +4 1
(x + 2) (x − 3) 5 x + 2 5 x − 3
\ I= ∫
(1 + t )(2 + t ) 2. Ans. 1 log | x − 1 | − 7 log | x + 2 | + 3 log | x + 3 | + C
3 3

( ) ( ) ( )
1 A B 2
(x + 3) 1 1 7 1
Let = + Hint: = − +3 1
(1 + t )(2 + t ) 1+ t 2 + t (x − 1) (x + 2) (x + 3) 3 x −1 3 x + 2 x+3
1 1
3. Ans. log | x + 2 | − log | x | − 1
\ I = A(2 + t) + B(1 + t) + C;
4 4 2x

( ) ()
Put t = –2, we get I = B(–2 + 1) ⇒ B = –1
Hint: 1 = 1 1 − 1 1 + 1  1 
Put t = –1, we get I = A(2 – 1) ⇒ A = 1 (x + 2) x 2 4 x + 2 4 x 2  x 2 
dt dt 4. Ans. 2 log | x − 1 | + 1 log | x + 2 | + 3 + C;
\ I= ∫1+ t −∫ 2 + t 3 3 x+2

Hint: x2 + 5
= log| 1 + t | − log | 2 + t | + C (x − 1) (x + 2) 2

= log
1+ t
2+t
+ C = log
1 + sin x
2 + sin x
+ C.  ( ) ( )
3 x −1 3 x + 2

= 2 1 + 1 1 − 3  1 2

 (x + 2) 

206 Mathematics–12
E:\AMIT_WORKS\Exam_Guru\EG_Mathematics-12_(working_02-06-2022)\EG_Mathematics-12_working\Open_Files\Chap_7\Chap_7
\ 17-Aug-2022 Amit Proof-5 Reader’s Sign _______________________ Date __________

5. Ans. 6 log | x +1| − 1 log(x 2+ 4) + 1 tan −1 x + C;


5 10 10 2 () 13. Ans. log
(e x − 1) (e x − 3)
(e x − 2) 2
+ C;
 − x + 1
( )
x2 + 5 Hint : Put ex = z and
Hint: = 6 1 +  5 5
(x + 1) (x 2 + 4) 5 x + 1  x 2 + 4  2 
= 2 
  3 2
z − 6z + 11z − 6  (z − 1 ) (z − 2 ) (z − 3) 

6. Ans. 1 log | x − 4 | − 1 log x 2 + 4x + 7
39 78
 x + 2
= 1 −2 1 + 1
z −1 z−2 z −3 ( )
− 8 tan −1  + C;
39 3  3  14. 1 log 1 − cos x − 1 log 1 + cos x + 2 log 3 + 2 cos x
10 2 5
Hint: 1
Hint: 1 sin x
(x − 4) (x 2 + 4x+ 7) =
sin x (3 + 2 cos x ) (1 + cos x )(1 − cos x )(3 + 2 cos x )
 −x− 8  Put cos x = z.
= 1
( )
1 +  39 39 
39 x − 4  x 2 + 4x + 7 
 
15. Ans. log
1 + 2 log x
2 + 3 log x
+C ;

x 3 Hint: Put log x = z and
7. Ans. + log | x | − log | 2x −1 | + C;
2
1 − x2
4
 x−2 
2
1
6z + 7z + 2
=
(2z + 1)
1
(3z + 2)
= 2 1 −3 1
2z + 1 2 + 3z ( ) ( )
Hint: =1+1
x (1 − 2x) 2 2  x (2x − 1)  x
16. Ans. log e −x 1 + C;
=
1 1 2
+ −
3  e
2 2  x 2x − 1 Hint: Put ex = z
1 = 1 = 1 −1
8. Ans. 1 log x + 1 − 4 + C; ⇒ x
e − 1 z(z − 1) z − 1 z
2 x −1 x −1
Hint: 3 3x +5 = 3x + 5 ex + 1 1
17. Ans. log − x + C; Hint : Put e x = z
x − x 2 − x + 1 (x + 1) (x − 1) 2 ex e


( ) ( )
=1 1 −1 1 +
2 x + 1 2 x − 1 (x − 1) 2
4 \ e dx = dz ⇒z dx = dz ⇒ dx =
x

1 1
dz
z
⇒ ∫ x x dx = ∫ dz
9. Ans.
3 log | x + 2 | + 1 log | x 2 + 1 | 1
+ tan −1 x + C; e (e + 1) (z + 1) z 2
5 5 5  1 − 1 + 1  dz
 2x + 1  = ∫ 
 z + 1 z z 2 
( )
2
x + x + 1 3 1
Hint: = +  5 5 18. log(ex) + log(ex + 1) – 2 log(2ex + 1) + C
(x + 2) (x 2 + 1) 5 x + 2  x 2 + 1 
  Hint: Put ex = z.
10. Ans. log | x − 1 | − 3 − 3 4
x − 1 2(x − 1) 2
+ C; 19. 1 log x − 1 +C
4 x4
2
Hint: x + x +3 1 = 1 + 3 + 3 n
(x −1) x − 1 (x − 1) 2 (x − 1)3 20. Ans. 1 log nx + C; Hint : Put x n = z
n x +1
2 − sin x
11. Ans. log + C; x n −1 = 1  1  = 1  1 − 1 
1 − sin x ⇒
x (x n + 1) n  z (z + 1)  n  z z + 1
n
1
Hint: Put sin x = z. It gives: = 1 − 1
(1 − z )( 2 − z ) 1− z 2 − z
21. Ans.
−1
log | tan φ + 1| + 1 log tan 2 φ − tan φ + 1
12. Ans. 1 log | 1 + cos x | + 1 log | 1 − cos x | 3 6
2 6  2 tan φ − 1
+ 1 tan −1   + C;
− 2 log | 1 + 2 cos x | + C; 3  3
 3
Hint: 1 1 (tan 3 φ + tan φ) d φ tan φ sec2 φ
= Hint:
sin x + sin 2x sin x (1 + 2 cos x) ∫ 3
tan φ + 1
=∫
tan 3 φ + 1

sin x
= Put tan φ = z \ sec2 f df = dz
(1 + cos x ) (1 − cos x) (1 + 2 cos x)
tan φ sec2 φ  
d φ = ∫ 3 z dz = ∫ 
z
Put cos x = z and 1
(1 + z) (1 − z) (1 + 2z)
\ ∫ 3
tan φ + 1 z +1
2  dz
 (z + 1) (z − z + 1) 

( ) ( ) ( )
= − 1 1 + 1 1 + 4 1
2 1 + z 6 1 − z 3 1 + 2z 3 z +1 ( )
= −1 ∫ 1 dz + 1 ∫  2 z + 1  dz
3  z − z + 1

Integrals 207
E:\AMIT_WORKS\Exam_Guru\EG_Mathematics-12_(working_02-06-2022)\EG_Mathematics-12_working\Open_Files\Chap_7\Chap_7
\ 17-Aug-2022 Amit Proof 5 Reader’s Sign _______________________ Date __________

  x4 (x 4 − 1) + 1
22. Ans. tan −1 x − 1 tan −1  x  + C; Hint: =
2  2 (x − 1) (x + 1) (x − 1) (x 2 + 1)
2

Hint: 1 = 1 x4 − 1 1
(x + 1) (x 2 + 2) (y + 1) (y + 2)
2 = +
(x − 1) (x 2 + 1) (x − 1) (x 2 + 1)
1 1 1 1
= − = − 1
y + 1 y + 2 x2 + 1 x2 + 2 = (x + 1) +
(x − 1) (x 2 + 1)

III. 1. Ans.
− 1 −1 x
14
tan
2 35 ()
+ 8 tan −1
x
5
+C () − x 1
1  1   2 − 2
= (x + 1) + +
x2 + 1 y +1 2  x −1  x 2 + 1 
Hint: =  
(x + 4) (x 2 + 25) (y + 4) (y + 25)
2

    (
6. Ans. log (x 2 + 2) − 1 log x 2 + 1 + C; )
= −1  1  + 8  1  2
7  y + 4  7  y + 25 
x3
Hint: ∫ x 4 + 3x 2 + 2 dx . Put x = z.
2
\x dx = dz
5 2
2. Ans. 3 log sin θ − 2 − + C;
(sin θ − 2) It gives
1 z  1  −1 2 
Hint: 3z − 12 = 3 + 5 ∫ 2  (z + 1) (z + 2)  dz = 2 ∫  z + 1 + z + 2  dz
( z − 2) z − 2 ( z − 2)2

() 7. log x − 1 − 3 −
2 3
3. Ans. 1 log x2 + 1 +  tan −1 ( x ) − tan −1 x  + C
1 1 
x − 1 2 ( x − 1)2
+C
3 x +4 3 2 2 

4.
x2
2
x
+ log + 1 + C
x −1 ( ) ( )
 2
8. 1 log x 2 + 1 − 1 log x 2 + 4 + 1 tan −1  x  + C
5 10 10  2

5. Ans.
x 2 + x + 1 log | x − 1) − 1 log | x 2 + 1 |
2 2 4
− 1 tan −1 x + C;
2
9. −
1
30 ( )
tan −1 sin θ + tan −1 ( 2 sin θ ) + C
2
2
15

Topic 7. Integration By Parts


When we have to integrate a product of two functions like ∫ f ( x ) g ( x ) dx, we use the method of integration by parts.
Here first step is to decide that out of the two given functions which function will be taken as first function and which function
will be taken as a second function. First function must be easily differentiable and second function must be easily integrable. After
taking this decision, write selected first function at first place and selected second function at second place.
The selection of first and second function is done by ILATE rule. This rule tells us the order in which first and second function
are taken.

ILATE Rule
I L A T E
1. Inverse Trigonometric function
2. Logarithmic function
3. Algebraic function
4. Trigonometric function
5. Exponential function
ILATE rule tells us the order in which we take two functions as first and second.
Note: This rule is not a hard and fast rule.

Formula for Integration by Parts


∫ f ( x ) g ( x ) dx = f ( x ) ∫ g ( x ) dx − ∫  f ′ ( x ) ∫ g ( x ) dx  dx

or ∫ (first function )(second function ) dx = [first function × integral of second function]


– Integral of [derivative of first function × integral of second function]

208 Mathematics–12
E:\AMIT_WORKS\Exam_Guru\EG_Mathematics-12_(working_02-06-2022)\EG_Mathematics-12_working\Open_Files\Chap_7\Chap_7
\ 17-Aug-2022 Amit Proof-5 Reader’s Sign _______________________ Date __________

Example 1. Evaluate: ∫ (9x + 11) e


5x
dx ⇒ I = (9x + 11)
e5x
5 ∫ ( )
1
− 9 × e5x dx
5
∫ (9x + 11) e dx
5x
Solution. I= 5x
⇒ I= (9x + 11) ∫ e5xdx − ∫ (9x + 11)′ ∫ e5xdx  dx ⇒ I = (9x + 11) e − 9 e5x + C
5 25

EXERCISE 7.7

∫ (sin x ) dx 
I. Multiple Choice Questions (MCQs) −1
5. [Delhi 2004, A.I. 2006 (C)]
∫ x e dx equals
2
3 x
1.
∫ (sec x) dx 
−1
6. [Delhi 2005]
1 2 2 2
(a) ( x + 1) e x + C (b) (b) ( x 2 + 1) e x + C
2
∫ ( x cot x ) dx 
2 −1
7. [A.I. 2007 (C)]
1 2
∫ (sec x ) dx
2
(d) ( x 2 − 1) e x + C
2
(c) ( x − 1) e x + C 8. −1
2
2. ∫x ∫ ( tan x ) dx
3 2
( log x ) dx = 9. −1

1 4
∫ x sec x dx
2
(a) x [ 8 (log x) 2 + 4 log x − 1 ] + C 10.
32
1 4 III. Long Answer Type Questions
(b) x [8 ( log x ) 2 + 4 log x + 1 ] + C
32
∫ (x + 1) e x dx ∫x a
2 2 x
1. 2. dx
1 4
(c) x [ 8 ( log x ) 2 − 4 log x − 1 ] + C x dx
∫ (3x + 7) sin
2
32 3. ∫ 1+ sin x 4. x dx
(d) None of these
( xn ) dx ( )
∫ (x) e ∫
2n − 1
3. If I = ∫ e x sin 2 x dx, the for what value of K, 5. 6. x sin x dx
KI = ex (sin 3x – cos 2x) + C? (log x) dx 
(a) 1 (b) 3 (c) 5 (d) 7
7. ∫ (x + 1) 2
[A.I. 2015]

4. ∫ tan
−1
x dx is equal to [NCERT Exemplar]
∫ log (x + 1) dx 
2
8. [V. Imp.] [Delhi 2006, A.I. 2006]
(a) ( x + 1) tan −1 x − x + C
(b) x tan −1 x − x+C
9. ∫ log  x + x 2 + a 2  dx


∫ (x + 1) e
x
(c) x − x tan −1 x + C 10. log (xe x) dx

(d) x − ( x + 1) tan −1 x + C  cos x + sin x 


11. ∫ (cos 2x) log  cos x − sin x  dx
 1 
5. ∫ log (log x) + (log x)  dx equals
∫  x (sin x) dx 
2 −1
 12. [V. Imp.] [A.I. 2009]
x
∫ (sin x) dx
−1 2
(a) x log (log x) + +C 13.
log x
∫  x (sec x) dx
−1
x 14.
(b) x log (log x) − +C
log x
∫ ( x cos x ) dx 
2 −1
15. [Foreign 2009]
log x
(c) x log (log x) + ∫  x (tan x)  dx
+C −1 2
16.
x
log x  sin −1 x 
(d) x log (log x) −
x
+C 17. ∫  x 2 
dx  [V. Imp.]

Evaluate each of the following Integrals (tan −1 x) dx


II. Short Answer Type Questions
18. ∫ (1 + x 2) 2
 [V. Imp.]

1. ∫ log x dx 2. ∫x
3
(log x) dx  1− x
∫  tan
−1
19. dx  [V. Imp.]
1+ x 
3. ∫( tan −1 x dx ) [Delhi 2008 (C)]
 x 
∫  sin
−1

∫( ) 20. dx  [V. Imp.]


4. x tan −1 x dx  [Delhi 2014 (C), A.I. 2006] x + a 

Integrals 209
E:\AMIT_WORKS\Exam_Guru\EG_Mathematics-12_(working_02-06-2022)\EG_Mathematics-12_working\Open_Files\Chap_7\Chap_7
\ 17-Aug-2022 Amit Proof 5 Reader’s Sign _______________________ Date __________

Answers 7.7
1 1 1
∫x
2
I. 1. (c) I =
3
e x dx = x log (log x) − ⋅x−∫ dx + ∫ dx
log x (log x) 2 (log x) 2
Put x2 = t, then 2x dx = dt
1 1 1 x2 2 x
\ I = ∫ te dt = [t e − e ] + C = e ( x − 1) + C
t t t = x log (log x) − +C
2 2 2 log x
II. 1. x log | x | – x + C;
2. (d) I = ∫x
3
(log x ) 2 dx
Hint: log x = (log x) × 1. Now integrate by parts.
1 4 1  1 4 4
= x ⋅ (log x ) 2 − ∫ ⋅ x 4 ⋅ 2 log x ⋅   dx 2. x (log x) − x + C;
4 4  x 4 16
x4 1 x 4 dx
∫ (log x ) x dx = (log x ) 4 − ∫ x ×
1 4 1 3
Hint:
= ⋅ x (log x ) 2 − ∫ x3 log x dx 4
4 2
4 4
1 11 1  1  = x ( log x ) − 1 × x + C
= x 4 (log x ) 2 −  x 4 log x − ∫ x 4 ⋅   dx  4 4 4
4 2 4 4  x  1
3. x(tan −1 x) − log (1 + x 2 ) + C;
1 4 1 1 2
x (log x ) 2 − x 4 log x + ∫ x 3dx
=
4 8 8 Hint: I = tan −1 x × 1 dx
∫ ( )
=
1 4 1
x (log x) 2 − x 4 log x +
1 4
x +C
(
= tan x x − ∫
−1
1 + x2
)
1 .x dx
4 8 32
1 2x dx
= x tan x − ∫
−1
1 4
= x [ 8 (log x) 2 − 4 log x + 1] + C 2 1 + x2
32
= x tan −1 x − 1 log 1 + x 2 + C
2 ( )
∫e
x
3. (c) I = sin 2 x dx
2
 x + 1
4.  (tan −1 x) − x + C;
Integrating by part, taking sin 2x as the Ist function, we get  2  2
I = sin 2 x ( e x ) − ∫ 2 cos 2 x ( e x ) dx
x 2 dx
∫ ( tan ) ( )
2
Hint: I = −1
x x dx = tan −1 x x − 1 ∫ 2

x
(
= e sin 2 x − 2 cos 2 x ⋅ e − ∫ − 2 sin 2 x ⋅ e dx
x x
) 2 2 x +1

= e sin 2 x − 2 e cos 2 x − 4∫ e sin 2 x dx


x x x
(
2  x 2 + 1 − 1
= tan −1 x x − 1 ∫  )  dx
( )
2 2  x2 + 1 
= ex sin 2x – 2ex cos 2x – 4I  
⇒ 5I = ex (sin 2x – 2cos 2x) + C x 2 tan −1 x − 1  − 1  dx
2 ∫  1 + x2 
= 1 
2
Comparing with KI = ex (sin 2x – 2cos 2x) + C, we get K = 5.
5. x sin −1 x + 1 − x 2 + C;
4. (a) I = ∫ tan
−1
x dx
∫ (sin ) ( ) x dx
Put x = tan2 q, then dx = 2 tan q sec2 q dq Hint: I = −1
x .1dx = sin −1 x x − ∫
1 − x2
I = 2∫ θ tan θ sec θ d θ
2
1 −2x dx
= x sin x + ∫
−1

Now, let tan q = z, then sec2 q dq = dz 2 1 − x2
 z2 1 z2 
I = 2∫ z tan −1 z dz = 2  tan −1 z − ∫ dz  = x sin −1 x + 1 × 2 1 − x 2 + C
 2 2 1 + z 2
 2
 z2 1 z2 + 1 − 1  6. x sec −1 x − log x + x 2 − 1 + C;
= 2  tan −1 z − ∫ dz 
2 2 1 + z2 
∫ (sec )
−1
Hint: I = x . 1 dx
= z2 tan–1 z – z + tan–1 z = (z2 + 1) tan–1 z – z


= q(1 + tan2 q) – tan q
I = tan −1
(
I = sec −1 x x − ∫ ) x x2 − 1
x dx
x (1 + x) − x + C
= x sec −1 x − ∫ dx
1 = x sec −1 x − log x + x 2 − 1 + C
5. (b) I = ∫ log (log x) dx + ∫ (log x) 2
dx x2 − 1
x3 cot −1 x x 2 1
Integrating first integral by parts, we get 7. + − log (x 2 + 1) + C;
3 6 6
1 1
I = x log (log x) − ∫ x dx + ∫
∫ (cot )
dx −1
x log x (log x) 2 Hint: I = x x 2 dx

210 Mathematics–12
E:\AMIT_WORKS\Exam_Guru\EG_Mathematics-12_(working_02-06-2022)\EG_Mathematics-12_working\Open_Files\Chap_7\Chap_7
\ 17-Aug-2022 Amit Proof-5 Reader’s Sign _______________________ Date __________

( )
2 x x x
x3 + 1 x3 dx = x a − 2xa 2 + ∫ 2a 2 dx
3 3 ∫ x2 + 1
= cot −1 x
log a ( log a ) (log a )
3
 
= x cot −1 x + 1 ∫  x − 1  22x   dx
2 x x x
= x a − 2xa 2 + 2a 3 + C
3 3  2  x + 1  log a ( log a ) ( log a )
3  2  3. x(tan x – sec x) – log | sec x | + log |sec x + tan x | + C
= x cot −1 x + 1  x − 1 log x 2 + 1  + C
3 3 2 2  x x (1 − sin x )
Hint: =
3 2 1+ sin x cos 2 x
= x cot −1 x + x − 1 log x 2 + 1 + C = x sec2 x – x sec x tan x
3 6 6

( )
8. x sec −1 x − x − 1 + C;
Now integrate two functions by parts.
2
(3x + 7) sin 2x 3 cos 2x
4. 3x + 7x − − + C;
∫ (sec ) 4 2 4 8
−1
Hint: I = x .1 dx
(3x + 7) (1 − cos 2x)
(
= sec −1 x x − ∫ ) x
2 x x x −1
dx
Hint: (3x + 7) sin 2 x =
2
1
= (3x + 7) − 12 (3x + 7)(cos 2x )
(
= x sec −1 x − 1 ∫ dx
2 x −1
) Now integrate two functions.
2

1  e xn  [x n − 1] + C
(
= x sec −1 x − 1 × 2 x − 1 + C
2 ) 5.
n 
2n −1 ( xy ) dx =
9. (x + 1) tan −1 x − x + C; Hint: ∫ ( x) e ∫x
n  xn 

e
 ( x ) dx
n −1

∫ ( tan )
−1
Hint: I = x dx Now substitute xn = z
dz
Put tan −1 x = θ ⇒ x = tan2 q ⇒ x­n – 1 dx =
n
⇒ dx = 2 tan q sec q dq
2
1
I = ∫ z e z dz
n
(
I = 2∫ θ × tan θ sec 2 θ d θ ) Now integrate by parts.
6. − 2x cos x + 4 x sin x + 4 cos x + C;
tan 2 θ − 2 tan 2 θ d θ
= 2θ
2 ∫ 2 Hint: I = ∫ x sin x dx
= θ tan 2 θ − ∫ sec 2 θ − 1 d θ ( ) Put x =z ⇒ dx = 2 x dz = 2z dz
= q tan q – tan q + q + C
2
I= ∫ z sin z × 2z dz
( )
= tan −1 x x − x + tan −1 x + C I = 2∫ z 2 sin z dz
= ( x + 1) tan
−1
x − x +C = 2  −z 2 cos z + ∫ 2z cos z dz 
10. x tan x + log | cos x | + C
{
= 2  −z 2 cos z + 2 z sin z − ∫ sin z dz 
  }
∫ x sec
2
Hint: I = x dx
= 2[–z2 cos z + 2z sin z + 2 cos z] + C
= x tan x − ∫ tan x dx = 2  −x cos x + 2 x sin x + 2 cos x  + C
= x tan x + log | cos x | + C
= −2x cos x + 4 x sin x + 4 cos x + C
III. I. (x2 + 1)ex + C
− log | x | x
7.
∫ ( x + 1)
2 x + log + C;
Hint: I = e dx x +1 x +1
I = ( x + 1) e − 2∫ ( x + 1) e dx
x 2 x
⇒ log x  1 
Hint: I = ∫ ( x + 1)2 dx = ∫ (log x )  ( x + 1)2  dx
I = ( x + 1) e x − 2 ( x + 1) e x − ∫ e x dx 
2
⇒  
Now integrate by parts
⇒ I = (x + 1)2ex­– 2(x + 1)ex + 2ex + C


I = [x2 + 2x + 1 – 2x – 2 + 2]ex + C
I = [x2 + 1]ex + C
I = − ( log x ( )∫(
) 1 +
x +1
dx
x x + 1)

) ∫( )
2 x
2x a x log x 1 − 1 dx
2. x a − ax
2 + 2 3 +C; ⇒ I= −
(x +1
+
x x +1
log a (log a) (log a)
log x
x 2a x 2xa x ⇒ I= − + log x + C
∫ x a dx = log a − ∫ log a dx
2 x
Hint: I = x +1 x +1

Integrals 211
E:\AMIT_WORKS\Exam_Guru\EG_Mathematics-12_(working_02-06-2022)\EG_Mathematics-12_working\Open_Files\Chap_7\Chap_7
\ 17-Aug-2022 Amit Proof 5 Reader’s Sign _______________________ Date __________

8. x log(x2 + 1) + 2 tan–1 x – 2x + C; 1  −θ cos 2θ + sin 2θ  C


= +
Hint: I = (
∫ log x + 1 × 1 dx
2
) 2  2 4 

= 1
(
 −θ 1 − 2 sin 2 θ )
2 
+ 2 sin θ 1 − sin θ  + C
( 2x 2
I = log x + 1 x − ∫ 2 dx
2
x +1
)
2 2 4 

(
I = x log x 2 + 1 − 2∫ ) ( x + 1) − 1 dx
2


 ( 2
1 2 sin θ − 1 θ + sin θ 1 − sin 2 θ 
= 
)
2
2 2 +C
x +1 2 

I = x log(x2 + 1) – 2[x – tan–1 x] + C
2 2 2 2
9. x log x + x + a − x + a + C;

1
4 ( )
=  2x 2 − 1 sin −1 x + x 1 − x 2  + C

Hint: I = (
∫ log x + x + a × 1 dx
2 2
) 13. x(sin −1 x) 2 + 2 (sin −1 x) 1 − x 2 − 2x + C;

∫ (sin )
−1 2
Hint: I = x dx . Put sin–1 x = q
⇒ ( 1
I = log x + x 2 + a 2 ( x ) − ∫
2
2x
x + a2
2 )
dx
⇒ x = sin q ⇒ dx = cos q dq

( ) ∫θ cos θ d θ = θ 2 sin θ − ∫ 2θ sin θ d θ


2
⇒ I = x log x + x 2 + a 2 − x 2 + a 2 + C I=

10. xex[log(xex) – 1] + C; I = θ 2 sin θ − 2  −θ sin θ + ∫ cos θ d θ 


 
Hint: I = ∫ ( x + 1) e
x
( )
log xe x dx ⇒ I = q2 sin q + 2q cos q – 2 sin q + C
( )
2
Put xex = z ⇒ I = sin −1 x x + 2 sin −1 x 1 − x 2 − 2x + C
⇒ (xex + ex)dx = dz
x2 1
⇒ (x + 1)ex dx = dz 14. (sec −1 x) − x 2 −1 + C;
2 2
⇒ I= ∫ (log z ) × 1 dz Hint: I = ∫ (sec
−1
)
x x dx
Integrate by parts
( x2 −
)
2

1
I = ( log z ) z − ∫ × z dz ⇒
−1
I = sec x ∫
1 × x dx
z 2 x x2 − 1 2
I = xex log(xex) – (xex) + C
( )
2
⇒ I = x sec −1 x − 1 ∫ 2x dx
 sin 2x  log  cosx + sin x  − 1 log sec 2x + C 2 4 x2 − 1
11. 
 2   cos x − sin x  2
( )
x 2 sec −1 x − 1  x 2 − 
( )
⇒ I= 2 1 +C
cos x + sin x
(cos 2x ) dx 2 4  
Hint: I = ∫ log cos x − sin x 2
I = x (sec −1 x ) − 1 x 2 − 1 + C
= log (
cos x + sin x sin 2x
cos x − sin x 2 )( )
2 2
3
x3 cos −1 x 2
− 1 − x + 1 (1 − x 2) 2 + C;
( 
−∫ cos x − sin x  )(
2  sin 2x

cos x + sin x  cos x − sin x )2  2
dx ( ) 15.
3 3 9
∫ (cos )
  −1
x x 2 dx
Hint: I =


= log
 (
cos x + sin x   sin 2x 
cos x − sin x   2  ) Substitute cos x = q

–1

dx = – sin q dq
⇒ x = cos q

sin 2x
−∫   dx
∫ θ cos θ ( − sin θ) d θ
2
 cos 2 x − sin 2 x  I=

( )
3 3
sin 2x log cos x + sin x = θ cos θ − ∫ cos θ d θ
cos x − sin x ∫
= − tan 2x dx θ 3
2
1 ( x 2 − ) sin −1 x + x − x 2  + C;
3
= θ cos θ −
1 − sin 2 θ cos θ ( )
12.
4 
2 1 1
 3 ∫ 3

∫ (sin )
−1 Put sin q = z ⇒ cos q dq = dz
Hint: I = x x dx
Put sin x = q
–1 =
θ cos3 θ − 1 1 − z 2 dz
3 3∫ ( )
⇒ x = sin q ⇒ dx = cos q dq 3 3

1 = θ cos θ − 1  z − z  + C
I = ∫ θ sin θ cos θ d θ = 2 ∫ θ sin 2θ d θ 3 3 3
3
=
1  θ cos 2θ
− +∫
cos 2θ d θ  = θ cos θ − sin θ + 1 sin 3 θ + C
2  2 2  3 3 9

212 Mathematics–12
E:\AMIT_WORKS\Exam_Guru\EG_Mathematics-12_(working_02-06-2022)\EG_Mathematics-12_working\Open_Files\Chap_7\Chap_7
\ 17-Aug-2022 Amit Proof-5 Reader’s Sign _______________________ Date __________

3 Put tan–1 x = q ⇒ x = tan q


( )
3
1
I = θ cos θ − 1 1 − cos 2 θ + 1 − cos 2 θ 2 +C ⇒ dx = sec2 q dq
3 3 9
θ
I= ∫ × sec 2 θ d θ
x3 cos −1 x 1 x2 1
− − + 1 − x2 ( ) (1 + tan θ)
3/ 2 2
2
= +C
3 3 9
(x 2 + 1) (tan −1 x) 2 θ sec2 θ
16. − x tan −1 x + 1 log (x 2 + 1) + C; I= ∫ dθ
2 2 sec4 θ

∫ ( tan )
2
−1 1 θ 1 + cos 2θ d θ
Hint: I = x x dx = ∫ θ cos 2 θ d θ =
2∫
( )
Substitute tan–1 x = q ⇒ x = tan q 2
⇒ dx = sec2 q dq = θ + 1 ∫ θ cos 2θ d θ
4 2
⇒ I=
∫θ
2
( tan θ sec θ) d θ 2
θ 2 1  θ sin 2θ sin 2θ d 
= + −∫ θ
2 2 4 2  2 2 
= θ 2 tan θ − ∫ 2θ tan θ d θ
2 2 2
= θ + θ sin 2θ + cos 2θ + C
( )
2 2
= θ tan θ − ∫ θ sec 2 θ − 1 d θ 4 4 8
2 2 2 

2 2
θ tan θ − θ sec2 θ d θ + θ d θ I = θ + θ tan θ2 + 1  1 − tan 2 θ 
4 2 1 + tan θ 8  1 + tan θ 
=
2 ∫ ∫
( tan x) −1 2
θ 2 tan 2 θ − θ tan θ − tan θ θ  θ 2 −1 1  1 − x2  + C
= ∫ d + +C I= + tan x  x 2  + 
2  2 4 2  1 + x  8  1 + x 2 
2 2 2
= θ tan θ − θ tan θ + log (sec θ ) + θ + C 19.
1
x cos −1 x − 1 − x 2  + C
2 2 2  

( ) ( )
2
= x tan −1 x − tan −1 x x
2
1 − x dx
∫ tan
−1
2 Hint: I =
1+ x
) ( )
2
tan −1 x
2 (
+ 1 log 1 + x 2 +
2
+C Put x = cos q
 
⇒ dx = –sin q dq

I = −∫  tan −1 1 − cos θ  (sin θ d θ )



 2 
 2 
2
( 1
)
=  x + 1 tan −1 x − x tan −1 x + log x 2 + 1 + C
2 ( )  1 + cos θ 
1 θ sin θ d θ
2∫
=−
−1 2
17. − sin x + log 1 − 1 − x + C;
x x = − 1 θ ( − cos θ ) + ∫ cos θ d θ 
2 
Hint: I =
 sin −1 x  −1  1 
∫  x 2  dx = ∫ sin x  x 2  dx ( ) = − 1 [ −θ cos θ + sin θ ] + C
2
1
Put sin–1 x = q ⇒ x = sin q = −  −θ cos θ + 1 − cos 2 θ  + C
2 
⇒ dx = cos q dq
1
 1  = −  −x cos −1 x + 1 − x 2  + C
I= ∫ (θ)  sin 2 θ  cos θ d θ 2 
−1
= ∫ θ ( cosec θ cot θ ) d θ = x cos x − 1 1 − x 2 + C
2 2
= θ ( − cosec θ ) + ∫ cosec θ d θ 
( )   
20. a  x + a  tan −1 x  − x  + C;
 a  a  a 
= −θ + log cosec θ − cot θ + C
sin θ x dx
∫ sin
−1
Hint: I =
−1 2 x+a
= − sin x + log 1 − 1 − x + C Substitute: x = a tan2 θ
x x
⇒ dx = 2a tan θ sec2 θ d θ
−1 2 −1 2
18. (tan x) − x tan 2x + (1 − x )2 + C; (
a tan 2 θ 2a tan θ sec 2 θ d θ
)
∫ sin
−1
4 I=
2 (1 + x ) 8 (1 + x ) a + a tan 2 θ
tan −1 x dx
∫ ( )
2
Hint: I = = ∫ sin −1 tan 2 θ 2a tan θ sec 2 θ d θ
( )
2
1 + x2 sec θ

Integrals 213
E:\AMIT_WORKS\Exam_Guru\EG_Mathematics-12_(working_02-06-2022)\EG_Mathematics-12_working\Open_Files\Chap_7\Chap_7
\ 17-Aug-2022 Amit Proof 5 Reader’s Sign _______________________ Date __________

(
= ∫ sin −1 (sin θ ) 2a tan θ sec2 θ d θ )
 2 
I = 2a  θ tan θ − 1 tan θ + 1 θ  + C
2 2 2
 
(
= 2a∫ θ tan θ sec 2 θ d θ ) = a[q tan2 q – tan q + q] + C
 2  2  
I = 2a θ tan θ − ∫ tan θ d θ  = a  x tan −1 x − x + tan −1 x  + C
 2 2   a a a a 

( ) 
( ) 
 2 
I = 2a  θ tan θ − 1 ∫ sec 2 θ − 1 d θ  + C = a  x + a tan −1 x − x  + C
 2 2   a a a

Topic 8. Repeating After Twice Integration


Example 1. Evaluate: ∫ (sin 4x) e
5x
dx (sin 4x) e5x
⇒ I=
5
∫ (sin 4x) e
5x
Solution. I = dx
4 e5x  e5x 
 −  (cos 4x) + ∫ (4 sin 4x) dx
5 5  5 
⇒ I = (sin 4x) ∫ e5x dx − ∫ (sin 4x)′ ∫ e5x dx  dx
(sin 4x) e5x 4  e5x 4 
5x 5x ⇒ I= −  (cos 4x) + I
⇒ I = (sin 4x) e − ∫ 4(cos 4x) e dx 5 5  5 5 
5 5
(sin 4x) e5x 4 16
5x
⇒ I= − (cos 4x) e5x − I
⇒ I = (sin 4x) e − 4 ∫ (cos 4x) e5x dx 5 25 25
5 5
5x
⇒ I=
(sin 4x) e5x ⇒ 41 I = e [5 (sin 4x) − 4 (cos 4x) ] + C
5 25 25
4 e5x
 − (cos 4x) ∫ e5xdx − ∫ (cos 4x)′ ∫ e5x dx  dx  ⇒ I= [5(sin 4x) − 4(cos 4x)] + C
5  41

EXERCISE 7.8
I. Multiple Choice Questions (MCQs)
3. ∫e
2x
⋅ sin x cos x dx =
1. ∫ e sin bx dx is equal to
ax
2x
(a) e (sin 2 x + cos 2 x) + C
e ax 8
(a) (a sin bx − b cos bx) + C 2x
a + b2
2 e (cos 2 x − sin 2 x)
(b) +C
8
e ax
(b) (a sin bx − b cos bx) + C e 2 x (sin 2 x − cos 2 x)
a − b2
2
(c) +C
8
e ax (d) None of these
(c) (a sin bx − b cos bx) + C
b2 − a 2
Evaluate each of the following Integrals
e ax
(d) (a sin bx + b cos bx) + C
a + b2
2 II. Long Answer Type Questions

∫ (sin mx) e dx
nx
1.
∫e
ax
2. cos (bx + c) dx equals

∫ (cos mx) e dx
nx
2.
e ax [a cos (bx + c) − b sin (bx + c)]
(a) + C1
∫ sec x dx 
3
(a 2 + b 2 ) 3. [V. Imp.]
ax
e [a cos (bx + c) + b sin (bx + c)]
(b)
(a 2 + b 2 )
+ C1 4. ∫ sin (log x) dx
(cos 2x) dx
(c)
e ax [a cos (bx + c) + b sin (bx + c)]
+ C1
5. ∫ sin 3 x
(a 2 − b 2 )
6. ∫ e .sin zx dx  [CBSE 2022]
x
(d) None of these

214 Mathematics–12
E:\AMIT_WORKS\Exam_Guru\EG_Mathematics-12_(working_02-06-2022)\EG_Mathematics-12_working\Open_Files\Chap_7\Chap_7
\ 17-Aug-2022 Amit Proof-5 Reader’s Sign _______________________ Date __________

Answers 7.8
1 2x
∫e
ax
I. 1. (a) I= sin bx dx
∫e 2∫
2x
3. (c) I= ⋅ sin x cos x dx = e (2 sin x cos x) dx
Integrating by parts, we get
1 2x 1
2∫
 − cos bx   cos bx  ⇒ I= e sin 2 x dx = I1
 b  ∫
I = e ax  − ae ax ⋅  −  dx 2
 b 
∫e
2x
Let I1 = sin 2 x dx
1 a
= − e ax cos bx + ∫ e ax cos bx dx
b b
2 x  − cos 2 x  2 x  − cos 2 x 

2  ∫
1 a sin bx sin bx  =e   − 2e ⋅   dx
 2 
= − e ax cos bx + e ax ⋅ − ∫ ae ax ⋅ dx 
b b b b  e 2 x cos 2 x
1 a a 2 =− + ∫ e 2 x .cos 2 x dx
= − e ax cos bx + 2 e ax sin bx − 2 ∫ e ax sin bx dx 2
b b b e 2 x ⋅ cos 2 x 2 x  sin 2 x  sin 2 x
 2  ∫
1 a2 =− +e  − 2e 2 x ⋅ dx
= 2 e ax ( a sin bx − b cos bx) − 2 I 2 2
b b
e 2 x cos 2 x 2 x sin 2 x
a 2
1 ax =− +e ⋅ − ∫ e 2 x sin 2 x dx
⇒ I+ I = 2 e ( a sin bx − b cos bx) 2 2
b2 b
e 2 x (sin 2 x − cos 2 x)
 a 2 + b2  1 = − I1
⇒ I  = 2 e ax (a sin bx − b cos bx) 2
 b 2 
 b
e 2 x (sin 2 x − cos 2 x)
ax \ I1 = − I1
e 2
\ I= (a sin bx − b cos bx) + C
a 2 + b2 e 2 x (sin 2 x − cos 2 x)
⇒ 2I1 =
∫e
ax
2. (b) I = cos(bx + c) dx 2
e 2 x (sin 2 x − cos 2 x)
Integrating by parts, we get ⇒ I1 =
4
sin (bx + c) sin (bx + c)
I = e ax ⋅ − ∫ ae ax ⋅ dx 1 e 2 x (sin 2 x − cos 2 x)
b b Hence, I= I1 = +C
2 8
e ax sin (bx + c) a ax
= − ∫ e sin (bx + c) dx enx  n sin mx − m cos mx  + C;
b b II. 1. ( ) ( )
m + n2 
2
e ax sin (bx + c) a ax a2
= + 2 e cos(bx + c) − 2 ∫ e ax cos(bx + c) dx
∫ (sin mx ) e
nx
b b b Hint: I = dx

 a 2  ax [b sin (bx + c) + a cos(bx + c)] e ax enx − m cos mx enx dx


= sin ( mx ) ( )n
n ∫
i.e., 1 + 2  ∫ e cos(bx + c) dx =
 b  b2
1 m
e ax [a cos(bx + c) + b sin (bx + c)] I = sin ( mx ) enx − ∫ cos ( mx ) enxdx
Thus, ∫ e ax cos(bx + c) dx = + C1 n n
(a 2 + b 2 )  nx

 ...(1) = 1 sin ( mx ) enx − m cos ( mx ) e


n n n
nx 
In order to express this result in a convenient form, we substitute + ∫ m sin ( mx ) e dx 
a = r cos q and b = r sin q n 
b 1 m 1
So that r= a 2 + b 2 andθ = tan −1 = sin ( mx ) enx −  cos ( mx ) enx + m I 
a n n  n n 
We have a cos (bx + c) + b sin (bx + c) 1 m m2
= sin ( mx ) enx − 2 cos ( mx ) enx − 2 I
n n n
= r [cos (bx + c) cos q + sin (bx + c) sin q]
= r cos (bx + c – q)  m2  1
1 + n 2  I = n 2  n sin ( mx ) − m cos ( mx ) e
nx

 b
= a 2 + b 2 cos (bx + c) − tan −1 
 a enx  n sin mx − m cos mx  + C
I= ( ) ( )
m + n2 
2
Hence, (1) is transformed into
b enx  n cos mx + n sin mx  + C;

e ax cos  bx + c − tan −1  2. ( ) ( )
 a m + n2 
2

∫e
ax
cos (bx + c) dx = + k.
a ² + b² Hint: Solve as we have solved Q.No. 1.

Integrals 215
E:\AMIT_WORKS\Exam_Guru\EG_Mathematics-12_(working_02-06-2022)\EG_Mathematics-12_working\Open_Files\Chap_7\Chap_7
\ 17-Aug-2022 Amit Proof 5 Reader’s Sign _______________________ Date __________

3. 1 sec x tan x + log sec x + tan x  + C; 5. − 1 [ cosec x cot x + 3 log | cosec x − cot x |] + C;
2 2

Hint: I = ∫ sec
3
(
x dx = ∫ sec x sec2 x dx ) Hint: I =  cos 2x   1 − 2 sin 2 x 
∫  sin 3 x  dx = ∫  sin 3 x 
dx

= sec x tan x − ∫ sec x tan x tan x dx = ∫ cosec3 x dx − 2∫ cosec x dx

⇒ (
I = sec x tan x − ∫ sec x sec2 x − 1 dx ) I = I1 – 2 log|cosec x – cot x| ...(1)

⇒ I = sec x tan x − ∫ sec x dx + ∫ sec x dx


3 I1 = ∫ cosec
3
(
x dx = ∫ cosec x cosec2 x dx )
⇒ I = sec x tan x – I + log(sec x + tan x) ⇒ I1 = – cosec x cot x + ∫ ( cosec x cot x )( − cot x ) dx

⇒ I=
1
2
sec x tan x + log sec x + tan x  + C ⇒ (
I1 = – cosec x cot x −∫ ( cosec x ) cosec 2 x − 1 dx )
4. x [sin (log x) − cos (log x) ] + C; ⇒ I1 = – cosec x cot x −∫ cosec3 x dx + ∫ cosec x dx
2
⇒ I1 = – cosec x cot x – I1 + log|cosec x – cot x|
Hint: I = ∫ sin (log x ) dx = ∫ sin x (log x ) × 1 dx
From eqn (1), we have
cos ( log x )
I = sin ( log x )( x ) − ∫ × x dx ⇒ I = I1 – 2 log | cosec x – cot x |
x
⇒ I = x sin ( log x ) − ∫ cos ( log x ).1 dx 1 1
I = − cosec x cot x + log cosec x − cot x
2 2
sin ( log x ) 
⇒ I = x sin ( log x ) − cos ( log x )( x ) + ∫ x dx  – 2 log|cosec x – cot x|
x  ex
⇒ I = x sin (log x) – x cos (log x) – I 6. [sin 2 x − 2 cos 2 x] + C
5
x sin log x − cos log x  + C
⇒ I=
2
( ) ( ) Hint: Put n = 1 and m = 2 in sol no. 1

Topic 9. Integrals of the Exponential Form


x+3 Integrate only the first part:
Example 1. ∫ ( x + 4)2 .e x dx = .

x+3 x x + 4 −1 x

 (
I =  f ( x ) ∫ e x dx − ∫ f ′ ( x ) ∫ e x dx dx 
 )
Solution. Let I = ∫ . e dx = ∫ . e dx
( x + 4) 2 ( x + 4) 2 +∫ f ′ ( x ) e xdx
 x+4 1  x
=f ( x ) e x − ∫ f ′ ( x ) e xdx + ∫ f ′ ( x ) e x dx
∫
= 2

 ( x + 4) ( x + 4) 2 
e dx

 1 1  =f (x)ex + C
∫  x + 4 − ( x + 4)2  e x dx
=
xe xdx 
1
Example 3. Evaluate: ∫ (x + 1)2 [V. Imp.]
1
Put =t  − dx = dt
x+4 ( x + 4) 2 xe xdx [(x + 1) − 1] e x dx
1 1
Solution. I = ∫ (x + 1)2 ∫ (x + 1)2
=
Let f(x) = \ f (x) = −
x+4 ( x + 4) 2
 1 1 
∫  x + 1 − (x + 1)2  e dx
x
⇒ I=
Using ∫ e x [ f ( x) + f ′ ( x)] dx = ex f(x) + C
x 1
\ I= e . +C  1  ex
∫  x + 1 e dx − ∫ (x + 1)2 dx
x
x+4 ⇒ I=
ex
Hence, I= + C. Integrate the first part only:
x+4
 ′ 
 1  e x dx
∫  f ( x ) + f ′ ( x ) e dx 1
x
Example 2. Evaluate: e xdx − ∫  e x dx  dx − ∫
I=
x +1 ∫ 
  x + 1)  ∫  (x + 1) 2
 
∫  f ( x ) + f ′ ( x ) e dx
x
Solution. I=
 1  x e x dx e x dx ex
= x + 1 e + ∫ 2 −∫ 2 = (x + 1)
+C
= f ( x ) e dx +
∫ ∫ f ′ ( x) e
x x
dx (x + 1) (x + 1)

216 Mathematics–12
E:\AMIT_WORKS\Exam_Guru\EG_Mathematics-12_(working_02-06-2022)\EG_Mathematics-12_working\Open_Files\Chap_7\Chap_7
\ 17-Aug-2022 Amit Proof-5 Reader’s Sign _______________________ Date __________

EXERCISE 7.9
I. Multiple Choice Questions (MCQs)
 sin 4x − 4 
∫ 1 − cos 4x  e
x
9. dx  [Delhi 2010]
1. ∫ e (1 − cot x + cot x ) dx =
x 2

(a) ex cot x + C (b) –ex cot x + C


 (2 − sin x)  2x
(c) ex cosec x + C (d) –ex cosec x + C 10. ∫  1 − cos x  e dx  [V. Imp.]

( x + 3) e x
2. ∫ ( x + 4) 2
dx =  [NCERT Exemplar]
11. ∫
(x − sin x) dx
1 − cos x
ex ex
(a) + C (b) +C  x + sin x 
x+4 x+3 12. ∫  1 + cos x  dx
1 ex
(c) + C (d) +C
( x + 4) 2 ( x + 4) 2  (2 − 5 sin 2x)  e5xdx
2
13. ∫  1 − cos 2x 
 log x − 1 
3. ∫  1 + ( log x )2  dx is equal to  1 
14. ∫ log (log x) + (log x)2  dx  [V. Imp.] [Delhi 2010 (C)]
x xe x
(a) +C (b) +C
( log x ) 2 + 1 1 + x2  1 + sin 2x 
∫  1 − cos 2x  e
−x
15. dx  [V. Imp.]
x log x  
(c) + C (d) +C
x2 + 1 ( log x ) 2 + 1
1 − sin x  − x
Evaluate each of the following Integrals 16. ∫  e  dx 
2 [V. Imp.] [A.I. 2013 (C)]
1 + cos x  
II. Short Answer Type Questions-I
∫ (sin x) a
x
(x − 3) e x dx 17. dx
1. ∫ (x + 1)3  [Delhi 2009]
2 x
III. Short Answer Type Questions-II
2. ∫ (x + 1) e 2 dx 
(x + 1)
[Delhi 2005, 2006 (C)
( x cos x) dx 
−1

(x − 4) e x dx
1. ∫ 1 − x2
[A.I. 2014 (C)]
3. ∫  [Delhi 2008]
(x − 2) 2
(1 − x) 2 e x dx 2.
(
x 2 + 1 log x 2 + 1 − 2 log x  dx
  )
[ A.I. 2011, 2014 (C)]
4. ∫ ∫ x4
(x 2 + 1) 2
 1 − sin x  x
2 3. ∫  e dx
 x + 2 x  1 − cos x 
5. ∫  x + 4  e dx
IV. Long Answer Type Questions
(5x + 14) e5x dx
6. ∫
(x + 3) 2 (sin −1
x − cos −1 x dx )
1. ∫ (sin −1
x + cos −1
x )
7. ∫ [sin (log x) + cos (log x)] dx  [V. Imp.]
x dx 2
2. ∫  [Delhi 2012]
8. ∫ 3x tan x + x sec x  dx
2 3 2
( x sin x + cos x )2
Answers 7.9
∫e ( x + 3) e x ( x + 4 − 1) x
x
I. 1. (b) I = (1 − cot x + cot 2 x) dx
2. (a) I = ∫ ( x + 4) 2
dx = ∫
( x + 4) 2
e dx

= ∫ e {(1+ cot x ) − cot x} dx


x 2
 1 1  x 1 x
= ∫ − 2
e dx = e +C
 x + 4 ( x + 4)  x + 4
= ∫ e { cosec x + ( − cot x )}dx
x 2

Q e x [ f ( x) + f ′ ( x)] dx = e x f ( x) 
  ∫ 

x
{
= − ∫ e cot x + −cosec x dx ( 2
)}
2
 log x − 1  (log x − 1) 2
= –e cot x + C
x Q e { f ( x) + f ′ ( x)} dx = e f ( x) 
 ∫
x x

3. (a) I = ∫  1 + (log x )2 
  dx = ∫ {1 + (log x )2 }2 dx

Integrals 217
E:\AMIT_WORKS\Exam_Guru\EG_Mathematics-12_(working_02-06-2022)\EG_Mathematics-12_working\Open_Files\Chap_7\Chap_7
\ 17-Aug-2022 Amit Proof 5 Reader’s Sign _______________________ Date __________

Put log x = t, then et = x ⇒ et dt = dx


   
=  x  ex +  4 2  ex
 1 2t  (x + 4 ) (x + 4)
\ I = ∫e     
t
2
− 2 2 
dt
 1 + t {1 + (t )}  Integrate only the first part.
1
It is of the form ∫ et [ f (t ) + f ′ (t )] dt , where f (t ) = . e5x + C;
1 + t2 6. Ans.
x+3
1 x

t
= e f (t ) + C = e
t
+C= +C (5x + 14) e5x  5(x + 3) − 1 5x
1 + t2 1 + (log x ) 2 Hint: = 2 e
(x + 3) 2  (x + 3) 
ex
II. 1. Ans.
( x − 1)2
x
+C;

x
=5 ( )
1 e5x −  1  e5x
x+3  (x + 3) 2 

Hint: (x − 3) e3 = [(x − 1) − 23 ] e Integrate only the first part.


(x − 1) (x − 1)
7. Ans. x sin (log x) + C;
 2  ex
= 1 2 −  Hint: Integrate only the first part.
 (x − 1) (x − 1)3  8. Ans. x3 tan x + C;

 1  x  2  x Hint: Integrate only the first part.
= 2 e −  3 e 9. Ans. [cot 2x] ex + C;
 (x − 1)   ( x − 1) 
Hint:
Integrate only the first part.
 sin 4x − 4  x  
e =  2 sin 2x cos 2x − 4  e x
( )

2. Ans. x − 1 e x + C;  1 − cos 4x  2 sin 2
2x
x +1  
= [cot 2x]ex – [2 cosec2 2x]ex
 x 2 + 1  x  (x 2 − 1) + 2  x Integrate only the first part.
Hint:  2 e =  2 e
 ( x + 1)   (x + 1) 
( )
x
10. Ans. − 2 cot x e 2 + C;
2
   
=  x − 1  ex +  2 2  ex  x x
 (x + 1 )   (x + 1)   2 − sin x  2x  2 − 2 sin 2 cos 2  2x
Hint:  e = e
 1 − cos x  
2 sin 2 x 
Integrate only the first part.
 2 

( )
 x  x x
3. Ans.  e  + C; =  − cot x  e 2 + cosec 2 x e 2
 x − 2  2  2

 x − 4  x  (x − 2) − 2  x Now integrate the first part only.
Hint:  2 e =  2 e x
 (x − 2)   (x − 2)  11. Ans. − x cot + C ;
2
 1 2  e x. x − 2 sin x cos x

x − 2
− 
(x − 2) 2 
Integrate only the first part. Hint: x − sin x = 2 2
 1 − cos x 2x
2 sin
x 2
4. Ans.  2e  + C;
 x + 1 = 1 x cosec2 x − cot x
2 2 2
 (1 − x) 2  x  (x 2+ 1 − 2x)  x
Hint:  2 2
e = 2 2 e Integrate only the first part.
 (x + 1)   (x + 1) 
12. Ans. x tan x + C; Hint : x + sin x
 2    2 1 + cos x
=  x + 1 2  e x −  2x 2  e x
x + 2 sin x cos x x
( ) ( ) ( )
 2   2 
 x + 1   x + 1  = 2 2 = sec2 x + tan x
 2 x 2 2 2
2 cos
 x  2
=  21  e x −  22 2  e x
 x + 1  (x + 1)  Integrate only the first part.

Integrate only the first part. 13. Ans. –(cot x)e5x + C;
   2 − 10 sin x cos x  5x
Hint:  2 − 5 sin 2x  e5x = 
x
5. Ans. xe + C; 2 sin 2 x
e
x+4  1 − cos 2x   

( )
2
x + 2 e x =  x 2 + 4x + 4  e x = – [5 cot x]e5x + [cosec2 x]e5x
Hint:  2 
x+4  (x + 4)  Now integrate only the first part.

218 Mathematics–12
E:\AMIT_WORKS\Exam_Guru\EG_Mathematics-12_(working_02-06-2022)\EG_Mathematics-12_working\Open_Files\Chap_7\Chap_7
\ 17-Aug-2022 Amit Proof-5 Reader’s Sign _______________________ Date __________

⇒ dx = –sin q dq
 
14. Ans. [x log (log x)] −  x  + C;
 log x  (cos θ) θ ( − sin θ) d θ = (cos θ) θ ( − sin θ) d θ = − θ cos θ d θ
∫ 2 ∫ sin θ ∫
Hint: Integrate only the first part twice. 1 − cos θ

−1 Now, integrate by parts


15. Ans. (cosec x) e −x + C;
2
3
2  
 1 + sin 2x  −x  sin x + cos x  −x 2. Ans. −1 1 + 12  3 log 1 + 12  − 2 + C;
e = 9  x    x  
Hint:  2 e
 1 − cos 2x   2 sin x 
1 x 2 + 1 log (x 2 + 1) − 2 log x 
=
2
[cosec x ] e−x + 12 (cosec x cot x) e− x  + C; Hint:
x4

Integrate only the first part.  2 


x 2 + 1 log  x +2 1
 x  = x 2 + 1 log  x 2 + 1 1
( )
−x
=  x 2  x3
16. Ans. − sec x e 2 + C; x 4 x2
2
 cos x − sin x  −x
 1 − sin x  −x 2 2 e 2 = 1 + 12 log 1 + 12  13
Hint:  e 2 =  x  x x
  2 cos 2 x 
 1 + cos x 
 2 
Substitute: 1 + 12 = z

( ) ( )
−x −x x
= 1 sec x e2 − 1 sec x tan x e2
2 2 2

Integrate only the first part.


2 2
( )
3. − cot x e x + C;
2

 1 − 2 sin x cos x 
17. ax
1 + ( log a )
( log a ) sin x − cos x  + C
2  Hint: 1 − sin x
1 − cos x(e =
x

 2
)
sin
2
2x
2  ex


2
Hint: I = ∫ (sin x ) a dx
( ) ( )
x
1 x x
= cosec2 e x − cot e x
x x 2 2 2
 
I = (sin x ) a − ∫ ( cos x )  a  dx

log a  log a 
( )
Now integrate − cot x e x by parts
2
 ax  1 cos x  a x  + sin x  a x  dx 
I = (sin x )  −  log a  ∫ ( )  log a  
 log a  log a 

 IV. 1. Ans. 2 ( 2x − 1) sin −1 x + x 1 − x  − x + C;
π 

 ax   ax  −1 −1
I = (sin x )  
 log a 
− ( cos x )  2 −
1
 ( log a )  ( log a )
( )
2 ∫ (sin x ) a dx
x Hint: sin −1 x − cos −1 x
sin x + cos x

I= a x  log a sin x − cos x  − 1 ( I )


(log a )2 
( ) 
(log a )2 = 2 (
sin −1 x − π − sin −1 x )
= 4 sin −1 x − 1
π π
2
 1  I = a x  log a sin x − cos x 
1 + 2 2 ( )  Now substitute: sin −1 x = θ ⇒ x = sin 2 θ
 ( log a )  ( log a )

 −x sec x 
2. Ans.   + tan x + C;
1 + ( log a )2  a x  log a sin x − cos x   x sin x + cos x 
 2 I = ( )
 ( log a ) 

(log a )2  
Hint: x2 =
(x sec x) (x cos x)
ax [x sin x + cos x]2 (x sin x + cos x) 2
I= ( log a ) sin x − cos x 
2
1 + ( log a )
 
x cos x
= ( x sec x )  2
III. 1. Ans. − (cos −1 x) 1 − x 2 + x  + C;  ( x sin x + cos x ) 
 
Hint: Substitute: cos–1 x = q ⇒ x = cos q Now integrate by parts.

Integrals 219
E:\AMIT_WORKS\Exam_Guru\EG_Mathematics-12_(working_02-06-2022)\EG_Mathematics-12_working\Open_Files\Chap_7\Chap_7
\ 17-Aug-2022 Amit Proof 5 Reader’s Sign _______________________ Date __________

Topic 10. Three More Formulae for Integration


First Formula
Solution. I= ∫ x 2 + 4x − 7 dx = ∫ (x + 2) 2 − 11 dx
a2 x 2 −11
1A. ∫ x 2 + a 2 dx =
2
log x + x 2 + a 2 +
2
x + a2 + C =
2
log (x + 2) + (x + 2) 2 − 11

x+2
a2 + (x + 2) 2 − 11 + C
1B. ∫ (x ± b) 2 + a 2 dx =
2
log (x ± b) + (x ± b) 2 + a 2 2
− 11
x ± b)  ⇒ I= log (x + 2) + x 2 + 4x − 7 +
+   (x ± b) 2 + a 2 + C 2
 2  x+2
 x 2 + 4x − 7 + C
2
Example 1. Evaluate: ∫ x 2 + 6x + 11 dx Third Formula

Solution.I = ∫ x 2 + 6x + 11 dx    ⇒ I = ∫ ( x + 3) + 2 dx
2
3A. ∫ a 2 − x 2 dx =
a2
2
sin −1 +()
x x
a 2
a2 − x2 + C

2
=log ( x + 3) +
2
( x + 3)2 + 2 +
x+3
2
( x + 3)2 + 2 + C 3B. ∫ a 2 − (x ± b) 2 dx =
a2
2
sin −1
a( )
x±b
+

=log ( x + 3) + x 2 + 6x + 11 +
x+3 2
2
x + 6x + 11 + C ( )
x±b
2
a 2 − (x ± b) 2 + C

Second Formula
Example 3. Evaluate: ∫ 11 + 6x − x 2 dx .
−a 2
2A. ∫ x 2 − a 2 dx =
2
log x + x 2 − a 2 Solution. I = ∫ 11 + 6x − x 2 dx
x
+ x2 − a2 + C −  x 2 − 6x − 11 dx = ∫ − ( x − 3) − 20 dx
2
2 ⇒ I= ∫  

2B. − a2 =∫ 20 − (x − 3) 2 dx
∫ (x ± b) 2 − a 2 dx =
2
log

(x ± b) + (x ± b) 2 − a 2 + ( )
x±b
2
(x ± b) 2 − a 2 + C
2

 20 

=20 sin −1  x − 3  + x − 3 20 − ( x − 3) + C
2
2

 
=10 sin −1  x − 3  + x − 3 11 + 6x − x 2 + C
Example 2. Evaluate: ∫ x 2 + 4x − 7 dx  20  2

EXERCISE 7.10
I. Multiple Choice Questions (MCQs) 1
(b) ( x + 4 ) x2 − 8x + 7 +
1. ∫ 1 + x 2 dx is equal to 2
x 1  9 log | x + 4 + x2 − 8x + 7 | + C
(a) 1 + x 2 + log x + 1 + x 2 + C
2 2
2 3
1
(b) (1 + x 2 ) 2 + C (c) ( x − 4 ) x2 − 8x + 7 −
3 2
3
2
(c) x (1 + x 2 ) 2 + C  3 2 log x − 4 + x2 − 8x + 7 + C
3
x2 x2
(d) 1 + x2 + log | x + 1 + x 2 | + C 1
2 2 (d) ( x − 4 ) x2 − 8x + 7 −
2
2. ∫ x 2 − 8 x + 7 dx is equal to
9
1 log x − 4 + x 2 − 8 x + 7 + C [NCERT]
(a) ( x − 4 ) x 2 − 8 x + 7 + 2
2
 9 log | x − 4 + x 2 − 8 x + 7 | + C 3. ∫ 1 − 4 x − x 2 dx equals

220 Mathematics–12
E:\AMIT_WORKS\Exam_Guru\EG_Mathematics-12_(working_02-06-2022)\EG_Mathematics-12_working\Open_Files\Chap_7\Chap_7
\ 17-Aug-2022 Amit Proof-5 Reader’s Sign _______________________ Date __________

x+2 5  x + 2 log x
(a) 1 − 4 x − x 2 + sin −1 (b) 16 + (log x) 2 −
2 2   +C 3
5 
x−2 5  x + 2  8 log log x + 16 + (log x) 2 + C
(b) 1 − 4 x − x 2 + sin −1   +C
2 2 5  log x
(c) 16 + (log x) 2 +
x +1 5  x + 2 2
(c) 1 − 4 x − x 2 − sin −1  +C
3 2  5   8 log log x + 16 + (log x) 2 + C

(d) None of these (d) None of these


Evaluate each of the following Integrals
16 + (log x) 2 II. Long Answer Type Questions
4. ∫ x
dx equals
1. ∫ x 2 + 4x + 7 dx
log x
(a) 16 + (log x) 2 −
2 2. ∫ x 2 + 2x − 5 dx

 8 log log x + 16 + (log x) 2 + C 3. ∫ 1 + 3x − x 2 dx

Answers 7.10
3 log x + 2 +
I. 1. (a) I = ∫ 1 + x dx = ∫ 1 + x dx
2 2 2
⇒ I=
2
( ) ( x + 2)2 + 3
 
( )(
x 2 a2
  Form ∫ a + x =
2 2
a + x2 + log | x + a 2 + x 2 | + C + x+2 x + 2) + 3 + C
2
 2 2  2
x 1
\ I= ∫ 1 + x 2 dx =
2
1 + x 2 + log| x + 1 + x 2 | + C
2 3 log x + 2 + x 2 + 4x + 7
⇒ I=
2
( )
2. (d) I= ∫ x 2 − 8 x + 7 dx = ∫ ( x 2 − 8 x + 16 ) + ( 7 − 16 ) dx

= ∫ ( x − 4 ) − 9 dx = ∫ ( x − 4 ) − 3 dx
2 2 2
( )
+ x+2
2
x 2 + 4x + 7 + C

x + 1 x 2 + 2x − 5 + C
Put x – 4 = t , then dx = dt so I = ∫ t − 3 dt
2 2
2. − 3 log (x + 1) + x 2 + 2x − 5 +
2

 x x2 − a2 a2 
  Form ∫ x 2 − a 2 dx = − log| x + x 2 − a 2 | + C Hint: I = ∫ x 2 + 2x − 5 dx = ∫ ( x + 1)2 − 6 dx
 2 2 

t t 2 − 32 32
\ I= − log t + t 2 − 32 + C ⇒ I = −3 log ( x + 1) + ( x + 1)2 − 6
2 2

=
( x − 4) x 2 − 8 x + 7
2

2( )(
+ x +1
2
x + 1) − 6 + C

9
log ( x − 4 ) + x2 − 8x + 7 + C  
3. 13 sin −1  2x − 3  + 2x − 3 1 + 3x − x 2 + C

2
8  13  4
x+2 5  x + 2
3. (a) 1 − 4 x − x 2 + sin −1  +C
2 2  5  Hint: I = ∫ 1 + 3x − x 2 dx = ∫ −  x 2 − 3x − 1 dx

log x
4. (c) 16 + (log x) 2 + 8 log log x + 16 + (log x) 2 + C
2
( ) ( )
 2  2
3
I = ∫ − x − − 13  dx = ∫
13 − x − 3 dx

( )  2 4 4 2
3 log x + 2 + x 2 + 4x + 7 + x + 2
II. 1.
2
( ) 2
x 2 + 4x + 7 + C

( ) ( )
2
  2x − 3 13 − x − 3
⇒ I = 13 sin −1  2x − 3  + +C
Hint: I = ∫ x + 4x + 7 dx = ∫
2
( x + 2)
2
+ 3 dx 8  13  4 4 2

Integrals 221
E:\AMIT_WORKS\Exam_Guru\EG_Mathematics-12_(working_02-06-2022)\EG_Mathematics-12_working\Open_Files\Chap_7\Chap_7
\ 17-Aug-2022 Amit Proof 5 Reader’s Sign _______________________ Date __________

II. DEFINITE INTEGRALS

Topic 11. Direct Evaluation of Definite Integrals


b π/4
dx
For finding ∫ f ( x ) dx we proceed as follows: Example 3. ∫
−π/4 1 + cos 2x
is equal to
a

(i) Find the indefinite integral ∫ f ( x ) dx . Let this be F(x). (a) 1 (b) 2 (c) 3 (d) 4
There is no need to keep integration constant C because if π/4
dx π/4
dx
we consider F(x) + C instead of F(x), we get
Solution. Let I = ∫ = ∫
− π / 4 1 + cos 2 x
2
−π/4 2 cos x
b π/4
b 1 1
∫ f ( x ) dx =  F ( x ) + C a = [F(b) + C] – [F(a) + C] sec 2 x dx = [ tan x ]− π/4
π/4
2 − π∫/ 4
=
a
2
= F(b) – F(a).
1 π  π  1 1
Thus, the arbitrary constant disappears in evaluating the =  tan − tan  −   = [1 + 1] = × 2 = 1
2 4 4  2 2
value of the definite integral. Hence, the correct option is (a).
b
(ii) Evaluate F(b) – F(a) =  F ( x ) , which is the value of π/4

∫ ( 2 tan )
a 3
b Example 4. Evaluate: x dx
∫ f ( x ) dx . 0

a π/4 π/4

∫ ( 2 tan x ) dx = ∫ 2 tan x tan


π/2
3 2
Example 1. ∫ cos x . esin x dx is equal to . Solution. x dx
0 0 0
π/2
Solution. Let I = ∫ cos x . esin x dx

π/4
=∫ 2 tan x (sec 2 x − 1) dx
0
Put sin x = t  cos x dx = dt
0
When x = 0 then t = sin 0 = 0; π/4
π π =∫ 2 tan x sec 2 x dx
When x = then t = sin = 1
2 2 0
1 π/4
\ I= ∫ et dt = [et ]10 = (e1 − e0 ) = e − 1  − ∫ 2 tan x dx = I1 − I 2
0
0
Hence, I = e – 1.
∫ 2 tan x sec
2
a
Consider x dx
1 π
Example 2. If ∫ 1 + 4 x 2 dx = 8 then a = ..
Put tan x = t ⇒ sec2x dx = dt
0
a
1 π
Solution. Given that: ∫ 1 + 4 x 2 dx = 8  ∫ 2 tan x sec 2 x dx = tan2 x
0
π/4
a

1

1
dx =
π Now I1 = ∫ 2 tan x sec 2 x dx
4 01 2 8 0
 + x 
4  tan 2 x 
π/4 π
a = 0 = tan 2 − tan 2 0 = 1
π1 4
 ∫  1 2  dx = 2 ⇒ I1 = 1
0  
  + x 2 
 2  π/4
2 tan x dx = 2 [ log | sec x |]0
π/4
1  −1 x  π
a I2 = ∫
  tan 1/ 2  = 2 0
1/ 2

 2[tan–1 2a – tan–1 0] =
π
0
= π
{
2 log sec − log | sec 0 |
4 }
2 =2 log 2 − log 1
π π
 tan 2a =
–1
 2a = tan 1
4 4 =2 × log 2 − 0 = log 2
1 2
 2a = 1  a = π/4

∫ ( 2 tan )
2 3
1 Hence, x dx = I1 – I2 = 1 – log 2.
Hence, the value of a = .
2 0

222 Mathematics–12
E:\AMIT_WORKS\Exam_Guru\EG_Mathematics-12_(working_02-06-2022)\EG_Mathematics-12_working\Open_Files\Chap_7\Chap_7
\ 17-Aug-2022 Amit Proof-5 Reader’s Sign _______________________ Date __________

EXERCISE 7.11
I. Multiple Choice Questions (MCQs) π/2
π/4 13. ∫ (sin 2x)  tan −1 (sin x)  dx
1. ∫ tan x dx is equal to
2
0
0
1
π π π π
∫ (sin )
−1
(a) −1 (b) 1 − (c) 1 + (d) −1 14. x dx
2 4 4 8 0
3
1 e2
 
2. ∫0 1 + x 2 dx is equal to 15. 1 1
∫  (log x) − (log x)2  dx
e
π π π π
(a) (b) (c) (d) 1
12 6 4 3
∫ (cos
−1
16. x) 2 dx
π/2
0
3. ∫ cosec x dx is equal to
2

π/4 1
dx
(a) –1 (b) 1 (c) 0 (d)
1 17. ∫ e x (e x+ 1)
0
2
1
Evaluate each of the following Integrals dx
II. Short Answer Type Questions-I
18. ∫ (x + 1) (x + 2) (x + 3)  [A.I. 2003]
0
π/4
2
1. ∫ 1 − sin 2x dx  [Delhi 2008] 5x 2dx
0
19. ∫ x 2 + 4x + 3  [A.I. 2010]
1
1
1− x π/2
2. ∫ 1+ x
dx  [A.I. 2008]
20. ∫
 x + sin x 
 1 + cos x  dx  [A.I. 2011]
0
0
a
a−x
3. ∫ dx  [Delhi 2008] π
a+ x
∫ [cos ax − sin bx ]
2
−a 21. dx  [Delhi 2015]
−π
1
(x − x )
3 1/ 3
π/4
4. ∫ x4
dx 22. ∫
dx
3
 [A.I. 2015]
1/ 3
0
cos x 2 sin 2x
π/4 π/2
sin x cos x
5. ∫ sin 4 x + cos 4 x
dx 23. ∫ x 2 sin x dx  [Delhi 2014 (C)]
0 0
π /3
(sin x + cos x ) dx
( )
π
6. ∫ sin 2x
 [Delhi 2010, A.I. 2014 (C)]
24. ∫e
2x
sin x +
π
dx  [Delhi 2006]
π/6 4
0
π/4
sin x + cos x
7. ∫ 9 + 16 sin 2x
dx  [V. Imp.] [Delhi 2014] III. Long Answer Type Questions
0 π/2
dx
π/2 1. ∫
8. ∫( )
tan x + cot x dx  [V. Imp.] [Delhi 2008, 2012] 0
a 2 cos 2 x + b 2 sin 2 x
0 π/4
(sin x cos x) dx
π/2
(sin 2x) dx
2. ∫ cos 2 x + sin 4 x
9. ∫ sin 4 x + cos 4 x
0
0
π/2
cos 2 x dx
π/2
(sin x cos x) dx 3. ∫ cos 2 x + 4 sin 2 x
10. ∫ 1 + sin 4 x
 [Delhi 2000] 0
0
π
a
x 4 dx dx
11. ∫
4. ∫ 3 + 2 sin x + cos x
0 a2 − x2 0

π π/2
 1 − sin x  x
12. ∫  1 − cos x  e dx 5. ∫ (cos 2x) log (sin x) dx  [Delhi 2012]
π/2 π/4

Integrals 223
E:\AMIT_WORKS\Exam_Guru\EG_Mathematics-12_(working_02-06-2022)\EG_Mathematics-12_working\Open_Files\Chap_7\Chap_7
\ 17-Aug-2022 Amit Proof 5 Reader’s Sign _______________________ Date __________

Answers 7.11

( )
π/4 π/4
 π π 
∫ tan x dx = ∫ (sec )
x − 1 dx = [ tan x − x ]0 ⇒ I = (sin 0 − 0) − sin − 
π/4
I. 1. 2 2
 2 2 
0 0

 π π  π
=  tan −  − (tan 0 − 0) = 1 −  − 0 = 1 −
 4 4  4
π
4



π  π −1
= ( 0 − 0) − 1 −  =
2  2 ( ) ( )
Hence, (b) is the correct answer. 3. (πa);
a
1
3
π π a − x dx

3
Hint: I =
∫0 1 + x 2 dx =  tan x  0 = tan 3 − tan 0 = 3 − 0 = 3
2. −1 −1 −1
a+x
−a
Hence, (d) is the correct answer. Put x= a cos q ⇒ dx = –a sin q dq
When x = –a then –a = a cos q
π/2
π π ⇒ cos q = –1 ⇒ q=p
∫ cosec x dx = [ − cot x ]
π/2
3. 2
= − cot + cot = 0 + 1 = 1
π/4
π/4
2 4 When x = a then a = a cos q
Hence, (b) is the correct answer. ⇒ cos q = 1 ⇒ q=0
II. 1. ( ) (
2 − 1 or 1 − 2 ; ) 0
 a − a cos θ 
( −a sin θ d θ)
π/4
\ I= ∫  a + a cos θ 
π

( )
Hint: I = 1 − sin 2x dx 0
0 ⇒ I = −a∫ sin θ/2 × 2 sin θ/2 cos θ/2 d θ
cos θ/2 1
π/4 π
= ∫ sin 2 x + cos 2 x − 2 sin x cos x dx

0
= −a∫ 2 sin 2 θ d θ
0 2
π
π/4
0
= ∫ (cos x − sin x ) dx I = −a∫ (1 − cos θ ) d θ = −a [θ − sin θ ]π
0
0
π
π/4
I = a [sin θ − θ ]π
0

or ∫ (sin x − cos x ) dx
0 = a[(sin 0 – 0) – (sin p – p)]
I = [sin x + cos x ] π/4 ⇒ I = a[(0 – 0) – (0 – p)] = pa.
0
4. 6;
 
=  1 + 1  − ( 0 + 1) ( )
 x − x3 1/3 
1

 2 2   Hint: I = ∫   dx
 x4 
1/3  

I =  2 − 1
1 1/ 3
 x − x3   dx 
( ) ∫  x3 
=  3 
2. π x
− 1 ; Hint: Substitute: x = cos θ 1/ 3
2
1 1/ 3
1  1  dx
 1− x 
Hint: I = ∫  1 + x  dx
I= ∫  x 2 − 1 x3
0
  1/3

Put x = cos q ⇒ dx = –sin q dq 1  −2 


Put =z ⇒  3  dx = dz
x2 x 
When x = 0 then 0 = cos q
dx
⇒ q = p/2 ⇒ = dz
x3 −2
When x = 1 then 1 = cos q 1 1 =z
When x = then
⇒ q=0 3 (1/3)2
0 ⇒ z=9
 1 − cos θ 
\ I= ∫  1 + cos θ 
( − sin θ d θ) When x = 1 then 1 =z
π/2 (1)2
( )( )
0 ⇒ z=1
= − ∫ sin θ/2 × 2 sin θ cos θ d θ 1
cos θ/2 2 2  4/3 

( )
dz = − 1  ( z − 1) 
π/2 1

∫ ( z − 1)
1/ 3
0 0 \ I=
⇒ I = − ∫ 2 sin 2 θ d θ = − ∫ (1 − cos θ ) d θ −2 2 4 
2
9  3  9
π/2 π/2
−3 0 − 16 = 6
= − [θ − sin θ ]
0
= [sin θ − θ ]π / 2
0 =
8
[ ]
π/2

224 Mathematics–12
E:\AMIT_WORKS\Exam_Guru\EG_Mathematics-12_(working_02-06-2022)\EG_Mathematics-12_working\Open_Files\Chap_7\Chap_7
\ 17-Aug-2022 Amit Proof-5 Reader’s Sign _______________________ Date __________

5. ()
π ;
8
When x = 0
⇒ z = –1
then sin 0 – cos 0 = z

π/4
 sin x cos x  dx When x = π then sin π − cos π = z
Hint: I = ∫  4 
sin x + cos 4 x 
4 4 4
0 ⇒ z=0
Divide numerator and denominator by cos4 x. 0
dz
π/4
tan x sec x dx 2 \ I= ∫ 9 + 16 (1 − z 2 )
∫ tan 4 x + 1
I= −1
0 0 0
Put tan2 x = z ⇒ 2 tan x sec x dx = dz dz dz
When x = 0 then tan2 0 = z
⇒ I= ∫ 25 − 16z 2 = ∫ (5)2 − ( 4z )2
−1 −1
⇒ z=0 0
I = 1 log 5 + 4z
When x = π then tan 2 π = z 40 5 − 4z −1
4 4
⇒ z=1 I = 1 log 5 − log 5 − 4 
1 40  5 5+ 4 
1  dz  1 1
∫ 2  z 2 + 1 = 2  tan
−1
z 
()
I=
0
0
I = 1 log1 − log 1 
40  9 
1 1 π π
=  tan −11 − tan −1 0 =  − 0 =
= 1 0 − log (3)  = 1 log (3).
−2
2 2  4  8
40   20
  
6.  2 sin −1  3 − 1  ;
 2 
8. ( 2π);
 π/2
π /3
(sin x + cos x ) dx Hint: I = ∫( tan x + cot x dx )
Hint: I = ∫ sin 2x
0
π/6 π/2
Substitute (sin x – cos x) = z sin x + cos x dx
⇒ I= ∫ sin x cos x
⇒ (cos x + sin x)dx = dz 0
(sin x – cos x)2 = z2 π/2
sin x + cos x dx
⇒ sin 2x = 1 – z2 = 2 ∫ sin 2x
0
π
When x= then sin π − cos π = z Substitute sin x – cos x = z
6 6 6
⇒ (cos x + sin x) dx = dz
 
⇒ z = 1− 3 Also sin 2x = 1 – z2
 2 
When x = 0 then sin 0 – cos 0 = z
π π
When x= π then sin − cos = z ⇒ z = –1
3 3 3
π π− π
 3 − 1 When x = then sin cos = z
3−1 2 2 2
⇒ =z ⇒ z=
2 2  2  ⇒ z=1
3 −1 1
3 −1 dz 1
2
dz \ I= 2∫ = 2 sin −1 z 
\ I= ∫ = sin −1
( z )1−
2
3 −1 1− z 2 −1
1− 3 1 − z2 2
2 = 2 sin −11 − sin −1 ( −1)
   
= sin −1  3 − 1 − sin −1  1 − 3 
 2   2  ⇒ I= ( )
2 π + π = 2π
2 2
−1 
3 − 1 sin −1  3 − 1
= sin  + 9.  π  ;
 2   2   2 
π/2
  sin 2x
= 2 sin −1  3 − 1 Hint: I = 
∫  sin 4 x + cos4 x  dx

 2  0

7.  1 log 3 ; π/2
2 sin x cos x
 20  = ∫ sin 4 x + cos4 x dx
0
π/4
(sin x + cos x ) dx
Hint: I = ∫
9 + 16 sin 2x
π/2
2 tan x sec2 x dx
0 I= ∫ tan 4 x + 1
Put sin x – cos x = z ⇒ (cos x + sin x)dx = dz 0
[Q sin 2x = 1 – z2] Put tan2 x = z and proceed as in Q.No. 5.

Integrals 225
E:\AMIT_WORKS\Exam_Guru\EG_Mathematics-12_(working_02-06-2022)\EG_Mathematics-12_working\Open_Files\Chap_7\Chap_7
\ 17-Aug-2022 Amit Proof 5 Reader’s Sign _______________________ Date __________

10.  π  ;
( ) ( )
π π
x x
 8  I = − ∫ cot e dx +
1
∫ cosec2 x e xdx
2 2 2
π/2 π/2
π/2
(sin x cos x ) dx Integrate only first integral by parts
Hint: I = ∫
{ ()}
1 + sin 4 x  π
0 x ex
I = −  cot +
Put sin2 x = z ⇒ sin x cos x dx = dz  2 π/2
2
When x = 0 then sin2 0 = z ⇒ z = 0
( ) ( )
π  π
cosec2 x e xdx  + 1 ∫ cosec 2 x e xdx
1
When x = π then sin 2 π = z ⇒ z = 1 2 ∫ 2  2 π/2 2
π/2
2 2

() ( )
π π
1  x ex  1 cosec 2 x e x dx
2  π/2 2 ∫
dz/2 = 1  −1 z 1 I = − cot −
\ I= ∫ tan   2
0
1 + z2 2  0 π/2

()
π
x e x dx
⇒ I = 1  tan −11 − tan −1 0 = π +1 ∫ cosec
2
2 8 2 2
π/2

() ()
 4
11.  3πa  ; = − cot π eπ − cot π eπ / 2 
 16   2 4 
a = – [0 – 1e ] = e .
p/2 p/2
x 4 dx
Hint: I = ∫ 13.  π − 1 ;
0 a2 − x2  2 
Substitute x = a sin q π/2

∫ (sin 2x )  tan (sin x ) dx


−1
⇒ dx = a cos q dq Hint: I =
0
When x = 0 then 0 = a sin q π/2
⇒ q=0 I = 2 ∫ (sin x )  tan −1 (sin x ) cos x dx
When x = a then a = a sin q 0
π/2
a sin θ ( a cos θ d θ )
4 4 Put sin x = z ⇒ cos x dx = dz
⇒ q= π  \I= ∫ When x = 0 then sin 0 = z
2 2 2
a − a sin θ2
0
⇒ z=0
π/2
sin 4 θ cos θ When x = π then sin π = z

4
I= a dθ
cos θ 2 2
0
⇒ z=1
π/2
1 1
∫ sin
4 4
⇒ I= a θ dθ
0
\ I = 2∫ z tan z dz = 2∫ tan z z dz
−1 −1
( )
0 0
4 π/2
Now integrate by parts
⇒ I= a ∫ (3 − 4 cos 2θ + cos 4θ) d θ
8 1 1
( )
 2 2
I =  2 tan −1 z z  − 2∫  1 2  z dz
0

4 π/2
 2 
0 0 1 + z  2
⇒ I = a 3θ − 2 sin 2θ + sin 4θ 
8  4  0
(
1 1  z 2 + 1 − 1
)
=  tan −1 z z 2  − ∫   dz
( )
( )

a 4  3π   0  z2 + 1 
⇒ I= − 0 + 0 − ( 0 − 0 + 0) 0 
8  2 
1
( )
1
 1 
I =  tan z z  − ∫ 1 − 2  dz
−1 2
⇒ I = 3π a 4
0  z + 1
16 0

( )
1 1
12. [ep/2] I =  tan −1 z z 2  −  z − tan −1 z 
 0  0

( )
π
1 − sin x e x dx
Hint: I = ∫ 1 − cos x I = ( tan z ) z + tan z − z 
−1 2 −1
1

π/2   0

I = ( z + 1) tan ( z ) − z 
1
π  1 − 2 sin x cos x  2 −1
 
∫  2 sin22 x 2  e dx
x 0
=
π/2 
2 

 ( π
4 ) 
I =  2 × − 1 − {( 0 + 1) 0 − 0}

( )
π

( ) ( )
1 cosec2 x − cot x e x dx
I= ∫ 2 2 2 I=
π
−1 − 0 =
π
−1
π/2 2 2

226 Mathematics–12
E:\AMIT_WORKS\Exam_Guru\EG_Mathematics-12_(working_02-06-2022)\EG_Mathematics-12_working\Open_Files\Chap_7\Chap_7
\ 17-Aug-2022 Amit Proof-5 Reader’s Sign _______________________ Date __________

14.  π − 1 ; { }
0
 2  I = −  −θ 2 cos θ + 2 θ sin θ − ∫ sin θ d θ 
  π/2
1 1

∫ (sin ) ( )
0
Hint: I =
−1
x dx = ∫ sin −1 x .1 dx I = −  −θ 2 cos θ + 2 {θ sin θ + cos θ}
 π/2
0 0
0
Integrate by parts = −  −θ 2 cos θ + 2θ sin θ + 2 cos θ 
π/2
1
( )
x dx
()
1
I =  sin −1 x x  − ∫  
 0 2 = − ( 0 + 0 + 2) −  0 + 2 π × 1 + 0 
0 1− x   2 
1 = –[2 – p] = p – 2
I =  x sin −1 x  + 1 ∫ −2x dx
1
1
 0 2 2 17. log ( e + 1) − − (log 2)  ;

0 1− x
 e 
1 1
I =  x sin −1 x  +  1 − x 2  1
dx
  0   0 Hint: I = ∫ e x (e x + 1)
−1 −1 0
I = 1sin 1 − 0 sin 0 +  1 − 1 − 1 − 0 
Put ex = z ⇒ ex dx = dz

( )
dz
=  π − 0  π
 + [ −1] = 2 − 1
⇒ z dx = dz ⇒ dx =
z
 2
When x = 0 then e0 = z ⇒ z = 1
15.  e ( e − 2) ; When x = 1 then e1 = z ⇒ z = e
 2  e e
dz  1 1 1
e2 
1 1  \ I= ∫ ( z + 1) z 2 = ∫  z + 1 − z + z 2  dz
Hint: I = ∫  log x − (log x )2  dx 1 1
e   e
⇒ I = log ( z + 1) − log z − 1 
e2 e2  z 1
 1  dx
∫  log x  dx − ∫ (log x )2
{ }
⇒ I=
e e ⇒ I =  log ( e + 1) − log e − 1 − ( log 2 − log1 − 1)
 e
Integrate only first integral by parts
1
e2 e2 e2 ⇒ I = log ( e + 1) − 1 − − log 2 + 0 + 1
I=  x  + 1 x dx dx  e 

 log x  ∫ (log x )2 x − ∫ (log x )2
 e e e ⇒ I = log ( e + 1) − 1 − log 2
e
e2 e2 e2
I =  x  +
 log x  e

e
dx −
(log x )2 e
dx
∫ (log x )2 1
18.  log
2 3 ()
8 − log 3 
2 
; ()
e2 1
   2  dx
I=  x  = e 2 − e  Hint: I = ∫ ( x + 1)( x + 2)( x + 3)
 log x  e  log e log e  0

( )( ) ( )
2 2 1
  1 1 
I=  e − e  =e −e = ∫ − 1 +1 1  dx
 2 log e log e  2 1  2 x +1 x+2 2 x+3
0

e2 − 2e e e Integrate and find the value.


I= = ( − 2)
16. [π – 2]
1
2 2 

5
19. 5 + log
2 2 ()
3 − 45 log 5 
2 4 
; ()
∫ (cos )
−1 2
Hint: I = x dx 2
5x 2 dx
2
 4x + 3 
0 Hint: I = ∫ x 2 + 4x + 3 = ∫ 5 1 − x 2 + 4x + 3  dx
Put cos x = q
–1
⇒ x = cos q 1 1

{ ( ) ( )}
⇒ dx = – sin q dq 2
 
When x = 0 then 0 = cos q = 5∫ 1 − − 1 1 + 9 1 dx
 2 x + 1 2 x + 3 
1
⇒ q= π
Now integrate.
2
When x = 1 then 1 = cos q 20.  π  ;
⇒ q=0  2 

( )
π/2
x + sin x

0
I = − ∫ (θ ) sin θ d θ
2 Hint: I = ∫ 1 + cos x
dx
0
π/2
Integrate by parts π/2  x + 2 sin x cos x 
 2 2  dx
I = −  −θ 2 cos θ + ∫ 2θ cos θ d θ 
0 = ∫  2 cos 2x 
  π/2 0  2 

Integrals 227
E:\AMIT_WORKS\Exam_Guru\EG_Mathematics-12_(working_02-06-2022)\EG_Mathematics-12_working\Open_Files\Chap_7\Chap_7
\ 17-Aug-2022 Amit Proof 5 Reader’s Sign _______________________ Date __________

π/2 π/2 23. (π – 2)


1 x sec2 x dx +
= ∫ ∫ tan x dx π/2

∫ (x )
2 2 2 Hint: I =
2
sin x dx
0 0
Integrate only first integral by parts 0
Integrate by parts

( )( ) ( )
π/2 π/2 π/2
I =  1 x tan x 2
π/2
−∫ 1 tan x 2 dx + tan x dx I =  −x 2 cos x + ∫ 2x cos x dx 

 2 2 0 2 2 ∫ 2

 0
0 0
π/2
x
π/2 π/2 π / 2. I =  −x 2 cos x + 2x sin x − ∫ 2 sin x dx 
I =  x tan  − ∫ tan x dx + ∫ tan x dx  0
 2 0 2 2 π/2
0 0
I =  −x 2 cos x + 2x sin x + 2 cos x 
 0
I =  π tan π − 0 = π
 2 4  2 I = [(0 + p × 1 + 0) – (0 + 0 + 2)]
I = (p – 2)
 sin (2aπ) sin (2bπ) 
21.  2π + − ;  2
 2a 2b  24. 
π  5 
∫ [cos ax − sin bx ]
2
Hint: I = dx
( )
π
π 2x
−π Hint: I = ∫ sin x+
4
e dx
π 0
⇒ I= ∫ cos 2 ax + sin 2 bx − 2 sin bx cos ax  dx Integrate by parts
 

( ) ( )
−π 2x 2x π
 
I = sin x + π e − ∫ cos x + π e dx 
( )( )
π
 1 + cos 2ax 4 2 4 2
⇒ I= ∫  + 1 − cos 2bx  0
2 2
−π

 (a + b) x   (b − a ) x  



( )
I = sin x +
π e 2x
4 2
− sin   − sin    dx
( ) ( )
π
 2   2   2x 2x 
− cos x + π e + ∫ sin x + π e dx  
Now integrate and find the value.  4 4 4 4 
0

( ) ( )
π
22.  6 

I = sin x +
π e2x − cos x + π e2x  1
− I
 5  4 2 4 4  0 4

{ ( ) ( )}
π/4 π
dx  
Hint: I = ∫ 3 5I = 1  2 sin x + π − cos x + π e2x 
0 cos x 2 sin 2x 4 4 4 4 0

{ ( ) ( )}
π/4 π
dx  
I= ∫ 3 I = 1  2 sin x + π − cos x + π e2x 
0 2 cos x sin x cos x 5 4 4 0

{ ( ) ( )} { ( ) ( )}
π/4
 
= ∫
dx I = 1  2 sin 5π − cos 5π − 2 sin π − cos π 
2 cos 4 x tan x 5 4 4 4 4 
0

π/4
sec4 x dx
π/4
( tan 2
)
x + 1 sec2 x dx
   
I = 1  2 × 1 + 1  − 2 × 1 − 1  
I= ∫ 2 tan x
= ∫ 2 tan x
5  2 2  2 2 
0 0
 
Sub: tan x = z ⇒ sec x dx = dz 2 I = 1 2 + 1 − 2 + 1 
5 2 2
When x = 0 then tan 0 = z
⇒ z=0 I= 1× 2= 2
5 5
When x = π
4
then tan ()
π
4
=z
III. 1.  π  ;
 2ab 
⇒ z=1 π/2
dx
1
( 2
)
1 z + 1 dz = 1  z 3/2 + 1  dz
I= ∫
1 Hint: I = ∫ a cos x + b 2 sin 2 x
2 2

2 ∫ 
\  0
2
0 z 0 z π/2
sec2 x dx
1 ⇒ I= ∫ a + b 2 tan 2 x
2
I = 1  2 z 5/ 2 + 2 z  0
2  5  0
Put tan x = z ⇒ sec2 x dx = dz

2 5 ( )
I = 1  2 + 2 − ( 0) = 1 × 12 = 6
 2 5 5
When x = 0
⇒ z=0
then tan 0 = z

228 Mathematics–12
E:\AMIT_WORKS\Exam_Guru\EG_Mathematics-12_(working_02-06-2022)\EG_Mathematics-12_working\Open_Files\Chap_7\Chap_7
\ 17-Aug-2022 Amit Proof-5 Reader’s Sign _______________________ Date __________

When x = π then tan π = z


I = −1
π/2 
(4 − 3cos x) − 4  dx
2
2 2
3 ∫ 
 4 − 3 cos 2 x 
⇒ z=∞ 0  

( )
∞ ∞
dz 1  −1 bz  π/2
\ I= ∫ a 2 + b2z 2 = ab  tan a  0 I=
−1  4
∫ 1 − 4 − 3 cos2 x  dx

0 3
0
1  −1 ∞ − 1  π − 0
⇒ I= tan tan −1 0 = π/2
ab  ab  2  I = −1  x  + 4
π/2 sec2 x dx

3   0 3 ∫ 1 + 4 tan 2 x
= π 0

( )
2ab
I = −1  π  + 4  tan −1 ( 2 tan x )
π/2

π  
3 2 3× 2   0
2.
6 3 −π + 2  tan −1 ∞ − tan −1 0
I=
π/4
(sin x cos x ) dx 6 3 
Hint: I = ∫ cos 2 x + sin 4 x −π + 2  π − 
0 I= 0 = −π + π = π
6 3  2  6 3 6
Divide numerator and denominator by cos x. 4

π/4 4. π
tan x sec 2 x dx 4
I= ∫ tan 4 x + tan 2 x + 1 dx
0 Hint: I = ∫ 3 + 2 sin x + cos x
Put tan x = z 2
⇒ 2tan x sec x dx = dz
2

When x = 0 then tan2 0 = z 2 tan x 1 − tan 2 x


Change sin x = 2 and cos x = 2
⇒ z=0 2x 2x
1 + tan 1 + tan
2 2
When x = π then tan2 π = z
4 4
and then put tan x = z
⇒ z=1 2
1 1
I = 1 ∫ 2 dz 1 dz 5.  −π + 1 log 2 + 1  ;
2 z + z +1 2 ∫
=
( )
2  8 4 4 
0 0 z+1 + 3 π/2
2 4
1
Hint: I = ∫ (cos 2x ) logsin x dx
  
I = 1 × 2  tan −1  2z + 1 
π/4

2 3  3  0
π/2
I= ∫ log (sin x ) cos 2x dx
 
I = 1  tan −1  3  − tan −1  1  
π/4
π/2
 3  3 
3 I = log (sin x ) sin 2x − ∫ cos x sin 2x dx 
 2 sin x 2  π / 4
 
I = 1  tan −1 3 − tan −1  1   π/2
3  3  I =  1 sin 2x log (sin x ) − ∫ cos 2 x dx 
 2  π / 4
I = 1 π − π = 1 × π = π
( )
1 1 + cos 2x dx 
π/2
3  3 6  3 6 6 3 I =  2 sin 2x log (sin x ) − ∫ 
 2
()
π/4
3. π
π/2
6
π/2 π/2 I =  1 sin 2x log (sin x ) − x − sin 2x 
cos x dx2 2
cos x dx  2 2 4  π / 4
Hint: I = ∫ cos 2 x + 4 sin 2 x
= ∫ 4 − 3 cos 2 x Evaluate the limits.
0 0

Topic 12. Properties of Definite Integrals


b b b a
1. ∫ f ( x ) dx = ∫ f ( z ) dz . This property means if the limits 2. ∫ f ( x ) dx = −∫ f ( x ) dx . This property means if the limits
a a a b
and the function of the definite integral do not change and of the definite integral are interchanged, then the value of
only variable changes, then the value of the definite integral definite integral becomes negative of what it was.
remains same. a
4 4 3. ∫ f ( x ) dx = 0 . This means if the lower and upper limits
For example: ∫( ) (
3x 2 + 5 dx = ∫ 3p 2 + 5 dp ) a
of the definite integral are same, its value is zero.
1 1

Integrals 229
E:\AMIT_WORKS\Exam_Guru\EG_Mathematics-12_(working_02-06-2022)\EG_Mathematics-12_working\Open_Files\Chap_7\Chap_7
\ 17-Aug-2022 Amit Proof 5 Reader’s Sign _______________________ Date __________

d b c d π π
2 2
f ( x ) dx = ∫ f ( x ) dx + ∫ f ( x ) dx + ∫ f ( x ) dx . sin x + cos x
4. ∫
π

a a b c
⇒ 2I =
∫ sin x + cos x
dx = ∫ 1 dx
0
= [ x ]0 2
0
This property is called split property. π π
= ∴ I =
b b 2 4
f ( x ) dx = ∫ f ( a + b − x ) dx So, R is true.
5. ∫ From (ii), A is also true. R is the correct explanation for A.
a a
a a
Hence, the correct option is (a)
π/2
6. ∫ f ( x ) dx = ∫ f ( a − x ) dx sin n x
0 0
Example 2. Evaluate: ∫ sin x + cos n x
n dx
0
2a a a
π/2
7. ∫ f ( x ) dx = ∫ f ( x ) dx + ∫ f ( 2a − x ) dx Solution. Let I = ∫
sin n x
dx  ...(1)
0 0 0
0
sin n x + cos n x
2a a
On applying the property
8. ∫ f ( x ) dx = 2∫ f ( x ) dx if f ( 2a − x ) = f ( x )
a a
f ( x ) dx = ∫ f ( a − x ) dx , we get
0 0
= 0 if f (2a – x) = –f (x).

0 0

( )
a a
9. (i) ∫ f ( x ) dx = 2∫ f ( x ) dx , when f (x) is an even function π/2 sin n π − x dx
=∫ 2
sin ( ) ( )
−a 0
i.e., f (– x) = f (x). n π − x + cos n π − x
0
a 2 2
(ii) ∫ f ( x ) dx = 0 , when f (x) is an odd function i.e., =∫
π/2
cos n x dx
−a
cos n x + sin n x
f (– x) = – f (x). 0
nT T π/2
cos n x dx
10. ∫0
f ( x)dx = n∫ f ( x)dx , where ‘T’ is the period of the
0
=∫
sin n x + cos n x
 ...(2)
0
function i.e., f(x + T) = f(x)
b + nT b
On adding (1) and (2), we get
π/2 π/2
11. ∫
a + nT
f ( x)dx = ∫ f ( x)dx , where f(x) is periodic with period
a
2I = ∫
 sin n x + cos n x 
 sin n x + cos n x  dx = ∫ dx
0 0
T and nŒI
π
π π
Questions Based on the Above Properties of ⇒ 2I =  x
 =  − 0 =2
0
 2  2
Definite Integrals ⇒ I= π
a a 4
π/2
Property: ∫ f ( x ) dx = ∫ f ( a − x ) dx sin n x dx π
\ ∫ =
0 0 sin n
x + cos n
x 4
0
π π/2
cos x π 2 tan n x
Example 1. Assertion (A):
∫ dx = Example 3. Evaluate: ∫ 1 + tan n x
dx
π 0
sin x + cos x 4 0
2
Reason (R): sin x π π/2
tan n x
π/2
sin n x
∫0 sin x + cos x dx = 4 Solution. Let I = ∫ dx = ∫ dx
0
1 + tan n x 0
sin x + cos n x
n

π
2
sin x Now this is the same question as the last example.
Solution. I = ...(i)
∫ sin x + cos x dx π/2
tan n x
π/2
sin n x dx π
a
0
a
\ I= ∫ 1 + tan n x
dx = ∫ =
sin n x + cos n x 4
∫ f ( x ) dx = ∫ f (a − x ) dx
0 0
Q Note:
0 0 π/2
π/2
sin n x dx π cos n x dx π
π 
1. ∫ =
sin n x + cos n x 4
2. ∫ =
sin n x + cos n x 4
sin  − x dx
π
0 0
2 2

I = ∫ π/2 π/2
π  π  1 π 1 π
0
sin  − x + cos  − x
2  2 
3. ∫ 1 + tan n x
dx =
4
4. ∫ 1 + cot n x
dx =
4
0 0
cos x
I= ∫ dx ...(ii) π/2 π/2
cos x + sin x tan n x dx π cot n x dx π
Adding equations (i) and (ii),
5.
∫ =
1 + tan n x 4
6. ∫ =
1 + cot n x 4
0 0

230 Mathematics–12
E:\AMIT_WORKS\Exam_Guru\EG_Mathematics-12_(working_02-06-2022)\EG_Mathematics-12_working\Open_Files\Chap_7\Chap_7
\ 17-Aug-2022 Amit Proof-5 Reader’s Sign _______________________ Date __________

EXERCISE 7.12
I. Multiple Choice Questions (MCQs) π/2
π/2
2008 sin x 7. ∫ [ 2 log sin x − log sin 2x ] dx  [V. Imp.]
1. ∫ 2008sin x + 2008cos x
dx = 0
0 III. Short Answer Type Questions-II
π π π
(a) 0 (b) p (c) (d) x dx
3 4 1. ∫ 1 + sin x  [Delhi 2010, A.I. 2004, 2011 (C)]
π/2
0
a
200 sin x + 100 cos x x dx
2. ∫
0
sin x + cos x
dx = 2. (a) Evaluate: ∫ x+ a−x
0
(a) 50p (b) 25p (c) 75p (d) 150p a
a − x dx
(b) Evaluate: ∫ x+ a−x
 [Delhi 2008 (C)]
π/2 0
φ( x )
3. The value of the integral ∫ π
dx is π
x tan x dx
0 
φ( x ) + φ  − x 
3. ∫ sec x + tan x
2  0
 [V. Imp.] [Delhi 2008, 2010 (C), A.I. 2008, 2017]
π π
(a) (b) 1
 2x − 1 
4 2
∫ tan
−1
4.  1 + x − x 2  dx
(c) p (d) None of these 0
π /3
dx
2a
f ( x) 5. ∫ 1 + tan x
 [Delhi 2007, 2011 (C)]
4. The value of the integral ∫ f ( x ) + f ( 2a −x )
dx is equal π/6
to 0 π
x sin x dx
6. ∫ 1 + cos2 x  [Delhi 2009 (C), A.I. 2012, 2014, 2017]
0
(a) a (b) 2a (c) 1 (d) 0
π
 4 + 3 cos x 
6
x
7. ∫ log  4 − 3 cos x  dx
5. The value of the integral ∫
3 9−x + x
dx is 0
8. If f (x) = f (a – x) and g (x) + g (a – x) = 4, prove that:
a a
1 3
(a) (b) (c) 2 (d) 1 ∫ f ( x ) g ( x ) dx = 2∫ f ( x ) dx
2 2 0 0
π  b
 − x 
π/2
4 9. If f (a + b – x) = f (x), then ∫ x f ( x ) dx = ?
6. The value of ∫ dx is

( )
1 a
0 sin x + cos x 1
10. ∫ log − 1 dx  [Delhi A.I. 2008, 2014]
x
π 2 π 0
(a) (b)
4 4 2 π/2
sin 2 x dx 
(c) 0 (d) None of these
11. ∫ sin x + cos x
[Delhi A.I. 2008, 2014 (C)]
0

Evaluate each of the following Integrals π/2

II. Short Answer Type Questions-I


12. ∫ log (tan x) dx  [Delhi 2004]
0
n π/2
(sin x − cos x) dx π
1. ∫ x n − x dx 2. ∫ 1 + sin x cos x 13. x dx
∫ a 2 cos2 x + b2 sin 2 x  [A.I. 2006, 2009 (C)]
0 0
0
π/2 π/2
dx dx
∫ ∫
3. Or π
x tan x dx
0 1 + cot x 0 1 + tan x 14. ∫ sec x cosec x  [V. Imp.] [Delhi 2007, 2008, 2011 (C)]
0
π/2 1

∫ (cos x ) dx ∫ x (1 − x )
2 n π π
4. 5. dx x dx x sin x dx
0 0
15. ∫ 1 + sin 2 x 16. ∫ 1 + sin x
π/4 0 0

6. ∫ log (1 + tan x) dx π b
f ( x ) dx
0 17. ∫ x log (sin x) dx 18. ∫ f (a + b − x) + f ( x)
 [V. Imp.] [A.I. 2011, Delhi 2013 (C), (SP)] 0 a

Integrals 231
E:\AMIT_WORKS\Exam_Guru\EG_Mathematics-12_(working_02-06-2022)\EG_Mathematics-12_working\Open_Files\Chap_7\Chap_7
\ 17-Aug-2022 Amit Proof 5 Reader’s Sign _______________________ Date __________

π/2 2
x 2 dx 
19. ∫ (sin 2x) (log tan x) dx  [A.I. Delhi 2011] 26. ∫ 1 + 5x [A.I. (North) 2016]
0 −2
a a
dx x 2 dx
20. ∫ 27. ∫ 1 + 5x
0 x + a2 − x2 −a

1 IV. Long Answer Type Questions


log (1 + x) dx
21. ∫ 1 + x2  [A.I. 2011 (C)] π/2
− π
1. (a) Prove: ∫ log (sin x) dx =  (log 2)
 2 
0
0
π/2

∫ [2 log cos x − log sin 2x] dx


π/2
22. − π
(b) Prove: ∫ log (cos x) dx =  (log 2)
0  2 
0
2π  [V. Imp.] [A.I. 2008]
dx
23. ∫ 1 + esin x
π
0 2. (a) Evaluate: ∫ log (1 + cos x) dx = −π (log 2)
π 0
ecos x dx
24. ∫ cos x
π
 [Delhi 2009]
0
e + e − cos x (b) Evaluate: ∫ log (1 − cos x) dx = −π ( log 2) [A.I. 2008]
0
π π/2
x sin x dx x sin x cos x dx [A.I. 2010 (C), Delhi 2014]
25. ∫ 1 + 3 cos2 x  [A.I. (East) 2016] 3. Evaluate: ∫ sin 4 x + cos 4 x
0 0

Answers 7.12
π/2 π/2
2008sin x φ( x )
I. 1. (d) I= ∫ 2008 + 2008cos x
sin x
dx  ...(1)
3. (a) I= ∫ π 
dx
 ...(1)
0 0 φ( x ) + φ  − x
2 
π 
sin  − x
π/2 2 
2008 π 
= ∫ π 
sin  − x
π 
cos  − x
dx π/2 φ  − x
2 
0
2008 2 
+ 2008 2 
= ∫ π 
dx  ...(2)
0 φ  − x + φ( x )
 a a
 2 
 Q ∫ f ( x) dx = ∫ f (a − x) dx   a a

 0 0   ∫
Q f ( x ) dx = ∫ f (a − x) dx 
π/2   0 0 
2008cos x
⇒ I= ∫0
2008cos x + 2008sin x
dx  ...(2)
Adding (1) and (2), we get
π/2 π
Adding (1) and (2), we get π  π π
π/2
2I = ∫ dx = [ x]02 =  − 0 = ⇒ I =
2  2 4
2008sin x + 2008cos x 0
2I = ∫ 2008sin x + 2008cos x
dx 2a
f ( x)
0
4. (a) I= ∫ f ( x ) + f ( 2 a −x )
dx  ...(1)
π/2 π
π  π 0
⇒ I= ∫ dx = [ x] 2
0
⇒ 2I =  − 0 ⇒ Ι =
2  4
2a
f ( 2a − x )
0 ⇒ I= ∫
0
f ( 2a − x ) + f ( x )
dx ... (2)
π/2 π/2
sin x + cos x sin x
2. (c) I = 100 ∫ sin x + cos x dx + 100 ∫ sin x + cos x dx  a a

0 0  Q ∫ f ( x) dx = ∫ f (a − x) dx 
π/2
 0 0 
 π  π/2 sin x π
= 100 ∫ dx + 100    Q ∫ dx =  Adding (1) and (2), we get
 4 sin x + cos x 4
0  0 2a 2a
f ( x ) + f ( 2a − x )
2I = ∫ f ( 2a − x ) + f ( x )
dx = ∫ dx
π
 π  π π 0 0
= 100 [ x ] 2 + 100   = 100  + 
0
 4  2 4 ⇒ 2I = [ x]02 a = (2a − 0) = 2a ⇒ I = a
3π 3
= 100 × = 75π 5. (b)
4 2

232 Mathematics–12
E:\AMIT_WORKS\Exam_Guru\EG_Mathematics-12_(working_02-06-2022)\EG_Mathematics-12_working\Open_Files\Chap_7\Chap_7
\ 17-Aug-2022 Amit Proof-5 Reader’s Sign _______________________ Date __________

π/2
π
4
−x π/2


( ) ( π2 − x) dx
sin π − x − cos
2

( ) ( π2 − x)
6. (c) I= dx  ...(1) I=
0 sin x + cos x 0 1 + sin π − x cos
a a
2
Using ∫ f ( x) dx = ∫ f ( a − x ) dx π/2
(cos x − sin x ) dx 
0 0 ⇒ I= ∫ 1 + sin x cos x
...(2)
0
π π 
π/2 −  − x Add (1) and (2)
4  2 
I= ∫  π  π 
dx
0
sin  − x + cos  − x
π/2
(sin x − cos x + cos x − sin x ) dx
 2   2  2I = ∫ 1 + sin x cos x
0
π
π/2 − +x π/2
π/2
4
= ∫ cos x + sin x
dx  ...(2) 2I = ∫ 0 dx = C  0 =C−C=0
0 0

Adding (1) and (2), we get 3. π


4
π/2 π/2
π π dx sin x dx
π/2
4
−x π/2 −
4
+x Hint: I = ∫ 1 + cot x
= ∫ sin x + cos x
 ...(1)
I+I= ∫0 sin x + cos x
dx + ∫
0 cos x + sin x
dx 0
a
0
a
Apply the property: ∫ f ( x ) dx = ∫ f ( a − x ) dx
π π
π/2 −x− +x 0 0
4 4
2I = ∫ dx
0 sin x + cos x
I=
π/2


( )
sin π − x dx
2

sin ( π − x ) + cos ( π − x )
π/2
0
= ∫0 sin x + cos x
dx = 0 ⇒ I = 0 0
2 2
 3  5 π/2
II. 1. 2n  n 2  − 2  n 2  ; cos x dx
3   5 
= ∫ cos x + sin x
 ...(2)
0
n
Add (1) and (2)
Hint: I = ∫x n − x dx
π/2

0 π/2
Apply the property:
2I = ∫ 1dx =  x  0 2
0
a a

∫ f ( x ) dx = ∫ f ( a − x) dx ⇒ I= π
4
0 0
π/2 π/2
dx cos x dx
∫ 1+ ∫
n or =
I= ∫ (n − x) n − ( n − x ) dx 0 tan x 0 cos x + sin x
0
Repeat the solution of first part.
n n
I= ∫ (n − x) (
x dx = ∫ n x − x 3/ 2
) dx 4.
π;
4
0 0 π/2

∫ cos
2
n Hint: I = x dx  ...(1)
I =  2n x3/ 2 − 2 x5 / 2  0
 3 5  0 Apply the property

( ) ()
a a
=  2n ( n ) − 2 ( n )  − [0 − 0]
3/ 2 5/ 2
∫ f ( x ) dx = ∫ f ( a − x ) dx
 3 5 
0 0

= 2n ( n ) − 2 ( n )
( )
3/ 2 5/ 2
π/2
π − x dx
3 5
∫ cos
2
I=
2
2. 0; 0
π/2
π/2
(sin x − cos x ) dx  = ∫ sin
2
x dx  ...(2)
Hint: I = ∫ (1 + sin x cos x ) ...(1)
0
0

On applying the property: Add (1) and (2)


π/2

∫ (sin )
a a
2
2I = x + cos 2 x dx
∫ f ( x ) dx = ∫ f ( a − x ) dx 0
0 0

Integrals 233
E:\AMIT_WORKS\Exam_Guru\EG_Mathematics-12_(working_02-06-2022)\EG_Mathematics-12_working\Open_Files\Chap_7\Chap_7
\ 17-Aug-2022 Amit Proof 5 Reader’s Sign _______________________ Date __________

π/2 π/2
π/2
= ∫ 1 dx =  x 
0
Hint: I = ∫ 2 logsin x − log (sin 2x ) dx
0 0
π
2I = π ⇒ I= π/2
 sin 2 x 
2 4 I= ∫ log  2 sin x cos x  dx
5. 1 ;
0

( )
(n + 1) (n + 2) π/2
tan x dx
1
= ∫ log 2
∫ x (1 − x )
n 0
Hint: I = dx

( )
π/2
0 tan x dx
Apply the property
I= ∫ log 2
 ...(1)
0
a a
Apply the property:
∫ f ( x ) dx = ∫ f ( a − x ) dx a a
0 0
∫ f ( x ) dx = ∫ f ( a − x ) dx
1
n 0 0
I= ∫ (1 − x ) 1 − (1 − x ) dx
0

1 1 I=
π/2



log  2 ( )  dx
 tan π − x

2
I= ∫ (1 − x ) x dx = ∫ x − x dx
n n n +1
( ) 0 



0 0

( )
π/2
cot x dx


I= x − x 
n +1 n + 2 1 = ∫ log 2
 ...(2)
0
 n + 1 n + 2 0
Add (1) and (2)
I=  1 − 1 = 1 π/2
 n + 1 n + 2  ( n + 1)( n + 2)  tan x × cot x  dx
2I = ∫ log  2 
( )(
2
π 0
6. log 2);

()
8 π/2 π / 2 
1
π/4 2I = ∫ 4
dx
log = −2 log 2  ∫ dx 
 0 
Hint: I = ∫ log (1 + tan x ) dx  ...(1) 0

Apply the property


0

π/2
2I = −2 ( log 2)  x  = −2 ( log 2)
0
π
2 ()
a a −π
∫ f ( x ) dx = ∫ f ( a − x ) dx I=
2
(log 2)
0 0 III. 1. p

( )
π/4 π
 π−x  x dx
I= ∫ log 1 + tan 4  dx Hint: I = ∫ 1 + sin x  ...(1)
0 0
π/4 Apply the property:
I= ∫ log 1 + 1 − tan x  dx a a
 1 + tan x 
0 ∫ f ( x ) dx = ∫ f ( a − x ) dx
( )
π/4 0 0
2
I= ∫ log
1 + tan x
dx  ...(2) π
( π − x ) dx = π ( π − x ) dx 
0 I= ∫ 1 + sin ( π − x ) ∫ 1 + sin x ...(2)
0 0
Add (1) and (2)
π/4 Add (1) and (2)
 2 
∫ log (1 + tan x ) ×
( )
2I = dx π
( x + π − x ) dx = ππ
(1 + tan x )  1
0  2I = ∫ 1 + sin x ∫ 1 + sin x
dx
0 0
π/4
= ∫ (log 2) dx
π
2I = π∫  1 − sin x  dx
0  cos 2 x 
0

2I = ( log 2)  x  = π log 2
π/4
π
0 4 (
= π∫ sec2 x − sec x tan x dx )
I=
π log
8 ()
( 2)

0

2I = π  tan x − sec x 
π
0
−π (log 2);
7. 2I = p[(tan p – sec p) – (tan 0 – sec 0)]
2
234 Mathematics–12
E:\AMIT_WORKS\Exam_Guru\EG_Mathematics-12_(working_02-06-2022)\EG_Mathematics-12_working\Open_Files\Chap_7\Chap_7
\ 17-Aug-2022 Amit Proof-5 Reader’s Sign _______________________ Date __________

2I = p[{0 – (–1)} – (0 – 1)] = 2p 2I = p[(sec p – tan p + p – (sec 0 – tan 0 + 0)]


I=p 2I = p[(–1 – 0 + p) – (1 – 0 + 0)]
a 2I = p[–1 + p – 1] = p(p – 2)
2. (a) ;
2
a I = π ( π − 2)
x dx 2
Hint: I = ∫ x+ a−x
 ...(1) 4. 0;
0
1
Apply the property:  2x −1 
∫ tan
−1
I=  2  dx
a a 0 1 + x − x 
∫ f ( x ) dx = ∫ f ( a − x ) dx  x − (1 − x ) 
1

∫ tan
0 0 −1
I= 1 + x (1 − x )  dx
a 0  
a−x
\ I= ∫ a−x + a−a+x
dx 1
= ∫  tan x − tan (1 − x ) dx
0 −1 −1

a 0
=∫ a − x dx  ...(2)
1 1
a−x+ x
∫ tan x dx − ∫ tan (1 − x ) dx
0 −1 −1
I=
Add (1) and (2) 0 0
a a
a a

 
2I = ∫  x + a − x  dx = ∫ dx = [ x ]0 = a
a Apply the property ∫ f ( x ) dx = ∫ f ( a − x ) dx only in second
0
 x + a − x  0 integral. 0 0

I= a
1 1

∫ ( tan x ) dx − ∫ tan

2 \ I=
−1 −1
1 − (1 − x ) dx
0 0
a
(b)
2 1 1

a
x dx
I= ∫ tan
−1
(
x dx − ∫ tan −1 x dx = 0 )
Hint: I = ∫ x+ a−x 5.
π;
0 0

0
12
Solve as we have solved. Q.No. 2 π /3
dx
3. π ( π − 2) ,
Hint: I = ∫ 1+ tan x
π/6
2 π
x tan x dx π/3
I= ∫  ...(1)  cos x 
sec x + tan x
0
I= ∫   dx 
cos x + sin x 
...(1)
π/6
Apply the property:
a a
Now apply the property
∫ f ( x ) dx = ∫ f ( a − x ) dx b b
0 0 ∫ f ( x ) dx = ∫ f ( a + b − x ) dx
π
( π − x ) tan ( π − x ) dx a a
I= ∫ sec ( π − x ) + tan ( π − x )
0
I=
π/3


(
3 6)
cos π + π − x dx
 ...(2)
− ( π − x ) tan x  dx
cos ( + − x ) + sin ( + − x )
π
I= ∫  π/6 π π π π
− [sec x + tan x ] 3 6 3 6
0
π
( π − x ) tan x dx  Add (1) and (2)
I= ∫ sec x + tan x
...(2) π/3
 sin x + cos x 
Add (1) and (2)
0
2I = ∫   dx
sin x + cos x 
π/6
π
( π − x + x ) tan x dx π
tan x dx π/3
2I = ∫ sec x + tan x
= π∫
sec x + tan x = ∫ dx =  x  π/6
π/3

0 0
π/6

tan x (sec x − tan x ) dx


π
π
2I = π∫ 2I = π − π = π ⇒ I=
sec2 x − tan 2 x 3 6 6 12
0
π π2 ;
6.
(
2I = π∫ sec x tan x − sec x + 1 dx 2
) 4
π
0 x sin x dx
2I = [sec x − tan x + x ] π I= ∫ 1 + cos2 x  ...(1)
0 0

Integrals 235
E:\AMIT_WORKS\Exam_Guru\EG_Mathematics-12_(working_02-06-2022)\EG_Mathematics-12_working\Open_Files\Chap_7\Chap_7
\ 17-Aug-2022 Amit Proof 5 Reader’s Sign _______________________ Date __________

Apply the property


( )∫
b
a+b
a a 9. f ( x ) dx ;
2
∫ f ( x ) dx = ∫ f ( a − x ) dx a
b

∫ x f ( x ) dx
0 0
Hint: I =
π
( π − x ) sin ( π − x ) dx a
I= ∫ 1 + cos 2 ( π − x ) Apply the property:
0
b b
π
( π − x ) sin x dx 
I= ∫ 1 + cos 2 x
...(2) ∫ f ( x ) dx = ∫ f ( a + b − x ) dx
0 a a

Add (1) and (2) b

π
sin x dx π
I= ∫ ( a + b − x ) f ( a + b − x ) dx
2I = π∫ 2 = −π  tan ( cos x ) 0
 −1  a

0
1 + cos x b

2I = –p[tan–1 (cos p) – tan–1 (cos 0)]


I= ∫ ( a + b − x ) f ( x ) dx
a
2I = –p[–tan–1 1 – tan–1 1] [Q f (a + b –x) = f (x) (given)]
π2
2I = 2π × π ⇒ I= b b

7. 0;
4 4 ⇒ I= ∫ ( a + b) f ( x ) dx − ∫ x f ( x ) dx
a a
π b
4 + 3 cos x  dx
Hint: I = ∫ log   ...(1) ⇒ I = ( a + b ) ∫ f ( x ) dx − I
 4 − 3 cos x 
0 a
Apply the property: b
a a
⇒ 2I = ( a + b ) ∫ f ( x ) dx
∫ f ( x ) dx = ∫ f ( a − x ) dx a
0 0

( )∫
b
 4 + 3 cos ( π − x ) 
π
a+b
I = ∫ log  ⇒ I= f ( x ) dx
 dx 2
0  4 − 3 cos ( π − x )  a
10. 0;
π

( ) ( )
4 − 3 cos x  dx
I = ∫ log 
1 1
 ...(2) 1 1 1− x
0
 4 + 3 cos x  Hint: I = ∫ log x
− dx = ∫ log
x
dx
0 0
Add (1) and (2)

( )
1
1 − x dx
( ) ∫ log
π
 4 + 3 cos x 4 − 3 cos x  I=  ...(1)
2I = ∫ log  4 − 3 cos x × 4 + 3 cos x  dx 0
x
0
π π
Apply the property:
π
= ∫ log1 dx = ∫ 0 dx = C  = C − C = 0 a a

0 0
0
∫ f ( x ) dx = ∫ f ( a − x ) dx
a 0 0
8. Hint: I = ∫ f ( x ) g ( x ) dx 1
1 − (1 − x ) 
∫ log 
0
I= dx
Apply the property: 1 − x 
0
a a

∫ f ( x ) dx = ∫ f ( a − x ) dx
( )
1
x
0 0
I= ∫ log 1 − x dx  ...(2)
a 0
I= ∫ f ( a − x ) g ( a − x ) dx Add (1) and (2)
0

( )( )
1
 1− x
× x
a 
I= ∫ f ( x ) 4 − g ( x ) dx 2I = ∫ log  x 1− x  dx
0
0
[Q f (a – x) = f (x) and g(a – x) = 4 – g(x) (given)] 1 1 1
a a ⇒ 2I = ∫ log1dx = ∫ 0 dx = C  =C−C=0
0
I = 4∫ f ( x ) dx − ∫ f ( x ) g ( x ) dx 0 0
0 0 ⇒ I=0
a
I = 4∫ f ( x ) dx − I
0
11.
1 log
2
( 2 +1 ; )
a π/2
sin 2 x dx
⇒ I = 2∫ f ( x ) dx Hint: I = ∫ sin x + cos x  ...(1)
0 0

236 Mathematics–12
E:\AMIT_WORKS\Exam_Guru\EG_Mathematics-12_(working_02-06-2022)\EG_Mathematics-12_working\Open_Files\Chap_7\Chap_7
\ 17-Aug-2022 Amit Proof-5 Reader’s Sign _______________________ Date __________

Apply the property Add (1) and (2)

( )
a a π π
f ( x ) dx = f ( a − x ) dx 1 − cos 2x
∫ ∫ 2I = π∫ sin x dx = π∫
2
2
dx
0 0 0 0
π/2 π
cos 2 x dx I = π  x − sin 2x 
I= ∫ cos x + sin x
 ...(2) 4  2  0
0
2
Add (1) and (2) I = π ( π − 0) − ( 0 − 0) = π
4 4
π
dx π2 ;
2I = ∫ sin x + cos x 15.
2 2
0 π
Now integrate and find the value. x dx
Hint: I = ∫ 1 + sin 2 x  ...(1)
12. 0 0
π/2 Apply the property:
Hint: I = ∫ log ( tan x ) dx  ...(1) a a
0 ∫ f ( x ) dx = ∫ f ( a − x ) dx
Apply the property: 0 0

a a π
( π − x ) dx
∫ f ( x ) dx = ∫ f ( a − x ) dx I= ∫ 1 + sin 2 x  ...(2)
0 0 0

π/2 Add (1) and (2)


I= ∫ log ( cot x ) dx  ...(2) π
0 2I = π∫  1  dx
Now find (1) + (2). It gives I = 0.  1 + sin 2 x 
0
π2 ; 1 1
13. Since =
2ab 1 + sin 2 ( π − x ) 1 + sin 2 x
π
x dx
Hint: I = ∫ a 2 cos2 x + b2 sin 2 x  ...(1) π/2
dx
0 \ 2I = 2π ∫
Apply the property: 0
1 + sin 2 x
a a π/2
sec2 x dx
∫ f ( x ) dx = ∫ f ( a − x ) dx ⇒ I= π∫
1 + 2 tan 2 x
0 0 0
π
( π − x ) dx 1 tan −1  2 tan x  π/2
I= ∫ a 2 cos2 x + b2 sin 2 x  ...(2) ⇒ I= π
2  0
0
Add (1) and (2)
π
dx
= π 1  tan −1
2 { 2 tan π
2 ( )} − tan −1 { }
2 tan ( 0) 

2I = π∫ 2
0
a cos 2
x + b 2 sin 2 x = π  tan −1 ( ∞ ) − tan −1
2
( 2 ×0 
 )
Now it is same as Q.No. 1 (Type-III) of Exercise 7.11
2
π2 ; = π  π − 0 = π
14. 2  2  2 2
4
π
Hint: I =
x tan x dx
∫ sec x cosec x 16. π ( π − 2);
2
0 π
π x sin x dx
I= ∫ x sin x dx 
2
...(1)
Hint: I = ∫ 1 + sin x  ...(1)
0
0
Apply the property: Apply the property
a a
a a
∫ f ( x ) dx = ∫ f ( a − x ) dx
∫ f ( x ) dx = ∫ f ( a − x ) dx 0 0
0 0
π π
( π − x ) sin ( π − x ) dx
∫ ( π − x )sin ( π − x ) dx ∫
2
I= I=
1 + sin ( π − x )
0 0
π π
( π − x ) sin x dx 
∫ ( π − x )sin =∫
2
I= x dx  ...(2) ...(2)
1 + sin x
0 0

Integrals 237
E:\AMIT_WORKS\Exam_Guru\EG_Mathematics-12_(working_02-06-2022)\EG_Mathematics-12_working\Open_Files\Chap_7\Chap_7
\ 17-Aug-2022 Amit Proof 5 Reader’s Sign _______________________ Date __________

Add (1) and (2) π/2 π/2


I= π ∫ logsin 2x dx − π log 2 ∫ dx
( )
π
sin x dx 2 2
2I = π∫ 0 0
1 + sin x
( )
π/2
I= π π π−
0

2 ∫ log (sin 2x ) dx − 2 log 2 2
0
 (1 + sin x ) − 1
π
0
2I = π∫  dx
1 + sin x 
0
π/2 2
I= π π
∫ logsin ( 2x ) dx − 4 (log 2)
π
1  2
2I = π∫ 1 − dx 0
dz
 1 + sin x 
0 Put 2x = z ⇒ dx =
2
π
π 1 x=0 then 0 = z
2I = π  x  − π∫ dx
0 1 + sin x x= π then 2 × π = z ⇒ z=p
0
2 2
π
2I = π 2 − π∫  1 − sin x  dx π 2
 cos 2 x  \ I = π ∫ log (sin z ) dz − π ( log 2)
0 4 4
0
π
2I = π − π∫ sec x − sec x tan x  dx
2 2 π 2
I = π ∫ logsin x dx − π ( log 2)
0 4 4
Now evaluate. 0
 b b 
2 Q ∫ f ( x ) dx = ∫ f ( z ) dz 
17. −π (log 2);  a a 
3
π 2 2
I = π × I − π ( log 2) = I − π ( log 2)
Hint: I = ∫ x log (sin x ) dx  ...(1)
4 π 4 4 4
0
 π/2 
Apply the property
Q I = π ∫ log (sin x ) dx 
a a  0 
∫ f ( x ) dx = ∫ f ( a − x ) dx 3I = − π 2 log 2 ⇒ I = π 2
0 0
4 4
( ) − ( log 2)
π
3
b−a
I= ∫ ( π − x ) log sin ( π − x ) dx 18.
2
;
0
f ( x ) dx
b
π Hint: I = ∫ f (a + b − x) + f ( x)  ...(1)
I= ∫ ( π − x ) logsin x dx  ...(2) a
0 On applying the property
Add (1) and (2) b b
π ∫ f ( x ) dx = ∫ f ( a + b − x ) dx
2I = π∫ log (sin x ) dx a a
0
f ( a + b − x ) dx
b
Since log sin(p – x) = log sin x we get I = ∫ f a + b − {a + b − x} + f ( a + b − x )
π/2 a

\ 2I = 2π ∫ log (sin x ) dx b
f ( a + b − x ) dx
0 I= ∫ f ( x) + f (a + b − x)  ...(2)
π/2 a
I = π ∫ log (sin x ) dx  ...(3) Add (1) and (2)
0 b
b
Apply the property 2I = ∫ dx =  x  a = b − a
a a a

∫ f ( x ) dx = ∫ f ( a − x ) dx I= b−a
0 0 2
π/2
19. 0
I = π ∫ log cos x dx  ...(4) π/2

0 Hint: I = ∫ (sin 2x ) log ( tan x ) dx  ...(1)


0
Add (3) and (4) Apply the property
π/2 a a
2I = π ∫ log (sin 2x ) − log 2 dx ∫ f ( x ) dx = ∫ f ( a − x ) dx
0 0 0

238 Mathematics–12
E:\AMIT_WORKS\Exam_Guru\EG_Mathematics-12_(working_02-06-2022)\EG_Mathematics-12_working\Open_Files\Chap_7\Chap_7
\ 17-Aug-2022 Amit Proof-5 Reader’s Sign _______________________ Date __________

( )
π/2 2π 2π
  dx dx
I= ∫ sin ( π − 2x ) log  tan π − x  dx I= ∫ 1 + esin(2π − x) = ∫ 1 + e− sin x
 2
0 0 0
π/2

I= ∫ sin 2x log (cot x ) dx  ...(2) ⇒ I= esin xdx
∫ 1 + esin x  ...(2)
0
0
π/2
2I = ∫ sin 2x log ( tan x × cot x ) dx Add (1) and (2)
0

π/2  1 esin x 
⇒ 2I = ∫ (sin 2x ) log (1) dx = 0 2I = ∫ 1 + esin x + 1 + esin x  dx
0
0
I=0 2π

20. π
2I = ∫ dx =  x  0 = 2π − 0    ⇒ I = p
0
4 π
a 24.
dx 2
Hint: I = ∫
0 x + a2 − x2 π
ecos x
Put x = a sin q ⇒ dx = a cos q dq
Hint: I = ∫ ecos x + e− cos x dx  ...(1)
0
When x = 0 then 0 = a sin q ⇒q=0 Apply the property:
When x = a then a = a sin q ⇒ θ= π a a
2 f ( x ) dx =
π/2 π/2
∫ ∫ f ( a − x ) dx
a cos θ d θ cos θ d θ 0 0
\ I= ∫ a sin θ + a cos θ
= ∫ sin θ + cos θ π
0 0 ecos(π − x)
Apply the property
I= ∫ ecos(π − x) + e− cos(π − x) dx
0
a a

∫ f ( x ) dx = ∫ f ( a − x ) dx
π
e − cos xdx
0 0
I= ∫ e− cos x + ecos x  ...(2)
0
and add. It gives
π/2
Add (1) and (2)
2I = ∫ dx π
π
0 2I = ∫ dx =  x  0 = π ⇒I= π
2
Now evaluate. 0
2π 2
π (log 2) 25.
21. 3 2
8 π
log ( x + 1) dx x sin x dx
1
Hint: I = ∫ x2 + 1 Hint: I = ∫ 1 + 3 cos2 x  ...(1)
0 0

Substitute: x = tan θ ⇒ dx = sec2 q dq Applying the property


log (1 + tan θ ) sec2 θ d θ
π/4
a a
I= ∫ ( tan θ + 1)
2 ∫ f ( x ) dx = ∫ f ( a − x ) dx
0
0 0
π/4
I= ∫ log (1 + tan θ) d θ
π
( π − x ) sin x dx 
0
I= ∫ 1 + 3 cos 2 x
...(2)
0
22. − π ( log 2)
2 Add (1) and (2)
Hint: See the solution Q.No. 7 (Type-II). π
sin x dx
23. p 2I = π∫
0
1 + 3 cos 2 x

dx
Hint: I = ∫ 1 + esin x  ...(1) Now put cos x = z.
0
8
Apply the property 26.
3
a a 2
x 2 dx
∫ f ( x ) dx = ∫ f ( a − x ) dx Hint: I = ∫ 1 + 5x . ...(1)
0 0 −2

Integrals 239
E:\AMIT_WORKS\Exam_Guru\EG_Mathematics-12_(working_02-06-2022)\EG_Mathematics-12_working\Open_Files\Chap_7\Chap_7
\ 17-Aug-2022 Amit Proof 5 Reader’s Sign _______________________ Date __________

Apply the property 1 8+8


=
3
[ ] = 163
b b

∫ f ( x ) dx = ∫ f ( a + b − x ) dx I=
8
a a 3
a3
27.
2  (2 − 2 − x) 2 2
x dx2 3
I= ∫  1 + 5(2 − 2 − x)  dx = ∫ 1 + 5−x Hint: Same solution as given in Q.No.26
−2   −2
IV. 1. Hint: See the solution as given in Q.No.17
2
x 25 x dx 2. Hint: See the solution as given in Q.No.17
I= ∫  ...(2)
1 + 5x
−2 π2
3. ;
16
Add (1) and (2)
Hint: First apply the property:

2I = ∫
2
(
x2 1 + 5x ) dx = 2 a a
∫x
2

−2 (1 + 5 ) x
−2
dx ∫ f ( x ) dx = ∫ f ( a − x ) dx
0 0

Add the two integral.


2I = 1  x3 
2
Now divide numerator and denominator by cos4x and put tan2x
3 −2
= z.

Topic 13. Odd and Even Functions


Even Functions: If f (–x) = f (x) then the function f (x) is called Let f (x) = x3 + sin3 x
even function. ⇒ f (–x) = (–x)3 + sin3 (–x) = –x3 – sin3 x
= –(x3 + sin3 x) = –f (x)
For example, if f (x) = 3x2 + 4
\ f (x) is an odd function.
then f (–x) = 3(–x)2 + 4 = 3x2 + 4 = f (x)
π/2
⇒ f (x) = 3x2 + 4 is an even function. \ I= ∫ (x 3
)
+ sin 3 x dx = 0
−π / 2
Odd Functions: If f (–x) = –f (x) then the function f (x) is called
π/2
odd function. Example 2. Evaluate: ∫ sin 2 x dx
−π / 2
For example, f (x) = x3 + x
π/2
⇒ f (–x) = (–x)3 + (–x) = –x3 – x Solution. I= ∫ sin 2 x dx
−π / 2
⇒ f (–x) = – (x3 + x) = – f (x)
Let f (x) = sin2 x
⇒ f (x) is an odd function.
⇒ f (–x) = f (x) ⇒ f (x) is an even function
Property of Even Functions
π/2 π/2
a a \ I= ∫ sin 2 x dx = 2 ∫ sin 2 x dx
∫ (even function ) dx = 2∫ ( even function ) dx −π / 2 0
−a 0
π/2 π/2
1 − cos 2x
Property of Odd Functions
=2∫
2
dx = ∫ (1 − cos 2x ) dx
0 0

a π/2
 x − sin 2x 
∫ (odd function ) dx = 0 =
 2  0
−a
π/2
Example 1. Evaluate: ∫
−π / 2
(x 3
+ sin 3 x dx ) (
=π − 1 sin π − 0 − 1 sin 0
2 2 2 )( )
( )
π/2
Solution. I= ∫ (x 3
+ sin 3 x dx )
π 1
= − × 0 − ( 0 − 0) =
2 2
π
2
−π / 2

240 Mathematics–12
E:\AMIT_WORKS\Exam_Guru\EG_Mathematics-12_(working_02-06-2022)\EG_Mathematics-12_working\Open_Files\Chap_7\Chap_7
\ 17-Aug-2022 Amit Proof-5 Reader’s Sign _______________________ Date __________

EXERCISE 7.13
I. Multiple Choice Questions (MCQs) π/2
π 4. The value of ∫ (cos x + x 5 sin 4 x) dx is
1. The value of ∫ (1 − x ) sin x cos x dx is
2 2
−π/2
−π (a) 0 (b) 1 (c) –2 (d) 2
π3 7 1/ 2
1+ x
(a) 0 (b) π − (c) 2p – p3 (d) − 2π 3
3 2 5. The value of ∫
−1/ 2
cos x log
1− x
dx is
π/2
1
2. ∫
2 2
sin x cos x ( sin x + cos x ) dx = (a) 0 (b)
−π/2 2
1
2 4 2 8 (c) − (d) None of these
(a) (b) (c) (d) 2
15 15 5 15
1
Evaluate each of the following Integrals
sin x − x 2
3. The value of ∫−1 3 − | x | dx is II. Long Answer Type Questions
a

∫ log  x + x 2 − 1 dx
1
sin x − x 2 1. Evaluate:
(a) 0 (b) 2 ∫ dx 
3−|x| −a
0
π/2
4 + 3 sin x 
log 
1
− x2 1
(c) 2∫ dx (d) 2 ∫
sin x − x 2
dx 2. Evaluate: ∫  4 − 3 sin x 
dx
0
3−|x| 0
3−|x| −π / 2

Answers 7.13
π
 5 − 3 2 4
∫ (1 − x ) sin x cos = 2 =2× =
2 2
I. 1. (a) Let I= x dx
 15  15 15
−π

Here, f(x) = (1 – x2) sin x cos2 x Hence, (b) is the correct answer.
1 1 1
\ f(– x) = [1 – (–x) ] sin (– x) cos (– x)
2 2 sin x − x 2 sin x x2
3. (c) I= ∫−1 3 − | x | dx = −∫1 3 − | x | dx − −∫1 3 − | x | dx
= – [1 – (–x2)] sin x cos2 x = –f (x)
\ I= 0 sin x
f(x) = is an odd function ;
 [ Here f(– x) = – f (x) so f is an odd function] 3−|x|

Hence, (a) is the correct answer. x2


f(x) = is an even function
π/2 π/2 3−| x|
2. (b) I= ∫ sin 3 x cos 2 x dx + ∫ sin 2 x cos3 x dx = I1 + I2 1
x2 − x2
1
−π/2 −π/2
\ I = 0 − 2∫ dx = 2∫ dx
3−| x| 3−| x|
Since sin3x . cos2 x is an odd function 0 0

π/2
Hence, (c) is the correct answer.
\ I1 = ∫ sin 3 x cos 2 x dx = 0 π/2 π/2 π
x5 sin 4 x dx = [sin x ] 2 π + I1
−π/2 4. (d) I=
−π/2
∫ cos x dx + ∫
−π/2

2
Since sin2 x cos3 x is an even function
 π  π 
π/2 π/2 = sin − sin  −   + I1
 2  2 
∫ ∫ sin
2 3 2 3
\ I2 = sin x cos x dx = 2 x ⋅ cos x dx
−π/2 0
 π π
π/2 1 =  sin + sin  + I1 = 2 + I1
 2 2
= 2 ∫ sin x (1 − sin x) cos x dx = 2 ∫ t (1 − t ) dt
2 2 2 2

π/2
0 0

1
I1 =
−π/2
∫ x5 sin 4 x dx
= 2 ∫ ( t 2 − t 4 ) dt  [Put sin x = t ⇒ cos x dx = dt]
0
Here, f(x) = x5 sin4 x
⇒ f(–x) = (–x)5 sin4 (–x) = –x5 (–sin x)4 = –x5 sin4 x = –f(x)
π
When x = 0, t = 0 and when x = ,t=1 \ f(x) is an odd function.
2
π/2

t t  3
 1 1
5 1 \ ∫ x5 sin 4 x dx = 0. Thus, I = 2 + 0 = 2
\ I = 2  −  = 2 −  −π/2

 3 5 0  3 5 Hence, (d) is the correct answer.

Integrals 241
E:\AMIT_WORKS\Exam_Guru\EG_Mathematics-12_(working_02-06-2022)\EG_Mathematics-12_working\Open_Files\Chap_7\Chap_7
\ 19-Aug-2022 Amit Proof 5 Reader’s Sign _______________________ Date __________

1/ 2 Here, f(x) is an odd function of x.


1+ x
5. (a) I= ∫ cos x log
1− x
dx 1/ 2
1+ x
∫ cos x log dx = 0
−1/ 2

\
1+ x −1/ 2
1− x
f(x) = cos x log
1− x Hence, (a) is the correct answer.
−1
1− x 1+ x

\ f(–x) = cos ( − x) log = cos x log  II. 1. 0; Hint: Prove that the function log  x + x 2 + 1 is an odd
1+ x  1 − x   
function.
1+ x
= − cos x log  = − f ( x) 2. 0; Hint: Prove that the function is an odd function.
 1 − x 

Case Based Question


1. The bridge connects two hills 100 feet apart. The arch on 50

the bridge is in a parabolic form. The highest point on the (iii) The integrand of the integral ∫ x 2 dx is ...... function.
−50
bridge is 10 feet above the road at the middle of the bridge (a) even (b) odd
as seen in the figure.
(c) neither odd nor even (d) none
(iv) The area formed by the curve x2 = 250y, x-axis, y = 0
and y = 10 is
1000 2 4
(a) (b)
3 3
1000
(c) (d) 0
3

(v) The area formed between x2 = 250y, y-axis, y = 2 and
y = 4 is
1000
(a) (b) 0
3

Based on the information given above, answer the 1000 2
(c) (d) none of these
following questions. 3

(i) The equation of the parabola designed on the bridge is Ans. (i) (b) x2 = –250y
(a) x2 = 250y (b) x2 = –250y
1000
(c) y = 250x
2
(d) y2 = 250y (ii) (a)
3
50 x2
(ii) The value of the integral ∫ −50 250
dx is (iii) (a) even
1000
1000 250 (iv) (c)
(a) (b) 3
3 3
(v) (d) none of these
(c) 1200 (d) 0

Author’s Comments
Questions based on following types are very important for Exams. So, students are advised to revise them thoroughly.
1. Questions of Integration based on (a) partial fractions, (b) by-parts
2. Questions based on Special Integrals.
3. Questions based on Definite Integrals by substitution.
4. Questions based on properties of definite integrals.

242 Mathematics–12
E:\AMIT_WORKS\Exam_Guru\EG_Mathematics-12_(working_02-06-2022)\EG_Mathematics-12_working\Open_Files\Chap_7\Chap_7
\ 17-Aug-2022 Amit Proof-5 Reader’s Sign _______________________ Date __________

IMPORTANT FORMULAE

1.  x n +1  2.  xn + 1 
∫ x dx =  n + 1 + C, n ≠ − 1 ∫ ax dx = a  n + 1 + C, n ≠ − 1
n n

3.
∫  f ( x ) + g(x) + φ (x) dx = ∫ f ( x ) dx + ∫ g(x) dx + ∫ φ(x) dx + C
(ax + b) n + 1
∫ (ax + b) dx =
n
4. ∫ k dx = kx + C 5.
a(n + 1)
+ C, n ≠ − 1

dx −1 dx −1
6. ∫ x n = (n − 1) x n − 1 + C, n ≠ 1 7. ∫ (ax + b)n = a (n − 1) (ax + b)n − 1 + C, n ≠ 1
dx  dx  1
8. ∫ x
= log | x | + C 9. ∫  ax + b  = a log | ax + b | + C
 dx  2 ax + b
∫e
x
10. ∫  = +C 11. dx = e x + C
ax + b  a

e ax + b ax
∫ a dx = log a + C
x
∫ e dx =
ax + b
12. +C 13.
a

a mx + n
∫a ∫ sin x dx = − cos x + C
mx + n
14. dx = +C 15.
m log a

16. ∫ cos x dx = sin x + C 17. ∫ tan x dx = log | sec x | + C


18. ∫ cot x dx = log | sin x | + C 19. ∫ sec x dx = log | sec x + tan x | + C
20. ∫ cosec x dx = log | cosec x − cot x | + C = − log | cosec x + cot x | + C
∫ sec ∫ cosec x dx = − cot x + C
2 2
21. x dx = tan x + C 22.

23. ∫ (sec x tan x) dx = sec x + C 24. ∫ (cosec x cot x) dx = − cosec x + C


f ′ ( x ) dx f ′ ( x ) dx
25. ∫ = log f ( x ) + C 26. ∫ = 2 f ( x) + C
f ( x) f ( x)
n +1
n  f ( x ) dx 1 x−a
27. ∫  f ( x ) f ′ ( x ) dx = n + 1 + C, n ≠ −1 28. ∫ x 2 − a 2 = 2a log x+a
+C

29.
dx
∫ a 2 − x 2 = 2a log
1 a+ x
a−x
+C 30.
dx
∫ a2 + x2 =
1
a
tan −1
x
a
+C ()
dx dx
31. ∫ x2 + a2
= log x + x2 + a2 + C 32. ∫ 2 2
= log x + x2 − a2 + C
x −a

33. ∫ a −x
dx
2 2
= sin −1 () x
a
+C 34. dx
∫ ( x ± B)2 − A 2 = 2A log
1 (x ± B) − A
(x ± B) + A
+C

dx 1 A + (x ± B) dx 1 −1 (x ± B)
35.
∫ A 2 − ( x ± B)2 = 2A log A − (x ± B)
+C 36. ∫ ( x ± B)2 + A 2 = A tan A
+C

dx dx
37. ∫ = log (x ± B) + (x ± B) 2 − A 2 + C 38. ∫ = log (x ± B) + (x ± B) 2 + A 2 + C
( x ± B) 2
−A 2
( x ± B) 2
+A 2

Integrals 243
E:\AMIT_WORKS\Exam_Guru\EG_Mathematics-12_(working_02-06-2022)\EG_Mathematics-12_working\Open_Files\Chap_7\Chap_7
\ 17-Aug-2022 Amit Proof 5 Reader’s Sign _______________________ Date __________

dx (x ± B)
39. ∫ = sin −1 +C
A − ( x ± B)
2 2 A

40. Integration by Parts

∫ f ( x ) g ( x ) dx = f ( x ) ∫ g ( x ) dx − ∫  f ′ ( x ) ∫ g ( x ) dx  dx

a2 x 2
∫  f ( x ) + f ′ ( x ) e dx = f ( x ) e + C ∫ x 2 + a 2 dx = log x + x 2 + a 2 + x + a2 + C
x x
41. 42.
2 2

43. ∫ x 2 − a 2 dx =
−a 2
2
log x + x 2 − a 2 +
x
2
x2 − a2 + C 44. ∫ a 2 − x 2 dx =
a2
2
sin −1 +()
x x
a 2
a2 − x2 + C

45. Properties of definite integrals

b b b a

(a) ∫ f ( x ) dx = ∫ f ( z ) dz (b) ∫ f ( x ) dx = −∫ f ( x ) dx
a a a b
a d b c d
(c) ∫ f ( x ) dx = 0 (d) ∫ f ( x ) dx = ∫ f ( x ) dx + ∫ f ( x ) dx + ∫ f ( x ) dx
a a a b c
a a b b
(e) ∫ f ( x ) dx = ∫ f ( a − x ) dx (f) ∫ f ( x ) dx = ∫ f ( a + b − x ) dx
0 0 a a
2a a a
(g) ∫ f ( x ) dx = ∫ f ( x ) dx + ∫ f ( 2a − x ) dx
0 0 0
2a a
(h) ∫ f ( x ) dx = 2∫ f ( x ) dx if f ( 2a − x ) = f ( x )
0 0
= 0 if f (2a – x) = –f (x).

a a
(i) ∫ f ( x ) dx = 2∫ f ( x ) dx , when f (x) is an even function i.e., f (– x) = f (x).
−a 0
= 0, when f (x) is an odd function i.e., f (– x) = – f (x).

COMMON ERRORS
ERRORS CORRECTIONS

3x + 4 )( 8
( 3x + 4 )
8

∫ (3x + 4) dx = ∫ (3x + 4) dx =
7 7
(i) +C. (i) +C
8 3×8
Generally students forget to divide by coefficient of x.
(ii) Generally students write
(ii) ∫ sin x dx = − cos x + C
∫ sin x dx = cos x + C
and ∫ cos x dx = sin x + C
and ∫ cos x dx = − sin x + C

(iii) Students find it difficult to identify and use the properties (iii) Learn the properties well and practice as much as you can.
of definite integral.

244 Mathematics–12
E:\AMIT_WORKS\Exam_Guru\EG_Mathematics-12_(working_02-06-2022)\EG_Mathematics-12_working\Open_Files\Chap_7\Chap_7
\ 17-Aug-2022 Amit Proof-5 Reader’s Sign _______________________ Date __________

REVISION CHART

Integration
It is an important concept in mathematics and, together with differentiation,
is one of the two main operations in Calculus.

Type I Type III


Type II
When denominator of the component fraction When denominator of the composite fraction
When denominator of the composite fraction
can be factorised into linear factors. has linear and quadratic factors.
has linear factors and squares of linear fraction.
x 2 + 4x + 7 4 x 2 − 7x + 8
3x 2 + 4 x + 7
( x − 1)( x + 2)( x − 3) ( x − 4)( x − 3)( x − 1)2 (
( x − 1) x 2 + 4 x + 9 )
A B C A Bx + C
= + + A B C D = +
x −1 x + 2 x − 3 = + + +
x − 4 x − 3 x − 1 ( x − 1)2 (
x − 1 x 2 + 4x + 9 )

Resolution of Composite Fractions into Partial Fractions


All the following formulae are applicable when degree of the numerator of the composite fraction
is smaller than degree of the denominator.

Type IV
ax 2 + b
When denominator and numerator of the composite fraction has pure quadratic factors, i.e.,
(cx 2
)(
+ d ex 2 + f )
az + b
Put x2 = z and write the above composite fractions
(cz + d )(ez + f )
Now change it into partial fraction and before integration put z = x2 and then integrate.

Integration by Parts
This method is used when the integrand is a product of two function. First decide which function of the two, is easily
differentiable and which function is easily integrable. Take the function which is easily differentiable as first function and
write it at first place and use the formulae:
∫ f ( x ) g ( x ) dx = f ( x ) ∫ g ( x ) dx − ∫ f ′ (x ) ∫ g ( x ) dx  dx
When the integral is in the form: ∫ f ( x ) + f ′ ( x ) e x dx , write it as ∫ f ( x ) e x dx + ∫ f ′ ( x ) e x dx
Evaluate only first integral, by parts, second integral will get cancelled.
Identity: ∫ e x f ( x ) + f ′ ( x ) dx = e x f ( x ) + C

Integrals 245
E:\AMIT_WORKS\Exam_Guru\EG_Mathematics-12_(working_02-06-2022)\EG_Mathematics-12_working\Open_Files\Chap_8\Chap_8
\ 17-Aug-2022 Amit Proof-4 Reader’s Sign _______________________ Date __________

Topics Covered
8 Area Between Curves

8.1 Area between Curves 8.2 Area between a Parabola and a Line
8.3 Area between two Parabolas 8.4 Area of Circles
8.5 Area of an Ellipse 8.6 Area between a Circle and a Line
8.7 Area between Ellipse and Line 8.8 Area between two Circles
8.9 Area between Circle and Parabola

C hapter map
AREA

Area between Curves Area between Simple Curves

Area between two circles Area between Ellipse and Line


& two parabolas

Area between a Circle and a Line


Area between circle
and parabola
Area between a Parabola and a Line

Area between Ellipse and Parabola

Topic 1. Area Between Curves


Take two curves: y = f (x) and y = g(x). Assume that these two curves are intersecting at x = a on the left hand side and x = b at right
hand side. Now draw the graph of these two curves.
Area between two curves y = f (x) and y = g(x) intersecting at x = a and x = b when integral is evaluated along x-axis.
Right point of intersection of two curves
i .e., x = b b

∫ [ y of upper curve − y of lower curve]dx or Area = ∫ [ f ( x) − g ( x)]dx


Left point of intersection of two curves a
i .e., x = a

246
If we do not want full area between two curves y = f (x) and
y = g(x), but we want to find area of a limited part between curves
y = f(x) and y = g(x) between x = c and x = d, first draw their graph. upper point of
intersection
We have drawn their graphs in all possible different positions: [ x of right side curve − x of left side curve] dy

lower point of
intersection
b
⇒ Area = ∫  f ( y ) − g ( y ) dy
a
Note: 1. Equation of x-axis is: y = 0 and equation of y-axis is
x = 0.
2. UC = Upper curve, LC = Lower curve, LPI = left point
of intersection, RPI = Right point of intersection and
RA = Required area.

Area Enclosed between Straight Lines


Example 1. The area of the region bounded by the curve y = x +
1 and the lines x = 2 and x = 3 is
7 9
(a) sq. units (b) sq. units
2 2
11 13
(c) sq. units (d) sq. units
2 2
Sol. Given equation of lines are
y = x + 1, x = 2 and x = 3

Area between these two curves y = f (x), y = g(x), x = c and


x = d is
d Required area
∫( )
yof upper curve − yof lower curve dx 3
 x2 
3

c = ∫ ( x + 1) dx =  + x 
2  2 2
d 9  4 
=  + 3 −  + 2
or Area = ∫  f ( x ) − g ( x ) dx 2  2 
c 15 7
= − 4 = sq.units
2 2
Take two curves x = f (y) and x = g(y) intersecting at y = a Hence, the correct option is (a).
on the lower side and y = b on the upper side. Then area between Example 2. The area of the region bounded by the curve x = 2y
them will be calculated as discussed below: + 3 and the lines y = 1 and y = –1 is
3
Area between curves x = f (y) and x = g(y) intersecting at y = (a) 4 sq. units (b) sq. units
2
a and y = b when integral is evaluated along y-axis. (c) 6 sq. units (d) 8 sq. units

Area Between Curves 247


E:\AMIT_WORKS\Exam_Guru\EG_Mathematics-12_(working_02-06-2022)\EG_Mathematics-12_working\Open_Files\Chap_8\Chap_8
\ 17-Aug-2022 Amit Proof-4 Reader’s Sign _______________________ Date __________

Sol. Given equations of lines are x = 2y + 3, y = 1 and y = –1

\ Required area = area of ∆BCD + area of ∆BEF


In ∆BCD: UC: y =5 – x, LC: y = 0,
Required area
LPI: x = – 2, RPI: x = 5.
1
In ∆BEF: UC: y = 0, LC: y = 5 – x,
= ∫ (2 y + 3) dy
−1 LPI: x = 5, RPI: x = 8.
1 2 1
 y  −1 + 3 [ y ] 1−1
= 2. 5 8


2
= (1 − 1) + 3 (1 + 1) = 6 sq.units
R.A. = ∫ yUC dx + ∫ yLC dx
−2 5
Hence, the correct answer is (c). 5 8
∴ R.A. =
∫ ([(5 − x) − 0] dx ) + ∫ [0 − (5 − x)] dx
−2 5
5 8

Area = ∫ (5 − x ) dx − ∫ (5 − x ) dx
−2 5
Example 3. Using integration, find the area of the region bounded
5 8
by the lines: x + y = 5 and the ordinates x = – 2, x = 8 and x-axis.
= − ∫ ( x − 5) dx + ∫ ( x − 5) dx = 29 (unit)2
Hint: The given equations are: x + y = 5, x = – 2, x = 8, and x-axis. −2 5
Their graph is shown in the fig.
EXERCISE 8.1
I. Multiple Choice Questions- (MCQ) (c) 15 sq units (d) 21 sq units
1. The area of a triangular region whose sides have the equations II. Long Answer Type Questions
y = 2x + 1 , y = 3x + 1 and x = 4 is 1. Find the area of the region bounded by the lines:
(a) 16 sq units (b) 4 sq units y = 3x + 2; x-axis and ordinates x = – 1, x = 1.

(c) 8 sq units (d) 20 sq units (Use method of integration).
2. The area bounded by the curve y = x, the x – axis and the
2. Using integration, find the area of the region bounded
ordinates x = – 2 and x = 3 is
17 by lines: y = x + 1, the x-axis and ordinates, x = – 2 and
(a) 13 sq. units (b) sq units x = 3. [Delhi 2007 (C)]
2 2
Answers 8.1
I. 1. (c) 8 sq units B (4,13)
Hint: Here equation of sides of triangle ABC are
Y
y = 2x + 1  ...(i) +
1
y = 3x + 1  ...(ii) =
3x
C (4,9)
x = 4  ...(iii)
y

Solving (i) and (ii), we have co-ordinates of point A (0, 1) 1


2x +
Solving (ii) and (iii), we have co-ordinates of point B (4, 13) y= x=4
A(0,1)
Solving (i) and (iii), we have co-ordinates of point C (4, 9)
X X
\ Required area = (area under the line AB, x-axis and x = 0,
x = 4) – (area under the line AC, x-axis and x = 0, x = 4) O D (4,0)
4 4

= ∫ ( 3x + 1) dx − ∫ ( 2 x + 1) dx
0 0
Y
4 4
 3x   2x2
2
  3 × 4 2  
= 
 2
+ x − 
0  2
+ x
0
= 
 2  
(
+ 4  − 0  −  ( 4 ) + 4 − 0 
2
)
248 Mathematics–12
= 28 – 20 = 8 sq. units. −2
LPI : x = ; RPI : x = 1
2. (a) 13 sq. units 3
2
−2 / 3 1
Hint: Here the curve is y = x, which represents a straight line
passing through origin and making an angle 45° with x-axis.
R.A. = ∫ [0 − (3x + 2)] dx + ∫ [(3x + 2) − 0] dx
−1 −2 / 3
We have to find out the area of shaded region in the figure.
For –2 ≤ x ≤ 0, y < 0 and 0 ≤ x ≤ 3, y > 0 −2 / 3 1
\ Required area = area of D OAB + area of D OCD ⇒ R.A. = − ∫ (3x + 2) dx + ∫ (3x + 2) dx
3 0 3 0 3 0 −1 −2 / 3
 x2   x2 
= ∫
0
y dx + ∫ ( − y ) dx
−2
= ∫ x dx − ∫ x dx =   −  
 2  0  2  −2 2. 8.5 sq. units
0 −2

 32 02   02 ( − 2 ) 
2
9 13 Hint : Draw the graph of the given equations

=  −  −  −  = +2 = sq. units.
 2 2   2 2  2 2 y = x + 1, x = – 2 and x = 3.

R.A. = area of ∆AEF + area of ∆ACD.

II. 1. 13 sq.units In ∆AEF : UC : y = 0,


3
LC : y = x + 1;
Hint: Draw the graph of the given
equation: LPI : x = – 2;
y = 3x + 2, x = – 1 and x = 1 RPI : x = – 1.
R.A. = Area of ∆ABC + area of
∆AED In ∆ACD : UC : y = x + 1;
In ∆ABC: UC: y = 0; LC : y = 0; LPI : x = – 1, RPI : x = 3
LC : y = 3x + 2; −1 3

LPI : x = – 1, RPI : x =
−2 R.A. = ∫ [0 − (x + 1)] dx + ∫ [(x + 1) − 0] dx
3 −2 −1
−1 3
In ∆AED : UC: y = 3x + 2; = − ∫ (x + 1) dx + ∫ (x + 1) dx
−2 −1
LC: y = 0;

Topic 2. Area between a Parabola and a Line


Parabola: Parabola is the locus of a point which moves in such the equations of the parabolas.
a way that its distance from a fixed point and fixed line is same. Parabolas of the form y = quadratic polynomial in x open either
Equations of Parabolas upwards or downwards. If the coefficient of x2 is positive, the
parabola opens upwards and when the coefficient of x2 is negative,
y = quadratic polynomial in x or
parabola opens downwards.
x = quadratic polynomial in y, represent equations of
Parabola y = x2 – 9, y = x2 + x – 2, will open upwards while the
parabolas.
parabolas y = 16 – x2, y = 12 – x – x2 will open downwards.
y = x2 x = y2
Parabolas of the form x = quadratic polynomials in y open either
y = x2 – 9 x = y2 – 16
on the right hand side or on the left hand side. If the coefficient of
y = x + x – 2 and x = y2 + y – 2
2
y2 is positive then the parabola will open on the right hand side
y = 16 – 9x2 x = 25 – y2 and when the coefficient of y2 is negative then the parabola will
2
y = 12 – x – x x = 7 – y – y2 are examples of open on the left hand side.

Area Between Curves 249


E:\AMIT_WORKS\Exam_Guru\EG_Mathematics-12_(working_02-06-2022)\EG_Mathematics-12_working\Open_Files\Chap_8\Chap_8
\ 17-Aug-2022 Amit Proof-4 Reader’s Sign _______________________ Date __________

Parabolas with equations, x = y2, x = y2 + 16, x = y2 + y – 2 2 3/ 2 4 1 1 2 4


will open on right hand side because coefficients of y2 are positive [ x ]0 − . [ x ]0
=
3 2 2
while the parabolas with equations : x = 25 – y2 and x = 7 – y2 will
open on left hand side because the coefficients of y2 are negative. 2 3/ 2 1 2 2 1
= [(4) − 0] − [(4) − 0] = × 8 − × 16
3 4 3 4
How to Draw Rough Graph of a Parabola?
16 4
Take a parabola equation: y = x2 – 4x – 5. = − 4 = sq. units
3 3
(i) Here the coefficient of x2 is positive, therefore this parabola Hence, the correct answer is (a).
will open upwards. Example 2. Draw a rough graph of the curve: y = x2 – 2x – 3
(ii) In the equation: y = x2 – 4x – 5, put y = 0. and x + y = 9 and using integration find the area between them.
⇒ x2 – 4x – 5 = 0 Solution. The given equations are: y = x2 – 2x – 3 and x + y = 9
⇒ (x + 1) (x – 5) = 0 First equation is: y = quadratic polynomial in x.
⇒ x = – 1, 5. ⇒ It is an equation of a parabola.
This means the parabola will cut x-axis at x = – 1 and Here the coefficient of x2 is positive, so the parabola will open
x = 5. upwards. It will cut x-axis at x = – 1, 3, it will cut y-axis y = – 3.
Draw the rough parabola with this information:
Now solve the two equations:
x2 – 2x – 3 = 9 – x
⇒ x2 – x – 12 = 0
⇒ (x – 4) (x + 3) = 0
⇒ x = – 3, 4
⇒ Two curves intersect at x = – 3, 4.
Now draw perpendicular upwards from x = – 3. It will cut the
parabola at D. Now draw perpendicular upwards from x = 4. This
(iii) In the equation: y = x2 – 4x – 5. perpendicular cuts parabola at E.
Put x = 0 ⇒ y = – 5.
This means the parabola will cut y-axis at (0, – 5).
(iv) Now draw the rough graph of parabola cutting x-axis at
x = – 1 and 5 and y-axis at y = – 5 and opening upwards.
This is the rough graph of the parabola.
Example 1. The area of the region bounded by parabola y2 = x
and the straight line 2y = x is
4
(a) sq. units (b) 1 sq. unit
3
2 1 Now join D and E. This line is the graph of the line x + y = 9.
(c) sq. units (d) sq. units
3 3 Now shade the region between these two curves:
Solution. Given equation of parabola is y2 = x...(i) In the shaded region upper curve is the line x + y = 9 or
and equation of straight line is 2y = x...(ii) y = 9 – x and lower curve is y = x2 – 2x – 3. Left point of
intersection is x = – 3 and right point of intersection x = 4.
4

∫ (9 − x) − (x
2
∴ Required area = − 2x − 3)  dx
−3
4 4

∫( )
2  x 2 x3 
⇒ Area = 12 + x − x dx = 12x + − 
 2 3 − 3
−3



(
A =  48 + 8 −

64
3 )(9
− −36 + + 9 
2

 )
Solving eqns. (i) and (ii) we get


 x
  = x ⇒
2
x2
= x  x2 = 4x


A = 83 −
 (
64 9 
+
3 2  )
2 4 155  343
⇒ A = 83 − = sq units.
 x(x – 4) = 0  \ x = 0, 4  6  6
4 4
x 343
Required area = ∫
0
x dx − ∫ dx
0
2
∴ Required area =
6
sq. units.

250 Mathematics–12
EXERCISE 8.2
I. Multiple Choice Questions- (MCQ) 3. Find the area enclosed by the parabola 4y = 3x2 and the
1. Area lying between the curves y2 = 4x and y = 2x is line 2y = 3x + 12. [AI 2009, Delhi 2010]
1 4. Find the area bounded by the curves : x2 = 4y and the line
(a) 2 sq units (b) sq units : x = 4y – 2.
3 3
3
[Delhi 2005, 2010, 2014 (C), AI 2006, 2013, 2013 (C)]
1
(c) sq units (d) sq units 5. Find the area lying between the curves : y2 = 4x and
4 4
y = 2x.[Delhi 2013 (C), 2017, AI 2017]
2. Area bounded by parabola y2 = x and straight line 2y = x is
6. Find the area of the region bounded by the curves:
4 y = x2 + 2, y = x, x = 0 and x = 3. [AI 2006 (C)]
(a) sq units (b) 2 sq units
3 7. Find the area of the region: {(x, y) : 0 ≤ y ≤ x2, 0 ≤ y ≤ x +
2 1 2, 0 ≤ x ≤ 3}. [CBSE S.P. 2014, 2015]
(c) sq units (d) sq units
3 3 8. Find the area of the region bounded by: y2 = 4x, x = 1,
II. Long Answer Type Questions x = 4 and x-axis in the first quadrant.
1. Find the area of the region bounded by the parabola [Delhi 2006, Foreign 2007]
y = x2 and y = | x |. 9. Find the area enclosed between the parabola y2 = 4ax and
[AI 2002, 2009 (C), 2012 (C), 2011 (C), 2013] the line: y = mx.[Delhi 2007 (C), AI 2006, 2007 (C)]
2. Find the area enclosed by the parabola : y2 = x and the line 10. Find the area of the region: {(x, y) : 0 ≤ y ≤ x2 + 1, 0 ≤ y ≤
y + x = 2 and x-axis in the first quadrant. x + 1, 0 ≤ x ≤ 2} [Delhi 2007 (C), AI 2006, 2007 (C)]
[AI 2005, 2006, 2009]

Answers 8.2
4 Hint:
I. 1. (b) sq unit
3
Hint : We are given y2 = 4x ... (1) and y = 2x  ...(2)
Solving (1) and (2) simultaneously, we get x = 0, 1
These two curves (1) and (2) intersect each other at O(0, 0) and
A (1, 2) Required area
1 1 1 1 1
Draw the graph of y = x2 and y = | x |.
= ∫
0
4 x dx − ∫ 2 x dx = 2∫ x dx − 2∫ x dx
0 0
2

0
Find points of intersection by solving x2 = x and x2 = – x.
It gives : x = 0, x = – 1 and x = 0, x = 1 shaded area on
1
 3 1
LHS = shaded area of RHS.
1
 x2   x2  2 × 2  32  2 1 ∴ Required area = 2 [area of the region (OPAMO)]
= 2   − 2   =  x  −  x  0
3
   2 0 3  0 For region (OPAMO) : UC : y = x, LC : y = x2, LPI: x = 0 and
 2 0 RPI : x = 1.
1
4 3 3
 4 4 1
= (1) 2 − ( 0 ) 2  − (1 − 0 ) = (1) − 1 = − 1 = sq. units
2 ( )
∴ R.A. = 2∫ x − x 2 dx
3  3 3 3 0

Hence, (b) is the correct answer. 7 sq.units


2.
6
Hint: Solve the two equations:
y2 = x and x + y = 2.
It gives : x = 1 and 4 : Draw graph of y2 = x and
x + y = 2.
Required Area = Area of (OCB) + Area of (CAB).

4
2. (a) sq. units
3
II. 1. 1 sq.units
3

Area Between Curves 251


E:\AMIT_WORKS\Exam_Guru\EG_Mathematics-12_(working_02-06-2022)\EG_Mathematics-12_working\Open_Files\Chap_8\Chap_8
\ 17-Aug-2022 Amit Proof-4 Reader’s Sign _______________________ Date __________

For (OCB): UC : y = x, LC : y = 0, LPI : x = 0 and RPI : x = 1


For (CAB): UC : y = 2 – x, LC : y = 0, LPI : x = 1, RPI: x = 2.
5. ()
1 sq.units
3
1 2 Hint : Draw a graph of y2 = 4x and y = 2x.
\ R.A. = ∫ ( )
x − 0 dx + ∫ ( 2 − x ) − 0 dx Find the points of intersection of two curves:
0 1
y2 = 4x and y = 2x ⇒ (2x)2 = 4x ⇒ x = 0, 1
1 2
= ∫ x dx + ∫ ( 2 − x ) dx R.A. = Shaded Area.
0 1

3. 27sq.units
Hint : Solve the two equations: 4y = 3x2 and 2y = 3x + 12, it

gives x = – 2 , 4. Now draw graph of the two curves.

3x + 12 In the Shaded Area: UC : y = 2 x, LC : y = 2x,


In the shaded region: UC : y =
2 LPI : x = 0, RPI : x = 1
3x 2 ,
LC : y = LPI : x = – 2, RPI : x = 4. 1
4
4
(
R.A. = ∫ 2 x − 2x dx )
 2 0
\ R.A. = ∫  3x + 12 − 3x  dx
−2
 2 4  21 sq.units
6.
4 2
3
R.A. = ∫ 8 + 2x − x 2 dx
4 ( ) Hint: Draw graph of: y = x2 + 2, y = x, x = 0 and x = 3.

−2

4. 9 sq.units
8
Hint: Find point of intersection of two curves:

2
y = x and y = x
+2
4 4
2
Solve: x = x + 2 ⇒ x = −1, 2
4 4
Now draw the graph.
R.A = Shaded Area: In the shaded Area ABCO
UC: y = x2 + 2; LC : y = x, LPI : x = 0, RPI : x = 3
3
R.A. = ∫ (x 2 + 2) − x  dx
0

3
(
⇒ R.A. = ∫ 2 − x + x 2 dx )
0

7. 43 sq.units
R.A. = The shaded region AOB 6

x + 2 , LC : y = x 2 , Hint: Solve : y = x2 and y = x + 2.



UC : y =
4 4 It gives: x = – 1, 2.
LPI : x = – 1 and RPI: x = 2
Now draw a graph of y = x2, y = x + 2 and x = 3.
2 2
∴ R.A. =  x + 2 x2 
∫ 
4 4 4 (
−  dx = 1 ∫ 2 + x − x 2 dx ) Total required area = area of the region (OAB) + area of the
−1 −1 region (ACDB)

252 Mathematics–12
In the shaded region:
For region (OAB) : UC : y = x2,
LC : y = 0, LPI : x = 0, RPI : x = 2. UC : y = 2 a x,
LC : y = mx, LPI: x = 0,
For region (ACDB) : UC : y = x + 2, LC : y = 0,
4a
LPI : x = 2, RPI : x = 3. RPI : x = 2
m
2 3 4a / m2

∴ R. Area =
∫ x dx + ∫ ( x + 2) dx
2

0 2
R.A. = ∫  2 a x − mx  dx
 
0

8. 28 sq.units 23
3 10.
6
Hint: Draw the graph of y2 = 4x, x = 1, x = 4 and x-axis.

Solve y = x2 + 1 and y = x + 1 to find their points of intersection:


x2 + 1 = x + 1
R.A. = Shaded region.
In the shaded region. UC : y = 2 x, ⇒ x(x – 1) = 0 ⇒ x = 0, 1
LC : y = 0, LPI : x = 1, RPI : x = 4. ∴ Area = area of (EOBA) + area of (ABDC)
4 4 Region (EOBA) : UC : y = x2 + 1,
 ( )
\ R.A. = ∫  2 x − 0 dx = 2∫ x dx
 LC : y = 0, LPI : x = 0, RPI : x = 1.
1 1
2 Region (ABDC) : UC : y = x + 1, LC : y = 0,
9. 8a sq.units
LPI : x = 1, RPI : x = 2
3m3
1 2
Hint: first solve y2 = 4ax and y = mx.
A = ∫ [(x 2 + 1) − 0] dx + ∫ [(x + 1) − 0] dx
4a 0
It gives: x = 0 and x = 2 . 1
m 1 2
Now draw graph.
⇒ A = ∫ (x 2 + 1) dx + ∫ (x + 1) dx
R.A. = Shaded region. 0 1

Topic 3. Area between two Parabolas


Example 1. Draw the rough sketch of: y = x2 and y2 = 8x and find ⇒ (x2)2 = 8x
the area between these curves, using integration. ⇒ x4 = 8x
Solution. The given equations are: y = x2 and y2 = 8x 4
⇒ x – 8x = 0
First solve these two equations and find their points of
⇒ x(x3 – 8) = 0
intersection.
⇒ x = 0 and x = 2.
y2 = 8x and y = x2
Now draw the graph of the parabola

Area Between Curves 253


E:\AMIT_WORKS\Exam_Guru\EG_Mathematics-12_(working_02-06-2022)\EG_Mathematics-12_working\Open_Files\Chap_8\Chap_8
\ 17-Aug-2022 Amit Proof-4 Reader’s Sign _______________________ Date __________

LC : y = x2,
LPI : x = 0, RPI : x = 2.
2
\ Required Area = ∫ 2 2 x − x 2  dx
0

( )
3 2
 3
=  2 2 × 2 x2 − x 
 3 3
0

3 2
 3
= 4 2 x2 − x 
y = x2 and y2 = 8x.  3 3
0
R.A. = Shaded region.  4 2 × 2 2 8  
In the shaded region: =  −  − (0) 
 3 3 
UC : y2 = 8x
⇒ y = 2 2 x, = 16 − 8  = 8 sq.units
 3 3  3

EXERCISE 8.3

I. Multiple Choice Questions- (MCQ) 3. Draw a rough sketch and find the area of the region
1. The area bounded by the curve 4y = x2 and 2y = 6 – x2 is between the parabola : y2 = 4ax and x2 = 4ay. Use method
(a) 8 sq. units (b) 6 sq. units of integration. [Delhi 2008]
(c) 4 sq. units (d) 10 sq. units 4. Draw a rough sketch and find the area of the region bounded
2. The area included between two curves y2 = 9x and x2 = 9y is by the two parabolas y2 = 8x and x2 = 8y by using method
(a) 21 sq. units (b) 27 sq. units of integration. [AI 2004, Delhi 2012]
(c) 36 sq. units (d) 18 sq. units 5. Draw the rough graphs of the parabolas: y = 2x2 and y = x2
+ 4 and using integration find the area enclosed between
II. Long Answer Type Questions them.
1. Find the area of the region bounded by the two parabolas 6. Draw rough sketch of the parabolas: y = 5x2 – 3 and
y = x2 and y2 = x, using integration. y = 3x2 + 5. Using integration, find the area between them.
2. Prove that the curves : y2 = 4x and x2 = 4y, divide the area 7. Find the area enclosed between the parabolas y = x2 – 9
of the square bounded by x = 0, x = 4, y = 4 and y = 0 into and y = 3 – 2x2. Use method of integration. [Delhi 2009]
three equal parts. Use method of integration.
8. Find the area enclosed between the parabolas: y = 2x2 – 6
[Delhi 2009, A.I. 2015] and y = – x2 – 3. Use method of integration.

Answers 8.3
I. 1. (a) 8 sq. units
In the shaded region: UC: y = x. LC: y = x2, LPI: x = 0,
2. (b) 27 sq. units RPI: x = 1.
1 sq.units 1
II. 1.
3 \ A=∫ ( )
x − x 2 dx
Hint: Solve the two equations: y = x2 and y2 = x to find their 0

points of intersection 2. Hint: Solve the two equations:


It comes: x = 0 and x = 1. Now draw their graphs:  y2 = 4x and x2 = 4y ⇒ x = 0, x = 4, or (0, 0) and (4, 4). Now
R.A. = Shaded region. draw the graph of y2 = 4x, x2 = 4y, x = 0, x = 4, y = 4
and y = 0.]

254 Mathematics–12
In the shaded region (OBQRO):
UC: y = 4, LC: y = 2 x , LPI : x = 0, RPI: x = 4.
4
⇒ Area of the region (OBQRO) = ∫ (4 − 2 )
x dx = 16 sq.units
3
0

In the shaded region (OAQBO):

x2
UC: y = 2 x, LC : y = , LPI : x = 0, RPI: x = 4.
4
4 2

∴ Area of the region (OAQBO) = ∫  2 x − x  dx = 16 sq.units R.A. = Shaded region.
 4 3 2
In the shaded area: UC: y = 2 2 x, LC : y = x ,
0

In the shaded region (OPQAO): 8


LPI: x = 0, RPI: x = 8,
x2 8
 2
UC: y =
4
, LC: y = 0, LPI: x = 0, RPI: x = 4 A = ∫  2 2 x − x  dx = 64 sq.units
 8  3
4
0
 2 
\ Area of the region (OPQAO) = ∫  x − 0 dx = 16 sq. units 5. 32 sq.units
 4  3 3
0
2
3. 16 a Hint: Solve: y = x2 + 4 and y = 2x2.
sq.units
3 ⇒ 2x2 = x2 + 4 ⇒ x = –2, 2
Hint: Find the points of intersection of the parabolas: y2 = 4ax Now draw the graph.
and x2 = 4ay
2
 2
⇒  x  = 4ax ⇒ x4 = 64a3x
 4a 

⇒ x(x3 – 64a3) = 0 ⇒ x = 0 and x = 4a


Now draw the graphs of the parabolas.

R.A. = Shaded region.


In the shaded area : UC: y = x2 + 4, LC: y = 2x2, LPI: x = – 2,
RPI: x = 2
2 2
( )
A = ∫ x 2 + 4 − 2x 2 dx = ∫ 4 − x 2 dx ( )
−2 −2

6. 64 sq.units
3
Hint: Solve: y = 5x2 – 3 and y = 3x2 + 5
R.A. = Shaded region. ⇒ x = – 2, 2. Draw graph.
In the shaded region: UC: y = 2 a x, In the shaded region : UC: y = 3x2 + 5,
LC: y = 5x2 – 3; LPI : x = – 2, RPI: x = 2.
x2
LC: y = , LPI: x = 0, RPI: x = 4a. 2 2
4a
4a
 x2  16a 2 sq. units
Area = ∫   ( )
(3x 2 + 5) − (5x 2 − 3)  dx = 2 ∫ 4 − x 2 dx
Required Area = ∫  2 a x −  dx = −2 −2
 4a  3
0 R.A. = Shaded region.
4. 64 sq.units
3
Hint: First solve the two equations:

y2 = 8x and x2 = 8y for x:

2
 2
⇒  x  = 8x ⇒ x4 = 512x
 8
⇒ x = 0 and x = 8.
Now draw the graph.

Area Between Curves 255


E:\AMIT_WORKS\Exam_Guru\EG_Mathematics-12_(working_02-06-2022)\EG_Mathematics-12_working\Open_Files\Chap_8\Chap_8
\ 17-Aug-2022 Amit Proof-4 Reader’s Sign _______________________ Date __________

In the shaded region : UC: y = 3x2 + 5,


8. 4sq.units
LC: y = 5x2 – 3; LPI : x = – 2, RPI: x = 2.
2 Hint : Solve the equations:

∫ (3x
2
Area = + 5) − (5x 2 − 3)  dx
−2
y = 2x2 – 6 and y = – x2 – 3
⇒ x = – 1, 1
2 Now draw the graph.
(
= 2 ∫ 4 − x 2 dx ) R.A. = Shaded region.
−2
7. 32 sq.units In the shaded region: UC: y = – x2 – 3,
LC: y = 2x2 – 6, LPI : x = – 1,
Hint: Solve the equation:
y = x2 – 9 and y = 3 – 2x2 RPI: x = 1.
⇒ x = – 2, 2 1 1

Now draw the graph:


A = ∫ (− x 2 − 3) − (2x 2 − 6)  dx (
= 3∫ 1 − x 2 dx)
−1 −1

R.A. = Shaded region.


In the shaded region:
UC: y = 3 – 2x2,
LC: y = x2 – 9, LPI: x = – 2,
RPI : x = 2.
2

∫ (3 − 2x ) − (x
2 2
R.A. = − 9)  dx
−2

2
(
= 3 ∫ 4 − x 2 dx )
−2

Topic 4. Area of Circles


The general equation of a circle is: (x – a)2 + (y – b)2 = r2. Here Take a circle (x – 3)2 + (y – 4)2 = 16.
(a, b) is the centre of the circle and r is the radius of the circle. ⇒ (x – 3)2 + (y – 4)2 = 42
Area of the circle purely depends upon its radius r, it does ⇒ centre (3, 4); radius: r = 4
not depend upon its centre (a, b). Take a point centre (3, 4).
Draw two perpendicular lines parallel to x-axis and parallel
How to draw graph of a circle? to y-axis passing through (3, 4).
Take general equation of the circle Now draw a circle with radius 4, as shown.
(x – a)2 + (y – b)2 = r2
Here centre is (a, b) and radius r.
Draw two ⊥ lines passing through (a, b). On the circles,
the right extreme point is A[(a + r), b], left extreme point is
B[(a – r), b], upper extreme point is C[a, (b + r)] and lower
extreme point is D[a, (b – r)]
Now draw a circle passing through these four points.

The two perpendicular lines divide the circle into 4 equal


parts. We shall find the area of one part by integration and multiply
it by 4.
Example 1. The area of the region bounded by the circle x2 + y2 = 1 is
(a) 2 p sq. units (b) p sq. units
(c) 3 p sq. units (d) 4 p sq. units
Sol. Given equation of circle is

256 Mathematics–12
   x2 + y2 = 1  y = 1 − x 2
Since the circle is symmetrical about the axes. 
(
− 8 sin −1 0 +
0
2 )
16 − 0 




)

= 4  8 × + 0 − (8 × 0 + 0) 
 2 
= 4(4p) = 16p sq. units
Check: r= 4
⇒ A = πr2 = π(4)2 = 16π sq. units.
Example 3. Draw a graph of the circle: (x – 2)2 + (y – 3)2 = 25
1 and using integration, find its area.
\ Required area = 4 × ∫ 1 − x 2 dx
Solution. Given equation of the circle is: (x – 2)2 + (y – 3)2 = (5)2
0

x 1 
1 ⇒ Centre (2, 3), r = 5
=4 1 − x 2 + sin −1 x  Draw the circle as shown.
2 2 0
 1 
= 4 0 + sin −1 (1) − 0 − 0
 2 
1 π
= 4 × × = π sq. units
2 2

Hence, the correct answer is (b).


Example 2. Draw a rough graph of the circle: x2 + y2 = 16 and
using integration, find its area.
Solution. Equation of the circle: x2 + y2 = 16. Here centre of the
circle is (0, 0) and radius is 4 units.
Now draw its graphs,
Area of the circle = 4 (shaded area PAD)
we shall find the area of the
25 − ( x − 2) + 3
one part of the circle and 2
In the shaded region: UC:
multiply it by 4.
LC: y = 3, LPI: x = 2, RPI: x = 7
R.A. = Area of
( )
7
the full circle. Area of the circle = 4∫  25 − (x − 2) 2 + 3 − 3 dx
 
In the shaded region 2
7
(OAC): UC: y = 16 − x 2 , = 4∫ 25 − (x − 2) dx
2

LC: y = 0, LPI: 0 and RPI : 2
7
 25 − 1 (x − 2) x−2
x = 4.
= 4  sin + 25 − (x − 2) 2 
 2 5 2 2
( 16 − x − 0) dx
4
Area of the circle = 4∫
( )
2
 25 5
0 ⇒ A= 4 sin −1 1 + 25 − 25
 2 2

()
4

16
= 4  sin
2
−1 x
4
+
x
2

16 − x 2 
0

− ( 25
2
sin − 1 0 +
0
2

25 − 0 
 )
()
4
 −1 x x  ⇒ A = 25π
= 4 8 sin + 16 − x 2 
 4 2 0 ⇒ Area of the circle = 25π sq. units




(
= 4  8 sin −1
4
4()
+
4
2
16 − 16 ) Verification: r = 5.
∴ A = πr2 = 25π sq. units

EXERCISE 8.4
I. Multiple Choice Questions- (MCQ) π π
(c) sq. units (d) sq. units
1. Area lying in the first quadrant and bounded by the circle 3 4
x2+ y2 = 4 and the line x = 0 and x = 2 is  [NCERT] II. Long Answer Type Questions
π 1. Draw a rough sketch of the curve: x2 + y2 – 6x = 40 and use
(a) p sq. units (b) sq. units
2 integration to find its area. [Delhi 2008]

Area Between Curves 257


E:\AMIT_WORKS\Exam_Guru\EG_Mathematics-12_(working_02-06-2022)\EG_Mathematics-12_working\Open_Files\Chap_8\Chap_8
\ 17-Aug-2022 Amit Proof-4 Reader’s Sign _______________________ Date __________

2. Draw a rough graph of the circle : 4x2 + 4y2 = 25 and using 5. Using integration, find the area of the circle: (x – 1)2 +
integration, find its area. (y – 3)2 = 64.
3. Using integration, find the area of the circle (x – 2)2 + 6. Find the area of the circle: x2 + y2 – 4x – 6y – 36 = 0 by
y2 = 49.
using integration.
4. Draw a graph of: x2 + (y – 3)2 = 49 and using integration,
find its area.

Answers 8.4
2 2 2
I. 1. (a) x + y =2 . It is symmetrical about x-axis and y-axis both. Draw the graph.
2 2 2 2 2
y =2 –x ⇒y =± 2 −x

In the first quadrant, y > 0

\ y = 22 − x 2
2
2
Required area = ∫ y dx = ∫
0
0
22 − x 2 dx

 x 2 a2 x
Form =  ∫ a 2 − x 2 dx = a − x 2 + sin −1 
 2 2 a
2
x 2 4 −1 x 
In the shaded region: UC: y = 25 − x 2 ,
Required area =  4 − x + sin
2 2 2  0 4
5
2  π LC: y = 0, LPI: x = 0, RPI: x = .
=  [ 0] + 2sin −1 (1)  = 2 × = π 2
 2  2 5/ 2
Area of the circle: A = 4 ∫ 25 − x 2 dx
Hence, (a) is the correct answer. 4
0
II. 1. 49πsq.units 3. 49πsq.units
Hint: given equation of the circle: Hint: Equation of the circle: (x – 2)2 + y2 = 49. Centre (2, 0),
x2 + y2 – 6x = 40 ⇒ (x – 3)2 + y2 = 49 r=7
Here centre O(3, 0) and r = 7. Draw the graph.
Now draw, its graph. In the shaded region: UC:

49 − ( x − 2)
2
y=

LC: y = 0, LPI :
x = 2, RPI : x = 9
Area of the circle =

( )
9
4∫ 49 − (x − 2) 2 dx
2

4. 49πsq.units
49 − ( x − 3)
2
In the shaded region: UC: Hint: Equation of the circle:
LC: y = 0, LPI: x = 3, RPI: x = 10 x2 + (y – 3)2 = 49. Centre (0, 3), r = 7
10 Draw the graph.
Area of the circle = 4∫ 49 − (x − 3) 2 dx
3

2.25π sq.units
4
Hint:
Given equation of the circle:
4x 2 + 4y 2 = 25 ⇒ x 2 + y 2 = 25
4
5.
Here centre (0, 0) and r =
2

258 Mathematics–12
In the shaded area: UC: y = 49 − x 2 + 3, d 6. 49π
LC: y = 3, LPI : x = 0, RPI: x = 7. Hint: Equation of the circle is:

Area of the circle = x2 + y2 – 4x – 6y – 36 = 0

( )
7 7
⇒ (x – 2)2 + (y – 3)2 = 49

4∫  49 − x 2 + 3 − 3 dx = 4∫ 49 − x 2 dx
  Here: Centre (2, 3) or r = 7
0 0

Draw the graph:


5. 64πsq.units
In the shaded region:
Hint: Circle equation is (x – 1)2 + (y – 3)2 = 64
Centre (1, 3), r = 8 UC: y = 49 − (x − 2) 2 + 3
Draw the graph.
LC: y = 3, LPI : x = 2,
RPI: x = 9
Area of the circle =

( )
9
4∫  49 − (x − 2) 2 + 3 − 3 dx
 
2

9
= 4∫ 49 − (x − 2) 2 dx
2

In the shaded region:

UC: y = 64 − (x − 1) 2 + 3,
LC: y = 3; LPI: x = 1, RPI: x = 9
Area of the circle =

( )
9
4∫  64 − (x − 1) 2 + 3 − 3 dx
1  
9
= 4∫ 64 − (x − 1) 2 dx
1

Topic 5. Area of an Ellipse


General equation of an ellipse is:
( x − h )2 + ( y − k )2 = 1 (x − h) 2 (y − k) 2
+ =1 ⇒ Centre (h, k)
a 2
b 2 a2 b2
Draw two lines, x = h parallel to y-axis and y = k parallel
to x-axis. These two lines intersect at a point (h, k), which is the
centre of the ellipse. On the line y = k, mark two points A(x = h
+ a) and C(x = h – a). On the line x = h, mark two points B(y =
k + b) and D(y = k – b). Now join these points A, B, C and D as
shown in the graph. This is the rough figure of the ellipse.
Note: If you know the values of a and b in the equation of the
(x − h) 2 (y − k) 2
ellipse + = 1, then the area of the ellipse is (πab).
a2 b2 2
x2 y
Example 1. Draw the rough sketch of the ellipse: + =1
Here (h, k) is the centre of the ellipse. Area of the ellipse 25 16
and using integration find the area of the ellipse.
purely depends on the values of a and b. Area of the ellipse is πab.
Solution. Equation of the ellipse is:
How to draw graph of an ellipse? x2 y
2
Let the equation of the ellipse be + = 1
25 16

Area Between Curves 259


E:\AMIT_WORKS\Exam_Guru\EG_Mathematics-12_(working_02-06-2022)\EG_Mathematics-12_working\Open_Files\Chap_8\Chap_8
\ 17-Aug-2022 Amit Proof-4 Reader’s Sign _______________________ Date __________

x2 y2
⇒ 2 + =1
( 5 ) ( 4 )2
⇒ a = 5 and b = 4 and
centre is (0, 0).
Now let us draw the rough
sketch of the ellipse.
The two axis divide the
area of the ellipse into 4 equal
parts, we shall work out only
one fourth area of the ellipse
lying in the first quadrant and will multiply it by 4. It will give
the full area of the ellipse.
In the shaded region: In the shaded region: UC:

x2 y
2 (x − 2) 2 (y − 1) 2
UC: + = 1 + = 1
25 16 25 16

y2 x 2 25 − x 2 ( y − 1)2 ( x − 2 )2
⇒ = 1− = ⇒ = 1−
16 25 25 16 25
⇒ y2 =
16
25
(
25 − x 2 ) ( y − 1)2 25 − ( x − 2)
2
⇒ =
16 25
⇒ y= 4 25 − x 2
5 16
⇒ (y – 1)2 = [25 − (x − 2) 2]
LC: y = 0; LPI : x = 0, RPI : x = 5. 25

( )
5 4
 4  ⇒ y= 25 − (x − 2) 2 + 1
Area of the ellipse = 4∫  25 − x 2 − 0 dx 5
 5 
0
5
LC: y = 1, LPI: x = 2, RPI: x = 7
16

2

( )
A= 25 − x dx 7
5  4 
0 Area of the ellipse = 4∫  25 − (x − 2) 2 + 1 − 1 dx
5
{ () }
5  
16  25 x x  2
= sin −1 + 25 − x 2 
5  2 5 2 0 7
16
5 ∫
25 − (x − 2) 2 dx
{ }
⇒ A=
=
16  25
5  2
(sin −1 1) +
5
2 () 25 − 25 2

= 16  25 sin −1  x − 2 
{ () }


25
sin −1 0 +
0 
25 − 0  5  2  5 
2 2 
 7
x−2
25 − (x − 2) 2 
( )
 +
16  25 π  2 2
⇒ A= × + 0 − 0 = 20π sq. units
5  2 2 
Verification: Here a = 5, b = 4 ⇒ A= {
16  25
5  2
sin − 1 1 +
5
2
25 − 25 }
\ Area of the ellipse = πab = 20π sq. units.

Example 2. Draw rough graph of the ellipse


( x − 2)2 + ( y − 1)2 = 1
 − { 25
2
× sin −1 0 +
0
2
25 − 0 
 }
25 16
and find the area using method of integration. ⇒ A=
16  25 π
5  2 2{
× + 0 − {0 + 0}
 }
Solution. Equation of the ellipse:
16 25 π
⇒ A= × × = 20 π sq.units
( x − 2)2 + ( y − 1)2 = 1 5 2 2
25 16
Area of the ellipse = 20π. Verification: a = 5, b = 4
Centre of the ellipse (2, 1), a = 5, b = 4.
Area of the ellipse = πab = 20π sq. units
Draw the graph of the ellipse:

260 Mathematics–12
EXERCISE 8.5
I. Multiple Choice Questions- (MCQ) II. Long Answer Type Questions
x2
y 2
1. Using integration, find the area of the ellipse:
1. The area of the ellipse + = 1 is
a 2 b2 (x − 1) 2 y 2
+ =1
π a2 25 9
(a) sq. units 2 2
4 2. Find the area of the ellipse: x + (y − 1) = 1. Use method
π b2 16 4
(b) sq. units of integration.
4 (x − 3) 2 (y − 2) 2
(c) 4pa sq. units 2 3. Find the area of the ellipse: + = 1. Use
16 9
(d) 4pb2 sq. units method of integration.

Answers 8.5
πa 2 In the shaded area: UC:
I. 1. (a) sq. units
4
y = 1 16 − x 2 + 1 ,

II. 1. 15πsq.units 2

(x − 1) 2 y 2 LC : y = 1; LPI : x = 0; RPI : x = 4
Hint: Equation of the ellipse:
+ = 1,
25 9
Centre (1, 0), a = 5, b = 3. Area of the ellipse
Draw the graph.
{ }
4 4
A = 4∫  1 16 − x 2 + 1 − 1 dx = 2∫ 16 − x 2 dx
 2 
0 0

3. 12πsq.units

2 2
Hint: Equation of the ellipse: (x − 3) + (y − 2) = 1

16 9
In the shaded region: Here centre (3, 2), a = 4, b = 3
3
UC: y = 25 − (x − 1) 2 , Draw the graph of the ellipse:
5
LC: y = 0,
LPI: x = 1; RPI: x = 6
6
A = 4∫  3 25 − (x − 1) 2 − 0 dx
 5 
1
6
12
⇒ A = ∫ 25 − (x − 1) dx
2
5
1

2. 8πsq.units
2
x 2 + (y − 1) = 1
Hint: Equation of the ellipse:
.
16 4
Here Centre (0, 1), a = 4, b = 2.
Draw the graph of the ellipse: In the shaded region:
3 16 − (x − 3) 2 + 2,
UC: y = LC: y = 2,
4
LPI : x = 3; RPI: x = 7
7
3
Area of the ellipse = 4∫  16 − (x −3) + 2 − 2 dx
2
 4 
3

7
= 3∫ 16 − (x − 3) 2 dx
3

Area Between Curves 261


E:\AMIT_WORKS\Exam_Guru\EG_Mathematics-12_(working_02-06-2022)\EG_Mathematics-12_working\Open_Files\Chap_8\Chap_8
\ 17-Aug-2022 Amit Proof-4 Reader’s Sign _______________________ Date __________

Topic 6. Area Between a Circle and a Line


Example 1. Find the area of the region in the first quadrant Example 2. Draw a rough graph of the function: {(x, y) : x2 + y2
enclosed by x-axis, the line y = x and the circle x2 + y2 = 8. ≤ 1 ≤ x + y, x, y ∈ R} and using the method of integration, find
[A.I. 2014 (C), Delhi 2015 (C), Delhi 2017] the area between them. [Delhi 2010, 2011 (C)]
Solution. The given equations are: x2 + y2 = 8 and y = x. Solution. The given equations are: x2 + y2 = 1 and x + y = 1
First, find their point of intersection First solve these two equations to find their points of
x2 + y2 = 8, y = x ⇒ x2 + x2 = 8 intersection.
2
⇒ 2x = 8
x2 + y2 = 1, x + y = 1 ⇒ x2 + (1 – x)2 = 1
⇒ x2 = 4
⇒ x2 + 1 – 2x + x2 = 1 ⇒ 2x2 – 2x = 0
⇒ x = 2 and – 2
Now draw the graph of the two curves: ⇒ 2x (x – 1) = 0 ⇒ x = 0, 1
On putting x = 0 and x = 1 in x + y = 1 respectively, we get
y = 1 and y = 0

Circle x2 + y2 = 8 with centre (0, 0)


r = 2 2, line: y = x
⇒ The points of intersection are (0, 1) and (1, 0)
The shaded region has two parts:
Total area = area of the region (OAB) + area of the region Now draw the graph of the circle with centre (0, 0), r = 1
(ABC) Now join the points (1, 0) and (0, 1).
In the region (OAB): UC: y = x; LC: y = 0, It gives the graph of the line.
LPI: x = 0, RPI: x = 2 Now we put (0, 0) in the equation 1 ≤ x + y.
In the region (ABC) = UC: y = 8− x , 2 It gives: 1 ≤ 0 (false), hence the required area does not include
the area containing (0, 0)
LC: y = 0, LPI: x = 2, RPI: x = 2 2
So the required area is (ABCDA). In the required area
( 8 − x − 0) dx
2 2 2
Area of the shaded region = ∫ ( x − 0) dx + ∫
2
(ABCDA)
0 2
2 2 2 UC: Circle: y = 1 − x2
∫ x dx + ∫
2
⇒ R.A. = 8 − x dx
0 2 LC: Line: y = 1 – x, LPI: x = 0 and RPI: x = 1

∫( )
2 2 1
1 2 8  x  x 
⇒ R.A. =  x 2  +  sin −1  + 8 − x2  Required area = 1 − x 2 − (1 − x) dx
2 0 2  2 2  2 2 0

 1
1 
⇒ R.A. =
2
[ 4 − 0] +  4 sin −1(1) + 2 22 8 − 8


1
=  sin −1 (x) +
x x2 
1 − x2 − x + 
 2 2 2 0




 1 
−  4 sin −1 
 2

+ 4 
 ⇒
 1 π
(
R.A. =  × + 0 − 1 +
 2 2
1
2 ) 
− (0 + 0 − 0 + 0) 


 2 (
R.A. = 2 +  4 × π + 0 − 4 × π + 2 
4  )( ) ⇒
 π 1
R.A. =  −
 4 2 (  π 1
)
− 0 = − sq. units
 4 2

( )
⇒ R.A. = 2 + [2p – p – 2] = 2 + p – 2 = p sq. units π 1
⇒ Area of the shaded region = p sq. units ⇒ Required area = − sq.units
4 2

262 Mathematics–12
EXERCISE 8.6
I. Multiple Choice Questions- (MCQ) functions and using integration find the area enclosed
2 2 between them.
1. Smaller area bounded by the circle x + y = 4 and the line x
+ y = 2 is  x 2 + y 2 ≤ 16  x 2 + y 2 ≤ 1
3.  4. 
(a) 2 (p – 2) sq. units (b) (p – 2) sq. units  x + y ≤ 4  x − y ≥ 1
(c) (2p – 1) sq. units (d) 2 (p + 2) sq. units
 x 2 + y 2 ≤ 9
5.  [A.I. 2011, 2012 (C)]
II. Long Answer Type Questions  y − x ≥ 3
1. Find the area of the region in the first quadrant enclosed
by the x-axis, the line y = x and the circle x2 + y2 = 32.  x 2 + y 2 ≤ 25
6. 
[Delhi 2014]  x + y ≤ −5
2. Using integration, find the area of the region: 7. Find the area bounded by the circle x2 + y2 = 16, the line
{(x, y) : | x − 1 | ≤ y ≤ }
5 − x 2 [CBSE S.P. Delhi 2010] 3y = x and x-axis in the first quadrant, using integration.
[Delhi 2017]
Draw a rough graph of each of the following sets of

Answers 8.6
I. 1. (b) (p – 2) sq. units
Hint: Equation of circle is x2 + y2 = 22. Its radius = 2 units
Y

B(0,2)
M
x+
y=

A(2,0)
2

X O X

Draw the graph of line y = x and the circle with centre (0, 0) and
Y
1 r = 4 2.
Area of quadrant OAMB = (π × 22 ) Shaded region = Region (AOB) + Region (ABC)
4
1 In the region (AOB)
= π × 4 = π sq. units
4 UC: y = x; LC: y = 0; LPI: x = 0, RPI: x = 4
1 In the region (ABC)
Area of quadrant DAMB = × OA × OB
2 32 − x 2; LC: y = 0; LPI: x = 4
UC: y =
1
= × 2 × 2 = 2sq. units RPI: x = 4 2.
2
So, required area = p – 2. 4 4 2

Hence (b) is the correct answer. Area of the shaded region = ∫ x dx + ∫ 32 − x 2 dx


2 2
= 4p sq. units 0 4

Or Required area = ∫
0
4 – x 2 dx − ∫ (2 – x)dx
0
  
2. − 5 + 5 sin −1 2 + sin −1  1  
2 2 5  5
2 2
 x 4 – x 2 4 –1 x   x2 
=  + sin  – 2 x –  5π 5    
2 2 2   2 0 or − because sin −1  1  = cos −1  2 
 0 4 2  5  5
4  4  π Hint: The given equations are:
= (0 – 0) + (sin –1 1 – sin –1 0) =  4 –  – (0 – 0)  = 2   – 2
2  2  2 y = |x – 1| and y =
5 − x2
= (p – 2) sq. units
 x − 1 when x ≥ 1
⇒ y=
II. 1. 4π  −(x − 1) when x < 1
Hint: Given equations: x2 + y2 = 32 and y = x, solve them. and y = 5 − x 2 is the upper part of the circle.
⇒x=4

Area Between Curves 263


E:\AMIT_WORKS\Exam_Guru\EG_Mathematics-12_(working_02-06-2022)\EG_Mathematics-12_working\Open_Files\Chap_8\Chap_8
\ 17-Aug-2022 Amit Proof-4 Reader’s Sign _______________________ Date __________

Solve: y =
5 − x 2 and y = x – 1 and y = – (x – 1) 4. ( π4 − 12 )sq.units
( x − 1) = 5 − x 2 ⇒ x 2 − 2x + 1 = 5 − x 2
Hint: Solve the two given equations:

⇒ 2x2 – 2x – 4 = 0 ⇒ x2 – x – 2 = 0
x2 + y2 = 1 and x – y = 1.

⇒ (x – 2) (x + 1) = 0 ⇒ x = 2, –1
Draw the graph. The point (0, 2) satisfies the two inequations. It gives: x = 1, 0 ⇒ Two points (1, 0) and (0, – 1)
∴ Shaded area is the required area. Now draw the graphs.

In the shaded region UC: y = 5 − x2 , In the shaded region: UC: y = (x – 1);


There are two LC: y = – (x – 1) LC: y = − 1 − x 2 (– sign indicates that the curve is below x-axis);
and y = x – 1, LPI: x = – 1 and MPI: x = 1, LPI: x = 0, RPI: x = 1
RPI: x = 2,
{ }
1

Area of the shaded region Area of the shaded region = ∫ (x − 1) − − 1 − x 2  dx


0
 

( )
2 1 2 1
= ∫ 5 − x 2 dx − ∫ −(x − 1)dx − ∫ (x − 1)dx = ∫ x − 1 + 1 − x 2 dx
−1 −1 1 0

2 1
= ∫ 5 − x 2 dx + ∫ (x − 1) dx − ∫ (x − 1) dx
−1
2
5.
4 2 (
9π − 9 sq.units
)
−1 1
Hint: Solve the two equations:

3. (4π − 8) sq.units x2 + y2 = 9 and y – x = 3 ⇒ x = – 3, 0

Points of intersection are (– 3, 0) and (0, 3).
Hint: Solve the two equations:

Now draw the graphs of two equations.
x2 + y2 = 16 and x + y = 4 for x. It gives: x = 0, 4.

∴ Points of intersection are: (0, 4) and (4, 0)
Now draw the graph of the two curves.

In the shaded area: UC: y = 9 − x2 ;


LC: y = x + 3; LPI: x = – 3, RPI: x= 0
0
In the shaded region: Area of the shaded region = ∫  9 − x 2 − (x + 3)  dx

−3
UC: y = 16 − x 2 ; LC: y = (4 – x)
9π − 9 sq. units
LPI: x = 0, RPI: x = 4; =
4 2


2
4
Area of the shaded region = ∫  16 − x − (4 − x)  dx
0

6. ( 4 2 )
25π − 25 sq.units

264 Mathematics–12
Hint: Find the point of intersection of x2 + y2 = 25 and x + y
Or, Area of the region:
= – 5. These are (0, – 5) and (– 5, 0). Now draw the graph. 2
{(x, y) : x2 + y2 ≤ r2 and x + y ≥ r} = r ( π − 2) sq. units.
4

7. 4π sq.units
3
Hint: x2 + y2 = 16,
3y = x
On solving these two equations we get the points of intersection.
( )
A 2 3, 2 and B −2 3, 2 ( )

In the shaded region: UC: y = – (x + 5);


LC: y = − 25 − x 2 , LPI: x = – 5, RPI: x = 0

∫  − (x + 5) − ( − )
0
Area of the shaded region =  25 − x 2  dx
−5

0 0
= − ∫ (x + 5) dx + ∫ 25 − x 2 dx
2 3 4
−5 −5
 x 
∫   dx + ∫ 16 − x dx
2
Area =
= 25π − 25 sq. units 0 3 2 3
4 2

()
2 3
Note: Smaller area between a curve x2 + y2 = r2 and a line  2 
4
=  x  x 16 x 
+  16 − x 2 + sin −1 
x + y = r is:  2 3 0 2 2 4 2 3
2
r
(π − 2) sq.units . = 4π sq. units .
4 3

Topic 7. Area between Ellipse and Line


Example 1. Draw the rough figure and find the area represented ⇒ x = 0, 5
 2
x2 y x y \ Points of intersection are (0, 4) and (5, 0).
by: (x, y) : + ≤ 1 ≤ +  . Use method of integration.
 25 16 5 4 2
y2
Now draw the graph of the ellipse x + = 1 , centre of the
Solution. Find the points of intersection of the two given equation: 25 16
2 ellipse (0, 0), a = 5, b = 4, join the points A(0, 4) and C(5, 0).
x2 y x y
+ = 1 and + = 1
25 16 5 4 x y
It gives the line + = 1.
4 x2 y
2 5 4
Put y = (5 − x) in + = 1,
Which area is to be shaded?
5 25 16
x 2 1 16 x y
It gives: + × (5 − x) 2 = 1 Put (0, 0) in + ≥ 1. It gives 0 > 1 (false). So the area
25 16 25 5 4
⇒ x2 + (5 – x)2 = 25 including (0, 0) is not in the shade. So shade the other area as
⇒ 2x(x – 5) = 0 shown in the graph. In the shaded region:
4
UC: y = 25 − x 2 , LC: y = 4 (5 − x), LPI: x = 0,
5 5
RPI : x = 5.
Area of the shaded region
5
 4 25 − x 2 − 4 (5 − x)  dx
A= ∫  5 5 
0

()
5
4  25 −1 x x 1 
⇒ A =  sin + 25 − x 2 + (x − 5) 2 
52 5 2 2 0

Area Between Curves 265


E:\AMIT_WORKS\Exam_Guru\EG_Mathematics-12_(working_02-06-2022)\EG_Mathematics-12_working\Open_Files\Chap_8\Chap_8
\ 17-Aug-2022 Amit Proof-4 Reader’s Sign _______________________ Date __________

⇒ A= (
4  25 π
× +0+0 − 0+0+
5  2 2
25 
2  )(
4  25π 25 
=A= 5  4 − 2 
  )
⇒ A = (5p – 10) = 5(p – 2)
⇒ Area of the shaded region = 5(π – 2) sq. units

EXERCISE 8.7
I. Multiple Choice Questions- (MCQ) 2. Draw the rough graph of the following ellipse and line and
1. The area of the smaller region bounded by the ellipse using integration, find the smaller area enclosed between
x2 y 2 x y
+ = 1 and st. line + = 1 is x2 y
2
9 4 3 2 them: + = 1 and 5x − 4y = 20.
3 16 25
(a) 3 (p – 2) sq. units (b) (p – 2) sq. units
4 2 Draw rough figure and find the area represented by each of
(c) 3p sq. units (d) 2p sq. units the following sets of equations. Use method of integration.
II. Long Answer Type Questions  x2 y
2
x y
3. (x, y) : + ≤1≤ − + 
1. Find the area of the smaller region bounded by the ellipse:  16 49 4 7
2
x2 y x y Use method
2 + 2 = 1 and the straight line : a + b = 1.  x2 y
2
x y
a b 4. (x, y) : + ≤1≤ − − 
of integration. [A.I. 2009 (C)]  49 4 7 2

Answers 8.7
3 2 9 π 9  2 9 π  3
I. 1. (b) (p – 2) sq. units =  ⋅ −  = ⋅  − 1 = (p – 2) sq. units
2 3 2 2 2 3 2 2  2
x2 y 2 ab (π − 2) sq.units
Hint: Equation of ellipse is + =1 II. 1.
9 4 4
y2 x2 4 Hint: find the points of intersection of two equations:


⇒ = 1 − ⇒ y 2 = (9 − x 2 )
4 9 9 2
x 2 + y = 1 and x + y = 1
2 a 2
b 2 a b

⇒ y= 9 − x2
3 Points are: (0, b) and (a, 0). Now draw the graph:
x y y x 2

Equation of line AB is + = 1 ⇒ = 1− ⇒ y = (3 − x)
3 2 2 3 3
Y

B(0, 2)

A(3, 0)
X X
O

Shade the smaller common region:


b a2 − x2 b
UC: y = , LC: y = − (x − a);
a a
LPI: x = 0; RPI: x = a.
Y
3 3
Area of the shaded region:
2 2
Required area (shaded region) = ∫ 9 − x dx − ∫ (3 − x) dx
{ }
2 a

30 30 A = ∫  b a 2 − x 2 − − b (x − a)  dx
a a 
3 3
0
2x 9 x 2  x2  a
=  9 − x 2 + sin −1  − 3x − 
3 2 2 3 0 3  2 0 ⇒ A = b ∫  a 2 − x 2 + (x − a)] dx
a  
0

2  9 −1  2 9 
= 0 + sin 1 − 0 − 0  − 9 − − 0 + 0  = ab (π − 2) sq. units.
3  2  3 2  4

266 Mathematics–12
Note: This is the standard formula for such questions. Please
 Points of intersection A( – 4, 0), B (0, 7).
commit to memory. 7 16 − x 2 ,
In the shaded region: UC: y =
2. 5(π − 2) sq.units 4

x2 y
2 LC: y = 7 (x + 4), LPI: x = – 4, RPI: x = 0.
Hint: Equations are + = 1. 4
16 25
0
5x – 4y = 20, points of intersection: A = 7 ∫  16 − x 2 − (x + 4)  dx
A(4, 0), B(0, – 5). 4  
−4

= 7(p – 2) sq. units

7 (π − 2) sq.units
4.
2
Hint : Equations are:

2
x 2 + y = 1 and −x − y = 1
49 4 7 2
Points of intersection: A(– 7, 0), B (0, – 2).

In the shaded region: LC: y =


−5 16 − x 2 , In the shaded region: UC: y = −2 (x + 7),
4 7
5 (x − 4), 2 49 − x 2 ,
UC: y = LPI: x = 0, RPI: x = 4. LC: y = − LPI: x = – 7, RPI: x = 0.
4 7
4 0
A = 5 ∫ (x − 4) + 16 − x 2  dx A = ∫ 
−2
(x + 7) + 2 49 − x 2  dx
4  
 7 7 
0
−7
= 5(p – 2) sq. units 0
= − ∫ (x + 7) − 49 − x 2  dx
2
3. 7 (π − 2) sq.units 7  
−7
= 7 ( π − 2) sq.units
2

2
y2 x y
Hint: Equations are: x +
= 1 and − + = 1
16 49 4 7

MOST IMPORTANT TOPICS

Topic 8. Area between two Circles


Example 1. Draw a rough graph of the circle: x2 + y2 = r2 and the circle: (x – r)2 + y2 = r2. Shade the common region and find the
area of the shaded region using integration.
Solution. Equations of the circles are:
x2 + y2 = r2 and (x – r)2 + y2 = r2.
Solve these two equations:
x2 + y2 = (x – r)2 + y2
⇒ x2 = x2 – 2rx + r2
⇒ 2rx = r2
⇒ x= r
2

Area Between Curves 267


E:\AMIT_WORKS\Exam_Guru\EG_Mathematics-12_(working_02-06-2022)\EG_Mathematics-12_working\Open_Files\Chap_8\Chap_8
\ 17-Aug-2022 Amit Proof-4 Reader’s Sign _______________________ Date __________

Now draw the graph of the two circles:


LPI: x = r , RPI: x = r.
2
R. Area = area of (AOBD) = 4 area of (APD)
r r
 2 
= 4 ∫  r − x − 0 dx = 4 ∫
2
r 2− x 2 dx
 
r/2 r/2

()
r
 r2 −1 x x 2 
⇒ R.A. = 4  sin + r − x2 
 2 r 2 r
2
 r 2 π   r 2 π r 2 r 2  
⇒ R.A. = 4  × + 0 −  × + r − 
Circle x2 + y2 = r2 has centre (0, 0), radius = r  2 2   2 6 4 4 

and circle: (x – r)2 + y2 = r2 has centre (r, 0) and radius = r.  πr 2  πr 2 r 3 r
⇒ R.A. = 4  − + ×  
Shade the common region. Join the two points of intersection  4  12 4 2 
of two circles. This line AB and x-axis divides the full required
 2πr 2 3r 2 
area into 4 equal parts. We shall evaluate only one part and ⇒ R.A. = 4  −
 12 8 
then we shall multiply it by 4. We choose an area (APD) to
work out the required area because this area is below simple  2πr 2 3r 2 
⇒ R.A. =  − sq. units.
circle x2 + y2 = r2.  3 2 

Here UC: y= r 2 − x 2 and LC: y = 0, Note: This is a standard formula for this type of questions.

EXERCISE 8.8
I. Multiple Choice Questions- (MCQ) II. Long Answer Type Questions
1. The area of the region enclosed between the two circles: 1. Find the area bounded by the curves: (x – 1)2 + y2 = 1 and
x2 + y2 = 4 and (x – 2)2 + y2 = 4. is x2 + y2 = 1. [Delhi 2007, A.I. 2013 (C)]
[Delhi 2008, A.I. 2010 (C)] 2. Find the area of the region enclosed between the two circles
x2 + y2 = 9 and (x – 3)2 + y2 = 9. [Delhi 2009, A.I. 2009]
(a) 8π − 2 3 sq. units
3 3. Find the area of the region enclosed between the two
circles: (x – 6)2 + y2 = 36 and x2 + y2 = 36.
(b) 8π + 2 3 sq. units
3 [Delhi 2009 (C), A.I. 2013 (C)]
(c) (3p – 2) sq. units 4. Find the area of the region enclosed between the two
(d) (3p + 2) sq. units circles: x2 + y2 = 1 and x2 + (y – 1)2 = 1.  [Delhi 2009]

Answers 8.8
I. 1. (a) 8π − 2 3 sq.units In the shaded region: UC: y = 4 − x 2;
3 LC: y = 0, LPI: x = 1, RPI: x = 2
Hint: Solve the two equations: R.A. = Area of (AODCA) = 4 area of (ABC)
x2 + y2 = 4 and (x – 2)2 + y2 = 4 ⇒ x = 1. 2
Required area = 4∫ 4 − x dx
2
Draw graph: x2 + y2 = 4 has centre (0, 0) and r = 2
(x – 2)2 + y2 = 4 has centre (2, 0) and r = 2. 1

II. 1. 2π − 3 sq.units
3 2
Hint: Take the equations:

x2 + y2 = 1 and (x – 1)2 + y2 = 1

Find their point of intersection, it is: x = 1 .
2

268 Mathematics–12
Draw the graph with the given information: RA = Area of the region (AODCA)
[x2 + y2 = 1, centre (0, 0), r = 1 = 4 area of (ABC)
and (x – 1)2 + y2 = 1, centre (1, 0), r = 1]
RA = Area of (AODCA) = 4 area of (ABC) In the shaded region: UC: y = 36 − x 2 ,
LC: y = 0, LPI: x = 3, RPI: x = 6
In the shaded region : UC: y = 1 − x 2 ,
6
1 Required area = 4∫ 36 − x 2 dx
LC: y = 0, LPI: y = , RPI: y = 1
2
3
1
R.A. = 4 ∫ 1 − x 2 dx  
4.  2π − 3  sq.units
1/ 2  3 2 

2. 6π − 9 3 sq.units Hint: The two given equation:


2
x2 + y2 = 1 and x2 + (y – 1)2 = 1

Hint:
Given equations: Point of intersection:
x2 + y2 = 9 and (x – 3)2 + y2 = 9 y=

1 , [(x 2 + y 2 = 1),
centre (0, 0), r = 1]
point of intersection: 2
3 and [x2 + (y – 1)2 = 1, centre (0, 1), r = 1].
x = , [x 2 + y 2 = 9, centre (0, 0), r = 3],
2
[(x – 3)2 + y2 = 9, centre (3, 0), r = 3]. Now draw their graphs
Draw their graphs.

RA = Area of the region (AODCA)


= 4 area of (ABC).
R.A. = Area of the region (ACDOA)
In the shaded region: UC: y = 9 − x 2; = 4 area of the region (ABC). Integration will be evaluated
3 along y-axis.
LC: y = 0, LPI: x = , RPI: x = 3
2
3 In the shaded region: UC: x = 1 − y 2,
Required area = 4 ∫ 2
9 − x dx .
1
3/ 2 LC: x = 0, LPI: y = , UPI: y = 1
2
3. 24π − 18 3 sq.units 1

2 2 2 2 \ RA = 4 ∫ 1 − y 2 dy
Hint: The two equations are: x + y = 36 and (x – 6) + y =
1/ 2
36, point of intersection: x = 3, [x2 + y2 = 36, centre (0, 0), r =
6] and [(x – 6)2 + y2 = 36, centre (6, 0), r = 6]. Note: You can make this question simpler by changing x to y

Draw their graphs. and y to x.

Area Between Curves 269


E:\AMIT_WORKS\Exam_Guru\EG_Mathematics-12_(working_02-06-2022)\EG_Mathematics-12_working\Open_Files\Chap_8\Chap_8
\ 17-Aug-2022 Amit Proof-4 Reader’s Sign _______________________ Date __________

Topic 9. Area between Circle and Parabola


Example 1. Make a rough sketch and find the area of the region Example 2. Draw a rough graph of: {4x2 + 4y2 ≤ 9 and y2 ≤ 4x}
(using integration): and using method of integration, find the area enclosed between
{(x, y) : x2 + y2 ≤ 2ax; y2 ≥ ax, x ≥ 0, y ≥ 0} [Delhi 2016] them.[Delhi 2008, A.I. 2003 (C), 2013]
Solution. The two equations are: Solution. The given equations are: 4x2 + 4y2 = 9 and y2 = 4x
x2 + y2 = 2ax ⇒ (x – a)2 + y2 = a2 (a circle) Solve these equations to find their points of intersection.
4x2 + 4(4x) = 9 ⇒ 4x2 + 16x – 9 = 0
y2 = ax (a parabola). 2
Solve these equations, to find their points of intersection. ⇒ 4x + 18x – 2x – 9 = 0 ⇒ 2x (2x + 9) – 1 (2x + 9) = 0
(x – a)2 + y2 = a2 ⇒ (x – a)2 + ax = a2 ⇒ (2x – 1) (2x + 9) = 0
2 2
⇒ x – 2ax + a + ax = a 2
⇒ x(x – a) = 0 1 −9
⇒ x = and x = (rejected)
⇒ x = 0, a. 2 2
Now (x – a)2 + y2 = a2 is a circle with centre (a, 0) and r = 1
⇒ x=
a and y2 = ax is a parabola which opens on a right hand side 2
and passes through (0, 0). Draw the graph. Now 4x2 + 4y2 = 9

()
2
3
⇒ x2 + y2 =
is a circle with centre (0, 0) and
2
3
r= .
2
y2 = 4x is a parabola which will open on
RHS and will pass through (0, 0).
So draw their graphs.

For shading the region put (a, 0) in the parabola equation.


y2 = ax. It gives: 0 ≥ a2 (false)
∴ Point (a, 0) is not in the required area.
So shade the other part as shown in the graph.
In the shaded region: R.A. = Area (ODBEO).
In the region (ODBEO)
UC: y= a 2 − (x − a) 2 , We want to find the area (AODCA) = 2(OACBO)
R.A. = Area of the region (AOBA) + Area of the region
LC: y= a x, (ABCA)
LPI: x = 0, RPI: x = a In the region (AOBA):
a UC: y = 2 x and LC: y = 0,
R.A. = ∫  a 2 − (x − a) 2 − a x  dx
 LPI: x = 0, RPI: x = .
1
0
2
 2 x − a x − a 2
R.A. =  a sin − 1  + a − (x − a) 2 In the region (ABCA): UC: y =
9
− x 2,
 2  a  2 4
a 1 3
2 a 32  LC: y = 0; LPI: x = ; RPI: x =
− x  2 2
3 0
 1/ 2 3/ 2 
9
 2  Required area = 2  ∫ 2 x dx + ∫ − x 2 dx 
R.A. =   a sin −1 0 + 0 − 2 a a a   0 1/ 2
4

 2 3  1 3

( )
3
a  − π
2
 4 × 2  22 9 2x x 9 2
−   + 0 − 0  ⇒ R.A. =  x  + 2  sin −1 + − x2 
3   8 3 2 4
 2  2   0 1
2



R.A. =  0 + 0 −

2a 2   πa 2  
− −
3   4  

3  2 2  8 2 (
R.A. = 8  1  + 2  9 × π + 0 )

 2
 4
2

3  12
2
R.A. =  πa − 2a  = a (3π − 8) sq.units

9
8 ()
−  sin −1
1
3
+
1
4
9 1

4 4  

270 Mathematics–12
⇒ R.A. =
8
×
3 4
2  9π  9
+ 2  −  sin −1
 16  8
1 1 
+ 2
3 4   ()
 2 2 9π 9 −1 1 2
()
=  3 + 8 − 4 sin 3 − 2 

=
2 2
3
 9π 9
+ 2  − sin −1
 16 8
1
3

4
2


()
2 2
= 
 3

2
2  9π 9 −1 1
 + 8 − 4 sin 3 ()

 2 9π 9
= 
 6
+
8 4
1
3 ()
− sin −1  sq.units

EXERCISE 8.9
I. Multiple Choice Questions- (MCQ) parabola y2 = 8x. Also find the area of the region between
1. The area of the region {(x, y): x2 + y2 ≤ 1 and y2 < 1 – x} is them. Using integration.
 π 4 ( 4 π − 5) 5. Find the area of the region bounded by: {(x, y) : y2 ≤ 4x
(a)  +  sq. units (b) sq. units
 2 3 6 and 4x2 + 4y2 ≤ 9} [A.I. 2008 (C)]

(c)
( 2π − 3) sq. units (d)
( π − 8) sq. units Note: Same question as Q. No. 1. Change x to y and
5 12 y to x.
II. Long Answer Type Questions 6. Using integration find the area of region:
1. Find the area of the circle 4x2 + 4y2 = 9 which is interior {(x, y) : x2 + y2 ≤ 16 and x2 ≤ 6y}[Delhi 2010 (C)]
to the parabola x2 = 4y.[A.I. 2010] Note: Same question as Q. No. 2. Change x to y and

2. Find the area of the circle x2 + y2 = 16 exterior to the y to x.
parabola: y2 = 6x by using integration.
7. Find the area of the region: {(x, y) : y2 ≤ 6ax and x2 + y2 ≤
[Delhi 2010, AI 2007, 2012]
16a2}. Using a method of integration. [A.I. 2013]
3. Find the area lying above x-axis and included between the
circle: x2 + y2 = 8x and the parabola: y2 = 4x. 8. Using integration, find the area of the triangle formed by
[Delhi 2008, A.I. 2008] positive x-axis and the tangent and normal to the circle
4. Sketch the region common to the circle x2 + y2 = 20 and ( )
x2 + y2 = 4 at a point 1, 3 .[Delhi 2015]

Answers 8.9
I. 1. (a)

II. 1. 2 + 9π − 9 sin −1 1 sq.units


6 8 4 3
Hint: The given equations are:
4x2 + 4y2 = 9 and x2 = 4y
Refer to Example 2,
1/ 2 3/ 2
9 − y 2 dy 
R.A. = 2  ∫ 2 ydy + ∫ 
 0 4 
1/ 2
On solving, we get
  Hint: Point of intersection of the two equations: x2 + y2 = 16
R.A. =  2 + 9 sin −1 2 2  sq. units. and y2 = 6x is x = 2.
 6 4 3 
The answer can be converted to same as in example. We shall find the area AOB. Total required area will be:
8π + 2 area (AOB)
2 + 9 sin −1 2 2
6 4 3 In the shaded area AOB.

= 2 + 9 cos −1 1 − 8 =

6 4 9
2 + 9 cos −1 1
6 4 ()
3
UC: y = 16 − x 2 ,

LC: y = 6x,

= 2 +
6 4 2
9 π − sin −1 1
3 ( ) LPI: x = 0 and RPI: x = 2.
2
= 2 + 9π − 9 sin −1 1
Req. area = 8π + 2∫  16 − x 2 − 6 x  dx
6 8 4 3  
0

2.
3 ( )
4 8π − 3 sq.units

 
( )
= 8π +  8π − 4 3  = 4 8π − 3 sq. units
 3 3  3

Area Between Curves 271


E:\AMIT_WORKS\Exam_Guru\EG_Mathematics-12_(working_02-06-2022)\EG_Mathematics-12_working\Open_Files\Chap_8\Chap_8
\ 17-Aug-2022 Amit Proof-4 Reader’s Sign _______________________ Date __________

3. 4π + 32 sq. units. LPI: x = 2, RPI: x = 2 5


3 2 2 5
2
Hint: Curves are: (x – 4) + y = 16, y = 4x.
2 2 ⇒ R.A. = 2∫ 2 2 x dx + 2 ∫ 20 − x 2 dx
0 2
Points of intersection x = 0 and x = 4 5. This equation reduces of Q. No. 1 if x is changed to y and y is
changed to x.
R.A. = area of OBC + area of BCD
{(x, y) : x2 ≤ 4y and 4x2 + 4y2 ≤ 9}
In (OBC) : UC: y = 2 x, LC: y = 0,
6. This equation reduces to Q. No. 2 if x is changed to y and y is
LPI: x = 0, RPI: x = 4 changed to x.
{(x, y), x2 + y2 ≤ 16 and y2 ≤ 6x.
2 2
7. 16a π + 4 3a sq.units
3 3
Curves are: x2 + y2 = 16a2, y2 = 6ax
Point of intersection : x = 2a.

In (BCD); UC: y = 16 − (x − 4) 2 , LC: y = 0,

LPI : x = 4 and RPI: x = 8


4 8
RA = ∫2 x dx + ∫ 16 − (x − 4) 2 dx
0 4 Area of the shaded region:
4
 32   2a 4a 
⇒ RA = 2 × 2 
x 4×8
 + 4π = 3 + 4π = 2  ∫ 6ax dx + ∫ 16a 2 − x 2 dx 
3  0 
 0 2a 

= ( 323 + 4π)sq. units.


=
16 a 2π 4 3a 2
3
+
3
sq. units

8. 2 3 sq.units
8 10  1 
4. + π − 20 sin −1   sq.units
3  5 Hint : Circle equation: x2 + y2 = 4

Hint: Point of intersection of x2 + y2 = 20 and y2 = 8x is x = 2.



RA = 2 area (AOBCA)
= 2 (area of AOB) + 2 (area of ABC).
In (AOB): UC: y = 2 2 x, LC: y = 0,
LPI : x = 0 and RPI : x = 2

(
∴ Tangent equation touching the circle at 1, 3 , x + 3 y = 4 .)
Normal : y = 3x.
Area of ∆OAP = Area of ∆POB + Area of ∆PBA
1 4

2
= ∫ 3x dx + ∫ 1 ( 4 − x ) dx = 2 3 sq.units
In (ABC) : UC: y = 20 − x , LC: y = 0, 0 1
3

272 Mathematics–12
Case Based Questions
1. A child cuts a pizza with a knife. Pizza is circular in shape
which is represented by x2 + y2 = 4 and sharp edge of knife y
represents a straight line given by x = 3 y. 3y
x= B( 3 , 0)
x2 + y2 = 4
O
(c) x' x
A(2, 0)

y'

y
3y
x= , 1)
x = 3y ( 3
B x2 + y2 = 4
Pizza (d) x' x
O A(2, 0)
x2 + y2 = 4

Based on the above information, answer the following y'


(iii) Value of area of the region bounded by circular pizza
questions.
and edge of knife in first quadrant is
(i) The point(s) of intersection of the edge of knife (line)
π π
and pizza shown in the figure is (are) (a) sq.units (b) sq.units
2 3
(a) (1, 3 ), (–1, – 3 ) (b) ( 3 , 1), (– 3 , –1)
π
(c) ( 2 , 0), (0, 3 ) (d) (– 3 , 1), (1, – 3 ) (c) sq.units (d) π sq.units
5
(ii) Which of the following shaded portions represent the
smaller area bounded by pizza and edge of knife in first (iv) Area of each slice of pizza when child cuts the pizza
quadrant? into 4 equal pieces is
π
(a) π sq.units (b) sq.units
=4

y 2
+y2

x = 3y (c) 3π sq.units (d) 2π sq.units


x2

) (v) Area of whole pizza is


3,1
O B( (a) 3π sq.units (b) 2π sq.units
(a) x' x (c) 5π sq.units (d) 4π sq.units
A(2, 0)
Ans. (i) (b) ( 3 , 1), (– 3 , –1)
=4

y
+y2

y' x = 3y
x2

1)
y 3,
=4

B(
+y 2

x = 3y O
(ii) (a) x' x
A(2, 0)
x2

3,
1)
O B(
(b) x' x
A(2, 0)
y'
π
(iii) (b) sq.units (iv) (a) π sq.units
3
y' (v) (d) 4π sq.units

Area Between Curves 273


E:\AMIT_WORKS\Exam_Guru\EG_Mathematics-12_(working_02-06-2022)\EG_Mathematics-12_working\Open_Files\Chap_8\Chap_8
\ 19-Aug-2022 Amit Proof-4 Reader’s Sign _______________________ Date __________

2. A mirror in the shape of an ellipse represented by


x2 y 2 y
+ = 1 was hanging on the wall. Arun and his sister
9 4 x y
+ =1
were playing with ball inside the house, even their mother 3 2
refused to do so. All of a sudden, ball hit the mirror and got x2 y 2
+ =1
x y 9 4
a scratch in the shape of the line represented by + =1 . (d) x' x
3 2 O

y'

2 3
(iii) The value of 9 − x 2 dx is
3 ∫0

Based on the above information, answer the following π


(a) (b) π
questions. 2
(i) Point(s) of intersection of ellipse and scratched (straight
3π π
line) is (are) (c) (d)
2 4
(a) (0, 2), (3, 0) (b) (2, 0), (0, 3)
2 3 x
(iv) The value of is
3 ∫0  3 
(c) (2, 3), (0, 0) (d) (0, 3), (3, 0) 1 −  dx
(ii) Area of smaller region bounded by the ellipse and line
(a) 0 (b) 1
is represented by
y (c) 2 (d) 3
(v) Area of the smaller region bounded by the mirror and
(0, 2) x2 y 2
+ =1 scratch is
9 4
π π
(a) x'
(3, 0)
x (a) 3  + 1 sq.units (b)  + 1 sq.units
O 2  2 
x y
+ =1
3 2 π  π
(c)  − 1 sq.units (d) 3  − 1 sq.units
2  2 
y'
Ans. (i) (a) (0, 2), (3, 0)
y
(ii) (b)
2
(0, 2) x y 2 y
+ =1
9 4 (0, 2) x 2 y 2
+ =1
(3, 0) 9 4
(b) x' O
x
(3, 0)
x y
+ =1

x' x
O
3 2
x y
+ =1
3 2
y'
y'
y
x y
2)

+ =1
3 2 3π
(0,

x2 y 2 (iii) (c)
+ =1 2
9 4
(c) x' x (iv) (d) 3
) O
0
(–3,
π 
(v) (d) 3  − 1 sq.units
2 
y'

274 Mathematics–12
Author’s Comments
Questions based on following types are very important for Exams. So, students are advised to revise them thoroughly.
1. Area between two circles. (Most Important)
2. Area between circle and parabola. (Most Important)
3. Area between two Parabolas.

IMPORTANT FORMULAE
1. (A) Area between two curves y = f (x) and y = g(x) intersecting at x = a and x = b, when integral is evaluated along
x-axis.

b b
( )
Area = ∫  yupper curve − ( ylower curve ) dx = ∫  f ( x ) − g ( x ) dx

a a

(B) Area between two curves x = f (y) and x = g(y) intersecting at y = a and y = b, when integral is evaluated along
y-axis.

b b
( )
Area = ∫  xright side curve − ( xleft side curve ) dy = ∫  f ( y ) − g ( y ) dy

a a
2
2. Area of a circle = pr (r is the radius of the circle).
2
x2 y
3. Area of the ellipse + = 1 is πab .
a 2 b2

COMMON ERRORS
ERRORS CORRECTIONS
(i) Students are unable to recognise whether the equation is a (i) Students are advised to have a sound practice of recognition
straight line, circle, parabola, ellipse, etc. of equation of different types of curves.
(ii) Even after recognition of the type of the curves, students (ii) Practice to draw the graph of the various functions.
are unable to draw a rough graph of the given equation.
(iii) Students make mistake in shading the required area. (iii) Check and draw the given dimension carefully.
(iv) Students do not write unit of area in AOI. (iv) Must write sq. units with the answer.

Area Between Curves 275


E:\AMIT_WORKS\Exam_Guru\EG_Mathematics-12_(working_02-06-2022)\EG_Mathematics-12_working\Open_Files\Chap_8\Chap_8
\ 17-Aug-2022 Amit Proof-4 Reader’s Sign _______________________ Date __________

REVISION CHART

Area between Parabola and circle Area between Ellipse and


Area between two circles and between two parabolas
Parabola

Area Between Curves


To find the area between two curves first find out where the curves meet, which determines the end points of integration which represent limits of
the function. Then find the UC = Upper Curve, LC = Lower Curve, RPI = Right point of intersection, LPI = Left point of intersection.
RPI

Area = ∫ [y
LPI
U.C. − y L.C. ] dx

Area Under Simple Curves


The area under a curve between two points can be found by doing a definite integral between the two points:
To find the area under the curve y = f(x) between x = a and x = b, integrate y = f(x) between the limits of a and b.

Ellipse and Line Circle and Line Parabola and a Line

276 Mathematics–12
E:\AMIT_WORKS\Exam_Guru\EG_Mathematics-12_(working_02-06-2022)\EG_Mathematics-12_working\Open_Files\Chap_9\Chap_9
\ 19-Aug-2022 Amit Proof-4 Reader’s Sign _______________________ Date __________

9 Differential Equations

Topics Covered
9.1 Order and Degree of a Differential Equation 9.2 General Solution of a Differential Equation
9.3 Solution by Method of Separation of Variables 9.4 D.E. Reducible to Variable Separable
9.5 Homogeneous Differential Equation 9.6 Linear Differential Equations

C hapter map
DIFFERENTIAL EQUATIONS

Differential General Solution of a


Equations Differential Equation

Order & Linear Differential Equations


Degree Homogeneous
Variable Separable
D.E. Differential Equations
y
dy
Type: dx + Py = Q F(x, y) = x n g  
x
 

Differential Equations (D.E.)


2
An equation involving the independent variable x, dependent variable y and the differential coefficients dy , d y2 , etc. is called a
differential equation or (D.E.). For example: dx dx
2
dy (ii) x d y2 = x + 3 , etc. are differential equations.
(i) = x 2 + 1
dx dx

Topic 1. Order and Degree of a Differential Equation


Order Degree
p q Order: It is the highest derivative.
 d m y  d m −1y 
f ( x, y )  m  + φ ( x, y )  m −1  + .......... = 0 Degree: It is the exponent of the highest derivative.
 dx   dx 

Order of a D.E.
The order of a differential equation is the highest order derivative of the dependent variables w.r.t. independent variable occurring
in the differential equation.
dy
(i) In the D.E.: − x2 + 3 = 0
dx
dy
The highest order of derivative is . Since its order is 1, therefore order of the D.E. is also 1.
dx
d 2y  d 3y  dy
(ii) In the D.E.: x 2 y 2
+ x 3  − y =0
dx  dx  dx

277
E:\AMIT_WORKS\Exam_Guru\EG_Mathematics-12_(working_02-06-2022)\EG_Mathematics-12_working\Open_Files\Chap_9\Chap_9
\ 17-Aug-2022 Amit Proof-4 Reader’s Sign _______________________ Date __________

3/ 2
d 3y  dy 
2
 d 3y 
2
The highest order derivative is
dx3
and its order is 3, (iv) 1 +    = 5 3 
 dx   dx 
 
therefore the order of the D.E. is 3. 2
5 3
(iii) In the DE: (y′′′)2 + (y″)4 + (y′)3 + y4 = 0  dy  + 3x + y +  d y  = 0
Solution. (i) x 2  dx3 
 dx 
The highest derivative is y′′′ and its order is 3, therefore 2
the order of the D.E. is 3.  d 3y 
This D.E. has highest order derivative  3  . Its order is
3, therefore order of the D.E. is also 3. dx
Degree of a D.E.
2
Degree of a differential equation, when it is a polynomial equation  d 3y 
in derivatives, is the highest power (positive integral index) of Now see the power of  3  . It is 2, therefore degree of the
D.E. is 2. dx
the highest order derivative involved in the given differential
equation.
2
dy
d 3y  d 2 y   dy 5 (ii) 1 − x 2 dx − 1 − y 2 dy = 0 ⇒ = 1− x
(i) + 2  2  +  dx  + y = 0 dx 1 − y2
dx3  dx   
d 3y In the D.E. highest order derivative is dy and its order is one,
The highest derivative of the equation is and its power therefore order of the D.E. is 1. dx
dx3
is 1. Therefore degree of this D.E. is 1. Since power of the highest derivative dy of the D.E. is one,
2
 d 2y  therefore degree of the D.E. is also 1. dx
dy
(ii) In the differential equation:   +  2  − sin 2 y = 0
 dx   dx  dy dy
2
(iii) y = x + a 2   + b2
 2  dx  dx 
highest order derivative is  d y2  .
 dx   y − x dy 
2 2
 dy  + b 2
Its order is 2, and power is 1. ∴ its degree is 1. ⇒ = a2
 dx   dx 
d 3y  d 2y  2 2
(iii) In the D.E.: 3 + sin  = y order is defined as 3 but  dy  − 2xy dy  dy  + b 2
dx  dx 2  ⇒ y 2 + x2
 dx  dx
= a2
 dx 
 d 2y 
degree is not defined due to sin  2  . 2
 dx  ⇒ ( ) dy
x 2 − a 2   − 2xy
 dx 
dy
dx
+ y 2 − b2 = 0
2
d y dy dx
Example 1. The degree of the differential equation 2 + e =0 2

is ____________ .
dx Here, highest order derivative is  dy  . Its order is 1.
 dx 
d 2y \ Order of the D.E. is 1. Since power of dy is 2, therefore
Solution. The degree of the differential equation 2 + edy dx
=0 dx
dx degree of the D.E. is 2.
is not defined.
3

( )
2
dy
2   dy  2  2  d 3y 
Example 2. The degree of the differential equation 1 + =x (iv) 1 +
    = 5 3
dx  dx   dx 
is ____________ .
3

( )
4
dy
2   dy  2   d3 y
Solution. The given differential equation is 1+ =x ⇒ 1 +    = 25  3 
dx  dx   dx 
Squaring both sides, we get 4
2  d 3y 
 dy  In this D.E. highest order derivative is  3 
1 +   = x2  dx 
 dx 
So, the degree of the equation is 2. ⇒ Order of the D.E. is 3 and degree of the D.E. is 4.
Example 3. Write the order and the degree of each of the following Example 4. The degree of the differential equation
2 2
differential equations:  d2y  dy   dy 
 dx 2 
+   = x sin   is
5
 d 3y 
2 dx dx
dy 
2
(i) x + 3x + y +  3  = 0
 dx   dx  (a) 1 (b) 2 (c) 3   (d) not defined
Solution. The degree of the given differential equation is not
(ii) 1 − x 2 dx − 1 − y 2 dy = 0  dy 
defined because the value of sin   on expansion will be
dx 
dy
2 in increasing power of   .
(iii) y = x dy + a 2  dy  + b 2  dx 
dx  dx 
Hence, the correct option is (d).

278 Mathematics–12
E:\AMIT_WORKS\Exam_Guru\EG_Mathematics-12_(working_02-06-2022)\EG_Mathematics-12_working\Open_Files\Chap_9\Chap_9
\ 17-Aug-2022 Amit Proof-4 Reader’s Sign _______________________ Date __________

Example 5. The degree of the differential equation 7. Which of the following is a second order differential

( )
 2  3/2 2 equation?
dy d y is
1 +  = (a) (y′)2 + x = y2 (b) y′y′′+ y = sin x
 dx  dx 2
(c) y′′ + (y′′)2 + y = 0 (d) y′ = y2
3 Solution.Second order differential equation is y′y′′+ y = sin x
(a) 4 (b) (c) not defined (d) 2
2 Hence, the correct option is (b).
Solution.The given differential equation is
Example 8. The order and degree of the differential equation
( )
2 3/2
 dy 
d 2y
( ) =y
2
1 +  =  d 3y  d 2y dy
4
 dx 2
dx   3  − 3 2 + 2 dx
4 are
 dx  dx
Squaring both sides, we have
(a) 1, 4 (b) 3, 4 (c) 2, 4 (d) 3, 2
( )
2 3 2
 dy   d 2 y 
1 +  = Solution.The given differential equation is
 dx   dx 2 

( ) =y
2 4
So, the degree of the given differential equation is 2.  d 3y  d 2y dy 4
 3  − 3 2 + 2 dx
Hence, the correct option is (d).  dx  dx
Example 6. The order and degree of the differential equation 3
Here the highest derivative is d 3y .
1
1 dx
d 2 y  dy  4
+   + x 5 = 0 respectively are
\ the order of the differential equation is 3
dx 2  dx 
(a) 2 and not defined (b) 2 and 2 and since, the power of highest order is 2
(c) 2 and 3 (d) 3 and 3
\ its degree is 2
Solution. Given differential equation is
1 Hence, the correct option is (d).
1
d 2 y  dy  4 Example 9. The order and degree of the differential equation
+   + x5 = 0
dx 2  dx 
( )
 2
dy  d 2 y
( )
1 1

d 2y dy 4 1 + = are:
+ = −x 5  dx  dx 2
dx 2 dx
Since the degree of
dy
is in fraction. (a) 2, 3 (b) 2, 3 (c) 2, 1 (d) 3, 4
dx 2
So, the degree of the differential equation is not defined as Solution.The given differential equation is
the order is 2.
( )
 2
Hence, the correct option is (a). dy  d 2 y
1 + =
Example 7. The degree of the differential equation:  dx  dx 2

( ) + y = 0 is
3
d 2y dy 5 Here, the highest derivative is 2,
+ 6
dx 2 dx
\ order = 2
(a) 1 (b) 2 (c) 3 (d) 5
and the power of the highest derivative is 1
Solution. The degree of the given differential equation is 1 as the
\ degree = 1
power of the highest order is 1.
Hence, the correct option is (a). Hence, the correct option is (c).

EXERCISE 9.1
I. Multiple Choice Questions (MCQs) 3. The degree of the differential equation
1. The order and degree of the differential equation 3 2
 d 2 y   dy 
 dy 
 2  +  dx  + sin  dx  + 1 = 0 is
dy
−4
dy
− 7x = 0 are respectively  dx     
dx dx
(a) 3   (b) 2   (c) 1    (d) not defined
(a) 1 (b) 2 and 1 (c) 1 and 1 (d) 1 and 2
1 and 2 [NCERT]
2. The degree of the differential equation 4. The order of the differential equation
3 d 2y dy
d y 
2 2 2 2x 2 −3 + y = 0 is
3 = 1 +  dy   is dx 2 dx
  
dx 2   dx  
(a) 2   (b) 1    (c) 0   (d) not defined
(a) 1    (b) 2   (c) 3   (d) 6 [NCERT]

Differential Equations 279


E:\AMIT_WORKS\Exam_Guru\EG_Mathematics-12_(working_02-06-2022)\EG_Mathematics-12_working\Open_Files\Chap_9\Chap_9
\ 17-Aug-2022 Amit Proof-4 Reader’s Sign _______________________ Date __________

5. The degree of the differential equation then write the value of (m + n).[S.P. 2014]
2
d y  dy  d y 2 2 3. Write the degree of the D.E.
2
+ 3   = x 2 log  2  is
dx dx
   dx  [NCERT Exemplar] 4 2
 d 2y  dy
x3  2  + x   = 0 [Delhi 2013]

(a) 1    (b) 2   (c) 3   (d) None of these  dx   dx 
II. Very Short Answer Type Questions 4 2
4. Write degree of the D.E.  dy  + 3x  d y  = 0
dy 3  dx   dx 2 
1. Determine the order and degree of: x+ = y2
dx dy
[Delhi 2013 (C)]
dx
2. If m and n are order and degree respectively, of the
5. Write the sum of the order and degree of the D.E.
differential equation:
2 3
dy  d 2y  3
d  dy  
y   + x3  2  − xy = sin x   = 0 [AI 2015]
dx  dx  
 dx   dx 

Answers 9.1
Therefore, the order of the differential equation is 2.
I. 1. We have dy dy
= 4 + 7x Hence, (a) is the correct answer.
dx dx
Squaring both sides, we get 5. The degree of the differential equation is not defined because the
 2  2
2 term x 2 log  d y  present in it is not a polynomial in d y .
dy =  dy  dy 2
16   + 49x 2 + 56x  dx  dx 2
dx  dx  dx
Hence, (d) is the correct answer.
It is of first order and second degree differential equation.
The order of the highest order derivative is 1 and its highest II. 1. Order = 1, Degree = 2 ;
positive integral exponent is 2.
∴ The order of the given differential equation is 1 and its degree Hint: x  dy  + 3 = y 2

 dx   dy 
is 2.
 dx 
Hence, (d) is the correct answer.
2
dy dy
3
2 3
⇒ x   + 3 = y2  
2  2 2  d 2 y   2   dx   dx 
2. 3 d y =   dy  
1 +   
dy
⇒  3 2  =  1 +   
2
dx   dx    dx    dx   2. m + n = 2 + 2 = 4
It is free from radicals and fractions.
2 3. Degree = 2
 
The order of the highest order derivative  d y2  is 2 and degree
is also 2.  dx  4. Degree = 1
Hence, (b) is the correct answer.
3. The degree of the differential equation is not defined because 5. Order = 2, Degree = 1
its left hand side is not a polynomial in dy .
dx \ Order + Degree = 2 + 1 = 3
Hence, (d) is the correct answer.
4. The highest order derivative present in the given differential 3 2
d  dy  = dy  d 2 y 
2 Hint:
0 ⇒ 3    2  = 0
equation is d y , which is of order 2. dx  dx   dx   dx 
dx 2

Topic 2. General Solution of a Differential Equation


The solution of a differential equation which contains arbitrary dy
constants called the general solution of the differential equation. ⇒ = a cos (x + b)
dx
For example: y = a sin (x + b) is a general solution of the
d 2y
d 2y ⇒ = –a sin (x + b)
differential equation: + y = 0 because the equation: y = dx 2
dx 2 d 2y
d 2y Now put the values of y = a sin (x + b) and = –a sin (x + b)
a sin (x + b) satisfies the differential equation + y = 0 as 2 dx 2
shown below: dx 2 in d y + y = 0, we get:
dx 2
y = a sin (x + b) – a sin (x + b) + a sin (x + b) = 0

280 Mathematics–12
E:\AMIT_WORKS\Exam_Guru\EG_Mathematics-12_(working_02-06-2022)\EG_Mathematics-12_working\Open_Files\Chap_9\Chap_9
\ 17-Aug-2022 Amit Proof-4 Reader’s Sign _______________________ Date __________

Particular Solution of A Differential Equation  dy 


The solution of a differential equation, free from arbitrary y + x
x 2 + y 2 − 2 y  dx  =0
constants i.e., the solution obtained from the general solution  dy 
by giving particular values of the arbitrary constants is called  dx 
particular solution. dy  dy 
 ( x2 + y 2 )
− 2 y  y + x = 0
For example: y = 2 sin x +
π
4 ( )
is the particular solution of
 ( x2 + y 2 )

dx
dy
− 2 y2
 dx
dy

− 2 xy = 0
d 2y dx dx
the differential equation: + y = 0.
dx 2 dy
Note: (i) General solution of order n equation has n arbitrary  ( x 2 + y 2 − 2 y 2 )
= 2xy
dx
constants and
dy
(ii) There is no arbitrary constant in a particular solution
  ( x2 − y 2 ) = 2xy
dx
of a differential equation.
\ Hence the correct option is (a).
Example 1. The number of arbitrary constants in the general
solution of a differential equation of order three is ____________ . Example 5. Family y = Ax + A3 of curves will correspond to a
Solution. The number of arbitrary constants in the solution is 3. differential equation of order
(a) 3 (b) 2 (c) 1 (d) not defined
Example 2. Number of arbitrary constants in the particular
solution of a differential equation of order two is two. Solution. The given equation is
Solution. False y = Ax + A3
Since particular solution of a differential equation has no arbitrary dy
Differentiating both sides, we get =A
constant. dx
Example 3. The differential equation representing the family of d2y
Again differentiating both sides, we have 2 = 0
circles x2 + (y – a)2 = a2 will be of order two. dx

Solution. False So the order of the differential equation is 2.
We know that the order of the differential equation is equal to the Hence, the correct option is (b).
number of arbitrary constants.
Example 6. The differential equation for which y = a cos x + b
Example 4. The differential equation of the family of curves x2 + sin x is a solution, is:
y2 – 2ay = 0, where a is arbitrary constant, is: d2y 2

dy
(a) + y=0 (b) d 2y − y = 0
dy dx 2 dx
(a) ( x 2 − y 2 ) = 2 xy (b) 2( x 2 + y 2 ) = xy
dx dx 2
(d) d y + (a − b) y = 0
2
(c) d y + (a + b) y = 0
(c) 2( x 2 − y 2 ) dy = xy (d) ( x 2 + y 2 ) dy = 2 xy dx 2 dx 2
dx dx
Solution. The given equation is
Solution. The given equation is
y = a cos x + b sin x
x2 + y2 – 2ay = 0 ..(1)
Differentiating w.r.t. x, we have dy
= – a sin x + b cos x
dx
dy dy
2x + 2 y . − 2a = 0
dx dx d2y
= – a cos x – b sin x
dx 2
dy dy
 x+ y −a = 0
dx dx d2y

= – (a cos x + b sin x)
dy dx 2
 x + ( y − a) = 0
dx d2y d2y

= –y  +y=0
dy dx 2 dx 2
 ( y − a) = –x
dx
Hence, the correct option is (a).
 y – a = −x Example 7. The differential equation of the family of curves y2
dy / dx
= 4a (x + a) is:
dy
y. +x dy  dy  dy

 a =y+ x dx (a) y 2 = 4  x +  (b) 2 y . = 4a
⇒ a=
dy dx  dx dx
dy dx
dx 2 2 2
(c) y . d 2y +  dy  = 0 (d) 2 x . dy + y  dy  − y = 0
Putting the value of a in eq. (1) we get dx  dx  dx  dx 

Differential Equations 281


E:\AMIT_WORKS\Exam_Guru\EG_Mathematics-12_(working_02-06-2022)\EG_Mathematics-12_working\Open_Files\Chap_9\Chap_9
\ 17-Aug-2022 Amit Proof-4 Reader’s Sign _______________________ Date __________

Solution. The given equation of family of curves is 2


dy dy
2
y = 4a (x + a) Solution. Given differential equation: y = x + r 1+  
dx  dx 

y2 = 4ax + 4a2 ...(1) and given solution: x2 + y2 = r2
Differentiating both sides, w.r.t. x, we get Take x2 + y2 = r2
dy
2y . = 4a dy
dx ⇒ 2x + 2y = 0
dx
dy y dy

= 2a ⇒ =a dy −x
dx 2 dx ⇒ = y
dx
Now, putting the value of a in eq. (1) we get
dy − x
On putting y = r 2 − x 2 and = in the differential
 y dy   y dy 
2
dx y
y2 = 4 x  + 4 . 
 2 dx   2 dx  dy dy
2
equation: y = x + r 1 +   , we get:
dx  dx 
2

y2 = 2 xy dy + y 2  dy 
dx  dx  2
 − x  −x 
y = x×  + r 1+  
2  y   y
 
y = 2 x dy + y  dy 
dx  dx 
−x 2 x 2 −x 2 x2 + y 2
dy  dy 
2 = + r 1+ 2 = +r
  
2x . + y.  − y = 0 y y y y2
dx  dx 

Hence, the correct option is (d). −x 2 r 2 −x 2 + r 2


y= +r 2 =
y y y
Example 8. Verify that x2 + y2 = r2 is a solution of the differential

equation: y = x
dy dy
+ r 1+   .
2
y=
(
−x 2 + x 2 + y 2 )= y 2
= y Verified.
dx  dx  y y

EXERCISE 9.2
I. Multiple Choice Questions (MCQs) 2. Verify that y = x sin x is the solution of the differential
1. The number of arbitrary constants in the general solution dy
equation x   = y + x x 2 − y 2
of the D.E. of fourth order are:  dx 
(a) 10 (b) 2 3. Verify that xy = log y + C is the solution of the differential
(c) 3 (d) 4 2
equation: dy = y
2. The number of arbitrary constants in a particular solution dx xy + 1
of a differential equation of the third order are: III. Short Answer Type Questions-II
(a) 3 (b) 2 1. Verify that the function y = aex + be2x is solution of the
(c) 1 (d) 0 d 2y dy
D.E. − 3 + 2y = 0 .[A.I. 2005]
II. Short Answer Type Questions-I dx 2 dx
2. Prove that x2 – y2 = c(x2 + y2)2 is the general solution of the
1. Verify that y = x 2 + 1 is the solution of the D.E.
differential equation (x3 – 3xy2)dx = (y3 – 3x2y)dy where c
dy xy
= . is a constant. [Delhi 2017]
dx x 2 + 1

Answers 9.2
I.1. (d) 4 dy dy
2. Find and put the values of y and in the equation:
2. (d) 0 dx dx
dy dy dy
II.1. Find
dx
and put the values of y and
dx
in the equation: x   = y + x x2 − y 2 .
 dx 
dy xy . dy dy
= 3. Find and put the values y and in the equation
dx x 2 + 1 dx dx

282 Mathematics–12
E:\AMIT_WORKS\Exam_Guru\EG_Mathematics-12_(working_02-06-2022)\EG_Mathematics-12_working\Open_Files\Chap_9\Chap_9
\ 17-Aug-2022 Amit Proof-4 Reader’s Sign _______________________ Date __________

dy y2 2
and d y2 in the given D.E.
= .
dx xy + 1 dx
dy 2 dy dy
III. 1. Find and d y2 and put the values of y, 2. Find
dx
and put in the D.E.
dx dx dx

Topic 3. Solution by Method of Separation of Variables


In this section we shall discuss three methods of solving first order \ x = C sec y
and first degree of differential equations. Hence, the required solution is x = C sec y .
13
dy  y
Differential Equations with Variables Separable Example 3. The solution of =  is y 2 3 − x 2 3 = c.
dx  x 
If first order and first degree differential equations are Solution. True
dy The given differential equation is
of the form: = f ( x ) g ( y ) . This type of differential
dx dy  y 
13
dy y1/3 dy dx
equation are called variable separable differential equations. The =   = ⇒ 1/3 = 1/3
dx  x  dx x1/3 y x
above equation can be rewritten as:
Integrating both sides, we get
dy dy
= f (x) g (y) ⇒ = f ( x ) dx dy dx
∫ y1 3 = ∫ x1 3 ∫y dy = ∫ x −1 3 dx
−1 3
dx g ( y) ⇒

dy
⇒ ∫ = ∫ f ( x ) dx 

1 −1 +1
y 3 =
1 −1 +1
.x 3 +c
g(y) 1 1
− +1 − +1
If we integrate both sides, we get the required solution with 3
3
arbitrary constant. This solution will be called general solution. 3 23 3 23
Example 1. The solution of the differential equation ydx + (x + xy) 
y = x +c
2 2
dy = 0 is ____________ . 2 2 2

y 3=x 3+ c
Solution. The given differential equation is 3
2 2  2 
   ydx + (x + xy) dy = 0 
y 3−x 3 =k  k = 3 c 
 (x + xy) dy = – y dx  x (1 + y) dy = – y dx
Example 4. Solution of differential equation x dy – y dx = 0
1+ y 1

dy = − dx represents:
y x
(a) a rectangular hyperbola
Integrating both sides, we get
1
(b) parabola whose vertex is at origin
∫ dy = − ∫ dx (c) straight line passing through origin
x
(d) a circle whose centre is at origin.
1  1
   
∫  y + 1 dy = − ∫ dx
x
Solution. The given differential equation is
dy y dy dx
   log y + y = – log x + log c xdy – ydx = 0  = ⇒ =
dx x y x
 log x + log y + log ey = log c
Integrating both sides, we get
   log (xy . ey) = log c
dy dx
\        xy = c e–y ∫y ∫x =

Hence, the required solution is xy = c e–y.
Example 2. The solution of differential equation cot y dx = x dy  log y = log x + log c  log y = log xc
is ____________ .  y = xc
which is a straight line passing through the origin.
Solution. The given differential equation is cot y dx = x dy
Hence, the correct answer is (c).
dy
⇒ = dx ⇒ tan y dy = dx Example 5. tan–1x + tan–1y = c is the general solution of the
cot y x x differential equation:
Integrating both sides, we get
dx dy 1 + y 2 dy 1 + x 2
(a) = (b) =
∫ tan y dy = ∫ x
 log sec y = log x + log c dx 1 + x 2 dx 1 + y 2

log sec y – log x = log c (c) (1 + x2) dy + (1 + y2) dx = 0
sec y (d) (1 + x2) dx + (1 + y2) dy = 0
     log
= log c
x
sec y x 1
Solution. Given equation is tan–1x + tan–1y = c

\ =c ⇒ = Differentiating w.r.t. x, we have
x sec y c
1 1 dy
x 1  + . =0

= C  = C 1 + x 2 1 + y 2 dx
sec y c 

Differential Equations 283


E:\AMIT_WORKS\Exam_Guru\EG_Mathematics-12_(working_02-06-2022)\EG_Mathematics-12_working\Open_Files\Chap_9\Chap_9
\ 22-Aug-2022 Amit Proof-4 Reader’s Sign _______________________ Date __________

 1  dy  1  y ex

 1 + y 2  dx = −  1 + x 2  

y +1
dy = x dx
e +1
dy  1 + y2  Integrating both sides, we get

= −
dx  1 + x 2  y ex
2
    (1 + x ) dy = – (1 + y ) dx 2 ∫ y + 1 dy = ∫ e x + 1 dx
 (1 + x2) dy + (1 + y2) dx = 0
y +1−1 ex
Hence the correct option is (c). 
∫ y +1
dy = ∫ e x + 1 dx
dy
Example 6. The differential equation y + x = c represents: 1 ex
dx 
∫ 1. dy − ∫ y + 1 dy = ∫ e x + 1 dx
(a) Family of hyperbolas (b) Family of parabolas
(c) Family of ellipses (d) Family of circles  y – log |y + 1| = log |ex + 1| + log k
Solution. Given differential equation is  y = log |y + 1| +log |ex + 1| + log k
dy
    y +x=c  y = log |k (y + 1) (ex + 1)|
dx
dy Hence, the correct option is (c).
    y
=c–x  y dy = (c – x) dx dy
dx Example 9. The solution of differential equation = e x − y + x 2e − y

\ Integrating both sides, we get is: dx

∫ y dy = ∫ (c − x)dx (a) y = e x − y − x 2e − y + c (b) e y − e x =


x3
+c
3
y2 x2 x2 y 2

= cx − +k ⇒ + − cx = k 3 x3
2 2 2 2 (c) e x + e y = x + c (d) e x − e y = +c
3 3
 x2 + y2 – 2cx = 2k which is a family of circles.
Solution. The given differential equation is
Hence, the correct option is (d).
dy dy
Example 7. The general solution of ex cos y dx – ex sin y dy = 0 is: = e x − y + x 2e − y  = e x . e− y + x2. e− y
(a) ex cos y = k (b) ex sin y = k dx dx
(c) e = k cos y (d) ex = k sin y
x
dy dy

= e − y (e x + x 2 )  − y = (e x + x 2 ) dx
Solution. The given differential equation is dx e
ex cos y dx – ex sin y dy = 0 
ey . dy = (e x + x 2 ) dx
 ex (cos y dx – sin y dy) = 0
 cos y dx – sin y dy = 0 [ ex  0] Integrating both sides, we have
sin y = ∫ (e x + x 2 ) dx
 sin y dy = cos y dx 

cos y
dy = dx ∫ e y dy
x3
Integrating both sides, we have 
ey = e x + +c
sin y 3
∫ cos y dy = ∫ dx x3

ey – ex =
+c
 – log |cos y| = x + log k 3
1 Hence, the correct option is (b).

log – log k = x
cos y General Solution of Variable Separable Differential
 1  1 Equations

log  =x  = ex
 k cos y  k cos y Example 10. Find the general solution of the differential equation:
1  1 dy 5x + 3 7
 = ex cos y  ex cos y = c = ,y≠− .

k c = k  dx 2y + 7 2
Hence, the correct option is (a). dy 5x + 3
Solution. = 2y + 7 .
Example 8. The general solution of the differential equation dx
(ex + 1) y dy = (y + 1) ex dx is: This is a variable separable differential equation.
(a) (y + 1) = k (e x + 1) On separating the variables, we get:
(b) y + 1 = e x + 1 + k (2y + 7)dy = (5x + 3)dx
(c) y = log [k (y + 1) (ex + 1)] ⇒ ∫ ( 2y + 7) dy = ∫ (5x + 3) dx
 x  1 1
(d) y = log  e + 1 + k ⇒    (2y + 7) 2 + C1 = (5x + 3) 2 + C2
 y +1  4 10
On multiplying throughout by 20, we get:
Solution. The given differential equation is
5(2y + 7)2 + 20C1 = 2(5x + 3)2 + 20C2
(ex + 1) y dy = (y + 1) ex dx ⇒ 5(2y + 7)2 = 2(5x + 3)2 + 20(C2 – C1)

284 Mathematics–12
E:\AMIT_WORKS\Exam_Guru\EG_Mathematics-12_(working_02-06-2022)\EG_Mathematics-12_working\Open_Files\Chap_9\Chap_9
\ 17-Aug-2022 Amit Proof-4 Reader’s Sign _______________________ Date __________

⇒ 5(2y + 7)2 = 2(5x + 3)2 + C This is a variable separable differential equation. On separating
This is the required general solution. variables we get: dy = tan2 2x dx
Example 11. Find the general solution of the differential equation:
dy 1 − cos 4x
(
⇒ ∫ dy = ∫ sec 2 2x − 1 dx ) ⇒ y = 1 tan 2x − x + C
2
=
dx 1 + cos 4x This is the required general solution.
dy 1 − cos 4x dy 2 sin 2 2x
Solution. = ⇒ = = tan 2 2x
dx 1 + cos 4x dx 2 cos 2 2x

EXERCISE 9.3
I. Multiple Choice Questions 5. Find the general solution of the differential equation:
dy 1− y 2   dy    2 dy 
1. The solution of + 2
= 0 when x = 0 and y = 1 ,  y − x  dx   = a  y + dx 
dx 1 − x 2
is
6. Find the general solution of the differential equation:
−1 −1
y=π −1 −1
y=π (1 + y2) (1 + log x) dx + x dy = 0. [A.I. 2008]
(a) sin x + sin (b) sin x − sin
2 2
7. Find the general solution of the differential equation:
(c) sin
−1
x + sin −1
y=π (d) sin −1
x − sin −1
y=π (1 + e2x)dy + (1 + y2)ex dx = 0. [A.I. 2008]
6 3
8. Find the general solution of the differential equation:
2
2. The Solution of dy = x log x + x is (x2 – yx2)dy + (y2 + x2y2)dx = 0.
dx sin y + y cos y
9. Find the general solution of the differential equation:
2
(a) y sin y = x 2 log x + C (b) y sin y = x + C dy
= e x + y + x 2e y. [A.I. 2006]
dx
2
(c) y sin y = x + log x + C
10. Find the general solution of the differential equation:
(d) y sin y = x log x + C dy
1 + x 2 + y 2 + x 2 y 2 + ( xy ) = 0 .
dx
3. The solution of cos y dy = e x + sin y + x 2esin y is
dx 11. Find the general solution of the differential equation:
3 3 ex (tan y)dx + (1 – ex) (sec2 y) dy = 0. [Delhi 2011]
x
(a) e − e
− sin y
+ x =C
−x
(b) e − e
− sin y
+ x =C 12. Find the general solution of the differential equation:
3 3
dy 1
3 3 = x sin 2 x +
x
(c) e + e
− sin y
+ x =C x
(d) e − e
sin y
+ x =C dx x log x
3 3
13. Find the general solution of the differential equation:
II. Very Short Answer Type Question dy

1. Find the general solution as the differential equation: e dx = ( x + 3)  [V. Imp.]


y dx − x dy 14. Find the general solution of the differential equation:
= 0.
y dy
sin   = x  [V. Imp.]
III. Short Answer Type Question-I  dx 
1. Solve the following differential equation (find general 15. Find the general solution of the differential equation:
solution) : sec2 x tan y dx + sec2 y tan x dy = 0.[A.I. 2007] dy
= e x − y + x 2e − y [CBSE 2022]
dx
IV. Short Answer Type Questions-II
16. Find the general solution of the differential equation:
1. Find the general solution of the differential equation:
dy
x 1 − y 2 dx + y 1 − x 2 dy = 0 . log   = ax + by  [V. Imp.]
 dx 
2. Find the general solution of the differential equation: 17. Find the general solution of the differential equation:
(1 + x) (1 + y2) dx + (1 + y)(1 + x2) dy = 0. dy
= e x + y − e− x + y
3. Find the general solution of the differential equation: dx
dy 18. Find the general solution of the differential equation:
= 1 + x + y + xy . [V. Imp.]
dx
x (cos y) dy = (xex log x + ex) dx.
4. Find the general solution of the differential equation:
dy 19. Find the general solution of the differential equation:
= 1 − x + y − xy .
dx (1 – x2)­ dy + xy dx = xy2 dx.

Differential Equations 285


E:\AMIT_WORKS\Exam_Guru\EG_Mathematics-12_(working_02-06-2022)\EG_Mathematics-12_working\Open_Files\Chap_9\Chap_9
\ 22-Aug-2022 Amit Proof-4 Reader’s Sign _______________________ Date __________

20. Find the particular solution of the differential equation: 28. Find the equation of the curve passing through the point
dy
=
x (2 log x + 1)
dx sin y + y cos y
when x = 1, then y = 0. 0, ( )
π
4
whose differential equation is sin x cos y dx +

21. Find the general solution of the differential equation: cos x sin y dy = 0
dy xe x log x + e x 29. Find the particular solution of the differential equation:
=
dx x log y dy
= e x + y + x 2e y when x = 0, then y = 0. [Delhi 2006]
dx
22. Find the general solution of the differential equation:
dy 3e 2x + 3e 4x 30. Find the particular solution of the following differential
= x −x dy
dx e +e equation: ( x + 1) = 2e −y − 1; y = 0 when x = 0.
dx
23. Find the general solution of the differential equation:
[Delhi 2012]
dy
= e x + y + e y x2 31. Find the particular solution of the differential equation:
dx
dy dy 2
24. Find the general solution of the differential equation: y − x   = a  y 2 +  if y = when x = a.
 dx   dx  a
y dx – x dy = xy dx
25. Find the general solution of the differential equation: 32. Find the particular solution: (tan y) dy = sin(x + y) +
sin(x – y) if y = 0 when x = 0. dx
dy + xy dx = x dx
26. Find the particular solution of the differential equation:
V. Long Answer Type Question
(x + x

3 2
+ x +1 ) dy
dx
= 2x 2
+ x; y = 1 when x = 0 1. Show that the general solution of the differential equation:
dy y 2 + y + 1
dy + = 0 is given by
27. For the differential equation: xy = ( x + 2) ( y + 2) , find dx x 2 + x + 1
dx
the solution of the curve passing through the point (1, – 1). (x + y + 1) = A(1 – x – y – 2xy) where A is a parameter.

Answers 9.3
2
I. 1.
dy
+
1 − y2
=0⇒
dy
=−
1 − y2 ⇒ − cos y + y sin y + cos y = x log x 2
dx 1− x 2 dx 1 − x2 2
2
x 1
− ∫ ⋅ 2 ⋅ 2x dx + ∫ x dx + C
dx dy 2 x

⇒   + =0 2
⇒ y sin y = x ⋅ 2 log x − ∫ x dx + ∫ x dx + C
2
1− x 1 − y2
2
dx dy ⇒ y sin y = x 2 log x + C
\ ∫ 1 − x2
+∫
1 − y2
=0
Hence, (a) is the correct answeer.


⇒ sin −1 x + sin −1 y = C  ...(1) dy x sin y
3. cos y dx = e e + x 2esin y

When x = 0, y = 1 , then from (1), we get Separating the variables, we get


2
− sin y x
∫ cos y e dy = ∫ (e + x ) dx
2

sin 0 + sin−1 −1 1
= C ⇒ 0 + π = C or C = π
2 6 6 Put e
− sin y
= t, ∴ e− sin y(− cos y) dy = dt
Hence, from (1) sin −1 x + sin −1 y = π is the required solution. 3 3
6
\ − ∫ dt = e x + x ⇒ − t = e x + x + C
3 3
Thus, (c) is the correct answer. 3
2
⇒ − e− sin y = e x + x + C
dy x log x + x 3
2. We have =
dx sin y + y cos y Hence (c) is the correct answer.
Seperating the variables, we get
II. 1. y = Cx
(sin y + y cos y) dy = (x log x 2 + x) dx
y dx − x dy
Integrating both sides, we get Hint: =0 ⇒ y dx = x dy
y
∫ (sin y + y cos y) dy = ∫ (x log x + x) dx
2
dy
∫ = dx
y ∫ x
⇒ ⇒ log y = log x + log C ⇒ y = Cx.

286 Mathematics–12
E:\AMIT_WORKS\Exam_Guru\EG_Mathematics-12_(working_02-06-2022)\EG_Mathematics-12_working\Open_Files\Chap_9\Chap_9
\ 17-Aug-2022 Amit Proof-4 Reader’s Sign _______________________ Date __________

III. 1. tan x tan y = C. 8. x − log y − 1 − 1 = C


2 2 x y
Hint: sec x tan y dx + sec y tan x dy = 0

2 2 Hint: (x2 – yx2)dy + (y2 + x2y2) dx = 0

⇒ ∫ sec y dy = −∫ sec x dx
tan y tan x 1− y 1 + x 2 dx = C
⇒ log tan y = – log tan x + log C
⇒ ∫ y 2 dy + ∫
x2
⇒ log(tan y tan x) = log C ⇒ tan y tan x = C 3
9. e x + e −y + x = C
IV. 1. 1 − x 2 + 1 − y 2 = C 3
dy  x + y
2 2 Hint:
= e + x 2e y 
Hint: x 1 − y dx + y 1 − x dy = 0 dx 
x dx y dy dy
⇒ ∫ +∫ = C1 ⇒ = e x × e y + x 2e y
2 2 dx
1− x 1− y
∫e dy = ∫ (x 2 + e x) dx + C
−y

2. tan −1 x + tan −1 y + 1 log(1 + x 2)(1 + y 2) = C
2 1 + x2 − 1 = C
2 2 10. 1 + y 2 + 1 + x 2 + 1 log
Hint: (1 + x)(1 + y ) dx + (1 + y)(1 + x )dy = 0 2 1 + x2 + 1
⇒ ∫ 1 + x2 dx + ∫ 1 + y2 dy = C dy
1+ x 1+ y Hint:
1 + x 2 + y 2 + x 2 y 2 + xy =0
dx
x2 + x + C dy
3. log ( y + 1) = ⇒ (1 + x 2) (1 + y 2) + xy =0
2 dx
Hint: dy = 1 + x + y + xy = (1 + x )(1 + y ) 2
⇒ 1 ∫ 2y dy + ∫ 1 + x dx = C
dx 2 1+ y 2 x
dy
⇒ ∫ = ( x + 1) dx
y +1 ∫
(
⇒ 1 2 1 + y2 + ∫
2
1+ x 2
x2
)
( x (dx)) = C
−x 2 + x + C
4. log ( y + 1) =
2 2
⇒ 1 + y 2 + ∫ 1 +2 x x(dx) = C
dy x
Hint: = (1 − x + y − xy )
dx Put 1 + x2 = z2 ⇒ x dx = z dz
z
dy
⇒ = (1 − x)(1 + y) ⇒ ∫
dy ⇒ 1 + y2 + ∫ 2 (z dz)
y +1 ∫
= (−x + 1) dx z −1
dx
z 2 dz
5. y = C(x + a) (1 − ay) ⇒ 1 + y2 + ∫ =C
z2 − 1
dy dy
Hint:  y − x  = a  y 2 +  11. tan y = C(1 − e x)
 dx   dx 
dy Hint: ex (tan y) dx + (1 – ex)(sec2 y) dy = 0


∫ = dx
y (1 − ay) ∫ x + a e x dx sec2 y dy
⇒ ∫ 1 − ex + ∫ tan y
=C
1 a  dx
⇒ ∫  y + 1 − ay  dy = ∫ x + a + log C 2
12. y = x − x sin 2x − 1 cos 2x + log(log x) + C
4 4 8
(log x) 2
6. log | x | + + tan −1 y = C
2 Hint: dy = x sin 2 x + 1

dx x log x
Hint: (1 + y2)(1 + log x)dx + x dy = 0

1 dx
⇒ ∫ dy = 2 ∫ x(1 − cos 2x) dx + ∫ x log x + C
⇒ ∫  1 + log x  dx + ∫ dy 2 = C
 x  1+ y 1 1
⇒ y = ∫ x dx − ∫ x cos 2x dx + log (log x) + C
2 2
7. ( tan y ) + tan
−1 −1 x
e =C
13. y = (x + 3) log (x + 3) − x + C
2x 2 x
Hint: (1 + e )dy + (1 + y )e dx = 0
dy
dy
Hint: e dx = (x + 3) ⇒
= log (x + 3)
x dx
⇒ ∫ dy 2 + ∫ e 2x dx = C
1+ y 1+ e ⇒ ∫ dy = ∫ log (x + 3) × 1 dx + C
Differential Equations 287
E:\AMIT_WORKS\Exam_Guru\EG_Mathematics-12_(working_02-06-2022)\EG_Mathematics-12_working\Open_Files\Chap_9\Chap_9
\ 17-Aug-2022 Amit Proof-4 Reader’s Sign _______________________ Date __________

14. y = x sin −1 x + 1 − x 2 + C dy 3e2x + 3e 4x dy 3e3x (e −x + e x)


Hint: = x −x  ⇒ =
dx e +e dx e x + e− x
 dy  = x ⇒ dy = sin −1 x
Hint: sin
3
 dx  dx 23. −e −y = e x + x + C
3
∫ dy = ∫ sin
−1
⇒ x dx
Hint:
dy
= ex+ y + e y x2 = ex + x2 e y ( )
−1 −1
(
1 −2x dx
⇒ y = ∫ (sin x ⋅ 1) dx = x sin x + ∫ ) dx
2 1 − x2
⇒ ∫e
−y
( )
dy = ∫ e x + x 2 dx
3
15. e y = x + e x + C 24. log y = log x − x + C
3
dy
= − x dx
x 2 1
Hint: dy = e x − y + x 2e −y ⇒ dy = e +y x
Hint: y dx − x dy = xy dx ⇒ ∫

y ∫ x
dx dx e
∫ e dy = ∫ (e
y x
+ x 2) dx 2
⇒ 25. − log (1 − y ) = x + C
2
16. eax e −by
+ =C ; dy
Hint: dy + xy dx = x dx ⇒ ∫
1− y ∫
a b = x dx
 dy  = ax + by
Hint: log
 dx  26. y = 1 log | x + 1 | + 3 log (x 2 + 1) − 1 tan −1 x + 1
2 4 2
dy
∫e dy = ∫ eax dx
( )
−by
⇒ = eax + by = eax × eby ⇒ 3 2
Hint: x + x + x + 1
dy 2
= 2x + x
dx dx
2
17. −e −y = e −x + e x + C 2x + x
⇒ ∫ dy = ∫ dx
dy
= ex + y − e y − x
( 1) x 2 + 1
x + ( )
Hint:


dy
dx
= e x × e y − e ye −x ⇒
dy
= (e x − e −x) e y
1
⇒ y= ∫
1
3 x − 1 dx
dx + ∫ 2 2 2
(
and C = 1
)
dx dx 2 x +1 x +1

∫e dy = ∫ (e x − e − x) dx
−y
⇒ 27. y = 2 log  x ( y + 2) + ( x − 2)

18. sin y = ( log x ) e + C


x
dy
Hint: ( xy ) = (x + 2 )(y + 2 )
Hint: (x cos y)dy = (xe log x + e )dx x x dx

x(
⇒ ∫ ( cos y ) dy = ∫ log x + 1 e x dx ) ⇒
y dy x+2
∫ y +1 = ∫ x
dx

⇒ sin y = (log x)ex + C ⇒ y – 2 log(y + 2) = x + 2 log x + C


Put (1, – 1), it gives C = – 2
19. ( y − 1) 1 − x 2 = Cy
28. sec y sec x = 2
Hint: (1 – x2)dy + xy dx = xy2 dx
Hint: sin x cos y dx + cos x sin y dy = 0

⇒ ∫ dy = −1 ∫ −2x dx
y ( y − 1) 2 1 − x 2 ∫ tan y dy + ∫ tan x dx = C


 1 1
∫  y − 1 − y  dy =
−1 log 1 − x 2
2 ( ) 29. 1 = −e x − x3 + 2
ey 3
20. y sin y = x 2 log x dy
Hint:
= e x + y + x 2e y
dx
dy x ( 2 log x + 1)
Hint: =
∫e dy = ∫ (e x + x 2) dx and C = 2
−y
dx sin y + y cos y ⇒

∫ y cos y dy + ∫ sin y dy = ∫ (log x ) ( 2x ) dx + ∫ x dx


( )

30. y = log 2x + 1
x x +1
21. y log y − y = (log x) e + C
dy
Hint: ( x + 1)
= 2e −y − 1
dy x e x log x + e x dx
Hint:
=
dx x log y y
( x + 1) dy = 2 −ye ⇒ ey dx
∫ 2 − e y dy = ∫ x + 1
⇒ ∫ log y dy = ∫ ( log x ) e dx + ∫ 1 e x dx x dx e
x y
31. + x +2 a = 0
22. y = e3x + C 1− y a

288 Mathematics–12
E:\AMIT_WORKS\Exam_Guru\EG_Mathematics-12_(working_02-06-2022)\EG_Mathematics-12_working\Open_Files\Chap_9\Chap_9
\ 17-Aug-2022 Amit Proof-4 Reader’s Sign _______________________ Date __________

Hint: y − x

dy dy
= a  y 2 +  = ay 2 + a

dy ⇒ ∫ sec y tan y dy = 2 ∫ sin x dx + C
dx dx  dx
V. 1. x + y + 1 = A (1 − x − y − 2xy )
dy dx
⇒ ∫ y (1 − ay ) = ∫ x + a 2
Hint: dy + y 2 + y + 1 = 0
1 −a dx dx x + x + 1
⇒ ∫ y dy − ∫ 1 − ay dy = ∫ x + a
⇒ ∫ 2 dy dx
y + y + 1 ∫ x2 + x + 1
+ =C
32. sec y + 2 cos x = 3
⇒ dy dx
∫ +∫ =C
( ) ( )
2 2 2 2
   
dy
Hint: ( tan y ) = sin ( x + y ) + sin ( x − y )
y+ 1 +  3 x+ 1 +  3
dx 2  2  2  2 

dy ⇒ 2 tan −1  2y + 1 + 2 tan −1  2x + 1 = C
⇒ tan y = 2 sin x cos y    
dx 3 3  3 3 

Topic 4. D.E. Reducible to Variable Separable


Example 1. Solve the following differential equation:
( x + y − 3) dy = 5.
⇒ ∫ ( )
z
z+5
dz = ∫ dx
dx
( z + 5) − 5 dz
Solution. ( x + y − 3) dy
dx
= 5 ⇒ ∫ z+5
= x+C
dy dz
Put


x+y–3= z
dy
= dz −1
⇒ 1+ =
dx dx ⇒ ∫ dz − ∫ ( )
5
z+5
dz = x + C

dx dx
⇒ z – 5 log| z + 5 | = x + C
\ ( x + y − 3) dy
dx
= 5 becomes ( z )
dz
dx
−1 = 5 ( ) ⇒ (x + y –3) – 5 log | x + y – 3 + 5 | = x + C

⇒ z ( )
dz
dx
−z = 5 ⇒ (x + y – 3) – 5 log | x + y + 2 | = x + C
⇒ y – 5 log | x + y + 2 | = 3 + C
⇒ z( ) dz
dx
= z + 5 ⇒
dz z + 5
dx
=
z ⇒ y – 5 log | x + y + 2 | = C1

EXERCISE 9.4
I. Multiple Choice Questions
(a) e2y = e3x + x3 + k (b) 1 e 2x = 1 (e3x + x3) + k
1. The solution of the equation (2y – 1) dx – (2x + 3) dy = 0 2 3
is 1 2 y 1 3x
(c) e = (e − x3 ) + k (d) e2y = e3x + x2 + k
2x − 1 2x + 3 2 3
(a) = C (b) =C
2y + 3 2y − 1 II. Short Answer Type Questions-II
2x − 1 2x + 1 1. Solve the differential equation: sin −1  dy  = ( x + y )
(c) = C (d) =C
2y − 1 2y − 3  dx 
dy
2. The solution of the differential equation 2. Solve the differential equation: ( x + y + 1) = 1
dx
dy
(x 2 + 1) + y 2 + 1 = 0, if it is given that y = 1 when 3. Solve the differential equation:
dy x+ y
dx = 1+ e
x=0 dx
1+ x 4. Solve the differential equation: dy = x + 2y − 1
(a) y = 2 + x2 (b) y = dx x + 2y + 1
1− x
5. Solve the differential equation: dy x+ y −3
1− x =
(c) y = x(x – 1) (d) y = dx x + y + 3
1+ x
6. Solve the differential equation: dy = ( 4x + y + 1)2
3. The solution of the differential equation dy = e3x − 2y + x 2e −2y dx
is dx 7. Solve the differential equation: dy = cos(x + y) + sin (x + y)
dx

Differential Equations 289


E:\AMIT_WORKS\Exam_Guru\EG_Mathematics-12_(working_02-06-2022)\EG_Mathematics-12_working\Open_Files\Chap_9\Chap_9
\ 17-Aug-2022 Amit Proof-4 Reader’s Sign _______________________ Date __________

Answers 9.4
2x + 3 1− x dz = 3z + 2 ⇒ 1 3z + 6 dz = dx
3 ∫ 3z + 2 ∫
I.1. (b) = C 2. (d) y =
2y − 1 1+ x dx
z+2

3. (b) 1 e2x = 1 (e3x + x3) + k 5. ( y − 3) + 3 log ( x + y ) = x + C


2 3

II.1. tan ( x + y ) − sec ( x + y ) = x + C dy x + y − 3


Hint:
= ,
dx x + y + 3
dy dy dz
Hint:
= sin ( x + y ) . Put x + y – 3 = z ⇒ 1+ =
dx dx dx
dy dz
Put x + y = z ⇒ 1+ = z+6
dx dx ⇒ ∫ z + 3 dz = 2∫ dx + C
\ dz − 1 = sin z ⇒ dz
∫ 1+ sin z = ∫ dx + C ⇒ z + 3 log(z + 3) = 2x + C
dx
⇒ (x + y – 3) + 3 log(x + y) = 2x + C
2. y − log ( x + y + 2) = C ⇒ y – 3 + 3 log(x + y) = x + C

dy −1  4x + y + 1
Hint: ( x + y + 1)
=1 6. tan 
2 
= 2x + C
dx
Put x + y + 1 = z dy
= ( 4x + y + 1) .
2
dy dz dy dz Hint:

⇒ 1+ = ⇒ = −1 dx
dx dx dx dx Put 4x + y + 1 = z

dx ( )
\ z dz − 1 = 1 ⇒ z dz − z = 1
dx ( ) ⇒ 4+
dy dz
=
dx dx

dy dz
=
dx dx
−4

( )
z dz dz = z 2 + 4
⇒ ∫ z + 1 = ∫ dx + C \ dz − 4 = z 2
dx

dx
 x+ y  1
3. log  xe+ y = 2x + C ⇒ ∫ 4 + z 2 dz = ∫ dx
e + 2 
 x + y 
Hint:

dy
= 1 + ex + y 7. log 1 + tan  = x + C.
dx   2  
dy dz dy
Put x + y = z ⇒ 1 + = Hint:
= cos ( x + y ) + sin ( x + y ).
dx dx dx

( )
dz − 1 = 1 + z ⇒ dz = z + 2
dx
e
dx
e Put x + y = z ⇒
dy dz
=
dx dx
−1

 dz  = dx + C ∴ The given equation becomes:


 e + 2  ∫
⇒  z . Now again put ez = p
dz − 1 = cos z + sin z
dx
dp
⇒ e z dz = dp ⇒ p dz = dp ⇒ dz = dz = 1 + cos z + sin z
p ⇒
dx
( )
dz 1
\ ∫ e z + 2 = x + C, becomes ∫ p (p + 2) dp = x + C ⇒
dz
∫ (1+ cos z ) + sin z = ∫ dx + C
4. 3(x + 2y − 1) + 4 log (3x + 6y − 1) = 9x + C
dz
dy x + 2y −1
⇒ ∫ 2 cos2 z + 2 sin z cos z = x + C
Hint:
= . Put x + 2y − 1 = z 2 2 2
dx x + 2y + 1
sec2 z dz
⇒ 1 + 2dy = dz ⇒ 1  dz − 1 = dy ⇒ 1∫ 2 = x+C
dx dx 2  dx  dx 2 1 + tan z
2
\ 1  dz − 1 = z
( )
dz 2z 1
⇒ = +
2  dx  z + 2 dx z + 2 ⇒ log 1 + tan z = x + C
2

290 Mathematics–12
E:\AMIT_WORKS\Exam_Guru\EG_Mathematics-12_(working_02-06-2022)\EG_Mathematics-12_working\Open_Files\Chap_9\Chap_9
\ 17-Aug-2022 Amit Proof-4 Reader’s Sign _______________________ Date __________

Topic 5. Homogeneous Differential Equations


Homogeneous Function dy dv
By substituting y = vx and =v+x in equation (1),
A function F(x, y) is a homogeneous function of degree n if: dx dx
we get:
 y  x
F(x, y) = x n g or F ( x, y ) = y nh  
 x  y dv dv
v+ x = f(v) ⇒ x = φ (v ) − v
or F(λx, λy) = λ F(x, y) n dx dx
For example: dv dx
(i) F1(x, y) = 5y 2 + 2xy + x2
⇒ ∫ φ (v ) − v = ∫ x
+ C Evaluate it.

Change x to λx and y to λy in the above function, we get: y


Now replace v by. This gives the general solution of the
F1(λx, λy) =5λ2y2 + 2λ2xy + λ2x2 x
given homogeneous differential equation.
= λ2(5y2 + 2xy + x2) = λ2F(x, y)
⇒ F1(x, y) = 5y2 + 2xy + x2 is a homogeneous function of (B) If the homogeneous differential equation is of the form:
dx  
degree 2. ⇒ = F ( x, y ) = φ  x  then substitute:
dy  y
 y
(ii) F2(x, y) = cos   dx dv
x x = vy and= v+ y
dy dy
Now change x to λx and y to λy. It gives:
Example 1. Correct substitution for the solution of the differential
 λy  = cos  y  = λ 0 cos  y 
F2(λx, λy) = cos
 λx   x  x equation of the type dx = f(x, y) , where f(x, y) is a homogeneous
dy
⇒ F(x, y) is a homogeneous function of degree 0. function of zero degree is y = vx.
Homogeneous Differential Equations Solution. True
xdy y  y
dy Example 2. Solution of = y + x tan is sin   = cx.
A differential equation of the form: = F ( x, y ) is said to be dx x  x
dx Solution. True
homogeneous if F(x, y) is a homogeneous function of degree zero The given differential equation is
or If F(λx, λy) = λ0 F(x, y). dy  y
x = y + x tan  
dx  x
For example:   

dy 3x 2 + y 2   dy = y + tan  y 

(i) = is a homogeneous differential equation  x
dx x
dx xy + 2y 2 dy dv
Put y = vx ⇒ =v+x
3x 2 + y 2 dx dx
because is a homogeneous function of degree dv vx  vx 
xy + 2y 2  v + x ⋅ = + tan  
dx x  x
zero, as shown below:
dv dv
 v + x = v + tan v ⇒ x = tan v
3x 2 + y 2 dx dx
F(x, y) =
xy + 2y 2 dv
=
dx
⇒ cot v dv =
dx

tan v x x
3 ( λx ) + ( λy )
2 2
⇒ F(lx, ly) = Integrating both sides, we get
( λx ) ( λy ) + 2 ( λy )2 dx
2 2 2
 log sin v = log x + log c
∫ cot v dv = ∫
λ  3x + y   3x 2 + y 2  x
= 2 2
λ  xy + 2y 
= λ0  ( )
 xy + 2y 2 
y
 log sin v – log x = log c  log sin = log xc

x
y

\     sin = xc
= l0 F(x, y) x
Example 3. Show that the differential equation:
How to Solve a Homogeneous Differential
dy
Equation? x2 = x 2 + xy + y 2 is a homogeneous differential equation and
dx
(A) We can solve a homogeneous differential equation of the then solve it.
type: Solution. The given differential equation is:
dy  y dy dy x 2 + xy + y 2
= F ( x, y ) = φ  ...(1) x2 = x2 + xy + y2 ⇒ =  ...(1)
dx  x dx dx x2

Differential Equations 291


E:\AMIT_WORKS\Exam_Guru\EG_Mathematics-12_(working_02-06-2022)\EG_Mathematics-12_working\Open_Files\Chap_9\Chap_9
\ 17-Aug-2022 Amit Proof-4 Reader’s Sign _______________________ Date __________

2 2 dy dv
Now take: F(x, y) = x + xy + y ...(2) ⇒ = v+ x
x 2 dx dx
dy
On changing x to λx and y to λy in the equation (2), we get, On putting this value of and y = vx in equation (3), we get
dx
( λx )2 + ( λx )( λy ) + ( λy )2 dv = x + x ( vx ) + ( vx )
2 2
F(lx, ly) = v+ x
( λx )2 dx x2
λ 2 x 2 + λ 2 xy + λ 2 y 2 v+ x
dv x 2 + vx 2 + v 2 x 2
F(lx, ly) = ⇒ =
λ 2x 2 dx x2

x 2+ xy + y 2 
0
dv
v+ x
x 2 1+ v + v 2 ( dv )
= v2 + v + 1
 = λ F ( x, y )
⇒ F(lx, ly) = λ 
0 ⇒ = ⇒ v+ x
 x2 dx x2 dx
dv
F(lx, ly) = l0 F(x, y) ⇒ x = v2 + 1
dx
⇒ Given equation is a homogeneous differential equation. Now on separating the variables, we get
dv dx
For solving the given equation: ⇒ = ⇒ ∫ 2dv = ∫ dx
v2 + 1 x v +1 x
dy x 2 + xy + y 2 ⇒ tan–1 (v) = log x + C
Take: = ...(3)
dx x2
y
and take y = vx ⇒ tan −1   = log x + C
 x
This is the required solution of the given differential equation.

EXERCISE 9.5
I. Multiple Choice Questions 2. Show that the family of curves for which the slope of the
1. A homogeneous differential equation of the form  x2 + y 2 
tangent at any point (x, y) on it is  is given by
dx = h  x  can be solved by making the substitution  2xy 
x2 – y2 = Cx. or y2 – x2 = Cx
dy  y
  dy
3. Solve the differential equation: x 2 = x 2 − 2y 2 + xy .
(a) y = 2x  (b) v = yx  (c) x = vy  (d) x = v dx
4. Solve the differential equation:
2. Which of the following is a homogeneous differential
equation?   x dy − y dx = ( )
x 2 + y 2 dx . [V. Imp.]
(a) ( 4x + 6y + 5 ) dy − ( 3y + 2x + 4 ) dx = 0
5. Solve the differential equation:
( )
(b) ( xy ) dx − x3 + y 3 dy = 0
  y  y
   x cos   + y sin    y dx =
(c) ( x + 2y ) dx + 2xy dy = 0
3 2
  x   x

(d) y dx + ( x − xy − y ) dy = 0
2 2 2   y  y
 y sin   − x cos    x dy = 0 [A.I. 2010 (C)]
 x x 
y  y 
3. If x sin   dy =  y sin   − x  dx and dy y
6. Solve the differential equation: x   − y + x sin   = 0 .
x   x    dx   x

 y 7. Solve the differential equation:


y (1) = π , then the value of cos   is equal to
x y
2 y dx + x log   dy − 2x dy = 0 .[A.I. 2010]
 x
[CBSE 2013] [HOTS] 8. Solve the differential equation:

(a) x    (b) 1    (c) log x   (d) ex ( )  x


1 + e x / y dx + e x / y 1 −  dy = 0 .

x  y
II. Short Answer Type Questions-II 9. Solve the differential equation: (x + y) dy + (x – y) dx = 0,
given that y = 1 when x = 0.
1. Show that the differential equation is a homogeneous and
solve it. 10. Solve the differential equation: x2 dy + (xy + y2­) dx = 0,
given that y = 1 when x = 1. [Delhi 2010]
( x − y ) dy
dx
= ( x + 2y ) [A.I. 2010, 2017] 11. Solve the differential equation:

292 Mathematics–12
E:\AMIT_WORKS\Exam_Guru\EG_Mathematics-12_(working_02-06-2022)\EG_Mathematics-12_working\Open_Files\Chap_9\Chap_9
\ 17-Aug-2022 Amit Proof-4 Reader’s Sign _______________________ Date __________

20. Solve the differential equation:


 2  y  π
 x sin  x  − y  dx + x dy = 0; given that y = 4 y  y 
  xy log   dx +  y 2 − x 2 log    dy = 0 .

 x   x 
when x = 1. [Delhi 2011 (C)]
12. Solve the differential equation: [Delhi 2010 (C)]
dy y y dy
− + cosec   = 0, given that y = 0 when x = 1 . 21. Solve the differential equation: x = y [ log y − log x + 1]
dx x  x dx
[A.I. 2011 (C), Delhi 2009]  [V. Imp.] [Delhi 2011]
13. Solve the differential equation:
dy 22. Solve the differential equation:
2xy + y 2 − 2x 2 = 0, given that y = 2 when x = 1.
dx (x2 + 3xy + y2) dx – x2 dy = 0
[Delhi 2007, 2008 (C)]
14. Show that the differential equation: dy x2 − y 2 + y
23. Solve the differential equation: =
y dy y dx x
x cos   = y cos   + x is homogeneous and solve
 x  dx  x dy y y
it. 24. Solve the differential equation: = + sin  
dx x  x
15. Show that the differential equation:
x
 x  dy  = x 2 + 5xy + 4y 2
  25. Solve the differential equation: x 2
(2y) e y  (dx) +  y − 2xe y  dy = 0 is a homogeneous and  dx 
 
  26. Solve the differential equation:
find its particular solution, given that: x = 0 when y = 1
 dx 
16. Solve the differential equation: y   = x ( log x − log y ) + 1 
[V. Imp.]
 dy 
( x dy − y dx ) y sin  xy  = ( y dx + x dy ) x cos  xy 
dy xy
27. Find the particular solution of the D.E. = given
dx x 2 + y 2
(
17. Solve the differential equation: y e x / y dx = x e x / y + y dy ) that y = 1 when x = 0. [Delhi 2015]
dy y ( 2y − x )
18. Solve the differential equation: = and
dx x ( 2y + x ) dy
=
y2
. [AI 2015]
28. Solve the differential equation:
y = 1, when x = 1. [Delhi 2008] dx xy − y 2
2 2
dy y 29. Show that the family of curves for which dy = x + y is
19. Solve the differential equation: x = y − x tan  
dx  x dx 2xy
[AI 2009] given by x2 – y2 = cx.[Delhi 2017]

Answers 9.5
y
I.1. A homogeneous differential equation of the form dx = h  x  y sin   − x
can be solved by making the substitution x = vy dy  y dy x y 1 ...(1)

⇒ = = −
dx  y x y 
Hence, (c) is the correct answer. x sin   sin  
x x
2. y 2dx + ( x 2 − xy − y 2) dy = 0 ⇒ dy = − y2
dy
dx x − xy − y 2
2 Now, put y = vx, so that = v + x dv ...(2)
dx dx
2
Dividing numerator and denominator on RHS by x , we get From (1) and (2), we get
    2
 y v sin v − 1 ⇒ x dv = v sin v − 1 − v = −1
dy  
x v + x dv =
=− 2
dx sin v dx sin v sin v
dx y y
1 − −   Separating the variables, we get
x x
[Homogenous equation]   − sin v dv = dx
x
Hence, (d) is the correct answer.
Integrating both sides, we get
 y   y  y
3. x sin   dy =  y sin   − x  dx cos v = log x + C ⇒ cos = log x + C
x
   x
   x

Differential Equations 293


E:\AMIT_WORKS\Exam_Guru\EG_Mathematics-12_(working_02-06-2022)\EG_Mathematics-12_working\Open_Files\Chap_9\Chap_9
\ 17-Aug-2022 Amit Proof-4 Reader’s Sign _______________________ Date __________

Since y (1) = π dv  x cos v + vx sin v  vx


⇒ v+ x = 
2 dx  vx sin v − x cos v  x
∴ cos π = log 1 + C ⇒ C = 0  2 
2 ⇒ x dv =  v cos v + v sin v  − v
dx  v sin v − cos v 
y
Hence, cos   = log x
 x  ⇒ x dv = 2v cos v
dx v sin v − cos v
Therefore, (c) is the correct answer.

⇒ v sin v − cos v dx
∫ v cos v
dv = 2 ∫
x
2 2 −1  x + 2y 
II. 1. log (x + xy + y ) = 2 3 tan   +C
3x  ⇒ log 
sec v 
= log x 2 + log C
 v 
dy x + 2y x + 2y
Hint:
= take: F ( x, y ) = sec v
dx x − y x− y ⇒ = Cx 2 ⇒ sec v = Cvx2
v
λ ( x + 2y ) y y
⇒ F ( λx, λy ) = = λ 0 F ( x, y ) ⇒ sec   = C   x 2 = xyC
λ( x − y)  x  x
Given differential equation is homogeneous 1 xy C
⇒ =
 y 1
Take dy = x + 2y  ...(1) cos
 x
dx x − y
2 2
2. Hint: Slope of the tangent is x + y   y   = C sin  y 
2xy 6. x 1 − cos
  x    x
dy x 2 + y 2
\ =  ...(1) y
dx 2xy y − x sin  
dy  x
It is homogeneous. Hint: =
dx x
F(lx, ly) = l0 F(x, y) ⇒ Homogeneous
3. 1 log x + 2 y = log | x | + C
vx − x sin v
2 2 x− 2 y ⇒ v + x dv = = v − sin v
dx x
2 2
Hint: dy = x − 2y2 + xy
y
7. Cy = log   − 1
dx x  x
F(lx, ly) = l0 F(x, y) ⇒ Homogeneous
2 2
Hint: dy =
y
dx y
Take dy = x − 2y2 + xy 2x − x log  
dx x  x
⇒ v + x dv = vx
=
v
4. y + x 2 + y 2 = Cx 2 , dx 2x − x log v 2 − log v

2 2 ⇒ x dv = −v + v log v
dy y + x + y dx 2 − log v
Hint: =
dx x
 log v − 2  dx C
F(lx, ly) = l0 F(x, y) ⇒ Homogeneous ⇒ ∫  v (log v − 1)  dv = −∫ x
+
2 2
dy y + x + y  (log v − 1) − 1
Take: =
dx x ⇒ ∫  v (log v − 1)  dv = − log x + log C
vx + v 2 x 2 + x 2
⇒ v + x dv = ∫
dv dv
v ∫ v (log v −1)
Put y = vx ⇒ − = − log x + log C
dx x

⇒ dv dx
∫ 1 + v2
=∫
x 8.
 x
 
y e y  + x = C ,

5. xy cos  y  = C1  x x
 x  dy = 0
 x Hint: 1 + e y  dx + e y 1 −

   y 
  y  y  x
 x cos  x  + y sin  x   y  
e y  x − 1
Hint: dy = 
dv e ( v − 1)
v
dx  y y  x ⇒ dx =
y 
y sin   − x cos    dy x ⇒ v+ y =
dy 1 + ev
  x  x 
1+ e y

294 Mathematics–12
E:\AMIT_WORKS\Exam_Guru\EG_Mathematics-12_(working_02-06-2022)\EG_Mathematics-12_working\Open_Files\Chap_9\Chap_9
\ 17-Aug-2022 Amit Proof-4 Reader’s Sign _______________________ Date __________

⇒ F(lx, ly) = l0 F(x, y) ⇒ Homogeneous


( 2
9. log y + x + 2 tan 2
=)
y π
 x 2
−1 
y
y cos   + x
Take: dy  x 
=
dy y − x dx y
Hint:
= ⇒ v + x dv = v − 1 x cos 

dx y + x dx v + 1  x
v +1 dx dv = v cos v + 1
⇒ ∫ v 2 + 1 dv = −∫ x
+C ⇒ v+x
dx cos v

2 ( )
1 log v 2 + 1 + tan − 1 v + log x = C ⇒ x dv = 1
dx cos v
⇒ ∫ cos v dv = ∫
dx
x
x
 y  + log x 2 = C
⇒ log(y2 + x­2) – log x2 + 2 tan −1 15. 2e y + log | y | = 2
 x
x  x
10. y + 2x = 3x 2 y Hint: 2ye y (dx) +  y − 2xe y  dy = 0
Hint: x2 dy + (xy + y2) dx = 0
x
2x e y − y
 xy + y 2  ⇒ dx = 0
x ⇒ F(λx, λy) = λ F(x, y)

dy
dx
= −
 x 2 
dv
⇒ v + x = − v + v2
dx ( ) dy
2y e y
x
⇒ x dv = − (v 2+ 2v) ⇒ − ∫ dv = ∫ dx y
dx v(v + 2) x dx 2 x e − y
= x
( )
dy
⇒ −1 ∫ 1 − 1 dv = log x + C 2ye y
2 v v+2
Put x = yv and dx = v + y dv
 y  = log | ex | dy dy
11. cot
 x
dv = 2vev − 1 dy
This gives: v + y ⇒ 2∫ evdv = −∫
 y 
Hint:  x sin 2   − y  dx + x dy = 0

dy 2ev y
  x 
 y
y 16. sec   = C xy
y − x sin   2 x
dy  x
⇒ = y y
dx x xy cos   + y 2 sin  
Hint: dy =  x  x
dv
⇒ v + x = v − sin 2 v dx y y
dx xy sin   − x 2 cos  
 x  x
12. cos  y  = log (xe) Dividing the numerator and denominator on RHS by x2, we get
 x 2
 y  cos  y  +  y  sin  y 
 x   x  
 x 
2  x
dy y y dy
Hint:
− + cosec   = 0 =
dx x  x dx  y  sin  y  − cos  y 
 x  x  x
⇒ v + x dv − v + cosec v = 0
dx dy
Put y = vx and = v + x dv
dx dx
x dv = − cosec v ⇒ ∫ sin v dv = −∫ dx + C
dx x v cos v − v 2 sin v
v + x dv =
2x dx v sin v − cos v
13. y=
1 − log | x | 2v cos v
⇒ x dv =
2 dx v sin v − cos v
Hint: 2xy + y 2 − 2x 2 dy = 0 ⇒ dy = 2xy +2 y
dx dx 2x  v sin v − cos v 
⇒ ∫  dv = 2∫ dx
2
⇒ v + x dv = 2v + v ⇒ ∫ dv2 = 1 ∫ dx v cos v  x
dx 2 v 2 x
⇒ log sec 2v = log | C |
 y  = log | Cx | vx
14. sin
 x x
y 17. e y = log y + C
y cos   + x x
dy x  x x  y
Hint:

dx
=
y Hint: y e dx =  x e y + y dy ⇒ dx = xe +x y
y
x cos     dy
x ye y

Differential Equations 295


E:\AMIT_WORKS\Exam_Guru\EG_Mathematics-12_(working_02-06-2022)\EG_Mathematics-12_working\Open_Files\Chap_9\Chap_9
\ 17-Aug-2022 Amit Proof-4 Reader’s Sign _______________________ Date __________

Put x = vy and dx = v + y dv ⇒ x dv = v log v ⇒ ∫ dv = ∫ dx


dy dy dx v log v x
v
v + y dv = ve v+ 1 ⇒ y dv = 1v 1 22.
−x
= log x + C ;
dy e dy e x+ y
dy 2
+ y2
⇒ ∫ ev dv = ∫ Hint: dy = x + 3xy
y dx x 2

2y dv 2 dv dx
18.
x
+ log ( yx ) = 2 ⇒ v + x = v + 3v + 1
dx
⇒ ∫ (v + 1)2 = ∫ x

dy y (2y − x) v(2v − 1) 23. y = x sin (log Cx) ;


Hint:
= ⇒ v + x dv =
dx x(2y + x) dx 2v + 1
dy x2 − y 2 + y
⇒ x dv = −2v ⇒ ∫ 2v + 1 dv = −2∫ dx Hint: =
dx 2v + 1 v x dx x
dv dv dx
2y ⇒ v+ x = 1− v + v ⇒ ∫
2
=∫
⇒ + log v + log x 2 = C. Now put (1, 1) dx 1− v 2 x
x
–1
⇒ sin v = log Cx
y
19. x sin   = C , 24. y = 2x tan −1 ( Cx ) ,
 x
dy y y dy y y
Hint: = − tan   Hint:
= + sin
dx x  x dx x x
dv dx + C
⇒ v + x dv = v − tan v ⇒ x dv = − tan v
⇒ v + x = v + sin v ⇒
dx ∫ cosec v dv = ∫ x
dx dx

⇒ ∫ cot v dv = −∫
dx + C
x
2 ()
⇒ log tan v = log Cx

25. log x + x =C ,
 2
20. x 1 + 2 log
 y   + 4y 2 log y + C = 0
( ) 2 ( x + 2y )
  x  
2 2
 y  dx +  y 2 − x 2 log  y   dy = 0 Hint: dy = x + 5xy2 + 4y ⇒ x dv = (2v + 1) 2

Hint: xy log
 dx x dx
 x  x  
dv dx ⇒ −1 =
y
−xy log    y  log  y  ⇒ ∫ (2v + 1)2 = ∫ x 2(2v + 1)
log x + C
dy  x  x  x
⇒ = =
dx 2 2  y   −  y
y
2
26. x = yeCy ;
y − x log log
 x  x  x
dx
Hint: y = x [ log x − log y + 1]

⇒ v + x dv = v log v 2 ⇒ x dv = v log v 2 − v dy
dx log v − v dx log v − v
 
⇒ dx = x log  x  + 1
v3 log v − v 2
⇒ x dv =  ⇒ ∫ dv = ∫ dx + C dy y   y 
dx log v − v 2 v3 x
dx dv
log v Put x = yv and =v+ y
⇒ ∫ dv − ∫ 1 dv = log | x | + C dy dy
v3 v
− log v 1 dv
⇒ − 2 = log | v | + log | x | + C ⇒ v + y = v [ log v + 1]
2v 2 4v dy
 y 
⇒ x 2 1 + 2 log    + 4y 2 ( log y + C) = 0 x 2 = log y
  x  27.
2y 2
21. y = xeCx ,
dy 28. y = x [ log y + C]
Hint: x
= y ( log y − log x + 1)
dx
dy y  y   dy 
⇒ = log   + 1 29. Hint: 2xy   = x2 + y2.
dx x   x  dx

dy Put y = vx and dy = v + x dv
Put y = vx and = v + dv dx dx
dx dx
2v dv dx
v + x dv = v [log v + 1] = v log v + v
⇒ ∫ 1 − v2 = ∫ x
dx

296 Mathematics–12
E:\AMIT_WORKS\Exam_Guru\EG_Mathematics-12_(working_02-06-2022)\EG_Mathematics-12_working\Open_Files\Chap_9\Chap_9
\ 17-Aug-2022 Amit Proof-4 Reader’s Sign _______________________ Date __________

Topic 6. Linear Differential Equations


dy Example 2. General solution of the differential equation of the
Definition: A differential equation of the form: + Py = Q dy
dx type + P1 x = Q1 is given by ____________ .
where P and Q are constant or function of x only is called as first dx
order linear differential equations. Solution. General solution of the differential equation of the type
dy
For example: + P1 x = Q1 is given by x × I.F. = ∫ Q × I.F. dy + c
dx
dy
(i) + xy = cos x,
x ⋅ e∫ = ∫ Q1 ⋅ e ∫
P1dy P1dy
dx 
dy + c .
dy y
(ii) + = x, dy
dx 1 + x 2 Example 3. The solution of the differential equation x + 2y = x 2
is ____________ . dx
dy
(iii) + y = e x are linear differential equations:
dx dy
Solution. The given differential equation is x + 2y = x 2
Another form of the linear differential equation is: dx
dx dy 2
+ Px = Q.        + y = x.
dy dx x
Here P and Q are either constant numbers or functions of Since, it is linear differential equation
y only. 2
\ P= and Q = x
For example: x
Integrating factor I.F.
(i) dx + x = sin y,
dy ∫ dx
2 2
= e∫
Pdx
= e x = e 2 log x = elog x = x 2
dx x
(ii) + = e y etc. are linear differential equations.
dy y 2 \ Solution is
dy y × I.F. = ∫ Q × I.F. dx + c
To solve linear differential equation: + Py = Q
dx
∫x
3
∫x.x
2
 y . x2 =
dx + c  y . x2 = dx + c
dy
We first write the given equation in the form: + Py = Q
dx 1 4 1 2
where P and Q are constants or function of x only.  y . x2 =
x +c  y= x + c . x–2
4 4
(i) Find the integrating factor (I.F.) = e∫
P dx
1 2
Hence, the solution is y = x + c . x–2.
4
(ii) Write the solution of the given differential equation as:
dy
y ( I.F.) = ∫ Q ( I.F.) dx + C Example 4. The solution of (1 + x 2) + 2xy − 4x 2 = 0 is
__________ dx
In case the linear differential equation is of the form:
dx Solution. The given differential equation is
+ Px = Q where P and Q are either constants or function of
dy dy
(1 + x 2) + 2xy − 4x 2 = 0
y only. dx
∫ P dy
dy 2xy 2
Here also find the I.F. = e and solution of the differential 
+ 2
= 4x 2
dx 1 + x 1+ x
equation will be:
Since it is a linear differential equation
x(I.F.) = ∫ Q ( I.F.) dy + C
\
2
P = 2x 2 and Q = 4x 2
Solution of Linear Differential Equations of the 1+ x 1+ x
Type Integrating factor I.F.
2x dx
dy ∫ 2 2
   = e ∫ = e 1+ x
Pdx
+ Py = Q = elog(1+ x ) = (1 + x 2)
dx
dy y \ Solution is y × I.F = ∫ Q × I.F. dx + c

Example 1. + = 1 is an equation of the type
dx x log x x
____________ . 4x
y × (1 + x2) = ∫ 1 + x 2 × (1 + x ) dx + c
2


dy y
Solution. The given differential equation + = 1 is of
dx x log x x
the type
dy
+ Py = Q. 
y × (1 + x2) = ∫ 4x
2
dx + c
dx

Differential Equations 297


E:\AMIT_WORKS\Exam_Guru\EG_Mathematics-12_(working_02-06-2022)\EG_Mathematics-12_working\Open_Files\Chap_9\Chap_9
\ 17-Aug-2022 Amit Proof-4 Reader’s Sign _______________________ Date __________

4 x3 + c dy  1  1

y × (1 + x2) = 
+ 1 −  y =
3 dx  x  x

y=
4 x3 + c(1 + x 2) −1
3 (1 + x 2) Here ( )
P = 1− 1
x
Hence, the required solution is
3 \ I.F. = e ∫

Pdx
=e
( 1 )dx = e(x −log x)
∫ 1− x
4 x + c(1 + x 2) −1 .
y=
3 (1 + x 2) log 1
= e x ⋅ e − log x = e x ⋅ e x = ex ⋅ 1
dy x
Example 5. General solution of + y = sin x is ___________ .
dx x 1
Hence, the required I.F. = e ⋅ .
dy x
Solution. The given differential equation is + y = sin x dx
dx Example 7. Solution of the differential equation of the type
Since, it it a linear differential equation dy
+ P1x = Q1 is given by x . I.F. = ∫ (I.F.)Q1dy .
\ P = 1 and Q = sin x
Integrating factor I.F. = e ∫
Pdx
= e∫
1. dx
= ex Solution. True
dy x + 2y
Example 8. The solution of differential equation = is
\ Solution is y × I.F. = ∫ Q × I.F. dx + c
dx x
x + y = kx2.

y . ex = ∫ sin x . e xdx + c ...(1) Solution. True
The given differential equation is
Let I = ∫ sin x . e x dx
I II dy x + 2y dy y
=  =1+2
(
   I = sin x . ∫ e x dx − ∫ D(sin x) . ∫ e x dx dx ) dx x dx x
dy 2y

− =1
   I = sin x . e x − ∫ cos x . e x dx dx x
I II
−2
Here, P= and Q = 1
(
I = sin x . e x − cos x . ∫ e x dx − ∫ D (cos x)∫ e x dx dx 
) x
Integrating factor I.F.
I = sin x . e − cos x . e − ∫ − sin x . e dx  −2 dx
x x x

= 12
−2
= e x = e −2 log x = elog x
x
I = sin x . e − cos x . e − ∫ sin x . e dx
x x x

\ Solution is y × I.F. = ∫ Q × I.F. dx + c

I = sin x . ex – cos x . ex – I
 I + I = ex (sin x – cos x) 1 = 1 × 1 dx + c
x2 ∫
   y ×

x2

2I = ex (sin x – cos x)
y y
= 1 dx + c ⇒ 2 = − 1 + c
x2 ∫ x2
e x (sin x − cos x)    


\ I= x x
2
     y = – x + cx2   y + x = cx2
From eq. (1) we get
Example 9. Integrating factor of the differential equation
e x (sin x − cos x) + c
y . ex = dy
2       cos x . + y sin x = 1 is
dx
y= ( sin x −2 cos x ) + c . e –x (a) cos x (b) tan x (c) sec x (d) sin x
Solution. The given differential equation is
Hence, the required solution is dy
cos x . + y sin x = 1
y= (
sin x − cos x
2
+ c . e–x ) dy sin x
dx
dy

+ y= 1  + tan x y = sec x
dy 1+ y dx cos x cos x dx
Example 6. The integrating factor of +y= is
dx x Here, P = tan x and Q = sec x
___________ .
\ Integrating factor = e ∫ = e∫
Pdx tan xdx
= elogsec x = sec x
Solution. The given differential equation is
1+ y Hence, the correct option is (c).
dy
+y= dy
dx x Example 10. Integrating factor of x – y = x4 –3x is
dx
dy y
+ y− = 1 (a) x (b) log x (c)
1
(d) – x
dx x x x

298 Mathematics–12
E:\AMIT_WORKS\Exam_Guru\EG_Mathematics-12_(working_02-06-2022)\EG_Mathematics-12_working\Open_Files\Chap_9\Chap_9
\ 17-Aug-2022 Amit Proof-4 Reader’s Sign _______________________ Date __________

Solution. The given differential equation is dy



+ y tan x = sec x
dy dy y dx
x – y = x4 –3x  − = x3 – 3
dx dx x Here, P = tan x and Q = sec x
1
\ I.F. = e ∫ = e∫
Pdx tan x dx
Here, P = – and Q = x3 – 3 = elogsec x = sec x
x
So, integrating factor Hence, the correct option is (b).
1 1
Example 14. The integrating factor of the differential equation
∫ − dx
= e x = e − log x = e x = 1
log
= e∫
Pdx
dy 1+ y
x +y= is:
dx x
Hence, the correct option is (c).
Example 11. Integrating factor of the differential equation x ex
(a) x    (b)    (c) xex   (d) ex
e x
(1 – x2) dy – xy = 1 is Solution. The given differential equation is
dx
x dy 1+ y dy 1+ y
(a) –x (b) (c) 1 − x 2 (d) 1 log(1 – x2)
dx
+y =
x

dx
=
x
−y
1 + x2 2
Solution. The given differential equation is dy 1 (1 − x)

= +y
dy dx x x
(1 – x2) dy – xy = 1  − x .y= 1 2
( )
dx dx 1 − x 2 1− x dy 1− x 1

− y =
1 dx x x
x
Here, P = – 2 and Q =
Here, P = − (
x )
1− x 1 − x2 1− x 1
and Q =

\ Integrating factor x

∫ Pdx = e
−x
∫ 1 − x2 dx 1 log (1 − x 2)
2
\ Integrating factor I.F. = e∫
Pdx
=e

x −1
x
dx
=e
( 1 )dx
∫ 1− x
I.F. = e = e2 = 1− x
Hence, the correct option is (c). = e(x − log x) = e x. e − log x
Example 12. The solution of the differential equation log 1
= e x. e x = e x. 1
dy x
+ y = e −x , y(0) = 0 is
dx Hence, the correct option is (b).
(a) y = ex (x – 1) (b) y = xe–x Example 15. The solution of x
dy
+ y = e x is:
–x
(c) y = x e + 1 (d) y = (x + 1) e–x dx
x
Solution. The given differential equation is (a) y = e + k (b) y = xex + cx
dy x x
+ y = e −x y
dx (c) y = x . ex + k (d) x = e + k
y y
Since, it is a linear differential equation
\ P = 1 and Q = e–x dy
Solution. The given differential equation is x + y = ex
\ I.F = e ∫
1. dx dx
= ex
dy y ex
So, the solution is 
+ =
dx x x
−x
y × I.F. = ∫ Q . I.F. dx + c  y . ex = ∫e . e xdx + c
1 ex
Here P = and Q =
 y . ex = ∫ 1. dx + c  y . ex = x + c
x x
1
∫ x dx
Put x = 0, y = 0, we have 0 = 0 + c \ c = 0 \ Integrating factor I.F. = e
= elog | x| = x
So, the solution is y ex = x  y = x . e–x So, the solution is
Hence, the correct option is (b). y × I.F. = ∫ Q × I.F. dx + k
Example 13. Integrating factor of the differential equation
e x × x dx + k
dy
+ y tan x − sec x = 0 is

y×x= ∫x
dx

y × x = ∫ e x dx + k
(a) cos x   (b) sec x   (c) e cos x   (d) e sec x
Solution. Given differential equation is 
y × x = ex + k
dy ex + k
+ y tan x − sec x = 0
\ y=
dx x x
Hence, the correct option is (a).

Differential Equations 299


E:\AMIT_WORKS\Exam_Guru\EG_Mathematics-12_(working_02-06-2022)\EG_Mathematics-12_working\Open_Files\Chap_9\Chap_9
\ 17-Aug-2022 Amit Proof-4 Reader’s Sign _______________________ Date __________

dy y
∫ ( tan x sec )
x e tan x dx
2
Example 16. Solution of differential equation + = sinx is: ⇒ yetan x =
dx x
(a) x (y + cos x) = sin x + c ⇒ yetan x = ∫ ( tan x ) e
tan x
(sec 2
x dx )
(b) x (y – cos x) = sin x + c 2
On putting tan x = z and sec x dx = dz on RHS,
(c) xy cos x = sin x + c
z
(d) x (y + cos x) = cos x + c We get: yetan x = ∫ z e dz
dy y On integrating by parts, we get:
Solution. The given differential equation is + = sinx
dx x
yetan x = ze − ∫ e dz = ze − e + C
z z z z

Since, it is a linear differential equation


= (tan x)etan x – etan x + C
1
\P= and Q = sin x −1 x
x y = ( tan x − 1) + Ce
tan

1
∫ x dx  dy  + y 2 log x
Integrating factor I.F. = e = elog x = x Example 18. Solve: ( x log x ) =
 dx  x
\ Solution is y × I.F. = ∫ Q × I.F. dx + c
Solution. The given differential equation:
y × x = ∫ sin x . x dx + c  y × x = ∫ x sin x dx + c
 dy  + y 2 log x
( x log x ) =
 dx  x
(
 yx = x . ∫ sin x dx − ∫ D(x)∫ sin x dx dx + c
) dy  1 
⇒ + y = 22
 yx = x(− cos x) − ∫ − cos x dx
dx  x log x  x
This is a linear equation of the form:
 yx = − x cos x + ∫ cos x dx

dy
 yx = – x cos x + sin x + c + f ( x ) y = g(x)
dx
 yx + x cos x = sin x + c 1
∫ x log x dx
 x (y + cos x) = sin x + c \ I.F. = e = elog (log x) = log x
Hence, the correct option is (a).  [Q elog f (x) = f (x)]
 dy 
Example 17. Solve the differential equation: cos 2 x Now the solution of the differential equation will be:
 dx 
+ y = tan x.[Delhi 2008 (C)] y(I.F.) = ∫ g(x) (I.F.) dx
Solution. The given differential equation: 2 log x dx
⇒ y log x = ∫
x2
= ∫ ( log x ) 2x dx
−2
( )
 dy  Now on integrating RHS by parts, we get:
cos 2 x + y = tan x
 dx 
 2x −1  1  2x −1 
y log x = log x   −∫  dx
x  −1 

dy
dx ( )
+ sec 2 x y = sec2 x tan x  −1 
−2 log x
It is a linear differential equation of the type: ⇒ y log x = + ∫ 2x −2 dx
x
dy sec2 x dx
+ f ( x ) y = g(x) \ I.F. = e∫ = e tan x −2 log x 2
dx ⇒ y log x = − +C
x x
So, the solution of the given differential equation:
−2
⇒ y log x = (1 + log x ) + C
y(I.F.) = ∫ g(x) (I.F.) dx x

EXERCISE 9.6
I. Multiple Choice Questions
∫ = ∫  Q1 e ∫  dy + C
P1 dy P1dy
1. The integrating factor of the differential equation (a) y e 
 
dy
x − y = 2x 2 is[NCERT]
(b) y e ∫ = ∫  Q1 e ∫ 
P1 dx P1 dx
dx  dx + C
1  
(a) e–x   (b) e–y   (c)    (d) x
x (c) x e ∫ = ∫  Q1 e ∫ 1  dy + C
P1 dy P dy

2. The general solution of a differential equation of the type  


dx + P x = Q (d) xe ∫ P1 dx = ∫  Q1 e ∫ P1 dx  dy + C
1 is
dy 1  

300 Mathematics–12
E:\AMIT_WORKS\Exam_Guru\EG_Mathematics-12_(working_02-06-2022)\EG_Mathematics-12_working\Open_Files\Chap_9\Chap_9
\ 17-Aug-2022 Amit Proof-4 Reader’s Sign _______________________ Date __________

3. If sin x is an integrating factor of the differential equation dy


dy 16. − y tan x = 2 sin x [A.I. 2004 (C)]
+ Py = Q then P can be dx
dx
(a) log sin x   (b) cot x   (c) sin x   (d) log cos x ( ) dy
17. 1 + x 2
dx
− 2xy = ( x + 2) ( x + 1) [Delhi 2005]
2 2

II. Short Answer Type Questions-I


Find the general solution of the following differential
18. ( x + 1) dy
2
dx
2
+ 2xy = x + 4 [A.I. 2008]
equations:
dy
dy 19. + y = ( cos x − sin x ) [Delhi 2009]
1. + y cot x = 2x + x 2 cot x, (x ≠ 0) dx
dx
2. ex dy + (yex + 2x) dx = 0  dy  − y = sin x
20. sec x [A.I. 2009 (C)]
 dx 
dy
3. + (sec x ) y = tan x [Delhi 2008 (C)] dy
dx 21. + y tan x = 2x + x 2 tan x

( )
dx
(
4. cos 2 x
 dx )
 dy  + y = tan x. 0 ≤ x < π
2 dy y
+ = cos x +
sin x
22.
2 dx x x
5. (1 + x )dy + 2xy dx = cot x dx (x ≠ 0).
dy y 1
23. + =
dy dx x log x x
6. x   + y − x + xy cot x = 0 ( x ≠ 0) [Delhi 2011]
 dx 
dy
Find the particular solution of the following differential 24. x cos x   + y ( x sin x + cos x ) = 1
 dx 
equations:
7.
dy
dx
π
+ 2y tan x = sin x and y = 0 when x = [A.I. 2008]
3
(
25. x x 2 + 1 ) dy
dx
= y (1 − x ) + x
2 3
log x

(
8. 1 + x 2
dy
dx
)
+ 2xy =
1 and given that:
1 + x2
III. Long Answer Type Question
dy
1. Solve D.E.: x   + y − x + xy cot x = 0. [Delhi 2015]
y = 0 when x = 1.  dx 

dy π dy
− 3y cot x = sin 2x and given that y = 2 when x = .
9. 2. Solve the differential equation: x   + y = x cos x + sin x
dx 2  dx 
[Delhi 2011 (C)]
given that y = 1 when x = π . [Delhi 2017]
Solve the following differential equations: 2
dy
10. ( x log x ) + y =
2 log x
[Delhi 2009, 2010] 3. Solve the D.E. [(tan–1 x) – y]dx = (1 + x2)dy.[A.I. 2017]
dx x 4. ydx – (x + 2y2) dy = 0
dy π
11.
dx
+ y cot x = 4x cosec x given that: y = 0 when x = .
2 5. ( x + y ) dy
dx
=1
[A.I. 2010]
6. y dx + (x – y2) dy = 0.
(
12. x 2 − 1
dy
dx )
+ 2xy = 2
2 .[Delhi 2000 (C), A.I. 2010]
x −1 7. (x + 3y 2) dy = y
dx
(
13. 1 + x 2 ) dy
dx
+ y = tan −1
x .[A.I. 2017] 8. (tan y – x) dy = (1 + y2) dx[A.I. 2012]
–1

9. y(sin 2x)dx – (1 + y2 + cos2 x) dy = 0. [Delhi 2011 (C)]


14. ( x + 1) dy
dx
+ 1 = e x − y, x ≠ −1 [Delhi 2007] 10. yey dx = (y3 + 2xey) dy and given that y(0) = 1.
11. (y3 – x) dy = y dx.
dy y
15. − = 2x 2 [Delhi 2003] 12. (tan–1 y – x) dy = (1 + y2)dx.[Delhi 2015, A.I. 2015]
dx x

Answers 9.6
dy y Hence, (c) is the correct answer.
I. 1. Given equation can be written as − =2x
dx x
2. I.F. = e ∫
P1 dy

1
which is of the form dy + Py = Q where P = − , Q = 2x The general solution is given by x (I.F.) = ∫ Q.(I.F.) dy + C
dx x
⇒ x ⋅ e∫ = ( Q e ∫ 1 ) dy + C
P1 dy P dy
1
∫ − dx ∫ 1
I.F. = e ∫ = log x −1
=1
P dx − log x
e x =e =e
x Hence, (c) is the correct answer.

Differential Equations 301


E:\AMIT_WORKS\Exam_Guru\EG_Mathematics-12_(working_02-06-2022)\EG_Mathematics-12_working\Open_Files\Chap_9\Chap_9
\ 17-Aug-2022 Amit Proof-4 Reader’s Sign _______________________ Date __________

3. Since sin x is the I.F. of the differential equation


dy  2x  cot x
dy ⇒ + y= (L.D.E.)
+ Py = Q, dx  1 + x 2  1 + x2
dx
2x dx
∫1 2
I.F. = e ∫ + x2
P dx
= sin x \ I.F. = e = elog (1+ x ) = (1 + x 2)

⇒ ∫ Pdx = log sin x ⇒ P = d (log sin x) 6. y = 1 − cot x + C


dx x x sin x
P= 1 × d (sin x) ⇒ P = 1 × cos x = cot x
sin x dx sin x dy
Hint: x
+ y − x + xy cot x = 0
dx
Hence, (b) is the correct answer.

II. 1. y = x 2 +
C dx x (
⇒ dy + 1 + cot x y = 1 (L.D.E.) )
sin x
I.F. = e
∫ ( 1x + cot x) dx
dy
Hint:
+ cot x ⋅ y = 2x + x2 cot x (L.D.E.) \ I.F. = elog (x sin x) = x sin x
dx
7. y = cos x − 2 cos 2 x
I.F. = e∫
cot x dx
= elog sin x = sin x

dy
\ y sin x = ∫ (2x sin x + x 2 cos x) dx Hint:
+ (2 tan x) y = sin x (L.D.E.)
dx
2x
2. ye x + x 2 + C \ I.F. = e∫
2 tan x dx
= e2 log sec x = elog sec = sec 2 x
x x
Hint: e dy + (ye + 2x) dx = 0

8. ( )
y 1 + x 2 = tan −1 x − π
4
dy
⇒ + y = −2xx ( L.D.E.)
dx e

dx (
Hint: 1 + x 2 dy + 2xy = 1 2
1+ x
)
I.F. = e∫
1⋅dx
= ex

⇒ dy +  2x 2  y = 1 (L.D.E.)
∴ Solution of the (L.D.E.) y e x = ∫ −2xx × e x dx
e
dx  1 + x 
(1 + x )
2 2

3. y (sec x + tan x ) = sec x + tan x − x + C 2x


∫ 1 + x2 dx (1 + x 2)
\ I.F. = e = elog = (1 + x 2)
dy
Hint:
+ (sec x) y = tan x (L.D.E.)
dx 9. y = 4 sin 3 x − 2 sin 2 x

∴ I.F. = e∫
sec x dx
⇒ I.F. = elog (sec x + tan x) dy
Hint:
− 3 cot x ⋅ y = sin 2x (L.D.E.)
= (sec x + tan x) dx
Solution is : y(sec x + tan x) −3
− 3 cot x dx = e − 3 log sin x = elog sin x 1
I.F. = e∫ =
= tan x (sec x + tan x ) dx
∫ sin 3 x

4. y = ( tan x − 1) + Ce − tan x 10. y log x = −2 (1 + log x ) + C


x
 dy  + y = tan x  dy  + y = 2 log x
Hint: cos 2 x
Hint: x log x

 dx   dx  x


dy
dx
( )
+ sec2 x y = tan x sec2 x (L.D.E.) ⇒
dy  1 
+ y = 22 (L.D.E.)
dx  x log x  x
sec2 x dx
\ I.F. = e∫ = e tan x 1
∫ x log x dx
I.F. = e = elog log x = log x
Solution of L.D.E:
ye tan x = ∫ (tan x sec2 x)e tanx dx Solution of L.D.E: y ⋅ log x = ∫ 22 log x dx
x
5. ( )
y 1 + x 2 = log | sin x | + C
π2
2 11. y sin x = 2x 2 −
Hint: (1 + x )dy + 2xy dx = cot x dx
2

302 Mathematics–12
E:\AMIT_WORKS\Exam_Guru\EG_Mathematics-12_(working_02-06-2022)\EG_Mathematics-12_working\Open_Files\Chap_9\Chap_9
\ 17-Aug-2022 Amit Proof-4 Reader’s Sign _______________________ Date __________

dy − ∫ 1 dx
=1
−1
Hint:
+ (cot x) y = 4x cosec x (L.D.E.) \ I.F. = e x = e − log x = elog x
dx x
\ I.F. = e∫
cot x dx
= elog sin x = sin x 16. 2 − cos 2x
y=
cos x 2 cos x
y= 1 log x − 1 + C
12. dy
(x 2 − 1) x +1 Hint:
+ (− tan x) y = 2 sin x (L.D.E.)
dx
Hint: (x 2 − 1) dy + 2xy = 22
\ I.F. = e − ∫ tan x dx = e − log sec x = 1 =
cos x
dx x −1 sec x

dy  2x  y
⇒ + y = 2 2 2 (L.D.E.) 17. = x + tan −1 x + C
dx  x 2 − 1 (x − 1) 1 + x2
2x dy
∫ x2− 1 dx 2 − 1) Hint: (1 + x 2)
+ (− 2x) y = (x 2 + 2) (x 2 + 1)
\ I.F. = e = elog(x = (x 2 − 1) dx

Solution of the (L.D.E.): dy  − 2x 


+ y = x 2 + 2 (L.D.E.)
dx  1 + x 2 
(
y x2 − 1 = ∫ 2
x
2 dx
−1
)
− 2x
∫ 1 + x2 dx 2 1
− tan − 1 x
\ I.F. = e = e − log (1 + x ) =
13. −1
y = (tan x − 1) + Ce , 1 + x2

Hint: (1 + x 2)
dy
+ y = tan −1 x 18. y(x 2 + 1) = 2 log x + x 2+ 4 + x x 2+ 4 + C
dx 2

dy  1  tan −1 x (L.D.E.) dy
Hint: (x 2 + 1)
+ 2xy = x 2 + 4
⇒ +  y= dx
dx  1 + x 2  1 + x2
1
∫ 1 + x2 dx dy  2x  x2 + 4
tan −1 x ⇒ + 2  y= 2 (L.D.E.)
\ I.F. = e =e dx  x + 1 x +1
Solution of the (L.D.E.): 2x
∫ x2 + 1dx 2 + 1)
tan −1 x tan −1 x  dx  \ I.F. = e = elog (x = (x 2 + 1)
ye = ∫ tan −1 x e  
1 + x2 
19. y = cos x + Ce −x
y x
14. e (x + 1) = e + C
dy
Hint: + y = cos x − sin x (L.D.E.)
dy dx
Hint: (x + 1) + 1 = ex − y
dx
\ I.F. = e∫
dx
= ex
dy
⇒ (x + 1) + 1 = e x × e− y
dx ye x = ∫ (cos x − sin x) e x dx
x
dy
⇒ e y + 1 e y = e Put e y = z 20. y = Cesin x − (1 + sin x)
dx x + 1 x +1

⇒ ey
dy dz
=
dx dx
\
dx x +1( )
dz + 1 z = e x (L.D.E.)
x +1
Hint: (sec x)

dy
dx
− y = sin x

1
∫ x + 1 dx dy
I.F. = e = elog (x + 1) = x + 1. ⇒ + (− cos x) y = sin x cos x (L.D.E.)
dx
Solution of (L.D.E.):
\ I.F. = e ∫
− cos x dx
= e − sin x
x
z (x + 1) = ∫ e × (x + 1) dx = ∫ e x dx = e x y e − sin x = ∫ sin x e − sin x (cos x dx)
x +1

15. y = x 3 + Cx 21. y = x 2 + C cos x

dy  − 1 dy
Hint:
+   y = 2x 2(L.D.E.) Hint:
+ y tan x = 2x + x 2 tan x (L.D.E.)
dx  x  dx

Differential Equations 303


E:\AMIT_WORKS\Exam_Guru\EG_Mathematics-12_(working_02-06-2022)\EG_Mathematics-12_working\Open_Files\Chap_9\Chap_9
\ 17-Aug-2022 Amit Proof-4 Reader’s Sign _______________________ Date __________

I.F. = e∫
tan x dx  2  x 2 log x x 2 + 1

= elog sec x = sec x y  x + 1 = ∫ 2 × dx = ∫ x log x dx
 x  x +1 x

22. y = sin x + C III. 1.xy sin x = sin x – x cos x + C
x
dy y
Hint:
+ − 1 + y cot x = 0
dy y sin x dx x
Hint:
+ = cos x + (L.D.E.)

1
dx x x
(
⇒ dy + 1 + cot x y = 1
dx x )
\ I.F. = ∫ dx
e x = elog x = x It is L.D.E.

Now solution: yx = ∫ x cos x dx + ∫ sin x dx + C


∫ + cot x
I.F. = e x
(1 ) dx = elog x + log sin x = elog( x sin x)

= x sin x
yx = x sin x − ∫ sin x dx + ∫ sin x dx + C
yx sin x = ∫ x sin x dx , Now evaluate.
log x
23. y= + C 2. y = sin x
2 log x
dy
Hint:
x   + y = x cos x + sin x
Hint: dy + y 1
= (L.D.E.)  dx 
dx x log x x

\ I.F. = e
1
∫ x log x dx
=e log (log x)
= log x


dy 1
dx
+
x()
y = cos x + sin x
x
It is L.D. Equation.
log x (log x) 2 I.F. = x.
y log x = ∫ dx = +C
x 2
( )
−1 − tan −1 x
3. y = tan x − 1 + ce
tan x + C
24. y=
x sec x
4. x = 2y 2 + Cy ;
dy
Hint: x cos x   + y(x sin x + cos x) = 1
Hint: y dx – (x + 2y2)dy = 0

 dx 
x + 2y 2 x
dx (
⇒ dy + tan x + 1 y = 1 (L.D.E.)
x x cos x ) ⇒ dx =
dy y
= + 2y
y

dx −  1  x = 2y (L.D.E.)
⇒ I.F. = e
∫ (tan x+ 1x ) dx = x sec x, ⇒
dy  y 

Now solution: − ∫ 1 dy
\ I.F. = e y
= e − log y = 1
y (x sec x) = ∫ 1 × x sec x dx y
x cos x
x 1
\ Solution: = ∫ 2y × dy + C
y y

= sec2 x dx
dy
 x 2 + 1 x 2 2 5. x = Ce y − (y + 1) , Hint : (x + y) =1
25. y  = log x − x + C dx
 x  2 4
dx
+ (− 1) x = y. (L.D.E.) ∴ I.F. = e ∫
− dy

dy
Hint:

dy
+
(
x 2 − 1 y x 2 log x
= 2
)
dx x x 2 + 1 ( x +1 ) ⇒ I.F. = e −y. Hence solution: xe −y = ∫ y e −y dy

It is L.D.E.
y2 C
6. x= + . Hint : y dx + (x − y 2) dy = 0
x 2 −1 3 y
∫ dx
( )
 2x 1
x x 2 +1 ∫  x2 +1 − x  dx
I.F. = e =e
dx +  1  x = y (L.D.E.)

 2  dy  y 
=e
( )
log x 2 +1 − log x
=e
log  x +1
 x 
2
= x +1 1
x ∫ y dy
\ I.F. = e = elog y = y

304 Mathematics–12
E:\AMIT_WORKS\Exam_Guru\EG_Mathematics-12_(working_02-06-2022)\EG_Mathematics-12_working\Open_Files\Chap_9\Chap_9
\ 17-Aug-2022 Amit Proof-4 Reader’s Sign _______________________ Date __________

2 dy − 2∫ 1 dy
7. x = 3y + Cy ; Hint: (x + 3y 2) =y \ I.F. = e y
= 12
dx
y
⇒ dx −  1  x = 3y (L.D.E.) Now solution: x2 = ∫ y 2e −y × 12 dy = ∫ e −y dy
dy  y  y y
− ∫ 1 dy 3
=1
−1
11. x = y + C Hint: (y − x) dy = y dx
3
\ I.F. = e y
⇒ I.F. = e − log y = elog y
y 4 y
−1 − tan −1 y
8. x = (tan y − 1) + C e dx +  1  x = y 2(L.D.E.)

dy  y 
Hint: (tan–1 y – x)dy = (1 + y2)dx

1
∫ y dy
   tan −1 y  \ I.F. = e = y. Now solution xy = ∫ y 3 dy.
⇒ dx +  21  x =  2 (L.D.E.)
dy  y + 1  y + 1  −1 y
12. x = tan −1 y − 1 + Ce − tan
1
∫ 1 + y 2 dy −1 y
\ I.F. = e = e tan Now solution: Hint: (tan–1 y – x)dy = (1 + y2)dx

x e tan
−1 y
= ∫ (tan −1 y) e tan
−1 y  dy 
 y 2 + 1 (2 dx
)−1
⇒ 1 + y dy = tan y − x

⇒ dx  1  tan −1 y
2 y2 C +  x=
9. − cos x = 1 + + 2
dy  1 + y  1 + y2
3 y
This is L.D. Equation of the type
Hint: (y sin 2x)dx – (1 + y2 + cos2 x)dy = 0

dx
⇒ (sin 2x) dx + 1 (− cos 2 x) = 1 + y
2
+ f ( y ) x = g ( y )
dy
dy y y
1
dx = dz ∫ 1+ y 2 dy −1 y
2
Put − cos x = z ⇒ sin 2x I.F. = e = e tan
dy dy
\ Solution of the D.E.
2
dz  1  z y + 1 (L.D.E.)  tan −1 y  tan −1 y
\ +  = −1 y
dy  y  y xe tan = ∫ e dy
 1 + y 2 
1
∫ y dy Put tan–1 y = t
\ I.F. = e =y
⇒ dy
\ Solution : zy = ∫ (1 + y ) dy + C
2 = dt
1 + y2
−1 y
10. x = Cy 2 − y 2 e −y \ xe
tan
= ∫ tet dt

Hint: yey dx = (y3 + 2x ey) dy



( tan y ) − e
−1 y −1 y −1 tan −1 y
xe tan = e tan +C
dx  − 2  x y 2 e −y
⇒ + = (L.D.E.)
dy  y  x = tan −1 y − 1 + Ce − tan
−1 y

Case Based Questions


1. A veterinary doctor was examining a sick cat brought by using Newton law of cooling which is governed by the
a pet lover. When it was brought to the hospital, it was dT
differential equation: α (T–70), where 70°F is the
already dead. The pet lover wanted to find its time of death. dt
He took the temperature of the cat at 11:30 pm which was room temperature and T is the temperature of the object
at time t.
94.6°F. He took the temperature again after one hour; the
Substituting the two different observations of T and
temperature was lower than the first observation. It was
t made, in the solution of the differential equation
93.4°F. The room in which the cat was put is always at dT
70°F. The normal temperature of the cat is taken as 98.6°F = k (T–70)where k is a constant of proportion, time
dt
when it was alive. The doctor estimated the time of death of death is calculated.

Differential Equations 305


E:\AMIT_WORKS\Exam_Guru\EG_Mathematics-12_(working_02-06-2022)\EG_Mathematics-12_working\Open_Files\Chap_9\Chap_9
\ 17-Aug-2022 Amit Proof-4 Reader’s Sign _______________________ Date __________

(i) The degree of the above given differential equation is denotes the number of weeks and y the number of children
(a) 1 (b) 2 who have been given the drops.

(c) 3 (d) 0 (i) The order of the above given differential equation is
(ii) Which method of solving a differential equation helped (a) 1 (b) 2
in calculation of the time of death? (c) 3 (d) 0
(a) Variable separable method (ii) Which method of solving a differential equation can be
dy
(b) Solving Homogeneous differential equation used to solve = k (50 – y)?
dx
(c) Solving Linear differential equation (a) Variable separable method
(d) All of these (b) Solving Homogeneous differential equation
(iii) If the temperature was measured 2 hours after 11.30 (c) Solving Linear differential equation
pm, will the time of death change?
(d) All of these
(a) Yes (b) No dy
(iii) The solution of the differential equation = k (50 – y)
(c) May be (d) Cannot say is given by dx
(iv) The solution of the differential equation
(a) log |50 – y| = kx + C
dT
= k (T–70) is given by (b) –log |50 – y| = kx + C
dt
(a) log |T – 70| = kt + C (c) log |50 – y| = log |kx| + C
(b) log |T – 70| = log k + C (d) 50 – y = kx + C
(c) T – 70 = kt + C (iv) The value of c in the particular solution given that
(d) T – 80 = kt + C y(0) = 0 and k = 0.049 is
1
(v) If t = 0 when T is 72, then the value of C is (a) log 50 (b) log
50
(a) –2 (b) 0 (c) 50 (d) –50
(c) 2 (d) log 2 (v) Which of the following solutions may be used to find
Ans. (i) (a) 1 (ii) (a) Variable separable method the number of children who have been given the polio
drops?
(iii) (b) No (iv) (a) log |T – 70| = kt + C
(a) y = 50 – ekx (b) y = 50 – e–kx
(v) (d) log 2
(c) y = 50 (1– e–kx) (d) y = 50 (e–kx – 1)
2. Polio drops are delivered to 50K children in a district. The
Ans. (i) (a) 1
rats at which polio drops are given is directly proportional
to the number of children who have not been administered (ii) (a) Variable separable method
the drops. By the end of 2nd week, half the children have (iii) (b) –log |50 – y| = kx + C
been given the polio drops. How many will have been given
the drops by the end of 3rd week can be estimated using the (iv) (b) log
1
dy 50
solution to the differential equation = k (50 – y) where x
dx (v) (c) y = 50 (1– e–kx) qqq

Author’s Comments
Questions based on following types are very important for Exams. So, students are advised to revise them thoroughly.
1. To find order and degree of the given differential equation.
2. To solve three types of differential equation: (i) Variable separables, (ii) Homogeneous and (iii) Linear differential equations.

306 Mathematics–12
E:\AMIT_WORKS\Exam_Guru\EG_Mathematics-12_(working_02-06-2022)\EG_Mathematics-12_working\Open_Files\Chap_9\Chap_9
\ 17-Aug-2022 Amit Proof-4 Reader’s Sign _______________________ Date __________

IMPORTANT FORMULAE
1. (i) If the homogeneous differential equation is of the type:
dy  y
= F ( x, y ) = φ  ...(1)
dx  x

dy dv
Then substituting y = vx and =v+x in (1)
dx dx
(ii) If the homogeneous differential equation in the form:
dy  
= F ( x, y ) = φ  x 
dx  y

then substitute: x = vy and dx = v + y dv


dy dy

dy
2. If the given equation in the form: + Py = Q where P and Q are constants or function of x only.
dx

(i) Then find the integrating factor (I.F.) = e∫


P dx

(ii) Write the solution of the given differential equation as: y ( I.F.) = ∫ Q ( I.F.) dx + C

dx
3. If the linear differential equation is of the form: + Px = Q where P and Q are either constants or function of y only.
dy
∫ P dy
Then, I.F. = e and solution of the differential equation will be:

x(I.F.) = ∫ Q ( I.F.) dy + C

COMMON ERRORS
ERRORS CORRECTIONS
(i) When the solution of an equation contains many terms including (i) It is mandatory to put the constant of integration.
“log”, students do not take the constant of integration as log C,
and rather take it as C.

(ii) While solving linear first order equation, if I.F. = e–log x. It is


=1
−1
generally taken as –x. (ii) I.F. = e − log x = elog x
x
(iii) Due to lack of practice students are unable to recognise the (iii) This can be avoided by solving more and more questions.
type of differential equations they are solving, so they apply
wrong method to solve the given differential equation.

(iv) Students are not able to integrate the function. (iv) This also requires more and more practice of integration of
different types of questions.

Differential Equations 307


E:\AMIT_WORKS\Exam_Guru\EG_Mathematics-12_(working_02-06-2022)\EG_Mathematics-12_working\Open_Files\Chap_9\Chap_9
\ 17-Aug-2022 Amit Proof-4 Reader’s Sign _______________________ Date __________

REVISION CHART

Differential Equations
An equation can be changed into differential equation

By differentiating only once, if the equation By differentiating the equation twice if it


has only one constant (then eliminating has two constants (then eliminating two
constant). constants).

General Solution of Differential Equations

Variable separable differential equation.


dy y
dy
= f ( x) g ( y) A homogeneous differential equation of the type = φ   is
dx dx x
 
dx = φ  x 
It is solved by separating variable as solved by putting y = vx or  y  is solved by putting x = vy.
dy  
dy
∫ g ( y ) = ∫ f ( x ) dx

Linear Differential Equations

A linear equation of the type


dy
+ f ( x) y = g ( x)
dx

is solved by calculating I.F.


f ( x ) dx
I.F. = e∫ = φ ( x )( Assume it )
Then solution of this type of linear differential equation is
y ( I.F.) = ∫ g ( x )( I.F.) dx

308 Mathematics–12
E:\AMIT_WORKS\Exam_Guru\EG_Mathematics-12_(working_02-06-2022)\EG_Mathematics-12_working\Open_Files\Chap_10\Chap_10
\ 17-Aug-2022 Amit Proof-3 Reader’s Sign _______________________ Date __________

Topics Covered
10 Vector Algebra

10.1 Types of Vectors 10.2 Section Formula for Position Vectors


10.3 Dot Product of Two Vectors 10.4 Angle between Two Vectors
10.5 Vector Product or Cross Product

C hapter map
VECTOR ALGEBRA

Types of Vectors and Properties

Zero Unit Parallel Equal Negative of Addition of


Vector Vector Vectors Vectors a Vector Vectors

Multiplication of a Unit Vectors along Section


Vector by a Scalar the Co-ordinate axes Formula

Vector joining two Unit Vectors in the


points A and B direction of
→ →
 → → → a+ b
 AB = OB − OA 

Dot Product of Two Vectors

Projection of Perpendicular Finding the angle Squaring of a


vectors on a line Vectors between two vectors Vector

Cross Product of Vectors

Cross product of Unit vector perpendicular


unit vectors to two given vectors

Introduction
Scalar Quantity: A physical quantity which has magnitude only is called scalar quantity. Length, breadth, area, volume, height etc.
are scalar quantities. Scalar quantities are added, subtracted and multiplied algebraically.
Vector Quantity: A physical quantity which has magnitude and direction is called vector quantity. Force, velocity, acceleration,
displacement etc. are examples of vector quantities.
Vector quantities cannot be added, subtracted and multiplied algebraically, we need different methods to find sum, difference
and product of vector quantities.
Representation of a Vector: A line segment AB with an arrow represents a vector.
→ →
It is denoted as AB or p .

309
E:\AMIT_WORKS\Exam_Guru\EG_Mathematics-12_(working_02-06-2022)\EG_Mathematics-12_working\Open_Files\Chap_10\Chap_10
\ 17-Aug-2022 Amit Proof-3 Reader’s Sign _______________________ Date __________

→
Here point A is called initial point and point B is called terminal point of the vector AB . The length of the vector AB is proportional
→
to the magnitude of vector AB .

Topic 1. Types of Vectors


→ Vectors in Order: If two or more vectors have same flow, they
Unit Vector of a Given Vector: Unit vector of vector a is a → → →
vector whose magnitude is one unit and is in the direction of vector are called vectors in order. Vectors AB, BC and CD have same
→ → ∧ flow, therefore these vectors are in order.
a . A unit vector of vector a is represented by a . Actually unit Vectors in Opposite Order: Two vectors not in same flow are
∧ →
→ →
vector a represents the direction of the vector a . called vectors in opposite order. Vector AB and CB are in
→  opposite order because they are not in the same flow.
Magnitude of a vector a is represented by a or a . Relation
→ → ∧
between vectors a , a and a is

→ → ∧ ∧
a
a = a a or a = →
a
Zero Vector: A vector whose initial and terminal points
→ → →
coincides is called zero vector. Vector AA , BB and OO etc.
are zero vectors.
Vectors in same order Vectors in opposite order
Coinitial Vectors: Two or more vectors having a same initial
→ → →
point are called coinitial vectors. Vectors OA , OB and OC are Addition of Vectors
coinitial vectors because they have same initial point O. Vectors are added with the help of triangle law of addition.
Triangle Law of Addition: If any two sides of a triangle represent
two vectors in magnitude and direction taken in order, then the
third side of the triangle taken in opposite order, represents the
magnitude and direction of the sum of the two vectors.

→ → → →
Opposite Vectors: Vectors AB and − AB or AB and BA
are pairs of opposite vectors.
Collinear Vectors: If two or more vectors are parallel to the same
line segment irrespective of their magnitudes and direction are → → → → → →
called collinear vectors. In the given figure, AB + BC = AC or a + b = c
Properties of Addition of Vectors
→ →
1. Vector addition is commutative: This means, if a and b
→ → → →
are two vectors then: a + b = b + a .
→ → →
→ → →
Vectors a, b and c are collinear vectors because all these 2. Vector addition is associative: This means, if a, b and c
vectors are parallel to the same line segment AB. ( → →
) →
are three vectors, then: a + b + c = a + b + c

( → →
)
Equal Vectors: If two vectors have the same magnitude and the
→ → Unit Vectors Along Positive Directions of X-axis,
same direction, they are called equal vectors. Here a = b Y-axis and Z-axis

OX, OY and OZ are three mutually perpendicular axes. i is the
( )
→
unit vector along positive direction of x-axis OX , j is the unit

310 Mathematics–12
E:\AMIT_WORKS\Exam_Guru\EG_Mathematics-12_(working_02-06-2022)\EG_Mathematics-12_working\Open_Files\Chap_10\Chap_10
\ 17-Aug-2022 Amit Proof-3 Reader’s Sign _______________________ Date __________

( ) → ∧
vector along positive direction of y-axis OY , k is the unit vector \ In DAOB,
→
OA + AB = OB
→ →

along positive direction of z-axis ( OZ ) .


→ → → →
or AB = OB − OA
→ ∧ ∧ ∧
Here OA = x1 i + y1 j + z1 k
→ ∧ ∧ ∧
and OB = x2 i + y2 j + z2 k
→ → →
Then AB = OB − OA
→
( ∧ ∧
or AB = x2 i + y2 j + z2 k − x1 i + y1 j + z1 k

)( ∧ ∧ ∧
)
→ ∧ ∧ ∧
⇒ AB = ( x2 − x1) i + ( y2 − y1) j + ( z2 − z1) k
∧ ∧ ∧ →
Hence i, j and k are mutually perpendicular unit vectors. and magnitude of vector AB = distance between points A
They are also called orthogonal unit vectors. and B
→
Position Vector of a Point AB = ( x2 − x1)2 + ( y2 − y1)2 + ( z2 − z1)2
Take any point P in space. If O (0, 0, 0) is the origin of the Example. If A(1, – 2, 3) and B(4, 1, 7) are two points in space,
→
Cartesian coordinate system, then the vector OP having initial →

point O (0, 0, 0) and terminal point P(x, y, z) is called position then AB = position vector of point B – position vector of
vector of point P. point A.


→
( ) ( ∧ ∧
AB = 4 i + j + 7 k − i − 2 j + 3 k
∧ ∧ ∧ ∧
)
AB = (3 i + 3 j + 4 k ) and
→ ∧ ∧ ∧

→
AB = (3)2 + (3)2 + ( 4)2 = 9 + 9 + 16 = 34
This means distance between two points A (1, – 2, 3) and
B (4, 1, 7) is 34 units.
Here P is any point in space and its coordinates are
→ ∧ ∧ ∧
(x, y, z), then position vector of point p is x i + y j + z k or Components of a Vector
→ → ∧ ∧ ∧ Vector Components of A Vector: Take a vector:
OP = p = x i + y j + z k → ∧ ∧ ∧ ∧ ∧ ∧
→ ∧ ∧ ∧ r = x i + y j + z k . Here x i, y j and z k are called vector
Magnitude of a Position Vector: If p = x i + y j + z k is a →
components of the vector r .
position vector of point P(x, y, z), then the magnitude of the → ∧ ∧ ∧
position vector of point P(x, y, z) represents the distance of the Scalar Components of a Vector: Take a vector r = x i + y j + z k
→ →
point P from origin and it is equal to: p = x2 + y 2 + z 2 . . Here x, y and z are called scalar components of vector r .
→ ∧ ∧ ∧
Vector Joining Two Points Example. In a vector: r = 4 i − 5 j + 7 k
∧ ∧ ∧
Two points A(x1, y1, z1) and B(x2, y2, z2) are in space, with respect 4 i, − 5 j and 7 k are vector components and 4, – 5 and 7 are
to Cartesian coordinate system and origin O(0, 0, 0). Join O to A, →
→ → scalar components of the vector r .
O to B and A to B. It gives a ∆OAB with three vectors OA, AB
→
in order and vector OB in opposite order. Unit Vector Along a Given Vector
→ ∧ ∧ ∧ →
Take a vector r = x i + y j + z k . Here r = x 2 + y 2 + z 2 . Then
→ ∧
→  ∧ ∧ ∧
xi + y j + zk 
unit vector along vector r will be : r = r = ± 
→  x2 + y 2 + z 2 
r  
→ ∧ ∧ ∧
Example 1. Find a unit vector along the vector r = 3 i + 2 j + 6 k .
→ ∧ ∧ ∧
Solution. r = 3i + 2 j + 6k

⇒ r = (3) 2 + (2) 2 + (6) 2 = 9 + 4 + 36 = 49 = 7

Vector Algebra 311


E:\AMIT_WORKS\Exam_Guru\EG_Mathematics-12_(working_02-06-2022)\EG_Mathematics-12_working\Open_Files\Chap_10\Chap_10
\ 17-Aug-2022 Amit Proof-3 Reader’s Sign _______________________ Date __________

∧  ∧ → ∧ ∧  Direction Ratios and Direction Cosines of a Vector


\ r = ±  3 i +
r 2 j + 6 k  → ∧ ∧ ∧
=± Take a vector r = x i + y j + z k then x, y and z are direction
→  7 
r   →
x y
→ ∧ ∧ ∧ → ∧ ∧ ∧ ratios of vector r and , and
2 2 2
Example 2. If a = i + j + 2 k, b = 3 i − 4 j + k and z x+y +z x + y 2+
2 → z2
→ ∧ ∧ ∧
are the direction cosines of the vector r .
c = 2 i + 3 j + 5 k . Find a unit vector along the vector x 2+ y 2+ z 2
→ → → →
r = 2a − b +3c. Note:
→ →
Solution. Given vectors are:
(i) Two vectors r and λ r are collinear vectors.
→ ∧ ∧ ∧ → ∧ ∧ ∧ x
a = i + j + 2 k, b = 3 i − 4 j + k , (ii) cos a = ,
x + y2 + z2
2
→ → → →
then r = 2a − b +3c y
cos b = and
( ) ( ) x + y2 + z2
2
∧ ∧ ∧ ∧ ∧ ∧
= 2 i + j + 2k − 3i − 4 j + k + z
3(2 i + 3 j + 5 k )
∧ ∧ ∧ cos g =
x + y2 + z2
2

∧ ∧ ∧ ∧ ∧ ∧ ∧ ∧ ∧ (iii) If a, b, c are drs and cos α, cos β, cos γ are dcs of a vector,
= 2 i + 2 j + 4 k − 3 i + 4 j − k + 6 i + 9 j + 15 k then

( ∧ ∧
= 2i −3i +6i + 2 j + 4 j + 9 j +

) ( ∧ ∧ ∧
) (a) cos2 a + cos2 b + cos2 g = 1 and


(4 k − k + 15 k ) = 5 i + 15 j + 18 k
∧ ∧ ∧ ∧ ∧ ∧ (b) sin2 a + sin2 b + sin2 g = 2
 
Example 1. If a = b , then necessarily it implies a = ± b .
 
  

Solution. If a = b then a = ± b which is true.
⇒ r = (5)2 + (15)2 + (18)2 = 25 + 225 + 324
Hence, the statement is True.

⇒ r = 574 units Example 2. Position vector of a point P is a vector whose initial
point is origin.
 ∧ → ∧ ∧
∧ 5 i +r15 j + 18 k Solution. True
\ r = ± → =± 
 574  Example 3. The vector in the direction of the vector iˆ − 2 ˆj + 2kˆ
r  
that has magnitude 9 is
Note: (i) Position vector is not a full vector. iˆ − 2 ˆj + 2kˆ
(ii) Difference between two position vectors is a full (a) iˆ − 2 ˆj + 2kˆ (b)
3
vector. (c) 3(iˆ − 2 ˆj + 2kˆ) (d) 9(i − 2 ˆj + 2kˆ)
ˆ

Vectors with equal Magnitudes, but different Solution. Let a = iˆ − 2 ˆj + 2kˆ
 
a
Directions Unit vector in the direction of a = 
∧ ∧ ∧ ∧ ∧ ∧
a
Take a vector r = x i + y j + z k . If the coefficients of i, j, k iˆ − 2 ˆj + 2kˆ iˆ − 2 ˆj + 2kˆ iˆ − 2 ˆj + 2kˆ
are interchanged with same signs or different signs, then the new = = =
(1) 2 + ( − 2) 2 + (2) 2 1+ 4 + 4 3
vector so determined will have the same magnitude but different
→ ∧ ∧ ∧  Vector of magnitude 9

direction. For example, if we take a vector r = x i + y j + z k
9(iˆ − 2 ˆj + 2kˆ)
→ ∧ ∧ ∧ → ∧ ∧ ∧
= = 3(iˆ − 2 ˆj + 2kˆ)
then vectors r1 = y i + x j + z k, r2 = x i − z j + y k , and 3
→ ∧ ∧ ∧
Hence, the correct option is (c).
r3 = z i − x j − y k are of the same magnitude but different
Example 4. The vector having initial and terminal points as
directions.
(2, 5, 0) and (–3, 7, 4), respectively is
Vectors with same direction, but different (a) − iˆ + 12 ˆj + 4kˆ (b) 5iˆ + 2 ˆj − 4kˆ
Magnitudes (c) − 5iˆ + 2 ˆj + 4kˆ (d) iˆ + ˆj + kˆ
→ ∧ ∧ ∧ → ∧ ∧ ∧ Solution. Let A and B be two points whose coordinates are given as
Take a vector r = x i + y j + z k . Then vector r = x i + y j + z k (2, 5, 0) and (– 3, 7, 4)
→ ∧ ∧ ∧ 
and vector λ r = λx i + λy j + λz k will have same direction, but  AB = (  3  2)iˆ  (7  5) ˆj  (4  0)kˆ

AB =  5iˆ  2 ˆj  4 kˆ
→ →
different magnitude or vector r and λ r have same direction but 
different magnitude. Hence, the correct option is (c).

312 Mathematics–12
E:\AMIT_WORKS\Exam_Guru\EG_Mathematics-12_(working_02-06-2022)\EG_Mathematics-12_working\Open_Files\Chap_10\Chap_10
\ 17-Aug-2022 Amit Proof-3 Reader’s Sign _______________________ Date __________

→ ∧ ∧ ∧ → → → ∧ ∧ ∧
Example 5. If vector r = 2 i − 3 j + 6 k . Find direction ratios and Example 7. If vector r = AB , A( a ) = 2 i − j + 3 k and
→ → ∧ ∧ ∧ →
direction cosines of vector r . B( b ) = 4 i + 2 j − 5 k . Find r and verify: cos 2 a +
→ ∧ ∧ ∧
Solution. r = 2 i −3 j + 6k cos2 b + cos2 g = 1.
→ → ∧ ∧ ∧ → ∧ ∧ ∧
Direction ratios of r are a = 2, b = –3, c = 6 Solution. a = 2 i − j + 3k , b = 4 i + 2 j − 5k

r= a 2 + b 2 + c 2 = 4 + 9 + 36 = 7 → → → →


then r = AB = b − a
\ Direction cosines of r are →  ∧ ∧ ∧  ∧ ∧ ∧
a 2 r =  4 i + 2 j − 5 k −  2 i − j + 3 k
l = cos α = = , m = cos β = b = −3 ,    
r 7 r 7
c 6 → ∧ ∧ ∧ ∧ ∧ ∧
 n = cos γ = = ⇒ r = 4 i + 2 j −5k − 2 i + j − 3k
r 7
→ ∧ ∧ ∧
Example 6. If position vector of vertices of DABC are A( a ) , = 2 i + 3 j − 8k
→ → →
B( b ) and C( c ) . Find the median vector AD . ⇒ a = 2, b = 3 and c = –8
→ → →

Solution. In DABC, A(a ) , B( b ) and C( c ) , D is the mid-point ⇒ r = r = a 2 + b2 + c2
of BC.
 
 
\ D b + c ⇒ r= 4 + 9 + 64 = 77
 2 
a 2 b 3 c −8
 → → → → →
→
→ Now cos a = = , cos β = = , cos γ = =
b + c b + c − 2a r 77 r 77 r 77
Now AD =  −a=
 2  2
2 2 2
cos2 a + cos2 b + cos2 g =  2  +  3  +  −8 
→ → →      
→ b + c −2a 77   77   77 
\ AD =
2
 → →  → → → → = 4 + 9 + 64 = 77 = 1
77 77 77 77
 b − a  +  c − a  AB + AC
= = ⇒ cos2 a + cos2 b + cos2 g = 1
2 2

EXERCISE 10.1
I. Multiple Choice Questions (MCQs)
1. In triangle ABC, which of the following is not true: II. Very Short Answer Type Questions
→ ∧ ∧ ∧ → ∧ ∧ ∧ → ∧ ∧ ∧
1. If a = 2 i + 3 j − 4 k, b = i + j + k and c = 3 i − 5 j + k .
→ → → → →
Find a vector r if r = 2 a + b + 3 c . [Delhi 2012]
→ ∧ ∧ ∧ → ∧ ∧ ∧
2. If a = 3 i − j + 4 k, b = i + 2 j − k and
→ ∧ ∧ ∧

        c = 2 i + 4 j + k . Find the magnitude of a vector
(a) AB + BC + CA = 0 (b) AB + BC − AC = 0 → → → →
        r = 2a + 3b + c.
(c) AB + BC − CA = 0 (d) AB − CB + CA = 0
  3. Using the vector method find the length of AB if the
2. The vectors AB = 3iˆ + 4kˆ and AC = 5iˆ − 2 ˆj + 4kˆ are the
coordinates of points A and B are (1, 2, 1) and (3, – 1, 4)
sides of a triangle ABC. Then the length of median through
respectively.
A is:
4. Write the position vector of the point A (2, 1, – 3). Also
(a) 18 (b) 72 (c) 33 (d) 288 write it’s magnitude.
   → ∧ ∧ ∧
3. If a , b and c are non coplanar vectors, then the vectors,
        5. I f t h e t w o v e c t o r s a = 3i + y j + 2k and
a − 2b + 3c , 2a + 3b − 4c and − 7b + 10c are → ∧ ∧ ∧
(a) collinear b = 5x i − 3 j + (2z − 4) k are equal, find the values of
x, y and z.
(b) vertices of a triangle
6. Write the scalar and vector components of the vector
(c) Vertices of a right angled triangle → ∧ ∧ ∧
(d) None of these r = 2i + 3 j − 7k .

Vector Algebra 313


E:\AMIT_WORKS\Exam_Guru\EG_Mathematics-12_(working_02-06-2022)\EG_Mathematics-12_working\Open_Files\Chap_10\Chap_10
\ 17-Aug-2022 Amit Proof-3 Reader’s Sign _______________________ Date __________

7. There are two points in space A (1, – 1, 1) and III. Short Answer Type Questions
→ →
→ ∧ ∧ ∧ → ∧ ∧ ∧
B (3, 4, 6). Find the vector r = AB and find scalar and 1. If vectors a = 2 i + j + k and b = 3 i + 2 j − 3 k find

vector components of the vector r . the direction ratios and direction cosines of the vector
→ → →
8. Write a vector which is equal in magnitude of vector r = 3a + 2b.
→ ∧ ∧ ∧
r = 3 i + 2 j − 5 k but different in direction. → ∧ ∧ ∧ → ∧ ∧ ∧
→ ∧ ∧ ∧ 2. Given three vectors: a = 3 i + j + 2 k, b = 4 i − 3 j + 2 k
9. S h o w t h a t t h e v e c t o r s : a = 2 i − 3 j + 4 k and → ∧ ∧ ∧
→ ∧ ∧ ∧ and c = 2 i + 5 j + 2 k . Show that the vectors
b = − 4 i + 6 j − 8 k are collinear.
10. Write two different vectors having same magnitude.

( → →
)
r = 2 a − b and c are collinear vectors.

11. Write two different vectors having same direction.


→ ∧ ∧ ∧
3. S h o w that

A( i + 2 j − k ),
∧ ∧
( ∧
B 3i + 4 j and

)
12. If AB = 3 i − j + 2 k and coordinates of point A are
(0, –2, – 1), find the coordinates of the point B.
( ∧ ∧ ∧
)
C 6 i + 3 j − 4 k are the vertices of a right angled triangle.
[Delhi 2005] 4. S h o w t h a t t h r e e p o i n t s ( ∧
A − 2 i + 3 j + 5k ,
∧ ∧
)
13. Write all unit vectors in the xy-plane.
14. Write a unit vector in xy-plane making an angle of 30° with ( ∧ ∧ ∧
)
B i + 2 j + 3 k and C 7 i − k are collinear. ( ∧ ∧
)
the positive direction of the x-axis.  [Delhi 2009]
→ ∧ ∧ ∧
15. Find the angles that the vector r = 2 i − 3 j + 6 k makes IV. Long Answer Type Questions
with positive direction of x, y and z-axis. → ∧ ∧ ∧ → ∧ ∧ ∧
→ ∧ ∧ ∧ 1. If vector a = 3 i + j − k and vector b = 2 i + 5 j + 4 k .
16. Write the value of p for which a = 3 i + 2 j + 9 k and
→ ∧ ∧ ∧ Find
b = i + p j + 3 k are parallel vectors.
→ → →
[Delhi 2009, AI 2014] (i) vector r = 3 a + 2 b, and work out
17. If coordinates of two points are P(1, 5, 4) and Q(4, –1, –2), →
→ (ii) a unit vector along r and
find direction ratios of the vector PQ .[A.I. 2008]

18. Write direction cosines of a line equally inclined to the (iii) 7 units vector along r .
positive direction of three coordinates axes.  [A.I. 2009]
[A.I. 2010, 2011, 2011 (C), Delhi 2011, 2014]
19. For what value of λ the vectors 2 i − 3 j + 4 k ( ∧ ∧ ∧
) and 2. Find the direction ratios and direction cosines of the vector:
(λ i + 6 j − 8 k ) are collinear?
∧ ∧ ∧
[Delhi 2011]
→ ∧ ∧
b = 2 i + 2 j − k and verify:

20. What is the cosine of the angle which the vector (i) cos2 a + cos2 b + cos2 g = 1
→ ∧ ∧ ∧
a = 2 i + j + k makes with y-axis? (ii) sin2 a + sin2 b + sin2 g = 2

Answers 10.1
  
I. 1. In DABC, by Triangle Law of addition AB + BC = AC  1. 5iˆ − 2 ˆj + 4kˆ + 1. 3iˆ + 4kˆ
       Now AD =
⇒ AB + BC = − CA ⇒ AB + BC + CA = 0 1+1
Hence, (c) is not correct answer. (5 + 3)i + ( − 2 + 0) ˆj + (4 + 4)kˆ
ˆ
2. Let AD be the median through A. So D is the mid-point of BC. =
2
So the point D divides BC in the ratio 1 : 1
8iˆ − 2 ˆj + 8k ˆ
= = 4iˆ − ˆj + 4kˆ
2

\ AD = (4) 2 + ( − 1) 2 + (4) 2 = 16 + 1 + 16 = 33

Hence, (c) is the correct answer.


3. Let the three points whose position vectors are given be A, B
and C respectively. Let O be the origin of vectors.
          
OA = a − 2b + 3c , OB = 2a + 3b − 4c and OC = − 7 b + 10c
           
\ AB = OB − OA = (2a + 3b − 4c ) − (a − 2b + 3c ) = a + 5b − 7c

314 Mathematics–12
E:\AMIT_WORKS\Exam_Guru\EG_Mathematics-12_(working_02-06-2022)\EG_Mathematics-12_working\Open_Files\Chap_10\Chap_10
\ 17-Aug-2022 Amit Proof-3 Reader’s Sign _______________________ Date __________

    


\ AC = OC − OA = ( − 7b + 10c ) − (a − 2b + 3c )
        
= − a − 5b + 7c = − (a + 5b − 7c ) = − AB ⇒ AC = − AB.
→ ∧ ∧  3 ∧ 1 ∧
14. r = (cos 30°) i + (sin 30°) j =   i +
 2  2
j ()
Hence, (a) is the correct answer.
→ ∧
II. 1. r = 14 i − 8 j − 4 k
∧ ∧

2
7 ()
15. α = cos −1 2 , β = cos −1 
− 3
 7 
, γ = cos −1 6
7 ()
→ → ∧ ∧ ∧ 16. p =
2. r = 221; r = 11 i + 8 j + 6 k 3
→
→ → ∧ ∧ ∧
3. AB = 22; AB = 2 i − 3 j + 3 k 17. drs of PQ are: 1, – 2, – 2
→ ∧ ∧ ∧ 1 , 1 , 1
4. Position vector a = 2 i + j − 3 k . 18. Hint: Put α = β = γ = θ in
3 3 3
→ →
Magnitude of a is a = 14. cos2α + cos2β + cos2γ = 1
3 19. λ = – 4
5. x = , y = − 3, z = 3.
5
20. cos β = 1 . Hint: Find dcs of the given vector.

6. (i) Scalar components of r : 2, 3, − 7. 2
→ ∧ ∧ ∧
12 , 7 , −3
(ii) Vector components of r = 2 i, 3 j, − 7 k III. 1. drs: 12, 7, – 3. dcs:
→ ∧ ∧ ∧ 202 202 202
→ ∧ ∧ ∧
7. (i) r = 2 i + 5 j + 5 k Hint: r = 12 i + 7 j − 3 k

(ii) Scalar components of r : 2, 5, 5
→ ∧ ∧ ∧ 2. (2 a − b ) = 2 i + 5 j + 2 k and
→ → ∧ ∧ ∧

(iii) Vector components of r : 2 i, 5 j, 5 k → ∧ ∧ ∧


→ ∧ ∧ ∧ c = 2 i + 5 j + 2k
8. r1 = 3 i + 2 j + 5 k .
There can be many such answers. Q (2 a − b ) = ( c )
→ → →


( )

9. b = − 2 a .
→ →
\ (2 a − b ) and c are collinear vector.
→ → →

∴ Vectors a and b are collinear.


→ ∧ ∧ ∧ →
→ ∧ ∧ ∧ 3. AB = 2 i + 2 j + k ⇒ AB = 9
10. a = 2 i + 3 j + 6 k
→ ∧ ∧ ∧ →
→ ∧ ∧ ∧
BC = 3 i − j − 4 k ⇒ BC = 26
and b = 3 i + 6 j + 2 k
are different vectors with same magnitude. There can be many → ∧ ∧ ∧ →
such answers. AC = 5 i + j − 3 k ⇒ AC = 35
→ ∧ ∧ ∧
⇒ → 2 → 2 → 2
11. a = 2 i + 3 j + 6 k AC = AB + BC
→ ∧ ∧ ∧
⇒ DABC is a right angled ∆.
and b = 4 i + 6 j + 12 k
are two different vectors with same direction. → ∧ ∧ ∧ → ∧ ∧ ∧
4. AB = 3 i − j − 2 k, BC = 6 i − 2 j − 4 k
Note: You can write many such different answers.
12. B (3, – 3, 1)
→
⇒ BC = 2 AB ( ) →

13. Unit vectors in xy-plane: ⇒ Points A, B and C are collinear.


→ ∧ ∧ ∧
IV. 1. (i) r = 13 i + 13 j + 5 k
∧ ∧ ∧

(ii) r = 13 i + 13 j + 5 k
11 3

(iii) 7 r =
→ ( ∧
7 13 i + 13 j+ 5 k
∧ ∧
)
11 3
2. drs: 2, 2, – 1, dcs: 2 , 2 , −1
3 3 3
−1
 →
x
∧  y ∧ (i) Put cos α = , cos β = 2 and cos γ =
2
r =  i +  j 3 3 3
 x 2 + y 2   x 2 + y 2  in cos2 a + cos2 b + cos2 g = 1.

(ii) Now find sin2 α, sin2 β and sin2 γ and put these values in
→ ∧ ∧
or r = (cos θ) i + (sin θ) j sin2 a + sin2 b + sin2 g = 2.

Vector Algebra 315


E:\AMIT_WORKS\Exam_Guru\EG_Mathematics-12_(working_02-06-2022)\EG_Mathematics-12_working\Open_Files\Chap_10\Chap_10
\ 17-Aug-2022 Amit Proof-3 Reader’s Sign _______________________ Date __________

Topic 2. Section Formula for Position Vectors


Internal Division  The position vector of the required point c which divides
 
If a line segment joining two points the join of the given vectors a and b is
( ) ( ) is divided by a third point

A a and B b



c = m1x2 + m2 x1
m1 + m2
R ( r ) in the ratio m : n internally, then:
→ 
  
1 ⋅ (2a − 3b ) + 3(a + b )
→ → =
→ 3+1
r = mb +na     
2a − 3b + 3a + 3b 5a 5 
m+n = = = a
4 4 4
External Division: If a line segment joining
Hence, the correct option is (d).
two points A ( a ) and B ( b ) is divided by the
→ →
→ ∧ ∧ ∧

point R ( r ) in the ratio m : n externally then:


→ Note: In a ∆ABC, if AB = a1 i + b1 j + c1 k
→ ∧ ∧ ∧ →


→ → and AC = a2 i + b2 j + c2 k , then AD =
r = mb −na → →
m−n AB + AC
Position Vector of Mid-Point: If R r ( )

2
∆ABC.
where AD is the median of

is the position vector of the mid-point of a


line segment joining position vectors of two Proof: In ∆ABC, position vectors of vertices are
points then:

→ → () ()
→ →
() →
A a , B b and C c . Then, if D is the mid-point of BC,
a+b
r = → →
2
Example 1. The position vector of the point which divides the position vector of D is b + c
 2
  
join of points 2a − 3b and a + b in the ratio 3 : 1 is → → → → → →
    Now: AB = b − a , AC = c − a
3a − 2b 7 a − 8b
(a) (b)
2 4 → → → → → →
  AB + AC b − a+ c − a
3a 5a =
(c) (d) 2 2
4 4
  → → → → →
Solution. The given vectors are 2a  3b and a  b and the ratio (b ) + ( c ) − 2 a b + c → →
is 3 : 1. = = − a = AD
2 2

EXERCISE 10.2
I. Multiple Choice Questions (MCQs) II. Very Short Answer Type Questions
 
1. The position vector of the points A and B are a and b 1. Write the position vector of the mid-point of AB if
respectively. P divides AB in the ratio 3 : 1. Q is the mid- A( 3, 4, – 2) and B(1, 2, 4). Use vector method.
point of AP. Then the position vector of Q is:  [Delhi 2010]
   
5a + 3b a + 3b 2. P and Q are two points with position vector
(a) (b)

8
3a + 5b

4
 
3a + b
( → →
) ( → →
P 3 a − 2 b and Q a + b . Write the position vector of )
(c) (d) a point R which divides the line segment PQ in the ratio
4 4 2 : 1 externally. [A.I. 2010]
2. The co-ordinates of the point which divides the join of the 3. A and B are two points with position vectors
point (2, –1, 3) and (4, 3, 1) in the ratio 3 : 4 internally, are
given by ( → →
) ( →
A 2 a − 3 b and B 6 b − a respectively. Write the

)
position vector of a point P which divides the line segment
 2 20 10   10 15 2
(a)  , ,  (b)  , ,  AB in the ratio 1 : 2 internally. [A.I. 2013]
7 7 7  7 7 7
4. L and M are two points with position vectors
 20 5 15 
( ) ( )
15 20 3
(c)  , ,  (d)  , ,  → → → →
 7 7 7  7 7 7 L a − 2 b and M 2 a + b . Find the position vector of

316 Mathematics–12
E:\AMIT_WORKS\Exam_Guru\EG_Mathematics-12_(working_02-06-2022)\EG_Mathematics-12_working\Open_Files\Chap_10\Chap_10
\ 17-Aug-2022 Amit Proof-3 Reader’s Sign _______________________ Date __________

a point which divides the join of points L and M externally


in the ratio 2 : 1. [Delhi 2016] 3. Find the position vector of a point R r which divides the ( )

III. Long Answer Type Questions


line segment joining two points P 2 a + b and Q a − 3 b ( → →
) ( → →
)
1. Find the position vector R r ( )

which divides the
externally in the ratio 1 : 2. Also show that P is the mid-

( )
∧ ∧ ∧ point of the line segment RQ.
line segment joining the points A 3 i + j + 2 k and [Delhi 2010]
( ∧ ∧ ∧
)
B i − j − k in the ratio 3 : 2 (i) internally (ii) externally.
4. The position vectors of points A and B are a and b
→ →

2. A line segment AB whose vectors are A r1 = 3 a + b − 2 c () (


→ → → →
) respectively. A point P divides AB in the ratio 3 : 1
() (→ → →
and B r2 = a + 3 b + 3 c is divided by a point R r in

) ( )
→ internally and point Q is the mid-point of AP. Find the
position vector of the points P and Q.
the ratio 5 : 2 internally. Find the point R r . ( )

[Sample Paper 2014]

Answers 10.2
I. 1. P divides AB  in the ratio 3 : 1. The position vector of A and B → →
 4. 3 a + 4 b
are a and b respectively.
   
3b + a 3b + a
Position vector of P = = Hint:

3+1 4
→ → →
Q is the mid-point of AP, so it divides AP in the ratio 1 : 1. →

 2(2 a + b ) − 1 . ( a − 2 b )
The position vector of A is a and the postion vector of r =
  2 −1
3b + a
P is   → → → →
4 4a + 2b − a + 2b
 3b + a   =
1. a +1.
\ PV of Q = 4 = 5a + 3b 1
1+1 8 → →
  = 3a + 4b
 3b + a  
a+ 5a + 3b ∧ ∧ ∧
Position vector of Q = 4 = III. 1. 9i − j + k
2 8 (i) ,
5
Hence, (a) is the correct answer. ∧ ∧ ∧
 20 5 15  (ii) − 3 i − 5 j −7 k
2. (d)  , , 
 7 7 7
Hint: See the solution of above questions no. 3 and 4.

( ∧
II. 1. 2 i + 3 j + k
∧ ∧
) → ∧ ∧ ∧


11 a + 17 b + 11 c
→ →

2. R ( r ) =
Hint: A(3, 4, –2)
⇒ a = 3i + 4 j − 2k 7
( )
→ ∧ ∧ ∧ → →
B(1, 2, 4) ⇒ b = i + 2 j + 4k 3. R 3 a + 5 b .
→ →

( )
→ ∧ ∧ ∧
a+b → →
Mid-point p.v. = r = = 2i + 3 j + k Mid-point of RQ is P 2 a + b
2
(
2. R 4 b − a
→ →
) → →  → →
Hint: a + 3b 
→ → → → 4. P  and Q  5a + 3b 

2( a + b ) − 1 . (3 a − 2 b ) 4   8 
r =
2 −1 Hint:

→ → → →
2a + 2b − 3a + 2b → →
= → → →
3b + a
→ →
1 A( a ) and B( b ) and P( p) =
= 4b − a 4

P( a )
→ →
→ → →
3. 3b + a
Midpoint of A( a ) and P( p) =
4
Hint:
→ → →
→ → → →

1(6 b − a ) + 2(2 a − 3 b ) a + 3b + 4a
r = =
1+ 2 8
→ → → → → →
6b − a + 4a − 6b → 3b + 5a
= =a =
3 8

Vector Algebra 317


E:\AMIT_WORKS\Exam_Guru\EG_Mathematics-12_(working_02-06-2022)\EG_Mathematics-12_working\Open_Files\Chap_10\Chap_10
\ 22-Aug-2022 Amit Proof-3 Reader’s Sign _______________________ Date __________

Topic 3. Dot Product of Two Vectors


Two vectors are multiplied in two ways: ∧ ∧ ∧ ∧ ∧ ∧
(x) (a) i ⋅ i = 1, j . j = 1, k ⋅ k = 1
(i) Scalar or dot product, (ii) Vector or cross product.
∧ ∧ ∧ ∧ ∧ ∧
Scalar or Dot Product (b) i ⋅ j = 0, j . k = 0, k ⋅ i = 0
→ → ∧ ∧ ∧ ∧ ∧ ∧
The scalar or dot product of two non-zero vectors a and b is (c) j ⋅ i = 0, k . j = 0, i ⋅ k = 0
→ → (xi) Dot product is distributive over addition and subtraction.
denoted by a ⋅ b and defined as: → → →
→ → → → If a, b and c are three non-zero vectors, then
a ⋅ b = a b cos θ = ab cos θ.
( )
→ → → → → → →
Here θ is the angle between two vectors (a) a ⋅ b + c = a ⋅ b + a ⋅ c

(b) a ⋅ ( b − c ) = a ⋅ b − a ⋅ c
→ →
→ → → → → → →
a and b and 0 ≤ θ ≤ π.
→ →
(i) a ⋅ b is called scalar product → ∧ ∧ ∧ → ∧ ∧ ∧
→ → (xii) If p = a1 i + b1 j + c1 k and q = a2 i + b2 j + c2 k , then
because a ⋅ b is a scalar number. → →
→ → p ⋅ q = a1a2 + b1b2 + c1c2 .
(ii) a ⋅ b is called dot product → ∧ ∧ ∧ → ∧ ∧
because for multiplication symbol ‘ ⋅ ’ has been used. (xiii) If two vectors p = a1 i + b1 j + c1 k and q = a2 i + b2 j +
 ∧ → →
→ → a 
(iii) Dot product of a and b is the b c2 k are perpendicular, then p ⋅ q = 0
→ → or a1a2 + b1b2 + c1c2 = 0 .
projection of a over b .
  
Example 1. If a is any non-zero vector, then (a ⋅ iˆ) iˆ + (a ⋅ ˆj ) ˆj
Properties of Scalar or Dot Product 
+ (a ⋅ kˆ)kˆ equals .

→ → → → Solution. Let a = a1iˆ + a2 ˆj + a3kˆ
(i) a ⋅ b = a b cos θ = ab cos θ is a scalar number. 
 a  iˆ = (a1iˆ + a2 ˆj + a3kˆ) ⋅ iˆ
(ii) Scalar product is commutative: This means for any two = a1
→ → → → → →  ˆ  ˆ
non-zero vectors a and b : a ⋅ b = b ⋅ a Similarly, a ⋅ j = a2 and a ⋅ k = a3
   
→ → → →
 (a ⋅ iˆ) ⋅ iˆ + (a ⋅ ˆj ) ˆj + (a ⋅ kˆ) ⋅ kˆ = a1iˆ + a2 ˆj + a3kˆ = a

(iii) If two non-zero vectors a and b are such that a ⊥ b then Hence, the value of the filler is a .
→ → → → → → Example 2. State true or false.
a ⋅ b = a b cos 90º = a b × 0 = 0   
The formula (a + b )2 = a 2 + b 2 + 2a × b is valid for non-zero
→ → → →  
(iv) For any two vectors a and b if a ⋅ b = 0 then either vectors a and b .
      2 
→ → → → → →
Solution. (a + b ) 2 = (a + b ) ⋅ (a + b ) = a 2 + b + 2a ⋅ b
a = 0, or b = 0 or a ⊥ b .
Hence, the given statement is False.
→2 
→ → →
Example 3. The value of l such that the vectors a = 2iˆ + λˆj + kˆ
(v) If a is any non-zero vector then a ⋅ a = a . 
and b = iˆ + 2 ˆj + 3kˆ are orthogonal, is
→ →
(vi) If angle between two non-zero vectors a and b is 0°, then −5 3
(a) 0 (b) 1 (c) (d)
→ → → → → → 2 2
a ⋅ b = a b cos 0º = a b .  
Solution. Given that a  2i  j  k and b  i  2 ˆj  3 kˆ
ˆ ˆ ˆ ˆ
→ →  
(vii) If angle between two non-zero vectors a and b is 180°, Since a and b are orthogonal
 
→ → → → → → → → 
ab = 0
then a ⋅ b = a b cos 180º = a b (− 1) = − a b  (2i + λj + k ) ⋅ (iˆ + 2 ˆj + 3kˆ) = 0
ˆ ˆ ˆ
∧ ∧ ∧ ∧ ∧ ∧ 
2 + 2l + 3 = 0
(viii) If a and b are two unit vectors then a ⋅ b = a b cos q  5 + 2l = 0  λ =
−5
= 1 × 1 × cos q = cos q Hence, the correct option is (d). 2
      
∧ ∧ Example 4. If a , b , c are three vectors such that a + b + c = 0
\ a ⋅ b = cos θ        
∧ ∧ ∧ and a = 2, b = 3, c = 5, then the value of a ⋅ b + b ⋅ c + c ⋅ a is
(ix) If a is a unit vector then a ⋅ a = 1 × 1 × cos 0º = 1 . (a) 0 (b) 1 (c) – 19 (d) 38

318 Mathematics–12
E:\AMIT_WORKS\Exam_Guru\EG_Mathematics-12_(working_02-06-2022)\EG_Mathematics-12_working\Open_Files\Chap_10\Chap_10
\ 17-Aug-2022 Amit Proof-3 Reader’s Sign _______________________ Date __________

       
Solution. Given that a = 2, b = 3, c = 5,  (2) 2 + (3) 2 + (5) 2 + 2(a . b + b ⋅ c + c ⋅ a ) = 0
         
and a+b +c = 0  4 + 9 + 25 + 2 (a ⋅ b + b ⋅ c + c ⋅ a ) = 0
             
(a + b + c ) ⋅ (a + b + c ) = 0 ⋅ 0 = 0
 38 + 2(a ⋅ b + b ⋅ c + c ⋅ a ) = 0
  2             
 a 2 + a ⋅ b + a ⋅ c + b ⋅ a + b + b ⋅ c + c ⋅ a + c ⋅ b + c
2
 2(a ⋅ b + b ⋅ c + c ⋅ a ) = – 38
= 0      
\ a ⋅ b + b ⋅ c + c ⋅ a = – 19
2 2 2       Hence, the correct option is (c).
 a + b + c + 2a ⋅ b + 2b ⋅ c + 2c ⋅ a = 0

EXERCISE 10.3
I. Multiple Choice Questions (MCQs) → → → →
  7. If a and b are perpendicular vector and a + b = 13 and
1. If a = 4iˆ + 6 ˆj and b = 3 ˆj + 4kˆ, then the vector form of
  → →
component of a along b is a = 5, find b . [A.I. 2014]
18 18 ˆ
(3iˆ + 4kˆ)
∧ ∧ ∧
(a) (b) (3 j + 4kˆ) 8. If a, b and c are mutually perpendicular unit vectors, then
10 (3) 25
∧ ∧ ∧
18 find the value of 2 a + b + c .  [A.I. 2015]
(c) (3 ˆj + 4kˆ) (d) 3 ˆj + 4kˆ
3 III. Short Answer Type Questions
  
2. I f a = 2iˆ + 2 ˆj + 3kˆ, b = − iˆ + 2 ˆj + kˆ, c = 3iˆ + ˆj and → → → →
   1. If a = b and angle between a and b is 60° and
a + tb is normal to c, then the value of t is
(a) 8 (b) 4 (c) 6 (d) 2
→ →
1 then find →
a .
a⋅b= ,
   2
3. If two out of three vectors a , b , c are unit vectors,
         
a + b + c = 0 and 2 (a ⋅ b + b ⋅ c + c ⋅ a ) + 3 = 0, then
→ →
( →
2. Find a and b if a + b ⋅ a − b = 8 and a = 8 b .

)( → →
) → →

∧ ∧ ∧ ∧ ∧ ∧ →
the third is of length 3. If a, b and c are three unit vectors such that a + b + c = 0 ,
(a) 3
  
(b) 2 (c) 1
   
(d) 0 ( ∧
find the value of a ⋅ b + b ⋅ c + c ⋅ a .
∧ ∧ ∧ ∧ ∧
)
4. If a , b , c are unit vectors such that a + b + c = 0, then the → → → → → → →
      4. If a, b and c are three vectors such that: a + b + c = 0
value of a ⋅ b + b ⋅ c + c ⋅ a =
→ → →
(a) 1 (b) 2 and a = 3, b = 4 and c = 5, find the value of
3
(c) −
2
(d) None of these ( a ⋅ b + b ⋅ c + c ⋅ a ) .[Delhi 2012]
→ → → → → →

→ → →
II. Very Short Answer Type Questions 5. If a, b and c are mutually perpendicular vectors then

show that ( a + b + c ) = a
→ ∧ ∧ ∧ → ∧ ∧ ∧ → → → → → 2 →2 →2 →2
1. If a = 2 i + j + 4 k and b = i − 2 j + 5 k find a ⋅ b . + b + c .
→ ∧ ∧ ∧ → ∧ ∧ ∧ → ∧
[Foreign 2009, A.I. 2009] c = 3i +
6. I f a = i + 2 j + 3 k, b = − i + 2 j + k a n d

( )
→ →
∧ ∧ → → →
2. If a = 3, b = 2 and angle between these vectors j − 2 k then find λ if a − λ b ⊥ c .
→ → → →
a and b is 60°, find a ⋅ b .[Delhi 2011 (C)] → →2 →2 →2 → →

( → ∧
)(
3. Find x if x − a ⋅ x + a = 12. [A.I. 2009]
→ ∧
) 7. Prove that: a + b = a + b
IV. Long Answer Type Questions
when a ⊥ b .

→ → →
4. For any three non-zero vector a, b and c , show that: → ∧ ∧ ∧ → ∧ ∧ ∧

( ) ( ) ( )
→ → → → → → → → → 1. I f a = 2i + 3 j + k and b = i + j + 4k find
a⋅ b − c + b⋅ c − a + c ⋅ a − b =0.
→ → → → → →
(3 a + 2 b ) ⋅ ( 2 a + 3 b ) .
→ → → →

5. If a and b are two non-zero vectors and a + b = a − b → → → →


→ → 2. a and b are two vectors, such that: a = 2, b = 3 and
then prove that: a ⊥ b . → →
a ⋅ b = 4 find:
6. Find the value of λ if vectors a = ( 2 i + λ j + 3 k ) and
→ ∧ ∧ ∧
→ → → →

b = (3 i + 2 j − 4 k ) are perpendicular to each other.


→ ∧ ∧ ∧ (i) a − b (ii) a + b
→ → → →
[Delhi 2008, 2011 (C), AI 2010 (C), 2011 (C)] (iii) 3 a + 2 b (iv) 2 a − 3 b

Vector Algebra 319


E:\AMIT_WORKS\Exam_Guru\EG_Mathematics-12_(working_02-06-2022)\EG_Mathematics-12_working\Open_Files\Chap_10\Chap_10
\ 17-Aug-2022 Amit Proof-3 Reader’s Sign _______________________ Date __________

3. The scalar product of the vector i + j + k (


) with unit
∧ ∧ ∧ → ∧ ∧ ∧
7. If a = i + 2j − 3 k and b = 3 i − j + 2 k, show that:
→ ∧ ∧ ∧

vector along the sum of the vectors ( 2 i + 4 j − 5 k ) ( a + b ) ⊥ ( a − b ). [Delhi 2007]


∧ ∧ ∧ → → → →

and ( λ i + 2 j + 3 k ) is equal to 1. Find the value of λ.


∧ ∧ ∧
8. Show that each of the following three given vectors is a
[A.I. 2008 (C), 2009]
→ → → →
→ ∧ ∧ ∧
( →

7
∧ ∧ ∧
unit vector a = 1 2 i + 3 j + 6 k , b = 1 3 i − 6 j + 2 k
7
) ( )
4. If two vectors a and b are such that a = 2, b = 1 , and →
1 ∧ ∧
( ∧
)
→ →
(
a ⋅ b = 1 then find the value of 3 a − 5 b ⋅ 2 a + 7 b .

)( → →
) → and c = 6 i + 2 j − 3 k . Also show that these three
7
vectors are mutually perpendicular vectors.
[Delhi 2011]

r = ( r ⋅ i ) i + ( r⋅ j) j +
→ → → ∧ ∧ → ∧ ∧ → →
5. For any vector r , show that 9. For any two non-zero vectors a and b show that:
( r⋅ k ) k .
→ ∧ ∧
 → → → →  → → → →
 a b + b a  is perpendicular to  a b − b a  .
→ → →
6. If a, b and c are mutually perpendicular vectors → ∧ ∧ ∧ → ∧ ∧ ∧
→ → → 10. If a = i − j + 7 k and b = 5 i − j + λ k then find the
and a = 3, b = 2 and c = 5 find the value of :
→ →
3a + 2b + 4 c .

value of λ so that: a + b ⊥ a − b . ( → →
) ( → →
) [A.I. 2013]

Answers 10.3
 
I. 1. a = 4iˆ + 6 ˆj and b = 3 ˆj + 4kˆ      3
⇒ (a ⋅ b + b ⋅ c + c ⋅ a) = −
   2
  a ⋅b  
Vector form of component of a along b is   2  b . Hence, (c) is the correct answer.
   |b| 
a ⋅ b = 4(0) + 6(3) + 0(4) = 18 → →
 II. 1. a ⋅ b = 20
| b | = 32 + 42 = 9 + 16 = 5
 → →
| b |2 = (5)2 = 25 2. a⋅b =3
   18 
(3 ˆj + 4kˆ)
→ →
\ Vector form of component of a and along b = 
 25  Hint: a . b = ab cos θ = 3 × 2 × cos 60°

Hence, (b) answer is correct.
 
a = 2iˆ + 2 ˆj + 3kˆ, b = − iˆ + 2 ˆj + kˆ

2. 3. x = 13
 
a + tb = (2iˆ + 2 ˆj + 3kˆ) + t ( − iˆ + 2 ˆj + kˆ)
= (2 − t ) iˆ + (2 + 2t ) ˆj + (3 + t )kˆ
( → ∧
)(
Hint: x − a ⋅ x + a = 12 ⇒ x − 1 = 12

→ ∧
) →2


Also c = 3iˆ + ˆj + (0) kˆ

 
Since a + tb is normal to c


( → →
) →
4. Hint: a ⋅ b − c + b ⋅ c − a + c ⋅ a − b ( → →
) →
( → →
)
 
(
) 
(
\ a + tb ⊥ c ⇒ a + tb ⋅ c = 0 )  → → → →
= a⋅b − a⋅c + b⋅c −b⋅a + c⋅a − c⋅b
→ → → → → → → →

⇒ (2 – t) 3 + (2 + 2t) 1 + 0 = 0 ⇒ 6 – 3t + 2 + 2t = 0
⇒ 8 – t = 0; t = 8. Hence, (a) is the correct answer. → → → → → → → → → → → →
  = a⋅b − c⋅a + b⋅c − a⋅b + c⋅a −b⋅c =0
3. Let | a | = | b | = 1
      →2 →2
| a + b + c |2 = (a + b + c ) 2 → → → → → →
5. Hint: a + b = a − b ⇒ a + b = a − b
[...Sq. of a vector = Sq. of its modulus]
         → → → →
= a 2 + b 2 + c 2 + 2(a ⋅ b + b ⋅ c + c ⋅ a ) 2 2 2 2
⇒ a +b + 2a ⋅ b = a +b − 2a ⋅ b
        
⇒ 0 = | a |2 + | b |2 + | c |2 + 2(a ⋅ b + b ⋅ c + c ⋅ a ) → → → → → → → →
 ⇒ 2a ⋅ b = − 2a ⋅ b ⇒ 4a ⋅ b = 0⇒ a ⊥ b
⇒ 1 + 1 + | c |2 − 3 = 0
        → →
 [Q | a | = | b | = 1 and 2 (a ⋅ b + b ⋅ c + c ⋅ a ) = − 3] 6. λ = 3. Hint: Find λ when a ⋅ b = 0
 
⇒ | c |2 =1 ⇒ | c | =1 →
 7. b = 12
\ The length of the third vector c is 1.
Hence, (c) is the correct answer. → → 2 → →
2 2
            Hint:
a+b = a + b + 2 a.b
4. (a + b + c ) 2 = a 2 + b 2 + c 2 + 2a ⋅ b + 2b ⋅ c + 2c . a → →
     
⇒ (0)2 = 1 + 1 + 1 + 2 (a ⋅ b + b ⋅ c + c ⋅ a ) = 0 169 = 25 + b2 + 0 [Q a ⊥ b ]

320 Mathematics–12
E:\AMIT_WORKS\Exam_Guru\EG_Mathematics-12_(working_02-06-2022)\EG_Mathematics-12_working\Open_Files\Chap_10\Chap_10
\ 17-Aug-2022 Amit Proof-3 Reader’s Sign _______________________ Date __________

( )
→ →
8. 6 1 → → →
∧ 2 ∧ 2 ∧ 2 ∧ 2
6. λ = Hint: a − λ b ⋅ c = 0 ⇒ λ = a . c
∧ ∧ 3 b ⋅c
Hint:
2a + b + c =4 a + b + c
→ →2 →2 →2
∧ ∧ ∧ ∧ ∧ ∧ 7. Hint: a + b = a + b
+ 4 a . b + 2b . c + 4 a . c
→2 →2 → → →2 →2
= 4(1) + (1) + (1) + 0 + 0 + 0 ⇒ a + b + 2a ⋅ b = a + b

∧ ∧ ∧
→ → → →
[∵ a, b and c are perp. to each other]
⇒2a ⋅ b = 0⇒ a ⊥ b

=6

⇒ 2 a + b + c = 6. ( → →
IV. 1. 3 a + 2 b ⋅ 2 a + 3 b = 309 )( → →
)
III. 1. a = 1

( →
Hint: 3 a + 2 b = 8 i + 11 j + 11k and


) ∧ ∧ ∧

( )
→ →
→ → ∧ ∧ ∧
Hint: a = b and θ = 60º

2 a + 3 b = 7 i + 9 j + 14 k
→ → → →

\ a ⋅ b = a ⋅ a = a 2 cos 60º = 1

→ →
2

Q a ⋅ b = a b cos θ ⇒ 1 = a b cos 60°
→ → → ( →
Now find 3 a + 2 b . 2 a + 3 b

)( → →
)
2 → →
2. (i) a − b = 5
()
→ 2 → 2 →
⇒1= a
1 ⇒ a =1 ⇒ a =1
2 2 Hint:

2.
→ →
a = 16 2 and b = 2 2
→ →2
Work out: a − b = a − b ⋅ a − b = a 2 + b 2 − 2 a . b
( → →
)( → →
) → →

63 63
→ →

Hint: a + b . a − b = 8
( → →
)( → →
) (ii) a + b = 21

2 2 Hint:
⇒ a –b =8 ⇒ 64b – b2 = 8 ­2

∧ ∧ ∧ ∧ ∧ ∧
3. a ⋅ b + b ⋅ c + c ⋅ a = − 3
2
→ →2
Work out a + b = a + b ⋅ a + b = a 2 + b 2 + 2 a . b( → →
)( → →
) → →

∧ ∧ ∧ → ∧ ∧ ∧2 → →
Hint: a + b + c = 0, and a + b + c
(iii) 3 a + 2 b = 2 30 ;

Hint: Work out



() () ()
∧ 2 ∧ 2
= a + b + c + 2 a ⋅ b + 2b ⋅ c + 2c ⋅ a = 0
∧ 2 ∧ ∧ ∧ ∧ ∧ ∧

Put a = b = c = 1.
∧ ∧ ∧
→ →2
( → →
3a + 2 b = 3 a + 2 b ⋅ 3 a + 2 b = 9a 2 + 4b 2 + 12 a b )( → →
) →→


→ →
→ → → → → → (iv) 2a − 3b = 7
4. a ⋅ b + b ⋅ c + c ⋅ a = − 25
Hint: Work out

→ → → → → → →2
Hint: a + b + c = 0, Now find a + b + c

→ → → → → → → → →
→ →2
( →
2 a − 3 b = 2 a − 3 b ⋅ 2 a − 3 b = 4a 2 + 9b 2 − 12 a b

)( → →
) →→

= | a |2 + | b |2 + | c |2 + 2 a . b + 2 b . c + 2 c . a

3. λ = 1 ,
→ → →
and put a = 3, b = 4 and c = 5 Hint: Find

5.  Hint: a + b + c ( → → → 2
)=a →2 →2
+ b + c
→2

( ∧ ∧
2 i + 4 j − 5k + λ i + 2 j + 3k

) ( ∧ ∧ ∧
) = ( λ + 2) i + ( 6 j − 2 k )∧ ∧ ∧

( → →
+2 a⋅b +b⋅c + c⋅a
→ → → →
)

( )
∧ ∧ ∧

( )=a
∧ ∧ ∧
→ → → 2 →2 →2 →2 (λ + 2) i + 6 j − 2 k
a + b + c + 2(0 + 0 + 0) Now ⋅ i + j + k =1
+ b + c 2 2 2
(λ + 2) + (6) + (− 2)
→ → →
Qa ⊥ b ⊥ c. 4. 0

Vector Algebra 321


E:\AMIT_WORKS\Exam_Guru\EG_Mathematics-12_(working_02-06-2022)\EG_Mathematics-12_working\Open_Files\Chap_10\Chap_10
\ 17-Aug-2022 Amit Proof-3 Reader’s Sign _______________________ Date __________

Hint: (3 a − 5 b ).(2 a + 7 b )
→ → → → → → ∧ ∧ ∧ →
7. Hint: a + b = 4 i + j − k and a − b = − 2 i + 3 j − 5 k
→ ∧ ∧ ∧


→ → → →
= 6a 2 + 21 a . b − 10 a . b − 35b 2 (
Now show that a + b ⋅ a − b = 0
→ →
)( → →
)
→ → →
→ ∧ ∧ ∧
5. Hint: Take r = a i + b j + c k and put it in 8. Hint: If : a = 1, b = 1 and c = 1

( ) ( ) ( )
→ → → →
→ ∧ ∧ → ∧ ∧ → ∧ ∧ →
r ⋅ i i + r ⋅ j j + r ⋅ k k, you will get it equal to r . Then show that: a ⋅ b = 0 ⇒ a ⊥ b
→ → → → → → → →
→ → →
b ⋅ c = 0 ⇒ b ⊥ c, c ⋅ a = 0 ⇒ c ⊥ a
6. 3 a + 2 b + 4 c = 497
Hint: Find
 → → → →  → → → →
9. Hint:  a b + b a  ⋅  a b − b a 
→ → →2 →2 →2 →2
3 a + 2 b + 4 c = 9 a + 4 b + 16 c 2 →2 2 →2
= → →
b − b a =0
→ → → → → → a
+ 12 a ⋅ b + 16 b ⋅ c + 24 c ⋅ a
 → → → →  → → → →
→ → → → → → → → →
\  a b + b a  ⊥  a b − b a 
Put a ⋅ b = b ⋅ c = c ⋅ a = 0 because a ⊥ b ⊥ c . 10. λ = ± 5

Topic 4. Angle between Two Vectors


If θ is the angle between two vectors  
Let q be the angle between the two diagonal vectors a + b and
→ →  
a and b then a − b then
   
→ → (a + b ) ⋅ (a − b )
cos q =    
a⋅b a+b a −b
cos θ = → →
,0≤θ≤π
a b (2iˆ − 2 ˆj ) ⋅ (4iˆ − 2 ˆj + 4kˆ)
=
  (2) 2 + ( − 2) 2 ⋅ (4) 2 + ( − 2) 2 + (4) 2
Example 1. The vector a + b bisects
 
the angle between the non-collinear vectors a and b if =
8+ 4
=
12
=
1
. 2 2 ⋅6 2 2 ⋅6 2

Solution. If vector a + b bisects the angle between non-collinear  q=
π
     4
vectors a and b then the angle between a + b and a is equal to 
   Hence the value of required filler is .
the angle between a + b and b .   4
      Example 3. If a and b are adjacent sides of a rhombus, then
a ⋅ (a + b ) a ⋅ (a + b )  
So, cos q =    =  ...(i) a ⋅b = 0 .
a a+b a a 2 + b2
   
   Solution. If a ⋅ b = 0 then a ⋅ b = a b cos 90°
b ⋅ (a + b )
Also, cos q =    [ q is same] So the angle between the adjacent sides of the rhombus should be
b ⋅ a+b
90° which is not possible.
  
b ⋅ (a + b ) Hence, the given statement is False.
=  ...(ii)  
b a 2 + b2 Example 4. The angle between two vectors a and b with
 
From eq. (i) and eq. (ii) we get, magnitudes 3 and 4 respectively and a ⋅ b = 2 3 is
      ππ 5π

a ⋅ (a + b )
= 
b ⋅ (a + b ) (a) π (b) (d) (c)
6 2
3  2
a a 2 + b2 b a 2 + b2 Solution. Here, given that a = 3, b = 4 and a ⋅ b = 2 3
 
a b    From scalar product, we know that
  =   â = b̂ ⇒ a = b   
a b a ⋅ b = a b cos θ
 
Hence, the required filler is a = b .
  
2 3 = 3 ⋅ 4 ⋅ cos θ
Example 2. The vectors a = 3iˆ − 2 ˆj + 2kˆ and b = − iˆ − 2kˆ are the
adjacent sides of a parallelogram. The acute angle between its 
cos q = 2 3 = 1
3⋅4 2
diagonals is .
  π
Solution. Given that a = 3iˆ − 2 ˆj + 2kˆ and b = − iˆ − 2kˆ  q=
3
   
 a + b = 2iˆ − 2 ˆj and a − b = 4iˆ − 2 ˆj + 4kˆ
Hence, the correct option is (b).

322 Mathematics–12
E:\AMIT_WORKS\Exam_Guru\EG_Mathematics-12_(working_02-06-2022)\EG_Mathematics-12_working\Open_Files\Chap_10\Chap_10
\ 17-Aug-2022 Amit Proof-3 Reader’s Sign _______________________ Date __________

Example 5. The value of l for which the vectors 3iˆ − 6 ˆj + kˆ and 1 3 2


 
= ⇒λ=
2iˆ − 4 ˆj + λkˆ are parallel is λ 2 3
2 Hence, the correct option is (a).
2 3
(a) (b) (c) 5 (d) Example 6. Find the angle between two vectors:
3  2 2 5
Solution. Let a = 3iˆ − 6 ˆj + kˆ → ∧ ∧ ∧ → ∧ ∧ ∧

b = 2iˆ − 4 ˆj + λkˆ a = 3 i + j + 2 k and b = i − 4 j + 5 k

Since the given vectors are parallel,
→ ∧ ∧ ∧
 Angle between them is 0° Solution. a = 3i + j + 2k
   
so a ⋅ b = a b cos 0 → ∧ ∧ ∧
 (3iˆ – 6 ˆj + kˆ) ⋅ (2iˆ − 4 ˆj + λkˆ) and b = i − 4 j + 5k
= 3iˆ − 6 ˆj + kˆ 2iˆ − 4 ˆj + λkˆ → →
a⋅b (3 − 4 + 10)
6 + 24 + λ = 9 + 36 + 1 ⋅ 4 + 16 + λ 2 cos q = =
→ →
9 + 1 + 4 1 + 16 + 25
a b
30 + l = 46 ⋅ 20 + λ 2 9 9
Squaring both sides, we get = =
900 + l2 + 60 l = 46(20 + l2)
14 × 42 588
 900 + l2 + 60l = 920 + 46l2 ⇒ q = cos −1  9 
2 2
 l – 46l + 60l + 900 – 920 = 0  14 3 
 – 45l2 + 60l – 20 = 0
 9l2 – 12l + 4 = 0 Projection of a Vector on another Vector
 (3l – 2)2 = 0 → →
AB is the projection of a on b .
 3l – 2 = 0
→ →
 3l = 2 → →
a⋅b
\ l = 2/3 Projection of a on b = →
Alternate method: b
 Example 7. Find the projection of
Let a = a1iˆ + a2 ˆj + a3kˆ
 → ∧ ∧ ∧ → ∧ ∧ ∧
and  b = b1iˆ + b2 ˆj + b3kˆ a = 3 i + j + 2 k on b = i − 2 j + 3 k .
 
If a || b → ∧ ∧ ∧ → ∧ ∧ ∧
a1 a2 a3 Solution. a = 3 i + j + 2 k, b = i − 2 j + 3 k
 = = → →
b1 b2 b3 → →
Projection of a on b = a ⋅ b = 3 − 2 + 6 = 7
3 −6 1 →

= = 1+ 4 + 9 14
2 −4 λ b

EXERCISE 10.4
I. Multiple Choice Questions (MCQs) → ∧ ∧ ∧
vector b = 2 i + 6 j + 3 k .
1. The angle between the vectors ( 2iˆ + 6 ˆj + 3kˆ) and
[A.I. 2011, Delhi 2013 (C), 2014, 2014 (C), 2015]
(12iˆ − 4 ˆj + 3kˆ )

(a) cos − 1 
1 

 9 
(b) cos − 1 
2. F i n d t h e p r o j e c t i o n o f ( b + c ) on a ,
→ → →
where
 10   11  → ∧ ∧ ∧ → ∧ ∧ ∧ → ∧ ∧ ∧
a = 2 i − 2 j + k, b = i + 2 j −2 k and c = 2 i − j + 4 k
9   1
(c) cos − 1  (d) cos − 1   [Delhi 2007, AI 2007, 2013 (C)]
 91   9 → → → →
3. If a = b = 3 and a ⋅ b = 3 , Find the projection of
2. If q is the acute angle and the vector iˆ ( sin θ) + ˆj ( cos θ) → →
is perpendicular to the vector iˆ − 3, then q = a on b . [Delhi 2010 (C)]
→ ∧ ∧ ∧ →
π π π π 4. Find λ when projection of a = λ i + j + 4 k on b =
(a) (b) (c) (d)
6 5 4 3 ∧ ∧ ∧
      2 i + 6 j + 3 k is 4 units. [Delhi 2012]
3. If a and b are vectors such that | a + b | = | a − b |, then → ∧ ∧ ∧
  5. Find the angle between the vector a = i − j + k and
the angle between a and b is: → ∧ ∧ ∧
(a) 120° (b) 60° (c) 90° (d) 30° b = i + j − k .[Delhi 2008]
→ → → →
II. Very Short Answer Type Questions 6. If a = 3, b = 2 and a ⋅ b = 3 , find the angle between
→ ∧ ∧ ∧ → →
1. Write the projection of vector a = 7 i + j − 4 k on the a and b .[Delhi 2009, AI 2008, 2011]

Vector Algebra 323


E:\AMIT_WORKS\Exam_Guru\EG_Mathematics-12_(working_02-06-2022)\EG_Mathematics-12_working\Open_Files\Chap_10\Chap_10
\ 17-Aug-2022 Amit Proof-3 Reader’s Sign _______________________ Date __________

∧ ∧ ∧ → ∧ ∧ ∧ → ∧ ∧ ∧
7. If a unit vector a makes an angle π with i, π with j and 6. If a = i + 3 j − 5 k and b = 3 i − j + 2 k find the
∧ 3 4 → → → → → →
an acute angle γ with k , then find the value of γ. projection of p = 2 a + 3 b on q = 3 a − 2 b .
[Delhi 2013] [Delhi 2011]
→ 7. If the coordinates of vectors A, B and C of ∆ABC are
8. Find a vector a of magnitude 5 2, making an angle of (1, 2, 3), (– 1, 0, 0) and (0, 1, 2) respectively then
π with x-axis, π with y-axis and an acute angle γ with find ∠ABC. [∠ABC is the angle between the vectors
4 2 → →
z-axis.[A.I. 2014] BA and BA ]

( ) ( ) ( )
∧ ∧ ∧ ∧ ∧ ∧ ∧ ∧ ∧ ∧
9. If a and b are two unit vectors, then find the angle between 8. I f A i + j + k , B 2 i + 5 j , C 3 i + 2 j − 3k and
∧ ∧
a and b if ( ∧ ∧
)
3 a − b is a unit vector.  [Delhi 2014 (C)] ( ∧ ∧ ∧
)
D i − 6 j − k are position vectors of points A, B, C and D
→ → → →
10. If a and b are perpendicular vectors such that: respectively. Find the angle between AB and CD and
→ → → → → →
a + b = 13 and a = 5 find the value of b . deduce that AB and CD are collinear.

[A.I. 2014] 9. Dot product of a vector with vectors i − j , i + k and ( )( )


∧ ∧ ∧ ∧

III. Short Answer Type Question ( i + j − 2 k ) are 1, – 6 and 3 respectively, find the vector.
∧ ∧ ∧

10. Dot product of a vector with the vectors ( i − j + k ) ,


→ → → ∧ ∧ ∧
∧ ∧ ∧
1. If a ⋅ b = 8 and b = 2 i + 6 j + 3 k then find the projection

(2 i + j − 3 k ) and ( i + j + k ) are respectively 4, 0 and


→ →
∧ ∧ ∧ ∧ ∧ ∧
of a on b . [Delhi 2009 (C)]
IV. Long Answer Type Questions 2. Find the vector. [Delhi 2005, 2013 (C)]
→ → →
→ → → → → → →
1. If a + b + c = 0 and a = 3, b = 5 and c = 7 . Show 11. If three non-zero vectors a, b and c are such that
→ → → →
→ → a + b + c = 0 . Show that the angle θ between the vectors
that the angle between a and b is 60°. → → 2 2 2
[V. Imp.] [Delhi 2008, 2014, A.I. 2008] b and c is given by cos θ = a − b − c .
2bc
→ → →
→ → →
2. If a, b and c are three mutually perpendicular vectors 12. If a, b and c are three vectors of magnitudes 3, 4 and 5
of equal magnitude, show that the vector a + b + c ( → → →
) respectively, such that each one is perpendicular to the sum
→ → →
→ → → of the other two vectors. Prove that: a + b + c = 5 2.
is equally inclined to vectors a, b and c . Also find the
angle. [V. Imp.] [Delhi 2011 (C), 2013 (C)]  [V. Imp.] [Delhi 2010 (C), 2011 (C)]
→ → → → →
3. Find the value of λ for which the angle between the 13. If a and b are two vectors such that: a + b = a . Prove
( )
→ ∧ ∧ ∧ → ∧ ∧ ∧ → → →
vectors a = 2λ 2 i + 4λ j+ k and b = 7 i − 2 j + λ k is that: 2 a + b ⊥ b . [Delhi 2013]
obtuse.[A.I. 2013 (C)] → → →
14. If a, b and c are three mutually perpendicular vectors
( )
∧ ∧
→ → →
4. If a and b are two unit vectors such that their sum is also
of equal magnitude, show that the vector a + b + c is
a unit vector, then find the angle between them. → → →
[Delhi 2014] equally inclined to vectors a, b and c . Also find the angle
∧ ∧
5. If a and b are two unit vectors such that their difference ( →
which a + b + c makes with a, b or c .
→ →
) → → →

is also a unit vector, find angle between them. [Delhi 2017]

Answers 10.4
I. 1. Let q be the angle between the vectors 
  | b| = (12 ) 2 + ( − 4 ) 2 + ( 3 ) 2 = 144 + 16 + 9
a = 2iˆ + 6 ˆj + 3kˆ and b = 12iˆ − 4 ˆj + 3kˆ
  169 = 13=
a⋅b
cos q =  
| a |⋅|b |  
a⋅b 9 9  9
  \ cos q =   = = ⇒ θ = cos − 1  
a ⋅ b = ( 2iˆ + 6 ˆj + 3kˆ ) ⋅ ( 12iˆ − 4 ˆj + 3kˆ ) | a | . | b | ( 7 )(13 ) 91  91
= 2(12) + 6(–4) + 3(3) = 24 – 24 + 9 = 9 Hence, (c) is the correct answer
  
| a | = 22 + 62 + 32 = 4 + 36 + 9 = 49 = 7 2. Let a = iˆ (sin θ ) + ˆj (cos θ ) and b = iˆ − 3 , ˆj

324 Mathematics–12
E:\AMIT_WORKS\Exam_Guru\EG_Mathematics-12_(working_02-06-2022)\EG_Mathematics-12_working\Open_Files\Chap_10\Chap_10
\ 17-Aug-2022 Amit Proof-3 Reader’s Sign _______________________ Date __________

 
Since a ⊥ b → → →2
10. b = 12   Hint: a + b = 169
∴ sin θ (1) + cos θ ( − 3 ) = 0
sin θ →2 →2 → →
⇒ sin θ − 3 cos θ = 0 ⇒ = 3 ⇒ a + b = 169
Qa ⊥ b
cos θ
π → →
⇒ tan θ = 3 ⇒ θ = . → →
3 III. 1. Projection of a on b = a ⋅ b = 8
→ 7
Hence, (d) is the correct answer. b

3. (c) 90° IV. 1. θ = 60°


→ → → →
→ →
→ → Hint: a + b + c = 0
II. 1. Projection of a on b = a ⋅ b = 8 .
→ 7 → → → → →2 →2
b ⇒a+b=−c⇒ a+b = −c

Projection of ( b + c ) on a =
→ (b + c )⋅ a = 2
→ →
→ → →
→2
⇒ a + b + 2a ⋅ b = c

→2 → → →2


2. a
→ → →2 →2 →2
⇒ 2 a b cos θ = c − a − b

→ →
→ → →2 →2 →2
3. Projection of a on b = a ⋅ b = 3 = 1 c − a − b
→ 3 1
b cos θ = =
→ → 2
2a b

4. λ = 5
 
→ →
→ →
2. Each angle = cos −1  1 
a⋅b  3
Hint: Projection of a on b =

=4
b → → →2
→ → Hint: a + b + c

−1 a⋅b
5. θ = cos −1   Hint: cos θ = → → →2 →2 →2
 3 → → → → → →
a b = a + b + c + 2a ⋅ b + 2b ⋅ c + 2c . a

→ →
a⋅b → → → → → → →
6. θ = 30° Hint: cos θ = → → Since a = b = c and a ⊥ b , b ⊥ c
a b
7. γ = 60° → → → → →2 →2
and c ⊥ a  \ a + b + c =3 a +0+0+0
∧ ∧ ∧ ∧
Hint: a = ( cos α ) i + (cos β) j + (cos γ) k
→ → → → → →

⇒ a+b+c = 3 a = 3 b = 3 c
We are given : α = π , β = π .
3 4
Since cos2 a + cos2 b + cos2 g = 1 ⇒ g = 60°. Angle between a and a + b + c

( → → →
)
8.

( )
a =5 i +k
∧ ∧

θ = cos −1 (
→ → → →
a ⋅ a + b + c )  = cos −1 
 → →
a⋅a


 → → → →  → → 
Hint: α = π , β = π , γ = ?
 a a + b + c   a 3 a 
4 2
Put the values in cos2 a + cos2 b + cos2 g = 1  
→2
1
⇒ θ = cos −1   = cos −1  1 
a
⇒ cos γ =  → 2
 
2  3 a  3
→  ∧ ∧ ∧
\ a = 5 2 ( cos α ) i + ( cos β ) j+ ( cos γ ) k 
  3. 0 < λ < 1
2

→   ∧ ∧
  ∧
⇒ a = 5 2  1  i + 0 j +  1  k  ⇒ a = 5 i + k
 2   2 
→ ∧ ∧
( ) Hint: The angle q between vector a and b is
→ →

→ →
a⋅b
9. θ = 30° cosθ = → → ...(1)
∧ ∧ ∧ a b
Hint:
3 a − b = c (Assume)
2 2
∧ ∧ ∧ The angle q is obtuse (Given)

⇒ 3 a − b = c ⇒ cos θ = 3 ⇒ θ = 30°
2 \ cos q < 0

Vector Algebra 325


E:\AMIT_WORKS\Exam_Guru\EG_Mathematics-12_(working_02-06-2022)\EG_Mathematics-12_working\Open_Files\Chap_10\Chap_10
\ 17-Aug-2022 Amit Proof-3 Reader’s Sign _______________________ Date __________

→ →
a⋅b
→ → < 0  [From (1)] 7. ∠ABC = cos −1  10 
 102 
a b
→ ∧ ∧ ∧ → ∧ ∧ ∧
→ →  → →  Hint: BA = 2 i + 2 j + 3 k and BC = i + j + 2 k

a⋅b < 0 Q a , b > 0 → →
  BA ⋅ BC
∧ ∧ ∧  ∧ ∧ ∧ cos ∠ABC =
 → →
⇒  2λ 2 i + 4λ j + k  ⋅  7 i − 2 j + λ k  < 0
BA BC
   
2

⇒14l – 8l + l < 0 ⇒ 14l2 – 7l < 0 → →
8. θ = 180° ⇒ AB and CD are collinear
⇒7l(2l – 1) < 0 or l . (2l – 1) < 0
→ ∧ ∧ ∧ →
Case I Hint: AB = i + 4 j − k ⇒ AB = 3 2

l > 0, 2l – 1 < 0
→ ∧ ∧ ∧ →
l > 0, λ<1
CD = − 2 i − 8 j + 2 k ⇒ CD = 6 2
2
\ 0<λ< 1
→ →
→ →
2 AB ⋅ CD
Also AB ⋅ CD = − 36 and cos θ = → →
= −1
Case II AB CD
λ < 0, 2λ − 1 > 0 rejecting as no
1  9.

( ∧
r = − 2 i + 3 j + 4k
∧ ∧
)
λ < 0, λ >  common solution is herre → ∧ ∧ ∧
2  Hint: Let the required vector be r = x i + y j + z k

Hence, the angle between the given vectors is obtuse if \ x – y = 1, x + z = –6 and x + y – 2z = 3.

2( )
λ ∈ 0, 1 . Solve them
→ ∧ ∧ ∧
4. θ = 120° 10. r = 2i − j + k
→ ∧ → ∧ → ∧ ∧ ∧
Hint: In ∆ABC, BC = a, CA = b Hint: Let the required vector be: r = x i + y j + z k

→ ∧
and let BA = c x – y + z = 4, 2x + y – 3z = 0 and x + y + z = 2.

∧ ∧ ∧ Now solve for x, y and z.
Also a + b = c,
→ → → → → → → → →2 →2
∧ ∧2 ∧2 11. Hint: a + b + c = 0 ⇒ b + c = − a ⇒ b + c = − a
⇒ a +b = c
→  a 2 − b2 − c2
∧2 ∧2 ∧ ∧ ∧2 ⇒ b 2 + c 2 + 2 b ⋅ c = a 2 ⇒ cos θ =
2bc
⇒ a + b + 2a ⋅ b = c

12. Hint: Given:

⇒1 + 1 + 2 × 1 × 1 × cos q = 1

⇒ cos θ =
−1
⇒ θ = 120°.

( → →
)
a⋅ b + c + b⋅ c + a + c⋅ a + b =0

( → →
) →
( → →
)
2 → → → → → →
5. θ = 60° ⇒a⋅b + b⋅c + c⋅a =0

→ ∧ → ∧ → ∧
Hint: In ∆ABC, BC = a, CA = b and BA = c → → →2 →2 →2 →2
∧ ∧ ∧ Now a + b + c = a + b + c
Also a − b = c → → → → → →
+ 2a ⋅ b + 2b ⋅ c + 2c ⋅ a
∧ ∧2 ∧2
⇒ a−b = c
= 9 + 16 + 25 + 2(0) = 50
∧ ∧ → → →
⇒1 + 1 − 2a ⋅ b = 1
a + b + c =5 2
∧ ∧


⇒ 2 a ⋅ b = 1 ⇒ 2 × 1 × 1 × cos θ = 1 → → → → →2 →2
13. Hint: a + b = a ⇒ a+b = a
⇒ cos θ = 1 ⇒ θ = 60°

2 →2 →2 → → →2 → → → →
→ → ⇒ a + b + 2a ⋅ b = a
⇒ b ⋅ b + 2a ⋅ b = 0
6. Projection of p on q = 76

491
∧ ∧ ∧ → ∧ ∧ ∧
( → →
⇒ b + 2a ⋅ b = 0⇒ 2a + b ⊥ b) →
( → →
) →

Hint: p = 11 i + 3 j − 4 k and q = − 3 i + 11 j − 19 k
→ → →
→ → 14. Hint: a, b , c are mutually ^ vectors of equal magnitude.
Now work out : p ⋅ q → → → → → → → → →
→ \ a . b = b . c = c . a = 0 and a = b = c  ...(1)
q

326 Mathematics–12
E:\AMIT_WORKS\Exam_Guru\EG_Mathematics-12_(working_02-06-2022)\EG_Mathematics-12_working\Open_Files\Chap_10\Chap_10
\ 19-Aug-2022 Amit Proof-3 Reader’s Sign _______________________ Date __________

( →
Let angle between a + b + c and a, b , c be a, b, g.
→ →
) → → → →
b

c
→ 2 Similarly, cos β = and cos γ =
( a + b + c ). a =
→ → → → → →
→ → → → a a + b + c a + b + c
cos α = → → → → → → → →

⇒ cos a = cos b = cos g [Using (1)]
a + b + c a a + b + c a

⇒a=b=g
→ Now cos2 a + cos2 b + cos2 g = 1
a
⇒ 3 cos2 a = 1

= 1 ⇒ 1
→ →
a + b + c

⇒ cos α =
α = cos −1
3 3

Topic 5. Vector Product or Cross Product


The vector product or cross product of two n̂ → → → → →
→ → 7. If a and b are two non-zero vectors and a × b = 0 then
non-zero vectors a and b is denoted by → →
→ →  a || b .
a × b and is defined as: b
∧ ∧ → ∧ ∧ → ∧ ∧ →
→ →
→ → ∧ θ 8. i × i = 0, j × j = 0 and k × k = 0
a × b =  a b sin θ  n . 
a ∧ ∧ ∧ ∧ ∧ ∧ ∧ ∧ ∧
→ →
9. i × j = k, j × k = i and k × i = j
Here θ is the angle between two vectors a and b and 0º ≤ ∧ ∧ ∧ ∧ ∧ ∧ ∧ ∧ ∧
∧ 10. j × i = − k, k × j = − i and i × k = − j
θ ≤ 180º and n is a unit vector perpendicular to both the vectors → ∧ ∧ ∧ → ∧ ∧ ∧
→ → → → ∧ 11. If a = a1 i + a2 j + a3 k and b = b1 i + b2 j + b3 k , then
a and b such that a, b and n form a right handed system ∧ ∧ ∧
→ → ∧ i j k
→ →
rotated from a to b in the direction of n .
a × b = a1 a2 a3
(i) ( a × b ) is called vector product because ( a × b ) is a
→ → → →
b1 b2 b3  
a ×b
vector. 
(ii) ( → →
)
a × b is called cross product because symbol ‘×’ is used
12. Direction of resultant vector can
be found using Right Hand Rule.
a


for multiplication. Example 1. The value of the expression b
 2  
Properties of cross product or vector product a × b + (a ⋅ b ) 2 is .
2      
Solution. a × b + (a ⋅ b )2 = ( a b sin θ) + ( a b cos θ)
→ →  2 2

1. F o r a n y t w o n o n - z e r o v e c t o r s a and b :
2 2 2
( a × b ) ≠ ( b × a ) but ( a × b ) = − ( b × a ).
 2
→ → → → → → → → b sin 2 θ + a b cos 2 θ
= a
2  2 
→ → → → → →
b ⋅ (sin 2 θ + cos 2 θ)
= a
2. For any two non-zero vectors a and b : a × b = b × a . 2  2  2 2
= a
b ⋅1 = a b
→ →
3. If a and b are two non-zero vectors then: (i) a ⊥ a × b

( → →
) 2 2
Hence, the value of the filler is a b .

and (ii) b ⊥ a × b

( → →
) 2 2 
Example 2. If a × b + a ⋅ b = 144 and a = 4, then b is equal
→ → → → → ^ → to .
4. If a is a non-zero vector then a × a = a a (sin 0°) n = 0  2  
Solution. a × b + (a ⋅ b )2 = 144
→ → →
   
( ) ( )
2 2
∴a × a = 0
 a b sin θ + a b cos θ = 144
→ → → → 2 2
5. a and b are two non-zero vectors and a || b then:  a 2 b sin 2 θ + a 2 b cos 2 θ = 144
 2  2
→ → → → ∧ → → → → → →  a 2 b (sin 2 θ + cos 2 θ) = 144  a 2 b = 144
a × b = a b (sin 0º ) n = 0 \ a × b = 0 when a || b
  
→ → → → →  a b = 12  4 ⋅ b = 12
6. For any two vectors a and b , if a × b = 0 then: (i) either 
→ → → →
\ 4⋅ b = 3
→ →
a = 0, (ii) or b = 0 (iii) a || b . Hence, the value of the filler is 3.

Vector Algebra 327


E:\AMIT_WORKS\Exam_Guru\EG_Mathematics-12_(working_02-06-2022)\EG_Mathematics-12_working\Open_Files\Chap_10\Chap_10
\ 17-Aug-2022 Amit Proof-3 Reader’s Sign _______________________ Date __________

  
Example 3. For any vector a , the value of (a × iˆ)2 + (a × ˆj )2 

4
sin q =

+ (a × kˆ) 2 is 5
(a) a 2

(b) 3a 2

(c) 4a 2

(d) 2a 2     4
Now a × b = a b sin θ = 10 ⋅ 2 ⋅ = 16
 5
Solution.Let a = a1iˆ + a2 ˆj + a3kˆ
Hence, the correct option is (d).
 2 2 2
 a 2 = a1 + a2 + a3 Example 5. The number of vectors of unit length perpendicular
 ˆ  
Now, a × i = (a1iˆ + a2 ˆj + a3kˆ) × iˆ to the vectors a = 2iˆ + ˆj + 2kˆ and b = ˆj + kˆ is
iˆ ˆj kˆ (a) one (b) two (c) three (d) infinite
= a a a
1 2 3 Solution. The number of vectors of unit length perpendicular to
1 0 0   
vectors a and b is c (let)
= iˆ (0 − 0) − ˆj (0 − a3 ) + kˆ(0 − a2 ) = a3 ˆj − a2 kˆ   
  c = ± (a × b )
 (a × iˆ) 2 = (a3 ˆj − a2 kˆ) ⋅ (a3 ˆj − a2 kˆ)
So, there will be two vectors of unit length perpendicular to vectors
2 2  
= a3 + a2 a and b .
 2 2 Hence, the correct option is (b).
Similarly (a × ˆj ) 2 = a1 + a3
 → ∧ ∧ ∧ → ∧ ∧ ∧
and (a × kˆ) 2 = a12 + a22 Example 6. If a = 2 i + j + 4 k and b = 2 i − 5 j + 2 k , find


 ˆ2  
\ (a × i ) + (a × ˆj )2 + (a × kˆ) 2
2 2 2 2 2 2
= a3 + a2 + a1 + a3 + a1 + a2
→ →
(i) a × b (ii) Show that a ⊥ a × b

( → →
) →
and b ⊥ a × b . ( → →
)
2 → ∧ ∧ ∧ → ∧ ∧ ∧
2 2 2
= 2(a1 + a2 + a3 ) = 2a Solution. a = 2 i + j + 4 k and b = 2 i − 5 j + 2 k
Hence, the correct option is (d). ∧ ∧ ∧
   i j k
Example 4. If a = 10, b = 2 and a ⋅ b = 12 , then value of a × b is → →
(a) 5 (b) 10 (c) 14 (d) 16 (i) a×b = 2 1 4 or
    2 −5 2
Solution. Given that a = 10, b = 2 and a ⋅ b = 12
  → → ∧ ∧ ∧
 a ⋅ b = a b cos a × b = i (2 + 20) − j (4 − 8) + k (− 10 − 2)
 12 = 10  2  cos q → → ∧ ∧ ∧
12 3 ⇒ a × b = 22 i + 4 j − 12 k
 cos q = =


\
20 5
sin q = 1 − cos 2 θ

( ) (
→ →
)(
∧ ∧
(ii) a ⋅ a × b = 2 i + j + 4 k ⋅ 22 i + 4 j − 12 k
∧ ∧ ∧ ∧
)
a ⋅ ( a × b ) = 44 + 4 – 48 = 0 ⇒ ( )
2 → → → → → →
 3 ⇒ a ^ a×b ,

sin q = 1−  
 5
9
→ → →  ∧ ∧ ∧  ∧ ∧ ∧
 
sin q = 1− b ⋅  a × b  =  2 i − 5 j + 2 k  ⋅  22 i + 4 j − 12 k 
25      


sin q =
16
25
= 44 – 20 – 24 = 0 ⇒

b ^ a×b ( → →
)
EXERCISE 10.5
I. Multiple Choice Questions (MCQs)
  3iˆ + 2 ˆj − kˆ and 12iˆ + 5 ˆj − 5kˆ will be

1. If a = 2iˆ + 2 ˆj − kˆ and b = 6iˆ − 3 ˆj − 2kˆ, then the value 115 51
  (a) (b)
of a × b is
14 194 14 144
(a) 2iˆ + 2 ˆj − kˆ (b) 6iˆ − 2 ˆj + 2kˆ
(c) iˆ − 10 ˆj − 18kˆ (d) iˆ + ˆj + kˆ (c)
64
(d) None of these
  14 194
2. The sine of the angle between the vector a = 3iˆ + ˆj + kˆ, b =
2iˆ − 2 ˆj + kˆ is      
4. If (a × b ) 2 + (a ⋅ b ) 2 = 144 and | a | = 4, then | b | =
74 25 (a) 16 (b) 8 (c) 3 (d) –3
(a) (b)
99 99   2   2
[| a × b | + (a ⋅ b ) ]
5. The value of = [HOTS]
(c) 37 (d) 5 2a 2 b 2
99 41 1 1
(a) 1 (b) (c) (2) (d)
3. The sine of the angle between the two vectors 2 4

328 Mathematics–12
E:\AMIT_WORKS\Exam_Guru\EG_Mathematics-12_(working_02-06-2022)\EG_Mathematics-12_working\Open_Files\Chap_10\Chap_10
\ 17-Aug-2022 Amit Proof-3 Reader’s Sign _______________________ Date __________

II. Very Short Answer Type Questions


() ()
→ →
() →

1. Find the values of λ and µ if a = 2 i + 6 j + 27 k ,



( ∧ ∧ ∧
) 3. If three points A a , B b and C c are collinear, prove
→ → → → → → →

( ∧ ∧ ∧
)
b = i + λ j + µ k and a × b = 0 . [A.I. 2009]
→ → → that a × b + b × c + c × a = 0 .
→ ∧ ∧ ∧ →
4. If a = i + 4 j + k and b = 3 i + 4 j + 5 k are two vectors,
∧ ∧ ∧
→ →
2. a and b are two non-zero vectors, show that: find:
( a − b ) × ( a + b ) = 2( a × b ) .
→ → → → → →
(i) A unit vector perpendicular to a and b , how many
→ →

3. Find the value of: i ⋅ ( j × k ) + j ⋅ ( i × k ) + k ⋅ ( i × j )


∧ ∧ ∧ ∧ ∧ ∧ ∧ ∧ ∧ such vectors can you find?
→ →
[A.I. 2008] (ii) Find a 7 units vector perpendicular to a and b .
→ →  [Foreign 2006, A.I. 2007 (C), Delhi 2009 (C)]
4. If a and b are two non-zero vectors such that: 5. Find a unit vector perpendicular to the plane of ∆ABC
→ → → → → →
a ⋅ b = a × b then find the angle between a and b . obtained by joining three points A(3, 2, 1), B(2, 3, 2) and
C(4, – 1, – 1).
( ) ∧
5. Write the value of: i × j ⋅ k + i ⋅ j . [A.I. 2012]
∧ ∧ ∧ ∧
6. Find a unit vector perpendicular to each of the vectors

6. Write the value of ( k × j ) ⋅ i + j ⋅ k . [A.I. 2012]


∧ ∧ ∧ ∧ ∧
( a + b ) and ( a − b )
→ → → → →
where a = 3 i + 2 j + 2k and
∧ ∧ ∧

→ ∧ ∧ ∧
III. Short Answer Type Questions-I b = i + 2 j− 2 k . [Delhi 2015, A.I. 2015]
→ → → →
1. If a and b are two non-zero vectors and θ is the angle 7. If a and b are unit vectors and q is the angle between
between them, then prove that: them, then prove that


→ →
a × b = a ⋅ b tan θ. [A.I. 2001] ( )
→ →

θ 1  
sin = a − b [CBSE 2022]
2 2
→ → → → → → → → →  →
2. If a, b and c are three non-zero vectors, then prove that: 8. I f a × b = c × d and a × c = b × d . S h o w t h a t

( ) ( ) ( ) ( a − d ) is parallel to ( b − c ) where a ≠ d and b ≠ c .


→ → → → → → → →
→ → → → → → → → → →
a× b+c +b× c+a +c× a+b =0
→ → → → [Delhi 2009]
3. If a = 3, b = 10 and a × b = 5, find the angle between → ∧ ∧ ∧ → ∧ ∧ ∧
→ →3 9. I f a = i + 4 j + 2 k, b = 3i − 2 j + 7k and
vectors a and b . → ∧ ∧ ∧ →
→ →
c = 2 i − j + 4 k . Find a vector d which is perpendicular
→ →
4. Vectors a and b are such that a = 3, b = 2 and → → → →
to both a and b and c ⋅ d = 18.
→ →
( ) →3 →
a × b is a unit vector. Find the angle between a and b .  [V. Imp.] [A.I. 2010, 2012]
[A.I. 2010] → ∧ ∧ → ∧ ∧ ∧ →
10. If a = 3 i − j and b = 2 i + j − 3 k then express b in
IV. Short Answer Type Questions-II → → → → → → →
→ ∧ ∧ ∧ → ∧ ∧ ∧ the form of b = b 1 + b 2 where b 1 || a and b 2 ⊥ a .
1. If a = 3 i + 2 j + 5 k and b = 4 i + 3 j + 2 k, find:  [V. Imp.] [A.I. 2013 (C)]

( )

→ → → → →
(i) a × b and show that (ii) a ⊥ a × b 11. Find a vector p which is perpendicular to both
→ ∧ ∧ ∧ → ∧ ∧ ∧ → →

(iii) b ⊥ a × b ( → →
) α = 4 i + 5 j − k and β = i − 4 j + 5 k and p ⋅ q = 21
→ ∧ ∧ ∧
→ ∧ ∧ ∧ → ∧ ∧ ∧
where q = 3 i + j − k . [A.I. 2014]
2. If a = i − j + 2 k and b = 2 i + j − k, → ∧ ∧ ∧
12. If r = x i + y j + z k find r × i ⋅ r × j + xy . ( → ∧
)( → ∧
)

(
find r = 2 a − b × a + 2 b .
→ →
) ( → →
) [Delhi 2015]

Answers 10.5
 
iˆ ˆj kˆ 2. (c) | a × b | = | 3iˆ − ˆj − 8kˆ | = 9 + 1 + 64 = 74
   
I. 1. a × b = 2 2 − 1 = iˆ(4 − 3) − ˆj (4 + 6) + kˆ ( − 6 − 12) |a ×b| 74
\ sin q =   =
6 −3 2 | a ||b | 9 +1+1 4 + 4 +1
= i − 10 ˆj − 18kˆ
74 74 74
Hence, (c) is the correct answer. = = =
3 11 99 99

Vector Algebra 329


E:\AMIT_WORKS\Exam_Guru\EG_Mathematics-12_(working_02-06-2022)\EG_Mathematics-12_working\Open_Files\Chap_10\Chap_10
\ 17-Aug-2022 Amit Proof-3 Reader’s Sign _______________________ Date __________

 
3. Leta = 3iˆ + 2 ˆj − kˆ, b = 12iˆ + 5 ˆj − 5kˆ → → → → → → → →
and use b × a = − a × b, c × b = − b × c
iˆ ˆj kˆ → → → →
  and a × c = − c × a .
a × b = 3 2 − 1
→ →
12 5 − 5 a×b 5
3. θ = π ; Hint: sin θ = → → ⇒ sin θ = =1
= i ( − 10 + 5) − ˆj ( − 15 + 12) + kˆ(15 − 24)
ˆ 6 3× 10 2
a b 3
= − 5iˆ + 3 ˆj − 9kˆ
  4. θ = π ,
|a ×b| 25 + 9 + 81
⇒ sin q =   = 4
| a| | b | 9 + 4 + 1 144 + 25 + 25 → →
Hint: a × b = 1 ⇒ 3 ×
2 sin θ = 1
115 3
=
14 194 → → ∧ ∧ ∧
Hence, (a) is the correct answer. IV. 1. (i) a × b = − 11 i + 14 j + k
   
4. Since (a × b ) 2 + (a ⋅ b ) 2 = | a |2 | b |2

2
 2
\ 144 = (4) | b | ⇒ | b | = 144 ÷ 16 = 9 ⇒ | b | = 3
 2
 →
( ) →
(ii) a ⋅ a × b = 0 ⇒ a ⊥ a × b( )
→ → → →

b ⋅(a × b) = 0 ⇒ b ⊥ (a × b)
Hence, (c) is the correct answer. → → → → → →
     
(iii)
5. | a × b |2 = | a |2 | b |2 sin 2 θ = a 2b 2 sin 2 θ [a = | a |, b = | b |]
    →
(a ⋅ b ) 2 = | a |2 | b |2 cos 2 θ = a 2b 2 cos 2 θ ∧ ∧ ∧
2. r = − 5  i − 5 j − 3k
     
| a × b |2 + (a ⋅ b ) 2 a 2b 2 sin 2 θ + a 2b 2 cos 2 θ
\ = → → → → → →
2 a 2b 2 2 a 2b 2
3. Hint: AB = b − a and AC = c − a .
a 2b 2 (sin 2 θ + cos 2 θ) 1 → →
= = Since AB and AC are collinear, therefore work out:
2 a 2b 2 2
Hence, (b) is the correct answer.

→ → →
AB × AC = 0 or b − a × c − a = 0 ( → →
) ( → →
) →

II. 1. λ = 3 and µ = 27 ; Expand it.


2

( ) ( )
→ → ∧ ∧ ∧ → ∧
Hint: a × b = (6µ − 27λ) i − (2µ − 27) j + (2λ − 6) k = 0 . 1 ∧ ∧ ∧ →
7 ∧ ∧ ∧
4. (i) r = ± 8 i − j − 4k (ii) p = ± 8 i − j − 4k
Now solve: 6m – 27l = 0, 2m –27 = 0 and 2l –6 = 0. 9 9

(
2. Hint: a − b × a + b
→ →
) ( → →
) ∧
5. r = ±
1 ∧i − ∧j + 2 k∧ ( → →
. Find AB and AC )
→ →  → → → → → 6
= a×a+a×b −b×a −b ×b
→ →
→ → → → → → ∧
AB × AC
=0+a×b+a×b−0 Hint: r =

→ →
→ → → → AB AC
[Q − b × a = a × b ]

3. 1 ,

( )

6. r = 1 2 i − 2 j − k ;
∧ ∧

( ) ( ) ( )
∧ ∧ ∧ ∧ ∧ ∧ ∧ ∧ ∧ 3

Hint: Find ( a + b ) and ( a − b ) .


Hint: i ⋅ j × k + j ⋅ i × k + k ⋅ i × j
→ → → →

= i ⋅ i + j ⋅ (− j ) + k ⋅ (k ) = 1 − 1 + 1 = 1
∧ ∧ ∧ ∧ ∧ ∧

Then work out r = ( a + b ) × ( a − b ) and r =



→ →
r . → → → ∧

4. θ = π →
r
4 ∧ ∧ 2
→ → → → ∧ ∧
Hint: a ⋅ b = a × b 7. a− b = 1 + 1 – 2 a .b
⇒ ab cos q = ab sin q ⇒ tan q = 1. = 2 – 2 cos q
5. 1 = 2 (1 – cos q)
( ) ∧ ∧ ∧
Hint: i × j ⋅ k + i ⋅ j = k ⋅ k + i ⋅ j = 1 + 0 = 1

∧ ∧ ∧ ∧ ∧ ∧




a−b
∧ 2
θ
= 4 sin2
6. − 1 2
θ
( )
∧ ∧
→ →
III. 1. Hint: a × b = a b sin θ and a ⋅ b tan θ
→ → → →
⇒ a − b = 2 sin
2
→ → → → θ 1 ∧ ∧
= a b cos θ tan θ = a b sin θ
⇒ sin = a− b
2 2
2. Hint: Expand

( → →
a× b+c +b× c+a +c× a+b =0 ) →
( → →
) →
( → →
) → 8. Hint: Prove that: a − d × b − c = 0 . ( → →
) ( → →
) →

Using given conditions.

330 Mathematics–12
E:\AMIT_WORKS\Exam_Guru\EG_Mathematics-12_(working_02-06-2022)\EG_Mathematics-12_working\Open_Files\Chap_10\Chap_10
\ 22-Aug-2022 Amit Proof-3 Reader’s Sign _______________________ Date __________

→ ∧ ∧ ∧ → → → →
9. d = 64 i − 2 j − 28 k Now b2 ⊥ a ∴ b2 ⋅ a = 0

Hint: Let d = λ a × b


( → →
) 3(2 – 3 l) – 1(1 + l) + 0(–3) = 0

⇒ 6 − 9λ − 1 − λ = 0 ⇒ − 10λ = − 5 ⇒ λ = 1


( ∧ ∧
d = λ 32 i − j − 14 k and c ⋅ d = 18

) → →
2
→ ∧ ∧ ∧

⇒ l(64 + 1 – 56) = 18 11. p = 7 i − 7 j − 7k

⇒ l=2 → → ∧ ∧ ∧
Hint: α × β = 21 i − 21 j − 21k,
() () () ()
→ ∧ ∧ → ∧ ∧ ∧
10. b1 = 3 i − 1 j, b2 = 1 i + 3 j − (3) k → ∧ ∧ ∧
2 2 2 2 and q = 3 i + j − k

( ) ( )
→ ∧ ∧ → ∧ ∧ ∧
→ → → ∧ ∧ ∧
Hint: a = 3 i − j, b = 2 i + j − 3 k
p = λ α × β = 21λ i − j − k and
→ → → → → → → → →
b = b1 + b2 and b2 ⊥ a . Since b1 || a, p ⋅ q = 21λ (3 − 1 + 1) = 21

→ ∧ ∧
\ Take b1 = 3λ i − λ j
⇒ λ=1
3
→ → → ∧ ∧ ∧
b2 = b − b1 = (2 − 3λ) i + (1 + λ) j − 3 k 12. 0

Case Based Questions


1. Solar Panels have to be instralled carefully so that the tilt
of the roof, and the direction to the sun, produce the largest (iii) 15 unit, 82 + 62 = 64 + 36 = 100 = 10 unit
  
possible electrical power in the solar panels. (iv) N = A × B
A surveyor uses his instrument to determine the coordinates i j k
of the four corners of a roof
N = 15 0 0 = −15(6 j − 8k ) = −90 j + 120k ; − 90, 120

where solar panels are to
be mounted. In the picture, 0 8 6
suppose the points are 2
(v) ( −90) + 120 = 8100 + 14400 = 22500 = 150
2
labelled counter clockwise
from the roof corner nearest to Answer of second part
the camera in units of metres  1 6 1 
F = 910  i − j + k  = 455i − 780 j + 130k
P1 (6, 8, 4), P2 (21, 8, 4), P3 (21, 16, 10) and P4 (6, 16, 10). 2 7 7 

(i) What are the components to the two edge vectors The dot product is just
   
defined by A = PV of P2 – PV of P1 and B = PV of F . N = 455(0) − 780( −90) + 130(120) = 85,800 watts.
P4 – PV of P1? (where PV stands for position vector).
2. A plane started from
(ii) Write the vector in standard notation with i, j and k airport O with a
i, j and k are the unit vectors along the three velocity of 120 m/s North
(where Air Speed
towards east. Air is B
axes). 50 m/s
  blowing at a velocity
(iii) What are the magnitudes of the vectors A and B and of 50 m/s towards the O A
in what units? north as shown in the Plane Speed 120 m/s East

(iv) What are the components to the vector N , perpendicular figure. The plane travelled 1 hr in OA direction with the
 
to A and B and the surface of the roof? resultant velocity. From A and B travelled 1 hr with keeping
 velocity of 120 m/s and finally landed at B.
(v) What is the magnitude of N and its units? The sun is
1  6 1  (i) What is the resultant velocity from O to A?
located along the unit vector S = i − j + k . If the
2 7  7 (a) 100 m/s (b) 130 m/s
flow of solar energy is given by the vector F = 910 S in (c) 120 m/s (d) 170 m/s
units of watts/metre2, what is the dot product of vectors
  (ii) What is the direction of travel of plane O to A with east
F with N , and the units for this quantity?
 5  12 
Ans. (i) 15, 0, 0 : 0, 8, 6 (a) tan −1   (b) tan −1  
 12   3
(ii) 15i + 0 j + 0k ; 0i + 8 j + 6k
(c) 40° (d) 30°

Vector Algebra 331


E:\AMIT_WORKS\Exam_Guru\EG_Mathematics-12_(working_02-06-2022)\EG_Mathematics-12_working\Open_Files\Chap_10\Chap_10
\ 19-Aug-2022 Amit Proof-3 Reader’s Sign _______________________ Date __________


(iii) What is the total displacement from O to A?
(i) Which team will win the game?
(a) 500 km (b) 468 km (a) Team B
(c) 432 km (d) 400 km (b) Team A

(iv) What is the resultant velocity from A to B? (c) Team C
(a) 120 m/s (b) 70 m/s (d) No one
(c) 170 m/s (d) 200 m/s
(ii) What is the magnitude of the teams combine force?

(v) What is the displacement from A to B? (a) 7 KN
(a) 550 km (b) 432 km (b) 1.4 KN
(c) 600 km (d) 612 km (c) 1.5 KN
 5 (d) 2 KN
Ans. (i) (b) 130 m/s (ii) (a) tan −1  
 12 
(iii) In which direction approx. the ring getting pulls:

(iii) (b) 468 km (iv) (c) 170 m/s (a) 2.0 radian

(v) (d) 612 km (b) 2.5 radian
3. Three friends Ganesh, Dinesh and Ramesh went for playing (c) 2.4 radian
a Tug of war game. Team A, B and C belong to Ganesh, (d) 3 radian
Dinesh and Ramesh respectively.
(iv) What is the magnitude of the force of Team B?
(a) 2 6 KN
p/3
B (b) 6 KN
 
a=
(c) 2 KN
b=
−2i + 4 j −4i + 0 j

(d) 2 6 KN
A

(v) How many KN force is applied by Team A?
p c=

−3i − 3 j 2 (a) 5 KN
π
C 3 (b) 4 KN
(c) 2 KN

(d) 16 KN
Teams A, B, C have attached a rope to a metal ring and
is trying to pull the ring into their own area (team areas Ans. (i) (a) Team B
shown here). (ii) (b) 1.4 KN
Team A pulls with F = – 4i + 0 j KN
1
(iii) (c) 2.4 radian
Team B pulls with F2 = −2i + 4 j KN (iv) (a) 2 6 KN

Team C pulls with F3 = −3i − 3 j KN


(v) (b) 4 KN

Author’s Comments
Questions based on following types are very important for Exams. So, students are advised to revise them thoroughly.
1. Questions based on the dot product, angle between two vectors, projection of a vector on another vector, condition of perpendicularity of
two vectors.
2. To find a unit vector perpendicular to two given vectors. (Most Imp.)

332 Mathematics–12
E:\AMIT_WORKS\Exam_Guru\EG_Mathematics-12_(working_02-06-2022)\EG_Mathematics-12_working\Open_Files\Chap_10\Chap_10
\ 17-Aug-2022 Amit Proof-3 Reader’s Sign _______________________ Date __________

IMPORTANT FORMULAE
→ ∧ ∧ ∧
1. Position Vector of a Point P(x, y, z): p = x i + y j + z k and its magnitude p = x2 + y 2 + z 2
2. Vector joining two points:

→ ∧ ∧ ∧
AB = ( x2 − x1) i + ( y2 − y1) j + ( z2 − z1) k .

→ ∧ ∧ ∧
3. (a) Direction ratios of a vector r = x i + y j + z k are x, y, z.
→ ∧ ∧ ∧
x y z
(b) Direction cosines of a vector r = x i + y j + z k are: , , where r = x2 + y 2 + z 2 .
r r r
4. Section formula for internal division of two position vectors:
→ →

mb +na
Internal division: r =

m+n
→ →

mb −na
External division: r =

m−n
→ →

a+b
Mid-point position vector: r =

2
→ → → →
5. Dot or scalar product of two vectors: a ⋅ b = a b cos θ = ab cos θ. Dot product is a scalar number.
→ → → → ∧
6. Cross product of two vectors: a × b =  a b sin θ  n
 
 → → →  → → →
Here  a × b  ⊥ a and  a × b  ⊥ b
   
→ ∧ ∧ ∧ → ∧ ∧ ∧
7. If a = a1 i + a2 j + a3 k and b = b1 i + b2 j + b3 k then
∧ ∧ ∧
i j k
→ → → →
a ⋅ b = a1a2 + b1b2 + c1c2
and a ×b = a1 a2 a3
b1 b2 b3
→ →
→ → a⋅b
8. Angle between two vectors: If θ is the angle between two vectors a and b then cos θ = → →
,0≤θ≤π
→ →
→ → a b
a⋅b
9. Projection of a on b = →
b

COMMON ERRORS
ERRORS CORRECTIONS
(i) Do not put vector sign. →
(i) If ‘a’ is a vector, it is written as a .

(ii) If two points A(x1, y1, z1) and B(x2, y2, z2) are given, students (ii) Students should write the vector as
write → ∧ ∧ ∧
→ ∧ ∧ ∧ AB = ( x2 − x1) i + ( y2 − y1) j + ( z2 − z1) k

AB = ( x − x ) i + ( y1 − y2 ) j + ( z1 − z2 ) k
1 2
→ → → → →

which is wrong. AB = b − a , where a and b are position vectors of



point A and B respectively.

Vector Algebra 333


E:\AMIT_WORKS\Exam_Guru\EG_Mathematics-12_(working_02-06-2022)\EG_Mathematics-12_working\Open_Files\Chap_10\Chap_10
\ 17-Aug-2022 Amit Proof-3 Reader’s Sign _______________________ Date __________

REVISION CHART

Vector Algebra
A vector has direction and magnitude both but scalar has only magnitude.

Magnitude of a vector a is denoted by | a | or a. It is non-negative scalar.

(i) Scalar quantity, (ii) Vector quantity, (iii) Representation of a Vector, (iv) Unit vector of a given vector, (v) Coinitial vectors, (vi) Collinear
vectors, (vii) Equal vectors, (viii) Vectors in same order (ix) Vectors in opposite order, (x) Addition of vectors with triangle law of addition.

Dot Product of Vectors


It can be defined in two ways Geometrically and Algebraically
→ → → →
a ⋅ b = a b cosθ

Properties of Vectors
→ → → →
(i) Vector addition is commutative: a + b = b + a .

(ii) Vector addition is associative:  a + b  + c = a +  b + c  .


→ → → → → →

   
∧ ∧ ∧
(iii) i , j and k are unit vectors along x-axis, y-axis and z-axis.
→ ∧ ∧ ∧
(iv) Position vector of a point P(x, y, z) = OP = x i + y j + z k .

Cross Product of Vectors


The cross product of the two vectors is defined as the vector whose magnitude is equal to product
of magnitudes of two vectors and sine of the angle between them.
→ → → → ∧
a × b = a b sinθ n

334 Mathematics–12
E:\AMIT_WORKS\Exam_Guru\EG_Mathematics-12_(working_02-06-2022)\EG_Mathematics-12_working\Open_Files\Chap_11\Chap_11
\ 18-Aug-2022 Amit Proof-5 Reader’s Sign _______________________ Date __________

11 Three Dimensional Geometry


Topics Covered
11.1 Distance Formula, Section Formula for Internal and External Division, Direction Angle of Line and Collinearity of
Three Points
11.2 Equations of a Line in Space
11.3 Finding Equation of a Line in Cartesian & Vector Form and their Interconversion
11.4 Two Lines in Space
11.5 Angle between two Lines

C hapter map
THREE DIMENSIONAL GEOMETRY

The Concept of Direction Ratios and Direction Cosines

Line

Distance formula, section formula Equations of a line in space Two lines in space
for internal and external divisions,
direction angle of line and
collinearity of three points

Finding equation of a line in Angle between two lines


cartesian and vector form and
their interconversion.

Topic 1. Distance Formula, Section Formula for Internal and External Division,
Direction Angle of Line and Collinearity of Three Points
Distance Formula
The distance formula between two points A(x1, y1, z1) and B(x2,
y2, z2) is given below:

AB = (x2 − x1) 2 + (y2 − y1) 2 + (z2 − z1) 2

or AB = (x1 − x2) 2 + (y1 − y2) 2 + (z1 − z2) 2 Section Formula for Internal Division

Proof: Take two points: A(x1, y1, z1) and B(x2, y2, z2)
Vector Form

 ∧ ∧ ∧ A point P( r ) divides the line segment AB, joining two given
AB = (x2 − x1) i + (y2 − y1) j + (z2 − z1) k → →
points A (r1) and B (r2) in the ratio m : n internally then the section

⇒ AB = (x2 − x1) 2 + (y2 − y1) 2 + (z2 − z1) 2 formula for internal division in vector form:

335
E:\AMIT_WORKS\Exam_Guru\EG_Mathematics-12_(working_02-06-2022)\EG_Mathematics-12_working\Open_Files\Chap_11\Chap_11
\ 19-Aug-2022 Amit Proof-5 Reader’s Sign _______________________ Date __________

mx2 − nx1 my2 − ny1 mz − nz1


x= ;y= ;z= 2
m−n m−n m−n

→ →
→ m r2 + n r1
r =
m+n

If P( r ) is the mid-point position vector of points To Find out Ratio m : n
→ →
A (r1) and B (r2) then: If we are given two points A(x1, y1, z1) and B(x2, y2, z2) and the
point P(x, y, z), which divides the line-segment AB in some ratio,
→ →
→ then we use the following formula, to find m and n.
r1 + r2
r = kx2 + x1 ky + y1 kz + z m
2 x= ;y= 2 ; z = 2 1 Here k =
k +1 k +1 k +1 n
  ∧ ∧ ∧  ∧ ∧ ∧
Note: Here: r =  x i + y j + z k  , r1 = x1 i + y1 j + z1 k, and Z B
 
 ∧ ∧ ∧ C (x2, y2, z2)
r2 = x2 i + y2 j + z2 k
k:1
Cartesian Form A
A point P(x, y, z) divides the line segment AB joining two points (x1, y1, z1)
A(x1, y1, z1) and B(x2, y2, z2) in the ratio m : n internally, then the
section formula for internal division in Cartesian form: O Y
mx2 + nx1 my2 + ny1 mz + nz1 X
x= ;y= ,z= 2
m+n m+n m+n

 kx + x ky + y1 kz2 + z1 
If P(x, y, z) is the mid-point of the line segment AB joining C 2 1 , 2 , 
 k +1 k +1 k +1 
the points A(x1, y1, z1) and B(x2, y2, z2) then:
x1 + x2 y + y2 z + z2 Example 1. The direction cosines of vector (2i + 2 j − k ) are
x= ;y= 1 ,z= 1 .................
2 2 2
 
Solution. Let a = 2i + 2 j − k direction ratios of a are 2, 2, – 1
Section Formula for External Division So, the direction cosines are
Vector Form 2 2 −1 2 2 −1

   , ,  , ,
4 + 4 +1 4 + 4 +1 4 + 4 +1 3 3 3
A point P( r ) divides the line segment AB joining the points
→ → Hence, the direction cosines of the given vector are
A (r1) and B (r2) in the ratio m : n externally then the section 2 2 −1
formula for external division in the vector form: , , .
→ → 3 3 3
→ m r2 − n r1 Example 2. Distance of the point (a, b, g) from y-axis is
r =
m−n (a) b (b) | b |
(c) | b | + | g | (d) α2 + γ 2
Solution. The given point is (a, b, g)
Any point on y-axis = (0, b, 0)
\ Required distance
Cartesian Form = (α − 0) 2 + (β − β) 2 + ( γ − 0) 2
Point P(x, y, z) divides the line segment AB joining two points
A(x1, y1, z1) and B(x2, y2, z2) in the ratio m : n externally, then the = α2 + γ 2
section formula for external division in Cartesian form: Hence, the correct option is (d).

336 Mathematics–12
E:\AMIT_WORKS\Exam_Guru\EG_Mathematics-12_(working_02-06-2022)\EG_Mathematics-12_working\Open_Files\Chap_11\Chap_11
\ 18-Aug-2022 Amit Proof-5 Reader’s Sign _______________________ Date __________

Example 3. If the direction cosines of a line are k, k, k, then ∧ ∧ ∧ ∧ ∧ ∧



(a) k > 0 (b) 0 < k < 1 2(3 i − j + 4 k) − 1(2 i + j + 3 k)
(ii) External division r =
1 −1 2 −1
(c) k = 1 (d) k = or →
∧ ∧ ∧ ∧ ∧ ∧
3 3 6 i − 2 j + 8k − 2 i − j − 3k
⇒ r =
Sol. If l, m, n are the direction cosines of a line, then 1
→ ∧ ∧ ∧
     l2 + m2 + n2 = 1 ⇒ r = (4 i − 3 j + 5 k)
So,    k2 + k2 + k2 = 1 ∧ ∧ ∧
1 \ Point P is (4 i − 3 j + 5 k)
    3k2 = 1  k = ±
3 Example 5. Find the ratio in which a plane x + y + z = 5 divides
Hence, the correct option is (d). the line segment AB where A(1, 2, –3) and B(3, 1, 2). Find the
point of intersection also.
Example 4. A point P divides the line segment AB in the ratio
Solution. Let the point of intersection of line segment AB and
2 : 1 (i) internally and (ii) externally when A(2, 1, 3) and plane x + y + z = 5 be P(x, y, z). This point divides AB in the
B(3, –1, 4). Find the point P. Use (a) Cartesian method and ratio k : 1.
(b) Vector method.

3k + 1 k+2
Solution. Point A(2, 1, 3) and B(3, –1, 4) \ x=,y=
k +1 k +1
(a) Cartesian Method. 2k − 3
2 × 3 +1× 2 6 + 2 8 and z=
(i) Internal division x = = = k +1
2 +1 3 3 Since this point lies on the plane x + y + z = 5
y=
2 × ( −1) + 1 × 1 −2 + 1 −1
2 +1
=
3
=
3
\ ( )( )(
3k + 1
k +1
+
k+2
k +1
+
2k − 3
k +1
= 5 )
2 × 4 + 1 × 3 8 + 3 11 ⇒ 3k + 1 + k + 2 + 2k – 3 = 5k + 5 ⇒ 6k = 5k + 5
z= = =
2 +1 3 3 ⇒ k= 5
\ Point P is , , (
8 −1 11
) ⇒ Ratio is 5 : 1 (internally)
3 3 3
( 2 × 3) − (1 × 2) = 6 − 2 = 4 (
For Point P 3k + 1 , k + 2 , 2k − 3 put k = 5
k +1 k +1 k +1 )
( )
(ii) External division x =
2 −1 1
= P 16 , 7 , 7
 2 × ( −1) − (1 × 1) −2 − 1 6 6 6
y= = = −3
2 −1 1 Direction Angles of a Line
z=
( 2 × 4) − (1 × 3) = 8 − 3 = 5 Suppose a line OP is passing through the origin O. This line makes
2 −1 1 angle α, b and g with the positive direction of x-axis, y-axis and
\ Point P is (4, –3, 5) z-axis respectively. These angles α, b and g are called direction
angles of the line OP.
→ If the given line is not passing through the origin, then imagine
(b) Vector Method: Let the point P be P( r ) another line passing through the origin and parallel to the given
line. Now, whatever angles this line is making with positive
directions of X, Y and Z-axis, will be the direction angles of the
given line.
Direction Cosines of a Line
If α, b and g are the direction angles of a line then cos α, cos b
and cos g are called the direction cosines of the line. Direction
∧ ∧ ∧ ∧ ∧ ∧

2(3 i − j + 4 k) + 1(2 i + j + 3 k) cosines cos α, cos b and cos g are also denoted by l, m and n.
(i) Internal division r =
2 +1
∧ ∧ ∧ ∧ ∧ ∧

6 i − 2 j + 8k + 2 i + j + 3k
⇒ r =
3
∧ ∧ ∧

8 i − j + 11 k
⇒ r =
3
∧ ∧ ∧
1 
⇒ Point P is  8 i − j + 11 k 
3 

Three Dimensional Geometry 337


E:\AMIT_WORKS\Exam_Guru\EG_Mathematics-12_(working_02-06-2022)\EG_Mathematics-12_working\Open_Files\Chap_11\Chap_11
\ 18-Aug-2022 Amit Proof-5 Reader’s Sign _______________________ Date __________

\ l = cos α, Direction Ratios of a Line


m = cos β and If A(x1, y1, z1) and B (x2, y2, z2) are two points on a line, then
n = cos γ a = x2 – x1, b = y2 – y1, c = z2 – z1 are direction ratio of the line AB.

Take a vector r passing through two points A(x1, y1, z1) Example 6. There are two points A(1, –1, 2) and B(3, 4, 5). Find
and B(x2, y2, z2) direction ratios and direction cosines of AB.
→ ∧ ∧ ∧ Solution. A(1, –1, 2) and B(3, 4, 5).
\ r = (x2 − x1) i + (y2 − y1) j + (z2 − z1) k
drs of AB : a, b, c :
where r= (x2 − x1) 2 + (y2 − y1) 2 + (z2 − z1) 2 a = (x2 – x1) = (3 – 1) = 2
→ b = (y2 – y1) = (4 + 1) = 5
Since α is the angle between r and positive direction of
→ ∧ c = (z2 – z1) = (5 – 2) = 3
x-axis or α is the angle between r and i . drs of AB : a, b, c = 2, 5, 3
→ ∧
r ⋅i Now find r= a 2 + b2 + c2
\ l = cos α = → ∧
r i ⇒ r= 4 + 25 + 9 = 38
 ∧ ∧ ∧ ∧ dcs of AB : l, m, n
(x2 − x1) i + (y2 − y1) j + (z2 − z1) k  ⋅ i a 2 b 5 c 3
  l= = ,m = = ,n = =
⇒ l = cos a = → ∧ r 38 r 38 r 38
r i
Collinearity of Three Points
(x2 − x1) x2 − x1 Three points A, B and C are collinear if the direction cosines of
⇒ l = cos α = = AB and BC are same or proportional.
r ×1 r ×1
Example 7. Show that the three points A(1, –1, 2), B(3, 2, 5) and
x2 − x1 C(5, 5, 8) are collinear.
⇒ l = cos α =
r Solution. Three points are A(1, –1, 2), B(3, 2, 5) and C(5, 5, 8)
y2 − y1 z −z drs of AB = a1, b1, c1 = (3 – 1, 2 + 1, 5 – 2)
Similarly m = cos β = and z = cos γ = 2 1 ⇒ a1, b1, c1 = (2, 3, 3)
r r
drs of AC = a2, b2, c2 = (5 – 1, 5 + 1, 8 – 2)
Hence, direction cosines of a vector or a line passing through
two points A(x1 ,y1, z1) and B(x2, y2, z2) are: ⇒ a2, b2, c2 = (4, 6, 6)
a 4 b 6 c 6
Since 2 = = 2, 2 = = 2, 2 = = 2
x2 − x1 y − y1 z −z a1 2 b1 3 c1 3
l = cos α = , m = cos β = 2 , n = cos γ = 2 1
r r r a2 b2 c2
⇒ = = =2
where r = (x2 − x1) 2 + (y2 − y1) 2 + (z2 − z1) 2 a1 b1 c1
⇒ Three points A, B and C are collinear.

EXERCISE 11.1
I. Multiple Choice Questions (MCQs) respectively. It makes an acute angle q with the positive
Choose the correct answer from the given options. z-axis, then q equals to:
1. If a line makes angles a, b, g with the co-ordinate axes, (a) 30° (b) 45° (c) 60° (d) 75°
then
4. If a line in the space makes angles a, b, g with the co-
(a) cos 2a + cos 2b + cos 2g – 1 = 0
ordinate axes, then cos 2a + cos 2b + cos 2g + sin2 a +
(b) cos 2a + cos 2b + cos 2g – 2 = 0
sin2 b + sin2 g is equal to
(c) cos 2a + cos 2b + cos 2g + 1 = 0
(d) cos 2a + cos 2b + cos 2g + 2 = 0 (a) – 1 (b) 0 (c) 1 (d) 2

1 1  II. Very Short Answer Type Questions


2. If  , , n  are the direction cosines of a line, then the
2 3  1. Find the distance of a point P(2, 3, 4) from y-axis.
value of n is: [Delhi 2010 (C)]
23 23 2 3 III. Short Answer Type Questions-I
(a) (b) (c) (d)
6 6 3 2 1. If a line makes angle 90°, 60° and θ with x, y and z-axis
respectively, where θ is an acute angle find θ.
3. A line AB in three-dimensional space makes angles 45°
and 120° with the positive x-axis and the positive y-axis [Delhi 2015, 2017]

338 Mathematics–12
E:\AMIT_WORKS\Exam_Guru\EG_Mathematics-12_(working_02-06-2022)\EG_Mathematics-12_working\Open_Files\Chap_11\Chap_11
\ 18-Aug-2022 Amit Proof-5 Reader’s Sign _______________________ Date __________

2. If a line passes through two points A(1, – 1, 1) and 6. The x-coordinate of a point on the line joining the point
B(2, 3, 5), find the direction ratios and direction cosines P(2, 2, 1) and Q(5, 1, –2) is 4. Find its z-coordinate.
of the line AB. [A.I. 2017]
3. If a line makes angle 45°, 60° and 60° with the positive
IV. Long Answer Type Questions
direction of x, y and z-axis. Find the direction cosines and
1. If cos α, cos b and cos g are the direction cosines of a line,
direction ratios of the line.
then show that: [CBSE 2022]
4. Show that the three points A(2, 1, 3), B(3, 4, 5) and
(i) cos2 α + cos2 b + cos2 g = 1 or l2 + m2 + n2 = 1
C(7, 16, 13) are collinear.
(ii) sin2 α + sin2 b + sin2 g = 2
5. Find the value of λ and µ if three points A(1, 1, 2),
B(3, 2, 5) and C(λ, µ, 17) are collinear. [A.I. 2004] (iii) cos 2α + cos 2b + cos 2g = – 1

Answers 11.1
I.1. (c) Since a line makes an angle α, b, g with co-ordinate axes, ⇒ cos q = cos 60°, q = 60°.
we have Hence, (c) is the correct answer.
cos2 α + cos2 b + cos2 g = 1 [ l2 + m2 + n2 = 1] 4. (c) As a line in the space makes angles a, b, g with the co-ordinate
⇒ 2 cos2 α + 2 cos2 b + 2 cos2 g = 2 axes, then
⇒ (2 cos2 α – 1) + (2 cos2 b – 1) + (2 cos2 g – 1) = 2 – 3 cos2 a + cos2 b + cos2 g = 1
⇒ cos 2α + cos 2b + cos 2g = – 1 Now cos 2a + cos 2b + cos 2g + sin2 a + sin2 b + sin2 g
or cos 2α + cos 2b + cos 2g + 1 = 0 = (2 cos2 a – 1) + (2 cos2 b – 1) + (2 cos2 g – 1)
Hence, (c) is the correct answer. + (1 – cos2 a) + (1 – cos2 b) + (1 – cos2 g)
1 1 = cos a + cos2 b + cos2 g = 1.
2
2. (a) As  , , n  are the d.c.’s of a line
2 3  Hence, (c) is the correct answer.
2 2 II. 1. Distance = 2 5 ,
1 1 2

\   +  +n = 1 Hint: Distance of a point

 2 3
1 1 P(x1, y1, z1) from y-axis = x12 + z12

⇒ + + n2 = 1
4 9 III. 1. θ = 30°.
1 1 9+ 4 Hint: Use the formula: cos2 a + cos2 b + cos2 g = 1

⇒ n2 = 1 −  +  = 1 −  
 4 9   36  2. drs = 1, 4, 4, dcs = 1 , 4 , 4
33 33 33
= 1 − 13 = 23 Hint: drs : a = x2 – x1, b = y2 – y1, c = z2 – z1
36 36
a b c
dcs : cos α = , cos β = , cos γ =
23 r r r
\ n= .
6
where r = a 2 + b2 + c2
Hence (a) is the correct answer.
3. (c) Let l, m and n be the direction cosines of the line, we have 1 1 1
3. dcs : cos 45°, cos 60°, cos 60° = , ,
2 2 2
1
l = cos 45° = , m = cos 120° drs : 2, 1, 1
2
4. Hint: Find dcs of AB and BC; dcs of AB = dcs of BC.
1
= – cos 60° = – and n = cos q 5. λ = 11, m = 6.
2
Hint: Find dcs of AB and AC, and equate them.

2 2
 1   1 2 [ l 2 + m 2 + n 2 = 1]

\   +  −  + cos θ = 1 6. z = –1.
 2   2
Hint: Take the drs of PQ in proportion.

1 1

⇒ + + cos 2 θ = 1 IV. 1. Hint: Take two points A(x1, y1, z1) and B(x2, y2, z2) on a line,
2 4
1 1 3 1 and r = (x2 − x1) 2 + (y2 − y1) 2 + (z2 − z1) 2

⇒ cos2 q = 1 −  +  =1− =
2 4 4 4
x2 − x1 y − y1
then cos α = , cos β = 2 and
1 r r

\ cos q =
2 z2 − z1
   cos γ =
[ q is an acute angle so cos q is positive] r

Three Dimensional Geometry 339


E:\AMIT_WORKS\Exam_Guru\EG_Mathematics-12_(working_02-06-2022)\EG_Mathematics-12_working\Open_Files\Chap_11\Chap_11
\ 18-Aug-2022 Amit Proof-5 Reader’s Sign _______________________ Date __________

Now put these values (i). (iii) Put cos 2a = 2 cos2 a – 1


In (ii) write sin2 a = 1 – cos2 a, sin2 b = 1 – cos2 b cos 2b = 2 cos2 b – 1
and     sin2 g = 1 – cos2 g cos 2g = 2 cos2 g – 1

Topic 2. Equations of a Line in Space


A line in space can be determined uniquely if: Equation of the required line in cartesian form
(i) direction ratios of the line and one point on it are given. x − x1 y − y1 z − z1
= =
(ii) two points on the line are given. a b c
Equations of a Line when its Direction Ratios and ⇒
x −1
=
y +1 z − 2
=
one Point on it are given 3 2 4

Vector Form Vector Form: Position vector of fixed point A on the required

()
line

( )

If A r1 is a fixed point and P r is any variable point on a line → ∧ ∧ ∧
r1 = x1 i + y1 j + z1 k = i − j + 2 k
∧ ∧ ∧

AP and m is any vector parallel to the line AP then equation of Direction ratio vector of the required line
the line AP in vector form: → ∧ ∧ ∧
→ → → m = a i +b j+ck
r = r1 + λ m → ∧ ∧ ∧
m = 3i + 2 j + 4k
Equation of the line in vector form:
→ → →
r = r1 + λ m
→ ∧ ∧ ∧ ∧ ∧ ∧
⇒    r = (i − j + 2 k) + λ(3 i + 2 j + 4 k)
If (x1, y1, z1) is the fixed point, P(x, y, z) is the variable point and
a, b, c are direction ratios of the line, then Equations of a Line when Two Points on it are
→ ∧ ∧ ∧ → ∧ ∧ ∧ given
r1 = x1 i + y1 j + z1 k, r = x i + y j + z k Let A(x1, y1, z1) and B(x2, y2, z2) are two points of a line AB.
→ ∧ ∧ ∧ Then direction ratios of the line AB are:
and m = a i + b j + ck a = (x2 – x1), b = (y2 – y1), c = (z2 – z1).
then the equation of the line AP can be written as:
Take A(x1, y1, z1), or B(x2, y2, z2) as fixed point and P(x, y, z) as
∧ ∧ ∧
 ∧ ∧ ∧  ∧ ∧ ∧
a variable point on it. Then the equations of line AB are:
x i + y j + z k =  x1 i + y1 j+ z1 k  + λ  a i + b j + c k 
    x − x1 y − y1 z − z1 x − x2 y − y2 z − z2
= = or = =
→ a b c a b c
 ∧ ∧ ∧  ∧ ∧ ∧
or r =  x1 i + y1 j+ z1 k  + λ  a i + b j + c k 
    Example 2. Find the equation of a line in cartesian and vector
forms passing through two points A(2, 1, 3) and B(4, –1, 1).
→ ∧ ∧ ∧
Here, m = a i + b j + c k can be called as a direction ratio Solution. Given points are:
vector of the line AB. A(x1, y1, z1) = A(2, 1, 3)
and B(x2, y2, z2) = B(4, –1, 1)
Cartesian Form Direction ratios of the line
If A(x1, y1, z1) is a fixed point and P(x, y, z) is a variable point a, b, c = x2 – x1, y2 – y1, z2 – z1
on a line AB and a, b, c are direction ratios of the line AB. Then = (4 – 2, –1 – 1, 1 – 3) = 2, –2, –2
equations of the line AB in Cartesian form:
or a, b, c = 1, –1, –1 (since drs are proportionate values)
x − x1 y − y1 z − z1
= = Cartesian form: Equation of the line in cartesian form
a b c
x − x1 y − y1 z − z1
Example 1. Find the equation of a line passing through a point = =
a b c
A(1, –1, 2) and its direction ratios are 3, 2, 4. Write the equation
in cartesian and vector form. x − 2 y −1 z − 3
⇒ = =
Solution. Cartesian Form: Point of the required line is 1 −1 −1
A(x1, y1, z1) = A(1, –1, 2) and direction ratios of the required x − x2 y − y2 z − z 2
line are: a, b, c = 3, 2, 4 or = =
a b c

340 Mathematics–12
E:\AMIT_WORKS\Exam_Guru\EG_Mathematics-12_(working_02-06-2022)\EG_Mathematics-12_working\Open_Files\Chap_11\Chap_11
\ 18-Aug-2022 Amit Proof-5 Reader’s Sign _______________________ Date __________

Equation of the required line in vector form:


y +1 z −1
⇒ x−4 = = → → →
1 −1 −1 r = r1 + λ m
These two equations are same.
→ ∧ ∧ ∧ ∧ ∧ ∧
Vector Form: P.V. of fixed point on the required line ⇒ r = (2 i + j + 3 k) + λ(i − j − k)
→ ∧ ∧ ∧ → ∧ ∧ ∧
→ →
r1 = 2 i + j + 3 k or r2 = 4 i − j + k →
or r = r2 + λ m
direction ratio vector of the required line
→ ∧ ∧ ∧ ∧ ∧ ∧
→ ∧ ∧ ∧ → ∧ ∧ ∧
⇒ r = (4 i − j + k) + λ(i − j − k)
m = a i +b j+ck ⇒ m = i − j−k

EXERCISE 11.2
I. Multiple Choice Questions (MCQs) 2. Write the equation of a line passing through the point
Choose the correct answer from the given options. A(α, β, γ) and parallel to z-axis.[A.I. 2014 (C)]
1. The equation of the line through (1, – 1, 2) and parallel to III. Short Answer Type Questions-I
the line joining (– 1, 0, 1) and (2, 3, – 1) is: 1. Vertices of a ∆ABC are: A(1, 1, 3), B(2, 3, 4) and
x +1 y − 3 z + 2 x −1 y +1 z − 2 C(4, 5, – 2). Find the equations of the median of AD in
(a) = = (b) = =
−1 0 −1 3 3 −2 Cartesian form, D is mid-point of BC.
x +1 y −1 z + 2 x −1 y +1 z − 2 2. Find the equation of (i) x-axis, (ii) y-axis, (iii) z-axis in
(c) = = (d) = =
2 3 −1 3 3 0 Cartesian and vector form.
2. The cartesian equations of a line are 3. Find the equations of lines passing through a point
6x – 2 = 3y + 1 = 2z + 2 A(2, 1, 3) and parallel to (i) x-axis, (ii) y-axis and (iii) z-axis
The d-ratios of the line are: in Cartesian and vector form.
(a) 1, 2, 3 (b) 2, 3, 1 IV. Short Answer Type Question-II
(c) 3, 2, 1 (d) None of these 1. Mid-points of the sides AB, AC and BC of ∆ABC are
3. The equation of straight line passing through the point P(2, 1, 7), Q(– 1, 3, – 5) and R(3, 5, – 1) respectively. Find
(a, b, c) and parallel to z-axis is:
the equations of the side BC in Cartesian form.
(a) x − a = y − b = z − c (b) x − a = y − b = z − c 2. Three vertices of a parallelogram ABCD are A(2, 1, 3),
1 1 0 0 1 1
B(6, 7, 5) and C(4, 9, 11). Find the fourth vertex D and
x−a y−b z −c
(c) = = (d) x − a = y − b = z − c write the equation of the diagonal BD in vector form.
1 0 0 0 0 1 [A.I. 2005]
II. Very Short Answer Type Questions 3. Find the equation of a line passing through a point
1. Find the equation of a line which is parallel to a vector A(3, 1, 4) and parallel to a line passing through two points
 ∧ ∧ ∧ B (1, 1, – 1) and C (5, 2, 7) in (i) Cartesian form and
 2 i − j + 3 k  and passes through a point A(5, – 2, 4). (ii) vector form.
[A.I. 2007]

Answers 11.2
I. 1. (b) Since the required line is parallel to the line joining (– 1, 1 1 1
0, 1) and (2, 3, – 1) which is d-ratios of the line are
, , or 1, 2, 3
6 3 2
x +1 y − 0 z −1 y z −1
= = or x + 1 = = Hence, (a) is the correct answer.
2 +1 3 − 0 −1 − 1 3 3 −2 3. (d) Since the required line which passes through the point
Its d-ratios will be same as d-ratios of the given line 3, 3, – 2. (a, b, c) is parallel to z-axis whose d-cosines are 0, 0, 1, so the
x−a y −b z −c
Hence, the required equation of the line is x − 1 = y + 1 = z − 2 equation of the required line is = − .
3 3 −2 0 0 1
Hence, (b) is the correct answer.
Hence, (d) is the correct answer.
2. (a) The cartesian equations of the line are
→  ∧ ∧ ∧  ∧ ∧ ∧
6x – 2 = 3y + 1 = 2z – 2 II. 1. r =  5 i − 2 j + 4 k  + λ  2 i − j + 3 k 
1 1    
x− y+
3 = 3 = z −1
or
1 1 1 or x − 5 = y + 2 = z − 4 .
2 −1 3
6 3 2

Three Dimensional Geometry 341


E:\AMIT_WORKS\Exam_Guru\EG_Mathematics-12_(working_02-06-2022)\EG_Mathematics-12_working\Open_Files\Chap_11\Chap_11
\ 18-Aug-2022 Amit Proof-5 Reader’s Sign _______________________ Date __________

2. drs of z-axis are (0, 0, 1) and fixed point on the line is (a, b, g) x − 2 y −1 z − 3
(iii) = =
0 0 1
\ x −α = y −β = z − γ.

0 0 1 → ∧ ∧ ∧ ∧
or r =  2 i + j + 3 k  + λ  k 
→ ∧ ∧ ∧ ∧    
or r =  α i + β j + γ k  + λ k
  x − 3 y − 5 z +1
IV. 1. = =
3 −2 12
x −1 y −1 z − 3 x − 3 y − 4 z −1
III. 1. = = or = = →  ∧ ∧ ∧
2 3 −2 2 3 −2  ∧ ∧ ∧
or r =  3 i + 5 j − k  + λ  3 i − 2 j + 12 k 
2. drs of x-axis: 1, 0, 0; drs of y-axis: 0, 1, 0;    

drs of z-axis: 0, 0, 1 and fixed point (0, 0, 0).


x = y = z or → ∧
(i) eqn. of x-axis: r = λ(i )
1 0 0
x = y = z or → ∧
r = λ( j)
(ii) eqn. of y-axis:
0 1 0
→ ∧
y
(iii) eq. of z-axis: x = = z or r = λ(k) drs of BC = drs of PQ
0 0 1 Hint: BC is parallel to PQ.

3. drs of x-axis = 1, 0, 0 2. D (0, 3, 9).
drs of y-axis = 0, 1, 0 Hint: Mid-point of AC = mid-point of BD.

drs of z-axis = 0, 0, 1 Equations of BD:
x − 2 y −1 z − 3  ∧→ ∧ ∧  ∧ ∧ ∧
(i) = = r =  6 i + 7 j + 5k + λ  3 i + 2 j − 2k
1 0 0
   
→ ∧ ∧ ∧ ∧
or r =  2 i + j + 3 k  + λ  i 
→ ∧ ∧ ∧ ∧ ∧
or r =  3 j + 9 k  + λ  3 i + 2 j − 2 k 
       

x − 2 y −1 z − 3 x − 3 y −1 z − 4
(ii) = = 3. = =
0 1 0 4 1 8
→  ∧ ∧ ∧ →  ∧ ∧ ∧  ∧ ∧ ∧
 ∧ or r =  3 i + j + 4 k  + λ  4 i + j + 8 k 
or r =  2 i + j + 3 k  + λ  j 
       

Topic 3. Finding Equation of a Line in Cartesian & Vector Form and


their Interconversion
To find the Fixed Point and Direction Ratios of a Hence, the vector equation of the given line is
Line From it’s Equations 
r = (5i − 4 j + 6k ) + λ(3i + 7 j + 2k )
Cartesian form
x − x1 y − y1 z − z1 x−5 y+4
Equations of a Line: = = . Here, (x1, y1, z1) Example 2. The vector equation of the line =
a b c 3 7
is the fixed point and a, b, c are direction ratios of the line. z−6 
= is r = 5i − 4 j + 6k + λ(3i + 7 j + 2k ) .
x−5 y+4 2
Example 1. The vector equation of the line =
3 7 Solution. The Cartesian form of the equation is
z−6
= is ............. x−5 y+4 z−6
2 = = =λ
3 7 2
Solution. The given equation is
x−5 \ Here x1 = 5, y1 = – 4, z1 = 6, a = 3, b = 7, c = 2
y+4 z−6
= = 
3 7 2 So, the vector equation is r
 
Here a = (5i − 4 j + 6k ) and b = (3i + 7 j + 2k )
 
= (5i − 4 j + 6k ) + λ(3i + 7 j + 2k )
  
Equation of the line is r = a + b λ Hence, the given statement is ‘true’.

342 Mathematics–12
E:\AMIT_WORKS\Exam_Guru\EG_Mathematics-12_(working_02-06-2022)\EG_Mathematics-12_working\Open_Files\Chap_11\Chap_11
\ 18-Aug-2022 Amit Proof-5 Reader’s Sign _______________________ Date __________

Example 3. Find the fixed point and direction ratios of the line: Conversion of Cartesian form of equations of a
3x + 2 = 5 – 2y = 3z. line into vector form and vector form of a line into
Solution. The given equation is: 3x + 2 = 5 – 2y = 3z Cartesian form
First write it in standard form Example 5. Change the equations of the line in vector form:
x − x1 y − y1 z − z1 x + 3 3 − 2y 5z + 4
= = =
= 2 3 1
a b c
x + 3 3 − 2y 5z + 4
( ) ( )
2 5 Solution. Write the equations = = in
3 x + 3 = − 2 y − = 3(z − 0) 2 3 1
2 standard form:

x− − 2
3( )y− 5 z−0
2=
x − (− 3)
=
( )
−2 y − 3
2 = 

5 z − 
− 4 
 5  
⇒ = 2 3 1
1 −1 1
3 2 3
x − (− 3) y−3 z− −
= − 32 =
4
5( )
( )

⇒ Fixed point − 2 , 5 , 0 2 1
3 2 2 5
 3 − 4
⇒ Fixed point is  − 3, ,  and drs are:
Direction ratios : 1 , − 1 , 1 or 2, – 3, 2  2 5 
3 2 3 −3 1
2, , or 20, – 15, 2.
Vector Form 2 5

∴ Equation of the line in vector form:
 ∧ ∧ ∧  ∧ ∧ ∧
Equation of a Line: r =  x1 i + y1 j + z1 k  + λ  a i + b j + c k  →  ∧
3 ∧ 4 ∧  ∧ ∧ ∧
    r =  − 3 i + j − k  + λ  20 i − 15 j + 2 k 
 2 5   
Here (x1, y1, z1) is the fixed point and a, b, c are direction ratios Example 6. Change into Cartesian form of equations of line:
of the line. → ∧ ∧ ∧
→ ∧ ∧ r = (3λ + 2) i + (5 − 2λ) j + (1 − 4λ) k .
Example 4. From the equation of a line r = (3λ + 2) i + (λ − 3) j + Solution. Given equation is:
∧ → ∧ ∧ ∧
(5λ − 2) k , find fixed point and direction ratios of the line. r = (3λ + 2) i + (5 − 2λ) j + (1 − 4λ) k
Write it in a standard form:
Solution. Write the equation in standard form as: →  ∧ ∧ ∧  ∧ ∧ ∧
r =  2 i + 5 j + k + λ  3 i − 2 j − 4 k
→  ∧ ∧ ∧  ∧ ∧ ∧    
r =  2 i − 3 j − 2 k + λ  3 i + j + 5 k ⇒ Fixed point is (2, 5, 1) and drs are: 3, – 2, – 4
   
∴ Equation of the line in Cartesian form
Here, fixed point is (2, – 3, – 2) and direction ratios are : x−2 y − 5 z −1
= =
3, 1, 5. 3 −2 −4

EXERCISE 11.3
I. Multiple Choice Questions (MCQs) x y z x y z
(c) = = (d) = =
Choose the correct answer from the given options. −1 5 3 4 −5 −2
x − 3 y − 2 z −1
1. The straight line = = is: II. Very Short Answer Type Questions
3 1 0
(a) parallel to x-axis (b) parallel to y-axis 1. Find the direction cosines of a line parallel to the line:
2x − 5 y + 4 6 − z [Delhi 2008 (C), 2009 (C)]
(c) parallel to z-axis (d) perpendicular to z-axis = =
4 3 6
2. The points (1, 2, 3), (4, 0, 4), (– 2, 4, 2), (7, – 2, 5) are:
(a) collinear 2. The equations of a line are: 5x – 3 = 15y + 7 = 3 – 10z.
Write the direction cosines of the line. [A.I. 2015]
(b) are the vertices of a square
(c) are the vertices of a rectangle 3. Find the vector equation of the line passing through
(d) None of these the point A(1, 2, –1) and parallel to the line:
5x – 25 = 14 – 7y = 35 z.[Delhi 2017]
3. The straight line passing through the point (4, – 5, – 2) and
(– 1, 5, 3) is: III. Short Answer Type Questions-I
x −4 y + 5 z + 2 x +1 y − 5 z − 3 1. Find the equation of a line which passes through a point
(a) = = (b) = =
1 −2 −1 1 2 −1 A(1, 2, 3) and parallel to the line:

Three Dimensional Geometry 343


E:\AMIT_WORKS\Exam_Guru\EG_Mathematics-12_(working_02-06-2022)\EG_Mathematics-12_working\Open_Files\Chap_11\Chap_11
\ 18-Aug-2022 Amit Proof-5 Reader’s Sign _______________________ Date __________

− x − 2 y + 3 2z − 6 and passes through a point given on the line:


= =
1 7 3 (2x – 1) = (3y – 2) = (z + 4)
[Delhi 2007, 2012 (C), A.I. 2009 (C), 2013 (C)] IV. Short Answer Type Question-II
2. Find the equation of a line parallel to the line:
1. Write the vector equation of the line:
→ ∧ ∧ ∧  ∧ ∧ ∧
x−5 y+4 6− z
   r =  i − 4 j + 4 k + λ  3 i + 2 j + 4 k   = = .[Delhi 2010, AI 2011]
    3 7 2

Answers 11.3
I. 1. (c) Parallel to z-axis Hint: Fixed point on the required line (1, 2, –1)

2. (a) Collinear Given line : 5x – 25 = 14 – 7y = 35z
3. (a) x − 4 = y + 5 = z + 2 ⇒
x−5
=
y−2
= z
1 −2 −1 1 − 1 1
5 7 35
−6
II. 1. 2 , 3 ,
7 7 7 Drs : 1 , −1 , 1 or 7, −5,1
5 7 35
y+4 6− z
Hint: 2x − 5 = = Now find the equation of the line.
4 3 6
5 x −1 y − 2 z − 3
x− III. 1. = =
⇒ 2 = y − ( −4) = z − 6 −2 14 3
2 3 −6 Hint: Fixed point on the required line (1, 2, 3). Let us find the

drs of the line : a = 2, b = 3, c = –6
drs of the given line:
Now a 2 + b 2 + c 2 = 4 + 9 + 36 = 7
r= −x − 2 = y + 3 2z − 6
=
1 7 3
\ cos a = a = 2 , cos b = b = 3
r 7 r 7 y+3 z −3
⇒ x+2 =
=
c −6 −1 7 3
cos g = =
r 7 2
6 2 −3 3
2. , , Drs : −1, 7, or − 2,14, 3
7 7 7 2
Hint: 5x – 3 = 15y + 7 = 3 – 10z
Now find the equation of the line.


⇒ ( ) (5 15 )
5 x − 3 = 15 y + 7 = −10 z − 3
10 ( ) 2.
2x − 1 3y − 2 z + 4
6
=
6
=
4
;



x−3 y+ 7
5= 15 =
z− 3
10
drs: 3, 2, 4 and fixed point:
1 2
2 3 (
, ,−4 )
1 1 −1 Now find the equation of the line.
5 15 10
→  ∧ ∧ ∧  ∧ ∧ ∧
IV. 1. r =  5 i − 4 j + 6 k  + λ  3 i + 7 j − 2 k 

⇒ drs : 1 , 1 , −1    
5 15 10
y+4 6− z

⇒ a = 6, b = 2, c = –3 Hint: x − 5 =
=
3 7 2
r= a 2 + b 2 + c 2 = 36 + 4 + 9 = 7

⇒ x − 5 = y − ( −4) z − 6
2 −3 =
\ cos a = 6 , cos b = , cos g =
3 7 −2
7 7 7

⇒ Fixed point (5, –4, 6)
3.

r = ( i + 2 j − k ) + λ (7 i − 5 j + k )
∧ ∧ ∧ ∧ ∧ ∧
drs : 3, 7, –2.

Topic 4. Two Lines in Space


Two lines in space will be in one of the following three forms: parallel. Such lines are called skew lines. Such lines can
(i) Intersecting lines: In space we can have two intersecting never be coplanar. There is distance between them. Such
lines. Such lines are coplanar. Distance between such two lines are not possible in two dimensional coordinate
lines is zero. geometry.
(ii) Non-intersecting and non-parallel lines: In space such (iii) Parallel lines: In space we can have two parallel lines.
two lines are possible which are neither intersecting nor They are also coplanar. We can find out the distance
between them.

344 Mathematics–12
E:\AMIT_WORKS\Exam_Guru\EG_Mathematics-12_(working_02-06-2022)\EG_Mathematics-12_working\Open_Files\Chap_11\Chap_11
\ 19-Aug-2022 Amit Proof-5 Reader’s Sign _______________________ Date __________

Shortest Distance between Two Skew Lines Cartesian Form


Take Cartesian equations of two lines:
Z
L2 x − x1 y − y1 z − z1 x − x2 y − y2 z − z2
= = and = =
T a1 b1 c1 a2 b2 c2

Q
d these lines are coplanar or intersecting when
S P
L1 x2 − x1 y2 − y1 z2 − z1
O Y a1 b1 c1 =0
X    a2 b2 c2
L1 : r = r1 + λ1 m1
  
L 2 : r = r2 + λ 2 m2 Note: The above given formula proves only when the two lines are
coplanar or intersecting, we can not find the point of intersection
of these lines with this method.
Example 1. Find the shortest distance between the two lines
whose vector equation are.
(i) Vector method: Take two skew lines: → ∧ ∧ ∧ ∧ ∧ ∧
 r = (i + 2 j + 3 k) + λ(i − 3 j + 2 k) and
→ → → → → →
→

l1 : r = r1 + λ1 m1 and l2 : r = r2 + λ 2 m2 ∧ ∧ ∧ ∧ ∧ ∧
 r = (4 i + 5 j + 6 k) + µ(2 i + 3 j + k)

\ Shortest distance between these lines:
[A.I. 2009, 2011, Delhi 2014 (C)]
 → → → → Solution. The given equations are:
 r1 − r2  ⋅  m1 × m2 
 
S.D. = → ∧ ∧ ∧ ∧ ∧ ∧
→ → r =  i + 2 j + 3k + λ  i − 3 j + 2 k
m1 × m2    
→  ∧ ∧ ∧  ∧ ∧ ∧
Note: This formula fails when lines are parallel. and r =  4 i + 5 j + 6 k + µ  2 i + 3 j + k
   
x − x1 y − y1 z − z1 → → → ∧ ∧ ∧
(ii) Cartesian form: Let = = and r = r1 − r2 = − 3 i − 3 j − 3 k
a1 b1 c1 ⇒
x − x2 y − y2 z − z2
= = be two skew lines in Cartesian
a2 b2 c2 → → ∧ ∧ ∧
and m1 × m2 = − 9 i + 3 j + 9 k
form. Then the shortest distance between them:
→ →
 → → → →
x2 − x1 y2 − y1 z2 − z1 \  r1 − r2  ⋅  m1 × m2  = 9 and m1 × m2 = 171 = 3 19
   
a1 b1 c1
a2 b2 c2  → → → →
S.D. =  r1 − r2  ⋅  m1 × m2 
  9 3
(a1b2− a2b1) 2 + (b1c2 − b2c1) 2 + (c1a2 − c2a1) 2 S.D. = → →
= =
3 19 19
m1 × m2
Note: This formula fails when lines are parallel.

x +1 y + 3 z + 5
Shortest Distance between Two Parallel Lines Example 2. Show that the lines: = = and
3 5 7
→ → → → → → x − 2 y − 4 z − 6 intersect. Also find the point of intersection.
Let r = r1 + λ1 m1 and r = r2 + λ 2 m2 be two parallel lines. Then = =
1 3 5
shortest distance between them: Solution. Equation of two given lines:  [Delhi 2009]

( r1 − r2 ) × m
x +1
3
=
y+3 z+5
= ...(1)
S.D. =  5 7
m
x−2 y−4 z−6
and = = ...(2)
Condition of Coplanarity of Two Lines or Condition of 1 3 5
Intersection of Two Lines x +1 y+3 z+5
→ → → → → → Let: = = =l
3 5 7
Two lines r = r1 + λ m1 and r = r2 + λ m2 are coplanar or
⇒ x = 3l – 1, y = 5l – 3, z = 7l – 5
intersecting when
Let P(3l – 1, 5l – 3, 7l – 5) be the point of intersection of
 → →  → →  two lines. Therefore, this is the common point of the two lines
 r1 − r2  ⋅  m1 × m2  = 0
and hence it lies on the second equation also.

Three Dimensional Geometry 345


E:\AMIT_WORKS\Exam_Guru\EG_Mathematics-12_(working_02-06-2022)\EG_Mathematics-12_working\Open_Files\Chap_11\Chap_11
\ 18-Aug-2022 Amit Proof-5 Reader’s Sign _______________________ Date __________

On putting the point P in the line (2), we get; x2 − x1 y2 − y1 z2 − z1 3 − 1 4 − 1 −2 − 3


3λ − 1 − 2 5λ − 3 − 4 7λ − 5 − 6 a1 b1 c1 = 2 3 −5
= =
1 3 5
a2 b2 c2 1 −3 2
3λ − 3 5λ − 7 7λ − 11
⇒ = =
1 3 5 2 3 −5
3λ − 3 5λ − 7
⇒ = and 5λ − 7 = 7λ − 11 = 2 3 −5
1 3 3 5 1 −3 2
⇒ 9l –9 = 5l – 7 and 25l –35 = 21l – 33
= 2(6 – 15) – 3(4 + 5) – 5(–6 – 3)
⇒ 4l = 2 and 4l = 2
1 = 2 × (–9) – 3 × 9 – 5(–9)
⇒ l= and l = 1
2 2 = –18 – 27 + 45 = – 45 + 45 = 0
Since both equation gives same value of λ, therefore the two ⇒ Two given lines intersect or two given lines are coplanar.
lines intersect and are coplanar. If we get two different values of → ∧ ∧ ∧ → ∧ ∧ ∧
λ on solving the above two equations, then lines do not intersect. Vector method: r1 = i + j + 3 k, r2 = 3 i + 4 j − 2 k
1
Put λ = in P(3l – 1, 5l – 3, 7l – 5), we get P , ,
2
1 −1 −3
2 2 2 ( )
→ → ∧ ∧
r2 − r1 = (3 i + 4 j − 2 k) − (i + j + 3 k)
∧ ∧ ∧ ∧

Note: (1) When we are asked to show that two lines are coplanar ∧ ∧ ∧ ∧ ∧ ∧
or intersecting and find points of intersection, use the = 3 i + 4 j − 2k − i − j −3k
above method. ∧ ∧ ∧
(2) When we want to show that two lines intersect or are = 2 i + 3 j − 5k
coplanar, you can use above method or the condition ∧ ∧ ∧
→ →   → →  → →
i j k
 r1 − r2  ⋅  m1 × m2  = 0 m1 × m2 = 2 3 −5
1 −3 2
x2 − x1 y2 − y1 z2 − z1
or a1 b1 c1 = 0 → → ∧ ∧ ∧
⇒ m1 × m2 = i ( 6 − 15) − j ( 4 + 5) + k ( −6 − 3)
a2 b2 c2
→ → ∧ ∧ ∧
Example 3. Show that the two lines: ⇒ m1 × m2 = −9 i − 9 j − 9 k
x −1 y −1 z − 3 x − 3 y − 4 z + 2 intersect or are → → → → ∧ ∧ ∧ ∧ ∧ ∧
= = and = =
2 3 −5 1 −3 2 (r2 − r1) ⋅ (m1 × m2) = (2 i + 3 j − 5 k) ⋅ (−9 i − 9 j − 9 k)

coplanar.
= –18 – 27 + 45 = 0
Solution. Given lines:
⇒ The two given lines intersect or two given lines are
x −1 y −1 z − 3 y−4 z+2
= = and x − 3 = = coplanar.
2 3 −5 1 −3 2
Determinant method: Note: This method is used when we want to show that either the
(x1, y1, z1) = (1, 1, 3); (x2, y2, z2) = (3, 4, –2) lines intersect or are coplanar and point of intersection is
a1, b1, c1 = 2, 3, –5; a2, b2, c2 = 1, –3, 2 not asked.

EXERCISE 11.4
I. Multiple Choice Questions (MCQs) II. Short Answer Type Questions-II
Choose the correct answer from the given options. → ∧ ∧ ∧ ∧ ∧ ∧
1. The shortest distance between the two lines whose vector 1. Show that the lines: r = (3 i + 2 j − 4 k) + λ(i + 2 j + 2 k)
equations are given by → ∧ ∧ ∧ ∧ ∧
      
    r = i + 2 j + k + λ i − j + k ( ) and r = (5 i − 2 j) + µ(3 i + 2 j + 6 k) intersect. Also find
their point of intersection. [Delhi 2013 (C), 2014 (C)]
r = 2i − j − k + µ ( 2i + j + 2k ) is

and
→  ∧ ∧ ∧  ∧ ∧
2 2 2. Show that the lines, r =  i + j − k  + λ  3 i − j  and
(a) 3 2 (b)    
2 3 →  ∧ ∧  ∧ ∧
r =  4 i − k  + µ  2 i + 3 k  are coplanar.
5 2    
(c) (d) None of these [Delhi 2013 (C), 2014]
2

346 Mathematics–12
E:\AMIT_WORKS\Exam_Guru\EG_Mathematics-12_(working_02-06-2022)\EG_Mathematics-12_working\Open_Files\Chap_11\Chap_11
\ 18-Aug-2022 Amit Proof-5 Reader’s Sign _______________________ Date __________

III. Long Answer Type Questions 2. Find the shortest distance between the following two
1. Find the shortest distance between the following two lines:
y −5 z −7
   x − 3 =
→ ∧ ∧ ∧
lines: r = (1 + 2λ) i + (1 − λ) j + λ k =
1 −2 1
→  ∧ ∧ ∧  ∧ ∧ ∧
and r =  2 i + j − k + µ  3 i − 5 j + 2 k and   x + 1 = y + 1 = z + 1 
    7 −6 1
[Delhi 2008 (C), 2012, A.I. 2007] [Delhi 2008, 2010, A.I. 2009 (C)]

Answers 11.4
I. 1. We have λ + 3 − 5 2λ + 2 + 2 2λ − 4
  Hence = =
a1 = i + 2 j + k , b1 = i − j + k 3 2 6
  λ − 2 = 2λ + 4 = 2λ − 4
and a2 = 2i − j − k , b2 = 2i + j + 2k

3 2 6
The S.D. between them is given by
    Solve for l
d=
( a2 − a1 ) ⋅ b1 × b2
 
(
...(1)
) On taking first two terms, we get
b1 × b2 λ−2
= 2λ + 4 ⇒ 6l + 12 = 2l – 4
3 2
 
( ) (
We have a2 − a1 = 2i − j − k − i + 2 j + k = i − 3 j − 2k )
⇒ 4l = –16 ⇒ l = –4
i j On taking last two terms, we get
k
  2λ + 4 = 2λ − 4
and b1 × b2 = 1 −1 1 ⇒ 12l + 24 = 4l – 8
2 6
2 1 2

⇒ 8l = –32 ⇒ l = –4
= (− 2 − 1) i + (2 − 2) j + (1 + 2)k We get same value of l = –4
= −3i + 3k
\ Two lines intersect.
 
b1 × b2 = 2
(− 3) + 0 + 3 2 2 Put l = –4 in x = l + 3, y = 2l + 2, z = 2l – 4
= It gives –1, –6, –12.
9 + 0 + 9 = 18 = 3 2
   
∴     ( a2 − a1 ) ⋅ b1 × b2
( ) 2. Hint: Equations of two given lines:

( )(
= i − 3 j − 2k ⋅ − 3i + 3k )

( ∧ ∧
r = i + j − k + λ 3i − j

) ( ∧ ∧
)
→ ∧ ∧ ∧ → ∧ ∧ ∧
= (1) (– 3) + (– 3) (0) + (– 2) (3)
⇒ r1 = i + j − k, m1 = 3 i − j + 0 k
=–3+0–6=–9
Hence, from (1), S.D. (d)


( ∧ ∧
r = 4 i − k + µ 2 i + 3k ) ( ∧ ∧
)
|−9| 9 3 3 2 → ∧ ∧ ∧ → ∧ ∧ ∧
= = = = units.
3 2 3 2 2 2
⇒ r2 = 4 i + 0 j − k, m 2 = 2 i + 0 j + 3 k
Thus, (a) is the correct answer. → → ∧ ∧ ∧

⇒ r2 − r1 = 3 i − j + 0 k
II. 1. (– 1, – 6, – 12)
∧ ∧ ∧
Hint: Equation of the given line i j k
( ) ( )
→ ∧ ∧ ∧ ∧ ∧ ∧ → →
r = 3i + 2 j − 4k + λ i + 2 j + 2k m1 × m 2 = 3 −1 0

r = (5 i − 2 j ) + µ (3 i + 2 j + 6 k )
→ ∧ ∧ ∧ ∧ ∧ 2 0 3
and
= ∧i ( −3 − 0) − ∧j (9 − 0) + k∧ ( 0 + 2)
Equations of the above line in cartesian form
→ → ∧ ∧ ∧
x−3 = y −2 = z+4 m1 × m 2 = −3 i − 9 j + 2 k
1 2 2
and x −
3
5 =
y+2 z
2
=
6
(r − r ) . (m × m )

2

1

1

2

= (3 i − j + 0 k ).( −3 i − 9 j + 2 k )
∧ ∧ ∧ ∧ ∧ ∧
x−3 = y −2 = z +4 = λ
Let
1 2 2 = –9 + 9 + 0 = 0
⇒ x = l + 3, y = 2l + 2, z = 2l – 4 Hence, two lines are coplanar.
Let this be the point of intersection of two lines.
III. 1. 10 ;
\ This point will lie on the second line also 59

Three Dimensional Geometry 347


E:\AMIT_WORKS\Exam_Guru\EG_Mathematics-12_(working_02-06-2022)\EG_Mathematics-12_working\Open_Files\Chap_11\Chap_11
\ 19-Aug-2022 Amit Proof-5 Reader’s Sign _______________________ Date __________

Hint: Equations of two given lines: ∧ ∧ ∧


= 3i − j − 7k



( ∧
) (

r = i + j + 0k + λ 2 i − j + k

) ∧ ∧ ∧



→ → ∧
m1 × m 2 = 3 i − j − 7 k
∧ ∧

r = ( 2 i + j − k ) + µ (3 i − 5 j + 2 k )
→ ∧ ∧ ∧ ∧ ∧ ∧
→ →
and

⇒ m1 × m 2 = 9 + 1 + 49 = 59
→ ∧ ∧ ∧ → ∧ ∧ ∧

⇒ r1 = i + j − 0 k, m1 = 2 i − j + k
(r − r ) . (m × m ) = 3 + 0 + 7 = 10
→ → →
1

2

( )
2 1
→ ∧ ∧ ∧ → ∧ ∧ ∧

(r − r ) . (m × m ) = 10
r2 = 2 i + j − k , m 2 = 3 i − 5 j + 2 k → → → →
→ → ∧ ∧ ∧ 2 1 1 2

⇒ r2 − r1 = i + 0 j − k S.D. =
→ →
59
m1 × m2
∧ ∧ ∧
i j k
→ →
and 2. 2 29 ; Hint:
m1 × m 2 = 2 −1 1
→ → ∧ ∧ ∧ → → ∧ ∧ ∧
3 −5 2
r2− r1 = (4 i + 6 j + 8 k); m1× m2 = 4 i + 6 j + 8 k
∧ ∧ ∧
= i ( −2 + 5) − j ( 4 − 3) + k ( −10 + 3) Try in the way we have done in Question No. 1.

Topic 5. Angle Between Two Lines


Z Condition of Parallelism of above Two Lines
>
b1 , c1 → →
a 1, m1 = λ m2
L2 <
q Cartesian Form
Q
<a Angle Between Two Lines
2, b2 , c
L1 > x − x2 y − y2 z − z2
O 2 x − x1 y − y1 z − z1
Y Let = = and = = be
a1 b1 c1 a2 b2 c2
X
the equations of two lines in Cartesian form.

Angle between these two lines = angle between their direction


ratios a1, b1, c1 and a2, b2, c2. If θ is the angle between these
lines, then:

a1a2 + b1b2 + c1c2


cos q =
a12 + b12 + c12 a22 + b22 + c22
Vector Form
→ → → → → → Condition of Perpendicularity of above Two Lines
Let r = r1 + λ m1 and r = r2 + µ m2 be the vector equations of
a1a2 + b1b2 + c1c2 = 0
two lines. Then angle between two lines = angle between their
→ → Condition of Parallelism of above Two Lines
direction ratios vectors m1 and m2 .
a1 b c
= 1= 1
If θ is the angle between two lines whose direction ratio a2 b2 c2
→ →
vectors are m1 and m2 , then: Assertion-Reason Type Questions
Direction: In the following questions, A statement of
→ →
Assertion (A) is followed by a statement of Reason (R). Mark the
m1 ⋅ m2
cos q = correct choice as given below.
→ →
m1 m2 (a) Both A and R are true and R is the correct explanation of
A
Condition of Perpendicularity of above Two Lines (b) Both A and R are true but R is NOT the correct explanation
of A.
→ →
m1 ⋅ m2 = 0 (c) A is true but R is false.
(d) A is false but R is True.
348 Mathematics–12
E:\AMIT_WORKS\Exam_Guru\EG_Mathematics-12_(working_02-06-2022)\EG_Mathematics-12_working\Open_Files\Chap_11\Chap_11
\ 18-Aug-2022 Amit Proof-5 Reader’s Sign _______________________ Date __________

Example 1. Assertion (A): P is a point on the line segment joining Example 3. Find the angle between two lines:
the points (3, 2, –1) and (6, –4, –2). If x coordinate of P is 5, then
its y coordinate is –2. x − 2 y +1 z − 3 x + 2 y −1 z + 4 .
= = and = =
Reason (R): The two lines x = ay + b, z = cy + d and x = a'y 3 1 2 1 3 3
+ b', z = c'y + d' will be perpendicular, iff aa' + bb' + cc' = 0. Solution. Equations of the lines are:
Solution. Assertion (A) is correct
x−2 y +1 z − 3 y −1 z + 4
Since P = (5, y, z) = = and x + 2 = =
3 1 2 1 3 3
Equation of line joining (3, 2, –1) and (6, –4, –2) is Direction ratios of two lines are:
x−3 y −2 z +1 x − 3 y − 2 z +1 a1, b1, c1 = 3, 1, 2 and a2, b2, c2 = 1, 3, 3
= = = = =
    6 − 3 −4 − 2 −2 + 1 3 −6 −1
If q is the angle between two lines then
so if point P lies on the line then it must satisfy the above equation
a1a2 + b1b2 + c1c2
5 − 3 y − 2 z +1 cos q =
    = =
3 −6 −1 a12 + b12 + c12 a22 + b22 + c22
5−3 y −2 3 ×1+1× 3 + 2 × 3
    = ⇒ cos q =
3 −6 9 +1+ 4 1+ 9 + 9
Hence y co-ordinate of P is –2.
Reason (R) is false. Since, the two lines x = ay + b, z = cy + ⇒ cos q = 12 12
=
d and x = a'y + b', z = c'y + d' will be perpendicular, iff aa' + cc' 14 × 19 266
+ 1 = 0.
 
Example 2. Assertion (A): The angle between the straight lines ⇒ q = cos −1  12 
 266 
x +1 y − 2 z + 3 x −1 y + 2 z − 3
= = and + = is 90° y + 2 z −1
2 5 4 1 2 −3 Example 4. If two lines x − 1 = = and x + 1 = y − 1
4 3λ 5 2λ 1
Reason (R): Skew lines are lines in different planes which are z+2
parallel and intersecting. = are perpendicular, find the value of l.
3
Solution. Assertion (A) is correct Solution. Equations of two lines are
x +1 y − 2 z + 3 x −1 y + 2 z − 3
Given : = = and + = x −1 y + 2 z −1 y −1 z + 2
2 5 4 1 2 −3 = = and x + 1 = =
4 3λ 5 2λ 1 3
Direction ratios of lines are
drs of the two lines:
a1 = 2, b1 = 5, c1 = 4 and a2 = 1, b2 = 2, c2 = –3
a1, b1, c1 = 4, 3l, 5     and a2, b2, c2 = 2l, 1, 3
As we know, the angle between the lines is given by
a1a2 + b1b2 + c1c2 Since two lines are perpendicular then
cos θ =
( a12 + b12 + c12 .)( a22 + b22 + c22 ) a1a2 + b1b2 + c1c2 = 0
⇒ 4 × 2l + 3l × 1 + 5 × 3 = 0
2 × 1 + 5 × 2 + 4 × ( −3)
⇒ cos θ = =0 ⇒ 8l + 3l + 15 = 0
(
2 2 + 52 + 4 2 . )( 12 + 22 + ( −3)
3
) ⇒ 11l = –15
∴ θ = 90°
−15
Reason (R) is wrong. ⇒ l=
11

EXERCISE 11.5
I. Multiple Choice Questions (MCQs) 2. The angle between the pair of lines with direction ratios
Choose the correct answer from the given options. (1, 1, 2) and ( 3 − 1, − 3 − 1, 4 is )
1. The angle between the lines
(a) 30° (b) 45° (c) 60° (d) 90°
x y z x −1 y −1 z −1
    = = and = = is: II. Short Answer Type Questions-I
2 2 −1 1 2 2
1. Find the angle between the following two lines:
4 −1  1 
(a) cos − 1   (b) cos  
9 3 − x + 2 y −1 z + 3 x + 2 2y − 8 z − 5
= = and = = .
−2 7 −3 −1 4 4
2 5
(c) cos −1   (d) cos −1  
9
  9 Also check whether the lines are parallel or perpendicular.

Three Dimensional Geometry 349


E:\AMIT_WORKS\Exam_Guru\EG_Mathematics-12_(working_02-06-2022)\EG_Mathematics-12_working\Open_Files\Chap_11\Chap_11
\ 18-Aug-2022 Amit Proof-5 Reader’s Sign _______________________ Date __________

2. Find the angle between the following pair of lines:


2. Show that three lines with direction ratios: 12 , − 3 , − 4 ;
→ ∧ ∧ ∧ ∧ ∧ ∧ 13 13 13
r = (2 i − 5 j + k) + λ(3 i + 2 j + 6 k) and 4 12 3 3 − 4 12
, , and , , are mutually perpendicular.
→ ∧ ∧ ∧ ∧ ∧ ∧ 13 13 13 13 13 13
r = (7 i − 6 j − 6 k) + µ(i + 2 j + 2 k). [A.I. 2008 (C)]
3. Show that angle between any two diagonals of a cube is
3. Find the value of λ, so that the following two lines are
perpendicular
x−5
5λ + 2
=
2 − y 1− z
5
=
−1
and cos −1 ()
1
3
. [HOTS]

x 2y + 1 1 − z [Delhi 2009, A.I. 2012 (C)] IV. Long Answer Type Questions
= = .
1 4λ −3
1. A line makes angles α, β, γ and δ with diagonals of a cube,
4. Show that the line through the points A(4, 7, 8),
B(2, 3, 4) is parallel to the line passing through the points prove that:
C(–1, – 2, 1) and D(1, 2, 5). 4
cos 2 α + cos 2 β + cos 2 γ + cos 2 δ = .

5. Show that the lines through the points A(1, – 1, 2) and 3
B(3, 4, – 2) is perpendicular to the line through the points
C(0, 3, 2) and D(3, 5, 6).  l + l + l   m + m2 + m3 
2. Verify that  1 2 3  ,  1  and
III. Short Answer Type Questions-II  3   3
1. Find the value of p, so that the lines:
1 − x 7y − 14 z − 3 7 − 7x y − 5  n1 + n2 + n3  can be taken as the direction cosines of

l1 : = = and l2 : =  
3 p 2 3p 1 3
6−z
= are perpendicular to each other. Also find the a line equally inclined to three mutually perpendicular
5
equation of a line passing through a point (3, 2, – 4) and lines with direction cosines,l1, m1, n1; l2, m2, n2 and l3,
parallel to line l1.[A.I. 2014] m3, n3.

Answers 11.5
x y z 6 1
I. 1. (a) Direction ratios of the line = = are 2, 2, − 1 = = = cos 60°
2 2 −1 12 2
Hence, q = 60°. Thus, (c) is the correct answer.
Its direction cosines are 2 , 2 , − 1
3 3 3 II. 1. θ = 90°;
y −1 z + 3
Direction ratios of the line x − 1 = y − 1 = z − 1 are 1, 2, 2 Hint: First line: −x + 2 = =
1 2 2 −2 7 −3
x − 2 y −1 z + 3
∴ Its direction cosines are , , 1 2 2 ⇒ = =
2 7 −3
3 3 3 drs : a1 = 2, b1 = 7, c1 = –3
2 1 2 2 −1 2
Now, cos q =     +     +     Second line:
x+2 = y −4 = z −5
 3 3  3  3  3   3 −1 2 4
drs : a2 = –1, b2 = 2, c2 = 4
2 4 2 2+4−2 4 Now use the formula
= + − = =
9 9 9 9 9 a1a2 + b1b2 + c1c2
cos q =

\
4
cos q = cos −1   ( )(
a1 + b12 + c12 a22 + b22 + c22
2
)
( )
9
Hence, (a) is the correct answer. 2. θ = cos −1 19
21
2. (c) We have direction ratios of two lines as (1, 1, 2) and → ∧ ∧ ∧ → ∧ ∧ ∧
Hint: m1 = 3 i + 2 j + 6 k and m 2 = i + 2 j + 2 k

( 3 − 1, − 3 − 1, 4 ) Now use the formula:
→ →

  cos q =
(
1 3 −1 −1 ) ( ) 3 + 1 + 2( 4) cos q = →
m1 . m 2

m1 m 2
( 3 − 1) + ( − )
2 2
2 2 2 2
1 +1 + 2 , 3 −1 + 4 3. l = 1.
y − 2 z −1
3 −1− 3 −1+ 8 Hint: First line = x − 5 =
=

⇒ cos q = 5λ + 2 −5 1
6 24 ⇒ drs : a1 = 5l + 2, b2 = –5, c1 = 1

350 Mathematics–12
E:\AMIT_WORKS\Exam_Guru\EG_Mathematics-12_(working_02-06-2022)\EG_Mathematics-12_working\Open_Files\Chap_11\Chap_11
\ 18-Aug-2022 Amit Proof-5 Reader’s Sign _______________________ Date __________

IV. 1. Hint: drs of diagonal OF = a, a, a


x y + 1 z −1
Second line : = 2= drs of diagonal AG = a, – a, – a
1 2λ 3
⇒ drs : a2 = 1, b2 = 2l, c2 = 3. drs of diagonal EC = a, – a, a
Since lines are perpendicular, therefore use drs of diagonal BD = a, a, – a
a1a2 + b1b2 + c1c2 = 0
4. Hint: drs of AB : 1, 2, 2 and drs of CD : 1, 2, 2
5. Hint: drs of AB : 2, 5, – 4 and drs of CD : 3, 2, 4
Now show that a1a2 + b1b2 + c1c2 = 0.
x−3 y−2 z+4
III. 1. p = 7; = =
−3 1 2
x − 1 y −2 z −3
Hint: First line = =
−3 p 2
p 7
drs : −3, , 2
7
x −1 y −5 z −6
Second line = =
−3p 1 −5
7
drs :
−3p
,1, −5 Now, find cos2 a, cos2 b, cos2 g and cos2 d and verify.
7
Now use a1a2 + b1b2 + c1c2 = 0 2. Hint: Here: l1l2 + m1m2 + n1n2 = 0
We get p = 7. l1l3 + m1m3 + n1n3 = 0 and l2l3 + m2m3 + n2n3 = 0

Fixed point on the required line (3, 2, –4) Now find angle between direction cosines of the first line:
drs of the first line : –3, 1, 2
l + l + l m + m2 + m3 n1 + n2 + n3
Now find the equation. 1 2 3 , 1 ,
3 3 3
2. Hint: Use a1a2 + b1b2 + c1c2 = 0
3. Try yourself. with l1, m1, n1 and l2 m2, n2 and l3, m3, n3.

Case Based Question


1. Consider the following diagram, where the four forces on (i) The cartesian equation of line along TA is
the cable are shown direction. x y z x y z − 14
    (a) = = (b) − =
Namely FTA , FTB , FTC and FTD −4 3 12 −4 3 7
x y z − 12
(d) x = y = z − 24
z
(c) = =
−3 4 12 3 4 12
(ii) The vector TD is
T
FTC (a) 8i − 6 j + 24k (b) −8i − 6 j + 24k
FTA
FTB FTD
D
(c) −8i − 6 j − 14k (d) 8i + 6 j + 4k
(iii) The length of the cable TB is
14 m C (a) 24 units (b) 17.2 units
A
O 8m (c) 27 units (d) 25 units
6m
B (iv) The length of cable TC is equal to the length of
6m 8m
y
(a) TA (b) TB
x
(c) TD (d) All of these
(v) The sum of all vectors along the cables is

(a) 76i (b) 46 j units (c) 56k (d) 96k
x y z − 14
Ans. (i) (b) − = (ii) (c) −8i − 6 j − 14k
−4 3 7
Based on the above information, answer the following (iii) (b) 17.2 units (iv) (d)All of these
questions. (v) (c) 56k

Three Dimensional Geometry 351


E:\AMIT_WORKS\Exam_Guru\EG_Mathematics-12_(working_02-06-2022)\EG_Mathematics-12_working\Open_Files\Chap_11\Chap_11
\ 18-Aug-2022 Amit Proof-5 Reader’s Sign _______________________ Date __________

Author’s Comments
Questions based on following types are very important for Exams. So, students are advised to revise them thoroughly.
1. Lines
(i) Distance between two skew lines. (Most Imp.)
(ii) (a) Foot of the perpendicular from a given point to a given line.
(b) Distance of a given point from a given line.
(c) Equation of a line drawn from a given point and perpendicular to a given line.
(d) Image of a given point in a given line.

IMPORTANT FORMULAE
1. cos2 a + cos2 b + cos2 g = 1 2. sin2 a + sin2 b + sin2 g = 2
3. cos 2a + cos 2b + cos 2g = –1
4. Collinearity of three points: If a1, b1, c1 are drs of the line segment AB, a2, b2, c2 are drs of the line segment BC, then three
a b c
points A, B and C are collinear if 1 = 1 = 1 .
a2 b2 c2
5. Equation of a line:
(i) In cartesian form: Equation of a line passing through a point A(x1, y1, z1) and direction ratio a, b, c :

       x − x1 = y − y1 = z − z1 .
a b c

→ →
(ii) In vector form: If r1 is the position vector of a fixed point on the line and m is the direction ratio vector of the line, then
equation of the line in vector form:

→ → →
r = r1 + λ m

Here r is position vector of any variable point on the line and l is a constant
→ ∧ ∧ ∧ → ∧ ∧ ∧ → ∧ ∧ ∧
and r = x i + y j + z k, r1 = x1 i + y1 j + z1 k and m = a i + b j + c k .

6. To find the image of a point A(x1, y1, z1) in a given line BC: x − x2 = y − y2 = z − z2 find the foot of the perpendicular
a b c
P(x3, y3, z3) drawn from a point A(x1, y1, z1).

352 Mathematics–12
E:\AMIT_WORKS\Exam_Guru\EG_Mathematics-12_(working_02-06-2022)\EG_Mathematics-12_working\Open_Files\Chap_11\Chap_11
\ 22-Aug-2022 Amit Proof-5 Reader’s Sign _______________________ Date __________

Now assume that A′(x4, y4, z4 ) be the image of the point A(x1, y1, z1) in the line BC: x − x2 = y − y2 = z − z2 , then point
P(x3, y3, z3) will be mid point of two points A(x1, y1, z1) and A′(x4, y4, z4). a b c

x1 + x4 y + y4 z + z4
Hence, = x3, 1 = y3, 1 = z3 . From this find values of x4, y4 and z4. The point A′(x4, y4, z4) is the required
2 2 2
image of the point A in the given line BC.

→ → → → → →
7. Shortest distance between two skew lines vector method: r = r1 + λ m1 and r = r2 + µ m2

 → →  → → 
 r1 − r2  ⋅  m1 × m2 
     S.D. =
→ →
m1 × m2

x − x1 y − y1 z − z1 x − x2 y − y2 z − z2
Cartesian Method: Lines = = and = =
a1 b1 c1 a2 b2 c2

x2 − x1 y2 − y1 z2 − z1
a1 b1 c1
a2 b2 c2
  S.D =
(a1b2 − a2 b1 ) 2 + (b1c2 − b2 c1 ) 2 + (c1a2 − c2 a1 ) 2
8. Condition of coplanarity of two lines or condition of intersection of two lines:

→ → → → → →  → →  → → 
   r = r1 + λ1 m1 and r = r2 + λ 2 m2 ∴  r1 − r2  ⋅  m1 × m2  = 0 [ S.D = 0]

9. Angle between two lines


 
m1 ⋅ m2 | a1a2 + b1b2 + c1c2 |
(i) cos q =   (ii) cos q =
| m1 | m2 | a12 + b12 + c12 a22 + b22 + c22

10. Condition of perpendicularity of two lines a1a2 + b1b2 + c1c2 = 0

a1 b1 c1
11. Condition of parallelism of two lines = =
a2 b2 c2

Three Dimensional Geometry 353


E:\AMIT_WORKS\Exam_Guru\EG_Mathematics-12_(working_02-06-2022)\EG_Mathematics-12_working\Open_Files\Chap_11\Chap_11
\ 18-Aug-2022 Amit Proof-5 Reader’s Sign _______________________ Date __________

COMMON ERRORS
ERRORS CORRECTIONS
(i) Students commit mistake in evaluation of value of a (i) Student can cross-check the value of a determinant by
determinant. evaluating the determinant along the other row or column.

(ii) Students commit mistake in evaluation of cross product of (ii) Practice the expansion of determinant along the different
two vectors. row and column.
(iii) Students commits mistake in writing the different formulae. (iii) Learn the formulae in writing again and again.
(iv) dr’s of the line (iv) drs. of the line
x − 1 1 − y 1 − 2z x − 1 1 − y 1 − 2z
= = = =
2 −2 3 2 −2 3
is usually taken as 2, –2, 3 x −1 = y −1 = z −1/ 2

2 2 −3 / 2
⇒ drs are 4, 4, –3
(v) Conversion from cartesian to vector form and vice-versa ∧ ∧ ∧ ∧ ∧ ∧

x −1 = y − 2 = z − 3 (v) i + 2 j + 3 k + λ  3 i + 4 j + 2 k 
e.g.  
3 4 2
∧ ∧ ∧ ∧ ∧ ∧
Vector form: 3 i + 4 j + 2 k + λ  i + 2 j + 3 k 
 

REVISION CHART

Three Dimensional Geometry


An object that has height, width and depth like any object in the real world. E.g., Your body is three-dimensional. Also known as 3D.

Direction Ratios and Cosines


Direction ratios provide a convenient way of specifying the direction
of a line in three dimensional space. Direction cosines are the cosines
of the angles between a line and the co-ordinate axes.

Distance and Section Interconversion of Different Forms


Shortest Distance
Formula (Cartesian and
Cartesian and Vector between Two Lines
vector forms) of
Form
Equations of Lines
in Space

354 Mathematics–12
E:\AMIT_WORKS\Exam_Guru\EG_Mathematics-12_(working_02-06-2022)\EG_Mathematics-12_working\Open_Files\Chap_12\Chap_12
\ 18-Aug-2022 Amit Proof-5 Reader’s Sign _______________________ Date __________

Topics Covered
12 Linear Programming

12.1 Manufacturing LPP (Maximization of Profit) 12.2 Minimisation of Objective Function and Diet Problems
12.3 Transportation Problems

C hapter map
LINEAR PROGRAMMING

Mathematical Graphical Method Corner Point Different types


Formulation of the of Method of solving of LPP
Problem Solving LPP a LPP

Optimisation Feasible Region


Problem

Objective Feasible Solution Infeasible Solution


Function

Non-negativity
restrictions
Diet Transportation Manufacturer Allocation
Constraints Problems Problems Problems Problems

Introduction
Linear programming is concerned with finding the optimal value (maximum or minimum) of a linear function of several variable
(called objective function) subject to the conditions that the variables are non-negative and satisfy a set of linear inequalities (called
linear constraints).
The common shaded region determined by all the constraints including the non-negative constraints x ≥ 0, y ≥ 0 of a linear
programming problem is called a feasible region or solution region for the problem. Points within and on the boundary of the feasible
region represent feasible solution of the constraints. Any point outside the feasible region is an infeasible solution.

Optimal Solution
Any point in the feasible region that gives the optimal value (maximum or minimum) of the objective functions is called an optimal
solution.

Corner Point Method


Corner point method for solving an LPP.
(i) Find the feasible region of the linear programming problem and determine its corner points (vertices).
(ii) Evaluate the objective function Z = ax + by at each corner point.
Let M and m respectively be the largest and smallest values at these points. If the feasible region is bounded, M and m
respectively, are the maximum and minimum values of the objective function.

355
E:\AMIT_WORKS\Exam_Guru\EG_Mathematics-12_(working_02-06-2022)\EG_Mathematics-12_working\Open_Files\Chap_12\Chap_12
\ 18-Aug-2022 Amit Proof-5 Reader’s Sign _______________________ Date __________

(iii) m is the minimum value of the objective function if the open half plane determined by ax + by < m has no point in common
with the feasible region, otherwise the objective function has no minimum value.
(iv) If two corner points of the feasible region are both optimal solutions of the same type i.e., both produce the same maximum
or minimum, then any point on the line segment joining these two points is also an optimal solution of the same type.
A few important linear programming problems are:
(i) Manufacturing problems. (ii) Diet problems and (iii) Transportation problems.

Topic 1. Manufacturing LPP (Maximization of Profit)


A manufacturer of different products is interested in maximisation Now draw the graph : Plot the points, and join them: To shade
of his profit. Questions based on this topic has constraint in terms the required feasible area. Put (0, 0) in
of inequation and an objective to maximise the profit.
2x + 3y < 30
Example 1. A product has 30 and 17 units of labour and capital
⇒ 0 < 30 (True)
respectively, which he can use to produce two types of goods X
and Y. To produce one unit of X, 2 units of labour and 3 units ∴ (0, 0) is in the shading.
of capital are required. Similarly, 3 units of labour and 1 unit of Put (0, 0) in 3x + y < 17
capital is required to produce one unit of Y. X and Y are priced at ⇒ 0 < 17 (True)
` 100 and ` 120 per unit respectively. If producer use his resources
∴ (0, 0) is in the shading.
to maximise the total revenue, number of units of X and Y should
be produced are To get point P solve:

(a) x = 2; y = 5 (b) x = 2; y = 7 2x + 3y =30 and

(c) x = 3; y = 8 (d) x = 4; y = 4 3x + y = 17, it gives (3, 8)


Solution. Let x be the number of units of X and y be the number From the graph: Feasible region = Shaded region = OAPD
of units of Y produced. We can display the information given in
Vertices of the shaded region are:
the question as:
x y Available units O(0, 0), A(0, 10), P(3, 8) and D ( )
17
3
,0
Labour 2 3 30
Now calculate the value of the objective function at the corner
Capital 3 1 17
points:
Revenue per unit 100 120
Vertices Z = 100x + 120y Value of Z
This problem can be formulated as: (in `)

Z = 100x + 120y (to be maximised) O(0, 0) Z = 100 × 0 + 120 × 0 = 0 0

2x + 3y ≤ 30, 3x + y ≤ 17, x ≥ 0, y ≥ 0. A(0, 10) Z = 100 × 0 + 120 × 10 = 1200 1200


Let us plot these inequations:
P(3, 8) Z = 100 × 3 + 120 × 8 = 1260 1260
A B

( )
Take 2x + 3y = 30 x 0 15 ;
17
y 10 0 D ,0 Z = 100 × 17 + 120 × 0 = 1700 1700
3
3 3 3

C D Z has maximum value at P(3, 8) and maximum profit = ` 1260

Take 3x + y = 17 17 ∴ x = 3 units and y = 8 units.


x 0 = 5.7
3
y 17 0 Hence, option (c) is the correct answer.

356 Mathematics–12
E:\AMIT_WORKS\Exam_Guru\EG_Mathematics-12_(working_02-06-2022)\EG_Mathematics-12_working\Open_Files\Chap_12\Chap_12
\ 18-Aug-2022 Amit Proof-5 Reader’s Sign _______________________ Date __________

EXERCISE 12.1
I. Multiple Choice Questions (MCQs) 4. A cooperative society of farmers has 50 hectares of land
Choose the correct answer from the given options. to grow two crops X and Y. The profits from crops X
and Y per hectare are estimated as ` 10,500 and ` 9,000
1. For an L.P.P. the objective function is Z = 4x + 3y, and the
respectively.To control weeds, a liquid herbicide has to be
feasible region determined by a set of constraints (linear
inequations) is shown in the graph. used for crops X and Y at the rates of 20 litres and 10 litres
per hectare. Further not more than 800 litres of herbicide
y should be used in order to protect fish and wildlife using
100 a pond which collects drainage from the land. How much
land should be allocated to each crop so as to maximise
80 (0, 80) the total profit of the society?
(a) Zmax = ` 4,95,000 at (30, 20)
60 (b) Zmax = ` 4,70,000 at (30, 10)
(c) Zmax = ` 4,60,000 at (30, 15)
0)

P
,4

40 (d) Zmax = ` 4,90,000 at (40, 20)


(0

5. A manufacturing company makes two models A and B of


Q(30, 20)
20 a product. Each piece of model A requires 9 labour hours
(60, 0) for fabricating and 1 labour hour for finishing. Each piece
R
0
x
of model B requires 12 labours hour for fabricating and
20 40 60 80 100
(40, 0) 3 labour hour for finishing. For fabricating and finishing
the maximum labour hours available are 180 and 30
Which one of the following statements is true? respectively. The company makes a profit of ` 8000 on
(a) Maximum value of Z is at R. each piece of model A and ` 12000 on each piece of model
(b) Maximum value of Z is at Q. B. How many pieces of model A and model B should be
(c) Value of Z at R is less than the value at P. manufactured per week to realise a maximum profit?
(d) Value of Z at Q is less than the value at R. (a) Zmax = ` 1,55,000 at (12, 8) (b)Zmax = ` 1,60,000 at
[CBSE 2022] (14, 5)

2. A factory owner wants to purchase two types of machines (c) Zmax = ` 1,65,000 at (12, 5) (d)Zmax = ` 1,68,000 at
A and B for his factory. The machine A requires an area (12, 6)
of 1000 m2 and 12 skilled men for running it and its daily 6. One kind of cake requires 200 g of flour and 25 g of fat
output is 50 units, whereas the machine B requires 1200 and another kind of cake require 100 g of flour and 50 g
m2 area and 8 skilled men, and its daily output is 40 units. of fat. The maximum number of cakes which can be made
If an area of 7600 m2 and 72 skilled men be available to from 5 kg of flour and 1 kg of fat assuming that there is no
operate the machines, how many machines of each type shortage of the other ingredients used in making the cakes
should be bought to maximise the daily output? is given by
(a) Zmax = 310 at (4, 8) (b) Zmax = 320 at (4, 3) (a) Zmax = 21 at (18, 5) (b) Zmax = 25 at (20, 12)
(c) Zmax = 280 at (4, 6) (d) Zmax = 300 at (4, 5) (c) Zmax = 30 at (20, 10) (d) Zmax = 40 at (20, 5)
3. Minimum and maximum value of Z = 3x + 9y subject to 7. A factory makes tennis rackets and cricket bats. A tennis
the constraints: x + 3y ≤ 60; x + y ≥ 10; x ≤ y; x ≥ 0, y ≥ 0 racket takes 1.5 hours of machine time and 3 hours of
are given by craftsman’s time in its making while a cricket bat takes
(a) Zmin = 60 at (5, 5) and Zmax = 180 at (15, 15) and 3 hours machine time and 1 hour of craftsman’s time. In
(0, 20) a day the factory has the availability of not more than 42
(b) Zmin = 50 at (5, 4) and Zmax = 170 at (15, 10) and hours of machine time and 24 hour of craftsman’s time.
(0, 20) What number of cricket bats and tennis rackets be made
(c) Zmin = 45 at (5, 6) and Zmax = 175 at (15, 15) and if the factory work at full capacity?
(5, 12) (a) Zmax = 18 at (4, 8) (b) Zmax = 17 at (14, 12)
(d) None of these (c) Zmax = 16 at (4, 12) (d) Zmax = 20 at (4, 11)

Linear Programming 357


E:\AMIT_WORKS\Exam_Guru\EG_Mathematics-12_(working_02-06-2022)\EG_Mathematics-12_working\Open_Files\Chap_12\Chap_12
\ 18-Aug-2022 Amit Proof-5 Reader’s Sign _______________________ Date __________

8. A manufacturer produces nuts and bolts. It takes 1 hour of 2. A merchant plans to sell two types of personal computers
work on machine A and 3 hours on machine B to produce a a desktop model and a portable model that will cost
package of nuts. It takes 3 hours on machine A and 1 hour ` 25,000 and ` 40,000 respectively. He estimates that
on machine B to produce a package of bolts. He earns a the total monthly demand of computers will not exceeds
profit of ` 17.50 per package on nuts and ` 7.00 per package 250 units. Determine the number of units of each type
on bolts. How many packages of each should be produced of computers, which the merchant should stock to get
each day so as to maximise his profit, if he operates his maximum profit if he does not want to invest more than
machines for at the most 12 hours a day? ` 70 lakhs and if his profit on the desktop model is ` 4,500
(a) Zmax = ` 78 at (3, 5) (b) Zmax = ` 73.50 at (3, 3) and on portable model ` 5000. [AI 2011]
(c) Zmax = ` 70 at (3, 4) (d) Zmax = ` 75 at (4, 3) 3. A manufacturer has three machines I, II and III installed in
9. A factory manufactures two types of screws A and B. his factory. Machine I and II are capable of being operated
Each type of screw requires the use of two machines an for atmost 12 hours whereas machine III must be operated
automatic and a hand operated. It takes 4 minutes on the for atleast 5 hours a day she produces only two items M and
automatic and 6 minutes on hand operated machines to N each requiring the use of three machines. The number
manufacture a package of screw A, while it takes 6 minutes of hours required for producing I unit of each M and N on
on automatic and 3 minutes on hand operated machine the three machines are given in the following table:
to manufacture a package of screw B. Each machine is
Items Number of hours required on machines
available for atmost 4 hours, on any day. The manufacturer
can sell a package of screws A at a profit of ` 7 and screw B I II III
at a profit of ` 10. Assuming that he can sell all the screws M (x) 1 2 1
he manufactures in a day, how many packages of each N (y) 2 1 1.25
type of screws should be produced in order to maximise She makes a profit of ` 600 and ` 400 on items. M and N
his profit?
respectively. How many of each item should she produce
(a) Zmax = ` 390 at (28, 25) so as to maximise her profit, assuming that she can sell all
(b) Zmax = ` 380 at (25, 20) the items that she produced? What will be the maximum
(c) Zmax = ` 400 at (30, 15) profit?
(d) Zmax = ` 410 at (30, 20) 4. A manufacturer makes two types of toys A and B. Three
10. A cottage industry manufactures pedestal lamps and machines are needed for this purpose and the time (in
wooden shades. Each requiring the use of a grinding/ minutes) required for each toy on the machines is given
cutting machine and a sprayer. It takes 2 hours on grinding/ below:
cutting machine and 3 hours on the sprayer to manufacture
Machines Profit
a pedestal lamp. It takes 1 hour on the grinding/cutting Types of toys
I II III in `
machine and 2 hours on the sprayer to manufacture a
shade. On any day the sprayer is available for at the most A (x) 12 min. 18 min. 6 min. 7.50
20 hours and the grinding, cutting machine for at the most B (y) 6 min. 0 min. 9 min. 5.00
12 hours. The profit from the sale of a lamp in ` 5 and that Time available 6 hrs. 6 hrs. 6 hrs.
from a shade is ` 3. Assuming that the manufacturer can on machine in
sell all the lamps and shades that he produces, how should (hrs)
he schedule his daily production in order to maximise his
Each machine is available for a maximum of 6
profit?
hours per day. If the profit on each toy of type A is
(a) Zmax = ` 26 at (5, 2) (b) Zmax = ` 30 at (4, 6) ` 7.50, and that on each toy of type B ` 5 show that 15 toys
(c) Zmax = ` 32 at (4, 4) (d) Zmax = ` 28 at (4, 5) of type A and 30 toys of type B should be manufactured in
II. Long Answer Type Questions a day to get maximum profit.
1. A company manufacture two types of novelty Souvenirs 5. An aeroplane can carry a maximum of 200 passengers.
made of plywood. Souvenirs of type A requires 5 minutes A profit of ` 1,000 is made on each executive class ticket
each for cutting and 10 minutes each for assembling. and a profit of ` 600 is made on each economy class
Souvenirs of type B requires 8 minutes for cutting and ticket. The airline reserves atleast 20 seats for executive
8 minutes each for assembling. There are 3 hours 20 class. However, atleast 4 times as many passengers prefer
minutes available for cutting and 4 hours for assembling. to travel by economy class than by the executive class.
The profit is ` 5 each for type A and ` 6 each for type B Determine how many tickets of each type must be sold
souvenirs. How many souvenirs of each type should the in order to maximise the profit of the airline. What is the
company manufacture in order to maximise his profit? maximum profit?

358 Mathematics–12
E:\AMIT_WORKS\Exam_Guru\EG_Mathematics-12_(working_02-06-2022)\EG_Mathematics-12_working\Open_Files\Chap_12\Chap_12
\ 18-Aug-2022 Amit Proof-5 Reader’s Sign _______________________ Date __________

6. A dealer wishes to purchase a number of fans and sewing per day is 16. If the profit on a ring is ` 300 and that on
machines. He has only ` 57,600 to invest and has space a chain is ` 190, find the number of rings and chains that
for atmost 20 items. A fan costs him ` 3,600 and a sewing should be manufactured per day, so as to earn maximum
machine ` 2,400. Profit on selling a fan and a sewing profit. Make it an LPP and solve it graphically.
machine are ` 220 and ` 180 respectively. Assuming that [Delhi 2011]
he can sell all the items that he can buy, how should he 13. A library has to accommodate two different types of books
invest his money in order to maximise his profit? Formulate on a shelf. The books are 6 cm and 4 cm thick and weight
this problem mathematically and solve it graphically.
1 kg and 1 1 kg each respectively. The shelf is 96 cm long
[A.I. 2007, 2010 (C)] 2
and at a time can support a weight of 21 kg. How should
7. Anil wants to invest atmost ` 12,000 in bonds A and B. the shelf be filled with the books of two types in order to
According to the rules, he has to invest atleast ` 2000 in include the greatest number of books? Make it an LPP and
bond A and atleast ` 4000 in bond B. If the rate of interest in solve it graphically. [A.I. 2010 (C)]
bond A is 8% per annum and on bond B is 10% per annum
14. A decorative item dealer deals in two items A and B. He
now how should Anil invest his money for maximum
has ` 15000 to invest and a space to store at the most 80
interest? Solve the problem graphically. [A.I. 2006]
pieces. Item A costs him ` 300 and item B cost him ` 150.
8. A man has ` 1,500 for purchase rice and wheat. A bag of He can sell item A and B at respective profits of ` 50 and
rice and a bag of wheat cost ` 180 and ` 120 respectively. ` 28. Assuming that he can sell all he buys, formulate the
He has a storage capacity of 10 bags only. He earns a profit LPP in order to maximise his profit and solve it graphically.
of ` 11 and ` 9 respectively per bag of rice and wheat. [Delhi 2012 (C)]
Formulate it as an LPP and solve it graphically.
15. Find graphically the maximum value of Z = 2x + 3y subject
[A.I. 2008, 2009 (C)] to the constraints: 3x + y = 6; x + 2y = 4 and x ≥ 0, y ≥ 0.
9. Solve the following linear programming problem [Delhi 2015]
graphically:
16. A manufacturer produces two products A and B. Both
Maximise: Z = 8x + 7y; subject to the constraints: products are processed on two different machines. The
3x + y ≤ 66, x + y ≤ 45, x ≤ 20, y ≤ 40, x ≥ 0, y ≥ 0 available capacity of first machine is 12 hours and that
[AI 2005 (C)] of second machine is 9 hours per day. Each unit of the
10. A manufacturer makes two products, A and B. Product A product A requires 3 hours on both machines and each unit
sells at ` 200 per unit and take 30 minutes to make and of product B requires 2 hours on first machine and 1 hour
product B sells at ` 300 per unit and takes 1 hour to make. on second machine. Each unit of product A is sold at ` 7
There is permanent order of 14 units of product A and 16 profit and that of B at a profit of ` 4. Find the production
units of product B. A working week consists of 40 hours level per day for maximum profit graphically.
of production and the weekly turnover must not be less [Delhi 2016]
than ` 10,000. If the profit on each of the product A is ` 20 17. Solve the following L.P.P. graphically:
and on product B ` 30, how much of each product should Maximise Z = 4x + y.
be produced so that the profit is maximum? Also find the

Subject to the following constraints x + y ≤ 50,
maximum profit and solve the problem graphically.
3x + y ≤ 90, x ≥ 10, x, y ≥ 0. [Delhi 2017]
11. If a young man rides his motorcycle at 25 km/hour he had
18. Solve the following L.P.P. graphically.
to spend ` 2 per km on petrol. If he rides at a faster speed
Maximise Z = 20x + 10y.
of 40 km/hour the petrol cost increases at ` 5/km. He has
` 100 to spend on petrol and wishes to find what is the
Subject to the following constraints x + 2y ≤ 28,
maximum distance he can travel within one hour. Express 3x + y ≤ 24, x ≥ 2, x, y ≥ 0.[Delhi 2017]
it as LPP and solve it graphically. 19. Maximise Z = x + 2y subject to the constraints
[Delhi 2007, 2013 (C), 2014 (C)] x + 2y ≥ 100, 2x – y ≤ 0, 2x + y ≤ 200, x, y ≥ 0. Solve the
12. A small firm manufactures gold rings and chains. The above L.P.P. graphically. [A.I. 2017]
total number of rings and chains manufactured per day is 20. Solve the following linear programming problem
atmost 24. It takes 1 hour to make a ring and 30 minutes graphically: Maximise Z = 34x + 45y, x + y ≤ 300,
to make a chain. The maximum number of hours available 2x + 3y ≤ 70, x ≥ 0, y ≥ 0. [A.I. 2017]

Linear Programming 359


E:\AMIT_WORKS\Exam_Guru\EG_Mathematics-12_(working_02-06-2022)\EG_Mathematics-12_working\Open_Files\Chap_12\Chap_12
\ 18-Aug-2022 Amit Proof-5 Reader’s Sign _______________________ Date __________

Answers 12.1
I. 1. (b) Z = 4x + 3y
x 50 0
at P (0, 40), Z = 4(0) + 3(40) = 120 Graph: x + y = 50
y 0 50
at Q (30, 20), Z = 4(30) + 3(20) = 180
at R (40, 0), Z = 4(40) + 3(0) = 160 x 0 40
2x + y = 80
\ Zmax = 180 at Q (30, 20) y 80 0

2. (b) Zmax = 320 at (4, 3)
Hint. Let x be the number of machines of type A and y be the
number of the machines of type B. Then we can display the
above information as:
Machine A Machine B Maximum
x y available
capacity
Area in m­2 1000 1200 7600
Men power 12 8 72
Output 50 40
Now formulate the above information in mathematical
Solve: x + y = 50 and 2x + y = 80 ⇒ B(30, 20)
inequations.
Feasible region = Shaded region OABC.
Z = 50x + 40y (To be
maximised) O = (0, 0), A = (40, 0),
1000x + 1200y ≤ 7600 B = (30, 20), C = (0, 50)
⇒ 5x + 6y ≤ 38; ZO = 0, ZA = 4,20,000, ZB = 4,95,000,
12x + 8y ≤ 72 ZC = 4,50,000
⇒ 3x + 2y ≤ 18 5. (d) Zmax = ` 1,68,000 at (12, 6)
and x ≥ 0 and y ≥ 0 Hint: Let the number of pieces of model A = x and number of
Or Z = 50x + 40y pieces of model B = y.
(to be maximised) Objective function: maximise:
5x + 6y ≤ 38; 3x + 2y ≤ 18 Z = 8000x + 12000y,
and Subject to the constraints:
x ≥ 0 and y ≥ 0. 9x + 12y ≤ 180
Now, let us draw the graph: ⇒ 3x + 4y ≤ 60 (Fabrication)
x + 3y ≤ 30, x ≥ 0, y ≥ 0
3. (a) Zmin = 60 at (5, 5) and Zmax = 180 at (15, 15)
x 0 20
and (0, 20) Graph: 3x + 4y = 60
y 15 0
Hint: Maximize: Z = 3x + 9y subject to the constraints:

x + 3y ≤ 60; x + y ≥ 10; x ≤ y; x ≥ 0 y ≥ 0.
x 0 30
x + 3y = 30
y 10 0

4. (a) Zmax = ` 4,95,000 at (30, 20)


Hint: Objective function maximise:

Z = 10,500x + 9000y, subject to the constraints: x + y ≤ 50 (land) Solve: 3x + 4y = 60 and x + 3y = 30 ⇒ (12, 6)
20x + 10y ≤ 800 (herbicide) Feasible region = Shaded region = OABC
Or x + y ≤ 50 and 2x + y ≤ 80, x ≥ 0, y ≥ 0. O = (0, 0), A = (20, 0), B = (12, 6), C = (0, 10)
ZO = 0, ZA = 1,60,000, ZB = 1,68,000,

360 Mathematics–12
E:\AMIT_WORKS\Exam_Guru\EG_Mathematics-12_(working_02-06-2022)\EG_Mathematics-12_working\Open_Files\Chap_12\Chap_12
\ 18-Aug-2022 Amit Proof-5 Reader’s Sign _______________________ Date __________

ZC = 1,20,000
x 0 8
⇒ Zmax = ` 1,68,000 at (12, 6) 3x + y = 24
y 24 0
⇒ Model A = 12 and model B = 6.
6. (c) Zmax = 30 at (20, 10) Solve: x + 2y = 28 and 3x + y = 24 ⇒ B(4, 12)
I kind of cakes = 20, II kind of cakes = 10.
Hint: Let the number of cakes of kind I = x and number of II
kind cakes = y.
Kind of Number of Flour (gms) Fat (gms)
cakes cakes
I x 200 25
II y 100 50
Total x+y 5000 1000
Objective function maximise: Z = x + y, subject to the constraints:
200x + 100y ≤ 5000
Feasible region = Shaded region OABC.
⇒ 2x + y ≤ 50 (Flour)
Vertices : O (0, 0) A = (8, 0), B = (4, 12),
25x + 50y ≤ 1000
C = (0, 14).
⇒ x + 2y ≤ 40 (Fat)
x ≥ 0 and y ≥ 0 (i) Z1 = x + y : Z1(O) = 0,
Graph: Z1(A) = 8, Z1(B) = 16,

x 0 25 Z1(C) = 14
2x + y = 50
⇒ Zmax =16 at (4, 12)
y 50 0


∴ Maximum number of tennis rackets = 4 and cricket bats
x 0 40 = 12.
x + 2y = 40
y 20 0 (ii) Z2 = 20x + 10y (Profit)

Solve: 2x + y = 50 and x + 2y = 40 Z2(O) = 0, Z2(A) = ` 160,
⇒ (x, y) = (20, 10) Z2(B) = ` 200,
Feasible region = Shaded region OABC. Z2(C) = ` 140
O = (0, 0), A = (25, 0), B = (20, 10), C = (0, 20) ∴ At Zmax Profit = ` 200, x = 4 and y = 12.
ZO = 0, ZA = 25, ZB = 30, ZC = 20. 8. (b) Zmax = ` 73.50 at (3, 3)
⇒ Zmax = 30 at B(20, 10) or x = 20, y = 10. Hint: Let the number of nut’s package = x and number of bolt’s

7. (c) Zmax = 16 at (4, 12) package = y.
Hint: Let x tennis rackets and y cricket bats are to be produced
in one day in the factory. Machine A Machine B
Items Number Profit
hours hours
Machine
Items Number Craftsmans Profit
hours Nuts x 1 3 17.50
Tennis
x 1.5 3 20 Bolts y 3 1 7.00
rackets
Cricket bats y 3 1 10
Time 12 12
Total time
42 24
available Objective function Maximise: Z = (17.50) x + 7y
Objective function: Subject to constraints:
(i) Maximize: Z1 = x + y (number of rackets and bats)
x + 3y ≤ 12, 3x + y ≤ 12, x ≥ 0, y ≥ 0

(ii) Maximise : Z2 = 20x + 10y (Profit) subject to the constraints:
x 0 12
1.5x + 3y ≤ 42 ⇒ x + 2y ≤ 28, 3x + y ≤ 24 Graph: x + 3y = 12
y 4 0
x ≥ 0, y ≥ 0
x 0 28 x 0 4
Graph: x + 2y = 28 3x + y = 12
y 14 0 y 12 0

Linear Programming 361


E:\AMIT_WORKS\Exam_Guru\EG_Mathematics-12_(working_02-06-2022)\EG_Mathematics-12_working\Open_Files\Chap_12\Chap_12
\ 18-Aug-2022 Amit Proof-5 Reader’s Sign _______________________ Date __________

Solve: x + 3y = 12 and 3x + y = 12 10. (c) Zmax = ` 32 at (4, 4)


⇒ (3, 3) Hint: Let the number of pedestal lamps = x, number of wooden
shades = y.
Objective function: maximise: Z = 5x + 3y.
Subject to the constraints:
2x + y ≤ 12; 3x + 2y ≤ 20; x ≥ 0, y ≥ 0
Machine (x) Pedestal (y) Wooden Time
lamps shades (hours)
Grinding/Cutting 2 1 12
Sprayer 3 2 20
Profit 5 3
Objective Function Maximise: Z = 5x + 3y
Subject to constraints 2x + y ≤ 12 and 3x + 2y ≤ 20
Feasible region shaded area OABC:
Vertices: O (0, 0), A = (4, 0), B = (3, 3), x 0 6
Graph: 2x + y = 12
C = (0, 4) y 12 0
⇒ Z0 = 0; ZA = 70, ZB = 73.50, ZC = 28
⇒ Zmax (Profit) = ` 73.50 at x = 3 and y = 3. x 0 20
9. (d) Zmax = ` 410 at (30, 20) 3x + 2y = 20 3
y 10 0
Hint:
Machine A (time) (x) B (time) (y) Time (minutes) Solve: 2x + y = 12 and 3x + 2y = 20 ⇒ (4, 4)

Automatic 4 min. 6 min. 240


Hand operated 6 min. 3 min. 240
Profit `7 ` 10
Objective function: maximise: Z = 7x + 10y subject to the
constraints:
4x + 6y ≤ 240 ⇒ 2x + 3y ≤ 120;
6x + 3y ≤ 240
⇒ 2x + y ≤ 80, x ≥ 0, y ≥ 0
x 0 60
Graph: 2x + 3y = 120
y 40 0 Feasible region = Shaded region OABC.

Vertices: O = (0, 0), A = (6, 0), B = (4, 4); C = (0, 10)
x 0 40
2x + y = 80 Now, ZO = 0, ZA = 30, ZB = 32, ZC = 30
y 80 0
⇒ Zmax (Profit) = ` 32 at x = 4, x = 4
Solve: 2x + 3y = 120 and 2x + y = 80 ⇒ (30, 20) II. 1. Zmax = ` 160, for x = 8 and y = 20.
Hint: Let the company manufactures x souvenirs of type A and
y souvenirs of type B.
Type of Type A Type B Maximum time
Work (min.) (x) (min.) (y) in (min.)
Cutting 5 8 200
Assembling 10 8 240
Profit `5 `6
Objective function: Maximise: Z = 5x + 6y subject to the
constraints:
5x + 8y ≤ 200 and 5x + 4y ≤ 120, x ≥ 0, y ≥ 0
Graph:
Feasible region = Shaded region OABC.
x 0 40
Vertices: O = (0, 0), A = (40, 0), B = (30, 20); 5x + 8y = 200
y 25 0
C = (0, 40)
ZO = 0, ZA = 280, ZB = 410, ZC = 400 x 0 24
⇒ Zmax (Profit) = ` 410 when x = 30 of type A and y = 20 of 5x + 4y = 120
y 30 0
type B.

362 Mathematics–12
E:\AMIT_WORKS\Exam_Guru\EG_Mathematics-12_(working_02-06-2022)\EG_Mathematics-12_working\Open_Files\Chap_12\Chap_12
\ 18-Aug-2022 Amit Proof-5 Reader’s Sign _______________________ Date __________

Solve: 5x + 8y = 200 and 5x + 4y = 120 ⇒ (8, 20) Feasible region = Shaded region OABC.
ZO = 0, ZA = 11,25,000, ZB = 11,50,000,
ZC = 8,75,000
⇒ M. Profit Zmax = ` 11,50,000 when x = 200, y = 50
3. 
4 units of M and 4 units of N and maximum profit: ` 4000
Hint: Let x and y be the number of items of M and N respectively,

Z = 600x + 400y (maximise) subject to the constraints:
x + 2y ≤ 12 (Machine I) 2x + y ≤ 12 (Machine II),

x + 5 y ≥ 5 (Machine III), x ≥ 0, y ≥ 0.
Feasible region = Shaded region 4
OABC. x 0 12
Vertices: O = (0, 0), A = (24, 0), Graph: x + 2y = 12
y 6 0
B = (8, 20), C = (0, 25)
ZO = 0, ZA = 120, ZB = 160, ZC = 150 x 0 6 x 0 5
2x + y = 12 ,x+ 5 y=5
⇒ Zmax (Profit) = ` 160 at x = 8, y = 20 y 12 0 4 y 4 0

2. Zmax (Profit) = ` 11,50,000,
when x = 200 and y = 50. Solve 2x + y = 12 and x + 2y = 12
Hint: Assume that the merchant deals in x desktop and y portable ⇒ x = 4, y = 4.
computers.
Desktop Portable
Computers (x) Computers (y)
Cost ` 25,000 ` 40,000 ` 70,00,000
Number x y 250
Profit ` 4,500 ` 5,000
Objective function: Maximise:
Z = 4,500x + 5,000y
subject to the constraints: x + y ≤ 250
25,000x + 40,000y ≤ 70,00,000
⇒ 5x + 8y ≤ 1400, x ≥ 0, y ≥ 0 Feasible region = Shaded region ABCDE.
x 0 250 Z = 600x + 400y : A(5, 0) = 3000, B(6, 0) = 3600,
Graph: x + y = 250
y 250 0 C(4, 4) = 4000, D(0, 6) = 2400, E(0, 4) = 1600.

x 0 280 4. Hint: Let x and y be the number of toys of type A and B
5x + 8y = 1400 respectively. Then maximise:
y 175 0

Z = 7.50x + 5y subject to the constraints:
Solve: x + y = 250 and 5x + 8y = 1400
12x + 6y ≤ 360
⇒ (x, y) = (200, 50)
Vertices: O = (0, 0), A = (250, 0), ⇒ 2x + y ≤ 60, x ≥ 0, y ≥ 0
B = (200, 50), C = (0, 175) 6x + 9y ≤ 360
⇒ 2x + 3y ≤ 120, x ≥ 0, y ≥ 0
and 18x ≤ 360 ⇒ x ≤ 20
x 0 30
Graph: 2x + y = 60
y 60 0

x 0 60
2x + 3y = 120
y 40 0

Solve: 2x + y = 60 and 2x + 3y = 120
⇒ x = 15, y = 30.

Linear Programming 363


E:\AMIT_WORKS\Exam_Guru\EG_Mathematics-12_(working_02-06-2022)\EG_Mathematics-12_working\Open_Files\Chap_12\Chap_12
\ 18-Aug-2022 Amit Proof-5 Reader’s Sign _______________________ Date __________

Solve: x = 20 and 2x + y = 60 Objective function: Maximise: Z = 220x + 180y, subject to the


⇒ x = 20, y = 20. constraints:
3,600x + 2400y ≤ 57,600
⇒ 3x + 2y ≤ 48 and x + y ≤ 20, x ≥ 0, y ≥ 0
x 0 16
Graph: 3x + 2y = 48
y 24 0

x 0 20
x + y = 20
y 20 0

Solve: 3x + 2y = 48 and x + y = 20 ⇒ (8, 12)
Vertices: O = (0, 0), A = (16, 0), B(8, 12),
C(0, 20)
Feasible region = Shaded region OABCD. ZO = 0, ZA = 3,520, ZB = 3,920, ZC = 3,600
O (0, 0), A (20, 0), B (20, 20), C(15, 30), D(0, 40)
ZO = 0, ZA = 150, ZB = 250, ZC = 262.50, ZD = 200
Maximum profit = ` 262.50, x = 15 and y = 30.
5. 40 tickets of executive class and 160 tickets of economy class;
M. Profit = ` 1,36,000.
Hint: Let the number of executive class tickets be x and number
of economy class tickets be y.
Maximize: Z = 1,000x + 600y subject to constraints:
x ≥ 20, y ≥ 4x, x + y ≤ 200
x 0 200
Graph: x + y = 200
y 200 0

Feasible region = Shaded region OABC.
x 0 20 40 ⇒ Maximum profit = ` 3,920 at x = 8, y = 12.
y = 4x
y 0 80 160 7. Max. interest = ` 1,160. He should invest ` 2,000 in bond A and

` 10,000 in bond B.
Solve: (i) x + y = 200 and y = 4x
Hint: Let ` x and y be invested in bond A and bond B respectively
⇒ x = 40, y = 160 objective function:
(ii) x + y = 200 and x = 20 ⇒ (20, 180)
(iii) y = 4x and x = 20 ⇒ (20, 80) Maximise : Z = 8x + 10y
100 100
Subject to the constraints: x + y ≤ 12000,
x ≥ 2,000, y ≥ 4,000. x ≥ 0, y ≥ 0.
x 0 12, 000
Graph: x + y = 12, 000
y 12, 000 0

Solve: x + y = 12,000 and x = 2,000
⇒ (2,000, 10,000)
Solve: x + y = 12,000 and y = 4,000 ⇒ (8,000, 4,000)
Vertices: A = (2, 000, 4,000), B(8,000, 4,000),
C(2,000, 10,000). ZA = 560, ZB = 1,040, ZC = 1,160

Feasible region = Shaded region ABC.


A (20, 80), B(40, 160), C(20, 180)
ZA = ` 68,000, ZB = ` 1,36,000, ZC = ` 1,28,000.
6. Max. profit = Zmax = ` 3,920 when deals with 8 fans and 12
sewing machines.
Hint: Let x and y be the number of fans and sewing machine he
deals:
Fans Sewing Machine
Cost ` 3,600 ` 2,400 57,600
Number x y 20
Feasible region = Shaded region ABC.
Profit 220 180 ⇒ Max. interest = ZC = ` 1,160 at x = ` 2,000, and y = ` 10,000.

364 Mathematics–12
E:\AMIT_WORKS\Exam_Guru\EG_Mathematics-12_(working_02-06-2022)\EG_Mathematics-12_working\Open_Files\Chap_12\Chap_12
\ 18-Aug-2022 Amit Proof-5 Reader’s Sign _______________________ Date __________

8. Profit = ` 100, x = 5 and y = 5


Hint: Objective function:

ZD = 320, ZE = 280 ⇒ Zmax = 325.5 at 21 , 69 .
2 2 ( )
Maximise: Z = 11x + 9y subject to the constraints: x + y ≤ 10,
180x + 120y ≤ 1,500
⇒ 3x + 2y ≤ 25, x ≥ 0, y ≥ 0

x 10 0
Graph: x + y = 10
y 0 10

x 25 0
3
3x + 2y = 25
y 0 25
2
Feasible region = Shaded region OABCDE.
Solve: x + y = 10 and 3x + 2y = 25 ⇒ (5, 5)
10. Maximum profit ; ` 1,440, when product A = 48 units and product

3 ( )
Vertices: O = (0, 0), A = 25 , 0 , B = (5, 5), B = 16 units are produced.
Hint: Objective function:

275 , Maximise: Z = 20x + 30y subjects to the constraints:
C = (0, 10). ZO = 0, ZA = ZB = 100, ZC = 90.
3 200x + 300y ≥ 10,000
⇒ 2x + 3y ≥ 100; 30x + 60y = 2400
⇒ x + 2y = 80, x ≥ 14, y ≥ 16,
x ≥ 0, y ≥ 0.

x 50 0
Graph: 2x + 3y = 100 100
y 0
3

x 80 0
x + 2y = 80
y 0 40

Solve: 2x + 3y = 100 and x + 2y = 80
Feasible region = Shaded region OABC.

(
9. Z max = 325.5 at 21 , 69
2 2 )
x 0 22
Hint: Graph: 3x + y = 66

y 66 0

x 45 0
x + y = 45
y 0 45

Solve: 3x + y = 66 and x + y = 45

⇒ ( 21 69
,
2 2 ) ⇒ (– 40, 60). Point of intersection of x = 14 and
2x + 3y = 100 is (14, 24); x = 14 and x + 2y = 80 is (14, 33),
Solve: x = 20 and 3x + y = 66 ⇒ (20, 6) y = 16 and 2x + 3y = 100 is (26, 16), y = 16 and x + 2y = 80 is
Solve: x = 20 and x + y = 45 ⇒ (20, 25) (48, 16).
Solve: y = 40 and x + y = 45 ⇒ (5, 40) Vertices: A = (26, 16), B = (48, 16),
C = (14, 33), D = (14, 24)
Here given Z = 8x + 7y
ZA = 1,000, ZB = 1,440,
O = (0, 0), A = (20, 0), B = (20, 6), C =
E = (0, 40)
( 21 69
,
2 2 )
, D = (5, 40), ZC = 1,270, ZD = 1,000
Feasible region = Shaded region ABCD.
ZO = 0, ZA = 160, ZB = 202, ZC = 325.5, ⇒ Zmax = ` 1,440 at x = 48, y = 16.

Linear Programming 365


E:\AMIT_WORKS\Exam_Guru\EG_Mathematics-12_(working_02-06-2022)\EG_Mathematics-12_working\Open_Files\Chap_12\Chap_12
\ 18-Aug-2022 Amit Proof-5 Reader’s Sign _______________________ Date __________

B = (8, 16), C = (0, 24)


11. Max. distance covered = 30 km, 50 km at a speed of 25 km/hr
40 3 ZO = 0, ZA = 4,800, ZB = 5,440, ZC = 4,560
and km at a speed of 40 km/hr.
3
Hint: Let the distance covered at the speed of 25 km/hr = x km
and the distance covered at the speed of 40 km/hr = y km.
∴ Objective function: Maximise: Z = x + y subject to the
constraints:
x y
2x + 5y ≤ 100; + ≤ 1, x ≥ 0, y ≥ 0
25 40

x 50 0
Graph: 2x + 5y = 100
y 0 20

x y x 0 25
+ =1 Feasible region = Shaded region OABC.
25 40 y 40 0
⇒ Max. Profit = ` 5,440 at x = 8 and y = 16.
y
Solve: 2x + 5y = 100 and x + =1 13. The maximum number of books = 18,
25 40
x = 12, and y = 6.

⇒ (
50 40
,
3 3 ) Hint: Let the number of books of first type = x and number of

books of second type = y
Vertices: O = (0, 0), A = (25, 0),

( )
Objective function: Maximise: Z = x + y subject to the
50 , 40
B = , C = (0, 20) constraints: 6x + 4y ≤ 96 ⇒ 3x + 2y ≤ 48
3 3
ZO = 0, ZA = 25, ZB = 30, ZC = 20 x + 3 y ≤ 21 ⇒ 2x + 3y ≤ 42, x ≥ 0, y ≥ 0
2
x 16 0
Graph: 3x + 2y = 48
y 0 24

Feasible region = Shaded area OABC.

(
⇒ Zmax = 30 at 50 , 40 .
3 3 )
12. Max. profit : ` 5,440 when x = 8, y = 16.
Hint: Let the number of gold rings = x; x 21 0
2x + 3y = 42
number of gold chains = y y 0 14

∴ Objective function Maximise: Z = 300x + 190y subject to the
constraints: Solve: 3x + 2y = 48 and 2x + 3y = 42 ⇒ (12, 6)
Vertices: O = (0, 0), A = (16, 0),
x + y ≤ 24, x + 1 y ≤ 16
2 B = (12, 6), C = (0, 14), ZO = 0, ZA = 16,
⇒ 2x + y ≤ 32, x ≥ 0, y ≥ 0 ZB = 18, ZC = 14
x 24 0 Feasible region = Shaded region OABC.
Graph: x + y = 24
y 0 24 ⇒ Zmax = 18 at (12, 6)

14. Maximum profit of ` 2680 is at (20, 60).
x 16 0 Hint: Maximise: Z = 50x + 28y
2x + y = 32
y 0 32
Subject to: x + y ≤ 80, 2x + y ≤ 100 x ≥ 0, y ≥ 0.
Now solve: x + y = 24 and 2x + y = 32 ⇒ (8, 16)
Vertices: O = (0, 0), A = (16, 0), 5 5 5 ( )
15. Maximum value of Z is 34 at 8 , 6 .

366 Mathematics–12
E:\AMIT_WORKS\Exam_Guru\EG_Mathematics-12_(working_02-06-2022)\EG_Mathematics-12_working\Open_Files\Chap_12\Chap_12
\ 18-Aug-2022 Amit Proof-5 Reader’s Sign _______________________ Date __________

16. A = 2 units, B = 3 units. Max. profit = ` 26 18. Maximise: z = 20x + 10y


Hint: Maximise Z = 7x + 4y subject to constraints 3x + 2y ≤ 12,
Subject to the constraint x + 2y ≤ 28, 3x + y ≤ 24, x ≥ 2, x, y ≥ 0
3x + y ≤ 9, x ≥ 0, y ≥ 0
x 0 4
3x + 2y = 12
y 6 0

x 0 3
3x + y = 9
y 9 0

Draw the graph of the inequation 3x + 2y ≤ 12 and 3x + y ≤ 9,
x ≥ 0, y ≥ 0
Now shade the feasible region, AECOA is the feasible region.
Corner points are A(3, 0), C(0, 6) and E(2, 3) on putting these
points in Z = 7x + 4y; point E(2, 3) maximise the profit.

x + 2y = 28 3x + y = 24

x 0 28 x 0 8
y 14 0 y 24 0

Corner points z = 20x + 10y
A(2, 0) 40
B(8, 0) 160
17. Maximise : z = 4x + y C(4, 12) 200 (Maximise)
Subject to constraints
D(2, 13) 170
x + y ≤ 50, 3x + y ≤ 90, x ≥ 10, x, y ≥ 0
x + y = 50 3x + y = 90 Maximum value is 200 at the point (4, 12).
x 0 50 x 0 30 19. z = x + 2y s.t x + 2y ≥ 100, 2x – y ≤ 0, 2x + y ≤ 200, x, y ≥ 0
y 50 0 y 90 0

x + 2y = 100
2x –y = 0 2x + y = 200

x 100 0 x 0 50 x 100 0
Corner points z = 4x + y y 0 50 y 0 100 y 0 200
A(10, 0) 40
B(30, 0) 120 → maximum Z(A) = 400 (max.), Z(B) = 250,
C(20, 30) 110
Z(C) = 100, Z(D) = 100
D(10, 40) 80
Maximum value is 120 at the point (30, 0). Maximum value is 400 at the point (0, 200).

Linear Programming 367


E:\AMIT_WORKS\Exam_Guru\EG_Mathematics-12_(working_02-06-2022)\EG_Mathematics-12_working\Open_Files\Chap_12\Chap_12
\ 18-Aug-2022 Amit Proof-5 Reader’s Sign _______________________ Date __________

20. Maximise: z = 34x + 45y



Subject to x + y ≤ 300, 2x + 3y ≤ 70, x ≥ 0, y ≥ 0
x + y = 300
x 0 300
y 300 0

2x + 3y = 70
x 0 35
70 ( 23.3) 0
y
3

( )
Z(A) = z 0 , 70 = 1050
3
Z(B) = z (35, 0) = 1190
⇒ Max (1190) at x = 35, y = 0.
Maximum value is 1190 at the point (35, 0)

Topic 2. Minimisation of Objective Function and Diet Problems


When a manufacturer is manufacturing some products, he is Solution. Let x and y be the number of packets of food P and Q
interested to minimise the cost of production. Here also we convert respectively. Obviously x ≥ 0, y ≥ 0. Mathematical formulation
the given conditions into equations called constraints and form of the given problem is as follows:
an objective to minimise the cost of production.
Minimise Z = 6x + 3y (vitamin A)
Example 1. (Diet problem) A dietician has to develop a special
subject to the constraints
diet using two foods P and Q. Each packet (containing 30 g) of
12x + 3y ≥ 240 (constraint on
food P contains 12 units of calcium, 4 units of iron, 6 units of
cholesterol and 6 units of vitamin A. Each packet of the same calcium), i.e. 4x + y ≥ 80 ...(1)
quantity of food contains 3 units of calcium, 20 units of iron, 4 4x + 20y ≥ 460 (constraint on iron), i.e. x + 5y ≥ 115 ...(2)
units of cholesterol and 3 units of vitamin A. The diet requires 6x + 4y ≤ 300 (constraint on
atleast 240 units of calcium, atleast 460 units of iron and at most
cholesterol), i.e. 3x + 2y ≤ 150 ...(3)
300 units of cholesterol. How many packets of each food should
be used to minimise the amount of vitamin A in the diet? What x ≥ 0, y ≥ 0 ...(4)
is the minimum amount of vitamin A? Let us graph the inequalities (1) to (4)
(a) 288 at (2, 72) (b) 150 at (15, 20) The feasible region (shaded) determined by the constraints (1) to
(c) 285 at (40, 15) (d) None of these (4) is shown in figure and note that it is bounded.

368 Mathematics–12
E:\AMIT_WORKS\Exam_Guru\EG_Mathematics-12_(working_02-06-2022)\EG_Mathematics-12_working\Open_Files\Chap_12\Chap_12
\ 18-Aug-2022 Amit Proof-5 Reader’s Sign _______________________ Date __________

The coordinates of the corner points L, M and N are (2, 72), (15, 20) and (40, 15)
respectively. Let us evaluate Z at these points:
Corner Point Z = 6x + 3y
(2, 72) 228
(15, 20) 150 ← Minimum
(40, 15) 285
From the table, we find that Z is minimum at the point (15, 20). Hence, the amount
of vitamin A under the constraints given in the problem will be minimum, if 15
packets of food P and 20 packets of food Q are used in the special diet. The minimum
amount of vitamin A will be 150 units.
Hence, option (b) is the correct answer.

EXERCISE 12.2
I. Multiple Choice Questions (MCQs)
Choose the correct answer from the given options.
1. Minimum value of Z = x – 5y + 20 subject to the constraints: x – y ≥ 0, – x + 2y ≥ 2, x ≥ 3, y ≤ 4 is given by
(a) Zmin = 2 at (4, 6) (b) Zmin = 4 at (4, 4) (c) Zmin = 3 at (4, 3) (d) Zmin = 6 at (4, 5)
2. Minimum value of Z = x – 7y + 190 subject to the constraints: x + y ≤ 8, x + y ≥ 4, x ≤ 5, y ≤ 5, x ≥ 0, y ≥ 0 equals.
(a) Zmin = 155 at (0, 5) (b) Zmin = 150 at (2, 5) (c) Zmin = 160 at (2, 3) (d) Zmin = 170 at (3, 5)

Answers 12.2
I. 1. (b) Zmin = 4 at (4, 4) 2. (a) Zmin = 155 at (0, 5)
Hint: Minimise Z = x – 5y + 20 subject to the constraints:
x 8 0
x – y ≥ 0, – x + 2y ≥ 2, y ≤ 4, x ≥ 3 Hint: Graph: x + y = 8

y 0 8
x 0 4
Graph: x − y = 0 x 4 0
y 0 4 x+ y=4
y 0 4
x 4 0
−x + 2y = 2
y 3 1

Solve: x – y = 0
and – x + 2y = 2
⇒ (2, 2)

Vertices are:
A = (5, 0),
B = (5, 3), C = (3, 5),
D = (0, 5), E = (0, 4),
F = (4, 0) ⇒ ZA = 195
Feasible region =
Shaded region ABCD. ZB = 174, ZC = 158,
Vertices: A = (3, 3), ZD = 155, ZE = 162,
( )
B = 3, 5 ,
2
ZF = 194.
C(6, 4), D = (4, 4) Feasible region = bounded region ABCDEF.
ZA = 8, ZB = 10.5, ZC = 6, ZD = 4 (min.) ⇒Zmin = 155 at x = 0 and y = 5.

Linear Programming 369


E:\AMIT_WORKS\Exam_Guru\EG_Mathematics-12_(working_02-06-2022)\EG_Mathematics-12_working\Open_Files\Chap_12\Chap_12
\ 18-Aug-2022 Amit Proof-5 Reader’s Sign _______________________ Date __________

Topic 3. Transportation Problems


In transport problems, we want to transport goods from one spot Total transportation cost
to destination spot at minimum cost. In these problems,
Z = 160x + 100 (5 – x) + 100y +
manufacturing spot and sale spot are given with their distances
and also cost of transportation per kilometre. With this information  120 (5 – y) + 100
constraint inequations and objective equations are formed. (x + y – 4) + 150 (8 – x – y)
Generally, questions are not asked on this topic, because it
= 10 (x – 7y + 190)
involves many inequations.
Example 1. There are two factories located on at place P and The problem becomes: Minimise:
the other at place Q. Cost in Rupees Z = 10(x – 7y + 190)
From these locations From To
a certain commodity A B C
is to be delivered to P 160 100 150
each of the three depots Q 100 120 100
situated at A, B and C.
The weekly requirements of the depots are respectively 5, 5 and
4 units of the commodity while the production capacity of the
factories at P and Q are respectively 8 and 6 units. The cost of
the transportation per unit is given in the table, then
Number of units should be transported from each factory to each
depot in order to minimise the transportation cost are subject to the constraints:
(a) x = 1; y = 2 (b) x = 3; y = 6 x + y ≤ 8, x + y ≥ 4, x ≤ 5,
(c) x = 2; y = 2 (d) x = 0; y = 5
y ≤ 5, x ≥ 0, y ≥ 0.
Solution. The problem can be explained diagrammatically as
below: Plot these inequations:
The shaded region ABCDEF is the feasible region.
Vertices of ABCDEF are: A = (0, 4), B = (0, 5),
C = (3, 5), D = (5, 3), E = (5, 0), F = (4, 0)
ZA = ` 1,620, ZB = ` 1,550, ZC = ` 1,580,
ZD = ` 1,740, ZE = ` 1,950, ZF = ` 1,940
Zmin = ZB = ` 1,550 at (0, 5)
x = 0, y = 5.
Let x units of the commodity be sent to A from P, then (5 – x)
units will be sent to A from Q.
Let y units of the commodity is sent to B from P then (5 – y)
units will be sent from Q to B.
Then 8 – (x + y) units of the commodity will be sent from
P to C and 6 – (5 – x) – (5 – y) = x + y – 4, units of the commodity
will be sent from Q to C.
The commodity sent to A, B and C from P and Q should not
come back. Hence, the optimal transport strategy will be to deliver 0, 5

∴ x ≥ 0, y ≥ 0, 8 – x – y ≥ 0, 5 – x ≥ 0, 5 – y ≥ 0, and 3 units from P to A, B and C and 5, 0, 1 units from Q to


x+y–4≥0 A, B and C at a minimum cost of ` 1,550.
⇒ x ≥ 0, y ≥ 0, x + y ≤ 8, x ≤ 5, y ≤ 5 and x + y ≥ 4. Hence, option (d) is the correct answer.

370 Mathematics–12
E:\AMIT_WORKS\Exam_Guru\EG_Mathematics-12_(working_02-06-2022)\EG_Mathematics-12_working\Open_Files\Chap_12\Chap_12
\ 18-Aug-2022 Amit Proof-5 Reader’s Sign _______________________ Date __________

EXERCISE 12.3
I. Long Answer Type Questions 2. An oil company has two depots A and B with capacities of
1. Two godowns A and B have a grain capacity of 100 quintals 7,000 L and 4,000 L respectively. The company is to supply
and 50 quintals respectively. They supply to 3 ration shops oil to three petrol pumps D, E and F whose requirements are
D, E and F whose requirements are 60, 50 and 40 quintals 4,500 L, 3,000 L and 3,500 L respectively. The distances
respectively. The cost of transportation per quintal from (in km) between the depots and the petrol pumps is given
the godowns to the shops are given in the following table: in the following table:

Transportation cost per quintal (in `) Distance in km


From A(100) B(50) From A B
To To
D(60) 6 4 D 7 3
E(50) 3 2 E 6 4
F(40) 2.50 3 F 3 2

How should the supplies be transported in order that the Assuming that the transportation cost of 10 litre of oil is
transportation cost is minimum? What is the minimum `1 per km, how should the delivery be scheduled in order
cost? that the transportation cost is minimum?

Answers 12.3
I. 1. The transportation of grain from two depots A and B to three ⇒ ZA = ` 560, ZB = ` 620,

ration shops in shown below: ZC = ` 610, ZD = ` 510


Each transfer in position
∴ x ≥ 0, y ≥ 0, 100 – x – y ≥ 0, 60 – x ≥ 0,
Feasible region = Shaded closed region ABCD.
50 – y ≥ 0, x + y – 60 ≥ 0
⇒ Zmin = ZD = ` 510 at (10, 50) ⇒ x = 10, y = 50.
Cost of transportation: Supply:
Z = 6x + 3y + 2.50 (100 – x – y) + 4(60 – x)
A B Total
+ 2(50 – y) + 3(x + y – 60). D 10 50 60
⇒ Z = 2.5x + 1.5y + 410 (To be minimised) E 50 0 50
∴ Question becomes: Minimise: F 40 0 40
Total 100 50 150
Z = 2.5x + 1.5y + 410 subject to the constraints:
x + y ≤ 100, x + y ≥ 60, x ≤ 60, y ≤ 50, x ≥ 0, y ≥ 0
2. Let x litres of oil is supplied from depot A to petrol pump D and
y litres of oil is supplied from depot A to petrol pump E then
Plot these inequations:
7,000 – (x + y) litres of petrol will be supplied from depot A to
x 100 0 x 60 0 petrol pump F.
x + y = 100,
, x + y = 60 Now as requirement of oil at petrol pump D, E and F are 4,500 L,
y 0 100 y 0 60
3,000 L, and 3,500 L. So quantity of oil transported from depot
Vertices : A = (60, 0), B = (60, 40), B to petrol pumps D, E, F will be (4,500 – x) L, (3,000 – y) L
C = (50, 50), D = (10, 50) and 3,500 – (7,000 – x – y)] L respectively.

Linear Programming 371


E:\AMIT_WORKS\Exam_Guru\EG_Mathematics-12_(working_02-06-2022)\EG_Mathematics-12_working\Open_Files\Chap_12\Chap_12
\ 18-Aug-2022 Amit Proof-5 Reader’s Sign _______________________ Date __________

∴ x ≥ 0, y ≥ 0, 7000 – x – y ≥ 0 or  x + y ≤ 7,000 ZA = ` 5,300, ZB = ` 5,550, ZC = ` 5,450,


4500 – x ≥ 0 ⇒ x ≤ 4500; 3000 – y ≥ 0 ZD = 4,400, ZE = ` 5000.

⇒ y ≤ 3000; and 3500 – [7000 – x – y] ≥ 0


⇒ 3500 – 7000 + x + y ≥ 0 ⇒ x + y ≥ 3500
Cost of transportation:
Z = 0.7x + 0.6y + 0.3[7000 – (x + y)] + 0.3 (4500 – x) + 0.4
(3,000 – y) + 0.2 (x + y – 3500)
⇒ Z = 0.3x + 0.1y + 3,950.
Now the question reduces to minimise:
Z = 0.3x + 0.1y + 3,950 subject to the constraints:
x + y ≤ 7,000, x + y ≥ 3,500

Feasible region = Shaded closed region ABCDE.
x ≤ 4,500, y ≤ 3,000, x ≥ 0, y ≥ 0.
⇒ Zmin = ZD = ` 4,400 at (500, 3,000)
x 7, 000 0 ⇒ x = 500, y = 3,000
Graph: x + y = 7000
y 0 7, 000

x 3, 500 0
x + y = 3, 500
y 0 3, 500

Vertices: A = (4,500, 0), B = (4,500, 2,500)
C = (4,000, 3,000), D = (500, 3,000), E = (3,500, 0)

Case Based Questions


1. A train can carry a maximum of 300 passengers. A profit of (ii) Which among these is a constraint for this LPP?
` 800 is made on each executive class ticket and a profit of (a) x + y ≥ 300 (b) y ≥ 3x
` 200 is made on each economy class ticket. The railway
reserves at least 40 seats for the executive class. However, (c) x ≤ 40 (d) y ≤ 3x
at least 3 times as many passengers prefer to travel by (iii) Which among these is not a corner point for this LPP?
economy class, than by executive class. It is given that the (a) (40, 120) (b) (40, 260)
number of executive class ticket is x and that of economy
class ticket is y. (c) (30, 90) (d) (75, 225)
(iv) The maximum profit is:
(a) ` 56000 (b) ` 85000
(c) ` 205000 (d) ` 105000
(v) Which corner point the objective function has minimum
value?
(a) x = 40, y = 120 (b) x = 40, y = 260
(c) x = 75, y = 225 (d) x = 30, y = 90
Ans. (i) (a) Maximise Z = 800x + 200y

Based on the above information, answer the following (ii) (b) y ≥ 3x (iii) (c) (30, 90)
questions: (iv) (d) ` 105000 (v) (a) x = 40, y = 120
(i) The objective function of the LPP is: 2. Suppose a dealer in rural area wishes to purchase a number
(a) Maximise Z = 800x + 200y of sewing machines. He has only ` 5760 to invest and has
(b) Maximise Z = 200x + 800y space for at most 20 items for storage. An electronic sewing
(c) Minimise Z = 800x + 200y machine costs him ` 360 and a manually operated sewing
(d) Minimise Z = 200x + 800y machine ` 240. He can sell an electronic sewing machine

372 Mathematics–12
E:\AMIT_WORKS\Exam_Guru\EG_Mathematics-12_(working_02-06-2022)\EG_Mathematics-12_working\Open_Files\Chap_12\Chap_12
\ 18-Aug-2022 Amit Proof-5 Reader’s Sign _______________________ Date __________

at a profit of ` 22 and a manually operated sewing machine (b) the optimal solution occurs at every point on the
at a profit of ` 18. line joining these two points.
(c) the LPP under consideration is not solvable.
(d) the LPP under consideration must be reconstructed.
Ans. (i) (c) x + y > 0 (ii) (b) (8, 12)
(iii) (c) (8, 12) (iv) (c) (8, 12)
(v) (b) the optimal solution occurs at every point on the

line joining these two points.
Based on the above information, answer the following
3. Deepa rides her car at 25 km/hr. She has to spend
questions:
` 2 per km on diesel and if she rides it at a faster speed of
(i) Let x and y denote the number of electronic sewing
machines and manually operated sewing machines 40 km/hr, the diesel cost increases to ` 5 per km. She has
purchased by the dealer. If it is assumed that the dealer ` 100 to spend on diesel. Let she travels x kms with a speed
purchased atleast one of the given machines then: 25 km/hr and y kms with a speed 40 km/hr. The feasible
(a) x + y ≥ 0 (b) x + y < 0 region for the LPP is shown below:
(c) x + y > 0 (d) x + y ≤ 0
(ii) Let the constraints in the given problem is represented
by the following inequalities:
x + y ≤ 20, 360x + 240y ≤ 5760, x, y ≥ 0
Then which of the following points lies in its feasible
region?
(a) (0, 24) (b) (8, 12)
(c) (20, 2) (d) None of these
(iii) If the objective function of the given problem is

maximise z = 22x + 18y, then its optimal value occurs
at: Based on the above information, answer the following
(a) (0, 0) (b) (16, 0) questions:
(c) (8, 12) (d) (0, 2) (i) What is the point of intersection of line l1 and l2?
(iv) Suppose the given shaded region APDO represent  40 50   50 40 
the feasible region corresponding to mathematical (a)  ,  (b)  , 
 3 3  3 3
formulation of given problem.
 −50 40   −50 −40 
(c)  ,  (d)  , 
 3 3  3 3 
(ii) The corner points of the feasible region shown in above
graph are:
 40 50 
(a) (0, 25), (20, 0),  , 
 3 3
(b) (0, 0), (25, 0), (0, 20)
 40 50 
(c) (0, 0),  ,  , (0, 20)
 3 3
Then which of the followings represents the coordinates
of one of its corner points?  50 40 
(d) (0, 0), (25, 0),  ,  , (0, 20)
 3 3
(a) (0, 24) (b) (12, 8)
(iii) If Z = x + y be the objective function and max Z = 30.
(c) (8, 12) (d) (6, 14)
The maximum value occurs at point:
(v) If an LPP admits optimal solution at two consecutive
vertices of a feasible region, then:  50 40 
(a)  ,  (b) (0, 0)
 3 3
(a) the required optimal solution is at the mid-point of
the line joining two points. (c) (25, 0) (d) (0, 20)

Linear Programming 373


E:\AMIT_WORKS\Exam_Guru\EG_Mathematics-12_(working_02-06-2022)\EG_Mathematics-12_working\Open_Files\Chap_12\Chap_12
\ 18-Aug-2022 Amit Proof-5 Reader’s Sign _______________________ Date __________

(iv) If Z = 6x – 9y be the objective function, then maximum Ans. (i) (a) linear programming problem
value of Z is: (ii) (a) Maximum investment is ` 50,000 and storage space
(a) –20 (b) 150 (c) 180 (d) 20 is for maximum of 60 pieces
(v) If Z = 6x + 3y be the objective function, then what is (iii) (b) ` 5,000 (iv) (a) ` 4,500
the minimum value of Z? (v) (c) ` 6,250
(a) 120 (b) 130 (c) 0 (d) 150 5. A small firm manufactures necklaces and bracelets. The
combined number of necklaces and bracelets that it can
 50 40  handle per day is at most 24. The bracelet takes one hour
Ans. (i) (b)  , 
 3 3 to make and the necklace takes half an hour. The maximum
 50 40 
(ii) (d) (0, 0), (25, 0),  ,  , (0, 20) number of hours available per day is 16. The profit on the
 3 3
 50 40  bracelet is ` 2 and the profit on the necklace is ` 1.
(iii) (a)  ,  (iv) (b) 150 (v) (c) 0
 3 3
4. A furniture dealer deals in only two items-tables and
chairs. He has ` 50,000 to invest and has storage space
of atmost 60 pieces. A table costs ` 2,500 and a chair
` 500. He estimates that from the sale of one table, he can
make a profit of ` 250 and that from the sale of one chair
a profit of ` 75. He wants to know how many tables and
chairs he should buy from the available money so as to
maximise his total profit assuming that he can sell all the
items which he buys.
Based on the above information, answer the following
questions: The graph of the equations formed is shown here.
(i) The above stated optimisation problem is an example
of:
(a) linear programming problem
(b) functional programming problem
(c) computer programming problem
(d) None of these
(ii) Which of the followings is constraints for the given
problem?
(a) Maximum investment is ` 50,000 and storage space
is for maximum of 60 pieces
(b) Minimum investment is ` 50,000 and storage space
is for minimum of 60 pieces
(c) Maximum investment is ` 50,000 and storage space
is for minimum of 60 pieces
(d) None of these
(iii) If he buys tables only, then his profit will be: Based on the above information, answer the following
(a) ` 500 (b) ` 5,000 questions:
(i) What is the first inequality formed?
(c) ` 7,500 (d) ` 2,500
(a) x + y ≤ 24 (b) x + y ≥ 24
(iv) Suppose he chooses to buy chairs only. Then, his
(c) x + y < 24 (d) x + y > 24
maximum profit is:
(ii) What is the second inequality formed?
(a) ` 4,500 (b) ` 7,500
(a) x + y ≤ 32 (b) x + 2y ≤ 32
(c) ` 4,000 (d) ` 5,700
(c) x + y ≥ 32 (d) x + 2y ≥ 32
(v) If he chooses 10 tables and 50 chairs, then his maximum
(iii) In order to maximize the profits per week how many
profit is: necklaces and bracelets should be made?
(a) ` 6,200 (b) ` 6,500 (a) 10 and 5 (b) 12 and 8
(c) ` 6,250 (d) ` 6,520 (c) 16 and 8 (d) 14 and 5

374 Mathematics–12
E:\AMIT_WORKS\Exam_Guru\EG_Mathematics-12_(working_02-06-2022)\EG_Mathematics-12_working\Open_Files\Chap_12\Chap_12
\ 18-Aug-2022 Amit Proof-5 Reader’s Sign _______________________ Date __________

(iv) What would be the profit if 5 necklaces and 5 bracelets


The graph of the equations formed is shown here:
are made?
(a) 15 (b) 30
(c) 17 (d) 25
(v) What would be the maximum possible profit?
(a) 20 (b) 24
(c) 16 (d) 32
Ans. (i) (a) x + y ≤ 24
(ii) (b) x + 2y ≤ 32
(iii) (c) 16 and 8
(iv) (a) 15
(v) (d) 32 Based on the above information, answer the following
questions:
6. A company produces two articles X and Y. There are two
(i) What is the first inequality formed?
departments through which the articles are processed viz.
assembling and finishing. The potential capacity of the (a) 4x + 2y ≥ 60 (b) 4x + 2y ≤ 60
assembly department is 60 hours a week and that of the (c) 2x + 4y ≥ 60 (d) 2x + 4y ≤ 60
finishing department is 48 hours a week. Product of one (ii) What is the second inequality formed?
unit of X requires 4 hours in assembling and 2 hours in (a) x + y ≤ 48 (b) x + 4y ≤ 48
finishing. Each of the unit Y requires 2 hours in assembling (c) 2x + y ≤ 48 (d) 2x + 4y ≤ 48
and 4 hours in finishing. Profit is ` 8 for each unit of X and (iii) In order to maximize the profits, per week how many
` 6 for each unit of Y. units of article X and Y should be made?
(a) 10 and 5 (b) 5 and 10
(c) 12 and 6 (d) 6 and 12
(iv) What would be the profit if 5, X articles and 5, Y articles
are made?
(a) ` 70 (b) ` 60 (c) ` 50 (d) ` 40
(v) What would be the maximum possible profit?
(a) ` 120 (b) ` 132 (c) ` 72 (d) ` 112
Ans. (i) (b) 4x + 2y ≤ 60 (ii) (d) 2x + 4y ≤ 48
(iii) (c) 12 and 6 (iv) (a) ` 70
(v) (b) ` 132

Author’s Comments
Questions based on following types are very important for Exams. So, students are advised to revise them thoroughly.
Questions based on Graphical Method of Solving LPP (bounded or unbounded) related to:
(i) Diet Problem (ii) Manufacturing Problem
(iii) Allocation Problem

IMPORTANT METHODS
Here it is very important to learn how to convert the word problems into inequations and objective functions.
Students are advised to have a very good practice of this process.
1. First form the objective functions and inequations.
2. Plot linear inequations and find the feasible region (shading).
3. Note the corner points of the feasible region.
4. Put these points in the objective functions and find the maximum and minimum value of the objective functions.

Linear Programming 375


E:\AMIT_WORKS\Exam_Guru\EG_Mathematics-12_(working_02-06-2022)\EG_Mathematics-12_working\Open_Files\Chap_12\Chap_12
\ 18-Aug-2022 Amit Proof-5 Reader’s Sign _______________________ Date __________

COMMON ERRORS
ERRORS CORRECTIONS
(i) Students commit mistake in forming inequations of (i) Give emphasis on conversion of word problems to
constraints and objective function. mathematical terms.
(ii) Confuse between atleast, atmost usually write ≤ for atleast (ii) Write ≥ for atleast and ≤ for atmost.
and ≥ for atmost.
(iii) Wrong shade of the feasible region. (iii) Correctness increases with maximum practice.
(iv) If feasible region is unbounded, then unable to identify (iv) Do many problems involving such questions.
whether the half plane determined by ax + by > M or ax
+ by < M has a point in common with feasible region or
not.
(v) Forget to write the final answer. (v) Reminding again and again.

REVISION CHART

Linear Programming Problem


(LPP)

Linear programming is concerned with finding the optimal value (maximum or minimum) of a linear function of several
variables (called objective function) subject to the conditions that the variables are non-negative and satisfy a set of linear
inequations (called linear constraints).
The common region shaded determined by all the constraints including the non-negative constraints x ≥ 0, y ≥ 0
of a linear programming problem is called a feasible region or solution region for the problem. Points within and on the
boundary of the feasible regions represents feasible solution of the constraints. Any point out of the feasible region is an
infeasible solution.
Any point in the feasible region that gives optimal value (maximum or minimum) of the objective function is called
an optimum solution.

Evaluate the objec tive


function Z = ax + by at m is the minimum value of
each corner point. Let M the objective function, if the
and m respectively be the open half plane determined
largest and smallest value of Corner Point Method by ax + by < m has no
these points. If the feasible of Solving a LPP point in common with the
region is bounded, M and m feasible region, otherwise
respectively use the maximum the objective function has no
and minimum values of the minimum value.
objective function.
Find the feasible region of
the linear programming
problem and determine
its corner points.

376 Mathematics–12
E:\AMIT_WORKS\Exam_Guru\EG_Mathematics-12_(working_02-06-2022)\EG_Mathematics-12_working\Open_Files\Chap_13\Chap_13
\ 18-Aug-2022 Amit Proof-5 Reader’s Sign _______________________ Date __________

13 Probability

Topics Covered
13.1 Probability 13.2 Conditional Probability
13.3 Multiplication Theorem on Probability 13.4 Independent Events
13.5 Partition of the Sample Space 13.6 Random Variable

C hapter map
PROBABILITY

Conditional Bayes' Independent


Probability Theorem Events

Theorem of
Total Probability
Understanding Properties of Multiplication Probability
Conditional Conditional Theorem on Distribution of a
Probability Probability Probability random variable

Topic 1. Probability
Probability is a function of outcomes of an experiment. Events
When two coins are tossed once or one coin is tossed twice we
Experiment get HH, TT, HT and TH as outcomes. Hence
The game we play is called an experiment. For example: (i) To get both heads is an event.
(i) Tossing of a coin 4 times is an experiment. (ii) To get both tails is another event.
(ii) Drawing of a card from a well shuffled pack of 52 playing (iii) To get one head and one tail is again an event.
cards is an experiment.
Favourable Outcomes
Outcomes The outcomes of an experiment which help an event to occur
The all possible results of an experiment are called outcomes. are called favourable outcomes to that event.
(i) When we toss a coin we get either a head or a tail. Here, When we toss 2 dice once and event is to get a doublet,
heads and tails are outcomes of the experiment. then the favourable outcomes will be:
{(1, 1), (2, 2); (3, 3); (4, 4), (5, 5), (6, 6)}
(ii) When we throw a die, we get either 1, 2, 3, 4, 5 or 6. These
Number of favourable outcomes of an event is represented
numbers 1, 2, 3, 4, 5 and 6 are outcomes of the experiment.
as n(E).
Sample space: A set of all possible outcomes of an

experiment is called sample space. When we toss a coin Probability of an Event E:
once, we get either a head or a tail. n(E)
Probability of event E = P(E) = n(S)
∴ Sample space = S = {H, T} and n(S) = 2
When we toss two coins, then all possible outcomes are 2
Example 1. If A and B are such that P(A  B) = and
3
HH, TT, TH or HT.
P(A   B) = 5 , then P(A) + P(B) is equal to .................... .
∴ S = {HH, TT, TH, HT} and n(S) = 4 9

377
E:\AMIT_WORKS\Exam_Guru\EG_Mathematics-12_(working_02-06-2022)\EG_Mathematics-12_working\Open_Files\Chap_13\Chap_13
\ 18-Aug-2022 Amit Proof-5 Reader’s Sign _______________________ Date __________

2 Example 4. A pair of dice is tossed once. If event A is to get a doublet


Solution. Here, P(A  B) = and P(A  B) = 5 and event B is to get an odd number on each die. Then find:
3 9
2 (i) P(A) (ii) P(B)

\ 1 – P(A  B) =
3 (iii) P(A ∩ B) (iv) P(A ∪ B)

P(A  B) = 1 − 2 = 1
3 3 (v) P (to get either A or B) (vi) P (to get neither A nor B)
Now P(A) + P(B) = 1 – P(A) + 1 – P(B) (vii) P(A′) (viii) P(B′) (ix) P(A′∩ B)
= 2 – [P(A) + P(B)] (x) P(A ∩ B′) (xi) P(A′ ∪ B′).
= 2 – [P(A  B) + P(A  B)] Solution. n(S) = 36
5 1 8 10
= 2− +  = 2− = (i) A = {a doublet} = {(1, 1), (2, 2), (3, 3), (4, 4),
 9 3 9 9
 (5, 5), (6, 6)}
10
Hence, the value of the filler is ⇒ n(A) = 6
9
n (A) 6 1
Example 2. A and B are events such that P(A) = 0.4, P(B) = 0.3 \ P(A) = = = .
and P(A  B) = 0.5. Then P(B  A) equals n (S) 36 6
2 1 3 1 (ii)
B = {an odd number on each die}
(a) (b) (c) (d)
3 2 10 5
= {(1, 1), (1, 3), (1, 5), (3, 1), (3, 3)), (3, 5),
Solution. Given that: P(A) = 0.4, P(B) = 0.3 and P(A  B) = 0.5
 (5,1), (5,3), (5, 5)}
P(A  B) = P(A) + P(B) – P(A  B)
0.5 = 0.4 + 0.3 – P(A  B) ⇒ n(B) = 9
P(A  B) = 0.4 + 0.3 – 0.5 = 0.2 n ( B) 9 1
\ P(B) = = = .
\ P(B  A) = P(A) – P(A  B) n (S) 36 4
1 (iii) (A ∩ B) = {(1, 1), (3, 3), (5, 5)} ⇒ n(A ∩ B) = 3
= 0.4 – 0.2 = 0.2 =
5
Hence, the correct option is (d). n(A ∩ B) 3 1
\ P(A ∩ B) = = =
Example 3. A pair of dice is tossed once. Find the probability n(S) 36 12
of getting:
(iv) P(A ∪ B) = P(A) + P(B) – P(A ∩ B)
(i) A doublet;
1 1 1 2 + 3 −1 4 1
(ii) An even number on both the dice. = + − = = =
6 4 12 12 12 3
(iii) Sum of the numbers on two dice is 7.
1
Solution. Here, n(S) = 36 (v) P(either A or B) = P(A ∪ B) =
3
(i) P (doublet) = P(D)
(vi) P (neither A nor B) = P(A′ ∩ B′)
D = {(1, 1); (2, 2); (3, 3); (4, 4); (5, 5); (6, 6)}
1 2
⇒ n(D) = 6 and n(S) = 36. = P ( A ∪ B)′ = 1 − P ( A ∪ B) = 1 − =
3 3
n(E) 6 1
∴ P(D) = = = . 1 5
n(S) 36 6 (vii) P(A′) = 1 – P(A) = 1 − =
(ii) P (even number on both the dice) = P(E) 6 6
E = {(2, 2); (2, 4); (2, 6); (4, 2); (4, 4); (4, 6); 1 3
(viii) P(B′) = 1 – P(B) = 1 − =
 (6, 2); (6, 4); (6, 6)} 4 4
⇒ n(E) = 9 and n(S) = 36 1 1 1
(ix) P(A′ ∩ B) = P(B) – P(A ∩ B) = − =
n(E) 9 1 4 12 6
-∴ P(E) = = = .
n(S) 36 4
(x) P(A ∩ B ′) = P(A) – P(A ∩ B) = 1 − 1 = 1
(iii) P (sum of the two numbers on two dice is 7) = P(E). 6 12 12
E = {(6, 1); (5, 2); (4, 3); (3, 4); (2, 5); (1, 6)}
(xi) P(A′ ∪ B′) = P(A ∩ B)′ = 1 – P(A ∩ B) = 1 − 1 = 11
n(E) 6 1 12 12
⇒ n(E) = 6 ⇒ P(E) = = = .
n(S) 36 6 Example 5. An urn contains 4 white, 3 black and 5 red balls. 3
Note: Remember:
balls are drawn one by one with replacement. Find the probability
(i) P(A′) = 1 – P(A)
of getting:
(ii) P(A ∪ B) = P(A) + P(B) – P(A ∩ B)
(i) All the balls white,
(iii) P(A or B) = P(A ∪ B)
(iv) P(Neither A nor B) = P(A′ ∩ B′) = P(A ∪ B)′ = (ii) all the balls of same colour.
 1 – P(A ∪ B) (iii) 2 white balls and 1 black ball
(v) P(A ∩ B′) = P(A) – P(A ∩ B) (iv) all the balls are of different colours,
(vi) P(A′ ∩ B) = P(B) – P(A ∩ B) (v) 2 white balls.

378 Mathematics–12
E:\AMIT_WORKS\Exam_Guru\EG_Mathematics-12_(working_02-06-2022)\EG_Mathematics-12_working\Open_Files\Chap_13\Chap_13
\ 18-Aug-2022 Amit Proof-5 Reader’s Sign _______________________ Date __________

Solution. The urn contains 4 white, 3 black and 5 red balls. 4×3× 2
4
Total number of balls = 12. C3 3 × 2 ×1
(i) P(3W) = 12 =
Draw is with replacement: C3 12 × 11 × 10
3 × 2 ×1
(i) P(3 balls white) = P(W).P(W).P(W)
4×3× 2 1
4 4 4 1 = = .
= × × = 12 × 11 × 10 55
12 12 12 27
4
(ii) P(3 balls of same colour) = P(3 white or 3 black or 3 red C2 × 3C1
(ii) P(2W and 1 black) = 12
balls) C3
= P(3 white) + P(3 black) + P(3 red) 4×3 3
×
= P(W) . P(W) . P(W) + P(B) P(B) P(B) + P(R) P(R) P(R) = 1 ×2 1 = 9
12 × 11 × 10 110
4 4 4 3 3 3 5 5 5
= × × + × × + × × 1× 2 × 3
12 12 12 12 12 12 12 12 12
(iii) P (same colour balls) = P(3W or 3 B or 3 R)
64 27 125 216 1 = P(3W) + P(3B) + P(3R)
= + + = = .
1728 1728 1728 1728 8 4
C3 3C3 5
C
= + 12 + 12 3
3! 12
(iii) P(2W and 1B) = P ( W ) ⋅ P ( W ) ⋅ P ( B) C3 C3 C3
2 !1!
( )( )( )
4 4 3 1 5×4×3
= 3 = 4 +1+
12 12 12 12 3 × 2 ×1
= 12
C3
(iv) P(balls of different colours)
4 + 1 + 10
 3! 
= 
 1! 1! 1!
P ( W ) ⋅ P ( B) P ( R ) = 6
4 3 5
12 12 12
=
5
( )( )( )
24
=
12 × 11 × 10
3× 2 ×1
(v) P(2 white) = P(2 white and 1 non-white) 15 × 6 3
= = .
= P(2W and 1 W′) 12 × 11 × 10 44

 3!  (iv) P(3 different colours balls) = P(1 W,1 B and 1 R)


=  P ( W ) ⋅ P ( W ) P ( W′ )
 2 !1! 4
C1 × 3C1 × 5C1
=
= 3 ( )( )( )
4 4 8
12 12 12
=
2
9
=
12

4×3×5
C3

Example 6. A box contains 4 white, 3 black and 5 red balls. 3 12 × 11 × 10


balls are drawn one by one without replacement. Find the: 3× 2 ×1
4×3×5 3
(i) P(3 white balls), = = .
2 × 11 × 10 11
(ii) P(2 white and 1 black ball),
4
(iii) P(all the 3 balls are of same colour) C2 × 8C1
(v) P(2 white and 1 non-white) = 12
(iv) P(3 balls are of different colours), C3
6×8
(v) P(2 balls are white). = 12 × 11 × 10
Solution. The given box contains 4 white, 3 black and 5 red balls. 3× 2 ×1
Total number of balls = 12 6×8 12
It is a case of draw without replacement: = 2 × 11 × 10 = 55

EXERCISE 13.1
I. Multiple Choice Questions (MCQs) drawn one by one with replacement. The P (4 white balls)
1. A pair of dice is tossed. The probability of getting a doublet is
is 1 1 3 1
(a) (b) (c) (d)
1 1 44 55 4 27
(a) (b)
6 4 II. Short Answer Type Questions
1 1 1. A card is drawn at random from a well shuffled pack of
(c) (d)
3 2 52 cards. Find the probability that it is neither an ace nor
2. A box contains 4 white, 3 black and 5 red balls. 3 balls are a king. [Delhi 2007]

Probability 379
E:\AMIT_WORKS\Exam_Guru\EG_Mathematics-12_(working_02-06-2022)\EG_Mathematics-12_working\Open_Files\Chap_13\Chap_13
\ 18-Aug-2022 Amit Proof-5 Reader’s Sign _______________________ Date __________

2. Two dice are thrown together. What is the probability that (ii) P (to get 3 balls of same colours)
sum of the numbers on the two dice is neither 9 nor 11? (iii) P(to get 3 balls of different colours),
[A.I. 2007] (iv) P(to get 2 white ball).
3. From a well shuffled pack of 52 playing cards, 4 cards are
5. A bag contains 7 red, 4 white and 5 black balls. Two balls
drawn one by one with replacement. Find the following
are drawn at random from the bag. What is the probability
probabilities:
that both the balls are white? [A.I. 2010]
(i) P(To get 4 spades)
(ii) P (To get 4 aces) 6. Two balls are drawn at random from a bag containing 2
white, 3 red, 5 green and 4 black balls, one by one without
(iii) P(To get cards of same suits)
replacement. Find the probability that the two balls drawn
(iv) P(To get 2 aces and 2 kings)
are of different colours. [Delhi 2009]
(v) P(To get 3 red cards).
7. An urn contains 7 red and 4 blue balls. Two balls are drawn
4. A box contains 5 white, 4 red and 3 black balls. 3 balls are
drawn at random without replacement. Find the following at random with replacement. Find the probability of getting
probabilities. (i) 2 red balls (ii) 2 blue balls (iii) 1 red and 1 blue ball.
(i) P(to get all the 3 balls red),  [Delhi 2007, A.I. 2007]

Answers 13.1
I. 1. (a)
1
2. (d)
1 (iii) 1
6 27 64
Hint: There are 13 cards of same suit and then 4 suits out
II. 1. 11 . of 52 cards.
13
4 4 \ P(4 cards of 4 suits drawn with replacement)
Hint: P(Ace) = P(A) =
, P(King) = P(B) =
52 52 = 4  13 × 13 × 13 × 13  = 1
P(A ∩ B) = 0  52 52 52 52  64
P(neither A nor B) = P(A′ ∩ B′) = P(A ∪ B)′ (iv) 6
28561
= 1 – P(A ∪ B)
Hint: There are 4 aces and 4 kings in 52 cards.
= 1 – [P(A) + P(B) – P(A ∩ B)]
\ P(2 aces and 2 kings drawn with replacement)

( )
2. 5
6 =  4!  4 × 4 × 4 × 4 = 6
 2! 2! 52 52 52 52 28561
Hint: A = {(6, 3), (5, 4), (4, 5), (3, 6)}
n(A) = 4 (v) 1
8
P(Sum 9) = P(A) = 4 Hint: There are 26 red cards out of 52 cards
36 \ P(3 red cards drawn with replacement out of 52 cards)
B = {(6, 5), (5, 6)} ⇒ n(B) = 2
= 26 × 26 × 26 = 1
Also A∩B= f 52 52 52 8
P(Sum 11) = P(B) 1
4. (i)
2 55
⇒ P(B) = Hint: 5 white, 4 red and 3 black balls.
36
P(A ∩ B) = 0 ⇒ Total number of balls = 12 balls
P(neither A nor B) = P(A′ ∩ B′) = P(A ∪ B)′ P(3 red balls drawn without replacement out of 12 balls)
4 4
= 1 – P(A ∪ B) C3 C 4
= = 12 1 =
= 1 – [P(A) + P(B) – P(A ∩ B)]
12
C3 C3 12 × 11 × 10
1× 2 × 3
3. (i) 1
= 4 × 6 1
256 =
12 × 11 × 10 55
Hint: There are 13 spades out of 52 cards 3
(ii)
\ P(4 spades drawn with replacement) 44
Hint: To get 3 balls of same colour means to get 3 balls of
= 13 × 13 × 13 × 13 = 1 white colour or 3 balls of red colour or 3 balls of black colour
52 52 52 52 256
out of 12 balls.
(ii) 1
28561 \ P(3 balls of same colour drawn without replacement out of
12 balls)
Hint: There are 4 aces out of 52 cards
= P(3W) + P(3R) + P(3B)
\ P(4 aces drawn with replacement) 5
C3 + 4C3 + 3C3 3
= 4 × 4 × 4 × 4 = 1 = 12
C3
=
44
52 52 52 52 28561

380 Mathematics–12
E:\AMIT_WORKS\Exam_Guru\EG_Mathematics-12_(working_02-06-2022)\EG_Mathematics-12_working\Open_Files\Chap_13\Chap_13
\ 18-Aug-2022 Amit Proof-5 Reader’s Sign _______________________ Date __________

P(To draw 2 balls of different colours out of 14 balls without


(iii) 3
11 replacement)
Hint: To draw three balls of different colours means to draw = P(1W 1R) + P(1W1G) + P(1W 1B)
1 white, 1 red and 1 black ball out of 12 balls)
+ P(1R 1G) + P(1R 1B) + P(1G 1B)
\ P(3 balls of different colours drawn without replacement out
of 12 balls)
=
( C × C ) + ( C × C ) + .....
2
1
3
1
2
1
5
1
5 14
C1 × 4C1 × 3C1 3 C2
= 12 =
C3 11
7 7. (i) 49
(iv) 121
22
Hint: There are 7 red and 4 black balls.

Hint: P(2W and 1 non-white drawn without replacement
out of 12 balls). Total number of balls = 11 balls
5 7
C2 × C1 7 P (2 red balls drawn with replacement)
= 12 =
C3 22 = P(1R) . P(1R)
1 = 7 × 7 = 49
5.
20 11 11 121
Hint: 7 red, 4 white and 5 black balls.

(ii) 16
Total number of balls = 16 balls 121
P(2 white balls are drawn without replacement out of 16 balls) Hint:
P(2B) = P(B) . P(B) = 4 × 4 = 16
4 11 11 121
C
= 16 2 = 1 (iii) 56
C2 20 121
6. 71 Hint: P(1R 1B) = 2P(R) . P(B)

91
Hint: There are 2 white, 3 red, 5 green and 4 black balls.
= 2 × 7 × 4 = 56
Total number of balls = 14 11 11 121

Topic 2. Conditional Probability


If E and F are two events associated with the same sample of a P(A  B) = P(A) + P(B) – P(A  B)
random experiment, then the conditional probability of event E 5 1 p 5 1 2p 2 2p
given that event F has already occurred is denoted by P(E|F) and = p+ −  − =  =
9 3 3 9 3 3 9 3
is given by 1
 p=
P ( E ∩ F) 3
P(E|F) = ; P ( F ) ≠ 0. 1
P ( F) Hence, p is equal to .
3
Properties of Conditional Probability Example 2. If A and B are two events such that P(A) > 0 and
Property 1: Let E and F be the events of a sample space S of an P(B′ )
P(A) + P(B) > 1, then P ( B A )  1 − .
experiment, then P(A)
(i) P(S|F) = 1, (ii) P(S|E) = 1, Solution. False [ P ( B A )
(iii) P(E|E) = 1, (iv) P(F|F) = 1. = P(A ∩ B) = P(A) + P(B) − P(A ∪ B) > 1 − P(A ∪ B) ]
P(A) P(A) P(A)
Property 2: Let A, B and F be three events of a sample space S 4 7 B
such that P(S) ≠ 0, then, Example 3. If P(A) = and P(A  B) = , then P   is
5 10  A
equal to
P[(A ∪ B) | F] = P(A|F) + P(B|F) – P(A ∩ B) | F]
1 1 17 7
Property 3: If E and F are two events of a sample space S then (a) (b) (c) 8 (d)
10 8 20
P(E′|F) = 1 – P(E|F).
4 7
Example 1. If A and B are two events such that P ( A B) = p, Solution. Given that: P(A) = and P(A  B) = 10
5
1 5
P(A) = p, P(B) = and P(A  B) = , then p is equal to ......... . B P(A ∩ B) 7/10 7
3 9 \ P   =
= =
1 5  A P(A) 4/5 8
Solution. Given that: P(A) = p, P(B) = and P(A  B) = Hence, the correct option is (c).
3 9
P(A ∩ B) 7 17  A
P ( A B) = =p Example 4. If P(A  B) = 10 and P(B) = , then P   equals
20
P(B) B
1 14 17 7 1
 P(A  B) = p . P(B) = p .
and (a) (b) (c) (d)
3 17 20 8 8

Probability 381
E:\AMIT_WORKS\Exam_Guru\EG_Mathematics-12_(working_02-06-2022)\EG_Mathematics-12_working\Open_Files\Chap_13\Chap_13
\ 22-Aug-2022 Amit Proof-5 Reader’s Sign _______________________ Date __________

7 17  A B A  P(A ∪ B)
Solution. Given that: P(A  B) = 10 and P(B) = (c) P   . P   = 1 (d) P  =
20  B  A B P(B)
 A P(A ∩ B) 7/10 14 Solution. Given that: A ≠ f and B  f,

\ P   = = =
B P(B) 17/20 17
 A P(A ∩ B)
Hence, the correct option is (a). then P   =
  B P(B)
3 2 3 Hence, the correct option is (b).
Example 5. If P(A) = , P(B) = and P(A  B) = , then
10 5 5
 B  A 7 9 4
P   + P   equals to Example 9. Let P(A) = , P(B) = and P(A  B) = . Then
 A  B  A′ 
13 13 13
P   is equal to
5 7  B
1 1
(a) (b) (c) 12 (d) 12 5
4 3 (a)
6
(b)
4
(c)
4
(d)
3 2 3 13 13 9 9
Solution. Here, P(A) = , P(B) = and P(A  B) =
10 5 5 7 9 4
Solution. Given that: P(A) = , P(B) = and P(A  B) =
P(A  B) = P(A) + P(B) – P(A  B) 13 13 13
3 3 2  A′  P(A ′ ∩ B) P(B) − P(A ∩ B)

= + − P(A ∩ B) P  = =
5 10 5  B P(B) P(B)
3 2 3 3+ 4−6 1 9 4 5
 P(A  B) = + − = = −
5
10 10
10 5 5 = 13 13 = 13 =
 A 9 9 9
 B
Now P   + P   13 13
B A
P(A ∩ B) P(A ∩ B) 1/10 1/10 1 1 7
Hence, the correct option is (d).
= + = + = + = Example 10. If A and B are such events that P(A) > 0 and
P(B) P(A) 2/5 3/10 4 3 12
 A′ 
Hence, the correct option is (d). P(B)  1 then P   equals
1 B′ 
Example 6. If A and B are two events such that P(A) = ,
1 1 2  A  A′ 
P(B) = and P ( A B) = , then P(A  B) equals (a) 1 – P   (b) 1 – P  
3 4 B B
3
(a) 1
(b) 3
(c) 1 (d) (c) 1 − P(A ∪ B) (d) P(A′)
12 4 4 16 P(B′ ) P(B′ )
1 1  A 1 Solution. Given that: P(A) > 0 and P(B)  1
Solution. Given that: P(A) = , P(B) = and P   =
2 3 B 4  A′  P(A ′ ∩ B′ ) 1 − P(A ∪ B)
 A P(Α ∩ B)
\ P   = =
P   = B′  P(B′ ) P(B′ )
B P(B) Hence, the correct option is (c).
1 P(Α ∩ B) 1 1 1
=  P(A  B) = × = Example 11. Two events E and F are independent. If P(E) = 0.3
4 1/3 4 3 12
 E  F
NowP(A  B) = 1 – P(A  B) and P(E  F) = 0.5, then P   − P   equals
F E
= 1 – [P(A) + P(B) – P(A  B)]
9 3 1 2 3 1 1
= 1 −  1 + 1 − 1  = 1 −  5 − 1  = 1 − = = (a)
7
(b)
35
(c)
70
(d)
7
 2 3 12   6 12  12 12 4
Solution. Given that: E and F are independent events such that
Hence, the correct option is (c).
P(E) = 0.3 and P(E  F) = 0.5
B
Example 7. If P(A) = 0.4, P(B) = 0.8 and P   = 0.6 then P(E  F) = P(E) + P(F) – P(E  F)
 A
P(A  B) is equal to 0.5 = 0.3 + P(F) – P(E).P(F)
(a) 0.24 (b) 0.3 (c) 0.48 (d) 0.96  0.5 – 0.3 = P(F) [1 – P(E)]  0.2 = P(F) (1 – 0.3)
B  0.2 = P(F).(0.7)
Solution. Given that: P(A) = 0.4, P(B) = 0.8 and P   = 0.6
 A 0.2 2

\ P(F) = =
 B P(A ∩ B) P(A ∩ B) 0.7 7
P  =  0.6 =
 A P(A) 0.4  E  F P(E ∩ F) P(E ∩ F)
\ P(A  B) = 0.6 × 0.4 = 0.24 Now P   − P   = −
F E P(F) P(E)
P(A  B) = P(A) + P(B) – P(A  B)
P(E).P(F) P(E).P(F)
= 0.4 + 0.8 – 0.24 = 1.20 – 0.24 = 0.96 = −
Hence, the correct option is (d). P(F) P(E)
Example 8. If A and B are two events and A  f, B  f, then 3 2 1
= P(E) – P(F) = − =
P(A ∩ B) 10 7 70
 A  A
(a) P   = P(A).P(B) (b) P   = Hence, the correct option is (c).
 B  B P(B)

382 Mathematics–12
E:\AMIT_WORKS\Exam_Guru\EG_Mathematics-12_(working_02-06-2022)\EG_Mathematics-12_working\Open_Files\Chap_13\Chap_13
\ 18-Aug-2022 Amit Proof-5 Reader’s Sign _______________________ Date __________

Example 12. In a college, 30% students fail in Physics, 25% fail (v) P(A|B′), (vi) P(B|A′),
in Mathematics and 10% fail in both. One student is chosen at
random. The probability that she fails in Physics if she has failed (vii) P(A′|B′) (viii) P(B′|A′).
in Mathematics is 9 8 47
Solution. Given: P(A) = , P ( B) = and P ( A ∪ B) = .
1 2 9 1 20 15 60
(a) (b) (c) (d)
10 5 20 3
Now P(A ∪ B) = P(A) + P(B) – P(A ∩ B)
Solution. Let E1 be the event that the student fails in Physics and
47 9 8
E2 be the event that she fails in Mathematics. ⇒ = + − P ( A ∩ B)
60 20 15
30 25

\ P(E1) = , P(E2) = 9 8 47 1
100 100 ⇒ P(A ∩ B) = + − =
10 20 15 60 5
and P(E1  E2) =
100 1

\ P ( E1 E 2 ) =
P(E1 ∩ E 2 ) 10/100 2
= = P ( A ∩ B) 5 1 15 3
(i) P(A|B) = = = × =
P(E 2 ) 25/100 5 P ( B) 8 5 8 8
Hence, the correct option is (b). 15
Example 13. A card is drawn from a well shuffled pack of 52 1
cards at random. Find the probability of getting an ace when it is P ( A ∩ B) 5 = 1 × 20 = 4
(ii) P(B|A) = =
given that card drawn was black card. P (A) 9 5 9 9
Solution. Total number of outcomes = n(S) = 52. 20
Let E be the event to get an ace and F be the event to get a
(iii) P(A′ | B) = 1 − P ( A | B) = 1 − 3 = 5
black card. 8 8
4 26
\ P(E) = and P(F) =
52 52 (iv) P(B′ | A) = 1 − P ( B | A ) = 1 − 4 = 5
9 9
Number of favourable outcomes for (E ∩ F) will be 2 black
aces. P ( A ∩ B′ ) P ( A ) − P ( A ∩ B)
2 (v) P(A | B′) = =
\ P(E ∩ F) = P ( B′ ) 1 − P ( B)
52
2 9 −1
P(E ∩ F) 52 1 20 5 = 15
Now P(E | F) = = = =
P(F) 26 13 8 28
52 1−
15
Example 14. A pair of dice is thrown. Find the probability of
P ( A ′ ∩ B) P ( B) − P ( A ∩ B)
getting a doublet if it is known that sum of the numbers on the (vi) P(B | A′) = =
two dice is 10. P ( A′ ) 1 − P (A)
Solution. A pair of dice has been thrown.
8 −1 8−3
∴ Total number of outcomes = n(S) = 36 15 5 = 15 = 5 × 20 = 20
E = Doublets = {(1, 1), (2, 2), (3, 3), (4, 4), (5, 5), = 9 11
1− 15 11 33
 (6, 6)} 20 20
F = {Total of two numbers on two dice is 10} P ( A′ ∩ B′ ) P ( A ∪ B) ′
= {(4, 6), (5, 5), (6, 4)} (vii) P(A′ | B′) = =
P ( B′ ) 1 − P ( B)
n ( F) 3
⇒ n(F) = 3 ⇒ P ( F) = =
n (S) 36 47 13
1 − P ( A ∪ B) 1 − 60 60
E ∩ F = Number of common outcomes of two events E = = =
1 − P ( B) 1− 8 7
and F = (5, 5)
15 15
⇒ n(E ∩ F) = 1 ⇒ P ( E ∩ F) = 1 13 15 13
36 = 60 × 7 = 28
The conditional probability of E when F has occurred =
1
P ( E ∩ F) 36 1
P ( E | F) = = = P ( A′ ∩ B′ ) P ( A ∪ B)′
P ( F) 3 3 (viii) P(B′ | A′) = =
36 P ( A′ ) 1 − P (A)
1
⇒ P(E | F) =
3 47 13
Example 15. If E and F are two events of the same sample space of an 1 − P ( A ∪ B) 1 − 60 60
= = =
47 Find 1 − P (A) 1− 9 11
experiment and P ( A ) = 9 , P ( B) = 8 and P(A ∪ B) = . 20 20
20 15 60
(i) P(A|B), (ii) P(B|A) 13 20 13
= × =
(iii) P(A′|B), (iv) P(B′|A), 60 11 33

Probability 383
E:\AMIT_WORKS\Exam_Guru\EG_Mathematics-12_(working_02-06-2022)\EG_Mathematics-12_working\Open_Files\Chap_13\Chap_13
\ 18-Aug-2022 Amit Proof-5 Reader’s Sign _______________________ Date __________

EXERCISE 13.2
I. Multiple Choice Questions (MCQs) 7. A pair of dice is thrown find the probabilities of getting a
1. It is given that the event A and B are such that P(A) = sum of 10 or more, if 5 appears on the first die.
1 1 2 [Delhi 2006 (C)]
, P(A | B) = and P(B|A) = , then P(B) is
4 2 3 8. A die is thrown thrice if event E is that first throw shows
1 2 1 and F is the event that last throw shows 6. Find P(E|F).
1 1
(a) (b) (c) (d) 9. A die is thrown twice and the sum of the numbers appearing
2 6 3 3
  is observed to be 8. What is the conditional probability that
( )
2. If P A ∩ B = 0.15, P(B) = 0.10 then P(A | B) is: the number 5 has appeared atleast once? [Delhi 2006]
1 1 1 1 10. A die is throw 3 times. Find P(E|F) when E = {4 appears
(a) (b) (c) (d)
3 4 2 6 on third throw} F = {6 and 5 appears respectively on first
3. If A and B are events such that P(A|B) = P(B|A), then: 2 throws}.
(a) A ⊂ B but A ≠ B (b) A = B 11. A black and red die are rolled:
(c) A ∩ B = f (d) P(A) = P(B) (a) Find the conditional probability of obtaining a sum
3 3 2 greater than 9 given that the black die results in 5.
4. If P(A ∪ B) = , P(A) = , P(B) = , then P(B|A) + (b) Find the conditional probability of obtaining the
5 10 5
P(A|B) = sum 8, given that the red die resulted in a number less
1 1 5 7 than 4.
(a) (b) (c) (d)
4 3 12 12 12. Given that two numbers appearing on throwing two dice
II. Very Short Answer Type Question are different. Find the probability of the event that the sum
of the numbers on the two dice is 4.
1. A coin is tossed three times: Find P(E|F) if:
13. Consider the experiment of throwing a die, if a multiple
(i) E = head on third toss; F = heads on first two tosses
of 3 comes up throw the die again. If any other number
(ii) E = atleast two heads; F = at most two heads.
comes, toss a coin. Find the conditional probability of the
(iii) E = at most two tails; F = atleast one tail. event that the coin shows a tail, given that atleast one die
III. Short Answer Type Questions shows 3.
1. A coin is tossed twice and four possible outcomes are 14. A die is tossed twice and the sum of the numbers appearing
equally likely. If E is the event, both head and tail have in two tosses is observed to be 7. Find the conditional
appeared and F be the event, at most one tail has appeared. probability that the number 2 appeared atleast once.
Find P(E), P(F), P(E ∩ F), P(E|F) and P(F|E). 15. One card is drawn from a well shuffled pack of 52 cards.
2. Consider the experiment of tossing a coin, if the coin shows If E is the event “the card drawn is a king or queen”
a head, toss it again, but if it shows a tail, then throw a die. and F is the event the card drawn is a queen or an ace”.
Find the conditional probability of the event that the die Find P(E|F). [Delhi 2005 (C)]
shows a number greater than 4, given that ‘there is atleast 16. In a certain school 20% of the students failed in English,
one tail’. 15% of the students failed in Mathematics and 10% of the
3. In a family there are 3 children. What is the probability students failed in both English and Mathematics. A student
that first child is a boy when it is known that last child is is selected at random. If he has failed in English what is
also a boy? the probability that he has also failed in Mathematics?
4. A family has two children what is the probability that both  [Delhi 2007]
the children are boys given that (i) at least one of them is 17. Ten cards numbered from 1 to 10 are placed in a box; mixed
a boy, (ii) elder child is a boy? up thoroughly and then one card is drawn randomly. It is
 [Delhi 2008, 2014 (C), A.I. 2010] known that the number on the drawn card is more than 3.
5. Assume that each born child is equally likely to be a boy or What is the probability that it is an even number?
a girl. If a family has two children, what is the conditional  [A.I. 2008]
probability that both are girls? Given that (i) youngest child 18. In a school there are 1000 students, out of which 430 are
is a girl (ii) atleast one child is a girl. girls. It is known that out of 430 girls, 10% of the girls
6. Mother, father and son line up at random for a family study in Class XII. What is the probability that a student
picture. Find P(E|F) when E = son is on one end and chosen randomly studies in Class XII, given that the chosen
F = Father is in the middle. student is a girl? [A.I. 2009 (C)]

384 Mathematics–12
E:\AMIT_WORKS\Exam_Guru\EG_Mathematics-12_(working_02-06-2022)\EG_Mathematics-12_working\Open_Files\Chap_13\Chap_13
\ 18-Aug-2022 Amit Proof-5 Reader’s Sign _______________________ Date __________

19. An instructor has a question bank consisting of 300 easy 40% read English newspaper and 20% read both Hindi
True/False questions, 200 difficult True/False questions, and English newspapers. A student is selected at random:
500 easy multiple choice questions and 400 difficult (i) Find the probability that she reads neither Hindi nor
multiple choice questions. If a question is selected at English newspaper.
random from the question bank, what is the probability (ii) If she read Hindi newspaper, find the probability that
that it will be an easy question given that it is Multiple she reads English newspaper also.
Choice Question? (iii) If she read English newspaper, find the probability that
20. In a hostel 60%, of the students read Hindi newspaper, she reads Hindi newspaper also.

Answers 13.2
1 P(A ∩ B) 1 ⇒ n(E) =4
I. 1. P(A|B) = ⇒ = ...(1) F= {H on first two tosses}
2 P(B) 2
2 P(A ∩ B) 2 ={HHH, HHT}
and P(B|A) = ⇒ =  ...(2) ⇒ n(F) =2, (E ∩ F) = {HHH}
3 P(A) 3
Dividing (1) by (2), wet get ⇒ n(E ∩ F) =1.
P(A) 3 4 4 1 1 3
= ⇒ P(B) = P(A) = × = (ii) P(E | F) =
7
P(B) 4 3 3 4 3 Hint: E = {atleast 2Hs)
Hence, (c) is the correct answer. = {HTH, HHT, THH, HHH}
()
2. P B = 1 – P(B) = 1 – 0.10 = 0.90 F = {at most 2 Hs)
(
P A∩B ) = 0.15 = 15 = 1 = {TTT, HTT, THT, TTH, THH HTH, HHT}
P A|B =( ) P B ()
0.90 90 6 ⇒ n(F) = 7
Hence, (d) is the correct answer. (E ∩ F) = {HHT, HTH, THH}
3. We have P(A|B) = P(B|A) ⇒ n(E ∩ F) = 3.
P ( A ∩ B) P(A ∩ B) 6
(iii) P(E | F) =

⇒ = 7
P(B) P(A) Hint: E = at most 2 tails
 (By definition of conditional probability) = no tail or 1 tail or 2 tails
1 1 = {HHH, THH, HTH, HHT, TTH, THT, HTT}
⇒ = ⇒ P(A) = P(B)
P(B) P(A) ⇒ n(E) = 7
Hence, (d) is the correct answer. F = at least 1 tail
4. P(A ∪ B) = P(A) + P(B) – P(A ∩ B) = one tail, 2 tails or 3 tails
3 3 2 = {THH, HTH, HHT, TTH, THT, HTT, TTT}
⇒ = + − P(A ∩ B)
5 10 5 ⇒ n(F) = 7
3 2 3 3+ 4− 6 1 E ∩ F = {THH, HTH, HHT, TTH, THT, HTT}
⇒ P(A ∩ B) = + − = =
10 5 5 10 10 ( )
⇒ n(E ∩ F) = 6 ⇒ P(E | F) = n E ∩ F = 6
1 n ( F) 7
P(A ∩ B) 10 1 1 2
P(B|A) =
P(A)
= =
3 3 III. 1. P ( E ) = 1 ; P ( F) = 3 ; P(E ∩ F) = ; P(E | F) = ;
2 4 2 3
10 P(F | E) = 1
1 S = {TT, HH, TH, HT}
P(A ∩ B) 10 5 1 1
P(A|B) = = = × = ⇒ n(S) = 4
P(B) 2 2 10 4
(i) E = both head and tail appear
5
1 1 4+3 7 = {TH, HT}
\ P(B|A) + P(A|B) = + = = ⇒ n(E) = 2
3 4 12 12
( )
Hence, (d) is the correct answer. P(E) = n E = 2 = 1
1 n (S) 4 2
II. 1. (i) P(E|F) =
2 (ii) P ( F) = 3 ;
Hint: n(S) = 8 4
E = {H on the third toss} F = at most one tail = no tail or 1 tail
= {HHH, THH, HTH, TTH} = {HH, TH, HT}

Probability 385
E:\AMIT_WORKS\Exam_Guru\EG_Mathematics-12_(working_02-06-2022)\EG_Mathematics-12_working\Open_Files\Chap_13\Chap_13
\ 18-Aug-2022 Amit Proof-5 Reader’s Sign _______________________ Date __________

⇒ n(F) = 3 (ii) F= elder child is a boy


( ) ={(b, g), (b, b)}
P(F) = n F = 3
n (S) 4 ⇒ n(F) =2
1 n(S) =4
(iii) P(E ∩ F) = ;
2 n ( F) = 2 = 1
E ∩ F = Common outcomes of E and F P(F) =
n (S) 4 2
= {TH, HT} E ∩ F = {(b, b)} ⇒ n(E ∩ F) = 1
⇒ n(E ∩ F) = 2
⇒ P(E ∩ F) = n ( E ∩ F) = 1
n ( E ∩ F) = 2 = 1 n (S) 4
\ P(E ∩ F) =
n (S) 4 2
1
2 P ( E ∩ F) = 4 = 1
(iv) P(E | F) = ; ⇒ P(E | F) =
3 P ( F) 1 2
1 2
P ( E ∩ F) 2
P(E | F) = = = 1×4=2
P ( F) 3 2 3 3 1
5. (i) P(E | F) = (ii) P(E |F) = 1
4 2 3
(v) P(F | E) = 1 Hint: S = {(b, b), (g, g), (b, g), (g, b)}
1 ⇒ n(S) = 4
P (F ∩ E ) 2 (i) E = both girls = {(g, g)}
P(F | E) = = =1
2 P (E ) 1
⇒ n(E) = 1
2. P(E | F) = ; 2
9 F = youngest child is a girl
1 = {(g, g), (b, g)}
3. P(E | F) =
2
Hint: Let b stands for boy and g stands for girl. ⇒ n(F) = 2
Sample space: {(b, b, b), (b, b, g), (b, g, b), (g, b, b), E ∩ F = {(g, g)} ⇒ n(E ∩ F) = 1
(g, g, b), (g, b, g), (b, g, g), (g, g, g)} ( )
P(E ∩ F) = n E ∩ F = 1
⇒ n(S) = 8. n (S) 4
Let E denotes the event that first child is a boy and F denotes ( )
the event that last child is a boy. P(F) = n F = 2 = 1
n (S) 4 2
\ E = {(b, b, b), (b, b, g), (b, g, b), (b, g, g)} 1
F = {(b, b, b), (b, g, b), (g, b, b), (g, g, b)} P ( E ∩ F) 4 1
P(E | F) = = =
⇒ n(F) = 4 P ( F) 1 2
E ∩ F = {(b, b, b), (b, g, b)} 2
⇒ n(E ∩ F) = 2. (ii) n(S) = 4, E = {(g, g)}
F = at least one child is a girl
⇒ P(F) = 4 , P(E ∩ F) = 2
8 8 = {(b, g), (g, b), (g, g)}
Now the required probability: ( )
⇒ n(F) = 3 ⇒ P(F) = n F = 3
2 n (S) 4
P(E ∩ F) 8 1
= P(E|F) = = = ⇒ P(E|F) = 1 E ∩ F = {(g, g)} ⇒ n(E ∩ F) = 1
P(F) 4 2 2
( )
8 P(E ∩ F) = n E ∩ F = 1
1 1 n (S) 4
4. P(E|F) = , P(F|E) =
3 2 1
( )
Hint: S = {(b, b), (g, g), (b, g), (g, b)} P(E | F) = P E ∩ F = 4 = 1
n(S) = 4 P ( F) 3 3
4
(i) E = both children are boys = {(b, b)}
6. P(E | F) = 1
⇒ n(E) = 1
F = at least one of them is a boy Hint: S = {MFS, MSF, FMS, FSM, SMF, SFM}
= {(b, g), (g, b), (b, b)} ⇒ n(S) = 6
⇒ n(F) = 3 E = Son is on one end
E ∩ F = {(b, b)} = {MFS, FMS, SMF, SFM}
1
⇒ P(E ∩ F) = ⇒ P(F) = 3 F = Father in the middle = {MFS, SFM}
4 4 ⇒ n(F) = 2
1
P(E ∩ F) 4 1 n ( F) 2 1
⇒ P(E|F) = = =
⇒ P(F) = = =
P(F) 3 3 n (S) 6 3
4 E ∩ F = {MFS, SFM} ⇒ n(E ∩ F) = 2

386 Mathematics–12
E:\AMIT_WORKS\Exam_Guru\EG_Mathematics-12_(working_02-06-2022)\EG_Mathematics-12_working\Open_Files\Chap_13\Chap_13
\ 18-Aug-2022 Amit Proof-5 Reader’s Sign _______________________ Date __________

n ( E ∩ F) = 2 = 1
(
P (E ∩ F) = n E ∩ F = 1
)

⇒ P(E ∩ F) =
n (S) 6 3 n (S) 36
1 1
P ( E ∩ F) = 3 = 1
( )
P(E | F) = P E ∩ F = 36 = 1
P(E | F) =
P ( F) 1 P ( F) 6 6
3 36
1
7. P(E | F) = 1
3 11. (a) P(A | B) = ;
3
Hint: Throwing of a pair of dice has 36 outcomes Hint: Two dice are tossed
\ n(S) = 36 \ n(S) = 36
E = Sum 10 or more A = sum greater than 9
= {(4, 6), (5, 5), (6, 4), (6, 5), (5, 6), (6, 6)}
= {(6, 4), (5, 5), (4, 6), (5, 6), (6, 5), (6, 6)}
F = 5 appear on the first die
B = black die result in 5
= {(5, 1), (5, 2), (5, 3), (5, 4), (5, 5), (5, 6)}
= {(5, 1), (5, 2), (5, 3), (5, 4), (5, 5), (5, 6)}
( )

⇒ n(F) = 6 ⇒ P(F) = n F = 6 = 1 n(B) = 6, n(S) = 36
n (S) 36 6 ( )
E ∩ F = {(5, 5), (5, 6)} \ P(B) = n B = 6 = 1
n (S) 36 6
⇒ n(E ∩ F) = 2
A ∩ B = {(5, 5), (5, 6)} ⇒ n(A ∩ B) = 2
n ( E ∩ F) 2 1
\ P(E ∩ F) = = = n ( A ∩ B) 2 1
n (S) 36 18 ⇒ P(A ∩ B) = = =
n (S) 36 18
1
P ( E ∩ F) 18 1 1
P(E | F) = = = ( )
P ( F) 1 3 P(A | B) = P A ∩ B = 18 = 1
6 P ( B) 1 3
1 6
8. P(E | F) = 1
6 (b) P ( A | B) = ;
Hint: n(S) = 216, 9
n(E) = n{first throw is 1} = 1 × 6 × 6 = 36 Hint: n(S) = 36
n(F) = n{last throw show 6} A = sum is 8 = {(6, 2), (5, 3), (4, 4), (3, 5), (2, 6)}
= 6 × 6 × 1 = 36 B = Red die show a number less than 4.
n(E ∩ F) = 1 × 6 × 1 = 6. = { (1, 1), (1, 2), (1, 3), (1, 4), (1, 5), (1, 6),
(2, 1), (2, 2), (2, 3), (2, 4), (2, 5), (2, 6),
2 (3, 1), (3, 2), (3, 3), (3, 4), (3, 5), (3, 6)}
9. P(E | F) =
11
( )
Hint: n(S) = 36 \ n(B) = 18 ⇒ P(B) = n B = 18 = 1
n (S) 36 2
E= Sum is 8 A ∩ B = {(3, 5), (2, 6)}
= {(2, 6), (3, 5), (4, 4), (5, 3), (6, 2)} ⇒ n(A ∩ B) = 2
F= number 5 appears at least once n ( A ∩ B) = 2 = 1
⇒ P(A ∩ B) =
= {(5, 1), (5, 2), (5, 3), (5, 4), (5, 5), n (S) 36 18
(5, 6), (1, 5), (2, 5), (3, 5), (4, 5), (6, 5)} 1
( )

⇒ n(F) = 11 ⇒ P(F) =
n ( F) 11
= P(A | B) = P A ∩ B = 18 = 1
n (S) 36 P ( B) 1 9
E∩F= {(3, 5), (5, 3)} 2
1
n ( E ∩ F) 2 12. P(A | B) = ,
⇒ n(E ∩ F) =
2 ⇒ P(E ∩ F) = = 15
n (S) 36 Hint: Two dice are thrown. n(S) = 36
2 B = dice have different number

\ P(E | F) = P ( E ∩ F) 36 2
= = 6! = 30
P ( F) 11 11 n(B) = 6P2 = ⇒ P(B) = 30 = 5
36 4! 36 6
1 A = sum of the numbers is 4 = {(1, 3), (2, 2), (3, 1)}
10. P(E | F) = ,
6 A ∩ B = {(1, 3), (3, 1)}
Hint: n(S) = 36 ⇒ n(A ∩ B) = 2
n(E) = n{4 appears on third throw} n ( A ∩ B) 2 1
\ P(A ∩ B) = = =
= 6 × 6 × 1 = 36 n (S) 36 18
n(F) = n{6 and 5 appears respectively on first 2 tosses} 1
=1×1×6=6 P ( A ∩ B) = 18 = 1 × 6 = 1
P(A | B) =
( ) P ( B) 5 18 5 15
n(E ∩ F) = n{6, 5, 4} = 1 ⇒ P(F) = n F = 6 6
n (S) 36

Probability 387
E:\AMIT_WORKS\Exam_Guru\EG_Mathematics-12_(working_02-06-2022)\EG_Mathematics-12_working\Open_Files\Chap_13\Chap_13
\ 18-Aug-2022 Amit Proof-5 Reader’s Sign _______________________ Date __________

13. P(A | B) = 0, 1
16. P(M | E) = = 50%
Hint: S = {(3, 1), (3, 2), (3, 3), (3, 4), (3, 5), (3, 6), (6, 1),
2
(6, 2), (6, 3), (6, 4), (6, 5), (6, 6), (1, H), (1, T), 20 , P(M) = 15 ,
Hint: P(E) =
(2, H), (2, T), (4, H), (4, T), (5, H), (5, T)} 100 100

⇒ n(S) = 20 10
P(E ∩ M) =
100
A = Coin shows a tail = {(1, T), (2, T), (4, T), (5, T)}
10
B = At least one die shows 3 P ( M ∩ E ) 100 1 50%
\ P(M | E) = = = =
= {(3, 1), (3, 2), (3, 3), (3, 4), (3, 5), (3, 6), (6, 3)} P (E ) 20 2
100

⇒ n(B) = 7 4
17. P(A|B) = ;
7
n ( B) = 7 , A ∩ B = φ
P(B) = Hint: S = {1, 2, 3, ...., 10}
n (S) 20

⇒ P(A ∩ B) = 0
⇒ n(S) = 10

P ( A ∩ B) 0 A = An even number = {2, 4, 6, 8, 10}



⇒ P(A | B) = = =0
P ( B) 7 B = Number is more than 3 = {4, 5, 6, 7, 8, 9, 10}
20
1
⇒ n(B) = 7
14. P(A | B) =,
3 n ( B) 7

⇒ P(B) = =
Hint: A die is tossed twice.
n (S) 10

\ n(S) = 36 (A ∩ B) = {4, 6, 8, 10}
A = Number of 2 appears at least once ⇒ n(A ∩ B) = 4

= {(2, 1), (2, 2), (2, 3), (2, 4), (2, 5), (2, 6), (1, 2), (3, 2), (4, n ( A ∩ B) = 4
P(A ∩ B) =
2), (5, 2), (6, 2)} n (S) 10
B = Sum of two numbers is 7 4
P ( A ∩ B) 10 4
= {(1, 6), (6, 1), (2, 5), (5, 2), (4, 3), (3, 4)} \ P(A | B) = = =
P ( B) 7 7

⇒ n(B) = 6 10
1
n ( B) 6 18. P(E|F) = ;
P(B) = = 10
n (S) 36
Hint: n(S) = 1000
A ∩ B = {(2, 5), (5, 2)} ⇒ n(A ∩ B) = 2
43
n ( A ∩ B) = 2 P(F) =

⇒ P(A ∩ B) = 100
n (S) 36
P(E ∩ F) = 43
2
P ( A ∩ B) 36 1 1000
⇒ P(A | B) = = =
P ( B) 6 3 43
36 P ( E ∩ F) = 1000 = 1
⇒ P(E | F) =
15. P(E | F) = 1 ; P ( F) 43 10
2 100
Hint:Card is drawn from 52 cards 19. P(E|M) = 5 ;
9

\ n(S) = 52
Hint:

E = {King or Queen}
E = easy D = Difficult Total

⇒ n(E) = 8
T = True/False 300 200 500
F = {Queen or Ace}
M = Multiple choice 500 400 900

⇒ n(F) = 8
Total 800 600 1400

⇒ P(F) = 8
52 500 = 5
P(E ∩ M) =
E ∩ F = {Queen} ⇒ n(E ∩ F) = 4 1400 14
n ( E ∩ F) = 4
P(E ∩ F) = P(M) = 900 = 9
n (S) 52 1400 14
4 5
P ( E ∩ F) 52 1 P ( E ∩ M ) = 14 = 5
P(E | F) = = =
⇒ P(E | M) =
P ( F) 8 2 P (M ) 9 9
52 14

388 Mathematics–12
E:\AMIT_WORKS\Exam_Guru\EG_Mathematics-12_(working_02-06-2022)\EG_Mathematics-12_working\Open_Files\Chap_13\Chap_13
\ 18-Aug-2022 Amit Proof-5 Reader’s Sign _______________________ Date __________

60 1
20. Given P(H) = ; (ii) 33 %,
100 3
P(E) = 40 ; P(H ∩ E) = 20 20
100 100 P(E ∩ H) 100 1
Hint: P(E | H) = = = = 33 1 %
(i) P(H′ ∩ E′) = 20%, P(H) 60 3 3
100
Hint: P(H′ ∩ E′) = 20% = P(H ∪ E)′

(iii) 50%,
= 1 – P(H ∪ E)
20
= 1 – [P(H) + P(E) – P(H ∩ E)] P(H ∩ E) 100 1
Hint: P(H | E) =
= = = 50%
P(E) 40 2
60 + 40 − 20  1 80 20 20
= 1 −  = − = = % 100
100 100 100  100 100

Topic 3. Multiplication Theorem on Probability


Multiplication Rules for Two Events ∴ Probability of getting both aces:
Let E and F be two events in the same sample space S. Then E ∩ F P(E ∩ F) = P(EF) = P(E) ⋅ P(F | E)
denotes the simultaneous occurrence of the event E and F. Event 4 3 1
E ∩ F is also written as EF. We want to calculate the probability = × =
52 51 221
of event (E ∩ F). We know.
Second Method: There are 4 aces and total number of cards
P ( E ∩ F)
P(E | F) = 52, number of favourable outcomes
P ( F)
⇒ P(E ∩ F) = P(F) × P(E | F)...(1) = n(E) = 4C2.
P ( E ∩ F) Total number of outcomes = n(S) = 52C2
Also P(F | E) =
P (E) 4×3
n ( E ) 4C2 2 1
⇒ P(E ∩ F) = P(E) . P(F | E) ...(2) \ P(E) = = = =
n (S) 52C2 52 × 51 221
From (1) and (2), we get: 2
P(E ∩ F) = P(E) ⋅ P(F | E) = P(F) ⋅ P(E | F) Example 2. Three cards are drawn successively without
This result is called the multiplication rule of probabilities. replacement from a pack of 52 well shuffled cards. What is the
probability that the first card is a king, second card is a queen
Multiplication Rules for Three Events
and third card is a jack?
If E, F and G are three events of same sample space, then:
Solution. Let E be the event that card drawn is a king. F be the
P(E ∩ F ∩ G) = P(E) ⋅ P(F | E) ⋅ P[G|(E ∩ F)]
event that card drawn is a queen and G be the event that card
Example 1. Two cards are drawn at random from a well shuffled
pack of 52 cards without replacement. What is the probability drawn is a jack.
that both cards are ace? Here we want to work out P(EFG) or P(E ∩ F ∩ G)
Solution. First Method: Let E be the event to draw first card an
Now, P(E) = 4 (4 kings out of 52 cards) and P(F/E)
ace and F be the event to draw second card an ace. 52
In a pack of 52 cards there are 4 ace.
= 4 (4 queens out of 51 cards) also
4 51
∴ P(E) = ,
52 4 (4 jacks out of 50 cards)
Now 1 ace has been selected so left out aces are 3 and total P(G|E ∩ F) =
50
cards are 51.
∴ P(EFG) = P(E) ⋅ P(F|E) P(G|E ∩ F)
∴Probability of Event F when E has already occurred =
8
P(F | E) =
3 = 4 × 4 × 4 =
51 52 51 50 16575

EXERCISE 13.3
I. Multiple Choice Questions (MCQs) II. Short Answer Type Questions-II
1. If A and B are two events such that P(A ∪ B) = 1. A box of oranges is inspected by examining three randomly
5 1 1
, P(A) = , P(B) = , the A and B are selected oranges drawn without replacement. If all the three
6 4 3
(a) mutually exclusive (b) dependent oranges are good, the box is approved for sale, otherwise,
(c) Independent (d) None of these it is rejected. Find the probability that a box containing

Probability 389
E:\AMIT_WORKS\Exam_Guru\EG_Mathematics-12_(working_02-06-2022)\EG_Mathematics-12_working\Open_Files\Chap_13\Chap_13
\ 18-Aug-2022 Amit Proof-5 Reader’s Sign _______________________ Date __________

15 oranges out of which 12 are good and 3 are bad ones, in the urn and then a ball is drawn at random. What is the
will be approved for sale. [Delhi 2006] probability that the second ball drawn is red?
2. Three cards are drawn successively, without replacement  [Foreign 2008 (C)]
from a pack of 52 well shuffled cards. What is the III. Long Answer Type Questions
probability that first two cards are kings and the third card 1. Bag I contains 3 red and 4 black balls and bag II contains 4
is an ace? red and 5 black balls. Two balls are transferred at random
3. An urn contains 5 red and 5 black balls. A ball is drawn from bag I to bag II and then a ball is drawn from bag
at random, its colour is noted and returned to the urn. II. The ball so drawn is red. Find the probability that the
Moreover, two additional balls of the same colour are put transferred ball were both black. [Delhi 2012]

Answers 13.3
I. 1. If A and B are any two events P(A ∪ B) = P(A) + P(B) – 1
P(A ∩ B) 3.
2
5 3 1 Hint: P(To draw 1R ball in second draw)


⇒ = + − P(A ∩ B)
6 4 3 = P(draw 1R from 5R and 5B) × P(draw 1R from 7R and
 1 3 5B ) + P (draw 1B from 5 red and 5B) × (draw 1R from 5 red
  P(A) = 1 − P(A) = 1 − 4 = 4  and 7B)
 


⇒ P(A ∩ B) =
3 1 5 9 + 4 − 10 3 1
+ − =
4 3 6 12
= =
12 4

10 12 10( 12 120 2 )( )( )
= 5 × 7 + 5 × 5 = 60 = 1

3 1 1
Also P(A) . P(B) = × =
4 3 4 III. 1. 4
17
 1

\ P(A ∩ B) = P(A) . P(B)  Each = 4  Hint: A = {2 red balls are transferred}

 
⇒ A and B are independent events. B = {2 black balls are transferred}
Again, as P(A ∩ B) ≠ 0, so A and B are independent of each C = {1R and 1B balls are transferred}
other. D = {1R ball is drawn from the II bag}
Hence, (c) is the correct answer.
P(D) = P(A) P(D/A) + P(B) P(D/B) + P(C) × P(D/C)
44

( ) ( ) ( )
II. 1. ,  3C  6  4C   3C × 4C1 
91 4 5

=  7 2 +  7 2 × +  17 × = 102
Hint: P (all the three oranges are good)  C2  11  C2  11  C2  11 21 × 11
12
C 44
()
= 15 3 =    C 
4 
C3 91 P  B ∩ D  =  7 2  × 4  = 24
 B   C2  11 21 × 11
2
2. ,
()
5525
  P  B ∩ D   24 
   B  =  21 × 11  = 24 = 4
Hint: P (First 2 cards are kings and third card is an ace)
P B = 


 4C 
=  52 2  ×
4
= ( )
2 

D 
B 
P(D)
()  102  102 17
 21 × 11 
 C2  50 5525

Topic 4. Independent Events


Definition of Independent Events Similarly: P(F | E) = P(F)
If the probability of occurrence of an event E is not-effected by P ( E ∩ F)
the occurrence of the event F, or if the probability of occurrence ⇒ = P ( F)
P (E)
of event F is not effected by occurrence of event E, then events
E and F are called independent events. ⇒ P(E ∩ F) = P(E) × P(F)
This means if P(E|F) = P(E) and P(F|E) = P(F) then the events ∴ Two events E and F are independent when
E and F are independent.
P ( E ∩ F) P(E ∩ F) = P(E) ⋅ P(F).
Now: P(E|F) = P(E) ⇒ = P (E)
P ( F) Here E ∩ F means the simultaneous occurrence of events E
P(E ∩ F) = P(E) × P(F) and F.

390 Mathematics–12
E:\AMIT_WORKS\Exam_Guru\EG_Mathematics-12_(working_02-06-2022)\EG_Mathematics-12_working\Open_Files\Chap_13\Chap_13
\ 18-Aug-2022 Amit Proof-5 Reader’s Sign _______________________ Date __________

Dependent Events Example 3. If A and B are independent events, then A and B


Two events E and F are said to be dependent events if: are also independent.
P(E ∩ F) ≠ P(E) ⋅ P(F) Sol. True
Example 4. If A and B are mutually exclusive events, then they
Difference Between Independent Events and will be independent also.
Mutually Exclusive Events Solution. False
(i) Term independent is defined in terms of probability of Example 5. Two independent events are always mutually
events, whereas mutually exclusive is defined in terms of exclusive.
events. Solution. False
(ii) Independent events may have common outcomes whereas Example 6. If A and B are two independent events, then
mutually exclusive events never have common outcomes. P(A and B) = P(A).P(B).
(iii) Two independent events having non-zero probabilities of Solution. True
occurrence cannot be mutually exclusive and conversely, Example 7. If A and B are independent events, then
i.e., two mutually exclusive events having non-zero
P(A  B) = 1 – P(A).P(B)
probabilities of occurrence cannot be independent.
Solution. True
Independent Experiments [ P(A  B) = 1 – P(A  B) = 1 – P(A).P(B)]
Two Experiment are said to be independent for every pair of Example 8. If A and B are independent events, then
events E and F, where E is associated with the first experiment P (exactly one of A, B occurs) = P(A).P(B) + P(B).P(A)
and event F is associated with second experiment, the probability Solution. True
of the simultaneous occurrence of the events E and F when the Example 9. If A, B and C are three independent events such that
two experiments are performed, is the product of P(E) and P(F), P(A) = P(B) = P(C) = p,
calculated separately on the bases of two experiments. i.e., then P (atleast two of A, B and C occur) = 3p2 – 2p3
P(E ∩ F) = P(E) ⋅ P(F). Solution. True
Independence of Events A, B and C Since P (atleast two of A, B and C occur)
Three events A, B and C are said to independent if: = p × p × (1 – p) + (1 – p) . p . p + p(1 – p).p + p . p . p
(i) P(A ∩ B) = P(A) ⋅ P(B) = 3p2 (1 – p) + p3 = 3p2 – 3p3 + p3 = 3p2 – 2p3
Example 10. If A and B are two independent events with
(ii) P(A ∩ C) = P(A) ⋅ P(C)
3 4
(iii) P(B ∩ C) = P(B) ⋅ P(C) P(A) = and P(B) = , then P(A  B) equals
5 9
(iv) P(A ∩ B ∩ C) = P(A) P(B) P(C)
4 8 1 2
If atleast one of the above condition is not true for three given (a) . (b) (c) (d)
15 45 3 9
events, we say that three events are not independent.
Solution. Given that: A and B are independent events such that
Properties of Independent Events 3 3 2
P(A) = \ P(A) = 1 − =
5 5 5
Property 1: If E and F are two independent events then E and F′
are also independent events. 4 4 5
P(B) = \ P(B) = 1 −
=
9 9 9
Property 2: If E and F are two independent events then E′ and
F′ are also independent events. 2 5 2

\ P(A  B) = P(A) . P(B) = . =
5 9 9
Property 3: If E and F are two independent events, then the
probability of occurrence of atleast one of E or F is given by Hence, the correct option is (d).
[1 – P(E′) × P(F′)]. Example 11. If two events are independent, then
Example 1. Let A and B be two events. If P(A/B) = P(A), then (a) they must be mutually exclusive
A is ............... of B. (b) the sum of their probabilities must be equal to 1
P(A ∩ B) P(A ∩ B) (c) (a) and (b) both are correct
Solution.  P ( A B) =  P(A) =
P(B) P(B)
(d) none of the above is correct
 P(A  B) = P(A).P(B)
Solution. For independent events A and B, P(A).P(B) = P(A  B)
So, A is independent of B. So, they will not be mutually exclusive.
Example 2. Let P(A) > 0 and P(B) > 0. Then A and B can be both If P(A) + P(B) = 1, they are exhaustive events and for
mutually exclusive and independent. independent events A and P(A  B)  0.
Solution. False Hence, the correct option is (d).

Probability 391
E:\AMIT_WORKS\Exam_Guru\EG_Mathematics-12_(working_02-06-2022)\EG_Mathematics-12_working\Open_Files\Chap_13\Chap_13
\ 18-Aug-2022 Amit Proof-5 Reader’s Sign _______________________ Date __________

Example 12. If the events A and B are independent, then Example 16. A flashlight has 8 batteries out of which 3 are dead.
P(A  B) is equal to If two batteries are selected without replacement and tested then
(a) P(A) + P(B) (b) P(A) – P(B) the probability that both are dead is
33 9 1 3
P(A) (a) (b) (c)
(d) 28
(c) P(A).P(B) (d) 56 64 14
P(B)
Solution. Required probability = P(dead).P(dead)
Solution. Since A and B are two independent events
3 2 3
\ P(A  B) = P(A).P(B) = . =
8 7 28
Hence, the correct option is (c). Hence, the correct option is (d).
Example 13. A bag contains 5 red and 3 blue balls. If 3 balls are Example 17. If two cards are drawn from a well shuffled deck
drawn at random without replacement, then the probability of of 52 playing cards with replacement, then the probability that
getting exactly one red ball is both cards are queens is
1 1 1 1 1 1 1 4
135 15 (a) × (b) + (c) × (d) ×
(a) 45 (b) (c) 15 (d) 13 13 13 13 13 17 13 51
196 392 56 29
4
Solution. Given that: Bag contains 5 red and 3 blue balls. Solution. Probability of getting Queen =
52
Probability of getting exactly one red ball if 3 balls are randomly
So, the required probability
drawn without replacement = P(Queen).P(Queen)
= P(R) . P(B) . P(B) + P(B) . P(R) . P(B) + P(B) . P(B) . P(R) 4 4 1 1
= . = . (with replacement)
52 52 13 13
5 3 2 3 5 2 3 2 5 Hence, the correct option is (a).
= . . + . . + . .
8 7 6 8 7 6 8 7 6
Example 18. Directions: In the following questions, a statement
30 30 30 90 15
= + + = = of Assertion (A) is followed by a statement of Reason (R).
336 336 336 336 56
Mark the correct choice as:
Hence, the correct option is (c). (a) Both A and R are true and R is the correct explanation of A.
Example 14. If a die is thrown and a card is selected at random (b) Both A and R are true but R is NOT the correct explanation
from a deck of 52 playing cards. The probability of getting an of A.
even number on the die and a spade card is (c) A is true but R is false.
1
(a) 1 (b) 1 (c) (d) 3 (d) A is false but R is True.
2 4 8 4
Let A and B be two independent events.
Solution. Let E1 be the event of getting even number on the die.
Example 19. Assertion (A): If P (A) = 0.3 and
E2 be the event of selecting a spade card. 2
3 1 13 1 P (A » B) = 0.8, then P (B) is .
= and P(E2) = = 7
\ P(E1) =

52 4
6 2 Reason (R) : P ( E ) = 1 – P(E), where E is any event.
1 1 1 Solution. (a) Both assertion (A) and reason (R) are true and
So P(E1  E2) = P(E1).P(E2) = . =
2 4 8 reason (R) is the correct explanation of assertion (A).
Hence, the correction option is (c). P(AUB') = P(A) + P(B') – P(A)P(B')
Example 15. A box contains 3 orange balls, 3 green balls and ∴ 0.8 = 0.3 + P(B') – 0.3P(B') fi 0.5 = P(B')(0.7)
2 blue balls. Three balls are drawn at random from the box without 5 5 2
fi P(B') = ∴ P(B) = 1 – =
7 7 7
replacement. The probability of drawing 2 green balls and one
blue ball is Example 20. Assertion (A): When two coins are tossed
3 2 1 167 simultaneously, then the probability of getting no tail is 1 .
(a) (b) (c) (d) 4
28 21 28 168 Reason (R): The probability of getting a head (i.e., no tail) in
Solution. Probability of drawing 2 green and 1 blue balls one toss of a coin is 1 .
= P(G) . P(G) . P(B) + P(G) . P(B) . P(G) + P(B) . P(G) . P(G) 2
Solution. (a) Both assertion (A) and reason (R) are true and
3 2 2 3 2 2 2 3 2 12 12 12 reason (R) is the correct explanation of assertion (A). Probability
= . . + . . + . . = + +
8 7 6 8 7 6 8 7 6 336 336 336 of getting no tail when two coins tossed simultaneously i.e., both
36 3 are heads.
= =
336 28 1 1 1
Probability of both head = × =
Hence, the correct option is (a). 2 2 4

392 Mathematics–12
E:\AMIT_WORKS\Exam_Guru\EG_Mathematics-12_(working_02-06-2022)\EG_Mathematics-12_working\Open_Files\Chap_13\Chap_13
\ 18-Aug-2022 Amit Proof-5 Reader’s Sign _______________________ Date __________

EXERCISE 13.4
I. Multiple Choice Questions (MCQs) 9. A speaks truth in 60% cases and B speaks truth in 90%
1. If E and F are independent events such that 0 < P(E) < 1 cases.
and 0 < P(F) < 1, then (i) In what percentage of cases are they likely to contradict
(a) E and F C (the complement of the event F) are each other in stating the same fact? In the case of
independent. contradiction do you think that statement of B will
(b) EC and FC are independent carry more weight as B speaks truth in more number
(c) P(E|F) + P(EC|FC) = 1 of cases than A?
(d) All of the above. (ii) In what percent of cases are they likely to agree in
stating the same fact? Do you think, when they agree
3
2. If A and B are two independent events with P(A) = and means both are speaking truth?
4 5
P(B) = , then P ( A' ∩ B') equals:  [V. Imp.] [Delhi 2005, 2013]
9 10. The probability of two students A and B coming to school
4 8 1 2
(a) (b) (c) (d)
5 45 3 9 in time are 3 and 5 respectively.
7 7
II. Short Answer Type Question-I Assuming that the events “A coming in time” and “B
1. A card is drawn from a well shuffled packs of 52 cards. coming in time” are independent, find the probability of
If event E is that the card drawn is a spade and event F is the event, only one of them coming to school in time.
that the card drawn is an ace. Show that the two events are
 [A.I. 2013]
independent.
11. Ramesh appears for an interview for two posts A and B,
III. Short Answer Type Questions-II
for which selection probabilities are independent. The
1. Three coins are tossed simultaneously. Consider the event
E, “three heads or three tails”, F, “at least two heads” and probability of his selection for post A is 1 and for post B
6
G, “at most two heads”. Of the events (E, F), (E, G) and is 1 .
(F, G) which are independent? Which are dependent? 7
Find the probability that Ramesh is:
2. A fair coin and an unbiased die are tossed. Let A be the
(i) P (selected only for post A)
event ‘head appears on the coin’ and B be the event ‘3 on
the die’. Check whether A and B are independent events (ii) P (selected only for post B)
or not. (iii) P (selected for any one post)
3. A die marked 1, 2, 3 in red and 4, 5, 6 in green, is tossed. (iv) P (selected for both the posts A and B)
Let A be the event, ‘number is even’, and B is the event, (v) P (selected for atleast one post)
‘the number is red’. Are A and B independent? (vi) P (selected for at most one post)
4. A pair of dice is tossed. Event E is to get 3 on the first die (vii) P (rejected for both posts). [A.I. 2007, (C)]
and event F is to get 2 on the second die. Show that the
12. Two persons A and B appear in an interview for two
two events E and F are independent.
vacancies for the same post. The probability that A will
5. One card is drawn at random from a well shuffled deck of 1 1
52 cards. In which of the following cases are the events E be selected is and B will be selected is . What is the
5 4
and F independent? probability that:
(i) E = the card drawn is spade. (i)
Both A and B will be selected.
F = the card drawn is an ace.
(ii)
Both A and B will be rejected
(ii) E = the card drawn is black.
(iii)
Only A will be selected
F = the card drawn is a king.
. (iv)
Only B will be selected.
(iii) E = the card drawn is a king or queen.
F = The card drawn is a king or jack. (v)
Any one of them will be selected.
1 7 (vi)
Atleast one of them will be selected.
6. Events A and B are such that P(A) = , P(B) = (vii)
At most one of them will be selected. [A.I. 2007 (C)]
2 12
1
and P (not A or not B) = . State whether A and B are
4 13. The probability of A of hitting a target is 4 and that of B
independent. [CBSE 2022] 5
2
is . They both fire at the target, find the probability that:
7. Given two independent events A and B such that 3
P(A) = 0.3, P(B) = 0.6, Find (i) P(A and B), (ii) P(A and (i) Only A will hit the target
not B); (iii) P(A or B); (iv) P (neither A nor B). . (ii) Only B will hit the target.
8. A and B are two independent events such that P(A ∪ B) = (iii) Any one of them will hit the target.
0.6, P(A) = 0.2, find P(B). (iv) Both of them will hit the target.

Probability 393
E:\AMIT_WORKS\Exam_Guru\EG_Mathematics-12_(working_02-06-2022)\EG_Mathematics-12_working\Open_Files\Chap_13\Chap_13
\ 18-Aug-2022 Amit Proof-5 Reader’s Sign _______________________ Date __________

(v) Both of them will not hit the target. Find the probability that:
(vi) Atleast one of them will hit the target. (i) Only A solves the question.
(vii) At most one of them will hit the target. (ii) Only B solves the question.
14. If A and B are two independent events such that (iii) Only C solves the question.
2 1 (iv) Only A and B solve the question.
P(A′ ∩ B) = and P ( A ∩ B′ ) = , then find the values
15 6 (v) Only A and C solves the question.
of P(A) and P(B). [Delhi 2015] (vi) Only B and C solves the question.
15. Two students A and B are given a question to solve. They (vii) All of them solve the question.
try to solve the question separately. Probability that A can (viii) Atleast one of them solve the question.
solve the question is 1 and probability that B can solve (ix) Atleast two of them solve the question.
2 (x) At most one of them solve the question.
the question is 1 .
3 (xi) At most two of them solve the question.
Find the probability that: [V. Imp.] (xii) No one solves the question.
(i) Neither A nor B solves the question (xiii) Questions is solved.
(ii) Only A solves the question. (xiv) Any one of them solve the question.
(iii) Only B solves the question. (xv) Any two of them solve the question.
(iv) Any one of them solve the question. 17. A and B throw a die alternatively till one of them gets a 6
(v) Both of them solve the question. and wins the game. Find their respective probabilities of
winning if A starts first. [Delhi 2006, 2013]
(vi) Atleast one of them solve the question.
18. A, B and C throw a die turn by turn, till one of them gets a
(vii) Question is solved. 6 and wins the game. Find their respective probabilities of
(viii) At most one of them solve the question. winning if A starts the game, if he loses the game, it goes
 [Delhi 2006, 2011] to B and then to C. [Delhi 2009]
16. A problem of statistics is given to three students A, B and 19. A die whose faces are marked 1, 2, 3 in red and 4, 5, 6 in
C, to solve. Their respective probabilities to solve the green is tossed. Let A be the event that number obtained is
even and B be the event that number obtained is red. Are
problem are 1 , 1 and 1 .
2 3 4 events A and B independent? [A.I. 2017]

Answers 13.4
I. 1. As E and F are independent events, so 4
P(F = ace) = P(E ∩ F)
P(E ∩ F) = P(E).P(F) 52
P(E ∩ FC) = P(E) – P(E ∩ F) 1.
= P (ace of spade) =
= P(E) – P(E).P(F) = P(E) {1–P(F)} 52
E and F are independent events.
⇒ P(E ∩ FC) = P(E) P(FC)
Q P(E ∩ F) = P(E) . P(F).
P(E ∩ F) P(E ) P(F)
Now, P(E | F) = = = P(E ) III. 1. (i) E and F are independent.
P(F) P(F)
(ii) E and G are dependent.
Similarly, P(EC | FC) = P(EC)
\ P(E | F) + P(EC | FC) = P(E) + P(EC) = 1 (iii) F and G are independent.
Hence, (d) is the correct answer. Hint: n(S) = 8, E = {HHH, TTT},
3 3 2 F = {HHT, HTH, THH, HHH}
2. P(A) = , P(A′) = 1 – P(A) = 1 − − G = {HHT, HTH, THH, HTT, THT, TTH, TTT}
5 5 5
4 4 5 E ∩ F = {HHH}, E ∩ G = {TTT}
P(B) = , P(B′) = 1 − P(B) = 1 − =
9 9 9 F ∩ G = {HHT, HTH, THH}
2 5 2 2. Event A and B are independent.
Since P(A′∩ B′) = P(A′) × P(B′) = × =
5 9 9 Hint:
[As A and B are independent events, so A′ and B′ are also S = {1H, 2H, 3H, 4H, 5H, 6H, 1T, 2T, 3T, 4T, 5T, 6T}
independent events]
A = {1H, 2H, 3H, 4H, 5H, 6H], B = {3H, 3T}
Hence, (d) is the correct answer.
A ∩ B = {3H}
II. 1. Events E and F are independent.
13 3. Events A and B are dependent.
Hint: n(S) = 52. P(E = spades) = Hint: A = {2, 4, 6}, B = {1, 2, 3} A ∩ B = {2}
52

394 Mathematics–12
E:\AMIT_WORKS\Exam_Guru\EG_Mathematics-12_(working_02-06-2022)\EG_Mathematics-12_working\Open_Files\Chap_13\Chap_13
\ 18-Aug-2022 Amit Proof-5 Reader’s Sign _______________________ Date __________

4. E and F are independent events.


(v) 7 . Hint: P(A) ⋅ P(B′) + P(A′) P(B)
Hint: E = {(3,1), (3,2), (3,3), (3,4), (3,5), (3,6)} 20
F = {(1,2), (2,2), (3,2), (4,2), (5,2), (6,2)}, E ∩ F = {3,2} (vi) 2 .
5. (i) E and F are independent events. 5
(ii) E and F are independent events. Hint: P(A) P(B′) + P(A′) P(B) + P(A) . P(B)
(iii) E and F are dependent events. (vii) 19 .
20
6. A and B are not independent events:
Hint: P(A) P(B′) + P(A′) P(B) + P(A′) P(B′)
Hint: P(A′ ∪ B′) = P(A ∩ B)′
4
⇒ P(A ∩ B) = 1 – P(A ∩ B)′ 13. (i) Hint: P(A) P(B′)
15
7. (i) 0.18, (ii) 0.12, (iii) 0.72, (iv) 0.28 2 Hint: P(A′) P(B)
(ii)
1 15
8. P ( B) = .
2 2
(iii) Hint: P(A) P(B′) + P(A′) P(B)
Hint: P(A ∪ B) = P(A) + P(B) – P(A ∩ B) 5
⇒ P(A ∪ B) = P(A) + P(B) – P(A) × P(B) 8 Hint: P(A) P(B)
(iv)
Now put the values. 15
1 Hint: P(A′) P(B′)
9. P (they contradict) = 42%. Not necessary. (v)
15
Hint: P (Contradict) = P(A) × P(B ′) + P(A′) × P(B)
14
(ii) P (they agree) = 58%. No. (vi)
15
Hint: P (they agree) = P(A) P(B) + P(A′) × P(B′) Hint: P(A) P(B′) + P(A′) P(B) + P(A) P(B)
10. P(only one of them comes to school in time) or 1 – P(A′) × P(B′)
26
= 49 . (vii) 7
15
Hint: P (only one of them comes to school in time) = P(A) P(B′)
Hint: P(A) P(B′) + P(A′) P(B) + P(A′) P(B′)
+ P(A′) × P(B)
14. P(A) = 1 and P(B) = 1
11. (i) 1 . Hint: P(A) × P(B′) 5 6
7
Hint: P(A′ ∩ B) = 2
5 15
(ii) . Hint: P(A′) P(B)
42
⇒ P(B) − P(A ∩ B) = 2 ...(1)
(iii) 11 . 15
42
and P(A ∩ B′) = ⇒ P(A) − P(A ∩ B) = 1 
1 ...(2)
Hint: P(A) × P(B′) + P(A′) P(B) 6 6
(iv) 1 . Hint: P(A) × P(B) On subtracting (1) from (2) we get;
42
P(A) − P(B) = 1 ⇒ P(A) = 1 + P(B)
(v) . 2 30 30
7
Since A and B are independent events. So,
Hint: P(A) P(B′) + P(A′) P(B) + P(A) × P(B)
P(A ∩ B) = P(A) × P(B)
41 .
(vi)
42 \ P(B) − P(A ∩ B) = 2
15
Hint: P(A′) × P(B′) + P(A) P(B′) + P(A′) P(B) 2
⇒ P(B) − P(A) × P(B) = ...(3)
(vii) 5 . 15
7 1 + P(B)
Hint: P(A′) × P(B′) Put P(A) = in (3) and solves it for P(B)
30
1 4 1 3
12. P(A) = ⇒ P(A′) = , P(B) = 4 ⇒ P(B′ ) = 4 15. (i) 1 . Hint: P(A′) P(B′)
5 5 3
1
(i) 1 . Hint: P(A)× P(B) (ii) . Hint: P(A) P(B′)
20 3
(ii) 3 . Hint: P(A′)× P(B′) (iii) 1 . Hint: P(A′) P(B)
5 6
1
(iii) 3 . Hint: P(A)× P(B′) (iv) 2 . Hint: P(A) P(B′) + P(A′) P(B)
20
1
(iv) . Hint: P(A′) ⋅ P(B) 1;
5 (v) Hint: P(A) × P(B)
6

Probability 395
E:\AMIT_WORKS\Exam_Guru\EG_Mathematics-12_(working_02-06-2022)\EG_Mathematics-12_working\Open_Files\Chap_13\Chap_13
\ 18-Aug-2022 Amit Proof-5 Reader’s Sign _______________________ Date __________

2. (xii) 1 . Hint: P(A′) P(B′) P(C′)


(vi)
3 4
Hint: P(A) P(B′) + P(A′) P(B) + P(A) . P(B) 3
(xiii) . Hint: 1 − P(A′) P(B′) P(C′)
or 1 – P(A′) × P(B′) 4
11
(vii) 2 . Hint: 1 – P(A′) × P(B′) (xiv)
24
.
3
5 Hint: P(A) P(B′) P(C′) + P(A′) P(B) P(C′)
(viii) .
6 + P(A′) P(B′) P(C)
Hint: P(A′) P(B′) + P(A′) P(B) + P(A) P(B′) 1.
(xv)
or 1 – P(A) × P(B) 4
Hint: P(A) P(B) P(C′) + P(A′) P(B) P(C)
16. (i) 1 . Hint: P(A) P(B′) P(C′)
4 + P(A) P(B′) P(C)
1 . Hint: P(A′) P(B) P(C′)
(ii) 17. P(A) = 6 , P(B) = 5
8 11 11
1 . Hint: P(A′) P(B′) P(C)
(iii) Hint: pA = , qA = 5 , pB = 1 , qB = 5
1
12 6 6 6 6

(iv) 1 . Hint: P(A) P(B) P(C′) P(A) = pA + (qAqB)pA + (qAqB) (qAqB)pA + ...
8
1 = 1 1 + 25 + 625 + ...
. Hint: P(A) P(B′) P(C)  
(v)
12 6  36 1296
25
1 . Hint: P(A′) P(B) P(C) It is a (infinite) G.P. series a = 1, r =
(vi) 36
24 1 a 
P(A) =
1 . Hint: P(A) P(B) P(C) 6 1 − r 
(vii)
24
36 30 25
3 18. P(A) = , P(B) = , P(C) =
(viii) . Hint: 1 − P(A′) P(B′) P(C′) 91 91 91
4
Hint: P(A) = pA + (qAqBqC)pA + ...
7.
(ix) P(B) = qApB + (qA qB qC)qA pB + ...
24
Hint: P(A) P(B) P(C′) + P(A) P(B′) P(C) P(C) = 1 – [P(A) + P(B)].
+ P(A′) P(B) P(C) + P(A) P(B) P(C) 3=1 3 1
17 19. P(A) = , P(B) = =
(x) 6 2 6 2
24 1
Hint: P(A′) P(B′) P(C′) + P(A) P(B′) P(C′) P(A ∩ B) = P(To get an even red number) =
6
+ P(A′) P(B) P(C′) + P(A′) P(B′) P(C) P(A) . P(B) = 1 × 1 = 1 ≠ P ( A ∩ B)
2 2 4
(xi) 23
. Hint: 1 − P(A) P(B) P(C) \ Events A and B are not independent.
24

Topic 5. Partition of The Sample Space


The events E1, E2, E3 ..... En represent a partition of the sample
space S if they are pairwise disjoint, exhaustive and have non- Solution. Here, P ( E1) = 1 , P ( E 2 ) = 1 and P ( E 3 ) = 1 . [E1, E2
2 3 6
zero probabilities. For example, E and E′ are partition of a sample and E3 are bag’s selection probabilities and represent a partition
space S because E ∩ E′ = φ, E ∪ E′ = S and P(E) ≠ 0 and P(E′) ≠ 0. of sample space]

Theorem of Total Probability Let event A is to draw a white ball.


Let E1, E2, E3 .... En be the partition of the sample space P(A | E1) = P (To draw a white ball from bag I) = 3

S and let A be any other event associated with sample space S then 8
P(A) = P(E1) × P(A | E1) + P(E2) × P(A | E2) + ... + P(En) . P(A | En) P(A | E2) = P (To draw a white ball from bag II) =
5
Example 1. There are three bags I, II and III. Bag I has 3 white, 8
5 black balls, bag II has 5 white, 3 black balls and bag III has 4
P(A | E3) = P (To draw a white ball from bag III) =
4 white, 4 black balls. A bag is selected at random and a ball is 8
drawn from it. What is the probability that the ball drawn is white? Now probability to draw a white ball
Bags are such that their selection probabilities are 1 , 1 and 1 P(A) = P(E1) × P(A | E1) + P(E2) × P(A | E2) +
2 3 6 P(E3) P(A | E3)
respectively.

396 Mathematics–12
E:\AMIT_WORKS\Exam_Guru\EG_Mathematics-12_(working_02-06-2022)\EG_Mathematics-12_working\Open_Files\Chap_13\Chap_13
\ 18-Aug-2022 Amit Proof-5 Reader’s Sign _______________________ Date __________

1 3 1 5 1 4 3 5 4 Example 3. Suppose that reliability of HIV test is specified as


= × + × + × = + + follows: Of people having HIV, 90% of the test detect the disease
2 8 3 8 6 8 16 24 48
9 + 10 + 4 23 but 10% go undetected. Of people free of HIV, 99% of the test
= = ⇒ P ( A ) = 23 are judge HIV negative but 1% are diagnosed as showing HIV
48 48 48
positive. From a large population of which only 0.1% have
Bayes’ Theorem (Most Important Topic) HIV one person is selected at random, given the HIV test and
Let E1, E2, E3 ... En be partition of sample space S and A be any the pathologist reports him/her as HIV positive. What is the
probability that the person actually has HIV?
other event of the sample space S. Then
Solution. Let E denotes the event that the person selected is
P(Ei | A)
actually having HIV and A the event that the person’s HIV test
P ( E i ) × P ( A|Ei )
= is diagnosed as positive. We need to find P(E/A).
P ( E1 ) × P ( A|E1 ) + P ( E 2 ) × P ( A|E 2 ) + ... + P ( E n ) P ( A|E n )
Also E′ denotes the event that the person selected is actually not
where i = 1, 2, 3, ... n. having HIV.
Example 2. A company has two plants to manufacture scooters. Clearly (E and E′) is a partition of the sample space of all people
Plant I manufactures 70% of scooters and plant II manufactures in the population. We are given that:
30%. At plant I, 80% scooters are rated of standard quality and
at plant II, 90% of the scooters are rated of standard quality. A P(E) = 0.1% = 0.1 = 0.001
100
scooter is picked up at random and is found to be of standard
⇒ P(E′) = 1 – P(E) = 1 – 0.001 = 0.999
quality. What is the chance that it has come from plant I?
P(A/E) = P (Person tested as HIV positive that he/she is
70 7 30 3
Solution. P(I) = = , P ( II ) = = actually having HIV) = 90% = 0.9 and P(A/E′) = P(Person
100 10 100 10
tested as HIV positive given that he/she is actually not-having
If A is the event that the scooter is of standard quality. Then:
HIV) = 1% = 0.01.
P ()
A
I
=
80
100 10
=
8
,P
A
()
II
=
90
100 10
=
9
Now, by Bayes’ theorem:

Now we want to find out P(I/A) P ( E ) × P ( A/E )


P(E/A) =
P ( E ) × P ( A/E ) + P ( E′ ) P ( A/E′ )
P I = ()
A
P ( I ) × P ( A/I )
P ( I ) × P ( A/I ) + P ( II ) × P ( A/II ) =
0.001 × 0.9
0.001 × 0.9 + 0.999 × 0.01
7 × 8 56
56 100 56 90
= 7 10 8
10
3 9
= 100 =
56 + 27
× = = = 0.083 (approx.)
× + × 100 83 83 1089
10 10 10 10 100
Thus the probability that a person selected at random is
∴ The probability that selected scooter has been manufactured actually having HIV has given that he/she is tested HIV positive
56 is 0.083.
in plant I = .
83

EXERCISE 13.5
I. Multiple Choice Questions (MCQs) to be red. Find the probability that it was drawn from
1. In a group of 400 people, 160 are smokers and bag II. [V. Imp.] [Delhi 2007, 2009 (C), A.I. 2007]
non-vegetarian, 100 are smokers and vegetarian and 2. A bag contains 3 red and 7 black balls. Two balls are
the remaining are non-smokers and vegetarian. The selected at random one by one without replacement. If the
probabilities of getting a special chest disease are 35%, second selected ball be red, what is the probability that the
20% and 10% respectively. A person is chosen from the first selected ball is also red? [Delhi 2014 (C)]
group at random and is found to be suffering from the
3. In a factory which manufactures bolts, machines A, B
disease. The probability that selected person is a smokers
and C manufactures respectively 30%, 50%, 20% of the
and non-vegetarian is:
bolts. Of their 3, 4 and 1 percent respectively are defective
90 29
(a) (b) bolts. A bolt is drawn at random from the product and is
1089 45 found to be defective. Find the probability that this is not
28 manufactured by machine B.[A.I. 2015, Delhi 2010 (C)]
(c) (d) None of these.
45 4. A box has 5 balls. Two balls are drawn at random and are
II. Short Answer Type Questions-II found to be both white. What the probability that the box
1. There are two bags I and II. Bag I contains 3 white and 2 has 4 white balls?
red balls and bag II contains 5 white and 4 red balls. One 5. Two cards are drawn from well shuffled deck of 52 cards,
ball is drawn at random from one of the bags and is found successively without replacement. Second card is found to

Probability 397
E:\AMIT_WORKS\Exam_Guru\EG_Mathematics-12_(working_02-06-2022)\EG_Mathematics-12_working\Open_Files\Chap_13\Chap_13
\ 18-Aug-2022 Amit Proof-5 Reader’s Sign _______________________ Date __________

be a spade, what is the probability that first card was also


a spade? are 1 , 1 and 1 if he comes by train, bus and scooter
4 3 12
III. Long Answer Type Questions respectively, but if he comes by other means of transport
then he will not be late. When he arrives, he is late. What
1. 40% students of a college reside in the hostel and the
is the probability that he comes by train? Or He comes by
remaining reside outside. At the end of the year, 50% of
bus? [Delhi 2008 (C), A.I. 2009]
the hosteliers got A grade while from outside students
only 30% got A grade in the examination. At the end of 8. A man is known to speak truth 3 out of 4 times. He throws
the year, a student of the college was chosen at random a die and reports that it is a six. Find the probability that
and was found to get A grade. What is the probability that it is actually a six.
the selected student was hostelier?  [Delhi 2011, 2014 (C) A.I. 2008 (C) 2009]
 [SP Delhi 2010, 2011 (C) 2012] 9. A card from a pack of 52 cards is lost. From the remaining
2. An insurance company insured 2000 scooter drivers, 4000 cards of the pack two cards are drawn and found to be both
car drivers and 6000 truck drivers. The probability of an diamonds. Find the probability that the lost card being a
accident involving a scooter, a car and a truck are 0.01, diamond. [Delhi 2010, 2012 (C)]
0.03 and 0.15 respectively. One of the insured person meets 10. A man is known to speak the truth 3 out of 5 times. He
with an accident. What is the probability that he is a scooter throws a die and reports that it is a number greater than
driver? [Delhi 2005, 2008, 2009, A.I. 2008, 2008 (C)] 4. Find the probability that it is actually a number greater
3. In a bulb factory, machines A, B and C manufacture 60%, than 4. [A.I. 2009]
30% and 10% bulbs respectively. 1%, 2% and 3% of the 11. In a factory which manufactures bolts, machine A, B and
bulbs produced respectively by A, B and C are found to
C manufacture respectively 25%, 35% and 40% of the
be defective. A bulb is picked up randomly from the total
bolts. Of their outputs 5, 4 and 2 percent are respectively
production and found to the defective. Find the probability
defective bolts. A bolt is drawn at random from the product
that this bulb was manufactured by the machine A.
and is found to be defective. What is the probability that
 [A.I. 2008]
it is manufactured by the machine B?
4. Three bags I, II, III contain balls as shown in the table
 [Delhi 2006, 2010 (C), A.I. 2014]
given below:
12. Suppose 5% of men and 0.25% of women have grey hair.
Bag Number of Number of Number of A grey haired person is selected at random. What is the
white balls black balls red balls probability of this person being male? Assume that there
I 1 2 3 are equal number of males and females. [Delhi 2011]
II 2 1 1 13. Given three identical boxes I, II and III, each containing
III 4 3 2 two coins. In the box I, both coins are gold coins, in box
II, both are silver coins and in the box III, there is one gold
A bag is chosen at random and two balls are drawn from it.
and one silver coin. A person chooses a box at random
They happen to be white and red, what is the probability
that they come from third bag? and takes out a coin. If the coin is of gold, what is the
probability that the other coin in the box is also of gold?
 [Delhi 2009, 2009 (C) A.I. 2009]
 [A.I. 2011]
5. Two groups are competing for the position on the Board of
Directors of a corporation. The probability that the first and 14. Suppose a girl throws a die. If she gets a 5 or 6, she tosses
second groups will win are 0.6 and 0.4 respectively. Further, a coin three times and notes the number of heads. If she
if the first group wins, the probability of introducing a new gets 1, 2, 3 or 4, she tosses a coin once again and notes
product is 0.7 and the corresponding probability is 0.3 if whether a head or tail is obtained. If she obtained exactly
the second group wins. Find the probability that the new two heads what is the probability that she threw 1, 2, 3 and
product was introduced by second group. [Delhi 2009] 4 with a die? [Delhi 2012, A.I. 2012]
6. There are three coins one is a two headed coin (having 15. Assume that the chances of a patient having a heart attack
head on both the faces) another is a biased coin that comes is 40%. It is also assumed that a meditation and yoga course
up tail 25% of times and third is an unbiased coin. One of reduce the risk of heart attack by 30% and prescription
the three coins is chosen at random and tossed, it shows of a certain drug reduces its chance by 25%. At a time a
heads, what is the possibility that it was the two headed patient can choose any one of the two options with equal
coins? [Delhi 2009, A.I. 2011 (C)] probabilities. It is given that after going through one of
7. A doctor is to visit a patient. From the past experience, it the two options the patient selected at random suffers a
is known that the probabilities that he will come by train, heart attack. Find the probability that the patient followed
bus, scooter or by other means of transport are respectively, a course of meditation and yoga. Interpret the result and
3 1 1 2 state which of the above stated method is more beneficial
, , and . The probability that he will be late
10 5 10 5 for the patient. [Delhi 2013, A.I. 2013 (C)]

398 Mathematics–12
E:\AMIT_WORKS\Exam_Guru\EG_Mathematics-12_(working_02-06-2022)\EG_Mathematics-12_working\Open_Files\Chap_13\Chap_13
\ 18-Aug-2022 Amit Proof-5 Reader’s Sign _______________________ Date __________

16. In a group of 400 people, 160 are smokers and non- 21. Two cards from a pack of 52 cards are lost. From the
vegetarians, 100 are smokers and vegetarians and remaining pack of cards one card is drawn and found to
the remaining are non-smokers and vegetarians. The be diamond. Find the probability that both the lost cards
probabilities of getting a special chest disease are 35%, being diamond.
20% and 10% respectively. A person is chosen from the 22. A box contains 5 gold and 4 silver coins, two coins are stolen
group at random and is found to be suffering from the chest from the box. Now one coin is drawn at random from the
diseases. What is the probability that the selected person box and is found to be of gold. Find the probability that
is a smoker and non-vegetarian? What value is reflected
lost coins also were of gold.
in the question? [Delhi 2013 (C)]
23. A box has 5 balls. Three balls are drawn at random and are
17. A card from a pack of 52 playing cards is lost. From the
found to be white. What is the probability that the box has
remaining cards of the pack, three cards are drawn at
random (without replacement) and are found be all spades. all the white balls?
Find the probability of the lost card being a spade. 24. In answering a question on a multiple choice test, a
 [Delhi 2014] student either knows the answer or guesses. Let 3 be
4
18. An urn contains 4 balls. Two balls are drawn at random the probability that he knows the answer and 1 be the
from the urn (without replacement) and are found to be 4
all white. What is the probability that all the four balls in probability that he guesses. Assuming that a student who
the urn are white? [A.I. 2014] guesses at the answer will be correct with probability 1 .
19. A bag A contains 4 black and 6 red balls and bag B contains 4
7 black and 3 red balls. A die is thrown. If 1 or 2 appears What is the probability that the student knows the answer
on it, then bag A is chosen, otherwise bag B is chosen. If given that he answered it correctly?
two balls are drawn at random (without replacement) from 25. A laboratory blood test is 99% effective in detecting a
the selected bag, find the probability of one of them being certain disease when it is in fact present. However, the test
red and another black. [Delhi 2015] also yields a false positive result for 0.5% of the healthy
20. In a class 5% of boys and 10% of girls have an I.Q. of more person tested. (i.e., if a healthy person is tested, then with
than 150. In the class 60% of students are boys and rest probability 0.005, test will imply that he has the disease).
are girls. If a student is selected at random and is found to If 0.1 percent of the population actually has the disease,
be having I.Q. of more than 150, find the probability that what is the probability that a person has the disease given
the student is a boy. [A.I. 2010 (C)] that his test result is positive?

Answers 13.5
I. 1. E1 : The person selected is a smoker and non-vegetarian.
II. 1. 10
E2 : The person selected is a smoker and vegetarian. 19
1 1
E3 : The person selected is a non-smoker and vegetarian. Hint:
P(I) = , P(II) =
2 2
A : Special chest disease: 2 4
P(R | I) = , P(R | II) =
5 9
160 100 140
P(E1) = , P(E 2 ) = , P(E 3 ) = P ( II ) . P ( R | II )
400 400 400 P(II | R) = ( )
P I . P ( R | I ) + P ( II ). P ( R | II )
35 20 10
P(A|E1) = , P(A|E 2 ) = , P(A|E 3 ) = 1×4 2
100 100 100
= 2 9 = 9
P(E1|A) = Probability that selected person is a smoker 1×2+1×4 1+2
and non-vegetarian is 2 5 2 9 5 9
P(E1 ) . P(A/E1 ) 2
=
P(E1 ).P(A|E1 ) + P(E 2 ).P(A|E 2 ) + P(E 3 ).P(A|E 3 ) = 9 = 2 × 45 = 10
19 9 19 19
160 35 45
× 2
400 100 2.
=
160 35 100 20 140 10
9
× + × + × Hint: E1 (Red ball), E2 (black ball)
400 100 400 100 400 100 3 ,P E = 7

\ P(E1) =
10
( 2 ) 10
5600
= Let A be the event that first ball is red
5600 + 2000 + 1400
2 3
5600 28 \ P(A | E1) = , P ( A | E 2 ) =

= 9 9
=
9000 45 P ( E1) × P ( A/E1)

\ P(E1 | A) =
Hence, (c) is the correct answer. P ( E1) P ( A/E1) + P ( E 2 ) × P ( A/E 2 )

Probability 399
E:\AMIT_WORKS\Exam_Guru\EG_Mathematics-12_(working_02-06-2022)\EG_Mathematics-12_working\Open_Files\Chap_13\Chap_13
\ 18-Aug-2022 Amit Proof-5 Reader’s Sign _______________________ Date __________

11 10
3. III. 1.
31 19
30 50 20 Hint: E1 = Hosteler and E2 = Outsider
Hint:
P(A) = , P(B) = , P(C) =
100 100 100
A be the event that the selected students got grade A
Let D be the event of defective bolt 2
\ P(E1) =
3 4 5
P(D | A) = , P ( D | B) =
100 100 3
and P(E2) =
1 5
P(D | C) =
100 1 3
P(A | E1) = and P(A | E2) =
P(Not manufactured by machine B) = 2 10
Now use
P(A|D) + P(C|D)
P ( E1).P ( A | E1)
P ( A ) P ( D | A ) + P ( C ) .P ( D | C ) P(E1 | A) =
P ( E1).P ( A | E1) + P ( E 2 ).P ( A | E 2 )
= ( )
P A .P ( D | A ) + P ( B).P ( D | B) + P ( C).P ( D | C)
Put the values and get the answer.
3
4. 1
10 2.
52
Hint: Assume that there are 4 cases 2000 1 4000 = 2
Hint:
P(S) = = , P(C) =
Case I = 5 white balls 12000 6 12000 6
Case II = 4 White and 1 Non white balls 6000 = 3
P(T) =
Case III = 3 White and 2 Non white balls 12000 6
Case IV = 2 White and 3 Non white balls Let A be the event of accident
\ P(A | S) = 0.01, P(A | C) = 0.03, P(A | T) = 0.15
1
P(I) = P(II) = P(III) = P(IV) = P (S).P ( A | S)
4 P(S | A) =
Let A represent that there are 4 white balls P (S).P ( A | S) + P ( C).P ( A | C)
5
C + P (T ).P ( A | T )
\ P(A | I) = 5 2 = 1
C2 1 × 0.01
4 = 6
C2 6 1 × 0.01 + 2 × 0.03 + 3 × 0.15
P(A | II) = 5 =
C2 10 6 6 6
3
C2 3 0.01 1
P(A | III) = = = =
5
C2 10 0.01 + 0.06 + 0.45 52
2
C 2 3.
P(A | IV) = 5 2 = 1 5
C2 10
Hint:
P(A) = 60 , P(B) = 30 , P(C) = 10
P ( II ).P ( A | II ) 100 100 100
P(II | A) =
P ( I ).P ( A | I ) + P ( II ).P ( A | II ) + Let D be the event of defective
P ( III ).P ( A | III ) + P ( IV ).P ( A | IV ) \ P(D | A) =
1
, P(D | B) =
2
,
100 100
1× 6
4 10 3
= P(D | C) =
1 ×1+ 1 × 6 + 1 × 3 + 1 × 1 100
4 4 10 4 10 4 10 P (A ) . P (D | A )
P(A | D) =
6 6 P A . P ( D | A ) + P ( B) . P ( D | B)
( )
= 10 = 10 = 6 = 3 + P (C) . P (D | C)
1+ 6 + 3 + 1 20 20 10
Put the values and get the answer.
10 10 10 10
5
4 4.
5. 17 17
1 1 1
Hint:
E1 = spades and E2 = Non spade Hint: P(I) = , P(II) = , P(III) =
3 3 3
1 3 A is the event to drawn 1W and 1R
\ P(E1) = and P ( E 2 ) =
4 4 1
C1 × 3C1 1
Let A be the event the first card is spade P(A | E1) = 6 =
C2 5
12 13
\ P(A | E1) = and P(A | E2) = 2
C1 × 1C1 1
51 51 P(A | E2) = =
4 3
P ( E1).P ( A | E1) C2
P(E1 | A) =
P ( E1).P ( A | E1) + P ( E 2 ).P ( A | E 2 ) 4
C1 × 2C1 2
P(A | E3) = 9 =
Put the values and get the answer. C2 9

400 Mathematics–12
E:\AMIT_WORKS\Exam_Guru\EG_Mathematics-12_(working_02-06-2022)\EG_Mathematics-12_working\Open_Files\Chap_13\Chap_13
\ 18-Aug-2022 Amit Proof-5 Reader’s Sign _______________________ Date __________

P ( E 3 ) .P ( A | E 3 ) 11
9.
P(E3 | A) =
P ( E1).P ( A | E1) + P ( E 2 ).P ( A | E 2 ) 50
+ P ( E 3 ).P ( A | E 3 ) Hint: Let E1 = Event that lost card is diamond,
E2 = Event that lost card is not diamond.
= 5
17 \ P(E1) = 13 = 1 , P(E2) = 3
2 52 4 4
5. Let A be the event that 2 cards are diamond,
9
P(A | E1) = Prob. of getting 2 diamond cards when 1
Hint: P(I) = 0.6, P(II) = 0.4
diamond card is lost.
A be the event of introducing the new product 12
C2 12 × 11
P(A | I) = 0.7 and P(A | II) = 0.3
\ P(A | E1) = 51=
C2 51 × 50
P II ) . P ( A | II )
(
P(II | A) = ( ) P(A | E2) = Prob. of getting 2 diamond cards when the lost
P I . P ( A | I ) + P ( II ) . P ( A | II )
card is other than diamond
0.4 × 0.3 0.12 13
= = C2 13 × 12
0.6 × 0.7 + 0.4 × 0.3 0.42 + 0.12 P(A | E2) = = 51
C2 51 × 50
0.12 = 12 = 2
= To get P(E1 | A) use
0.54 54 9
P ( E1).P ( A | E1)
4 P(E1 | A) =
6. P ( E1).P ( A | E1) + P ( E 2 ).P ( A | E 2 )
9
Now put the values.
1
Hint:
P(I) = P(II) = P(III) = 3
3 10.
7
3 1 Hint: Let E1 and E2 be the events that 5 or 6 occurs and 5 or 6
P(H | I) = 1, P(H | II) = . P(H | III) =
4 2 does not occur respectively.
P ( I ) .P ( H | I ) 2 1 2
Find P(I | H) = \ P(E1) = = and P(E2) =
P I .P ( H | I ) + P ( II ).P ( H | II )
( ) 6 3 3
+ P ( III ).P ( H | III ) Let A be the event that man reports that 5 or 6 has occurred
3 2
Put the values and get the answer. \ P(A | E1) = and P(A | E2) =
5 5
4
7. (i) 1 ; (ii) To get P(E1 | A) use
2 9
P ( E1).P ( A | E1)
3 1 1 2 P(E1 | A) =
Hint: P(T) = , P(B) = , P(S) = , P(O) = P ( E1).P ( A | E1) + P ( E 2 ).P ( A | E 2 )
10 5 10 5
1 1 1 1×3
P(L | T) = , P(L | B) = , P(L | S) = 3 =3
4 3 12 = 1 33 25 2 =
× + × 3+ 4 7
P(L | O) = 0 3 5 3 5
P (T ).P ( L | T ) 28
(i) P(T | L) = 11.
P T .P ( L | T ) + P ( B).P ( L | B)
( ) 69
25 35
+ P (S).P ( L | S) + P ( O ).P ( L | O ) Hint:
P(E1) =
100
, P(E2) =
100
P ( B).P ( L | B) 40
(ii) P(B | L) = and P(E3) =
P (T ).P ( L | T ) + P ( B).P ( L | B) 100
+ P (S).P ( L | S) + P ( O ).P ( L | O ) Let D be the event of defective bolts
Put the values and get the answer. 5 4
\ P(D | E1) = , P(D | E2) = ,
100 100
3
8. 2
8 P(D | E3) =
100
Hint: Let E1 and E2 be the events that the man gets a 6 and the
man does not get a 6. Now find P(E2 | D)
1 5 P ( E 2 ) .P ( D | E 2 )
\ P(E1) = and P(E2) = =
6 6 P ( E1).P ( D | E1) + P ( E 2 ).P ( D | E 2 ) + P ( E 3 ).P ( D | E 3 )
Let A be the event that the man speaks truth.
20
3 1 12.
\ P(A | E1) = and P(A | E2) = 21
4 4
Now apply Bayes’ theorem, Hint: Let E1 and E2 be the events of selection of male and female
P ( E1).P ( A | E1) respectively.
P(E1 | A) =
P ( E1).P ( A | E1) + P ( E 2 ).P ( A | E 2 ) A is the event of grey hair
1
Now put the values. \ P(E1) = P(E2) =
2

Probability 401
E:\AMIT_WORKS\Exam_Guru\EG_Mathematics-12_(working_02-06-2022)\EG_Mathematics-12_working\Open_Files\Chap_13\Chap_13
\ 18-Aug-2022 Amit Proof-5 Reader’s Sign _______________________ Date __________

P(A | E1) = P(grey hair men) = 5% P(E1 | A)


P(A | E2) = P (grey hair women) = 0.25% P ( E1).P ( A | E1)
=
Now find P(E1 | A) P ( E1).P ( A | E1) + P ( E 2 ).P ( A | E 2 ) + P ( E 3 ).P ( A | E 3 )
P ( E1).P ( A | E1)
P(E1 | A) = 17. 10
P ( E1).P ( A | E1) + P ( E 2 ).P ( A | E 2 ) 49
2 Hint: E1 = Lost card spade
13.
3 E2 = Lost card is not spade
Hint: Let E1, E2 and E3 be the events that boxes I, II and III are 13 = 1 , P E = 39 = 3
chosen respectively. \ P(E1) =
52 4
( 2 ) 52 4
1 Let A be the event that three cards are spades.
\ P(E1) = P(E2) = P(E3) =
3 12 13
Let A be the event that the other coin drawn is of gold. C3 C3

\ P(A | E1) = 51 and P(A | E2) = 51
1 C3 C3
\ P(A | E1) = 2 = 1 , P(A | E2) = 0, P(A | E3) =
2 2 To find P(E1 | A) use
We want to work out the probability of the box containing both P ( E1).P ( A | E1)
gold coins, i.e., box I, i.e., P(E1 | A). Use P(E1 | A) =
P ( E1).P ( A | E1) + P ( E 2 ).P ( A | E 2 )
P ( E1).P ( A | E1)
P(E1 | A) = 3
P ( E1).P ( A | E1) + P ( E 2 ).P ( A | E 2 ) 18.
5
+ P ( E 3 ).P ( A | E 3 ) Hint: There are 3 possible cases for the 4 balls in the urn,
Put the values.
(i) E1 = 2W and 2 non W
8
14. (ii) E2 = 3W and 1 non W
11
Hint: Let E1 be the event to get 5 or 6. (iii) E3 = 4W
2 1 1
\ P(E1) = = \ P(E1) = P(E2) = P(E3) =
6 3 3
and E2 be the event to get 1, 2, 3, 4, Let A is the event to draw 2 white balls.
4 2 2 3
\ P(E2) = = C2 1 C2 1
6 3
\ P(A | E1) = 4 = , P(A | E 2 ) = 4 =
C2 6 C2 2
A is the event to get 1 head,
4
3 1 C2
\ P(A | E1) = , P(A | E2) = P(A | E3) = =1 4
8 2 C2
To find P(E2 | A). Use Now find P(E3 | A) and use
P ( E 2 ) .P ( A | E 2 ) P ( E 3 ) .P ( A | E 3 )
P(E2 | A) = P(E3 | A) =
P ( E 2 ).P ( A | E1) + P ( E 2 ).P ( A | E 2 ) P ( E1).P ( A | E1) + P ( E 2 ).P ( A | E 2 )
Now put the values. + P ( E 3 ).P ( A | E 3 )
22
14 19.
15. first method is better. 45
29
Hint: P(A) = 2 = 1 , P(B) = 4 = 2
Hint: E1 = event of treatment by yoga. 6 3 6 3
E2 = event of treatment by prescribing certain drugs Let A be the event to draw 1 R and 1B.
6
1 C1 × 4C1 6 × 4 24 8
\ P(E1) = P(E2) =
\ P(1R 1Bl | A) = 10 = = =
2 C2 45 45 15
Let A be the event that person has heart attack. 3
C1 × 7C1 3 × 7 21 7
\ P(A | E1) = 40% of 70% = 0.28 P(1R 1Bl | B) = 10 = = =
C2 45 45 15
P(A | E2) = 40% of 75% = 0.30
P(1Bl 1R) = P(A) × P(1Bl 1R | A) + P(B)P.(1Bl 1R | B)
Now find P(E1 | A) and use
P ( E1).P ( A | E1) = 1 × 8 + 2 × 7 = 8 + 14 = 22
P(E1 | A) = 3 15 3 11 45 45
P ( E1).P ( A | E1) + P ( E 2 ).P ( A | E 2 ) 3
20.
28 7
16.
45 Hint: E1 = Number of boys, E2 = Number of girls
Hint: E1 = Smoker and non-vegetarian A = The student have an I.Q. more than 150.
E2 = Smoker and vegetarian \ P(E1) = 60% = 0.6, P(E2) = 40% = 0.4
E3 = Non-smoker vegetarian P(A | E1) = 5% = 0.05, P(A | E2) = 10% = 0.1
Now find P(E1 | A) and use
160 , P(E ) = 100 , P(E ) = 140
\ P(E1) = 2 3 P ( E1).P ( A | E1)
400 400 400 P(E1 | A) =
Find P(E1 | A) by using P ( E1).P ( A | E1) + P ( E 2 ).P ( A | E 2 )

402 Mathematics–12
E:\AMIT_WORKS\Exam_Guru\EG_Mathematics-12_(working_02-06-2022)\EG_Mathematics-12_working\Open_Files\Chap_13\Chap_13
\ 18-Aug-2022 Amit Proof-5 Reader’s Sign _______________________ Date __________

22 2
21. 425 23.
3
Hint: E1 = both lost cards are diamond. Hint: There are 3 possible cases
E2 = 1 lost card is diamond and 1 non-diamond E1 = 3 White and 2 Non-white
E3 = both lost cards are non-diamond
E2 = 4 White and 1 Non-white
Let A be the event that card drawn is a diamond.
13 E3 = 5 White
C 2 1
\ P(E1) = 52 2 = , P(E1) = P(E2) = P(E3) =
C2 34 3
13
C1 × 39C1 13 A be the event that 3 drawn balls are white
P(E2) = 52 = , 3 4
C2 34 C3 1 C 4

\ P(A | E1) = 5 = , P(A | E2) = 5 3 =
C2 19 39 C3 10 C3 10
P(E3) = = 52
C2 34 5
C3
P(A | E3) = 5 =1
P(A | E1) = 11 , P(A | E2) = 12 C3
50 50
Now find P(E3 | A)
P(A | E3) = 13
50 P ( E 3 ) .P ( A | E 3 )
P(E3 | A) =
Now find P(E1 | A) P ( E1).P ( A | E1) + P ( E 2 ).P ( A | E 2 )
P ( E1).P ( A | E1) + P ( E 3 ).P ( A | E 3 )
P(E1 | A) = 12
P ( E1).P ( A | E1) + P ( E 2 ).P ( A | E 2 ) 24.
13
+ P ( E 3 ).P ( A | E 3 ) Hint: E1 = student knows the answer.
3
22. 14 E2 = student guesses the answer
Hint: E1 = both stolen coins are of gold. \ P(E1) = 3 , P(E2) = 1
E2 = 1 gold and 1 silver coins are stolen 4 4
E3 = both silver coins are stolen. Let A be the event that answer is correct.
5
C 5 \ P(A | E1) = 1 (He knows the answer)
P(E1) = 9 2 = , 1
C2 18 P(A | E2) = (Out of 4 choices he guesses 1 answer)
4
5
C1 × 4C1 10 To find P(E1 | A) use
P(E2) = 9 = ,
C2 18 P ( E1).P ( A | E1)
P(E1 | A) =
C2 3 4 P ( E1).P ( A | E1) + P ( E 2 ).P ( A | E 2 )
P(E3) = =
C2 18 9 22
25.
Let A be the event that one coin drawn is of gold 133
3 5 Hint: E1 = The person has disease
\ P(A | E1) = , P(A | E2) = 4 , P(A | E3) =
7 7 7 E2 = The person does not have disease
Now find: P(E1 | A) A = Blood test is positive
P ( E1).P ( A | E1) \ P(E1) = 0.1% = 0.001
P(E1 | A) =
P ( E1).P ( A | E1) + P ( E 2 ).P ( A | E 2 ) P(E2) = 1 – 0.001 = 0.999
+ P ( E 3 ).P ( A | E 3 ) P(A | E1) = 99% = 0.99, P(A | E2) = 0.005
5 ×3 Now find P(E1 | A)
= 18 7 = 3 P ( E1).P ( A | E1)
5 × 3 + 10 × 4 + 3 × 5 14 P(E1 | A) =
18 7 18 7 18 7 P ( E1).P ( A | E1) + P ( E 2 ).P ( A | E 2 )

Topic 6. Random Variable


A random variable is a real valued function whose domain is the Probability Distribution of a Random Variable
sample space of the experiment. A description giving the values of the random variable along
S = {HHH, HHT, HTH, THH, HTT, THT, TTH, TTT} with the corresponding probabilities is called the probability
If X denotes the number of heads obtained, then X is a random distribution of the random variable X. Let:
variable and for each outcome, its value is given:
X: x1 x2 x3 ... xn
X(HHH) = 3, X(HHT) = 2, X(HTH) = 2, X(THH) = 2,
P(X) p1 p2 p3 ...pn
X(HTT) = 1, X(THT) = 1, X(TTH) = 1, X(TTT) = 0
⇒ (X = 0, 1, 2, 3). Here, pi > 0 and p1 + p2 + p3 .... + pn = 1

Probability 403
E:\AMIT_WORKS\Exam_Guru\EG_Mathematics-12_(working_02-06-2022)\EG_Mathematics-12_working\Open_Files\Chap_13\Chap_13
\ 18-Aug-2022 Amit Proof-5 Reader’s Sign _______________________ Date __________

Example 1. The probability distribution of a discrete random Solution. The probability distribution of X is:
variable X is given below:
X 0 1 2 3 4 5 6 7
X 2 3 4 5
P(X) 0 k 3k 4k k 2k 5k 3k 2
2 2
5 7 9 11
P(X) 7
k k k k
We know that: ∑ pi = 1
i=0
The value of k is
(a) 8 (b) 16 (c) 32 (d) 48 \ 0 + k + 3k + 4k + k + 2k2 + 5k2 + 3k2 = 1
n ⇒ 10k2 + 9k – 1 = 0 ⇒ 10k2 + 10k – k – 1 = 0
Solution. We know that ∑ P(Xi ) = 1
i=1 ⇒ 10k(k + 1) – 1(k + 1) = 0 ⇒ (k + 1)(10k – 1) = 0
5 7 9 11

\   + + + =1 ⇒ k = –1 and k = 1 ; (k = − 1 not possible)
k k k k 10
32 (i) k= 1
  
= 1  k = 32. 10
k
(ii) P(X ≤ 2) = P(X = 0) + P(X = 1) + P(X = 2)
Hence, the correct option is (c).
Example 2. A coin is tossed three times. Find the probability = 0 + k + 3k = 4k = 4 × 1 = 2
10 5
distribution.
\ P(X ≤ 2) = 2
Solution. The sample space of the tossing of a coin three times is: 5
S = {HHH, HHT, HTH, THH, HTT, THT, TTH, TTT} (iii) P(X ≥ 6) = P(X = 6) + P(X = 7) = 5k2 + 3k2 = 8k2

( )
 Here, there are 8 outcomes and each outcomes is equally 2
1 8 2
likely, to occur, therefore the probability of occurrence of = 8× = =
10 100 25
each outcomes is 1 . 2
8 \ P(X ≥ 6) =
If number of heads x, is taken the random variable, then: 25
(iv) P(X > 4) = P(X = 5) + P(X = 6) + P(X = 7)
X{HHH} gives x = 3 ⇒ P ( x = 3) = 1
8 = 2k2 + 5k2 + 3k2
X{HHT, HTH, THH} gives:

( )
2
x= 2 ⇒ P(x = 2) = 3 = 10k2 = 10 ×
1
=
1
8 10 10
X{HTT, THT, TTH} gives:
\ P(X > 4) = 1
x= 1 ⇒ P(x = 1) = 3 10
8
X{TTT} gives: (v) P(2 < X < 6) = P(X = 3) + P(X = 4) + P(X = 5)

x= 0 ⇒ P(x = 0) = 1 = 4k + k + 2k2 = 5k + 2k2


8
( )
2
The above information can be displayed as: 1 1
= 5× +2×
10 10
0 xi 1 2 3
5 2 52 13
1 pi 3 3 1 = + = =
10 100 100 25
8 8 8 8
This is called the probability distribution of a random 13
\ P(2 < X < 6) =
variable X. 25
Example 3. A random variable X has the following probability (vi) P(3 ≤ X ≤ 5) = P(X = 3) + P(X = 4) + P(X = 5)
distribution: = 4k + k + 2k2 = 5k + 2k2

( )
X 0 1 2 3 4 5 6 7 2

P(X) 0 k 3k 4k k 2k 5k 3k 2
2 2 = 5× 1 +2× 1
10 10
Determine: (i) k, (ii) P(X ≤ 2), (iii) P(X ≥ 6), (iv) P(X > 4), 5 2 52 13
= + = =
(v) P(2 < X < 6), (vi) P(3 ≤ X ≤ 5). 10 100 100 25

404 Mathematics–12
E:\AMIT_WORKS\Exam_Guru\EG_Mathematics-12_(working_02-06-2022)\EG_Mathematics-12_working\Open_Files\Chap_13\Chap_13
\ 18-Aug-2022 Amit Proof-5 Reader’s Sign _______________________ Date __________

EXERCISE 13.6
I. Multiple Choice Questions (MCQs) drawn at random (without replacement) from the urn. Find
1. A random variable X has the probability distribution the probability distribution of the number of white balls.
 [Delhi 2012 (C)]
X 0 1 2 3 4 5 6 7 8
3. Out of a group of 30 honest people, 20 always speak truth.
P(X = x) a 3a 5a 7 a 9a 11a 13a 15a 17 a
Two persons are selected at random from the group. Find
Then the value of a is: the probability distribution of number of selected persons
who speak the truth. [Delhi 2013 (C), A.I. 2013 (C)]
1 2 5 7
(a) (b) (c) (d) 4. A class has 15 students whose ages are 14, 17, 15, 14, 21,
81 81 81 81
17, 19, 20, 16, 18, 20, 17, 16, 19 and 20 years. One student
2. The distribution of a random variable X is given below:
is selected in such a way that each has the same chance
X=x − 2 −1 0 1 2 3 of being chosen and the age x of the selected student is

1 1 3 recorded. What is the probability distribution of the random
P(X = x) k 2k k variable x?  [A.I. 2014 (C)]
10 5 10
The value of k is: 5. Three cards are drawn successively with replacement
from a well shuffled pack of 52 playing cards. Find the
1 2 3 7 probability distribution of the number of spades.
(a) (b) (c) (d)
10 10 10 10  [A.I. 2015]
3. A random variable X has the probability distribution:
6. A die is thrown again and again until three sixes are
X 1 2 3 4 5 6 7 8 obtained. Find the probability of obtaining the third six in
P( X ) 0.15 0.23 0.12 0.10 0.20 0.08 0.07 0.05 sixth throw of the die. [Delhi 2009]

7. A random variable x has the following probability
For the events, E = {X is prime number} and F = {X < 4}
distribution.
the probability of P(E ∪ F) is:
(a) 0.50 (b) 0.77 (c) 0.35 (d) 0.87 xi 0 1 2 3 4 5 6 7
II. Short Answer Type Questions-II pi 0 k 2k 2k 3k k 2
2k 2 2
7k + k
1. Find the probability distribution in three throws of a pair Determine:
of dice. [A.I. 2007 (C), Delhi 2006 (C), 2010 (C)] (i) k, (ii) P(x < 3); (iii) P(x > 6); (iv) P(0 < x < 3)
2. An urn contains 4 white and 6 red balls. Four balls are  [A.I. 2011]

Answers 13.6
Now P(E ∪ F) = P(E) + P(F) – P(E ∩ F)
I. 1. In a probability distribution of random variable X, ∑ p = 1 i
= 0.62 + 0.50 – 0.35 = 0.77
\ a + 3a + 5a + 7a + 9a + 11a + 13a + 15a + 17a = 1

Hence, (b) is the correct answer.
1
⇒ 81a = 1 ⇒ a =
xi 0 1 2 3
81
II. 1. p(xi ) 125 75 15 1
Hence, (a) is the correct answer.
216 216 216 216
2. In a probability distribution of random variable X, ∑p i =1
xi 0 1 2 3 4
1 1 3 2.

\ + k + + 2k + + k =1 pi 15 80 90 24 1
10 5 10 210 210 210 210 210
 1 1 3 1
⇒  + +  + k + 2k + k = 1 ⇒ k = xi 0 1 2
 10 5 10  10
3. pi 9 40 38
Hence, (a) is the correct answer.
87 87 87
3. Event E = {X is a prime number}
P(E) = P(X = 2) + P(X = 3) + P(X = 5) + P(X = 7) xi 14 15 16 17 18 19 20 21
4. pi 2 1 2 3 1 2 3 1
= 0.23 + 0.12 + 0.20 + 0.07 = 0.62
15 15 15 15 15 15 15 15
Event F = {X < 4}

\ P(F) = P(X = 1) + P(X = 2) + P(X = 3) xi 0 1 2 3
= 0.15 + 0.23 + 0.12 = 0.50 5. pi 27 27 9 1
P(E ∩ F) = P(X = 2) + P(X = 3) = 0.23 + 0.12 = 0.35 64 64 64 64

Probability 405
E:\AMIT_WORKS\Exam_Guru\EG_Mathematics-12_(working_02-06-2022)\EG_Mathematics-12_working\Open_Files\Chap_13\Chap_13
\ 18-Aug-2022 Amit Proof-5 Reader’s Sign _______________________ Date __________

625 1 5 xi 0 1 2 3 4 5 6 7
6. Hint: p = 6 , n = 5 and q = 6
23328 pi 0 1 2 2 3 1 2 17
10 10 10 10 100 100 100
P(To get third 6 in the sixth throw)
(i) k = 1 , (ii) p(x < 3) = 3 ,
1 10 10
= P(to get 2 sixes in first five throws) ×
6 17 3
(iii) p(x > 6) = , (iv) p(0 < x < 3) =
5
(
= C2 p q 2 3 1
6
. ) 100 10
1 + 37p = 2 + 26p
1
7. k =
10 ⇒  11p = 1 ⇒   p = 1
11

Case Based Questions


1. A coach is training 3 players. He observes that the player A (v) What is the probability that at least one of A, B or C
can hit a target 4 times in 5 shots, player B can hit 3 times will hit the target's?
in 4 shots and the player C can hit 2 times in 3 shots. 59 2
(a) (b)
From the situation, answer the following. 60 5
3 1
(c) (d)
5 60
2 1
Ans. (i) (c) (ii) (a)
5 10
13 1
(iii) (d) (iv) (b)
30 60
59
(v) (a)
60
2. In answering a question on a multiple choice test for class
3
XII, a student either knows the answer or guesses. Let
5
2
be the probability that he knows the answer and be the
5

(i) Let the target is hit by A, B: the target is hit by B and probability that he guesses. Assume that a student who
C: the target is hit by A and C. Then, the probability guesses at the answer will be correct with probability
that A, B and C all will hit, is 1
. Let E1, E2, E be the events that the student knows
3
4 3 2 1 the answer guesses the answer and answers correctly
(a) (b) (c) (d)
5 5 5 5 respectively.
(ii) Referring to (i), what is the probability that B, C will
hit and A will lose?
1 3 7 4
(a) (b) (c) (d)
10 10 10 10
(iii) With reference to the events mentioned in (i), what is
the probability that any two of A, B and C will hit?
1 11 17 13
(a) (b) (c) (d)
30 30 30 30
Based on the above information, answer the following.
(iv) What is the probability that 'none of them will hit the (i) What is the value of P(E1)?
targets? 2 1 3
1 1 (a) (b) (c) 1 (d)
(a) (b) 5 3 5
30 60 (ii) Value of P(E | E1) is
1 2 1 2
(c) (d) (a) (b) 1 (c) (d) 415
15 15 3 3

406 Mathematics–12
E:\AMIT_WORKS\Exam_Guru\EG_Mathematics-12_(working_02-06-2022)\EG_Mathematics-12_working\Open_Files\Chap_13\Chap_13
\ 19-Aug-2022 Amit Proof-5 Reader’s Sign _______________________ Date __________

k =2 (v) The probability that Iqbal processed the form and


(iii) ∑ P ( E|E k ) P ( E k )
k =1
equals committed an error is:
(a) 0.009 (b) 0.006 (c) 0.008 (d) 0.68
11 4 1 Ans. (i) (b) 0.04 (ii) (c) 0.008
(a) (b) (c) (d) 1
15 15 5 17
(iii) (b) 0.047 (iv) (c) (v) (a) 0.009
k =2 47
(iv) Value of ∑ P (Ek ) is
4. A shopkeeper sells three types of flower seeds A1, A2,
k =1
and A3. They are sold as a mixture where the proportions
1 1 3
(a) (b) (c) 1 (d) are 4:4:2 respectively. The germination rates of the three
3 5 5 types of seeds are 45%, 60% and 35% respectively.
(v) What is the probability that the student knows the
answer it correctly?
2 5 9 13
(a) (b) (c) (d)
11 3 11 3
3 11
Ans. ( ) (d) (ii) (b) 1 (iii) (a)
5 15
9
(iv) (c) 1 (v) (c)
11
3. In an office, three employees Vinay, Sonia and Iqbal
process incoming copies of a certain form. Vinay process
50% of the forms. Sonia processes 20% and Iqbal the
remaining 30% of the forms. Vinay has an error rate of
0.06, Sonia has an error rate of 0.04 and Iqbal has an error
rate of 0.03. Based on the above information answer the following
Based on the above information answer the following: questions:
(i) The probability of a randomly chosen seed to germinate:
(a) 0.69 (b) 0.39 (c) 0.49 (d) 0.59
(ii) The probability that the seed will not germinate given
that the seed is of type A3:
15 65 75 55
(a) (b) (c) (d)
100 100 100 100
(iii) The probability that the seed is of the type A2 given that
a randomly chosen seed does not germinate.
(i) The conditional probability that an error is committed 22 55 51 16
(a) (b) (c) (d)
in processing given that Sonia processed the form is: 51 51 16 51
(a) 0.0210 (b) 0.04 (c) 0.47 (d) 0.06 (iv) The probability that it is of the type A1 given that a
(ii) The probability that Sonia processed the form and randomly chosen seed does not germinate is:
committed an error is: 22 5 1 16
(a) (b) (c) (d)
(a) 0.005 (b) 0.006 (c) 0.008 (d) 0.68 51 51 16 51
(iii) The total probability of committing an error in (v) The probability that it will not germinate given that the
processing the form is seed is of type A1
55 65 35 45
(a) 0 (b) 0.047 (c) 0.234 (d) 1 (a) (b) (c) (d)
100 100 100 100
(iv) The manager of the company wants to do a quality
check. During the inspection, he selects a form at 65
Ans. (i) (c) 0.49 (ii) (b)
random from the days’ output of processed forms. If 100
the form selected at random has an error, the probability 16 22
(iii) (d) (iv) (a)
that the form is NOT processed by Vinay is: 51 51
30 17 30 55
(a) 1 (b) (c) (d) 37 (v) (a)
47 47 100

Probability 407
E:\AMIT_WORKS\Exam_Guru\EG_Mathematics-12_(working_02-06-2022)\EG_Mathematics-12_working\Open_Files\Chap_13\Chap_13
\ 18-Aug-2022 Amit Proof-5 Reader’s Sign _______________________ Date __________

Author’s Comments
Questions based on following types are very important for Exams. So, students are advised to revise them thoroughly.
1. Questions based on conditional probability.
2. Questions related to Bayes’ theorem.
3. Questions based on Independent Events.

IMPORTANT FORMULAE

n (E)
1. Probability of an event E = P ( E ) = where n(S) is total number of outcomes in the experiment and n(E) is the total
n (S)

number of favourable outcomes of event E.

P ( E ∩ F)
2. Conditional probability of event E when event F has already occurred =
P ( F)

3. A, B and F are three events of a sample space S such that P(S) ≠ 0; then
 ( A ∪ B) 
P
 F () ()
A B  ( A ∩ B) 
 = P F +P F −P F 
  

4. If E and F are two events of the same sample space S then P ( )


E′
F
= 1− P
E
F
. ()
B′( )
5. P A =
P ( A ∩ B′ ) P ( A ) − P ( A ∩ B)
P ( B′ )
=
1 − P ( B)

B′( )
6. P A′ =
P ( A′ ∩ B′ ) P ( A ∪ B)′ 1 − P ( A ∪ B)
P ( B′ )
=
P ( B′ )
=
1 − P ( B)

7. Multiplication theorem: P ( E ∩ F) = P ( F) ⋅ P ()
E
F
= P (E)⋅ P
F
E ()
8. Two events E and F are independent if P(E ∩ F) = P(E) . P(F)

9. Theorem of total probability: Let E1, E2 be the partition of a sample space S and A be any other event associated with sample
   
space S, then P ( A ) = P ( E1) ⋅ P  A  + P ( E 2 ) P  A  .
 E1   E2 
10. Bayes’ Theorem: Let E1, E2 and E3 be the partition of sample space S and A be any other event of the sample space S.

 
P ( E1) P  A 
Then P  E1  =  E1 
 A      
P ( E1) . P  A  +P ( E 2 ) P  A  + P ( E3 ) P  A 
 E1   E2   E3 

408 Mathematics–12
E:\AMIT_WORKS\Exam_Guru\EG_Mathematics-12_(working_02-06-2022)\EG_Mathematics-12_working\Open_Files\Chap_13\Chap_13
\ 18-Aug-2022 Amit Proof-5 Reader’s Sign _______________________ Date __________

COMMON ERRORS
Errors Corrections
(i) Students are unable to recognise whether to add two (i) (a) If two or more events are connected with ‘OR’; add
probability or multiply two probabilities them. (b) If two or more events are connected with ‘and’
multiply them.

() ()
(ii) In conditional probability, students are unable to work
A = P ( A ∩ B) B = P ( A ∩ B)
out P(A/B) or P(B/A). (ii) (a) P (b) P
B P ( B) A P (A)
(iii) Sometimes students are unable to understand that the (iii) If word with replacement is written, it is with
question based on replacement or without replacement. replacement. If nothing is mentioned or written without
replacement, it is a case of without replacement.
(iv) Mistakes in identifying different ‘Events’ in Bayes’ (iv) More practice of such questions
theorem
(v) Mistakes in identifying the probability of different (v) Drilling in such problems
events in Bayes’ theorem
(vi) P(A ∪ B) is taken as P(A) + P(B) always. (vi) P(A ∪ B) = P(A) + P(B) only for mutually
exclusive events
(vii) Bayes' Theorem: Students get confused by P(E|A) and E  P ( A|E ) P ( E )
P(A|Ei). Hence write the formula wrongly. Sometimes (vii) Bayes' Theorem: P  i  = i i
A ∑ P ( A|Ei ) P ( Ei )
they also consider events of P(E1|A1), P(E2|A2), etc.
where the symbol have their usual meaning.

REVISION CHART

Probability
If there are n elementary events associated with a random experiment and
Conditional Probability m of them are favourable to an event A, then the probability of happening or Bayes' Theorem
occurrence of A, denoted by P(A), is given by P(A) = m/n = Number of favourable
cases/total number of possible cases.

Independent Events

Random Variables & Its Probability Distribution


Probability Distribution of a Probability Distribution gives the values of the random variables along with the
random variables corresponding probabilities

Random Variable

Probability 409
E:\AMIT_WORKS\Exam_Guru\EG_Mathematics-12_(working_02-06-2022)\EG_Mathematics-12_working\Open_Files\SP_1 to 3\SP_1 to 3
\ 18-Aug-2022 Amit Proof-5 Reader’s Sign _______________________ Date __________

Sample Paper– 1
(Solved)
Time Allowed: 3 hours Maximum Marks: 80

General Instructions:
1. This question paper contains two parts A and B. Each part is compulsory. Part–A carries 24 marks and Part–B
carries 56 marks.
2. Part–A has Objective Type Questions and Part–B has Descriptive Type Questions.
3. Both Part A and Part B have choices.
Part-A:
1. It consists of two Sections–I and II.
2. Section–I comprises of 16 very short answer type questions.
3. Section–II contains 2 case studies. Each case study comprises of 5 case-based MCQs. An examinee is to attempt
any 4 out of 5 MCQs.
4. Internal choice is provided in 5 questions of Section I. You have to attempt only one of the alternatives in all
such questions.
Part-B:
1. It consists of three Sections–III, IV and V.
2. Section–III comprises of 10 questions of 2 marks each.
3. Section–IV comprises of 7 questions of 3 marks each.
4. Section–V comprises of 3 questions of 5 marks each.
5. Internal choice is provided in 3 questions of Section–III, 2 questions of Section–IV and 3 questions of Section–V.
You have to attempt only one of the alternatives in all such questions.

Part–A
Section–I
All questions are compulsory. In case of internal choices attempt any one.
Q1. The relation “less than” in the set of natural numbers is 1
(a) only symmetric (b) only transitive (c) only reflexive (d) equivalence relation
Sol. Let R be the relation in N defined by “less than” where N denotes the set of natural numbers.
The relation R is not reflexive, since a is not less than a for any natural number a.
The relation R is not symmetric, since if (a, b) ∈ R, then a is less than b but b is not less than a, i.e., (b, a) ∉ R
The relation R is transitive, since a < b and b < c ⇒ a < c.
Hence, (b) is the correct answer.
Q2. For real x, let f (x) = x3 + 5x + 1. Then 1
(a) f is one-one but not onto in R. (b) f is onto R but not one-one in R.
(c) f is one-one and onto in R. (d) f is neither one-one and onto in R.
Sol. For x1, x2 ∈ R,
f(x1) = f(x2)
⇒ x13 + 5x1 + 1 = x23 + 5x2 + 1
⇒ x13 – x23 + 5(x1 – x2) = 0
⇒ (x1 – x2)(x12 + x1x2 + x22 + 5) = 0 \ x1 = x2
Therefore, f (x) is one-one.

410
Again, as f (x) = x3 + 5x + 1 is a polynomial function so it is continuous in R.
f is continuous at every point of its domain R. It takes all values in (– ∞, ∞). Therefore, f (x) is onto. Hence, f (x)

is one-one and onto.

\ (c) is the correct answer.
Q3. Write the principal value of [tan–1 (– 3 ) + tan–1 (1)] 1
OR
 3π 
Find the principal value of tan–1  tan  .
 5

Sol. tan–1 (– 3 ) + tan–1 (1) = –tan–1 π π π π


3+ =− + =−
4 3 4 12
OR
 2π  −1  2π  −1  2π  2π
tan–1 tan  π − 5  = tan  − tan  = − tan  tan  = −
   5   5  5
Q4. If A is a matrix of order 3 × 2, then find the order of the matrix A′. 1
Sol. 2 × 3
Q5. If A is a square matrix of order 3 and |A| = 5, then find the value of |2A′|. 1
OR
If A is a square matrix such that A2 = A, then find (I – A)3 + A.
Sol. 40 OR I
Q6. A square matrix A is said to be skew-symmetric, if ___________. 1
Sol. A = –A′ (or, A′ = –A)
Q7. Find ∫ x log x dx .
4
1
OR
2x
Find ∫ 3
x2 + 1
dx .

x5 1 x5
Sol. ∫ x 4 log x dx = log x ⋅
5


⋅ dx
x 5
x 5 ⋅ log x x 5
= − +c
5 25
OR
2
Let x + 1 = t \ 2x dx = dt
2x 1 3 3
3 ∫ x +1
2
dx =
∫ 3
t ∫
dt = t −1/ 3 dt = t 2 / 3 + c = ( x 2 + 1) 2 / 3 + c
2 2
3
Q8. Evaluate
∫ | 2 x − 1 | dx .
1
1

3
3 3
1 
Sol. ∫ | 2 x − 1 | dx = ∫
1 1
(2 x − 1)dx =  (2 x − 1) 2  = 6
4 1
Q9. How many arbitrary constants in the particular solution of a differential equation of second order? 1
OR
Determine the order of the differential equation y′ + 5y = 0.
Sol. 0

Sample Paper–1 (Solved) 411


E:\AMIT_WORKS\Exam_Guru\EG_Mathematics-12_(working_02-06-2022)\EG_Mathematics-12_working\Open_Files\SP_1 to 3\SP_1 to 3
\ 18-Aug-2022 Amit Proof-5 Reader’s Sign _______________________ Date __________

OR
dy
The differential equation y′ + 5y = 0 can be written as + 5y = 0.
dx
As the order of the highest order derivative is 1, so the order of the given differential equation is 1.
 
Q10. If the projection of a = iˆ − 2 ˆj + 3kˆ on b = 2iˆ + λkˆ , is zero, then find the value of l. 1

  a ⋅ b
Sol. Projection of a on b = 
|b |

       =
( )(
i − 2 j + 3k ⋅ 2i + 0 j + λ k
)= 2 + 3λ
2 2 2
2 +0 +λ 4 + λ2
2 + 3λ
Given that = 0 ⇒ 2 + 3λ = 0 ⇒ λ = − 2
4+λ 2
3

Q11. Find the magnitude of the vector a = 2iˆ − 6 ˆj − 3kˆ . 1

Sol. We have a = 2iˆ − 6 ˆj − 3kˆ

| a |= (2) 2 + (−6) 2 + (−3) 2 = 4 + 36 + 9 = 49 = 7

     
Q12. If | a | = 3, | b | = 2 and a ⋅ b = 3 find the angle between a and b . 1
 
Sol. Let q be the angle between a and b , then
 
a ⋅b 3 3 π
cos θ =   = = = cos 30° ⇒ θ = 30° or
a b 3(2) 2 6

 
Hence, angle between a and b is 30° or p/6.
Q13. Find the d.c’s of a line whose direction ratios are 2, 3, – 6 1
Sol. a = 2, b = 3, c = –6
( 2) + ( 3) + ( − 6 ) = 4 + 9 + 36 = 7
2 2 2
a 2 + b2 + c2 =
2 3 −6
Hence, the d.c's of the given line are , , .
7 7 7
Q14. Find the vector equation of the line joining the points whose position vectors are 2iˆ − ˆj + kˆ and iˆ + 2 ˆj − 3kˆ  1
 
Sol. Let a = 2iˆ − ˆj + kˆ and b = iˆ + 2 ˆj − 3kˆ
  
b − a = (iˆ + 2 ˆj − 3kˆ) − (2iˆ − ˆj + kˆ) ⇒ b − a = − iˆ + 3 ˆj − 4kˆ

   
Vector equation of the line is r = a + λ (b − a )

⇒ r = 2iˆ − ˆj + kˆ + λ (− iˆ + 3 ˆj − 4kˆ).
1 1
Q15. If A and B are two independent events with P(A) = and P(B) = , then find P(B′ | A) 1
3 4
P ( B′  A ) P(B′)P(A)
Sol. P(B′/A) = = = P(B′) = 1 − P(B)
P(A) P(A)
1 3
    = 1 – =
4 4
3 3 1
Q16. Let E and F be events with P(E) = , P(F) = and P(E ∩ F) = . Are E and F independent? 1
5 10 5
412 Mathematics–12
3 3 9 1
Sol. P(E) . P(F) =
× = and P(E ∩ F) = , since P(E ∩ F) ≠ P(E) . P(F)
5 10 50 5

\ Events E and F are not independent.
Section-II
Both the Case study-based questions are compulsory. Attempt any 4 sub-parts from each question (17–18). Each
sub-part carries 1 mark.
Q17. A window is in the form of semi-circle with a rectangle on its diameter. The total perimeter of the window is 10 m.

D C

2y 2y

A 2x B
Based on the above information answer the following questions:
(i) From the figure, the perimeter (P) of the window is 1
(a) 2x + 4y + px (b) x + 4y + px
(c) 2x + y + px (d) 2x + 4y + x
(ii) Area(A) of whole window is given by 1
πx 2 πx 2
(a) xy + (b) 4xy + px2 (c) 4 xy + (d) xy + px2
2 2
(iii) Area(A) of window in terms of x is given by 1
πx 2
(a) x – 2x2 – (b) 10x – x2 – px2
2
πx 2 πx 2
(c) 10x – x2 –
(d) 10x – 2x2 –
2 2
(iv) For maximum / minimum value of area, values of x and y respectively should be 1
10 5 1 5 10 3 1 3
(a) , (b) , (c) , (d) ,
π+4 π+4 π+4 π+4 π+4 π+4 π+4 π+4
(v) Dimensions (length and breadth) of window is 1
2 10 20 10
(a) , (b) ,
π+4 π+4 π+4 π+4
20 1 2 4
(c) , (d) ,
π+4 π+4 π + 4 π + 4
Sol. (i) (a) (ii) (c) (iii) (d) (iv) (a) (v) (b)
Q18. In a hostel, 60% of the student read Hindi newspaper, 40% read English newspaper and 20% read both Hindi and
English newspapers.

Sample Paper–1 (Solved) 413


E:\AMIT_WORKS\Exam_Guru\EG_Mathematics-12_(working_02-06-2022)\EG_Mathematics-12_working\Open_Files\SP_1 to 3\SP_1 to 3
\ 18-Aug-2022 Amit Proof-5 Reader’s Sign _______________________ Date __________


Based on the above information answer the following:
(i) A student is selected at random, find the probability that a student reads both Hindi and English newspaper.1
1 2 4
(a) (b) (c) 3 (d)
5 5 5 5
(ii) A student is selected at random, find the probability that a student reads Hindi or English newspaper. 1
1 4 2
(a) (b) (c) 3 (d)
5 5 5 5
(iii) A student is selected at random. Find the probability that student neither reads Hindi or English newspaper.
 1
2 1 4
(a) (b) 3 (c) (d)
5 5 5 5
(iv) If student reads Hindi newspaper, find the probability that she reads English newspaper. 1
2 4 1
(a) (b) 3 (c) (d)
5 5 5 3
(v) If student reads English newspaper, find the probability that she reads Hindi newspaper. 1
1 2 1
(a) (b) (c) (d) 3
2 5 3 5
Sol. (i) (a) (ii) (b) (iii) (c) (iv) (d) (v) (a)

Part–B
Section–III
All questions are compulsory. In case of internal choices attempt any one.
1
Q19. Prove that sin–1 (2 x 1 − x 2 ) = 2 cos −1 x, ≤ x ≤ 1 . 2
2
Sol. Put x = cos q ⇔ q = cos–1 x
L.H.S. = sin–1 (2 x 1 − x 2 )
   = sin–1 (2 cos q sin q) = sin–1 (sin 2q) = 2q = 2 cos–1 x = R.H.S.
Q20. What positive value of x makes following pair of determinant equal? 2
2x 3 16 3
=
5 x 5 2
         
OR
6 −3 2
Find the value of D = 2 −1 2
−10 5 2

2x 3 16 3
Sol. We have =
5 x 5 2
On expanding, we get
2x2 – 15 = 32 – 15
⇒ 2x2 – 15 = 17
⇒ 2x2 = 32
⇒ x2 = 32 ÷ 2 = 16

414 Mathematics–12
\ x = ± 16
⇒ x = ±4
The positive value of x = 4
Hence, for x = 4, the given pair of determinant is equal.
OR
6 −3 2
D= 2 −1 2
−10 5 2
Expanding in terms of elements along the first row, we get
−1 2 2 2 2 −1
D= 6 – (– 3) +2
5 2 −10 2 −10 5
= 6 (– 2 – 10) + 3 (4 + 20) + 2 (10 – 10) = – 72 + 72 + 2 (0) = 0.
Q21. Examine the continuity of the function f (x) = 2x2 – 1 at x = 3. 2
Sol.      f (x) = 2x2 – 1
     lim x→3
f (x) = lim
x→3
(2x2 – 1)

= 2 lim
x→3
x2 – lim
x→3
1 = 2(3)2 – 1
        = 2 (9) – 1 = 18 – 1 = 17
Also    f (3) = 2(3)2 – 1 = 2(9) – 1 = 17
\    lim
x→3
f (x) = f (3)
Hence, f (x) = 2x2 – 1 is continuous at x = 3.
x4
Q22. Find the local maximum and local minimum, if any, of f ( x) = , x ≠ 1.  2
x −1
( x − 1) 4 x 3 − x 4 ⋅ 1
Sol. f ′ (x) = ( x − 1) 2
4 x 4 − 4 x 3 − x 4 x 3 (3 x − 4)
⇒ f ′ (x) = =
( x − 1) 2 ( x − 1) 2
 4
3x3  x − 
 3 4
For max. or min., f ′(x) = 0 ⇒ 2
= 0 ⇒ x = 0,
( x − 1) 3
3(−) (−)
When x is slightly < 0, we have f ′( x) = = + ve
(+)

When x is slightly > 0, we have f ′( x) = 3(+ ) (−) = − ve


(+)
\ As x increases through 0, f ′(x) changes sign from + ve to – ve, So x = 0 is a point of local maximum and local
max. value is f (0) = 0
4 3(+ ) (−)
When x is slightly < , f ′( x) = = − ve
3 (+)
4 3(+ ) (+ )
When x is slightly > , f ′( x) = = + ve
3 (+)
4
As increases through , f ′( x) changes sign from – ve to + ve
3

Sample Paper–1 (Solved) 415


E:\AMIT_WORKS\Exam_Guru\EG_Mathematics-12_(working_02-06-2022)\EG_Mathematics-12_working\Open_Files\SP_1 to 3\SP_1 to 3
\ 18-Aug-2022 Amit Proof-5 Reader’s Sign _______________________ Date __________

4
So x = is a point of local minimum and local minimum value is
3
4
4 256
4  
  3
f   =   =
81 = 256 × 3 = 256
3
  4 1 81 1 27
−1
3 3
a
sin 2 x x
Q23. Evaluate: ∫ (1 + cos x ) 2
dx OR Evaluate:

0
a − x2
2
dx  2

sin 2 x (1 − cos x ) dx =
2
1 − cos x
Sol.    I = ∫ (1 + cos x ) 2
dx =
∫ (1 + cos x ) 2 ∫ 1 + cos x
dx

2sin 2 x / 2 x  x  x
   =
∫ 2cos x / 2
2
2 ∫ 2  ∫
dx = tan 2 dx =  sec 2 − 1 dx = 2 tan − x + C
2

OR
a
x
Let I =

0
a − x2
2
dx

1
Put a2 – x2 = t, so that –2x dx = dt ⇒ xdx = – dt
2
When x = 0, t = a2 and when x = a, t = a2 – a2 = 0
0 0 0 1 1
1 dt 1 − 12 1 1
\   I = −

2 ∫t
a2
1
2
=−
2 2 ∫
t dt = −  2t 2  = − [ 0 − ( a 2 ) 2 ] = ( a 2 ) 2 = a
a
2  a2

Q24. Find the area of the region between the curves y2 = 4x and x = 3. 2
2
Sol. The equation of parabola is y = 4x. It contains only even powers of y.

Y
x=3

y=2 x B

X′ X
O A

B′

Y′

416 Mathematics–12
\ The parabola is symmetrical about x-axis.

In the first quadrant, y > 0

\ y= 2 x
Required area = Area OBB′ = 2 × Area OAB (shown shaded)
      = 2 × Area under y
       = 2 x between x = 0 and x = 3
3 3
       = 2 ∫ y dx = 2 ∫ 2 x dx
0 0
3
 3
3
 x2  8
       = 4   = × 3 2 = 8 3 sq. units
 3  3
 2 0
dy
Q25. Solve = 2x3 – x , given y = 1, when x = 0 2
dx
dy
Sol. We have = 2x3 – x
dx
Integrating both sides, we get
x4 x2 1 1
y = ∫ ( 2 x3 − x ) dx ⇒ y = 2 ⋅ − + C ⇒ y = x4 − x2 + C
     4 2 2 2
Now, y = 1, and x = 0
\   1 = 0 + C ⇒ C = 1
1
Hence, y = x2 (x2 – 1) + 1
2  
 
Q26. Find a unit vector perpendicular to each of the vectors a and b where a = 5iˆ + 6 ˆj − 2kˆ and b = 7iˆ + 6 ˆj + 2kˆ .2
iˆ ˆj kˆ
 
Sol. a × b = 5 6 −2 = 24iˆ − 24 ˆj − 12kˆ
7 6 2
  2 2 1
Unit vector perpendicular to both a and b is iˆ − ˆj − kˆ .
3 3 3
 ˆ ˆ    
ˆ ˆ ˆ ˆ
Q27. If a = i − j + 2k and b = 2i + j − k , find (2a − b) × (a + 2b ) . 2
 
Sol. 2a − b = 2(iˆ − ˆj + 2kˆ) − (2iˆ + ˆj − kˆ) = − 3 ˆj + 5kˆ
 
ˆ ˆ ˆ ˆ ˆ ˆ ˆ ˆ
= i − j + 2k + 2(2i + j − k ) = 5i + j
a + 2b

iˆ ˆj kˆ
   
∴ (2a − b ) × (a + 2b ) = 0 − 3 5 = iˆ (0 − 5) − ˆj (0 − 25) + kˆ(0 + 15)
5 1 0

         = − 5iˆ + 25 ˆj + 15kˆ = 5(− iˆ + 5 ˆj + 3kˆ).
Q28. Find the probability distribution of the number of heads in two tosses of a coin. 2
OR
The probability of finding a green signal on a busy crossing X is 30%. What is the probability of finding a green
signal on X on two consecutive days out of three?

Sample Paper–1 (Solved) 417


E:\AMIT_WORKS\Exam_Guru\EG_Mathematics-12_(working_02-06-2022)\EG_Mathematics-12_working\Open_Files\SP_1 to 3\SP_1 to 3
\ 18-Aug-2022 Amit Proof-5 Reader’s Sign _______________________ Date __________

Sol. Let X be a random variable which is the number of heads obtained in two independent tosses of a coin S = {HH,
HT, TH, TT}
Then, X(HH) = 2, X(HT) = 1, X(TH) = 1, X(TT) = 0
\ X can take values 0, 1, 2.
1
Now, P(X = 0) = P(No head) =
4
2 1
P(X = 1) = P(Exactly 1 head) = =
4 2
1
P(X = 2) = P(Exactly 2 heads) =
4
Hence, the probability distribution of the R.V is given by
Xi 0 1 2
1 1 1
P(Xi )
4 2 4
OR
30 3
Probability of green signal on crossing X = =
100 10
3 7
Probability of not a green signal on crossing X = 1 – =
10 10
Probability of a green signal on X on two consecutive days out of three
3 3 7 7 3 3 63
           = × × + × × =
10 10 10 10 10 10 500

Section-IV
All questions are compulsory. In case of internal choices attempt any one.
Q29. Let N be the set of natural numbers and R be the relation on N × N defined by (a, b) R (c, d) iff ad = bc for all a,
b, c, d ∈ N. Show that R is an equivalence relation. 3
Sol. (i) Reflexive: For any (a, b) ∈ N × N
       a ⋅ b = b ⋅ a \ (a, b) R (a, b) thus R is reflexive
(ii) Symmetric: For (a, b), (c, d) ∈ N × N
       (a, b) R (c, d) ⇒ a ⋅ d = b ⋅ c ⇒ c ⋅ b = d ⋅ a
       (c, d) R (a, b) \ R is symmetric
(iii) Transitive: For any (a, b), (c, d), (e, f ) ∈ N × N
(a, b) R (c, d) and (c, d) R (e, f )
⇒ a ⋅ d = b ⋅ c and c ⋅ f = d ⋅ e
⇒ a⋅d⋅c⋅f= b⋅c⋅d⋅e ⇒ a⋅f=b⋅e
\ (a, b) R (e, f ), \ R is transitive.
\ R is an equivalance relation.
dy
+ (cos x)x, then find
2
Q30. If y = e x cos x
. 3
dx
u = (cos x)x ⇒ y = e x
2
cos x
Sol. Let +u
dy du
= e x cos x (2x ⋅ cos x – x2 ⋅ sin x) +
2

\
dx dx

418 Mathematics–12
log u = log (cos x)x ⇒ log u = x log(cos x)
Differentiating w.r.t. ‘x’
1 du du
= log(cos x) – x tan x ⇒ = (cos x)x {log(cos x) – x tan x}
u dx dx
Therefore,
dy
= e x cos x (2x ⋅ cos x – x2 ⋅ sin x) + (cos x)x {log(cos x) – x tan x)
2

dx
Q31. Prove that f (x) = x|x| is differentiable for all real values of x. 3
OR
d2y π
If x = a (q – sin q), y = a (1 + cos q), find 2
at q = .
dx 2
Sol. f (x) = x|x|
 x 2 , if x ≥ 0

⇒ f (x) =  2
 − x , if x < 0
f ( 0 − h ) − f ( 0) − ( −h) − 0
2
Lf ′ (0) = lim = lim
h→ 0 −h h → 0 −h
−h2
= hlim = lim h = 0
→0 − h h→ 0
f ( 0 + h ) − f ( 0) h2 − 0
Rf ′ (0) = lim = lim = lim h = 0
h→ 0 h h→ 0 h h→ 0
\ Lf ′ (0) = Rf ′ (0)
Hence, f (x) is differentiable for all real values of x.
OR
dx
     x = a(q – sin q) ⇒ = a(1 – cos q)…(1)

dy
     y = a(1 + cos q) ⇒ = –a sin q…(2)

dy dy dx − a sin θ sin θ
Now,   = ÷ = =−
dx d θ d θ a (1 − cos θ) 1 − cos θ
θ θ
2sin cos
dy 2 2 = − cot θ

⇒    =−
dx θ 2
2sin 2
2
Differentiating again w.r.t. x, we get
d²y  θ 1 dθ 1 θ 1
= −  − cosec²  . = cosec² ×
dx ²  2  2 dx 2 2 a (1 − cos θ)

 dx dθ 1 
 d θ = a (1 − cos θ) ⇒ dx = a (1 − cos θ) 
 
θ
cosec²
d²y 2
=
dx ² 2a (1 − cos θ)

Sample Paper–1 (Solved) 419


E:\AMIT_WORKS\Exam_Guru\EG_Mathematics-12_(working_02-06-2022)\EG_Mathematics-12_working\Open_Files\SP_1 to 3\SP_1 to 3
\ 18-Aug-2022 Amit Proof-5 Reader’s Sign _______________________ Date __________

π
cosec² 2
 d²y 1 4 = 1 . ( 2) = 1
  π = . π  2 a (1 − 0) a
dx ² θ = 2 
2 a 1 − cos 
 2
Q32. Find the intervals on which the function f (x) = sin4 x + cos4 x, 0 < x < 2p is strictly increasing or decreasing. 3
Sol.      f (x) = sin4 x + cos4 x, 0 < x < 2p
\    f ′(x) = 4 sin3 x ⋅ cos x + 4 cos3 x(– sin x)

       = –4 sin x cos x (cos2 x – sin2 x)

⇒ f ′(x) = –2 sin 2x cos 2x = –sin 4x
     f ′(x) = 0 ⇒ – sin 4x = 0
π π

⇒ sin 4x = 0 ⇒ 4x = 0, p, 2p ⇒ x = 0, ,
4 2
π π
For 0 < x < , –sin 4x < 0 ⇒ f ′(x) < 0, for 0 < x <
4 4

 π
Hence, f is strictly decreasing in the interval  0,  .
 4
π π
Again sin 4x < 0 for p < 4x < 2p i.e., for <x<
4 2
π π

⇒ –sin 4x > 0 for <x<
4 2
 π π
Hence, f is strictly increasing in the interval  ,  .
 4 2
Q33. Find the general solution of the differential equation ye ydx = (y3 + 2xe y)dy. 3
dy
Sol.      ye ydx = (y3 + 2xe y)dy ⇒ y ⋅ ey = y3 + 2xey
dx
dx 2

\   − x = y 2 ⋅ e− y
dy y
1 1
−2 ∫ dy log 1
y −2 log y y2
I.F. (Integrating factor) = e =e =e =
y2

\  Solution is
1 1
x⋅ 2
= ∫ y 2 ⋅ e − y ⋅ 2 dy + c = ∫ e − y dy + c
        y y

x

⇒        = –e–y + c or x = –y­2e–y + cy2
y2

Q34. Find the area included between the two curves y2 = 9x and x2 = 9y. 3
OR
Using integration, find the area of the region bounded by the triangle whose vertices are (2, –2), (4, 5) and (6, 2).
Sol. As shown in the figure, we have to find the area OAPBO.
Solving the given two equations simultaneously, we have

420 Mathematics–12
       x4 = 81y2 = 81(9x) = 729 x
⇒   x4 – 729x = 0 ⇒ x(x3 – 729) = 0

⇒       x = 0, x3 = 729 = (9)3 ⇒ x = 9

\       x = 0 at O and x = 9 at P

Y
x 2 = 9y
y 2 = 9x
A P(9,9)

B
X′ X
O M
(9,0)

Y′

Now, Area OAPBO = Area OAPMO – Area OBPMO


9 9 9 9 x2
= ∫ y1dx − ∫ y2 dx = ∫ 3 x1/2 dx − ∫ dx
         0 0 0 0 9
9 9
 x3/2  1  x3  1
= 3  −   = 2[(9)3/2 ] − [(9)3 ]
3 9  3 0 27
 
          2 0

1
        = 2(32)3/2 – × 729 = 2 × 27 – 27 = 54 – 27 = 27 sq. units
27
OR
Let A(2, –2) ; B(4, 5) ; C(6, 2)

6
B(4, 5)
4

2 C(6, 2)

X′ X
0 2 4 6 8

–2 A(2, –2)
Y′

Sample Paper–1 (Solved) 421


E:\AMIT_WORKS\Exam_Guru\EG_Mathematics-12_(working_02-06-2022)\EG_Mathematics-12_working\Open_Files\SP_1 to 3\SP_1 to 3
\ 18-Aug-2022 Amit Proof-5 Reader’s Sign _______________________ Date __________

Equations of the lines


2
         AB : x = (y + 9)
7
2
         BC : x = – (y – 11)
3
         AC : x = y + 4
2  −2  5 5 2
        ar(DABC) = ∫−2 ( y + 4)dy +  3
 ∫2 ( y − 11)dy − ∫−2 ( y + 9)dy
7
1 1 1
= [( y + 4) 2 ]2−2 − [( y − 11) 2 ]52 − [( y + 9) 2 ]5−2
            2 3 7
            = 16 + 15 – 21 = 10
Q35. Find the particular solution of the differential equation: 3
dy  y π
       x = y − x tan   , given that y = at x = 1.
dx  x  4
Sol. The differential equation can be written as:
dy y y dy dv
      = − tan , let y = vx  \ =v+ x
dx x x dx dx
dv 1
⇒    v + x
= v – tan v ⇒ cot v dv = – dx
dx x
Integrate both sides
y c
log sin v = –log |x| + log c ⇒ log sin = log
x x
y π
⇒    x ⋅ sin
= c , Put y = and x = 1
x 4
π 1

⇒    sin = c or c=
4 2

y 1
\   Particular solution is x . sin   =

 x 2
Section-V
All questions are compulsory. In case of internal choices attempt any one.
 1 2 −3
Q36. If A =  3 2 −2 , then find A–1 and use it to solve the following system of the equations:  5
 
 2 −1 1
x + 2y – 3z = 6
3x + 2y – 2z = 3
2x – y + z = 2
OR
Using properties of determinants, prove that

(b + c) 2 a 2 bc
     (c + a ) 2 b 2 ca = (a – b)(b – c)(c – a)(a + b + c)(a2 + b2 + c2)
(a + b) 2 c 2 ab

422 Mathematics–12
 0 1 2  0 1 2
1 1

Sol. |A| = 7; adj (A) =  −7
 –1
7 −7  ; A = adj A =  −7 7 −7 
|A| 7
 −7 5 −4   −7 5 −4 
The system of equations in Matrix form can be written as:
 x 6
    A ⋅ X = B, where X =  y  ; B =  3
   
z  2

 x  0 1 2  6   7   1
 1 1
      X = A B ⇒ y = −7 7 −7 3 = −35 =  −5

–1   
  7   7    
z  −7 5 −4  2  −35  −5
\     x = 1, y = –5, z = –5
OR
(b + c) 2 a 2 bc b2 + c2 a2 bc
  (c + a ) 2 b 2 ca = c2 + a2 b2 ca   (C1 → C1 – 2C3)
(a + b) 2 c 2 ab a 2 + b2 c2 ab

a 2 + b 2 + c 2 a 2 bc
         = a 2 + b 2 + c 2 b 2 ca   (C1 → C1 + C2)
a 2 + b 2 + c 2 c 2 ab

a 2 + b2 + c2 a2 bc
2 2
         = 0 b − a ca − bc (R2 → R2 – R1, R3 → R3 – R1)
0 c 2 − a 2 ab − bc

a 2 + b 2 + c 2 a 2 bc
= (b − a )(c − a ) 0 b+a −c
0 c + a −b
        
Expand along C1
         = (a2 + b2 + c2)(b – a)(c – a)(–b2 – ab + c2 + ac)
         = (a – b)(b – c)(c – a)(a + b + c)(a2 + b2 + c2)
Q37. Find the number of distinct real values of l, for which the vectors −λ 2iˆ + ˆj + kˆ, iˆ − λ 2 ˆj + kˆ and iˆ + ˆj − λ 2 kˆ are
coplanar. 5

OR
          
If a , b , c are three non-zero vectors such that a . (b × c ) = 0 and a and c are not parallel prove that a = λb + µc ,
where l and µ are scalars.

Sol. The given vectors −λ 2iˆ + ˆj + kˆ, iˆ − λ 2 ˆj + kˆ and iˆ + ˆj − λ 2 kˆ are coplanar if

−λ 2 1 1
1 −λ 2 1 =0
1 1 −λ 2

Sample Paper–1 (Solved) 423


E:\AMIT_WORKS\Exam_Guru\EG_Mathematics-12_(working_02-06-2022)\EG_Mathematics-12_working\Open_Files\SP_1 to 3\SP_1 to 3
\ 18-Aug-2022 Amit Proof-5 Reader’s Sign _______________________ Date __________

⇒ –l2 (l4 – 1) – 1 (–l2 –1) + 1 (1 + l2) = 0


⇒ –l6 + l2 + l2 + 1 + 1 + l2 = 0
⇒ –l6 + 3l2 + 2 = 0
⇒ l6 – 3l2 – 2 = 0
⇒ (1 + l2)2 (l2 – 2) = 0
⇒ l2 – 2 = 0
⇒ l= ± 2

OR
         

( )
a⋅ b ×c =0⇒a ⊥ b ×c ( ) ∵ a ≠ 0 and b × c ≠ 0 
 
  
    = But b × c is a vector ^ to the plane of b and c . 
     
∴ a ⊥ b × c ⇒ a lies in the plane of b and c .
 
∴ a can be expressed as a linear combination of b and c.
  
⇒ There exist l and µ such that a = λb + µ c

Q38. Solve the following linear programming problem (L.P.P.) graphically. 5


Maximize Z = 5x + 3y such that
3x + 5y ≤ 15
5x + 2y ≤ 10
x, y ≥ 0
OR
The corner points of the feasible region determined by the system of linear constraints are as shown below:
Y
B(3,4)
4

2 C(0,2)

1
A(7,0)
O X
1 2 3 4 5 6 7
(0,0)

Answer each of the following:


(i) Let Z = 5x + 7y be the objective function. Find the maximum and minimum value of Z and also the
corresponding points at which the maximum and minimum value occurs.
(ii) Let Z = px + qy, where p, q > 0 be the objective function. Find the condition on p and q so that the maximum
value of Z occurs at A(7, 0) and B(3, 4). Also mention the number of optimal solutions in this case.
Sol. Equations corresponding to the given constraints are
3x + 5y = 15
5x + 2y = 10
x = 0, y = 0
424 Mathematics–12
Drawing the bounding lines corresponding to the given inequalities and considering their common solution space,
we find the feasible region is given by the shaded area OABC.

Y
8

6
(0,5) 5x + 2y = 10
4
C(0,3) B(20/19,45/19)
2
3x + 5y = 15
A (5,0)
(2,0) X
O 2 4 6 8
(0,0)

Now, solving the equations 3x + 5y = 15 and 5x + 2y = 10 simultaneously, we get the co‑ordinates of B as


 20 15 
 ,  .
19 19
 20 15 
The vertices of the convex polygon OABC are O(0, 0), A(2, 0), B  ,  and C(0, 3).
 19 19 

Vertex of the feasible region Corresponding value Z = 5x + 3y

At O (0, 0) Z = 5(0) + 3(0) = 0

At A (2, 0) Z = 5(2) + 3(0) = 10

 20 45   20   45  235
At B  ,  Z = 5  + 3  =
 19 19   19   19  19

At C(0, 3) Z = 5(0) + 3(3) = 9

235 7  20 45 
Hence, the maximum value of Z = 5x + 3y is i.e., 12 and is attained at the vertex B  ,  .
19 19  19 19 
OR
(i) Feasible region is bounded.
Maximum of Z must occur at the corner point of the feasible region.
We have Z = 5x + 7y
Corner points of the feasible region are
(0, 0), (0, 2), (3, 4) and (7, 0)
At (0, 0) Z = 5(0) + 7(0) = 0
At (0, 2) Z = 5(0) + 7(2) = 14

Sample Paper–1 (Solved) 425


E:\AMIT_WORKS\Exam_Guru\EG_Mathematics-12_(working_02-06-2022)\EG_Mathematics-12_working\Open_Files\SP_1 to 3\SP_1 to 3
\ 18-Aug-2022 Amit Proof-5 Reader’s Sign _______________________ Date __________

At (3, 4) Z = 5(3) + 7(4) = 43


At (7, 0) Z = 5(7) + 7(0) = 35
We see that maximum value of Z is 43 and minimum value of Z is 0.
(ii) Since maximum value of Z occurs at A(7, 0) and B(3, 4)

\  7p + 0 = 3p + 4q

⇒  7p = 3p + 4q

⇒  7p – 3p = 4q

⇒  4p = 4q

⇒  p =q
Number of optimal solutions are infinite.
qqq

426 Mathematics–12
Sample Paper– 2
(Unsolved)
Time Allowed: 3 hours Maximum Marks: 80

General Instructions: Same as Sample Paper-1

Part–A
Section – I

All questions are compulsory. In case of internal choices attempt any one.
Q1. Let A = {1, 2, 3}. Then number of equivalence relation containing (1, 2) is 1
(a) 1 (b) 2 (c) 3 (d) 4
Q2. A function f from the set of natural numbers to integers 1
 n −1
 , when n is odd
f(n) =  2 is
 − n , when n is even
 2
(a) one-one but not onto (b) onto but not one-one (c) one-one and onto both (d) neither one-one nor onto 1
Q3. A relation R is defined in set A = {1, 3, 5, 7, 9} such that: R = {(a, b) : a + b ≤ 10}, a ∈ A, b ∈ A}. Write relation
R in roster form. 1
OR
Take a set A = {2, 3, 5} and relation R is defined in set A as: R = {(a, b) : a ≥ b and a, b ∈ A}. Is the relation R
reflexive? Give reason.
Q4. Are the following matrices equal? 1

 1 7 3 1 7 7 
A=  , B=  
 2 4 0 2 × 3  2 4 – 1 2×3

 4 x + 2
Q5. If A =  2 x − 3 x + 1  is symmetric, then find the value of x. 1
 
OR
Write a 2 × 2 matrix which is both symmetric and skew symmetric.

1 3 –2
Q6. If Δ = 4 – 5 6 , write the co-factor of a32 (the element of third row and 2nd column). 1
3 5 2

∫ ( x + 2) dx 
3
Q7. Evaluate: 1
OR
2
Evaluate: ∫−1
x dx

427
E:\AMIT_WORKS\Exam_Guru\EG_Mathematics-12_(working_02-06-2022)\EG_Mathematics-12_working\Open_Files\SP_1 to 3\SP_1 to 3
\ 18-Aug-2022 Amit Proof-5 Reader’s Sign _______________________ Date __________

Q8. Find the area of the region bounded by y 2 = 9x, x = 2, x = 4 and the x-axis in the first quadrant. 1
dy x
Q9. Solve: dx = 2  1
x +1
OR
 d  dy 
3

Write the sum of the order and degree of the D.E.  =0  


 dx  dx 
→ →
  
Q10. For what value of λ the vectors a = 2i + λ j + k and b = i − 2 j + 2k are perpendicular to each other? 1
→   
Q11. Write the magnitude of position vector | p | = xi + y j + zk . 1
→ →
Q12. Find the work done in moving an object along a vector d = 8i + 2 j − 5k if the applied force is F = 7i − k . 1
x + 3 3 − 2 y 5z + 4
Q13. Change the equations of the line in vector form: = = . 1
2 3 1
→ ∧ ∧ ∧
Q14. Change into Cartesian form of equations of line: r = (3λ + 2) i + (5 − 2λ) j + (1 − 4λ) k . 1
3 3 1
Q15. Let E and F be events with P(E) = , P(F) = and P(E ∩ F) = . Are E and F independent? 1
5 10 5
Q16. Two balls are drawn at random with replacement from a box containing 10 black and 8 red balls. Find the probability
that both balls are red.  1
Section-II
Both the Case study-based questions are compulsory. Attempt any 4 sub-parts from each question (17–18). Each
sub-part carries 1 mark.
Q17. An open tank with a square base and vertical sides is to be constructed from a metal sheet so as to hold a given
quantity of water.

Based on the above information, answer the following:


(i) The volume of tank (V) is 1
2
(a) x y (b) xy
(c) x + y (d) xy2
(ii) Total Surface Area (S) of tank in terms of x is 1
4V
(a) x2 + 4V (b) x2 +
x
4V 4V
(c) x + (d) x2 –
x x

428 Mathematics–12
dS
(iii) For maximum or minimum value of surface area S, is 1
dx
4V 4V
(a) 2x + (b) x – 2
x2 x
4V 4V
(c) x + (d) 2x –
x2 x2
dS
(iv) From = 0, relation between length (x) and volume (V) is 1
dx
(a) x2 = 2V (b) x3 = 2V (c) x = 2V (d) x = 2V2
(v) For minimum surface area (S), relation between length (x) and height (y) is 1
(a) x = 2 + y (b) x = 2 – y
(c) x = y (d) x = 2y
Q18. A laboratory blood test is 99% effective in detecting a certain disease when it is in fact, present. However, the
test also yields a false positive result of 0.5% of the healthy person tested (i.e., if a healthy person is tested, then
with probability 0.005, the test will imply he has the disease).


Based on the above information answer the following:
(i) Probability that a person has a disease is 1
(a) 0.001 (b) 0.01 (c) 0.1 (d) 0.0001
(ii) Probability that a person does not have the disease is 1
(a) 0.09 (b) 0.999 (c) 0.092 (d) 0.091
(iii) Probability that a person having a disease and blood test positive is 1
(a) 0.91 (b) 9.9 (c) 0.99 (d) 9.2
(iv) Probability that a person not having a disease but has positive blood test is 1
(a) 0.004 (b) 0.002 (c) 0.003 (d) 0.005
(v) Find the probability that a person has the disease given that his test result is positive.  1
129 118 198 108
(a) (b) (c) (d)
1179 1190 1197 1107

Sample Paper–2 (Unsolved) 429


E:\AMIT_WORKS\Exam_Guru\EG_Mathematics-12_(working_02-06-2022)\EG_Mathematics-12_working\Open_Files\SP_1 to 3\SP_1 to 3
\ 18-Aug-2022 Amit Proof-5 Reader’s Sign _______________________ Date __________

Part–B

Section–III
All questions are compulsory. In case of internal choices attempt any one.

  1  1 
Q19. Evaluate cos cos −1  −  + sin −1  −    2
  7   7 
 1 −1 5
Q20. Show that the matrix A = −1 2 1 is a symmetric matrix.
 2
 5 1 3
 
OR
6 –3 2
Find the value of Δ = 2 –1 2
–10 5 2
x 2 – 25
Q21. Examine the continuity of the function f (x) =  2
x+5

Q22. Find the intervals in which the function: f ( x ) = sin x + π on [ 0, π ] ( ) 4


(a) strictly increases (S↑) (b) strictly decreases (S↓) 2
dx
Q23. Evaluate: ∫ (2 − x) ( x 2 + 3)
 2

OR
π 1 + cos x

Evaluate: ∫π /2 1 − cos x
dx

Q24. Find the area of the region {(x, y) : x2 ≤ y ≤ x}. 2


dy
Q25. Solve = 2x3 – x, given y = 1, when x = 0. 2
dx
→ → → →
Q26. Find the magnitude of the vector a × b if a = 2iˆ + kˆ, b = iˆ + ˆj + kˆ  2

Q27. Find the equation of the line passing through the point i + j − 3k and perpendicular to the lines r = i + λ(2
i + j − 3k ) and →
r = (2 i + j + k ) + μ( i + j + k ). 2
Q28. Given P(A) = 0.5, P(B) = 0.4, P(A ∩ B) = 0.3, then, find P(A′|B′). 2
OR
1 7 1
Events A and B are such that P(A) = , P(B) = and P(not A or not B) =
2 12 4
State whether A and B are independent or not.
Section-IV

All questions are compulsory. In case of internal choices attempt any one.
Q29. Let A = {1, 2, 3} and define a relation R on A as follows:
R = {(1, 1), (2, 2), (3, 3)}
Prove that R is an equivalence relation. 3

430 Mathematics–12
dy y dy y
Q30. If y = x sin y, prove that x = 1 – x cos y or dx = x (1 – x cos y) . 3
dx
Q31. Is f (x) = | x – 1 | + | x – 2 | differentiable at x = 2? 3
OR
d2 y
If x = cos θ + θ sin θ, y = sin θ – θ cos θ, then find
dx 2
Q32. Find the intervals on which the following function is (i) increasing (ii) decreasing : f (x) = x3 – 12x2 + 36x + 17. 3
8
Q33. Evaluate: ∫ ( x + 2)( x 2
+ 4)
dx . 3

Q34. Find the area of the region bounded by x2 = 4y, y = 2, y = 4 and the y-axis in the first quadrant. 3
OR
x2 + y =
2
Find the area of the ellipse 2 2 1.
a b
Q35. Find the particular solution of the differential equation 3
dy π
– 3y cot x = sin 2x; y = 2 when x =
dx 2
Section-V

All questions are compulsory. In case of internal choices attempt any one.
Q36. Using matrices, solve the following system of equations:
x + 2y – 3z = 6; 3x + 2y – 2z = 3; 2x – y + z = 2
5
OR
1 −2 0
If A =  2 1 3 , find A–1.
 0 −2 1
 
Using A–1, solve the system of linear equations :
x – 2y = 10
2x + y + 3z = 8
– 2y + z = 7.
2x −1 3y + 2 2 − z
Q37. Find the equation of a line passing through a point A(1, 2, 3) and parallel to the line = = . 5
1 2 1
OR
x −1 y −1 z − 2
Find a point on the line: = = at a distance of 3 units from a point (3, 2, 2).
1 2 −1

Q38. Solve the following linear programming problem (L.P.P.) graphically. 5



Maximize Z = 5x + 3y subject to constraints;
3x + 2y ≤ 20
2x + y ≤ 12
x, y ≥ 0

Sample Paper–2 (Unsolved) 431


E:\AMIT_WORKS\Exam_Guru\EG_Mathematics-12_(working_02-06-2022)\EG_Mathematics-12_working\Open_Files\SP_1 to 3\SP_1 to 3
\ 18-Aug-2022 Amit Proof-5 Reader’s Sign _______________________ Date __________

OR
The corner points of the feasible region determined by the system of linear constraints are as shown below:

Let F = 4x + 6y be the objective function. Find the minimum value of F and also the corresponding point at which
minimum value occurs.

Answers

1. (b) 2  2. (c) one-one and onto both


3. R = {(1, 1), (1, 3), (1, 5), (1, 7), (1, 9), (3, 1), (3, 3), (3, 5), (3, 7), (5, 1), (5, 3), (5, 5), (7, 1), (7, 3), (9, 1)}.
OR
R is a reflexive relation because (2, 2), (3, 3) and (5, 5) ∈ R.
4. No, A ≠ B
5. x = 5 OR A null matrix of order 2 × 2. 6. – 14
1 3
7. (x + 2)4 + C OR 8. (16 – 4 2 ) sq. units
4 2
1 5 →
9. y = log | x2 + 1 | + C OR 3 10. l = 11. | p | = x2 + y2 + z2
2 2

→  3 ∧ 4 ∧  ∧ ∧ ∧
12. 61 units 13. r =  − 3 i + j − k  + λ  20 i − 15 j + 2 k 
 2 5   
x−2 y − 5 z −1
14. = −2 = −4
3
16
15. No, E and F are not independent. 16.
81
17. (i)–(a), (ii)–(b), (iii)–(d), (iv)–(b), (v)–(d)
18. (i)–(a), (ii)–(b), (iii)–(c), (iv)–(d), (v)–(c) 19. 0 20. OR 0

432 Mathematics–12
21. f (x) is continuous at each real number other than – 5.

{ π
} { ( )} π
22. f (x) is S↑ in the interval: 0 < x < 4 or x ∈ 0, 4

f (x) is S↓ in the interval: {


4 } { ( )}
π < x < π or x ∈ π , π
4

1 1 2  x  π
23. − log 2 − x + log x 2 + 3 + tan −1   + C OR 2 –
7 14 7 3  3 2
1 1
24. sq. units 25. y = x2 (x2 – 1) + 1 26. 6
6 2

27. r = i + j − 3k + λ ( 4i − 5 j + k )
2
28. OR A and B are not independent events
3
sec3 θ
31. No, f (x) is not differentiable at x = 2. OR
θ
32. f (x) is increasing for x < 2, x > 6 and decreasing for 2 < x < 6
x+2  32 − 8 2  πa 2
33. log + tan –1 x + C 34.   OR
x2 + 4 2  3  4
35. y = – 2 sin2 x + 4 sin3 x
36. x = 1, y = – 5 and z = – 5 OR x = 4, y = – 3 and z = 1
x −1 y −2 z −3
37. = =
3 4 −6
OR

(
(2, 3, 1) and 4 , 5 , 5
3 3 3 )
38. Maximum value of Z is 32 at (4, 4) OR
Minimum value of Z is 12 and occurs at any point on line segment.
qqq

Sample Paper–2 (Unsolved) 433


E:\AMIT_WORKS\Exam_Guru\EG_Mathematics-12_(working_02-06-2022)\EG_Mathematics-12_working\Open_Files\SP_1 to 3\SP_1 to 3
\ 18-Aug-2022 Amit Proof-5 Reader’s Sign _______________________ Date __________

Sample Paper– 3
(Unsolved)
Time Allowed: 3 hours Maximum Marks: 80
General Instructions: Same as Sample Paper-1

Part–A
Section – I
All questions are compulsory. In case of internal choices attempt any one.
Q1. Let R = {(3, 3), (6, 6), (9, 9), (12, 12), (6, 12), (3, 9), (3, 6)} 1
be a relation on the set A = {3, 6, 9, 12}.
The relation is
(a) reflexive only (b) reflexive and transitive only
(c) reflexive and symmetric only (d) an equivalence relation.
Q2. Let f : R → R be defined as f (x) = 3x. 1
(a) f is one-one onto (b) f is many-one onto
(c) f is one-one but not onto (d) f is neither one-one nor onto.
Q3. A relation R is defined from set A = {1, 3, 4, 5} to set B = {2, 4, 6} as: R = {(a, b) : b is divisible by a}, a ∈ A
and b ∈ B. Write the relation R in roster form. 1
OR
If set A = {1, 2, 3, 4} and set B = {5, 6, 7, 8} and a relation R from set A to set B is defined as: R = {(a, b) : a +
b = 5, a ∈ A, b ∈ B}. Is the relation R an empty relation? Give reason.
Q4. If a matrix has 18 elements, what are the possible orders it can have? What, if it has 5 elements? 1
2 4 6
Q5. If A =  3 5 7  , show that (A′)′ = A. 1

OR
1 2 3
If A =  4
 5 6  and k is a scalar, show that (kA)′ = kA′.
1 2 3
Q6. Write the co-factors of elements of the second row of the determinant:  −4 3 6  . 1
2 −7 9 

Q7. Find ∫ sin 3x dx 1
OR
b
Evaluate : 1
x
dx ∫
a
Q8. Find the area of the region bounded by y2 = 9x, x = 2, x = 4 and the x-axis in the first quadrant.  1
d2y dy
Q9. Determine the order and degree of equation: 3 2
−5 + 2y = 0  1
dx dx
OR
Show that y = x2 + 2x + C is a solution of differential equation y′ – 2x – 2 = 0.
   ∧ ∧ ∧  ∧ ∧ ∧
Q10. Find a ⋅ b if a = 3 i − j + 2 k and b = 2 i + 3 j + 3 k  1
 ∧ ∧ ∧
Q11. Write the scalar and vector components of position vector: r = 4 i − 5 j + 7 k  1

434
 1 ∧ 1 ∧ 1 ∧
Q12. Compute the magnitude of vector: c = i+ j+ k 1
3 3 3
→ ∧ ∧ ∧
Q13. From the equation of a line r = (3λ + 2) i + (λ − 3) j + (5λ − 2) k , find fixed point and direction ratios of the line. 1
→ ∧ ∧ ∧
Q14. Change into Cartesian form of equations of line: r = (3λ + 2) i + (5 − 2λ ) j + (1 − 4λ ) k . 1
3 1
Q15. If P(A) = and P(B) = , find P(A ∩ B) if A and B are independent events. 1
5 5
Q16. Compute P(A|B) if P(B) = 0.5 and P(A ∩ B) = 0.32 1
Section-II
Both the Case study-based questions are compulsory. Attempt any 4 sub-parts from each question (17–18). Each
sub-part carries 1 mark.
Q17. Rahul buy some pepsi cans. All cans are of closed right cylindrical cans of volume 128 p cm3.
r

r
   
Based on the above information answer the following:
(i) Height of cylindrical can is 1
128 182 812 821
(a) 2 (b) 2 (c) 2 (d) 2
r r r r
(ii) Total surface area (S) of cylinder in terms of r is 1
265 256π 256π 256π
(a) π + 2πr 2 (b) + πr 2 (c) + 2πr 2 (d) + πr 2
r r r r
dS
(iii) For maximum or minimum surface area, = 0 which is equal to 1
dr
64π 46π 64 46
(a) πr − 2 = 0 (b) πr − 2 = 0 (c) π − 2 = 0 (d) π − 2 = 0
r r r r
(iv) For minimum surface area of cylindrical can, value of r is 1
(a) 3 cm (b) 1 cm (c) 4 cm (d) 2 cm
(v) Surface area of cylindrical can is minimum at h = .................. cm 1
(a) 2 (b) 4 (c) 6 (d) 8
Q18. A class consists of 100 students. 25 of them are girls and 75 boys. 20 of them are rich and remaining poor, 40 of
them are fair complexioned.

Based on the above information answer the following:


Sample Paper–3 (Unsolved) 435
E:\AMIT_WORKS\Exam_Guru\EG_Mathematics-12_(working_02-06-2022)\EG_Mathematics-12_working\Open_Files\SP_1 to 3\SP_1 to 3
\ 18-Aug-2022 Amit Proof-5 Reader’s Sign _______________________ Date __________

(i) Probability of selecting a fair complexioned student is 1


2 1 3 4
(a) (b) (c) (d)
5 5 5 5
(ii) Probability of selecting a rich student is 1
2 1 3 4
(a) (b) (c) (d)
5 5 5 5
(iii) Probability of selecting a girl 1
2 3 1 3
(a) (b) (c) (d)
5 4 4 5
(iv) Find the probability of selecting a fair complexioned rich girl. 1
(a) 0.1 (b) 0.21 (c) 0.12 (d) 0.02
(v) Find the probability of selecting a fair complexioned rich boy. 1
(a) 0.06 (b) 0.14 (c) 0.16 (d) 0.02

Part–B
Section–III
All questions are compulsory. In case of internal choices attempt any one.
a b π
Q19. If tan–1   + tan −1   = , find the value of x. 2
 x  x 2
Q20. Using determinants, find the area of the triangle with vertices (– 3, 5), (3, – 6), (7, 2). 2
OR
For what value of k, the points (1, 5) (k, 1) and (11, 7) are collinear?
Q21. Discuss the continuity at the point ‘a’ of the function 2
 x3
 2
, 0< x<a
 a
f (x) =  a , x=a
 a3
 2a − 2 , x>a
 x
Q22. If f (x) = (x – 4)3 (x + 1)2 find a point of local maxima, point of local minima and the point of inflexion.  2
x
( x − 3)e
Q23. Evaluate: ∫ ( x − 1)3
dx  2
OR
π/ 2

Evaluate :  x x 

2
 sin cos + x − 1 dx
0
2 2
Q24. Find the area of the parabola y2 = 4ax bounded by its latus rectum. 2
 dy 
Q25. Solve the differential equation sin–1   = x + y. 2
 dx 
Q26. Prove by vector method that in a right angled triangle, the square of the hypotenuse is equal to the sum of the
squares of the other two sides. 2
x +1 y + 3 z + 5 y−4 z−6
Q27. Show that the lines: = = and x − 2 = = intersect. Also find the point of intersection. 2
3 5 7 1 3 5

1 1 1
Q28. A problem in mathematics is given to 3 students whose chances of solving it are , , What is the probability
2 3 4
that the problem is solved? 2

436 Mathematics–12
OR
The odds against solving a problem by A and B are 3 : 2 and 2 : 1. Find the probability of getting the problem
solved.
Section-IV
All questions are compulsory. In case of internal choices attempt any one.
Q29. Let W denote the words in the English Dictionary. Show that the relation R is defined as {(x, y) ∈ W × W: the
words x and y have at least one letter in common} is reflexive, symmetric but not transitive. 3
 4x   2 + 3x  dy 5
Q30. If y = tan–1  2 
+ tan −1   , prove that = ⋅ 3
 1 + 5x   3 − 2x  dx 1 + 25 x 2
Q31. Prove that the greatest integer function defined by f (x) = [x], 0 < x < 3 is not differentiable at x = 1 and x = 2. 3
OR
d2y π
If x = 2 cos t – cos 2t, y = 2 sin t – sin 2t, find 2
at t = ⋅
dx 2
100
Q32. Find the interval in which the function f given by f (x) = x + sin x – 1 strictly increasing? 3
Q33. Evaluate: ∫ cos x cos 2x cos 3x dx 3
Q34. Find the area of the region bounded by the curves y2 = 4x, y-axis and the line y = 3. 3
OR
Find the area of the circle 4x + 4y = 9 which is interior to the parabola x2 = 4y
2 2

dy  dy 
Q35. Solve: y − x = a  y2 +   3
dx  dx 
Section-V
All questions are compulsory. In case of internal choices attempt any one.
 −8 5 
Q36. Show that A =  2 4  satisfies the equation x2 + 4x – 42 = 0. Thus, find A–1. 5
 
OR
Find the matrix X for which 1 − 4 X =  − 16 − 6 
3 − 2  7 2 
Q37. Find the equation of a line passing through a point A(2, 1, 3) and perpendicular to each of the following two lines:
∧ ∧
→ ∧
  ∧ ∧ ∧
 →
 ∧ ∧ ∧  ∧ ∧ ∧

r =  i + j + 2 k  + λ  3 i + j + 2 k  and r =  2 i − j − k  + µ  5 i + 4 j + 9 k 
        5
OR
x +1 y + 3 z + 5
Show that the lines: = = and x − 2 = y − 4 = z − 6 intersect. Also find the point of intersection.
3 5 7 1 3 5

Q38. Solve the following linear programming problem (L.P.P.) graphically. 5


Maximize Z = 22x + 18y

Subject to the constraints
x + y ≤ 20
360x + 240y ≤ 5760
and x ≥ 0, y ≥ 0
OR
The corner points of the feasible region determined by the system of linear constraints are as shown here:

Sample Paper–3 (Unsolved) 437


E:\AMIT_WORKS\Exam_Guru\EG_Mathematics-12_(working_02-06-2022)\EG_Mathematics-12_working\Open_Files\SP_1 to 3\SP_1 to 3
\ 18-Aug-2022 Amit Proof-5 Reader’s Sign _______________________ Date __________

y
6

3 (0,3)

2
(1,1)
1
(3,0)
(0,0) 1 2 3 4 5 6 x

Answer the following:


(a) Let Z = px + qy, where p, q > 0. Find the condition on p and q so that the minimum of Z occurs at (3, 0) and
(1, 1).
(b) Let Z = 4x + 6y be the objective function. Find the maximum and minimum value of Z and also the
corresponding points at which the maximum and minimum value occurs.

Answers
1. (a) reflexive only
2. (a) f is one-one onto
3. R = {(1, 2), (1, 4), (1, 6), (3, 6), (4, 4)}.
OR Yes, because a + b ≠ 5 for all a and b, where a ∈ A, b ∈ B
4. 1 × 18, 18 × 1, 2 × 9, 9 × 2, 3 × 6, 6 × 3; 1 × 5 and 5 × 1 6. – 39, 3, 11
cos 3x b
7. – + C OR log 8. (16 – 4 2 ) sq. units
3 a
9. Order = 2, Degree = 1 10. 9
11. Vector components: 4i, − 5 j , 7 k ; Scalar components: 4, – 5, 7 12. 1

( ˆ ) ( ˆ )
13. r = 2iˆ − 3 ˆj − 2k + λ 3iˆ + ˆj + 5k ; Fixed point is (2, – 3, – 2) and direction ratios are : 3, 1, 5.
x−2 y − 5 z −1 3
14. = − 2 = − 4 15. 16. 0.64
3 25
17. (i)—(a), (ii)—(c), (iii)—(a), (iv)—(c), (v)—(d)
18. (i)—(a), (ii)—(c), (iii)—(c), (iv)—(d), (v)—(a) 19. ab
20. 46 sq. units OR k = – 19 21. f (x) is continuous at x = a.
22. x = –1 is the point of L. maxima; x = 1 is the point of L. minima, x = 4 is the point of inflexion.
ex π3 π 1
23. + C OR – +
(x – 1) 2 24 2 2
8 2  1 −1 −3 
24. a sq. units 25. tan (x + y) – sec (x + y) = x + C 27. P  , , 
3 2 2 2 
3 3 –3 π 
28. OR 31. OR 32.  , π
4 5 2 2 
438 Mathematics–12
sin 6x + sin 2x + x + sin 4x + C
33. 24 8 4 16
9 2 9 2 2
34. sq. units OR + sin –1 35. y = C(x + a) (1 – ay)
4 6 4 3
− 2 5 6 2
 21 42   
36. A–1 =   OR X = 11
 1 4 2
 2 
 21 21 
x−2 y −1 z − 3  1 −1 −3 
37. = = OR P , , 
1 − 17 7 2 2 2 
38. Maximum value of Z = 392 and is attained at (8,12)
OR
q
(a) p =
2
(b) Maximum value of Z is18 at (0, 3) and minimum value of Z is 10 at (1, 1). qqq

Sample Paper–3 (Unsolved) 439

You might also like